You are on page 1of 678

McGILL UNIVERSITY

FACULTY OF SCIENCE

DEPARTMENT OF
MATHEMATICS AND STATISTICS

MATH 140 2008 09 — CALCULUS 1

Information for Students


(Fall Term, 2008/2009)
Pages 1 - 23 of these notes may be considered the
Course Outline for this course.
These notes have a page number in the upper right hand corner of every page; those are the

numbers referred to in the Table of Contents. If you print selected pages from the .pdf document,

you will need to use the sequential numbering of the pages in the document, counting this cover

as page 1: for that purpose it is best to first view the document online, and you should be able to

see those sequential numbers somewhere at the bottom of the screen.

W. G. Brown
December 2, 2008
Information for Students in MATH 140 2008 09

Contents 1.7.5 Terminology . . . . . 18


1.8 High Technology and MATH
1 General Information 1 140 . . . . . . . . . . . . . . . 19
1.1 Instructors and Times . . . . 1 1.8.1 Keep your e-mail ad-
1.2 Course Description . . . . . . 2 dresses up to date . . 19
1.2.1 Calendar Description of 1.8.2 Use of Calculators and
MATH 140 . . . . . . 2 Computer Algebra Sys-
1.2.2 Other beginning calcu- tems . . . . . . . . . . 20
lus courses . . . . . . 2 1.8.3 Use of the Internet . . 20
1.2.3 How much of my pre- 1.9 Which problems should I work? 20
vious calculus course do 1.9.1 There are no public or
I need to remember? . 3 private restricted lists
1.2.4 Late transfer from MATH of textbook problems
150 . . . . . . . . . . . 3 to which you can con-
1.3 Tutorials; Tutors’ Coordinates 3 fine your preparation
1.4 Evaluation of Your Progress . 5 for testing in MATH 140. 20
1.4.1 Your final grade . . . 5 1.9.2 Are the type of prob-
1.4.2 WeBWorK . . . . . 6 lems I find on WeB-
1.4.3 Quizzes . . . . . . . . 7 WorK problems indica-
1.4.4 Final Examination . . 9 tive of what I need to
1.4.5 Supplemental Assess- know in this course? . 21
ments . . . . . . . . . 9 1.9.3 Problems discussed in
1.4.6 Machine Scoring. . . . 9 the lectures . . . . . . 21
1.4.7 Plagiarism. . . . . . . 10 1.9.4 And anyhow, the course
1.4.8 Keep all your graded is not concerned only
materials! . . . . . . . 11 with problem solving. 21
1.4.9 There will be no graded 1.9.5 Repairing your precal-
written assignments. . 12 culus foundations. . . 21
1.5 Published Materials . . . . . 12 1.10 Last Weeks of Term (to be
1.5.1 Required Text-Book . 12 updated) . . . . . . . . . . . . 22
1.5.2 Optional Reference Books 12 1.10.1 Office Hours (additional
1.5.3 Recommended Video Ma- to regular office hours) 22
terials . . . . . . . . . 13 1.10.2 Review Tutorials (ad-
1.5.4 Other Calculus Text- ditional to regular tu-
books . . . . . . . . . 13 torials) . . . . . . . . 22
1.5.5 Website . . . . . . . . 14 1.11 Some of the best ways to en-
1.6 Syllabus . . . . . . . . . . . . 14 sure failure in MATH 140 . . 22
1.7 Preparation and Workload . . 16
1.7.1 Prerequisites. . . . . . 16 2 Solutions to Quiz Problems 24
1.7.2 Calculators . . . . . . 17 2.1 Draft Solutions to Quiz Q1 . 24
1.7.3 Self-Supervision . . . . 17 2.2 Draft Solutions to Quizzes Q3 31
1.7.4 Escape Routes . . . . 18 2.2.1 Monday Versions . . . 31
Information for Students in MATH 140 2008 09

2.2.2 Wednesday Versions . 34 5.1.9 Appendix D. Trigonom-


2.2.3 Thursday Versions . . 36 etry. TO BE DISCUSSED
2.2.4 Friday Versions . . . . 38 AFTER §1.2 . . . . . 2010
2.3 Draft Solutions to Quizzes Q4 39 5.1.10 §1.1 Four Ways to Rep-
2.3.1 Monday Versions . . . 40 resent a Function. . . 2011
2.3.2 Wednesday Versions . 42 5.2 Supplementary Notes for the
2.3.3 Thursday Versions . . 45 Lecture of September 8th, 2008 2019
2.3.4 Friday Versions . . . . 48 5.2.1 §1.2 Mathematical Mod-
2.4 Draft Solutions to Quizzes Q5 51 els: A Catalog of Es-
2.4.1 Monday Versions . . . 51 sential Functions. . . . 2019
2.4.2 Wednesday Versions . 54 5.2.2 Appendix D. Trigonom-
2.4.3 Thursday Versions . . 57 etry . . . . . . . . . . 2023
2.4.4 Friday Versions . . . . 59 5.3 Supplementary Notes for the
Lecture of September 10th, 2008 2034
3 Information Specifically for Stu- 5.3.1 §1.3 New Functions from
dents in Lecture Section 001 63 Old Functions. . . . . 2034
5.3.2 Appendix D. Trigonom-
4 Information Specifically for Stu- etry (continued) . . . 2039
dents in Lecture Section 002 2001 5.4 Supplementary Notes for the
4.1 Timetable for Lecture Section Lecture of September 15th, 2008 2047
002 of MATH 140 2008 09 . . 2001 5.4.1 Appendix D. Trigonom-
etry (conclusion) . . . 2047
5 Notes for Lecture Section 002 2003
5.4.2 §1.4 Graphing Calcu-
5.1 Supplementary Notes for the
lators and Computers
Lecture of September 3th, 2008 2003
(OMIT) . . . . . . . . 2053
5.1.1 Lecture style in Lec-
5.4.3 §1.5 Exponential Func-
ture Section 002 . . . 2003
tions . . . . . . . . . . 2054
5.1.2 A. Diagnostic Test: Al-
5.4.4 §1.6 Inverse Functions
gebra . . . . . . . . . 2005
and Logarithms . . . . 2055
5.1.3 B. Diagnostic Test: An-
5.5 Supplementary Notes for the
alytic Geometry . . . 2005
Lecture of September 17th, 2008 2060
5.1.4 C. Diagnostic Test: Func-
5.5.1 §1.6 Inverse Functions
tions . . . . . . . . . . 2005
and Logarithms (con-
5.1.5 D. Diagnostic Test: Trigonom-
clusion) . . . . . . . . 2060
etry . . . . . . . . . . 2005
5.6 Supplementary Notes for the
5.1.6 Appendix A. Numbers,
Lecture of September 22th, 2008 2073
Inequalities, and abso-
5.6.1 §2.1 The Tangent and
lute values. . . . . . . 2005
Velocity Problems. . . 2074
5.1.7 Appendix B. Coordi-
5.6.2 §2.2 The Limit of a Func-
nate Geometry and Lines 2010
tion. . . . . . . . . . . 2074
5.1.8 Appendix C. Graphs of
5.7 Supplementary Notes for the
Second-Degree Equations 2010
Lecture of September 24th, 2008 2081
Information for Students in MATH 140 2008 09

5.7.1 §2.4 The Precise Defi- 5.14.1 §3.6 Derivatives of log-


nition of a Limit (OMIT) 2081 arithm functions (ad-
5.7.2 §2.3 Calculating Lim- ditional note) . . . . . 2176
its Using the Limit Laws. 2084 5.14.2 §3.6 Derivatives of log-
5.8 Supplementary Notes for the arithm functions . . . 2176
Lecture of September 29th, 2008 2094 5.14.3 §3.7 Rates of Change
5.8.1 §2.5 Continuity. . . . . 2094 in the Natural and So-
5.9 Supplementary Notes for the cial Sciences . . . . . . 2183
Lecture of October 3rd, 2007 2104 5.14.4 §3.8 Exponential Growth
5.9.1 §2.6 Limits at Infinity; and Decay (OMIT) . . 2186
Horizontal Asymptotes. 2104 5.14.5 §3.9 Related Rates. . . 2186
5.9.2 §2.7 Derivatives and Rates 5.15 Supplementary Notes for the
of Change. . . . . . . 2111 Lecture of October 29th, 2008 2189
5.9.3 §2.8 The Derivative as 5.15.1 §3.7 Rates of Change
a Function. . . . . . . 2117 in the Natural and So-
5.10 Supplementary Notes for the cial Sciences . . . . . . 2189
Lecture of October 08th, 2008 2120 5.15.2 §3.9 Related Rates (con-
5.10.1 §2.8 The Derivative as clusion). . . . . . . . . 2191
a Function (continued). 2121 5.15.3 §3.10 Linear Approxi-
5.10.2 2 Review . . . . . . . 2126 mations and Differentials 2196
5.11 Supplementary Notes for the 5.16 Supplementary Notes for the
Lecture of October 15th, 2008 2132 Lecture of November 03rd, 2008 2199
5.11.1 §3.1 Derivatives of Poly- 5.16.1 §3.10 Linear Approxi-
nomials and Exponen- mations and Differen-
tial Functions . . . . . 2132 tials (conclusion) . . . 2199
5.11.2 §3.2 The Product and 5.16.2 §3.11 Hyperbolic Func-
Quotient Rules . . . . 2139 tions . . . . . . . . . . 2202
5.12 Supplementary Notes for the 5.16.3 3 Review . . . . . . . 2208
Lecture of October 20th, 2008 2142 5.17 Supplementary Notes for the
5.12.1 §3.3 Derivatives of Trigono- Lecture of November 05th, 2008 2214
metric Functions. . . . 2142 5.17.1 §4.1 Maximum and Min-
5.12.2 §3.4 The Chain Rule. 2150 imum Values . . . . . 2214
5.13 Supplementary Notes for the 5.18 Supplementary Notes for the
Lecture of October 22nd, 2008 2156 Lecture of November 10th, 2008 2223
5.13.1 §3.4 The Chain Rule 5.18.1 §4.2 The Mean Value
(conclusion). . . . . . 2156 Theorem . . . . . . . 2223
5.13.2 §3.5 Implicit Differen- 5.19 Supplementary Notes for the
tiation. . . . . . . . . 2159 Lecture of November 12th, 2008 2233
5.14 Supplementary Notes for the 5.19.1 §4.3 How Derivatives
Lecture of October 27th, 2008 2176 Affect the Shape of a
Graph . . . . . . . . . 2233
Information for Students in MATH 140 2008 09

5.20 Supplementary Notes for the 5.24.1 Solutions to December,


Lecture of November 17th, 2008 2241 2007, Final Examina-
5.20.1 §4.4 Indeterminate Forms tion in MATH 140 2007
and L’Hospital’s Rule 2241 09 (One of several ver-
5.20.2 Sketch of Solutions to sions) . . . . . . . . . 2328
Problems on the De- 5.25 Supplementary Notes for the
cember, 2003 Final Ex- Lecture of December 02nd, 2008 2337
amination . . . . . . . 2250 5.25.1 Solutions (continued)
5.21 Supplementary Notes for the to December, 2007, Fi-
Lecture of Wednesday, Novem- nal Examination in MATH
ber 19th, 2008 . . . . . . . . . 2261 140 2007 09 (One of
5.21.1 §4.5 Summary of Curve several versions) . . . 2337
Sketching . . . . . . . 2261 5.25.2 Some additional solved
5.21.2 §4.6 Graphing with Cal- problems in my notes 2340
culus and Calculators 2276
5.21.3 Sketch of Solutions to 6 Assignments from Previous Years 3001
Problems on one of sev- 6.1 Fall 1998 Problem Assignments 3001
eral versions of the De- 6.2 Fall 1999 Problem Assignments 3001
cember, 2004 Final Ex- 6.2.1 First Fall 1999 Prob-
amination . . . . . . . 2277 lem Assignment, with
5.22 Supplementary Notes for the Solutions . . . . . . . 3001
Lecture of November 24th, 2008 2287 6.2.2 Second Fall 1999 Prob-
5.22.1 §4.7 Optimization Prob- lem Assignment . . . . 3011
lems . . . . . . . . . . 2287 6.2.3 Third Fall 1999 Prob-
5.22.2 §4.8 Newton’s Method lem Assignment, with
– OMIT . . . . . . . . 2293 Solutions . . . . . . . 3012
5.22.3 Sketch of Solutions to 6.2.4 Fourth Fall 1999 Prob-
Problems on One Ver- lem Assignment . . . . 3020
sion of the December, 6.2.5 Fifth Fall 1999 Prob-
2005 Final Examination 2293 lem Assignment . . . . 3021
5.23 Supplementary Notes for the 6.3 2000/2001 Problem Assignments,
Lecture of November 26st, 2008 2304 with Solutions . . . . . . . . . 3023
5.23.1 §4.9 Antiderivatives . 2304 6.3.1 First 2000/2001 Prob-
5.23.2 Draft Solutions to the lem Assignment, with
December, 2006, Final Solutions . . . . . . . 3023
Examination in MATH 6.3.2 Second 2000/2001 Prob-
140 2006 09 (one version) 2315 lem Assignment, with
5.24 Supplementary Notes for the Solutions . . . . . . . 3034
Lecture of December 01st, 2008 2328 6.3.3 Third 2000/2001 Prob-
lem Assignment, with
Solutions . . . . . . . 3041
Information for Students in MATH 140 2008 09

6.3.4 Fourth 2000/2001 Prob- 7.3.2 Second 2005/2006 Writ-


lem Assignment, with ten Assignment W2 , with
Solutions . . . . . . . 3050 Sketch of Solutions . . 3102
6.3.5 Fifth 2000/2001 Prob- 7.3.3 Third 2005/2006 Writ-
lem Assignment, with ten Assignment W3 . . 3106
Solutions . . . . . . . 3056 7.3.4 Fourth 2005/2006 Writ-
6.3.6 Sixth 2000/2001 Prob- ten Assignment W4 . . 3107
lem Assignment, with 7.3.5 Fifth 2005/2006 Writ-
Solutions . . . . . . . 3061 ten Assignment W5 . . 3108
6.4 Fall 2006 Written Assignments, 7.4 Quizzes from MATH 140 2006
with Draft Solutions . . . . . 3065 09, with Draft Solutions . . . 3110
6.4.1 Draft Solutions to W1 , 7.4.1 Draft Solutions to Quiz
First Written Assign- Q1 . . . . . . . . . . . 3110
ment, Fall, 2006 . . . 3065 7.4.2 Draft Solutions to Quiz
6.4.2 Draft Solutions to W2 , Q2 . . . . . . . . . . . 3118
Second Written Assign- 7.4.3 Draft Solutions to Quiz
ment, Fall, 2006 . . . 3071 Q3 . . . . . . . . . . . 3125
6.4.3 Draft Solutions to W3 , 7.4.4 Draft Solutions to Quiz
Third Written Assign- Q4 . . . . . . . . . . . 3133
ment, Fall, 2006 . . . 3079 7.5 Quizzes from MATH 140 2007
6.4.4 Draft Solutions to W4 , 09, with Draft Solutions . . . 3141
Fourth Written Assign- 7.5.1 Draft Solutions to Quiz
ment, Fall, 2006 . . . 3083 Q1 . . . . . . . . . . . 3141
7.5.2 Draft Solutions to Quiz
7 Some Tests and Quizzes from Pre- Q2 . . . . . . . . . . .
3150
vious Years 3086 7.5.3 Draft Solutions to Quiz
7.1 Fall 1998 Class Quiz, with So- 3160
Q3 . . . . . . . . . . .
lutions . . . . . . . . . . . . . 3086 7.5.4 Draft Solutions to Quiz
7.2 Last Three Tutorial Quizzes Q4 . . . . . . . . . . .
3171
in 2000/2001 (many versions) 3088
7.2.1 Fourth 2000/2001 Tu- 8 Examinations from Previous Years 3187
torial Quizzes . . . . . 3088 8.1 December 1996 Final Exami-
7.2.2 Fifth 2000/2001 Tuto- nation in 189-122A . . . . . . 3188
rial Quizzes . . . . . . 3091 8.2 December 1997 Final Exami-
7.2.3 Sixth 2000/2001 Tuto- nation in 189-140A . . . . . . 3189
rial Quizzes . . . . . . 3094 8.3 December 1998 Final Exami-
7.3 2005/2006 Written Assignments 3099 nation in 189-140A . . . . . . 3192
7.3.1 First 2005/2006 Writ- 8.4 May 1999 Supplemental Ex-
ten Assignment W1 , with amination in 189-140A . . . . 3194
Sketch of Solutions . . 3099 8.5 December 1999 Final Exami-
nation in 189-140A . . . . . . 3195
Information for Students in MATH 140 2008 09

8.6 December 1999 Special Final 8.22 May, 2008, Supplemental/Deferred


Examination in 189-140A . . 3197 Examination in MATH 140 2007
8.7 December 2000 Final exami- 09 . . . . . . . . . . . . . . . 3247
nation in 189-140A . . . . . . 3198
8.8 May 2001 Supplemental/Deferred 9 WeBWorK 4001
Examination in 189-140A . . 3199 9.1 Frequently Asked Questions (FAQ) 4001
8.9 December 2001 Final Exami- 9.1.1 Where is WeBWorK? 4001
nation in 189-140A . . . . . . 3200 9.1.2 Do I need a password
8.10 May 2002 Supplemental/Deferred to use WeBWorK? . 4001
Examination in 189-140A . . 3202 9.1.3 Do I have to pay an
8.11 December 2002 Final Exami- additional fee to use WeB-
nation in MATH 140 2002 09 3204 WorK? . . . . . . . . 4002
8.12 May 2003 Supplemental/Deferred 9.1.4 When will assignments
Examination in MATH 140 2002 be available on WeB-
09 . . . . . . . . . . . . . . . 3206 WorK? . . . . . . . . 4002
8.13 December 2003 Final Exami- 9.1.5 Do WeBWorK assign-
nation in MATH 140 2003 09 3208 ments cover the full range
8.14 May 2004 Supplemental/Deferred of problems that I should
Examination in MATH 140 2003 be able to solve in this
09 . . . . . . . . . . . . . . . 3213 course? . . . . . . . . 4002
8.15 December 2004 Final Exami- 9.1.6 May I assume that the
nation in MATH 140 2004 09 distribution of topics
(One of several versions) . . . 3217 on quizzes and final ex-
8.16 May 2005 Supplemental/Deferred aminations will paral-
Examination in MATH 140 2004 lel the distribution of
09 . . . . . . . . . . . . . . . 3222 topics in the WeBWorK
8.17 December, 2005, Final Exam- assignments? . . . . . 4002
ination in MATH 140 2005 09 9.1.7 WeBWorK provides
(One of several versions) . . . 3226 for different kinds of
8.18 May, 2006, Supplemental/Deferred “Display Mode”. Which
Examination in MATH 140 2005 should I use? . . . . . 4003
09 . . . . . . . . . . . . . . . 3230 9.1.8 WeBWorK provides
8.19 December, 2006, Final Exam- for printing assignments
ination in MATH 140 2006 09 in “Portable Document
(One of several versions) . . . 3234 Format” (.pdf), “PostScript”
8.20 May, 2007, Supplemental/Deferred (.ps) forms. Which should
Examination in MATH 140 2006 I use? . . . . . . . . . 4003
09 . . . . . . . . . . . . . . . 3238 9.1.9 What is the relation be-
8.21 December, 2007, Final Exam- tween WeBWorK and
ination in MATH 140 2007 09 WebCT (myCourses)? 4003
(One of several versions) . . . 3242 9.1.10 Which browser should
I use for WeBWorK? 4004
Information for Students in MATH 140 2008 09

9.1.11 What do I have to do 4 Summary of Course Require-


on WeBWorK? . . . 4004 ments, as of 16 November, 2008;
9.1.12 How can I learn how (all dates are subject to change) 11
to use WeBWorK? . 4004 5 Values of trigonometric func-
9.1.13 Where should I go if tions at some “convenient” points
I have difficulties with (cf. Exercise 5.4) . . . . . . . 2028
WeBWorK? . . . . . 4005 6 Differentiation Rules from [1,
9.1.14 Can the WeBWorK §3.1] . . . . . . . . . . . . . . 2135
system ever break down 7 Differentiation Rules from [1,
or degrade? . . . . . . 4005 §3.2] . . . . . . . . . . . . . . 2140
9.1.15 How many attempts may 8 Some Antiderivatives . . . . . 2305
I make to solve a par- 9 1998 Problem Assignments . 3001
ticular problem on WeB- 10 Summary of solutions to Fall,
WorK? . . . . . . . . 4006 2006 written assignment W1 ,
9.1.16 Will all WeBWorK as- MONDAY version . . . . . . 3067
signments have the same 11 Summary of solutions to Fall,
length? the same value? 4006 2006 written assignment W1 ,
9.1.17 Is WeBWorK a good WEDNESDAY version . . . . 3069
indicator of examina- 12 Summary of solutions to Fall,
tion performance? . . 4006 2006 written assignment W1 ,
THURSDAY version . . . . . 3070
10 Contents of the DVD disks for 13 Summary of solutions to Fall,
Larson/Hostetler/Edwards 5001 2006 written assignment W1 ,
FRIDAY version . . . . . . . 3071
11 References 6001
11.1 Stewart Calculus Series . . . 6001
11.2 Other Calculus Textbooks . . 6002 List of Figures
11.2.1 R. A. Adams . . . . . 6002
11.2.2 Larson, Hostetler, et al. 6003 1 Sketch of Solution to [1, Ex-
11.2.3 Edwards and Penney . 6003 ercise 36, p. A9]: thickened
11.2.4 Others, not “Early Tran- portions of R . . . . . . . . . 2009
scendentals” . . . . . . 6004 2 Graph of the Function sin x . 2041
11.3 Other References . . . . . . . 6004 3 Graph of the Function cos x . 2041
4 Graph of the Function tan x . 2042
5 Graph of the Function cot x . 2043
List of Tables 6 Graph of the Function sec x . 2044
7 Graph of the Function csc x . 2045
1 Instructors and Times . . . . 1 8 Invertible restriction of the Func-
2 Schedule and Locations of Tu- tion sin x . . . . . . . . . . . 2064
torials, as of December 2, 2008 9 Graph of the Inverse Sine Func-
(subject to change) . . . . . . 4 tion . . . . . . . . . . . . . . 2065
3 Tutors’ Coordinates, as of De- 10 Reflection of the restriction of
cember 2, 2008 . . . . . . . . 5 sin x in the line y = x . . . . 2066
Information for Students in MATH 140 2008 09

11 Graph of the Function arccos x 2073


12 Graph of the Function
√ √ f (x) =
2 2
x + x + 1 − x − x . . . 2129
sin θ
13 Computing bounds for . 2142
θ
x 2
14 Graph of the Function 2
x +3
and its horizontal asymptote,
y = 1 . . . . . . . . . . . . . . 2237
15 Graph of the Function cos2 x−
2 sin x . . . . . . . . . . . . . 2238
16 Graph of the Function x ln x . 2240
17 Portion of the graph of y =
sin x − tan x, showing its ver-
tical asymptotes . . . . . . . 2264
sin x
18 Graph of f (x) = for x >
x
0 . . . . . . . . .r. . . . . . . 2267
x
19 Graph of f (x) = and
x−5
its horizontal asymptote . . . 2270
20 Graph of y = ex − 3e−x − 4x 2273
21 Graph of the Function x−ln(1+
x2 ) . . . . . . . . . . . . . . . 2327
2
22 Graph of xe−2x . . . . . . . 2341
23 Showing a discontinuity in a
graph . . . . . . . . . . . . . 3072
(x − a)2 − b2
24 Graph of the Function
(x − a)2 + b2
and its horizontal asymptote,
y = 1, when a = 2, b = 5 . . . 3085
25 Problem 4 on Thursday Quiz
Q3 . . . . . . . . . . . . . . . 3165
26 Problem 1 on Friday Quiz Q3 3166
Information for Students in MATH 140 2008 09 1

1 General Information
Distribution Date: This version is as of December 2, 2008
(all information is subject to change)
Pages 1 - 23 of these notes may be considered the Course Outline for this course.

These notes may undergo minor corrections or updates during the term: the
definitive version will be the version accessible at

http://www.math.mcgill.ca/brown/math140a.html

or on myCourses, at

http://www.mcgill.ca/mycourses/ 1

Some of the details concerning this course will be different from past years.
Students are advised not to make assumptions based on past years’ opera-
tions. Publications other than this document may contain unreliable infor-
mation about this course.
All details of the course could be subject to discretionary change in case of
force majeure.

1.1 Instructors and Times

INSTRUCTOR: Dr. D. Serbine Prof. W. G. Brown


(Course Coordinator)
LECTURE SECTION: 1 2
CRN: 362 365
OFFICE: BURN 1242 BURN 1224
OFFICE HOURS: MWF 09:30→10:30 M 15:00→16:00
(subject to change) F 10:00→11:00
or by appointment
TELEPHONE: (514)-398-3853 (514)-398-3836
E-MAIL:2 SERBIN@ BROWN@
MATH.MCGILL.CA MATH.MCGILL.CA
CLASSROOM: ADAMS AUD ADAMS AUD
CLASS HOURS: MWF 8:35–9:25 h. MW 16:35–17:55 h.

Table 1: Instructors and Times


1
Please note that the statements about MATH 140 in an SUS publication called Absolute Zero were
not given to instructors of this course to check, and some of them are not currently correct.
Information for Students in MATH 140 2008 09 2

1.2 Course Description


1.2.1 Calendar Description of MATH 140
MATH 1404 CALCULUS 1. (3 credits. 3 hours lecture; 1 hour tutorial. Prerequisite:
High School Calculus. Not open to students who have taken MATH 120, MATH 139
or CEGEP objective 00UN or equivalent; not open to students who have taken or are
taking MATH 122 or MATH 130 or MATH 131, except by permission of the Department
of Mathematics and Statistics. Each Tutorial section is enrolment limited.) Review
of functions and graphs. Limits, continuity, derivative. Differentiation of elementary
functions. Antidifferentiation. Applications.

1.2.2 Other beginning calculus courses


• MATH 139 Calculus (4 credits); Instructor = Professor S. Drury. This
course (whose prerequisite is a course in functions) is intended for students with no
previous knowledge of calculus; it is not open to students who have had one term of
College level Calculus. ... Students may apply at the Department of Mathematics
and Statistics before the end of the Course Change Period for authorization to
register in this course; they must bring copies of their transcripts5 . This course
covers approximately the same material as MATH 140. MATH 139 uses the same
textbook as MATH 140.

• MATH 150 Calculus A (4 credits); Instructor = Dr. M. Willems. This


course, together with either of its 4-credit sequels, MATH 151 Calculus B and
MATH 152 Calculus E6 , covers most of the material of courses MATH 140/MATH
139, MATH 141 Calculus 2, and MATH 222 Calculus 3, in only two semesters7 . A
prior or concurrent course in Vector Geometry (e.g. MATH 133) is recommended.8
2
Please do not send e-mail messages to your instructors through the WebCT or WeBWorK3 systems;
rather, use the addresses given in Table 1.1 on page 1.
3
E-mail messages generated by the Feedback command in WeBWorK should be used sparingly,
and confined to specific inquiries about WeBWorK assignments.
4
The previous designation for this course was 189-140, and the version given in the fall was labelled
189-140A; an earlier number for a similar course was 189-122.
5
Authorization for registration in MATH 139 2008 09 will be available beginning on Tuesday, 26
August, 2008, and ending on Tuesday, 16 September, 2008. For details, see the following URL:

http://www.math.mcgill.ca/brown/incoming.htm

6
Open only to students in the Faculty of Engineering
7
MATH 152 lacks one topic in MATH 222, as it is deferred to another Engineering Mathematics
course.
8
Note that MATH 150/151 uses a different textbook from MATH 140/141.
Information for Students in MATH 140 2008 09 3

1.2.3 How much of my previous calculus course do I need to remember?


Since the gaps in students’ knowledge from their high school calculus courses are different
for different students, we will be covering the entire syllabus of a first calculus course,
i.e., the same material that is covered in MATH 139, described above. However, we do
assume a working knowledge of the prerequisites for a first calculus course — algebra,
trigonometry, and functions.9

1.2.4 Late transfer from MATH 150


Some students from MATH 150 may be permitted by their Faculty to transfer into
MATH 140 after the end of the Change of Course Period. If you are in this category,
please send an e-mail message to Professor Brown as soon as your transfer has been
approved, so that a WeBWorK account may be opened for you in this course, and that
your date of entry into this course is recorded.

1.3 Tutorials; Tutors’ Coordinates


1. Every student must be registered in one lecture section and one tutorial section.
Tutorials begin in the week of September 08th, 2008. The last tutorials in all tuto-
rial sections will be during the week of November 24th, 2008. Table 2 gives times,
locations, and the tutor’s name for each of the tutorials; Table 3 gives the tutors’
coordinates. The information in these tables is subject to change. We try
to publicize changes but sometimes we are not informed in advance.10

2. You are expected to write quizzes only in the tutorial section in which
you are registered.11 You do not have a licence to move from one tutorial
section to another at will, even if you find the time, location, or person-
nel of your tutorials either temporarily or permanently inconvenient; in
the latter case the onus is on you to transfer formally to another tuto-
rial section, to change your other classes, or to drop MATH 140 2008
09. Please remember that transfers must be completed by the Course
Change (drop/add) deadline (September 16th, 2008), and are subject
to the maximum capacities established for each tutorial section12 .
9
While there will be a brief review of some topics from trigonometry, students who are wholly lacking
in trigonometry should consider registering in MATH 112 2008 09, a course available only in the fall
term. For students in some faculties MATH 112 cannot be taken for academic credit. While that may
be painful, it may be preferable to failing MATH 140 because of a lack of adequate preparation.
10
The current room for your tutorial should always be available by clicking on “Class Schedule” on
MINERVA FOR STUDENTS, http://www.mcgill.ca/minerva-students/.
11
In some time slots there may be several tutorial sections, meeting in different rooms.
12
Your instructors do not have the ability to change the maximum capacities of tutorials.
Information for Students in MATH 140 2008 09 4

All tutorial sections meet first during the week of September 08th, 2008
# CRN Day Begins Ends Room Tutor
T003 368 Mon 13:35 14:25 BURN 1B24 El-Hajj, L.
T004 369 Mon 14:35 15:25 BURN 1214 El-Hajj, L.
T005 370 Mon 14:35 15:25 ENGTR 0060 Bridgeman, L.
T006 371 Mon 15:35 16:25 RPHYS 115 Wong, M.
T007 372 Mon 14:35 15:25 RPHYS 115 Wong, M.
T008 373 Mon 15:35 16:25 ENGMC 12 Bridgeman, L.
T009 374 Wed 13:35 14:25 WONG 1030 Zhang, X.
T010 375 Wed 14:35 15:25 RPHYS 114 Zhang, X.
T011 376 Wed 14:35 15:25 BURN 1B45 Nashaat, S.
T012 377 Wed 15:35 16:25 BURN 1B36 Nashaat, S.
T013 378 Wed 14:35 15:25 BURN 1B39 Gauthier-Shalom, G.
T014 379 Wed 13:35 14:25 BURN 1214 Gauthier-Shalom, G.
T015 380 Thurs 16:05 16:55 BURN 1B39 Kazachkov, I.
T016 381 Thurs 17:05 17:55 BURN 1B39 Kazachkov, I.
T017 382 Wed 15:35 16:25 MAASS 328 Cordy, B.
T018 383 Wed 16:35 17:25 BURN 1B23 Cordy, B.
T019 384 Fri 14:35 15:25 BURN 1B23 Haldane, E.
T020 385 Fri 15:35 16:25 BURN 1B23 Haldane, E.

Table 2: Schedule and Locations of Tutorials, as of December 2, 2008 (subject to change)

3. The tutorials in MATH 140 are short. About one-third of the total tutorial time
is devoted to testing, through quizzes every few weeks, and subsequent discussion
of the solutions to the quiz problems.
The remaining time will be devoted to brief discussion of the solution of specific
kinds of problems. Students must not assume that they will be exposed in lectures
and tutorials to detailed model solutions for every type of Calculus 1 problem. It is
essential that you supplement these classes with serious work on your own, carefully
reading the textbook and solving problems therein. If you encounter difficulties,
take them to the tutors during one of their many office hours: you may attend the
office hours of any tutor in the course, and are not restricted to those of the tutor
of the tutorial in which you are registered.
Information for Students in MATH 140 2008 09 5

Tutor E-mail address Office Office Hours


BURN Day Begins Ends Day Begins Ends
Bridgeman, L. leilabridge@gmail.com 1115 W 13:30 16:30
Cordy, B. bcordy@math.mcgill.ca 1032 Th 10:00 13:00
El-Hajj, L. elhajj@math.mcgill.ca 1133 W 10:00 13:00
Gauthier-Shalom, G. ggauthier@math.mcgill.ca 1036 T 10:00 13:00
Haldane, E. ehaldane@math.mcgill.ca 1134 F 11:00 14:00
Kazachkov, I. kazachkov@math.mcgill.ca 1029 Th 13:00 15:00 Th 18:00 19:00
Nashaat, S. nashaat@math.mcgill.ca 1132 T 13:00 14:30 Th 16:30 18:00
Wong, M. wong@math.mcgill.ca 1031 M 13:00 14:30 Th 15:00 16:30
Zhang, X. xzhang@math.mcgill.ca 1020 T 15:00 18:00
During her/his office hours, a tutor is available to all students in the course,
not only to the students of her/his tutorial section.

Table 3: Tutors’ Coordinates, as of December 2, 2008

1.4 Evaluation of Your Progress


1.4.1 Your final grade
(See Table 4, p. 11) Your grade in this course will be a letter grade, based on a percentage
grade computed from the following components:
1. Assignments submitted over the Web: Seven WeBWorK homework assignments
(cf. §1.4.2) — counting together for a total of 10% of the final grade. All WeB-
WorK assignments must be completed by their posted expiration dates and times.13
2. Quizzes graded by your Tutor: Four tutorial quizzes will be given at the tutorials
— counting together for 20% of the final grade. Quizzes Q1 , Q3 , Q4 , Q5 will
count equally in the 20% allocated to the quizzes; Q5 counts equally with the other
quizzes, even though it will be 45 minutes long, while Q1 , Q2 , . . . Q4 will each be
25 minutes long.
3. The final examination — counting for 70%.
Where a student’s performance on the final examination is superior to her performance
on the tutorial quizzes, 20% of the final examination percentage will replace the quiz
13
Note that WeBWorK uses the “American” slash notation for dates, rather than the European
notation that may be in more common usage in Québec: “ab/cd/07” means day #cd of month ab of
year 2008.
Information for Students in MATH 140 2008 09 6

grades in the calculations. It is not planned to permit the examination grade to replace
the grades on WeBWorK assignments.

1.4.2 WeBWorK
The WeBWorK system, developed at the University of Rochester — is designed to
expose you to a large number of drill problems, and where plagiarism is discouraged.
WeBWorK is accessible only over the Web. Details on how to sign on to WeBWorK
are contained in Appendix 9 to these notes, page 4001. WeBWorK assignments carry
a due date and time; only answers submitted by the due date and time will count. The
WeBWorK assignments will be labelled A1 , . . ., A7 .

Numbers of permitted attempts at WeBWorK questions. The assignments


have limits to the numbers of times a student may attempt a problem. However, for each
assignment An there will be a companion “Practice” Assignment Pn (n = 1, 2, . . . , 7) with
an unlimited number of attempts at similar problems, but in which the specific data will
be different. You may prepare yourself on the Practice assignment before attempting
the actual assignment. The practice assignments DO NOT COUNT in your term mark,
even though a grade is recorded. Practice assignment Pn is usually due 1 week before
assignment An . Another assignment which will not count will be Practice Assignment
P0 , which is a general introduction to the WeBWorK system.

Due dates and times for WeBWorK assignments. The due dates for WeB-
WorK assignments will be on specified Sundays, about 23 : 30h (with the exception of
Assignment P4 , which will be due late in the evening on the Monday which is Canada’s
Thanksgiving Day); last minute changes in the due dates may be announced either on
WeBWorK, on WebCT, or by an e-mail message14 As mentioned in the WeBWorK
FAQ (cf. Appendix §9) if you leave your WeBWorK assignment until the hours close
to the due time on the due date, you should not be surprised if the system is slow to re-
spond. This is not a malfunction, but is simply a reflection of the fact that other students
have also been procrastinating! To benefit from the speed that the system can deliver
under normal conditions, do not delay your WeBWorK until the last possible day! If
a systems failure interferes with the due date of an assignment, arrangements may be
made to change that date, and an e-mail message may be broadcast to all users (to the
e-mail addresses on record), or a note posted in the course announcements on WebCT;
but slowness in the system just before the due time will not normally be considered a
systems failure.15
14
Be sure that your e-mail addresses are correctly recorded. See 1.8.1, p. 19 of these notes.
15
Should you find that the system is responding slowly, do not repeatedly press the ENTER key on
your keyboard, as you could submit your solutions more than once, thereby depleting the number of
Information for Students in MATH 140 2008 09 7

Precalculus WeBWorK assignments. Assignments A1 and A2 are based on precal-


culus material. These assignments have been scheduled to be due before the last day to
change courses; they can help you decide whether you are ready for MATH 140.

1.4.3 Quizzes
1. There will be 5 quizzes, numbered Q1 , . . . , Q5 , 4 of them administered at the
tutorials, and 1 at a lecture. These quizzes will be graded, and returned. The
primary purpose of a quiz is to diagnose possible gaps in your understanding.
No provision is being made for “makeup” quizzes for students who miss one of Q1 ,
Q3 , Q4 , Q5 .16 17 The grading formula permits the quiz component of the final grade
to be replaced by the final examination grade, if this is to the student’s advantage.

2. Quiz contents. The quizzes will be based on a selection of current topics in the
syllabus of the course, most of which topics will have been discussed in the lectures
before the quiz; sometimes different quizzes in the same set will select different
topics for examination. Where it is practical quiz topics may have been discussed
in tutorials; but that cannot always be the case, and is not practical for Q1 , Q2 . The
quizzes are usually not based directly on WeBWorK assignments. The specific
topics on quizzes will not be announced in advance: we want the quizzes to help
evaluate the level of your preparation, not to become targets of cramming. Please
do not ask your tutors speculate about the content of coming quizzes, as they
will not have advance information. To prepare for a quiz you should be working
exercises in the textbook based on the material currently under discussion at the
lectures, and you should have attempted any open WeBWorK assignments. But,
unlike the WeBWorK assignments — where evaluation is based on correct answers
alone — students will usually be expected to provide full solutions.18

3. Length of quizzes. Quiz Q5 will be about 45 minutes long; quizzes Q1 , . . . Q4


will be about 25 minutes long. However, quizzes Q1 , Q3 , Q4 , Q5 will count equally
in the calculation of the quiz grade, and quiz Q2 will not count at all.
attempts that have been allowed to you for a problem: this will not be considered a systems failure.
16
Students who miss Q2 for medical or religious reasons will be given an opportunity to write this
important diagnostic quiz at one other time.
17
Students who miss a quiz for medical reasons must obtain a medical note to be shown to their
lecturer — not to the Teaching Assistant: a vague report of illness is not sufficient.
18
In Math 140 and Math 141 the general rule is that full solutions are expected to all problems, unless
you receive explicit instructions to the contrary: ALWAYS SHOW YOUR WORK! The solutions in the
Student Solution Manual [3] to the textbook can serve as a guide to what should be included in a “full”
solution.
Information for Students in MATH 140 2008 09 8

4. Part Marks. TA’s are being instructed not to award part marks unless a student
has made substantial progress towards the solution of the question. It is hoped that
the number of marks received by the student should be suggestive of the extent of
her understanding of the problem. In that way the quizzes can serve their intended
purpose of providing students with a measure of their progress in the course, and
can help students anticipate their performance on the final examination.

5. Special significance of Q1 , Q2 , and Q5 . Quizzes Q1 , Q2 can serve as diagnostic


tools, indicating your preparedness for MATH 140. If your grades on these quizzes
are very low, you should reconsider the wisdom of remaining in MATH 140. Quiz
Q5 , because it is expected to be longer than the preceding quizzes, and because it
is administered later in the term, tends to provide some information about your
likely performance on the final examination. Of course, the quiz will not contain
questions from the very last topics in the course.

6. You are expected to write quizzes Q1 , Q3 , Q4 and Q5 only in the tutorial section,
and Q2 only in the lecture section in which you are registered.

7. Return of graded quizzes. Your tutors will normally bring graded quizzes with
them to the tutorial to be returned to you. University regulations do not permit us
to leave unclaimed materials bearing names and student numbers in unsupervised
locations; you may be able to recover an unclaimed quiz from the tutor who graded
it, during her/his regular office hours.

8. “Raw” versus “Scaled” Quiz Grades. After he receives all quiz data from
T.A.’s, Professor Brown may make small upward adjustments in the grades in
certain tutorial sections, in order to reduce variations between sections, based on
analysis of the average or median performance. The “Raw” grade can often give
you a good indication of how you might perform on the final examination.

9. Posting of grades. Grades on Quizzes will eventually be uploaded to WebCT,


where you will be able to check that your grades have been properly recorded.
Report any error by e-mail to Professor Brown and to your T.A., within one month
of the date it has been posted.

10. Plagiarism. (cf. §1.4.7) When a T.A. discovers evidence of cheating, she/he is
obliged to bring it to the attention of the instructors, who may have to forward the
materials to the Associate Dean of the Student’s Faculty for possible disciplinary
action. Such actions have been taken in the past in this course.

11. Students with Disabilities who wish to write any of their quizzes at the OSD
office need to make prior arrangements with that office before any quiz.
Information for Students in MATH 140 2008 09 9

12. Diagnostic trigonometry quiz. Quiz Q2 , to be administered in the lectures


of October 1st, is a multiple choice diagnostic quiz which will help evaluate your
preparation in trigonometry19 : the grade on Q2 does not count in your term mark,
but it is essential that you write it, in order to expose any deficiencies early in the
term20 .

1.4.4 Final Examination


A 3-hour-long final examination will be scheduled during the regular examination period
for the fall term (December 04th, 2008 through December 19th, 2008). You are advised
not to make any travel arrangements that would prevent you from being present on
campus at any time during this period.

1.4.5 Supplemental Assessments


Supplemental Examination. Faculty regulations require that we advise you that
there will be a supplemental examination in this course. For information about Supple-
mental Examinations, see

http://www.mcgill.ca/student-records/exam/supplemental/.

Note, in particular, that a Supplemental Examination may be written only by a student


who has obtained a grade of F or D as a grade in the course, and that the grade on the
Supplemental Examination counts in your average as though you have taken the course
again — without a term work component; don’t be confused by the word supplemental .

There is No Additional Work Option. “Will students with marks of D, F, or J


have the option of doing additional work to upgrade their mark?” No. (“Additional
Work” refers to an option available in certain Arts and Science courses, but not available
in MATH 140 2008 09.)

1.4.6 Machine Scoring.


“Will the final examination be machine scored?” It is possible that the final examination,
or part of it, could be machine scored. Multiple choice problems, possibly machine scored,
could also appear on some quizzes. (Machine grading, if implemented in whole or in part,
would be a change from the practice of past years.)
19
There could also be problems on trigonometry on other quizzes, particularly Q1 .
20
For students who miss Q2 for medical or religious reasons (which must be appropriately documented)
there will be another opportunity to write Q2 outside of the lecture hours.
Information for Students in MATH 140 2008 09 10

Answer-only Problems. It is possible that some of the problems on your final ex-
amination and/or quizzes will request that the answer only be given, and will not carry
part marks which could be based on the work leading up to the answer.

1.4.7 Plagiarism.
While students are not discouraged from discussing methods for solving WeBWorK
assignment problems with their colleagues, all the work that you submit — whether
through WeBWorK assignments, or on tutorial quizzes or the final examination must
be your own. The Senate of the University requires the following message in all course
outlines:

“McGill University values academic integrity. Therefore all students must under-
stand the meaning and consequences of cheating, plagiarism and other academic
offences under the Code of Student Conduct and Disciplinary Procedures. (See
http://www.mcgill.ca/integrity for more information).
“L’université McGill attache une haute importance à l’honnêteté académique. Il
incombe par conséquent à tous les étudiants de comprendre ce que l’on entend
par tricherie, plagiat et autres infractions académiques, ainsi que les conséquences
que peuvent avoir de telles actions, selon le Code de conduite de l’étudiant et des
procédures disciplinaires. (Pour de plus amples renseignements, veuillez consulter
le site http://www.mcgill.ca/integrity).”

It is a violation of University regulations to permit others to solve your


WeBWorK problems, or to extend such assistance to others; you could be
asked to sign a statement attesting to the originality of your work. The
Handbook on Student Rights and Responsibilities21 states in ¶A.I.15(a) that

“No student shall, with intent to deceive, represent the work of another person
as his or her own in any academic writing, essay, thesis, research report,
project or assignment submitted in a course or program of study or represent
as his or her own an entire essay or work of another, whether the material so
represented constitutes a part or the entirety of the work submitted.”

You are also referred to the following URL:

http://www.mcgill.ca/integrity/studentguide/

21
http://upload.mcgill.ca/secretariat/greenbookenglish.pdf
Information for Students in MATH 140 2008 09 11

Item # Due Date Details


P0 DOES NOT COUNT: introduces WeBWorK
P1 14 Sept 08 DOES NOT COUNT; practice for A1
P2 14 Sept 08 DOES NOT COUNT; practice for A2
WeBWorK A1 21 Sept 08
Assignments A2 21 Sept 08
(cf. §1.4.2) P3 28 Sept 08 DOES NOT COUNT; practice for A3
10% A3 05 Oct 08
P4 13 Oct 08 DOES NOT COUNT; practice for A4
A4 19 Oct 08
P5 26 Oct 08 DOES NOT COUNT; practice for A5
A5 02 Nov 08
P6 09 Nov 08 DOES NOT COUNT; practice for A6
A6 18 Nov 08
P7 23 Nov 08 DOES NOT COUNT; practice for A7
A7 02 Dec 08 A1 –A7 count equally, but may have different
numbers of problems.
Quizzes Q1 22–26 Sept 08 25 minutes long
(cf. §1.4.3) Q2 01 Oct 08 DOES NOT COUNT; 25 minutes long
20% or 0% Q3 20–24 Oct 08 25 minutes long
Q4 03–07 Nov 08 25 minutes long
Q5 17–24 Nov 08 45 minutes long
Quizzes Q1 , Q3 , Q4 , Q5 count equally.
Final Exam 04–19 Dec 08 Date of exam to be announced by Faculty
70% or 90%
Supplemental 01–04 May 09 Only for students who do not obtain standing
Exam at the final. Supplemental exams count in your
average like taking the course again, with no
term mark component.

Table 4: Summary of Course Requirements, as of 16 November, 2008; (all dates are


subject to change)

1.4.8 Keep all your graded materials!


Grades will be posted on WebCT during the term; check periodically to see that your
grades are recorded correctly, and advise Professor Brown and/or your TA if you find a
grade missing or incorrectly recorded. You may need to produce the graded assignment
Information for Students in MATH 140 2008 09 12

or quiz, so these should be retained until the end of the term.22

1.4.9 There will be no graded written assignments.


Students desiring the benefits of graded, written assignments should attempt to keep
their solutions in a notebook that could be shown to a tutor at his/her office hours. One
reason we are discontinued the practice of grading written assignments is that we found
that there was excessive collaboration. While you are urged to discuss problems with
your colleagues, anything that you submit for grading should be your own work.

1.5 Published Materials


1.5.1 Required Text-Book
The textbook for the course is J. Stewart, SINGLE VARIABLE CALCULUS: Early
Transcendentals, Sixth Edition, Brooks/Cole (2008), ISBN 0-495-01169-X, [1].
This book is the first half of a longer textbook, J. Stewart, CALCULUS: Early
Transcendentals, Sixth Edition, Brooks/Cole (2008), ISBN 0-495-01166-5, [2],
which covers the material for Calculus 3 (MATH 222) as well, but may not be the
text-book for that course at the present time. The textbook will be sold in the McGill
Bookstore bundled with its Student Solutions Manual (see below). The ISBN number
for the entire bundle is 0-495-42966-X.

1.5.2 Optional Reference Books


Students are urged to make use of the Student Solutions Manual:
• D. Anderson, J. A. Cole, D. Drucker, STUDENT SOLUTIONS MANUAL
FOR STEWART’S SINGLE VARIABLE CALCULUS: Early Transcen-
dentals, Sixth Edition, Brooks/Cole (2008), ISBN 0-495-?????-?, [3]. This
book is also sold “bundled” with the text book; we expect the Bookstore to stock
the bundle numbered ISBN 0-495-42966-X [4].
The publishers of the textbook and Student Solutions Manual also produce
• a “Study Guide”, designed to provide additional help for students who believe
they require it: R. St. Andre, STUDY GUIDE FOR STEWART’S SIN-
GLE VARIABLE CALCULUS: Early Transcendentals, Sixth Edition,
Brooks/Cole (2008), ISBN 0-495-01239-4, [5]. (The “Study Guide” resembles the
Student Solutions Manual in appearance: be sure you know what you are buying.)
22
The justification for quizzes and assignments is mainly as learning experiences: for that reason alone
you should retain all materials that could help you prepare for your final examination.
Information for Students in MATH 140 2008 09 13

• a “Companion” which integrates a review of pre-calculus concepts with the contents


of Math 140, including exercises with solutions: D. Ebersole, D. Schattschneider,
A. Sevilla, K. Somers, A COMPANION TO CALCULUS. Brooks/Cole (1995),
ISBN 0-534-26592-8 [38].

1.5.3 Recommended Video Materials


Use of the following materials is recommended, but is not mandatory23 .

Text-specific DVDs for Stewarts Calculus, early transcendentals, 6th edi-


tion [videorecording]. The publisher of Stewart’s Calculus has produced a series of
videodisks, [14]. These will initially be available for reserve loan at the Schulich Library.
There may not be DVD viewing equipment freely available in the library; the intention
is that interested students borrow disks for viewing on their own equipment at home.
Disk 1 covers Chapters 1-6 of the textbook.

Larson/Hostetler/Edwards DVD Disks A set of video DVD disks produced for an


earlier edition of another calculus book, [28] Calculus Instructional DVD Program, for use with
(inter alia) Larson/Hostetler/Edwards, Calculus of a Single Variable: Early Transcendental
Functions, Third Edition [29] is produced by the Houghton Mifflin Company. Copies have
been requested to be placed on reserve in the Schulich Library. In Appendix §10 of these notes
there are charts that indicate the contents of these disks that pertain to MATH 14024 .

1.5.4 Other Calculus Textbooks


While students may wish to consult other textbooks, or other editions of the recom-
mended textbooks, instructors and teaching assistants in Math 140 will normally refer
only to the prescribed edition of the prescribed textbook for the course. Other books
can be very useful, but the onus is on you to ensure that your book covers the syllabus to
at least the required depth; where there are differences of terminology, you are expected
to be familiar with the terminology of the textbook.25
In your previous calculus course(s) you may have learned methods of solving problems
that appear to differ from those you find in the current textbook. Your instructors will
be pleased to discuss any such methods with you personally, to ascertain whether they
are appropriate to the present course. In particular, any methods that depend upon
23
No one is planning to check whether you have used any of these aids; a student can obtain a perfect
grade in the course without ever consulting any of them. No audio-visual or calculator aid can replace
the systematic use of paper and pencil as you work your way through problems. But the intelligent use
of some of these aids may assist your in understanding the subject matter.
24
prepared for the 5th Edition of our textbook
25
It is hoped that there will be multiple copies of the textbook on reserve in the Schulich library.
Information for Students in MATH 140 2008 09 14

the use of a calculator, or the plotting of multiple points, or the tabulation of function
values, or the inference of a trend from a graph should be regarded with scepticism.

1.5.5 Website
These notes, and other materials distributed to students in this course, will be accessible
at the following URL:

http://www.math.mcgill.ca/brown/math140a.html

The notes will be in “pdf” (.pdf) form, and can be read using the Adobe Acrobat reader,
which many users have on their computers. This free software may be downloaded from
the following URL:
26
http://www.adobe.com/prodindex/acrobat/readstep.html

The questions on some old examinations will also be available as an appendix to these
notes on the Web.27 It is expected that most computers in campus labs should have the
necessary software to read the posted materials.
Where revisions are made to distributed printed materials — for example these in-
formation sheets — we expect that the last version will be posted on the Web.
The notes and WeBWorK will also be available via a link from the myCourses URL:

http://www.mcgill.ca/mycourses/

but some other features of WebCT28 have not yet been implemented. In particular,
please do not attempt to send instructors or TA’s mail through the myCourses system
— use your regular e-mail client from outside of myCourses.

1.6 Syllabus
In the following list section numbers refer to the text-book [1]. The syllabus will include
all of Chapters 1, 2, 3, 4, with omissions, as listed below.29
26
At the time of this writing the current versions appear to be Versions 8.1 or 9, depending on your
operating system.
27
There is no reason to expect the distribution of problems on quizzes or in assignments and exami-
nations from previous years be related to the frequencies of any types of problems on the examination
that you will be writing at the end of the term.
28
cf. Appendix 9 to these notes, p. 4001
29
If a textbook section is listed below, you should assume that all material in that section is examina-
tion material even if the instructor has not discussed every topic in his lectures; however, the instructors
may give you information during the term concerning topics that may be considered subsidiary.
Information for Students in MATH 140 2008 09 15

Diagnostic Tests The textbook includes (in the edition adopted for this year’s course)
four multiple choice diagnostic tests. You should administer these to yourself (hon-
estly, without peeking at the answers), in order to detect areas of weakness. If your
score on these tests is very low, you probably aren’t ready for a calculus course,
and should consider transferring immediately to MATH 112 2008 09, rather than
exposing yourself to a high risk of failure in MATH 140.30

Chapter 0: A Preview of Calculus. This is motivational material, and may not all
be discussed in the lectures. Read it.

Chapter 1: Functions and Models. Some parts of §1.2 may not be discussed in the
lectures, but you should read the whole section — in particular the definitions of
various kinds of functions — as this terminology may be used from time to time.
Omit §1.4.
The Principle of Mathematical Induction, [1, p. 77] has occasionally been included
in the syllabus of MATH 140. It will not be included this semester, because there
isn’t enough time. You may find problems in written assignments or examinations
from previous years based on this Principle.

Chapter 2: Limits and Derivatives. [1, §2.4] will be discussed at the lectures, but
is not examination material. Omit [1, pp. 138-140].

Chapter 3: Differentiation Rules. Omit [1, §3.7], with the exception of text and
exercises concerned with moving particles, [1, pp. 221-223, 230-231]). Omit [1,
§3.8]. You are encouraged to read the parts of [1, §§3.7-3.8] that pertain to your
own fields of interest.

Chapter 4: Applications of Differentiation. Omit “slant asymptotes [1, §4.5, pp.


312-313, and Exercises ##57-69 on p. 315]. Omit §4.6, §4.8.

Exercises that require technology Students are not expected to be able to solve
exercises that require the use of calculators or computers. You may wish to try
such problems, as a challenge, as some of them can be solved with clever use of
paper and pencil.
Do not assume that a topic is omitted from the syllabus if it has not been tested in a
WeBWorK assignment or a quiz, or if it has not appeared on any of the old examinations
in the course! Some topics to not lend themselves to this type of testing; others may have been
omitted simply because of lack of space, or oversight. By the same token, you need not expect every
topic in the course to be examined on the final examination.
30
Actually, these tests are not sufficiently demanding for students in MATH 140: it’s not sufficient to
be able to get the right answer — we would want you to be able to write a full solution to any one of
these problems in a style comparable to that of the Student Solutions Manual to the textbook.
Information for Students in MATH 140 2008 09 16

Problems Plus The exercises and other material that appear in [1, Principles of Prob-
lem Solving, pp. 76-81], and in the “Problems Plus” subsections following the later
chapters may be omitted.

The following appendices in the textbook contain some prerequisite material for this
course:

Appendix A: Numbers, Inequalities, and Absolute Values. (see information for


Chapters 1 and 3 above)

Appendix B: Coordinate Geometry and Lines.

Appendix C: Graphs of Second-Degree Equations. You are expected to be famil-


iar with the material concerning the circle, [1, pp. A16-A17]. The remainder of the
material should be familiar to most students, but will not be assumed.

Appendix D: Trigonometry. You are assumed to be familiar with the material in [1,
pp. A24–A31].

The other appendices will not be required in MATH 140.


Please do not ask the tutors to provide information as to which topics should be
emphasized. Unless you are informed otherwise by the instructors in the lecture sections
or published notes — printed, or mounted on the Web — you should assume that all
materials listed are included in the syllabus. You are not expected to be able to reproduce
proofs of the theorems in the textbook.

1.7 Preparation and Workload


1.7.1 Prerequisites.
All students must have completed a course in functions — in particular, familiarity
with trigonometric functions is assumed; it is your responsibility as a student to verify
that you have this necessary background to benefit from this course. While a course in
high school calculus is the stated “prerequisite” for this course, stronger students having
only the prerequisite for MATH 139 (viz., “a course in functions”) have normally been
permitted to register at their own risk in MATH 140. Some of the prerequisites are
reviewed in Appendices A, B, C, D of your textbook [1]; solutions to odd-numbered
problems can be found in the Student Solution Manual [3].
Because weakness in pre-calculus topics can contribute to failure of a student in
MATH 140 or MATH 141, students will be tested on prerequisites in both the WeB-
WorK assignments A1 , A2 and quizzes Q1 (functions), Q2 (trigonometry)31 . Students
31
The grade on Q2 the grade does not count in your term mark.
Information for Students in MATH 140 2008 09 17

who do not believe they can remedy deficiencies on their own might wish to consider
MATH 11232 . MATH 112 2008 09 is scheduled at the same time as Section 1 of MATH
140 2008 09.

1.7.2 Calculators
The use of calculators is not permitted in either quizzes or the examination in this course.
Students whose previous mathematics courses have been calculator-oriented would be
advised to make particular efforts to avoid the use of a calculator in solving problems
in this course, in order to develop a minimal facility in manual calculation. This means
that you are urged to do all arithmetic by hand. If, for a problem presented to you
in this course, you find that you cannot solve it without using a calculator,
then you are almost certainly solving the problem in an unacceptable way.
Do not assume that, in excluding the use of calculators, your instructors are “Lud-
dites”33 ! We believe that you will master the materials in Calculus 1 and 2 best by relying
on manual calculations. In some more advanced mathematics and statistics courses you
will be encouraged to use calculators and computers.

1.7.3 Self-Supervision
MATH 140 is not a high-school course, and McGill is not a high school. The
monitoring of your progress before the final examination is largely your own responsibil-
ity. While the tutors and instructors are available to help you, they cannot do so unless
and until you identify the need for help. WeBWorK and quizzes are designed to assist
you in doing this.

Time Demands of your Other Courses. Be sure to budget enough time to attend
lectures and tutorials, for private study, and for the solution of many problems. Don’t be
tempted to divert calculus study time to courses which offer instant gratification. While
the significance of the tutorial quizzes in the computation of your grade is minimal, these
are important learning experiences, and can assist you in gauging your progress in the
course. This is not a course that can be crammed for: you must work steadily through
the term if you wish to develop the facilities needed for a strong performance on the final
examination.
32
MATH 112 Fundamentals of Mathematics (3) (Fall. Not open to students who have taken CEGEP
course 201-101. Open only to those students who are deficient in a pre-calculus background.) Equations
and inequalities, graphs, relations and functions, exponential and logarithmic functions, trigonometric
functions and their use, mathematical induction, binomial theorem, complex numbers.
33
opposed to technological innovation
Information for Students in MATH 140 2008 09 18

Working Problems on Your Own. An effective way to master the calculus is


through working large numbers of problems from the textbook. Your textbook was
selected partly because of the availability of an excellent Student Solutions Manual [3];
this manual has brief but complete solutions to most of the odd-numbered exercises in
the textbook. The skills you acquire in solving textbook problems could have much more
influence on your final grade than either WeBWorK or the quizzes.

The real uses of WeBWorK and the quizzes. Students often misunderstand the
true significance of WeBWorK assignments and the quizzes. While both contribute
to your grade, they help you estimate the quality of your progress in the course. Take
proper remedial action if you are obtaining low grades on quizzes34 , or if you require
many attempts before being able to solve a problem on WeBWorK. However, while
both WeBWorK and the quizzes have a role to play in learning the calculus, neither
is as important as reading your textbook, working problems yourself, and attending and
listening at lectures and tutorials.

Does a high grade on WeBWorK indicate the likelihood of a high grade on the
final examination? NO! The primary purpose of the WeBWorK assignments is as
an aid to learning; but, as your work is not being done under examination conditions, you
should not use the WeBWorK grades as indicators of your likely examination grade.
The grades on the quizzes are somewhat more useful for that purpose.

1.7.4 Escape Routes


At any time, even after the last date for dropping the course, students who are experi-
encing medical or personal difficulties should not hesitate to consult their advisors or the
Student Affairs office of their faculty. Don’t allow yourself to be overwhelmed by such
problems; the University has resource persons who may be able to help you.

1.7.5 Terminology
Do not be surprised if your instructors and tutors use different terminology from what
you have heard in your previous calculus course, particularly if that course was at a high
school. Sometimes the differences are purely due to different traditions in the professions.

“Negative x”. Your instructors and tutors will often read a formula −x as minus x,
not as negative x. To a mathematician the term negative refers to real numbers which
are not squares, i.e. which are less than 0, and −x can be positive if x itself is negative.
34
An unwise remedy would be is to miss the quizzes, and thereby avoid an unwelcome message.
Information for Students in MATH 140 2008 09 19

However, mathematicians will sometimes refer to the operation of changing a sign


as the replacement of x by “its negative”; this is not entirely consistent with the usual
practice, but is an “abuse of language” that has crept into the professional jargon.

Inverse trigonometric functions; f n . A formula like sin−1 x will be read as the


inverse sine of x — never sine to the minus 1 or sine to the negative 1 . However, if we
write sinn x, where n is a positive integer, it will always mean (sin x)n . These conventions
apply to any of the functions sin, cos, tan, cot, sec, csc; they also apply to the hyperbolic
functions, which we will meet in [1, §3.11]: sinh, cosh, tanh, coth, . . . . We will usually
not write exponents on general functions, so a formula like f 2 (x) does not have an obvious
meaning, and we will avoid writing it when f is other than a trigonometric or hyperbolic
function.

Logarithms. If you were taught to interpret log x as being the logarithm to base 10,
you should now forget that convention, at least in your mathematics courses; nowadays
mathematicians rarely use logarithms to base 10.35 Most often, if your instructor speaks
of a logarithm, and writes log x, he will be referring to the base e, i.e. to loge ; that is,
he is referring to the function that calculus books call ln. When a logarithm to some
other base is intended, it will either be denoted by an explicit subscript, as log2 , or some
comment will be made at the beginning of the discussion, such as “all logarithms in this
discussion are to the base 2”.

Your instructors try to think like mathematicians even when lecturing to their classes,
and to use the language and terminology we use when talking to each other.

1.8 High Technology and MATH 140


1.8.1 Keep your e-mail addresses up to date
• (From http://www.mcgill.ca/email-policy/): “ The McGill E-mail Address points
to the McGill e-mail box by default for all students. Students can have e-mail
addressed to their McGill E-mail Address point to a mailbox other than their
McGill e-mail box but it is the student’s responsibility to ensure that the alternate
mailbox is viable (so that delivered e-mail can be accessed promptly). Policy
recommendation is that students use their McGill e-mail box. ... Any official
McGill e-mail sent to students will be addressed to the McGill E-mail Address. It
is the student’s responsibility to ensure that time-critical e-mail is accessed, read
35
Your calculator keyboard may be using this convention, but you won’t be using the calculator in
this course.
Information for Students in MATH 140 2008 09 20

and acted upon in a timely fashion.” You can enter or change a forwarding e-
mail address by going to http://webmail.mcgill.ca, and logging in to your student
mailbox at po-box.mcgill.ca.

• The WeBWorK system permits you to designate any e-mail address; that is also
the address that will be used if you send a FEEDBACK message.

1.8.2 Use of Calculators and Computer Algebra Systems


You are urged to do all calculations manually, and to avoid the use of calculators and
computer algebra systems until you have completed MATH 141. You should not use a
calculator or computer in the solution of WeBWorK problems, as it prevents you from
developing skill for detecting errors in manual calculations — a skill that you will need
for the quizzes and final examination. It is also foolish to use a calculator to produce
data to enter into WeBWorK, since WeBWorK will accept the data that you would
type into your calculator, and will do the calculations internally for you.

1.8.3 Use of the Internet


Students are expected to be able to access materials through the Internet, whenever
required. Here are some of the uses that are expected:
• To access and submit WeBWorK (§1.4.2) assignments

• To access WebCT, where course grades and announcements will be mounted

• To access the web page for the course — also available through WebCT — where
these and other notes will be available in .pdf form; the site also contains notes
and examinations from previous years.

• In Lecture Section 002 the instructor will post notes regularly. Use of these notes
is optional, but they will probably be available only as .pdf files.

1.9 Which problems should I work?


1.9.1 There are no public or private restricted lists of textbook problems to
which you can confine your preparation for testing in MATH 140.
The syllabus is defined in terms of sections of the textbook, and the intention is that you
should be able to solve most of the problems in the designated sections, as well as other
types of questions that could be placed before you. Of course, there are certain questions
in the book that are not appropriate to this course, e.g., a small number of problems
near the ends of the exercise sets, which can be difficult and require some assistance; or
Information for Students in MATH 140 2008 09 21

problems requiring the use of computers; or problems that involve concepts that have
been excluded from the course.

1.9.2 Are the type of problems I find on WeBWorK problems indicative of


what I need to know in this course?
Only partially! The WeBWorK problems are simply a sampling, and some important
topics don’t appear at all in the WeBWorK assignments. The course is not defined by
the WeBWorK problems.

1.9.3 Problems discussed in the lectures


The problems cited by your instructors could be appropriate for their use at that time,
or could be a useful introduction to a topic, or could be otherwise worthy of attention.
If your instructor does refer to a problem, it would be wise to consider it, as his past
or subsequent lectures could refer to it. But you should not assume that every such
problem will ultimately end up on a test or examination. We are trying to introduce you
to a beautiful area of mathematics, and we sincerely wish that you will excel on tests
and examinations; but we are not teaching a high school course, and do not encourage
memorization or rote-learning.

1.9.4 And anyhow, the course is not concerned only with problem solving.
We are also trying to convey to you an understanding of basic theorems of the subject.
We want you to learn precisely what the theorems are saying, and how these results
can be applied. At the ends of the textbook chapters there are “Concept Checks” and
True-False Quizzes that can help you evaluate your understanding. These questions are
usually not in a form that lends itself to examinations, but they are excellent questions
to help you evaluate your progress. Some of the questions could be reformulated as
acceptable test questions.

1.9.5 Repairing your precalculus foundations.


Some students may find, through WeBWorK assignments A1 and A2 or Quiz Q1 , that
they need remedial work to rebuild the foundations required for a first calculus course.
If you are in this category, you are urged to expedite that study, so that it has minimal
effect on your progress through MATH 140. One of our goals in MATH 140 is to prepare
you for MATH 141, so that you will not have to face similar deficiencies when you enter
that course.
Information for Students in MATH 140 2008 09 22

1.10 Last Weeks of Term (to be updated)


1.10.1 Office Hours (additional to regular office hours)

Tutor Office Additional Office Hours


BURN Weekday Month Date Begins Ends
L. Bridgeman 1115 Wed Nov 26 12:00 13:30
I. Kazachkov 1029 Thurs Nov 27 15:00 16:00
L. Bridgeman 1115 Fri Nov 28 15:00 16:30
S. Nashaat 1132 Tue Dec 02 14:30 16:00
G. Gauthier-Shalom 1036 Wed Dec 03 13:00 16:00
S. Nashaat 1132 Thurs Dec 04 15:00 16:30
L. Bridgeman 1115 Fri Dec 05 16:00 18:00
(updated to December 2, 2008)
During her/his office hours, a tutor is available to all students in the course,
not only to the students of her/his tutorial section.

1.10.2 Review Tutorials (additional to regular tutorials)

Tutor Classroom Review Tutorial


BURN Weekday Month Date Begins Ends Topic
M. Wong LEA 26 Wed Dec 03 15:05 16:25 §4.1, 4.2, etc.
B. Cordy LEA 26 Wed Dec 03 16:35 17:25 §4.7, etc.
X. Zhang LEA 26 Wed Dec 03 17:35 16:25 §4.4 etc.
E. Haldane MAASS 10 Fri Dec 05 14:35 15:25 Chapter 4
L. El-Hajj MAASS 10 Fri Dec 05 15:35 16:25 Chapter 4
(updated to December 2, 2008)
Seating space permitting, review tutorials are available to all students in the course,
not only to the students of the TA’s tutorial section. For safety reasons, the number of
persons in the room is limited to those who can be seated.

1.11 Some of the best ways to ensure failure in MATH 140


1. Decide in advance that you won’t write quizzes, because a low quiz grade
will be replaced by the exam grade. There has been a strong correlation
between failure to write quizzes and failure in the course. The main purpose of
the quizzes is to provide you with timely feedback about the level of your progress
in the course, by grading your performance on problems that could be typical of
those in the textbook or on the examination.
Information for Students in MATH 140 2008 09 23

2. Don’t bother coming to class. Lectures can be painful: they are scheduled
either early in the morning or late in the afternoon, and the instructors don’t
consciously make them entertaining, as there is work to be done. That notwith-
standing, students who attempt to learn the material entirely on their own usually
find that it takes even more time than if they had gone to class, and is likely to
reinforce misunderstood concepts.

3. Attend tutorials only when there is a quiz.

4. Use a calculator or a computer or human aid when working WeBWorK


assignments. Since calculators are not permitted on tests or examinations, you
weaken the usefulness of WeBWorK assignments if you don’t solve the questions
(by yourself, and) by hand.

5. Spend more time on WeBWorK assignments than on working problems


in the textbook. Before attempting WeBWorK problems you should read the
relevant sections of the textbook, particular any worked problems. Cover the solu-
tions, and see if your solution resembles that in the book. Then work some of the
low-, odd-numbered problems in the textbook, and compare your solutions with
those in the Student Solution Manual. Only then are you ready to derive full ben-
efit from the WeBWorK assignment. Make the assignment into a learning aid,
rather than a chore or a way of collecting a few marks.

6. When tests in your other courses are imminent, skip Calculus.


Information for Students in MATH 140 2008 09 24

2 Solutions to Quiz Problems


2.1 Draft Solutions to Quiz Q1
Release Date: Wednesday, 15 October, 2008
These are sketches of possible solutions to types of problems on the quiz; they haven’t
been fully checked, and there could be some errors or misprints that need to be
corrected.
There were many versions of these quizzes. The solutions below are to typical examples,
with specific choices of data. In all cases it was intended that students provide supporting
explanations for their answers. Some of the quizzes contained a question in which the
student had to match a function with one of 5 sketches of graphs. If you gave your
students guidance about what sort of solution you expected, you can now grade with
that guidance in mind. If you gave no guidance, then it’s difficult to expect more than
a choice of one of the 5 possibilities, since it will be difficult to create a grading scheme.
Remember that the grades for the various sections will be analyzed, and there may
be some upward adjustment to reduce grading differences between the tutorial sections.
Notwithstanding this observation, you should try, with the other TA’s from the day of
your tutorial, to minimize the spread, so that the grades students receive will carry some
useful information.
¡√ ¢
1. [3 MARKS] Find the domain of the function h(x) = (x + 8)2 4 5x − 25 .
Solution: This function is the product of two functions. Thep first, the polynomial
(x + 8)2 , is defined for all real numbers x. But the second, 4 5(x − 5), is defined
only if the function under the root sign is not negative. Thus this latter factor
requires that x ≥ 5, and that restriction is sufficient for h to be well defined: the
domain is (5, +∞).

x+1
2. [3 MARKS] Find the domain of the function f (x) = .
x−2
Solution: This function is the quotient of two functions. The numerator is defined
when the argument of the square root is not negative, i.e., for x ≥ −1. The
denominator is defined for all x; but, it being a denominator, we require that it be
non-zero, requiring the exclusion of the value x = 2 from the domain. Thus the
domain of f is obtained by puncturing the interval [−1, +∞) at x = 2:
[−1, 2) ∪ (2, +∞) .

8 x − 11
3. [3 MARKS] Find the domain of (f ◦g)(x) if f (x) = , g(x) = . Express
x − 11 x+9
as an interval or union of intervals.
Information for Students in MATH 140 2008 09 25

µ ¶
x − 11 8 4 x+9
Solution: (f ◦ g)(x) = f (g(x)) = f = x−11 =− · , at all
x+9 x+9
− 11 5 x + 11
points in the domain of g which are also in the domain of f . We must be cautious
not to base our reasoning on the reduced form of the function given finally above:
all steps in the computation of the composite function must be meaningful at any
point in its domain. In this final form the factor x + 9 does not appear in the
denominator, and we might be tempted to conclude that −9 is in the domain
of the composite function; but that is not the case, because the first step in the
calculation requires that g be defined, and −9 is not in the domain of g.
The first function to be applied is g, whose domain is R − {−9}, since we must
delete from the domain of the numerator function x − 11 the one point where the
denominator would be equal to 0. Then we have to apply the function f , whose
domain is — by similar reasoning — R − {11}; but it is not x = 11 that must be
deleted from the domain R − {−9} — we must delete the value x, if any, where
g assumes the value 11: that is, we must delete from the domain any point(s) x
x − 11
such that 11 = . The only point x with this property is x = −11. Thus the
x+9
domain of f ◦ g is R − {−9, −11}.

4. [3 MARKS] Showing all your work, find the inverse function f −1 (x) of f (x) =
7 − 5x
.
6 − 2x
7 − 5x
(a) Set y = f (x) = . Note that the point x = 3, which causes the denomi-
6 − 2x
nator of the fraction to be 0, is not in the domain of f .
7 − 6y
(b) Solve for x in terms of y: x = ; excluded is the value y = 52 .
5 − 2y
(c) Rewrite, naming the independent variable x: the inverse function is given by
7 − 6x
f −1 (x) = . Its domain is R − { 52 }.
5 − 2x
[3 MARKS] Showing all your work, find the inverse function f −1 (x) of f (x) =
5. √
10x + 7.
√ 7
(a) Set y = f (x) = 10x + 7. Note that the domain of f is x ≥ − .
10
y2 − 7
(b) Solve for x in terms of y: squaring and reduction yield x = .
10
(c) Rewrite, naming the independent variable x: the inverse function is given by
x2 − 7
f −1 (x) = . Its domain is [0, +∞] (corresponding to the fact that the
10
Information for Students in MATH 140 2008 09 26

image of f is [0, +∞]. (For the purposes of this course it wouldn’t be wrong to
think of the domain of f −1 as being R; but the algebraic relationship between
f and its inverse is more attractive if we take the domain of each to be the
image of the other.)

6. [3 MARKS] Showing all your work, find the inverse function f −1 (x) of f (x) =
¡ ¢1
6 − x3 5 .
¡ ¢1
(a) Set y = f (x) = 6 − x3 5 . Note that the domain of f is R.
(b) p
Solve for x in terms of y: taking the 5th power and reducing yield x =
3
6 − y5.
(c) Rewrite, naming
√ the independent variable x: the inverse function is given by
−1 3
f (x) = x = 6 − x5 . Its domain is R.

7. [3 MARKS]
√ Showing all your work, find the inverse function f −1 (x) of f (x) =
9 + x + 2.

(a) Set y = f (x) = 9 + x + 2. Note that the domain of f is x ≥ −2, the set of
all points where the square root is defined.
(b) Solve for x in terms of y: subtracting 9 from both sides and squaring yield
x = (y − 9)2 − 2.
(c) Rewrite, naming the independent variable x: the inverse function is given by
f −1 (x) = (x − 9)2 − 2 = x2 − 18x + 79. Its domain is [9, +∞) (corresponding
to the fact that the image of f is [9, +∞]. For the purposes of this course it
wouldn’t be wrong to think of the domain of f −1 as being R; but the algebraic
relationship between f and its inverse is more attractive if we take the domain
of each to be the image of the other.)

[3 MARKS] Showing all your work, find the inverse function f −1 (x) of f (x) =
8. √
81 − x2 , (0 ≤ x ≤ 9).

(a) Set y = f (x) = 81 − x2 . If we had not additional information, we could
observe that the domain of f is −9 ≤ x ≤ 9, the set of all points where
the square root is defined. However, we have been told in advance that the
domain is only the interval 0 ≤ x ≤ 9.
p
(b) Solve for x in terms of y: squaring both sides and reducing yield x = ± 81 − y 2 .
If we had no additional information, we would have to stop here, and assert
that f does not have an inverse. However, we are told that the domain of
f is 0 ≤ x ≤ 9; this permits us to drop the minus sign, and to assert that
Information for Students in MATH 140 2008 09 27


y = 81 − x2 . We
√ can also observe that the values of this function range
between 0 and + 81 = 9.
(c) Rewrite, naming
√ the independent variable x: the inverse function is given by
−1
f (x) = 81 − x2 . Its domain is [0, 9] (corresponding to the fact that the
image of f is [0, 9]). Thus it happens that f is its own inverse!
9. [3 MARKS] Showing all your work, solve the equation x6 2x − 2x = 0.
Solution:
¡ ¢
x6 2x − 2x = 0 ⇔ x6 − 1 2x = 0
⇔ x6 − 1 = 0
since 2x is non-zero for all values of x ,
of which both +1 and −1 are (the only real) solutions.
10. [3 MARKS] Showing all your work, solve the equation e2x − 3ex + 2 = 0.
Solution: The given equation is a quadratic equation in ex . It factorizes to yield
(ex − 1) (ex − 2) = 0 .
This equation is satisfied when either of the two factors is 0, i.e., when either Case
1: ex = 1 or Case2: ex = 2.
Case 1: ex = 1. To solve this equation take (natural) logarithms of both sides, to
find that x = 0 uniquely.
Case 2: ex = 2. Here again, if we take logarithms of both sides, we obtain x =
ln 2.
Thus the given equation is satisfied only for x = 0, ln 2.
11. [3 MARKS] Solve the inequality x2 ex − 6ex < 0.
Solution:
¡ ¢
x2 ex − 6ex < 0 ⇔ x2 − 6 ex < 0
⇔ x2 − 6 < 0
since ex is positive for all x
³ √ ´³ √ ´
⇔ x+ 6 x− 6 <0
√ √
⇔ − 6<x< 6
since the product can be negative only if precisely one of the two factors is negative,
and that can be arranged only by placing x between the roots of the quadratic
function x2 − 6.
Information for Students in MATH 140 2008 09 28

12. [3 MARKS] Showing all your work, find in the interval [0, 2π) all solutions to the
equation sin x cos x − 4 sin x = 0.
Solution:
sin x cos x − 4 sin x = 0 ⇔ (sin x) · (cos x − 4) = 0.
But the second factor can never be zero, since the cosine cannot exceed 1 in magni-
tude. Thus we can divide by this second factor, leaving sin x = 0, which is satisfied
in the given interval for x = 0, π.

13. [3 MARKS] Showing all your work, find, in the interval [0, 2π), all solutions of the
equation sin2 x = 4 sin x + 5.
Solution: The given equation is quadratic in sin x:

sin2 x = 4 sin x + 5 ⇔ (sin x + 1)(sin x − 5) = 0 .

Of the two factors on the left side of this equation, the second can never be 0,
since the magnitude of the sine cannot exceed 1. Thus the equation is equivalent
to sin x + 1 = 0, which is equivalent to sin x = −1, which is satisfied in the given
interval only for x = 3π
2
.

14. [3 MARKS] Showing all your work, find, in the interval [0, 2π), all solutions of the
equation sin2 x − cos2 x = 0. (Hint: You may wish to use a trigonometric identity.)
Solution:
sin2 x − cos2 x = 0 ⇔ − cos 2x = 0 .
The cosine is equal to 0 only at odd integer multiples of π2 . For x ∈ [0, 2π),
2x ∈ [0, 4π); in this interval there are 4 such multiples: 2x = π2 , 3π
2
, 5π
2
, 7π
2
; hence
x = π4 , 3π
4
, 5π 7π
4
, 4
. (A more naive way to solve the equation would be to factorize
the difference of squares into (sin x − cos x)(sin x + cos x). The product can vanish
only with either of the factors is zero. None of the factors is 0 when the cosine is
0; (why?). Hence the equation is equivalent to, after dividing by cos2 x, (tan x −
1)(tan x+1) = 0. These values for the tangent function are attained at the bisectors
of the quadrants, etc.

15. [3 MARKS] Showing all your work, find, in the interval [0, 2π], all solutions of the
equation 2 sin x cot x + cot x = −1 − 2 sin x.
Solution:

2 sin x cot x + cot x = −1 − 2 sin x ⇔ (2 sin x + 1) (cot x + 1) = 0


⇔ 2 sin x + 1 = 0 or cot x + 1 = 0
or both.
Information for Students in MATH 140 2008 09 29

Case 1: 2 sin x + 1 = 0 ⇔ sin x = − 12 ⇒ x = 7π 11π


6
, 6 in the given interval [0, 2π).
3π 7π
Case 2: cot x + 1 → x = 4
, 4 in the given interval [0, 2π).

Thus, in the interval 0 ≤ x < 2π, there are precisely the 4 solutions given above.

16. [3 MARKS] Find all solutions of the following equation in the interval [0, 2π):
3 cot x − tan x = 0.
Solution: First observe that the tangent cannot be 0 at any solution to this equa-
tion, since the cotangent
√ must be defined.
√ Multiplying the equation by tan x yields
3 − tan2 x = 0 ⇔ ( 3 − tan x)( 3 + tan x) = √0. The equation is thus satis-
fied precisely for points where the tangent is ± 3, i.e., at the multiples of π3 :
x = π3 , 2π
3
, 4π
3
, 5π
3
.

17. [3 MARKS] Use an addition or subtraction formula to simplify the following equa-
tion. Then find all solutions in the interval [0, 2π): cos x cos 2x + sin x sin 2x = 0.
Solution: cos x cos 2x + sin x sin 2x = 0 ⇔ cos(2x − x) = 0 ⇔ cos x = 0 which is
satisfied in the given interval only for x = π2 and x = 3π
2
.

18. [3 MARKS] Use an addition or subtraction formula to simplify the following equa-
tion. Then find all solutions in the interval [0, 2π): sin 7x cos 6x − cos 7x sin 6x = 0.
Solution: sin 7x cos 6x − cos 7x sin 6x = 0 ⇔ sin(7x − 6x) = 0 ⇔ sin x = 0, which
is satisfied in the given interval only for x = 0, π.

19. [3 MARKS] Use a double- or half-angle formula to solve the following equation in
the interval [0, 2π): sin 2x + cos x = 0.
Solution: sin 2x + cos x = 0 ⇔ 2 sin x · cos x + cos x = 0 ⇔ (2 sin x + 1)(cos x) = 0.
The equation is satisfied when at least one of the 2 factors is zero. The first factor
is satisfied when sin x = − 12 , hence at x = 7π6
and x = 11π6
in the given interval.
The second equation is satisfied for x = π2 and x = 3π 2
in the given interval. These
are the 4 solutions to the given equation in the given interval.

20. [2 MARKS] This was a multiple choice question with 5 possible ¡answers ¢to the
question “Which of the following is the graph of the function 4 csc πx + π3 .
Solution: One of the “solutions” offered was a sine curve — not the graph of a
cosecant, which has to have points of infinite discontinuity. One way to eliminate
fallacious solutions was to evaluate the function at certain “special” points and
check whether the proposed graph had the desired property. For example, the
cosecant will be have an infinite discontinuity (and be undefined) when its argument
is 0. Of the 4 solutions remaining, two of them have this property. Of these, one
is the reflection of the other in the x-axis: One simply has to test whether the sign
Information for Students in MATH 140 2008 09 30

of the function was appropriate when, for example, the argument of the cosecant
is π2 , i.e., when x = π5 .

21. [2 MARKS] This was a multiple choice question with 5 possible answers to the
question “Which of the following is the graph of the function 2 sec (4πx − π).
Solution: One of the “solutions” offered was the negative of a tangent curve —
not the graph of a secant, which can never cross the x-axis. One way to eliminate
fallacious solutions was to evaluate the function at certain “special” points and
check whether the proposed graph had the desired property. For example, the
secant will be have an infinite discontinuity (and be undefined) when its argument
is π2 , i.e., with the given data, when x = 83 . Of the 4 solutions remaining, two of
them have this property. Of these, one is the reflection of the other in the x-axis:
One simply has to test whether the sign of the function was appropriate when, for
example, the argument of the secant is 0, i.e., when x = π4 .

22. [2 MARKS] This was a multiple choice question with 5 possible¡ answers ¢ to the
π
question “Which of the following is the graph of the function tan x + 4 .
Solution: Two of the “solutions” offered were reflections of a tangent curve in
the x-axis — i.e., graphs of functions which are decreasing rather than increasing.
Of the three solutions remaining, only one of them crosses the x-axis when the
argument of the tangent is 0, i.e., when x = − π4 .

23. [2 MARKS] This was a multiple choice question with 5 possible answers ¡ to
¢ the
question “Which of the following is the graph of the function 15 sec π x − 12 .
Solution: The given function has infinite discontinuities where the argument of
the secant is an odd integer multiple of π2 . Thus one can check the behavior of
the graph when, for the given data, x = 1; this eliminates 3 of the 5 proposed
solutions. Of the remaining two solutions, one is the reflection of the other in the
x-axis; we can check the sign of the function when the argument of the secant is
0, i.e., when, for the given data, x = 12 . This eliminates one of the remaining 2
possible solutions.

24. [3 MARKS] Showing all√ details of your work, determine the domain of g ◦ f , if
f (x) = x2 and g(x) = x − 9. Use interval notation to express the result as an
interval or union of intervals.
Solution: The function f has R as its domain. Function g(x) is defined wherever
the argument of the√square root pis not negative, i.e., for x ≥ 9 with the given data.
2
Hence (g ◦ f )(x) = x − 9 = (x − 3)(x + 3). This function is defined whenever
the factors x − 3 and x + 3 of the argument of the square root are both of the same
Information for Students in MATH 140 2008 09 31

sign — i.e., both positive or both negative. In the first case,


¾ ½ ¾
x−3 ≥ 0 x ≥ 3
⇔ ⇔ x ≥ +3 ,
x+3 ≥ 0 x ≥ −3

while, in the second case


¾ ½ ¾
x−3 ≤ 0 x ≤ 3
⇔ ⇔ x ≤ −3 .
x+3 ≤ 0 x ≤ −3

Thus the domain is the union of the intervals [3, ∞) and (−∞, −3].

2.2 Draft Solutions to Quizzes Q3


Release Date: Sunday, November 02nd, 2008
subject to correction
(The quizzes were administered during the week of October 20th, 2008.)

The following solutions are for typical problems on each of the versions.

2.2.1 Monday Versions


Instructions

1. Show all your work. Marks are not given for answers alone.

2. You must enclose this question sheet in your folded answer sheet.

3. Time = 25 minutes

4. No calculators are permitted


 √ 
 x + 31 if x < −6 
1. [10 MARKS] Let f (x) = ab if x = −6 . What values must be
 
x + 10 + a if x > −6
chosen for a and b in order to make this function continuous at x = −6?
Solution:

(a) [1 MARK] In each of the intervals (−∞, −6), (−6, +∞) the function is of one
of the types that we know from [1, Theorem 7, p. 124] to be continuous; the
two functions that are pieced together at x = −6 are continuous at that point.
This fact will ensure that we can determine the 1-sided limits of the function
at x = 6 by simply taking the value of the respective functions.
Information for Students in MATH 140 2008 09 32

(b) In the explanation following [1, Definition 1, p. 119], the textbook reduces the
proof of continuity at a point to 3 steps:
• The point in question must be in the domain of the function — i.e., the
function must have been defined at the point.
• The 2-sided limit of the function must exist at the point. More precisely
– The limit from the left must exist at the point.
– The limit from the right must exist at the point.
– The two one-sided limits must be equal.
• The 2-sided limit of the function must equal the function value.
i. [1 MARK] The function has been defined at x = 6, to be equal to the
product ab of the constants that need to be determined.

ii. [2 MARKS] lim − f (x) = −6 + 31 = 5 (by the continuity of the com-
x→−6
position of the square root function and the linear function x + 31).
iii. [2 MARKS] lim + f (x) = (−6) + 10 + a = 4 + a (by the continuity of the
x→−6
linear function x + 10 + a).
iv. [2 MARKS] Imposing the condition that the two one-sided limits must
exist yields the constraint 5 = 4 + a, implying that a = 1, and that the
2-sided limit be the common value of the 1-sided limits, i.e., 5.
v. [2 MARKS] Imposing the condition that the limit be equal to the function
value yields the equation 5 = ab = 1b.
We have determined the values of a, b, and ensured that, with the values a = 1,
b = 5, the function so defined is, indeed, continuous.

2. [10 MARKS] By using appropriate algebraic √ manipulations,


√ compute the following
x2 + 2 − 11
limit or show that it does not exist: lim . Don’t use L’Hôpital’s
x→3 x−3
rule or epsilon-delta methods!
Solution: The limit cannot be evaluated using the “Quotient Law”, since both
numerator and denominator have limit equal to 0 as x → 3.
√ √ Ã√ √ √ √ !
x2 + 2 − 11 x2 + 2 − 11 x2 + 2 + 11
lim = lim ·√ √
x→3 x−3 x→3 x−3 x2 + 2 + 11
(x2 + 2) − 11
= lim ¡√ √ ¢
x→3 (x − 3) x2 + 2 + 11
(x − 3)(x + 3)
= lim ¡√ √ ¢
x→3 (x − 3) x2 + 2 + 11
Information for Students in MATH 140 2008 09 33

x+3
= lim √ √ .
x→3 x2 + 2 + 11
After the transformations we have effected on the function, the numerator and
denominator of the quotient now are both functions whose limit we can easily
find (by virtue of the continuity of polynomials, the square root function, and
composition of continuous functions); moreover, the limit of the denominator is
not zero. We may then apply the Quotient Law, and conclude that
√ √
x2 + 2 − 11 x+3
lim = lim √ √
x→3 x−3 x→3 x2 + 2 + 11
lim (x + 3)
x→3
= ¡√ √ ¢
lim x2 + 2 + 11
x→3
3+3 3
= √ √ =√ .
9 + 2 + 11 11

3. [10 MARKS] Use the limit definition of the derivative to compute the derivative of
1
f (x) = √ at x = −3. Don’t use any of the differentiation rules of Chapter 3
x+4
of Stewart!
Solution:
1 1
√ −√
x+4 −3 + 4
f 0 (−3) = lim
x→−3 x − (−3)

1− x+4
= lim √
x→−3 (x + 3) x + 4
√ √
1− x+4 1+ x+4
= lim √ · √
x→−3 (x + 3) x + 4 1 + x+4
1 − |x + 4|
= lim √ √
x→−3 (x + 3) x + 4(1 + x + 4)
1 − (x + 4)
= lim √ √
x→−3 (x + 3) x + 4(1 + x + 4)
since x + 4 is positive for x near −3,
−1
= lim √ √
x→−3 x + 4(1 + x + 4)
−1 1
= lim √ √ =− .
x→−3 −3 + 4(1 + −3 + 4) 2
Information for Students in MATH 140 2008 09 34

2.2.2 Wednesday Versions


Instructions
1. Show all your work. Marks are not given for answers alone.
2. You must enclose this question sheet in your folded answer sheet.
3. Time = 25 minutes
4. No calculators are permitted

1. [10 MARKS] Show that the equation ln x = −3x + 6 has at least one root between
x = 1 and x = 2. Justify all your steps!
Solution:
(a) [2 MARKS] Define a function f (x) = ln x − (−3x + 6) = ln x + 3x − 6.
(b) [2 MARKS] We will apply the Intermediate Value Theorem to the function f
on the interval [1, 2].
(c) [2 MARKS] The function, being a sum of a logarithm and a polynomial, both
of which are continuous wherever they are defined, is also continuous. The
domain is (0, +∞), which includes the interval [1, 2].
(d) [2 MARKS] f (1) = ln 1 + 3 − 6 = −3. f (2) = ln 2 + 6 − 6 = ln 2. Since x > 1,
ln 2 > 0. Thus the function has values of opposite signs at the ends of the
interval [1, 2].
(e) [2 MARKS] By the Intermediate Value Theorem there exists at least one point
c between 1 and 2 where f (c) = 0. Equivalently, the equation ln x = −3x + 6
has a solution between 1 and 2.
2. [10 MARKS] By using appropriate algebraic √ manipulations, compute the following
2x2 − 5
limit or show that it does not exist: lim . Don’t use L’Hôpital’s rule or
x→−∞ −5x + 2
epsilon-delta methods!
Solution: We cannot evaluate this limit using the Quotient Law, since both nu-
merator and denominator become infinite as x → −∞.
q ¡ ¢

2
2x − 5 x2 2 − x52
lim = lim ¡ ¢
x→−∞ −5x + 2 x→−∞ x −5 + 2
√ q¡ x ¢
2
x · 2 − x52
= lim ¡ ¢
x→−∞ x −5 + x2
Information for Students in MATH 140 2008 09 35

q
|x| 2 − x52
= lim ¡ ¢
x→−∞ x −5 + 2
q x
−x 2 − x52
= lim ¡ ¢
x→−∞ x −5 + 2
x
since, as x → −∞, x is eventually negative
q
− 2 − x52
= lim
x→−∞ −5 + 2
³ q x ´
lim − 2 − x52
x→−∞
= ¡ ¢
lim −5 + x2
x→−∞
√ √
− 2 2
= = .
−5 5

3. [10 MARKS] Use the limit definition of the derivative to compute the derivative of
1
f (x) = . at x = −3. Don’t use any of the differentiation rules of Chapter
(x + 4)2
3 of Stewart!
Solution:
1 1

(x + 4)2 (−3 + 4)2
f 0 (−3) = lim
x→−3 x − (−3)
1 − (x + 4)2
= lim .
x→−3 (x + 3)(x + 4)2

In this fraction we cannot apply the Quotient Law, since both numerator and
denominator approach 0. But we can factorize the numerator and then remove
from numerator and denominator a common factor which is not equal to 0 at
x = −3:
−x2 − 8x − 15
f 0 (−3) = lim
x→−3 (x + 3)(x + 4)2

−(x + 3)(x + 5)
= lim
x→−3 (x + 3)(x + 4)2

−(x + 5)
= lim ,
x→−3 (x + 4)2
Information for Students in MATH 140 2008 09 36

at which point we can use the Quotient Law:


−((−3) + 5)
f 0 (−3) = = −2 .
(−3 + 4)2

2.2.3 Thursday Versions


Instructions
1. Show all your work. Marks are not given for answers alone.
2. You must enclose this question sheet in your folded answer sheet.
3. Time = 25 minutes
4. No calculators are permitted

1. [15 MARKS] By using appropriate algebraic ³ manipulations,


´ compute the following

limit or show that it does not exist: lim x + x2 + 4x . Don’t use L’Hôpital’s
x→−∞
rule or epsilon-delta methods!
Solution: We cannot evaluate the limit of this sum by adding the limits of the
summands, as both of them are infinite, and of opposite signs.
à !
³ √ ´ ³ √ ´ x − √x2 + 4x
lim x + x2 + 4x = lim x + x2 + 4x · √
x→−∞ x→−∞ x − x2 + 4x
¡√ ¢2
x2 − x2 + 4x
= lim √
x→−∞ x − x2 + 4x
x2 − |x2 + 4x|
= lim √
x→−∞ x − x2 + 4x
x2 − |x(x + 4)|
= lim √
x→−∞ x − x2 + 4x
x2 − |x| · |x + 4|
= lim √
x→−∞ x − x2 + 4x
x2 − (−x) · (−(x + 4))
= lim √
x→−∞ x − x2 + 4x
since x and x + 4 are ultimately negative,
x2 − x2 − 4x
= lim √
x→−∞ x − x2 + 4x
−4x
= lim √ .
x→−∞ x − x2 + 4x
Information for Students in MATH 140 2008 09 37

At this point we have transformed the problem to one where a limit law still can’t
be applied: here we have a ratio where both numerator and denominator become
infinite. However, it is possible to divide a factor x from both numerator and
denominator.
³ √ ´ −4x
2
lim x + x + 4x = lim q ¡ ¢
x→−∞ x→−∞
x − x2 1 + x4
−4x
= lim √ q
x→−∞ 4
x− x2 · 1 + x
−4x
= lim q
x→−∞ 4
x − |x| · 1 + x
−4x
= lim q
x→−∞ 4
x+x· 1+ x
since, as x → −∞, x is eventually negative,
−4
= lim q
x→−∞
1 + 1 + x4
and we have now transformed the problem to the limit of a fraction where numer-
ator and denominator both have a finite limit as x → −∞, and where the limit of
the denominator is not 0.
³ √ ´ −4
lim x + x2 + 4x = √ = −2 .
x→−∞ 1+ 1+0
2. [15 MARKS] Use the limit definition of the derivative to compute the derivative of
x+1
f (x) = 2 at x = 0. Don’t use any of the differentiation rules of Chapter 3 of
x +4
Stewart!
Solution:
x+1 0+1
2
− 2
f 0 (0) = lim x + 4 0 + 4
x→0 x−0
4(x + 1) − (x2 + 4)
= lim
x→0 4x (x2 + 4)
−x(x − 4)
= lim
x→0 4x (x2 + 4)
1 x−4 1 −4 1
= − lim 2 =− · = .
4 x→0 x +4 4 4 4
Information for Students in MATH 140 2008 09 38

2.2.4 Friday Versions


Instructions
1. Show all your work. Marks are not given for answers alone.
2. You must enclose this question sheet in your folded answer sheet.
3. Time = 25 minutes
4. No calculators are permitted
1. [15 MARKS] By using appropriate algebraic µ manipulations,
¶ compute the following
4 1
limit or show that it does not exist: lim √ − . Don’t use L’Hôpital’s
x→0 x x + 16 x
rule or epsilon-delta methods!
Solution: This function is the difference of two functions, neither of which has
a finite limit at x = 0; indeed, both of these functions have infinite limits of
opposite signs on the two sides of 0. We certainly cannot apply the Difference Law
here. However, by taking the two expressions to a common denominator, we can
transform the function into a function with the same limit — a functions whose
limit we will eventually be able to evaluate.
µ ¶ √
4 1 4 − x + 16
lim √ − = lim √
x→0 x x + 16 x x→0 x x + 16
µ √ √ ¶
4 − x + 16 4 + x + 16
= lim √ · √
x→0 x x + 16 4 + x + 16
16 − |x + 16|
= lim √ ¡ √ ¢
x→0 x x + 16 · 4 + x + 16
16 − (x + 16)
= lim √ ¡ √ ¢
x→0 x x + 16 · 4 + x + 16
since x + 16 > 0 for x near 0,
−x
= lim √ ¡ √ ¢
x→0 x x + 16 · 4 + x + 16
−1
= lim √ ¡ √ ¢,
x→0 x + 16 · 4 + x + 16
in which fraction numerator and denominator both have limits, and the limit of
the denominator is not 0.
µ ¶
4 1 1 1
lim √ − =− =− .
x→0 x x + 16 x 4(4 + 4) 32
Information for Students in MATH 140 2008 09 39

2. [15 MARKS] Use the limit definition of the derivative to compute the derivative of
1
f (x) = at x = −1. Don’t use any of the differentiation rules of Chapter
(x + 2)2
3 of Stewart!
1 1
2

(x + 2) (−1 + 2)2
f 0 (−1) = lim
x→−1 x − (−1)
1 − (x + 2)2
= lim
x→−1 (x + 1)(x + 2)2

−x2 − 4x − 3
= lim
x→−1 (x + 1)(x + 2)2

−(x + 1)(x + 3)
= lim
x→−1 (x + 1)(x + 2)2

−(x + 3)
= lim ,
x→−1 (x + 2)2

where the fraction now has numerator and denominator which both have a limit
as x → −1, and where the limit of the denominator is not 0.

−(x + 3) − lim (x + 3) −1 + 3
0 x→−1
f (−1) = lim = =− = −2 .
x→−1 (x + 2)2 lim (x + 2) 2 (−1 + 2)2
x→−1

2.3 Draft Solutions to Quizzes Q4


Release Date: Saturday, November 15th, 2008
subject to correction
(The quizzes were administered during the week of November 03rd, 2008.)

The following solutions are for typical problems on each of the versions.

Instructions
1. Show all your work. Marks are not given for answers alone.

2. You must enclose this question sheet in your folded answer sheet.

3. Time = 25 minutes.

4. No calculators are permitted.


The following are typical versions of each quiz, with solutions.
Information for Students in MATH 140 2008 09 40

2.3.1 Monday Versions


1. [10 MARKS] Compute the derivatives of the following functions. Fully simplify
your answers! You may use all the differentiation rules of Chapter 3 of Stewart.

(a) f (x) = 4sin x


Solution: This problem could be solved by “logarithmic differentiation”. That
method, however, is not discussed in the textbook until [1, §3.6], but problems
of this type appear in [1, §3.4].
d ¡ sin x ¢ d ¡ ln 4 ¢sin x
f 0 (x) = 4 = e
dx dx
d ¡ (ln 4)(sin x) ¢
= e
dx
¡ ¢ d
= e(ln 4)(sin x) · ((ln 4)(sin x))
dx
by the Chain Rule
¡ (ln 4)(sin x) ¢ d
= e · ln 4 sin x
dx
since ln 4 is a constant
= 4sin x (ln 4) cos x

sec 4x
(b) g(x) =
cot 4x
Solution: There are several ways of attacking this problem. It’s not com-
pletely clear what “simplify your answer as much as possible” means, except
that there shouldn’t be obvious further possible cancellations, like the same
factor in numerator and denominator of a fraction; and, where there is an ob-
vious factorization, that should have been done. When we have trigonometric
identities, there are many variations of solutions that could be considered
“simplified as much as possible”. One solution could be the following:
µ ¶
0 d sec 4x
g (x) =
dx cot 4x
((sec 4x · tan 4x)4) · cot 4x − sec 4s · ((− csc2 4x) · 4)
=
µ cot2 ¶
4x
sec 4x + sec 4x · csc2 4x
= 4
cot2 4x
¡ 2 ¢
= 4 sec 4x tan 4x + sec2 4x .
p √
(c) h(x) = 3x + x.
Information for Students in MATH 140 2008 09 41

Solution:
d ³¡ √ ¢1 ´
h0 (x) = 3x + x 2
dx
1 ³¡ √ ¢ 1 −1 ´ d ¡ √ ¢
= 3x + x 2 · 3x + x
2 µdx ¶
1 ³¡ √ ¢− 12 ´ 1 1 −1
= 3x + x · 3 + x2
2 2
³ ´ µ ¶
1 ¡ √ ¢− 12 1 −1
= 3x + x · 3+ x 2
2 2
µ ¶
1 1 1
= p √ · 3+ √ .
2 3x + x 2 x

2. [10 MARKS] In answering each of the following problems, show all your work, and
simplify your answer as much as possible.
¡ √ ¢
(a) Find f 0 (x), when f (x) = arctan x − x2 + 1 .
Solution: By the Chain Rule,
µ ¶
0 1 1 2 − 12
f (x) = √ · 1 − · (1 + x ) · 2x
1 + (x − 1 + x2 )2 2
µ ¶
1 x
= √ · 1− √
1 + x2 + (1 + x2 ) − 2x 1 + x2 1 + x2
Ã√ !
1 1 1 + x2 − x
= · √ · √
2 1 + x2 − x 1 + x2 1 + x2
Ã√ !
1 1 1 + x2 − x
= ·√ ¡√ ¢· √
2 1 + x2 1 + x2 − x 1 + x2
1 1
= ·
2 1 + x2

sin (4x2 )
(b) Evaluate lim without using L’Hospital’s Rule.
x→0 sin (5x2 )
Solution:
 
sin (4x2 )  sin (4x2 ) 1 4x2 
lim = lim  · · 
x→0 sin (5x2 ) x→0 4x2 sin (5x2 ) 5x2
5x2
Information for Students in MATH 140 2008 09 42

µ µ ¶¶ µ ¶
sin (4x2 ) 1 4x2
= lim · µ ¶ · lim
x→0 4x2 sin (5x2 ) x→0 5x2
lim
x→0 5x2
µ ¶
sin (4x2 )
lim
x→0 4x2 4
= µ 2
¶ · lim
sin (5x ) x→0 5
lim 2
x→0 5x
1 4 4
= · = .
1 5 5
2
3. [10 MARKS] Suppose that x2 + 64y 2 = (x2 + 2y 2 + 8x) . Determine an equation
— simplified as much as possible — for the tangent to this curve at the point with
coordinates (0, −4).
Solution: Differentiate both sides of the equation “implicitly” with respect to x:
¡ ¢
2x + 64 · 2y · y 0 = 2 x2 + 2y 2 + 8x · (2x + 2 · 2y · y 0 + 8)
2 (x2 + 2y 2 + 8x) (2x + 8) − 2x
⇒ y0 =
128y − 8y (x2 + 2y 2 + 8x)
2 · 32 · 8
⇒ when (x, y) = (0, −4), y0 = = 1.
128(−4) − 16(−4)3
It follows that the equation of the tangent — the line through (0, −4) with slope
1 — is y = −4 + 1(x − 0), i.e., x − y = 4.

2.3.2 Wednesday Versions


√ x
1. [10 MARKS] Let f (x) = 64 − x2 + x arcsin .
8
(a) Determine the domain of definition of f .
Solution: The function is defined as the sum of two functions: both of those
functions must be defined in the domain of f — i.e., the domain of f is
the intersection of the domains of the two summands. The second of the
summands is a product, so it is defined only where its factors are defined.
Thus the domain of f is the intersection of the domains of three functions:
√ x
64 − x2 , x, and arcsin . In the case of the last function listed, it is a
8
composition: its domain of definition consists of those x for which the first
x
function applied — — is defined, restricted to those values for which the
8
second function, the arcsine, is defined on the image of the first function
applied.
Information for Students in MATH 140 2008 09 43


i.64 − x2 is defined wherever the argument is non-negative, i.e., wherever
64 − x2 ≥ 0, i.e., wherever (x − 8)(x + 8) ≤ 0, i.e., wherever −8 ≤ x ≤ +8.
ii. The function x is defined on all of R.
x x
iii. arcsin is defined wherever −1 ≤ ≤ +1, i.e., on the interval −8 ≤ x ≤
8 8
+8.
It follows that the domain of f is the interval [−8, +8].
x
(b) Show that f 0 (x) = arcsin . Show all your steps!
8
Solution:

0 d√ 2
d ³ x´
f (x) = 64 − x + x arcsin
dx dx 8
by the Sum Rule of Differentiation
d√ dx x d x
= 64 − x2 + · arcsin + x · arcsin
dx dx 8 dx 8
by the Product Rule of Differentiation
1 ¡ ¢ 1 −1 d ¡ ¢ x 1 d ³x´
= · 64 − x2 2 · 64 − x2 + 1 · arcsin + x · q ·
¡ ¢2 dx 8
2 dx 8
1 − x8
by the Chain Rule of Differentiation
1 x 1
= √ · (−2x) + arcsin + x · q ¡ ¢2
2 64 − x 2 8
8 1− x 8
x x x x
= −√ + arcsin + q ¡ ¢2 = arcsin
64 − x2 8 8
8 1 − x8

2. [10 MARKS] Compute the derivatives of the following functions. Fully simplify
your answers! You may use all the differentiation rules of Chapter 3 of Stewart.

(a) f (x) = 2 x
Solution: This problem could be solved by “logarithmic differentiation”. That
method, however, is not discussed in the textbook until [1, §3.6], but problems
of this type appear in [1, §3.4].

d ³ √x ´ d ¡ ln 2 ¢√x
f 0 (x) = 2 = e
dx dx
d ³ (ln 2)√x ´
= e
dx
³ √ ´ d ¡ √ ¢
= e(ln 2) x · (ln 2) x
dx
Information for Students in MATH 140 2008 09 44

by the Chain Rule


³ √ ´ d ³ 1´
(ln 2) x
= e · ln 2 x2
dx
since ln 2 is a constant


x 1 1 −1 2 x−1 · (ln 2)
= 2 (ln 2) · x 2 = √
2 x

(b) g(x) = tan 2x · sec 2x


Solution: There are a number of different ways of formulating a solution.
Suppose that we temporarily define u = 2x, G(u) = tan u · sec u. Then, by
the Product Rule of Differentiation,
¡ ¢
G0 (u) = sec2 u · sec u + tan u · sec u tan u = sec u · sec2 u + tan2 u .

Then g(x) = G(2x), so


d(2x) ¡ ¢
g 0 (x) = G0 (2x) · = 2G0 (2x) = 2 sec(2x) · sec2 2x + tan2 2x .
dx

x−9
(c) h(x) = √
x+9
Solution: This function could be differentiated by the Quotient Rule, followed
by the Chain Rule for each of the summands. Here’s another way, using the
Reciprocal Rule and the Chain Rule:
µ√ ¶
0 d x−9
h (x) = √
dx x+9
µ√ ¶
d x + 9 − 18
= √
dx x+9
µ ¶
d 18
= 1− √
dx x+9
µ ¶
d d 1
= 1 − 18 √
dx dx x+9
à !
−1 d ¡√ ¢
= 0 − 18 √ 2 · x+9
( x + 9) dx
µ ¶
18 1 1
= √ 2 · · √ +0
( x + 9) 2 x
9
= √ √ 2
x · ( x + 9)
Information for Students in MATH 140 2008 09 45

3. [10 MARKS] Consider the curve C defined by the equation ye−5y + 5 = x2 + 6x +


xy + 13.

(a) Show that the point P = (−2, 0) lies on C. In how many points does C
intersect the x-axis? Justify!
Solution:
i. P will lie on C iff its coordinates satisfy the equation of C. When (x, y) =
(−2, 0), so
¡ −5y ¢ ¡ ¢
ye + 5 − x2 + 6x + xy + 13
¡ ¢ ¡ ¢
= 0e0 + 5 − (−2)2 + 6(−2) + (−2)(0) + 13 = 5 − (4 − 12 + 13) = 0 ,

showing that the coordinates of the point satisfy the equation of the curve,
which is what we mean when we say that the point is on the curve.
ii. The points (x, 0) where the curve crosses the x-axis are those whose ab-
scissa x satisfies the equation 0e−5(0) + 5 = x2 + 6x + x(0) + 13, i.e.,
5 = x2 + 6x + 13, i.e., (x + 4)(x + 2) = 0. One of the points is the point
(−2, 0); there is one other — (x, y) = (−4, 0).
(b) Find the equation of the tangent line to C at P .
Solution: We find the slope of the tangent by differentiating in the equation
of C “implicitly” with respect to x:

y 0 e−5y + ye−5y · (−5) · y 0 = 2x + 6 + y + xy 0 + 0

and then assign the values (x, y) = (−2, 0):

y 0 e0 + 0e0 · (−5) · y 0 = 2(−2) + 6 + 0 + (−2)y 0 + 0 ,


2
which implies that y 0 = at (x, y) = (−2, 0). The equation of the line with
3
2
slope 32 , passing through the point P is y = 0 + (x − 2), or 2x − 3y = 4.
3

2.3.3 Thursday Versions


1. [10 MARKS] Let f and g be differentiable functions defined for all real numbers.
The following values of f , g and their derivatives are known:

x f (x) g(x) f 0 (x) g 0 (x)


3 5 −3 2 3
5 2 3 2 2
Information for Students in MATH 140 2008 09 46

(a) Let u(x) = g(f (x)). Give a general formula which expresses u0 (x) in terms of
f , g and their derivatives. Using this formula, compute u0 (3).
Solution:

u0 (3) = g 0 (f (3)) · f 0 (3) by the Chain Rule,


0 0
= g (5) · f (3) from the table
= 2 · 2 = 4.
p
(b) Let v(x) = f (x) · g(x). Give a general formula which expresses v 0 (x) in
terms of f , g and their derivatives. Using this formula, compute v 0 (5).
Solution:
µ ¶
0 1 1 p
v (5) = · f (5) 2 · f (5) · g(5) + f (5) · g 0 (5)
−1 0
by the Chain Rule
2
1 1 p
= · f (5)− 2 · f 0 (5) · g(5) + f (5) · g 0 (5)
2
1 −1 √
= · 2 2 · 2 · 3 + 2 · 2 from the table
2 √
7 7 2
= √ = .
2 2

2. [10 MARKS] Compute the derivatives of the following functions. Fully simplify
your answers! You may use all the differentiation rules of Chapter 3 of Stewart.

(a) f (x) = 4sec x


Solution: This problem could also be solved by using “logarithmic” differenti-
ation. That method, however, is not discussed in the textbook until [1, §3.6],
but problems of this type appear in [1, §3.4].
d sec x d ¡ ln 4 ¢sec x
f 0 (x) = (4 ) = e
dx dx
d ¡ (ln 4)(sec x) ¢
= e
dx
¡ ¢ d
= e(ln 4)(sec x) · ((ln 4)(sec x))
dx
by the Chain Rule
¡ (ln 4)(sec x) ¢ d
= e · ln 4 · sec x
dx
since ln 4 is a constant
sin x
= 4 (ln 4) sec x · tan x
Information for Students in MATH 140 2008 09 47

r
1
(b) g(x) = x6 sin
x
Solution:
r õ ¶ 21 !
d 6 1 d 1
g 0 (x) = x · sin + x6 · sin
dx x dx x
r µ ¶ 1 −1 µ ¶
1 61 1 2 1
= 6x5 sin + x sin · − 2
x 2 x x
r
1 x6
= 6x5 sin − q
x 2 sin 1 · x2
x
r
5 1 x4
= 6x sin − q
x 2 sin 1
x

−4 + sin x
(c) h(x) =
8x + cos x
Solution: By the Quotient Rule,

(0 + cos x)(8x + cos x) − (−4 + sin x)(8 − sin x)


h0 (x) =
(8x + cos x)2
8x · cos x + cos2 x + sin2 x + 32 − 12 sin x
=
(8x + cos x)2
8x · cos x + 33 − 12 sin x
= .
(8x + cos x)2

x √
3. [10 MARKS] Let f (x) = x arccos − 9 − x2
3
(a) Determine the domain of definition of f .
Solution: Since 9 − x2 = −(x + 3)(x − 3) is negative precisely when either
x < −3 or√x > +3, these intervals must be deleted from R in the domain of the
function 9 − x2 . The arccosine function is defined only on the interval −1 ≤
x
x ≤ 1, so arccos is defined only on the interval −3 ≤ x ≤ 3; multiplication
3
x
of arccos by x does not affect the domain. To summarize: both of the
3
summands have, as their domain, the interval −3 ≤ x ≤ 3, so their sum has
the same domain.
Information for Students in MATH 140 2008 09 48

x
(b) Show that f 0 (x) = arccos . Show all your steps!
3
Solution:
 
x −1 1 1 ¡ ¢ 1 −1
f 0 (x) = 1 arccos + x · q ¡ ¢ ·  − · 9 − x2 2 · (−2x)
3 2 3 2
1 − x3
x x x x
= arccos − √ +√ = arccos
3 9−x 2 9−x 2 3

2.3.4 Friday Versions


1. [10 MARKS] Compute the derivatives of the following functions. Fully simplify
your answers! You may use all the differentiation rules of Chapter 3 of Stewart.

(a) f (x) = 4x sin x


Solution: This problem could be solved by “logarithmic differentiation”. That
method, however, is not discussed in the textbook until [1, §3.6], but problems
of this type appear in [1, §3.4].

d ¡ x sin x ¢ d ¡ ln 4 ¢x sin x
f 0 (x) = 4 = e
dx dx
d ¡ (ln 4)(x sin x) ¢
= e
dx
¡ ¢ d
= e(ln 4)(x sin x) · ((ln 4)(x sin x))
dx
by the Chain Rule
¡ (ln 4)(x sin x) ¢ d
= e · ln 4 · (x sin x)
dx
since ln 4 is a constant
x sin x
= 4 (ln 4)(sin x + x cos x)

(b) g(x) = tan sin x
Solution: This function is the composition of three functions: first applied is
the sine function, then the square root function, finally the tangent function.
√ 1 1
g 0 (x) = sec2 sin x · √ · cos x .
2 sin x
Information for Students in MATH 140 2008 09 49

sec x
(c) h(x) =
6 + sec x
Solution: We could apply the Quotient Rule in the usual way. However, it’s
easier to differentiate the cosine than the secant, so we can first observe that
sec x sec x cos x 1
h(x) = = · = ,
6 + sec x 6 + sec x cos x 6 cos x + 1
which, because its numerator is 1, can be differentiated using the Reciprocal
Rule or the Chain Rule for the reciprocal of a function:
−1 6 sin x
h0 (x) = 2
· (−6 sin x) = . (1)
(6 cos x + 1) (6 cos x + 1)2
Technically the “simplification” which I proposed introduced an error! The
cos x 2n + 1
ratio is not defined when the cosine is 0, i.e., whenever x = π,
cos x 2
where n is any integer. Such points should be excluded from equation (1);
also excluded would have to be those points x where cos x = − 16 , which are
not in the domain of h, so the function cannot be differentiable there.
2. [10 MARKS] Consider the curve C defined by the equation y · cos(4y) + 2x + 1 =
x2 − xy − 7.
(a) Show that the point P = (−2, 0) lies on C. In how many points does C
intersect the x-axis? Justify!
Solution:
i. P will lie on C iff its coordinates satisfy the equation of C. When (x, y) =
(−2, 0), so
¡ ¢
(y · cos(4y) + 2x + 1) − x2 − xy − 7
¡ ¢
= (0 · cos 0 + 2(−2) + 1) − (−2)2 − (−2)0 − 7
= (0 − 4 + 1) − (4 − 0 − 7) = −3 − (−3) = 0 ,
showing that the point is on the curve.
ii. The points (x, 0) where the curve crosses the x-axis are those whose ab-
scissa x satisfies the equation 0 · cos(0) + 2x + 1 = x2 − 0 − 7, i.e.,
2x + 1 = x2 − 7, i.e., (x − 4)(x + 2) = 0. One of the points is the point
(−2, 0); there is one other — (x, y) = (4, 0).
(b) Find the equation of the tangent line to C at P .
Solution: We begin by differentiating “implicitly” with respect to x in the
equation for C:
y 0 · cos(4y) + y · (− sin(4y)) · 4y 0 + 2 + 0 = 2x − (y + xy 0 ) − 0 .
Information for Students in MATH 140 2008 09 50

We need this information only when (x, y) = (−2, 0):


y 0 (−2) · cos 0 + 0(0) · 4y 0 (−2) + 2 = 2(−2) − (0 − 2y 0 (−2)) ⇒ y 0 (−2) = 6 .
The tangent line has slope 6 and passes through (−2, 0); its equation is y =
0 + 6(x − (−2)), i.e., 6x − y + 12 = 0.
3. [10 MARKS] In answering each of the following problems, show all your work, and
simplify your answer as much as possible.
¡√ ¢
(a) Find f 0 (x), when f (x) = −x2 + 1 arccos x.
Solution:
µ ¶ ³√ ´
0 d√ 2 −1
f (x) = −x + 1 · arccos x + −x2 + 1 · √
dx −x2 + 1
µ ¶ ³ ´
1 1 √ −1
= · √ · (−2x) · arccos x + −x + 1 · √
2
2 2
−x + 1 −x2 + 1
x arccos x
= −√ − 1.
−x2 + 1
t3
(b) Evaluate lim without using L’Hospital’s Rule.
t→0 tan3 7t
Solution:
µ ¶3
t3 t
lim = lim
t→0 tan3 (7t) t→0 tan(7t)
µ ¶3
t
= lim
t→0 tan(7t)

by the Product Law for Limits


µ ¶3
t cos(7t)
= lim
t→0 sin(7t)
µ ¶3
t
= lim cos(7t) · lim
t→0 t→0 sin(7t)
µ ¶3
t
= cos(0) · lim
t→0 sin(7t)

since the cosine function is continuous at 0


µ ¶3
t
= lim
t→0 sin(7t)
µ ¶3
1 7t
= lim
7 t→0 sin(7t)
Information for Students in MATH 140 2008 09 51

µ ¶3
1 7t
= 3 lim
7 t→0 sin(7t)
1 1
= 3 (1)3 = .
7 343

2.4 Draft Solutions to Quizzes Q5


Release Date: Thursday, November 27th, 2008
subject to correction
(The quizzes were administered during the week of November 17th, 2008.)
The following solutions are for typical problems on each of the versions.

Instructions Write your full solutions on the answer sheet. On any part of
any question part marks will be awarded only if the work is worth at least
half of the available marks.

2.4.1 Monday Versions


1. [12 MARKS] Use the definitions of the hyperbolic functions to find the limits. In
every case you are expected to first write the function in terms of exponentials,
and then to show all your work finding the limit. IT IS NOT SUFFICIENT TO
SIMPLY WRITE THE ANSWER; YOU MUST NOT USE L’HOSPITAL’S RULE.
(a) Find lim tanh x .
x→∞
(b) Find lim tanh x .
x→−∞
(c) Find lim sinh x .
x→0
(d) Find lim sech x .
x→∞

Solution:
(a)
ex − e−x
lim tanh x = lim
x→∞ x→∞ ex + e−x
ex (1 − e−2x )
= lim x
x→∞ e (1 + e−2x )

1 − e−2x
= lim
x→∞ (1 + e−2x )
1−0
= lim =1
x→∞ 1 + 0
Information for Students in MATH 140 2008 09 52

(b)

ex − e−x
lim tanh x = lim
x→−∞ x→−∞ ex + e−x
e−x (e2x − 1)
= lim −x 2x
x→−∞ e (e + 1)
2x
e −1
= lim 2x
x→−∞ e +1
0−1
= lim = −1
x→−∞ 0 + 1

(c)

ex − e−x
lim sinh x = lim
x→0 x→0 2
1−1
= =0
2

(d)
2
lim sech x = lim
x→∞ x→∞ ex + e−x
Here the denominator is the sum of two terms; the first approaches +∞, while
the second approaches 0 as x → ∞. Thus the denominator → ∞, and the
fraction → 0.

2. [6 MARKS] Two sides of a triangle have lengths 10 m and 14 m. The angle


between them is increasing at a rate of 2◦ /min. How fast is the length of the
third side increasing when the angle between the sides of fixed length is 60◦ ? The
instructors are aware that you do not have the use of a calculator.
Solution: Denote the angle between the√sides of fixed length by θ. By the Law of
Cosines the length of the third side is 102 + 142 − 2 · 10 · 14 · cos θ, which I will
denote by f (θ). Then
1 1
f 0 (θ) = ·√ · (−2 · 10 · 14 · (− sin θ)) .
2 2 2
10 + 14 − 2 · 10 · 14 · cos θ
so
d 1 1 dθ
f (θ) = · √ · (−2 · 10 · 14 · (− sin θ)) · .
dt 2 2 2
10 + 14 − 2 · 10 · 14 · cos θ dt
Information for Students in MATH 140 2008 09 53

When θ = π3 ,
³π ´ 1 2 · 10 · 14 · sin π3 2π
f0 = ·p 2 ·
3 2 10 + 142 − 2 · 10 · 14 · cos π3 180

1 2 · 10 · 14 · 23 π
= ·q ·
2 102 + 142 − 2 · 10 · 14 · 1 90
2

= √ .
18 13

3. [6 MARKS] Showing all your work, determine the global (absolute) maximum,
x
and the global (absolute) minimum values of the function 2 on the interval
x + 25
[−1, 10].
Solution: This function is the ratio of two polynomials, and is continuous wherever
it is defined; its domain is the portion of R where the denominator is non-zero,
i.e., all of R; we are instructed to confine our attention to the closed subinterval
[−1, 10], and so we may apply the “Closed Interval Method” [1, p. 275]: the function
will attain a global maximum and a global minimum at points in the interval.
Differentiation yields
1 (x2 + 25) − x(2x + 0) (5 − x)(5 + x)
2 = ,
2
(x + 25) (x2 + 25)2
which can vanish only at the points x = ±5. Of these two points, only x = 5 is in
the interval of interest, so it is the only critical number relevant to this problem.
−1
At the left end-point x = −1, the function is equal to ; at the right end-point
26
10
x = 10, the function is equal to . The global extrema will be attained at two
125
of these numbers. Considered the tabulated values
x −1 5 10
1 5 1 10
function value − 26 50
= 10 125

−1
we see that the global minimum value of is attained at x = −1 , and the global
26
1
maximum value of is attained at the internal critical point x = 5 ; the right
10
end-point is neither a global minimum point nor a global maximum point.
4. [6 MARKS] Showing all your work, find the linearization L(x) of the function
1
f (x) = √ at a = 0, and use it to approximate the value of f (5). You do not
7+x
Information for Students in MATH 140 2008 09 54

need to simplify your numerical answer, as the instructors are aware that you do
not have the use of a calculator.
Solution:
1 1 3
f (x) = √ ⇒ f 0 (x) = − · (7 + x)− 2
7+x 2
1 3
⇒ f 0 (a) = − · 7− 2
2
⇒ L(x) = f (a) + f 0 (a) · (x − a)
1 1 −3 1 ³ x´
⇒ L(x) = 7 − · 7 · (x − 0) = √ 1 −
2 2
2 7 14
1 1 −3 9
⇒ L(5) = 7 2 − · 7 2 · (5) = 3 ≈ 0.242977 . . . .
2 2 · 72
The correct value is approximately 0.288675. . . .

2.4.2 Wednesday Versions


1. [6 MARKS] Showing all your work, find the linearization L(x) of the function
f (x) = ln 9x at a = 19 , and use it to approximate the value of f (9). You should
simplify your answer as much as possible, but the instructors are aware that you
do not have the use of a calculator.
Solution:

f (x) = ln 9x
1
⇒ f 0 (x) =
µ ¶ x
1
⇒ f0 = 9
9
µ ¶ µ ¶ µ ¶
1 0 1 1
⇒ L(x) = f +f · x−
9 9 9
µ ¶ µ ¶
1 1
= ln 9 · +9· x−
9 9
= 9x − 1 .

Hence L(9) = 9(9)−1 = 80. This “approximate” value is a very bad approximation;
the correct value of the function is approximately 4.39445. Why is this so bad an
approximation? For one thing, the point where we wish to approximate the value
is “far” from the point of tangency. If we had used the approximation to find,
say, f (0.1), we would have obtained the approximation −0.1, which is not so bad
Information for Students in MATH 140 2008 09 55

an approximation of ln(0.9), whose correct value is −0.1054. Here the percentage


error is approximately 5%, whereas the error in approximating f (9) was about
75.6
4.4
= 1700%.
dx dy dz
2. [4 MARKS] If z 2 = x2 + y 2 , = 4, and = 2, find when x = 24 and y = 18.
dt dt dt
Solution: When (x, y) = (24, 18), z 2 = 900 = 302 , so z = ±30. Differentiating the
dz dx dy
defining equation implicitly with respect to z yields 2z · = 2x · + 2y · ,
dt dt dt
dz
implying that z · = 4x + 2y. When (x, y) = (24, 18),
dt
dz 4x + 2y 4(24) + 2(18) 22
= = =∓ .
dt z ±30 5

3. [3 MARKS] By applying the definition of the hyperbolic function in terms of ex-


ponentials, find the exact value of sinh ln 2.
Solution:
eln 2 − e− ln 2
sinh ln 2 =
2
1
2−
= 2 = 3.
2 4

4. [8 MARKS] The altitude of a triangle is increasing at a rate of 5 cm/min, while


the area of the triangle is increasing at a rate of 4 cm2 /min. Determine the rate at
which the base of the triangle is changing when the altitude is 2 cm and the area
is 70 cm2 .
Solution: To express our solution concisely, let’s denote the lengths of the base and
height of the triangle by b and h, and the area by A. Then the relationship from
1
which we will be extracting our information will be A = · bh. Differentiating im-
2
µ ¶ db
dA 1 dh db 5b + ·h
plicitly with respect to time t yields = b· + · h , so 4 = dt .
dt 2 dt dt 2
db
70 5 · 70 + ·2
When h = 2 and A = 70, b = 2 · = 70, so 4 = dt , so db = −171.
2 2 dt
5. [6 MARKS] Showing all your work, determine the global (absolute) maximum and
x
global (absolute) minimum values of 2 on the interval [−1, 20].
x + 100
Information for Students in MATH 140 2008 09 56

x
Solution: Let f (x) = .
x2 + 100
0 1(x2 + 100) − x(2x)
f (x) =
(x2 + 100)2
(10 − x)(10 + x)
=
(x2 + 100)2
The critical points are x = ±10, and the end-points of the domain — where the 2-
sided derivative is not defined — are −1 and 20. These 4 points are the candidates
for absolute extrema. There is no need to use the first or 2nd derivative tests,
since, in any case, we must test 3 of these points. The point x = −10 is outside of
the prescribed domain of the function, and can be ignored.
1
f (−1) = −
101
10 1
f (10) = =
200 20
20 1 1
f (20) = = < = f (10).
500 25 20
The global maximum occurs at x = 10, where the global maximum value is seen to
1
be . The global minimum occurs at x = −1, where the global minimum value is
20
1
seen to be − . The end-point x = 20 does not figure as a global extremal point.
101
sinh x
6. [3 MARKS] Evaluate the following limit without using L’Hospital’s Rule: lim .
x→∞ ex
(Hint: Remember the definition of the hyperbolic function in terms of exponen-
tials.)
Solution:
ex − e−x
sinh x 2
lim = lim
x→∞ ex x→∞ ex
ex − e−x
= lim
x→∞ 2ex
1 − e−2x
= lim
x→∞
¡ 2 −2x ¢
lim 1 − e
= x→∞
lim 2
x→∞
1−0 1
= = .
2 2
Information for Students in MATH 140 2008 09 57

2.4.3 Thursday Versions


1. [6 MARKS ALTOGETHER] Express the value of each expression in terms of ex-
ponentials, reduced as much as possible. The instructors are aware that you do
not have the use of a calculator.

(a) sinh(ln 4)
(b) sinh 2

Solution:

(a)

eln 4 − e− ln 4
sinh ln 4 =
2
1
4−
= 4 = 15 .
2 8

e2 − e−2 e4 − 1
(b) sinh 2 = = .
2 2e2
2. (a) [4 MARKS] √ Showing all your work, find the linear approximation of the func-
tion g(x) = 3 1 + x at a = 0.

(b) [2 MARKS] Use the linear approximation from part (a) to approximate 3 0.93.
Your answer should be simplified as much as possible, but you are not expected
to perform calculations for which you would normally require a calculator.

(c) [2 MARKS] Use the linear approximation from part (a) to approximate 3 1.05.
Your answer should be simplified as much as possible, but you are not expected
to perform calculations for which you would normally require a calculator.
Solution:
√ 1 2 1
(a) Since g(x) = 3
(1 + x)− 3 . Hence g(0) = 1, g 0 (0) = . The
1 + x, g 0 (x) =
3 3
linear approximation about x = 0 is L(x) = 1 + 31 (x − 0).
√ √
(b) To approximate 3 0.93 = 3 1 − 0.07 we take L(−0.07) = 1 − 0.07 3
= 293
300

0.97667; the correct value is 0.9761....
√ p
(c) To approximate 3 1.05 = 3 (1 + 0.05) we take L(0.05) = 1 + 0.05 3
= 305
300

1.01667; the correct value is 1.016396. . . .
Information for Students in MATH 140 2008 09 58

3. [6 MARKS] Showing all your work, determine the global (absolute)√maximum,


and the global (absolute) minimum values of the function f (t) = 5t 64 − t2 on
the interval [−5, 8].
Solution: This function is a product of a polynomial and a root function, and is
continuous wherever it is defined; its domain is the portion of R where the argument
of the square root function is non-negative, i.e., [−8, 8]. Since the given interval is
closed, the function will attain a global maximum and a global minimum at points
in the interval. Differentiation yields
√ 1 1
f 0 (t) = 5 64 − t2 + 5t · · √ · (−2t)
2 64 − t2
10(32 − t2 )
= √
64 − t2
√ √
10(4 2 − t)(4 2 + t)
= √ .
64 − t2
√ √
The critical points of the function are t = ±4 2. However, since (−4 2)2 = 32 >
(−5)2 , the negative of these is outside of the interval of definition of the function,
and can be ignored. By the “Closed Interval Method” we need to test the one
remaining critical point and the two end-points
√ of the interval, which are the 3
candidates for extrema: t = −5, t = 4 2, t = 8. We need only evaluate the
function at these 3 points and compare the values; there is no advantage to be
gained in using the First or Second Derivative Tests:
t √ f (t)
−5
√ −25 49 = −175 < 0 .
4 2 160 > 0
8 0

The global maximum point is at t = 4 2, and the global maximum value is 160.
The global minimum point is at t = −5, and the global minimum value is −175.
The point t = 8 is neither a maximum point nor a minimum point. Note that the
question asked for the values, not the locations where those extremal values are
attained.
5 + sinh x
4. [4 MARKS] Find the derivative of G(x) = .
5 − sinh x
Solution: This function can be differentiated by the Quotient and Chain Rules,
but I prefer the following approach (which gives the same answer):
5 + sinh x 10 − (5 − sinh x)
=
5 − sinh x 5 − sinh x
Information for Students in MATH 140 2008 09 59

1
= 10 · −1
µ ¶ 5 − sinh x
d 5 + sinh x −1
⇒ = 10 · · (0 − cosh x)
dx 5 − sinh x (5 − sinh x)2
−10 cosh x
= .
(5 − sinh x)2

5. [6 MARKS] Showing all your work, determine all the critical numbers of the func-
tion f (θ) = 2 cos θ − sin2 θ.
Solution: The critical points are the points where either f 0 = 0 or where f is
not differentiable. This function is a sum of multiples of sines and cosines, and is
differentiable everywhere; f 0 (θ) = −2 sin θ − 2 sin θ cos θ = −2 sin θ(1 + cos θ). The
derivative vanishes where either sin θ = 0 or where 1 + cos θ = 0, or both. The
sine vanishes when its argument is an integer multiple of π; the cosine is equal to
−1 when its argument is an odd multiple of π. Thus the critical numbers are all
the integer multiples of π, since the first factor vanishes at all of these points; the
vanishing of the second factor occurs only at some of these points, so it adds no
points to the list of critical points. The set of critical points is infinite!
This function is periodic with period equal to 2π;
³ √ ´³ √ ´
f (θ) = 2 cos θ − (1 − cos2 θ) = cos θ + 1 + 2 cos θ + 1 − 2 .

When θ is an even integer multiple of π the function has its local maxima, of value 2;
when θ is an odd integer multiple of π the function has its local minima of −2. It is an
even function. f 00 (θ) = 2(2 cos θ + 1)(cos θ − 1). The second factor is either negative or
0; the first factor changes sign around the points where cos θ = − 21 ; thus the inflection
¡ ¢
points are at θ = 2n ± 23 π, where n is any integer.

2.4.4 Friday Versions


1. [6 MARKS ALTOGETHER] The radius of a circular disk is given as 50 cm with
a maximum error in measurement of ±0.2 cm.

(a) [4 MARKS] Showing all your work, use differentials to estimate the maximum
error in the calculated area of the disk.
(b) [2 MARKS] Showing all your work, determine the maximum relative error.

The instructors are aware that you do not have use of a calculator.
Solution:
Information for Students in MATH 140 2008 09 60

dA
(a) Denote the radius by r, and the area function by A(r) = πr2 . Then = 2πr,
dr
so the increment in area corresponding to an increment in radius of ∆r is
given approximately by ∆A = dA dr
· ∆r. When r = 50, and ∆r = ±0.2,
∆A = (2π · 50)(±0.2) = ±20π.
±20π 1
(b) The relative error is approximately 2
= .
π(50) 1250
2. [6 MARKS] Showing all your work, find the global (absolute)
√ maximum and the
global (absolute) minimum values of the function f (t) = 4 t·(10−t) on the interval
[0, 10].
Solution:

(a) The given function is a product of a “root” function and a polynomial, and is
continuous wherever the factors are defined. Since it is continuous on a closed
interval, it will attain global extrema there.
(b) We have, by the Product and Quotient Rules of Differentiation,

4 1 5
f (t) = t · (10 − t) = 10t 4 − t 4
5(2 − t)
f 0 (t) = 3 .
4t 4
We see that this function has a critical point at t = 2. Such a point could
be a local extremum, or might be only an inflection point. We don’t need to
follow up on this part of the investigation here, since we are interested only
in global extrema.)
(c) To locate the global extrema, we need to test the function at every (internal)
critical point, and also at the end-points of the interval, i.e., at 0 and 10. I
tabulate the results:
x f (x)
0 √ 0
2 42·8>0
10 0
We see that the global minimum value is 0, and that it is attained at two
points
√ in this closed interval — at the end-points. The global maximum value
4
is 2 · (10 − 2), a value which is attained at only one point in the interval.
The problem, as stated, asked only for the extremal values.

3. [18 MARKS ALTOGETHER] Use the definitions of the hyperbolic functions to


find the limits. In every case you are expected to first write the function in terms
Information for Students in MATH 140 2008 09 61

of exponentials, and then to show all your work finding the limit. IT IS NOT
SUFFICIENT TO SIMPLY WRITE THE ANSWER; YOU MUST NOT USE
L’HOSPITAL’S RULE.

(a) [3 MARKS] lim tanh x


x→−∞

(b) [3 MARKS] lim sinh x


x→−∞

(c) [3 MARKS] lim sech x


x→∞

(d) [3 MARKS] lim coth x


x→∞

(e) [3 MARKS] lim+ coth x


x→0

(f) [3 MARKS] lim− coth x


x→0

Solution:

(a)

ex − e−x
lim tanh x = lim
x→−∞ x→−∞ ex + e−x
e−x (e2x − 1)
= lim −x 2x
x→−∞ e (e + 1)
2x
e −1
= lim 2x
x→−∞ e +1
0−1
= lim = −1
x→−∞ 0 + 1

(b)

ex − e−x
lim sinh x = lim
x→−∞ x→−∞ 2
x
e e−x
= lim − lim
x→−∞ 2 x→−∞ 2
−x
e
= 0 − lim
x→−∞ 2
= −∞

(c)
2
lim sech x = lim
x→∞ x→∞ ex + e−x
Information for Students in MATH 140 2008 09 62

Here the denominator is the sum of two terms; the first approaches +∞, while
the second approaches 0 as x → ∞. Thus the denominator → ∞, and the
fraction → 0.
(d)

ex + e−x
lim coth x = lim
x→+∞ x→+∞ ex − e−x
ex (1 + e−2x )
= lim x
x→∞ e (1 − e−2x )

1 + e−2x
= lim
x→∞ (1 − e−2x )
1+0
= lim =1
x→∞ 1 − 0

(e)

ex + e−x
lim+ coth x = lim+
x→0 x→0 ex − e−x
ex (1 + e−2x )
= lim+ x
x→0 e (1 − e−2x )

1 + e−2x
= lim+
x→0 1 − e−2x

When x > 0 ex > 1, so e−x < 1, and 1 − e−2x > 0. The denominator is
approaching 0 as x → 0+ ; the numerator approaches 2. Thus the faction
remains positive, but approaches +∞.
(f) This problem is similar to the preceding, except that the denominator is ap-
proaching 0 on the negative side; the one-sided limit is −∞.
Information for Students in Lecture Section 1 of MATH 140 2008 09 63

3 Information Specifically for Students in Lecture


Section 001

UPDATED TO December 2, 2008


Information for Students in Lecture Section 2 of MATH 140 2008 09 2001

4 Information Specifically for Students in Lecture


Section 002
4.1 Timetable for Lecture Section 002 of MATH 140 2008 09
Distribution Date: Wednesday, September 03rd, 2008
This revision as of October 22nd, 2008. (All information is subject to change.)

This Week, Until


MONDAY WEDNESDAY (Next Sun. 24:00)
SEPTEMBER
01 Labour Day 03 §1.1, App. A
All Tutorial Sections meet first during the week of September 08, 2008.
08 §1.2, App. D 10 §1.3, App. D P1 , P2
Course changes must be completed by Tuesday, September 16, 2008
15 §1.5, App. D, §1.6 17 §1.6 A1 , A2
Deadline for withdrawal from course with fee refund = September 21, 2008
Quiz Q1 at your third tutorial
22 §2.1, §2.2 24 §2.4(not exam. material), P3 , Q1
§2.3
29 §2.5
OCTOBER
Quiz Q2 at lecture of 01 October, 2008
01 Q2 A3
06 §2.6, §2.7 08 §2.8 P4 (Mon. 13 Oct.,
23:30h)
Next week only students of cancelled Monday tutorials T003–T008 may sit in on tutorials
on Wednesday, Thursday, or Friday, space permitting.
13 Canadian Thanksgiving 15 §3.1,§3.2 A4
Deadline for withdrawal (with W) from course = Oct. 19, 2008
Quiz Q3 at tutorials, 20-24 October, 2008
20 §3.3, §3.4 22 §3.4, §3.5 Q3 , P5
27 §3.6, §3.9 29 §3.7(part), §3.9 A5
Notation: An = WeBWorK Assignment An due at 23:30h on Sunday this week
Pn = WeBWorK Practice Assignment Pn due at 23:30h on Sunday this week
(The Practice Assignments do not count in your grade.)
Qn = Quiz Qn will be administered at the tutorials this week.
X = reserved for eXpansion or review
Section numbers refer to the text-book.

UPDATED TO December 2, 2008


Notes for Lecture Section 002, MATH 140 2008 09 2002

This Week
MONDAY WEDNESDAY (Until Sun. 24:00)
NOVEMBER
Quiz Q4 at tutorials, 03-27 November, 2008
03 §3.10, §3.11 05 §4.1 Q4 , P6
10 §4.2 12 §4.3 A6
Quiz Q5 at tutorials, 17-21 November, 2008
17 §4.4 19 §4.5 — omit “slant asymp- Q5 , P7
totes”
Next week’s tutorial is the last.
24 §4.7 26 §4.9 A7
DECEMBER
Last lectures in both lecture sections are on Tuesday, December 02nd, 2008.
01 X 02 (Tuesday, replaces lecture
lost on Canadian Thanks-
giving) X
Notation: An = WeBWorK Assignment An due at 23:30h on Sunday this week
Pn = WeBWorK Practice Assignment Pn due at 23:30h on Sunday this week
(The Practice Assignments do not count in your grade.)
Qn = Quiz Qn will be administered at the tutorials this week.
X = reserved for eXpansion or review
Section numbers refer to the text-book.
Notes for Lecture Section 002, MATH 140 2008 09 2003

5 Notes for Lecture Section 002


5.1 Supplementary Notes for the Lecture of September 3th,
2008
Release Date: Wednesday, September 3rd, 2008
subject to correction

Tutorials in MATH 140 2008 09 begin next week.

5.1.1 Lecture style in Lecture Section 002


Lecture content The timetable on pages 2001, 2002 will show you approximately
what I plan to discuss at each lecture. I suggest that you look through the material in
advance. If you have time to try some of the exercises, and find some that cause you
difficulty, you are welcome to bring them to my attention; perhaps I may be able to work
some of these examples into the lecture.

What goes on the chalkboard? — Should I take notes? Your instructor believes
strongly that students should not sit in the lecture feverishly copying notes for fear of
missing some essential topic; in this course most of what you need to know is contained
in the textbook. You should take notes, but you should be trying to think at the same
time. The chalkboard will be used for

• statement/illustration of specific definitions and theorems

• sketching solutions to problems, or classes of problems

• a scratchpad

Some of this material will be useful to you in learning the material in the course. Even
when the material on the board is equivalent to something in your textbook, the act of
writing may help you remember it. But much of the material will be restatements of
your textbook, so you should normally not panic if you miss something.

Graphs Several topics in the syllabus, culminating in [1, §4.5], are concerned with
sketching of curves. Our emphasis is on qualitative properties of the graphs of functions,
but not on the production of extremely precise graphs. You can expect to see me draw on
the chalkboard sketches that are extremely crude approximations of functions, sometimes
even caricatures of the true graph. Mathematicians do not base proofs on sketches of
graphs — the role of a sketch is usually only to assist the reader to visualize the verbal or
Notes for Lecture Section 002, MATH 140 2008 09 2004

symbolic reasoning which accompanies it. Sometimes a graph is used help one discover a
phenomenon, but the result would not be acceptable to a mathematician unless it could
be proved in a non-graphical way.36

These supplementary notes Section and paragraph headings follow the order of
topics in the textbook. While some of the comments or explanations may be helpful in
understanding the book, the notes are not required reading for examination purposes.
Sometimes it may happen that the discussion of a topic or an exercise evolves during the
lecture into one which requires more detail than is practical to write on the chalkboard.
In such cases you may be referred at the following lecture to supplementary material
that will be contained in the notes placed on the Web. Such evolutions are spontaneous
and not planned, and cannot be announced in advance.

Timing and corrections The notes will usually not be posted until after the lecture.
While I do try to check the notes before posting them, there will inevitably be errors:
if you see something that doesn’t look right, please ask. The notes will be progressively
corrected as misprints and other errors are discovered.

The supplementary notes for the first lectures are unusually long, as the text covers
much material in an introductory way, or as review of material that you are expected
to know well already. I will certainly not have had time to discuss all of these issues in
class. The beginning of the lecture will be spent in discussing details of the course.

TEXTBOOK DIAGNOSTIC TESTS


36
This is why I usually avoid in the textbook problems that appear to be drawing inferences from
graphs.
Notes for Lecture Section 002, MATH 140 2008 09 2005

5.1.2 A. Diagnostic Test: Algebra


5.1.3 B. Diagnostic Test: Analytic Geometry
5.1.4 C. Diagnostic Test: Functions
5.1.5 D. Diagnostic Test: Trigonometry
Four tests are provided to help you decide whether you are ready for this course. Some
of the topics tested will be reviewed very briefly in the first two weeks of the term, and
are discussed in the Appendices to the textbook. Test yourself honestly to determine
whether you need to do some major repairs in your background at the beginning of
the term. If your performance is weak, review the Appendices to the textbook; if your
performance is truly dismal, you may wish to consider taking MATH 112 2008 09, a
course we offer only in the fall term.

TEXTBOOK CHAPTER 0. A PREVIEW OF CALCULUS.


This chapter is motivational. Most of the issues raised will be considered in depth later
in MATH 140 or MATH 141.

TEXTBOOK APPENDICES.

5.1.6 Appendix A. Numbers, Inequalities, and absolute values.


This appendix reviews some topics of Precalculus, to which most students have already
been exposed. If you haven’t seen much of this material before, you probably aren’t
ready for a calculus course.

The Number Systems The appendix begins with a brief discussion of the number
systems in which we will be working. A rigorous knowledge of this material is not
expected, but you should know the terminology, as it will be used constantly in the
Notes for Lecture Section 002, MATH 140 2008 09 2006

course.37 The hierarchy of number systems we will meet in this course, with the symbols
I will usually use for the sets are:
N the positive integers, called the natural numbers: 1, 2, 3, . . .
Z the integers, positive and non-positive: . . . , −2, −1, 0, 1, 2, . . .
Q the fractions, or rational numbers, positive, negative, or zero:
all fractions of the form m n
, where m and n are integers, and
n 6= 0
R the real numbers
We can represent all of these sets as points on the real line. Each of the sets I have listed
is properly contained
√ in the next. For example, there are real numbers which are not
rational, e.g., 2 — this fact was known to the ancient Greeks, and is a theorem38 of
Euclid.
The hierarchy I have given can be refined — algebraists study number systems that
can be interposed between the systems which I have listed. This topic is beyond the
present course.

Set Theory Notation You are expected to have an intuitive understanding of the
following concepts, and to be familiar with the notation. Here again, you aren’t expected
to bring a rigorous knowledge of the subject — in fact, it is surprisingly difficult to build
this subject on a firm foundation. You should be familiar with the following concepts:
• The concept of a set, made up of elements, which can also be called members of
the set. I will usually denote a set by a capital letter, e.g., S, and its members as
lower case letters, e.g., x, and write x ∈ S to indicate that element x is a member
of — or belongs to — set S.
• The union of 2 sets S, T , written S ∪ T , whose members are all x which are either
in S or in T or in both S and T .
• The intersection of 2 sets S, T , written S ∩ T , whose members are all x which are
in both S and T .
• The empty or null set, written ∅ or ∅. Don’t confuse this symbol with the Greek
letter phi , written as Φ in upper case and φ or ϕ in lower case.
• Sets written by giving a list of their members, placed between braces (curly brack-
ets), as {. . . , −4, −2, 0, 2, 4, . . .}, to represent the even integers.
37
Students at McGill interested in seeing these concepts developed carefully would usually meet them
first in MATH 242, given to Majors and Honours students in Mathematics every fall. This course
requires MATH 140 and MATH 141 as prerequisites.
38
You are not expected to know how to prove this theorem.
Notes for Lecture Section 002, MATH 140 2008 09 2007

• Sets written in “set-builder” notation: e.g., {x|x is an even integer} or

{x|There exists n ∈ N such that x = 2n}.

Some authors use the symbol : instead of |, as in {x : x is an even integer.}

Intervals On the real line, R, we consider for real numbers a ≤ b four types of finite
intervals with lower end-point a and upper end-point b:

Notation Set Description Verbal Description


(a, b) {x|a < x < b} open interval
[a, b) {x|a ≤ x < b} closed on left, open on right .
(a, b] {x|a < x ≤ b} open on left, closed on right
[a, b] {x|a ≤ x ≤ b} closed interval

In addition, we consider the following semi-infinite and infinite intervals:

Notation Set Description Verbal Description


(a, ∞) {x|a < x} ray open on left
[a, ∞) {x|a ≤ x} ray closed on left
.
(−∞, b) {x|x < b} ray open on right
(−∞, b] {x|x ≤ b} ray closed on right
(−∞, ∞) R or {x|x ∈ R} real line

Are −∞ and ∞ numbers? When we first meet the symbols +∞, −∞, they are used
to describe certain unbounded situations, but are not intended to be treated as numbers.
We will be devising methods for working with these situations in such a way that, should
you choose to casually treat these situations as though ±∞ are numbers39 , you will not
obtain incorrect results, provided you follow certain rules and restrictions that we will
be developing. But, whether or not you choose to work with them as though they were
numbers, you must understand that, when we use the word(s) (real) number we will
never intend that ±∞ be included.

Inequalities An inequality is a statement of one of the forms

a < b, a ≤ b, a > b, a ≥ b,

where a and b are real numbers. You should be familiar with standard methods for
working with inequalities — methods which are similar to, but more complicated than
39
in the extended real number system
Notes for Lecture Section 002, MATH 140 2008 09 2008

methods for working with of equations. Our interest here is only in simple properties,
like the following (for appropriate real numbers a, b, c):
a<b ⇒ b>a (2)
a≤b ⇒ b≥a (3)
a<b ⇒ a+c<b+c (4)
a≤b ⇒ a+c≤b+c (5)
¾
a<b
⇒ a+c<b+d (6)
c<d
¾
a<b
⇒ ac < bc (7)
c>0
¾
a<b
⇒ ac > bd (8)
c<0
1 1
0<a<b ⇒ 0< < (9)
b a
where I have used the symbol ⇒ to mean implies, connecting a hypothesis to a logical
conclusion.
When we speak of solving an inequality whose statement involves a variable x, we
mean to describe, in as simple a way as possible, the set of real numbers x which satisfy
the inequality.

Absolute Value and Square Root The absolute value of a real number x can be
defined by ½
x if x ≥ 0
|x| = .
−x if x < 0
For two real numbers, x, y, |x − y| can be interpreted as the distance between x, y on
the line R; thus |x| may be interpreted as the distance from x to the origin.
Some of the simple properties of this function are
−|x| ≤ x ≤ |x| (10)
|xy| = |x||y| (11)
|x| = 0 if and only if x = 0 (12)
¯ ¯
¯1¯
¯ ¯= 1 (x =6 0) (13)
¯ x ¯ |x|
¯ ¯
¯ x ¯ |x|
¯ ¯= 40
¯ y ¯ |y| (y 6= 0) (14)

40
[1, Exercise 66, p. A10]
Notes for Lecture Section 002, MATH 140 2008 09 2009

As a consequence of the geometric interpretation mentioned earlier, we have the following


special case of the “Triangle Inequality”:
|x + y| ≤ |x| + |y| , (15)
where the triangle is is a “degenerate” triangle with vertices x, 0, −y.

A Exercises
t t t
0 1 3

Figure 1: Sketch of Solution to [1, Exercise 36, p. A9]: thickened portions of R

[1, Exercise 36, p. A9] Solve the inequality x3 + 3x < 4x2 in terms of intervals, and
illustrate the solution set on the real number line.
Solution:
x3 + 3x < 4x2 ⇔ x3 − 4x2 + 3x < 0
¡ ¢
⇔ x x2 − 4x + 3 < 0
⇔ x(x − 1)(x − 3) < 0
The solution set will consist of all real numbers x for which the product of the
three factors x − 0, x − 1, x − 3 is negative. For each of these expressions the sign
will change only at a specific point on the real line. Thus, if we divide R into 4
parts by the points 0, 1, 3, the behavior of each of these factors will be the same
throughout each of the 4 intervals determined by the points. We can tabulate this
situation:
Interval x x−1 x−3
x<0 <0 <0 <0
0<x<1 >0 <0 <0
1<x<3 >0 >0 <0
3<x >0 >0 >0
where we exclude the points 0, 1, 3 since this “strict” inequality is clearly not satis-
fied when both sides are 0. Now we need only determine the signs of the products,
which we can show on an expanded table:
Interval x x − 1 x − 3 x(x − 1)(x − 3)
x<0 <0 <0 <0 <0
0<x<1 >0 <0 <0 >0 ,
1<x<3 >0 >0 <0 <0
3<x >0 >0 >0 >0
Notes for Lecture Section 002, MATH 140 2008 09 2010

showing that the product is negative when precisely an odd number of the 3 factors
are negative. The given inequality is satisfied precisely for points in the following
union of intervals: (−∞, 0) ∪ (1, 3).

5.1.7 Appendix B. Coordinate Geometry and Lines


Read this appendix, to be sure that you are familiar with all topics mentioned. This
material will not be reviewed in the lectures.

5.1.8 Appendix C. Graphs of Second-Degree Equations


Circles Read this appendix, to be sure that you are familiar with all topics mentioned.
This material will not be reviewed in the lectures.

Parabolas

Ellipses

Hyperbolas You are not expected to be familiar with the contents of these three
subsections, although it would occasionally be helpful.

5.1.9 Appendix D. Trigonometry. TO BE DISCUSSED AFTER §1.2

CHAPTER 1. FUNCTIONS AND MODELS


Many of the concepts in this chapter should be review for most students, since they
should have appeared in a good pre-calculus course; some of the terminology and notation
may be new. Some of the material in [1, §1.6] may be new.
In discussing [1, §1.2] I will also review material from [1, Appendix D]; if any of this
material is new to you, you should take steps to reinforce your background in trigonom-
etry before a deficiency impedes your progress. A very small number of students may
find themselves hopelessly deficient in both high school trigonometry and algebra, and
might seriously consider transferring from MATH 140 2008 09 to MATH 112 2008 09,
a course given only in the fall term; or delaying MATH 140 until the second term, and
taking other steps to rebuild precalculus foundations.
Notes for Lecture Section 002, MATH 140 2008 09 2011

5.1.10 §1.1 Four Ways to Represent a Function.


The purpose of the section is to prepare you for subsequent discussions in which functions
will be specified in a variety of ways. Time at the lecture did not permit discussion of
most of the details in the following notes. You should read the textbook, and may
find the notes below useful; you will find some parts of the discussion of even and odd
functions below which are indicated as being beyond the course.

Functions To specify a function we need two sets. The first, called the domain of
the function, is the set on which the function “acts”. The second is the set where the
function’s values are located. Some authors call this “target” the codomain.
The textbook defines [1, p. 11] the range of a function f as “the set of all possible
values of f (x) as x varies throughout the domain.” Many mathematicians now avoid the
word range 41 and prefer to call the set of values the image of f .
When we write f (x), we normally mean the point to which the point x in the domain
is mapped by the function f ; sometimes we may, carelessly, talk about “the function
f (x)”, but the intention is to reserve this notation for the general point of the “range”
(image). The “variable” x is sometimes called the independent variable; if we write
y = f (x), older terminology would call y the dependent variable.
One part of the definition of a function is non-negotiable: we always require that,
for every point x of the domain, there be exactly one point to which it is mapped: this
means never more than one image point; and never fewer than one, i.e., every point
in the domain must be mapped somewhere: there must never be any doubt about the
image of every single point of the domain.

Graph of a function The graph of f : A → B is the set of points in42 R2 whose first
coordinate x ranges over all points in the domain, where the second coordinate is the
corresponding value f (x). This set may be interpreted “graphically”, as points in the
plane, and that representation may be “sketched”. The “sketch” obtained may be useful
in solving problems, but mathematicians never base a serious proof on a sketch; if we
make a sketch, it is only as a visual aid. You are expected to follow this practice in your
written solutions.

Theorem 5.1 (Vertical Line Test [1, p. 16]) A set S of points in R2 is the graph of
a function if and only if no vertical line passes through two distinct points in S.
41
One reason for avoiding the use of this word is that some authors have used the same term for the
target, which could include points which are not values of the function.
42
R is the set of real numbers or, when interpreted geometrically, “the real line”; R2 is the set of
“ordered pairs” of real numbers, or, when interpreted geometrically, “the real plane” or “the Cartesian
plane”.
Notes for Lecture Section 002, MATH 140 2008 09 2012

Proof: The condition is necessary (the “only if” part of the theorem) because the presence
in S of two points with the same x-coordinate (the abscissa) would entail that that value
of x was mapped on to43 two distinct points. Otherwise we can interpret S as the graph
of a function whose domain is the set of all numbers that appear as an x-coordinate for
some point in S: every point in this domain is mapped on to exactly one point, so the
function is “well defined”. For each point x of this domain there is now exactly one point
y such that (x, y) ∈ S, and so, if we define a function f by x 7→ y, then S is the graph
of f .
Conversely, suppose a set of points meets every vertical line at most once. Then, if
the graph meets the line x = a in a point (a, b), the points are the graph of a function
that maps a to b.44 ¤
When a function f has domain A, modern mathematical terminology refers to the
function45 f : A → B, where B is the “target” or codomain. The choice of a codomain
is sometimes unclear in a particular context, since there is often no harm in enlarging
the set; when we say that B is the codomain we do not insist that every point in B
be realized as the image under the function of a point in the domain A; all we ask is
that the image or range should be contained in the codomain. There are good algebraic
reasons for being fussy about the codomain, but students are not likely to see them in
MATH 140 or MATH 141, except possibly in connection with inverse functions (to be
introduced in [1, §1.6]).
Suppose that f : A → B is a given function. If we have a formula for computing
f (x) when x is given, a useful notation is to denote the function √ by x 7→ f (x); thus,
for example, the square root function could be denoted by x 7→ x. (Note the peculiar
symbol — a horizontal arrow with a short vertical foot.)

Representation of Functions I wish to make it clear that no one is going to expect


you to answer silly questions like “How many ways are there to represent a function?”
The count of 4 in the textbook is simplistic, in that two of the four ways proposed are
not useful in general: a table of values can be used to define a function only if its domain
is finite, and the value of the function is prescribed for every point in the domain; it is
normally not acceptable to define a function by a graph, unless the nature of the graph
can be described without any ambiguity. There could also be other ways than the given
4 in which the values of a function could be specified. Aside from that, some of the ways
described in this section are problematic.
43
Some English dictionaries do not accept onto as a single word.
44
A symbol currently used by some mathematicians to indicate the end of a proof.
45
The arrow notation is not generally used in your textbook, although you can see it at the bottom
of [1, Figure 3, p. 12 margin], and is introduced here so that, at least, you can see the kind of notation
that is used by mathematicians today. Sometimes we may modify the notation f : X → Y , writing
f
X →Y.
Notes for Lecture Section 002, MATH 140 2008 09 2013

When a function f is represented by its graph — i.e., by the set of points

{(x, f (x))|x in the domain of f } ,

this graph will always have the property that there is at most one point where it intersects
a vertical line. That property is called by the textbook the Vertical Line Test. There
are no exceptions to this rule: if a curve crosses a vertical line line more than once, then
the curve is not the graph of a function; thus, for example, a circle is never the graph of
a function46 .
As mentioned above, a complete definition of a function must specify

• the domain;

• the set where the function takes its values — we could call this the “target” or the
codomain);

• the rule that assigns to each point in the domain precisely one point in the target.

If there is even one point in the domain where the action of the function has not been
specified precisely, the function is not “well defined”! The textbook proposes four ways
of representing functions, some of which have restrictions in their applicability if the
representation is to be interpreted as defining the function. Of these “numerically (by
a table of values)” is possible only if the domain of the function is finite. Similarly, the
method described as “visually (by a graph)” is not adequate if by graph one thinks of
a sketch — a curve drawn on a piece of paper; if, however, the graph is specified in an
unambiguous way, as, for example “the line through points (1, −2) and (4, 2)”, or “the
upper half-circle with centre (1, −2) and radius 5”, then the curve is precisely specified;
the second example — the circle — would still not define a function, since it violates the
“Vertical Line Test”. My objection is to the word visually, not to by a graph.
When a function is described by an algebraic formula, there is often a “natural”
domain — the largest set for which the formula “makes sense”. So, for example, one
might be inclined to say that “the function x1 has domain all real numbers except 0”.
But, if the inventor wishes to specify a domain that is smaller than this set, and use
the same formula, that is completely “legal”. There are often good reasons to restrict a
function to a smaller domain than that for which a defining formula might be meaningful.
So a formula is not enough to thoroughly specify a function: it is necessary to specify
its domain precisely: if that is not done, the reader will usually take the domain to be
the “natural” domain.
46
except for the circle with radius 0, whose graph consists of just one point
Notes for Lecture Section 002, MATH 140 2008 09 2014

Piecewise Defined Functions Mathematicians use the term piecewise to describe a


definition of a function where the domain is dissected into (subsets of) mutually disjoint
sets, within each of which it may be possible to define the function in some unified way.
(The reason for taking the sets to be mutually disjoint is to avoid the possibility that
two parts of the definition would be in conflict.) The graph appears to have been created
by cutting and pasting pieces of various graphs. When we cut and paste intervals that
are disjoint, some of those intervals will contain their end-points, and others will not
(since we don’t want the intervals to overlap). This can lead to definitions where there
are abrupt changes in the behavior of a function as we pass through specific points of
the domain. We will return to this issue when we discuss continuity in [1, §2.5].

Example 5.2 [1, Review Exercise 18, p. 74] Find an expression for the function whose
graph consists of the line segment from the point (−2, 2) to the point (−1, 0), together
with the top half of the circle with centre the origin and radius 1.”
Solution: (Here is an example where a function can be defined through a precise descrip-
tion of its graph.) The first part of the graph is an oblique line segment that crosses
once every line x = a with −2 ≤ a ≤ −1; the second is a semicircle crossing once every
line x = a where −1 ≤ x ≤ +1. Since no vertical line is crossed more than once, this
is, indeed, the graph of a function. The slope of the line from which the segment is se-
0−2
lected is −1−(−2) = −2; since the line passes through the point (−1, 0), its (point-slope)
equation is y − 0 = (−2)(x − (−1)), or y = −2x − 2. If we denote the function by f , we
have shown that f (x) = −2x − 2 for −2 ≤ x ≤ −1.
The equation of the circle centred at the origin, having radius 1, is x2 +y√2 = 1; solving
for y, and choosing the solution in the 2
√ upper half-plane, we obtain y = + 1 − x ; thus,
for −1 ≤ x ≤ 1, we have f (x) = 1 − x2 . The two intervals whose union forms the
domain overlap at the point x = −1; fortunately the two descriptions for the function
are consistent at that point. One formula for the function f is, therefore
½
−2x − 2 if −2 ≤ x ≤ −1
f (x) = √ .
1 − x2 if −1 < x ≤ 1

Another equally correct formula for the same function would be


½
−2x − 2 if −2 ≤ x < −1
f (x) = √ .
1 − x2 if −1 ≤ x ≤ 1

Symmetry: Even and Odd Functions. This topic will not be discussed in depth
at this time. Symmetry refers to properties of the graph of a function under which the
graph could be “lifted up” and “moved” (without any distortions or changes of distance),
so that the curve obtained is identical with the original graph. For example, the graph
Notes for Lecture Section 002, MATH 140 2008 09 2015

of the function sin is not changed when the whole plane is shifted to the right a multiple
of 2π units.
At this time the textbook considers only 2 special types of such “motions”. A function
f is even if it has the property that

f (−x) = f (x) (16)

for every point in the domain. So, if x is in the domain, condition (16) refers to f (−x);
if it is permitted to mention f (−x), then −x must also be in the domain of f . Not only
must an even function have the property that its domain be symmetric around the point
0; but, moreover, points which are symmetrically located with respect to 0 must have
the same function value. To prove that a function f is not even, it suffices to show that
there is a real number x such that either
1. f is defined at one of x, −x, but not at the other; or
2. f is defined at both x and −x, but the values of the function are different at the
two points.
Just one such pair is enough to permit the inference that f is not even, since evenness
requires that no such pair exist.
A function f is odd if it has the property that

f (−x) = −f (x) (17)

Here, as for evenness, a function can be odd only if its domain is symmetric around 0.
The equation must hold for all pairs of points in the domain; if there is one real number
x where either the equation fails or the function is defined at only one of the points x,
−x, the function is not odd. The terminology is easily remembered by thinking about
the properties of even and odd powers of x.
Here is a theorem about even and odd functions; you can skip it at this time:
Theorem 5.3 Let f be a function whose domain is R. Then f may be ex-
pressed as the sum of an even function fe and an odd function fo , both with domain
R.
Proof: (This proof is called constructive; not only will we prove the existence of
the functions postulated in the enunciation of the theorem, but we will show how
to construct the functions.)
We can define, for all x ∈ R,
f (x) + f (−x)
fe (x) = (18)
2
f (x) − f (−x)
fo (x) = (19)
2
Notes for Lecture Section 002, MATH 140 2008 09 2016

1. For all x ∈ R,
1 1
f (x) = (f (x) + f (−x)) + (f (x) − f (−x))
2 2
= fe (x) + fo (x) (20)
= (fe + fo )(x)

according to the definition of the sum fe + fo , [1, p. 41]. Since functions f


and fe + fo have the same domain — R — and have the same action on every
point x in the domain, as shown in (20), the functions are said to be equal.47
2. The function fe is defined in (18). The proof that this function is even is left
to the student.
3. The function fo is defined in (19). The proof that this function is odd is left
to the student.

Exercise 5.1 1. Show that the only function that is both even and odd is the constant
function 0.
2. Let r be a fixed real number. Is the constant function f given by f (x) = r even?
Is it odd?
3. Give examples to show that there exist functions that are neither even nor odd.
4. There is a function that is both even and odd. What is it?

Some examples of 2nd-degree equations whose graphs are symmetric about one of
the coordinate axes can be found in [1, Appendix C, pp. A18-A21].

Increasing and Decreasing Functions Here are the textbook’s definitions:


Definition 5.1 [1, p. 20]
1. A function f is called increasing on an interval I if

f (x1 ) < f (x2 ) whenever x1 < x2 in I .

2. A function f is called decreasing on an interval I if

f (x1 ) > f (x2 ) whenever x1 < x2 in I .


47
This is the definition of what we mean when we say two functions are equal: they have the same
domain, and they act in the same way on every point of that domain. The student who thinks there
is no need for a definition here, must remember that, hitherto, the = sign has been used only between
numbers. We are now applying it to a different type of object; whenever we do that we must clarify
precisely what we mean by this symbol and language in this new context.
Notes for Lecture Section 002, MATH 140 2008 09 2017

There are variations of the terms increasing and decreasing whose meanings are obvious;
you could see these variations when you use other textbooks, or look at old examinations.
Among these are

• non-decreasing: f is non-decreasing on interval I, if, whenever x1 < x2 on I,


f (x1 ) ≤ f (x2 ).

• non-increasing, defined analogously to the preceding

• strictly decreasing, which is a term used by some authors for what Stewart calls
decreasing

• strictly increasing, which is a term used by some authors for what Stewart calls
increasing.

1.1 Exercises Remember, in problems like [1, Exercises 1.1.27-1.1.31, p. 22], when
the textbook asks for the domain of the function, it always intends the largest possible
or maximal or natural domain for which the formula could be meaningful; the range
or image in such a situation will be that corresponding to the largest possible domain.
However, there will be situations where we will wish to prescribe for a function a domain
which does not contain all points in the largest possible domain. This will happen, in
particular, in optimization problems, where the domain may be restricted by certain
conditions specific to the problem under discussion. It will also happen when we have
to define the inverse trigonometric functions: in that case we shall have to restrict the
sizes of the domains of the functions in order that they will be “invertible”.
√ √
[1, Exercise 30, p. 22] Find the domain of the function g(u) = u + 4 − u.
Solution: as mentioned above, the textbook intends that we find the largest possible
domain for this function — the set of all u for which the definition √
makes sense and
for which the function is uniquely defined. The first summand, u makes sense
for all u ≥ 0; the second summand makes sense for 4 − u ≥ 0, i.e., for u ≤ 4.
For the sum to make sense we need that both of these conditions be satisfied
simultaneously; thus 0 ≤ u ≤ 4 is the (largest possible) domain.

[1, Exercise 70, p. 23] Determine whether f is even, odd, or neither: f (x) = 1+3x3 −
x5 .
Solution: This is not a problem to be attempted without some prior experimenta-
tion. After some experimentation you should have convinced yourself that

• odd integer powers of x define an odd function;


• even integer powers of x define an even function;
Notes for Lecture Section 002, MATH 140 2008 09 2018

• the sum of two odd functions is odd;


• the sum of two even functions is even;
• polynomials in only odd powers of x define an odd function;
• polynomials in only even powers of x define an even function

From this experimentation one might be inclined to be suspicious about a sum


which involves both odd and even powers.48 That suspicion can be borne out by
judicious choices of values for x. For example, we find that f (1) = 1 + 3 − 1 = 3;
f (−1) = 1 − 3 + 1 = −1. Thus x = 1 provides an example where f (−x) 6= f (x),
so f is not even; and f (−x) 6= −f (x), so f is not odd either.
(Compare this function with the function of [1, Exercise 69, p. 23], which is a sum
of multiples of even powers only: that function is, indeed, even.)

48
The summand 1 is x0 , and 0 is even.
Notes for Lecture Section 002, MATH 140 2008 09 2019

5.2 Supplementary Notes for the Lecture of September 8th,


2008
Release Date: Monday, September 8th, 2008
subject to correction

Preparing for MATH 140. One theme I will be returning to repeatedly in this brief
introduction is the need to repair deficiencies so that you are ready to work through
a calculus textbook for students in Science and Engineering. I am not concerned with
whose fault it is that these deficiencies may be present: our job is to quickly repair the
foundations so that we can start building. Except for the introduction of new terminology
and new notation, §1.1 is a review of material you should have seen in a precalculus
course; if you haven’t seen anything about functions before, you aren’t ready for a
calculus course at McGill, and should register in MATH 112 2007 09. The terminology
and notation will be used in the coming sections, and you should not have difficulty
learning them. Familiarity with the material on symmetry — in particular, the discussion
of even and odd functions in my notes — will not be assumed at present.

Tutorial contents. Because we have very little tutorial time available for review, this
week’s tutorials will be discussing (mainly review) material that we may not reach in
this lecture section until Wednesday; this is an exceptional situation — I will usually try
to have topics appear first in the lecture.

5.2.1 §1.2 Mathematical Models: A Catalog of Essential Functions.


Applications of the Calculus Many of you are taking this course because it is re-
quired in your programme — often because there are applications of the calculus to your
specific discipline. Since we have to instruct all of you together in this one course, it’s
not practical to discuss those applications here, so most of what you see in the course
will be sufficiently general that it is relevant to all of your areas. While I have to omit
discussion of specific applications in the lectures, that doesn’t mean that you have to skip
that material when you are reading the textbook: I urge you to read any applications
to your area, even if the syllabus states that the pages involved are not contained in the
formal syllabus. While the textbook applications will be specific, mine will usually be
general enough to apply to any area of interest.

What should I retain from this section of the textbook? While the title of the
section refers to models, i.e., to specific types of applications of the calculus to concrete
situations, this section also serves to remind students of a multitude of definitions and
facts from precalculus and earlier mathematics courses which we will be assuming in the
Notes for Lecture Section 002, MATH 140 2008 09 2020

subsequent chapters. I will discuss much of the material superficially; the exception is
the discussion of trigonometry, on which I will be amplifying slightly in the next lectures.

Linear Models The author defines a function f to be linear 49 if there exist real
constants m and b such that, for every x, f (x) = mx + b.
50
Exercise 5.2 Is the squaring function x 7→ x2 linear?
Solution: NO! Here is a proof by reductio ad absurdum, i.e., by showing that, if the
statement were true, it would imply an absurd contradiction. Suppose that there existed
real numbers m and b such that
x2 = mx + b for all x . (21)
We can complete the proof in various ways.
1. One way to show that constraint (21) cannot be satisfied by all x is to choose
several values of x which lead to a contradiction. By specializing values of x
we can determine information about m and b, eventually obtaining contradictory
information. If we take x = 0, we find that 0 = 0 + b, so b = 0. If we take x = 1,
we find that 1 = m + b = m + 0 = m. Then, if we take x = 2, we find that
4 = m · 2 + 0 = 1(2) + 0 = 2, which is a contradiction.51
2. Another approach would be to determine precisely those values of x for which the
given equation could be satisfied. The equation is quadratic, so, by completing the
square, we obtain
³ m ´2 m2
x− =b+ ,
2 4
2
implying that there couldn’t be any solutions at all unless b + m4 ≥ 0; and that,
even then, the only solutions are
r
m m2
x= ± b+ .
2 4
Thus there are at most 2 solutions to (21), which contradicts the claim that the
equation should hold for all real numbers in R.
49
In some areas of mathematics, the word linear would be reserved for the case where b = 0, i.e.,
where the graph of f is a line passing through the origin. In such cases the word that is used to describe
functions of the generality we want here is affine. In this course I will usually follow the textbook’s use
of language, so I am not likely to use the word affine again.
50
I probably won’t have time to discuss this example in the lecture.
51
This is an example of a proof by contradiction or by reductio ad absurdum: one assumes the falsity of
a statement, shows that the falsity would imply some nonsense, and then concludes that the statement
must have been true.
Notes for Lecture Section 002, MATH 140 2008 09 2021

When we take the narrow definition of linear (i.e. the definition where b = 0), we find
that it is equivalent to the property

f (x1 + x2 ) = f (x1 ) + f (x2 ) .

Exercise 5.3 Some students in MATH 140 appear to believe that the square root func-
tion is linear; see if you can prove that the square root also fails to be linear.
[1, Examples 1, 2, 3, pp. 26-27] on fitting a line to empirical data, and on interpolation
and extrapolation may be omitted: the topic they refer to is in [2, §14.7, Exercise 55, p.
932], and is beyond both MATH 140 and MATH 141.

Polynomials A polynomial is a function of the form

P (x) = an xn + an−1 xn−1 + · · · + a2 x2 + a1 x1 + a0 x0

where n is a non-negative integer, and a0 , a1 , . . . , an are real numbers. We usually write


just x for x1 , and a0 for a0 x0 . If a0 = a1 = · · · = an = 0 the polynomial is the zero
polynomial. Otherwise there will be at least one non-zero coefficient among a0 , a1 , . . . , an ,
and we can assume that n is such that an 6= 0; we call n the degree of the (non-zero)
polynomial.52 When a0 is the only non-zero coefficient, the polynomial is constant,
having degree 0, and may be identified with the real number a0 : thus we may think
of the real numbers as being contained in the set of polynomials. Of course you are
expected to know how to solve quadratic equations, of the form

a2 x2 + a1 x1 + a0 = 0 .

While you may be accustomed to doing so by using the “quadratic formula”, you are
encouraged to also become familiar with the (equivalent) method of completion of the
square seen above, and which will be discussed in the lectures. You are not expected to
know how to solve higher degree equations, except where the polynomial f has an obvious
factorization into lower degree polynomials; or, where the polynomial f has an obvious
root 53 or zero a, i.e., a number a such that f (a) = 0 — in that case f (x) = (x − a) · g(x),
where g is a polynomial of degree 1 lower than that of f .54 There will be specific facts
that you should know about polynomials; these will be discussed as we meet examples
during the term.
52
Zero is also a polynomial, but there the term degree is difficult to define, if we wish it to have
expected properties: some mathematicians refuse to use the term degree for the polynomial 0; others
define the degree to be −∞.
53
This use of the word root is not the same as that in the next paragraph: mathematicians sometimes
use precisely the same English word with totally different intended meanings.
54
This is an application of the Factor Theorem, or of the Remainder Theorem, which you should have
seen in high school.
Notes for Lecture Section 002, MATH 140 2008 09 2022

Power Functions If a is any real number, the textbook calls a function of the form
x 7→ xa , a power function.55 We will be defining what we mean by fractional, negative,
and, more generally, irrational powers.

Rational Functions A rational function maps the independent variable on to the


ratio of two polynomials, where the denominator is not the zero polynomial. Since
the denominator could be a non-zero constant polynomial, e.g. 1, every polynomial is a
1 x2 − 2x
rational function. Other “degenerate” examples are , : note that such functions
x x−2
1
need not be defined for all real numbers x; for example is not defined when x = 0,
x
2
x − 2x
and is undefined when x = 2 — in both cases the formula cannot be evaluated
x−2
because of a 0 denominator.

Algebraic Functions The textbook defines an algebraic function as one that can be
“constructed using algebraic operations (such as addition, subtraction, multiplication,
division, and taking roots) starting with polynomials”. The intention of the word root
here is that one does not simply confine oneself to nth roots, where n is a positive integer:
one must permit any solution of any polynomial equation, even if that solution cannot
be expressed in terms of nth roots.56

Trigonometric Functions The prerequisites for this course require a good basic
knowledge of trigonometry. While trigonometry originated in the study of metric prop-
erties of triangles, our main interest will be in properties and relations between trigono-
metric functions — functions defined not only for angles in the range that can appear
in a triangle, but for over the whole real line — possibly excluding specific values where
the functions are not defined. I will be reviewing some of this material on trigonometry
in the next few lectures.

Exponential Functions We will meet exponential functions next in [1, §1.5]; both
here and there the discussion of these functions is not rigorous. The intention is that
you should thoroughly understand what is meant by an integer power of a positive real
number: if n > 0, an is the product of n copies of a; if n < 0, an is the product of n
55
Power function is not a “standard” term.
56
This is difficult for students to grasp, since you know that the solutions of a quadratic equation
can be expressed in terms of square roots. It is also possible to express solutions of 3rd and 4th degree
equations in terms of (higher order) roots, although you probably haven’t studied these in high school;
but it is not necessarily true for polynomial equations of degree greater than 4. BUT DON’T PANIC
— YOU DON’T NEED TO KNOW ABOUT THIS IN MATH 140!
Notes for Lecture Section 002, MATH 140 2008 09 2023

copies of a1 ; if a = 0, an is defined to be 1. You should then generalize this understanding


r
by taking roots: for integers r and s, where s > 0, a s is the (real) sth root of ar — and,
if s is an even integer, this sth root is chosen to be the positive root rather than the
negative one. Finally, if we wished to make this definition rigorous, we would need to be
able to think of every real number as being a limit of a sequence of rational numbers.

Logarithmic Functions You are expected to be familiar with simple properties of


the logarithm — to any base — from your precalculus prerequisite. We will be reviewing
and proving some of these properties briefly in connection with [1, §1.6].

Transcendental Functions You are not expected to be able to determine whether


a function is transcendental . We will not be using this terminology — even though it
appears in the title of the textbook.

5.2.2 Appendix D. Trigonometry


YOU MAY HAVE WORK TO DO, BUT DON’T PANIC! Some students are
going to see from this appendix that they are not adequately prepared in this subject for
a Science/Engineering version of a calculus course. You’re going to have to work a little
harder than you may have planned, and it has to start now — not later in the term.
Whether you knew before about this deficiency, or are just discovering it now, it needs
to be remedied before it impedes your progress in the course; it’s not important at this
time to find someone to blame for the fact that you weren’t taught — or haven’t learned
— enough trigonometry. If this is the only gap in your preparation, there is no reason
to panic — it can be fixed! Your textbook and its Student Solutions Manual alone may
be enough, or you may need to use other materials from the library; if you were good
enough to get into McGill, you should be able to do this work on your own.

Angles

What is an angle? Our definitions of trigonometric functions may involve an


angle. Intuitively an angle is traced out by moving a ray — one of the two parts into
which a point divides an (infinite) line. Think of, for example, the ray which is the part
of the x-axis to the right of the origin; we can call this the initial ray of the angle. Turn
this ray in the counterclockwise direction (defined as the positive direction) in the plane,
pivoting on the origin. The final resting position could be called the terminal ray defining
the angle. Any ray may be used in this construction; indeed, we can also define an angle
in terms of a finite line segment pivoting on one of its ends, with the understanding that
the angle is that common property that is not changed when the segments are extended
in the direction away from the pivot point.
Notes for Lecture Section 002, MATH 140 2008 09 2024

Measuring angles There are several methods of angular “measure” in use.

• We could express an angle in terms of the fraction it makes of a full revolution, or


of the angle formed by turning until the first time that one ray is perpendicular to
the other (a right angle).

• The most commonly used system defines a full revolution to be 360 degrees 57 and
expresses all angles in terms of degrees (which may be further subdivided into
minutes and seconds).

• At the time of the French revolution a decimal method was proposed which divides
a right angle into 100 equal parts.

• In the calculus we find it most convenient to measure angles in radians (to be


defined below). One reason for this preference will be seen when we determine the
derivatives of the trigonometric functions, in [1, §3.3].58

Radians A radian is the angle at the centre of a circle subtended by an arc whose
length is the radius of the circle. (This definition is somewhat premature, as we have not
yet defined what we mean by the length of a curve.) The number of radians in a straight
angle59 is denoted by the symbol π — the lower case of a letter of the Greek alphabet.
Since a straight angle can also be defined to be 180 degrees we have the relationships
that

π radians = 180 degrees (22)


π radians = 2 right angles (23)
2π radians = 1 revolution (24)

which permits us to convert from one system of measurement to another. The number
π is not an integer, nor is it a rational number; nor is it an algebraic number (a number
which is a solution of a polynomial equation with integer coefficients). In Calculus 3
students will see how to approximate π:

π ≈ 3.1415926535897932384626433832795028841971693993751...
57
written 360◦
58
Notwithstanding the preceding comment, we do need in the real world to be able to discuss the
trigonometric functions of angles that happen to have been measured in degrees. When, for example, we
write sin 31◦ , we are, in effect, using a slightly different function from the function which we routinely
denote by sin in this course; when we write sin 31, we will mean the sine of an angle of 31 radians; but
31π
sin 31◦ will mean sin .
59
180
half a revolution, or 2 right angles
Notes for Lecture Section 002, MATH 140 2008 09 2025

to any desired number of decimal places. The approximation is not based on measuring
a concrete angle, as we don’t have instruments capable of that: it is derived using purely
mathematical methods. It is not correct to write pi = 3.14, nor to equate π to a
decimal number with any finite number of decimal digits; that’s why, in this
course, it’s better just to write π.60

Length of an arc of a circle The rationale for radian measure was that the angle,
measured in radians, would be equal — numerically — to the length of the arc it subtends
in a circle of radius 1. An angle of measure θ in a circle of radius r would subtend an
θ
arc equal to 2π times the circumference of the circle, i.e.,

θ
arc length subtended by angle θ = × 2πr = θr . (25)

The Four Quadrants Angles in the intervals 0 < θ < π2 , π2 < θ < π, π < θ < 3π 2
,

2
< θ < 2π, are said to lie respectively in the First, Second , Third , and Fourth
Quadrants. More generally, any angle θ is said to lie in the quadrant containing the
angle ¹ º
θ
θ− 2π .

Trigonometric Functions While the usual introduction of students to these functions


is through the angles of a right-angled triangle, I will skip that for now and define the
functions for the full domain in which we are interested. The definitions will involve
ratios of lengths of sides of a right-angled triangle. But, departing from the traditional
approach, these lengths will be signed — either positive or negative. For these definitions
consider the initial ray to be the positive x-axis in the plane, and rotate counterclockwise
through an angle θ until the ray — pivoting on the origin — rests in some terminal
position. The values of the functions will depend only on the this final position — not
on the way in which the position was reached; thus, if we happen to have rotated through
a number of complete revolutions before finally resting in this position, the number of
those complete revolutions will not affect the values of the functions in any way. Let
P (x, y) be any point on the terminal ray — except for the origin. The definitions will
all involve only the ratio of the numbers x and y — and they won’t depend on which
point was chosen on the terminal ray!
60
On WeBWorK you should simply type pi , and the system will supply an approximation to the
accuracy that it recognizes — don’t use a calculator.
Notes for Lecture Section 002, MATH 140 2008 09 2026

Definitions of the 6 trigonometric functions We are going to define the func-


tions so that they have the following properties:

√ y
the sine of θ = sin θ =
x2 +y 2

the cosine of θ = cos θ = √ x


x2 +y 2
y
the tangent of θ = tan θ = x

the cotangent of θ = cot θ = x (26)


y

x2 +y 2
the secant of θ = sec θ = x

x2 +y 2
the cosecant of θ = csc θ = y

All six of these functions can be expressed as the ratios of the lengths of sides of a right
angled triangle with one vertex at the origin, one at the point P (x, y) on the terminal
ray, and the third vertex at the foot of the perpendicular dropped from P to the x-axis.
We could call x the length
p of the “adjacent” side of the triangle, y the length of the
“opposite” side, and x + y 2 is then the length of the longest side of the triangle,
2

which, following the ancient Greeks, we call the hypotenuse. But note that, while the
hypotenuse is always of positive length, the other two sides can be of positive or negative
length depending on the direction in which the length is measured from the origin.

Theorem 5.4 The ratios given in chart (26) are independent of which point P is chosen
on the terminal ray, avoiding the origin.

Proof: This can be proved by taking 2 different points, and showing, using similar
triangles, that the ratios corresponding to them are equal. ¤
Only after having proved Theorem 5.4 can we call (26) definitions of the 6 functions:
that is becausepthe statements we have made — giving the values of the functions in
terms of x, y, x2 + y 2 — could have been ambiguous, in that the values being assigned
to the functions could have depended on P (x, y); we have now eliminated that possibility,
and are clearly assigning a specific value to each of the functions at each of the points
of its domain.

The domains of the trigonometric functions The definitions given above are
valid everywhere except where the denominators of the ratios are 0: those points must be
excluded from the domains, since fractions aren’t defined having a 0 denominator. Thus,
for example, the tangent function is not defined when x = 0, i.e., when the terminal ray
Notes for Lecture Section 002, MATH 140 2008 09 2027

is along the y-axis, i.e., for odd numbers of right angles. Since a right angle has measure
π
2
radians, the domain of the tangent function is
n π ¯¯ o
R − (2n + 1) ¯ n ∈ Z .
2
Since the symbol x was not available — I was using it to denote the abscissa of the point
P (x, y) whose position was referred to in my definitions — I named the independent
variable in my definitions as θ. Now we can use the symbol x for the independent
variable of these functions, on the understanding that, if we are called upon to refer to
the coordinates of the point P that appears in the definition, we will no longer be able
to use x as the name of the abscissa of that point.

Periodicity of the trigonometric functions All six of the functions I have de-
fined have the property that, if the terminal ray of the angle be rotated through a further
angle of 1 revolution, or 2π radians, there is no change in the function value. Indeed, for
2 of these functions — the tangent and cotangent — this property of periodicity holds for
an angle of 12 revolution or π radians. We call these angles the periods of the respective
functions, and say that the functions are periodic. When we draw the graphs of these
functions, the period is a length of graph, measured horizontally, which repeats itself
infinitely often to the right and left. Thus, to evaluate the functions it suffices to study
them for one convenient interval whose length is the period. For the sine, cosine, secant,
and cosecant, we could work with a “fundamental” interval of, for example [0, 2π] or
[−π, π] (omitting the points where the functions
¡ ¢ are not defined); for the tangent and
cotangent it is convenient to work with − π2 , π2 and (0, π) respectively. Within these
intervals it is useful to determine the values of the functions at convenient multiples and
submultiples of π.

Values of trigonometric functions at convenient points For each of the an-


gles 0, π6 , π4 , π3 , π2 , etc., it is possible to calculate all 6 of the functions (except where
they are undefined) using simple geometric constructions — involving either isosceles or
equilateral triangles.

Exercise 5.4 In [1, Example 3, p. A27] the textbook determines the entries in a table
for the sine and cosine. You should be able to complete Table 5 on page 2028 for the
other functions. Rather than memorizing this table, you should be able to carry out
the calculations whenever that is required. The author has included entries for many
angles in the first quadrant — i.e., between 0 and π2 — but fewer and fewer as the angle
increases. The same methods could be used to determine similar angles that have not
been shown in the table, e.g., ± 11π
6
.
Notes for Lecture Section 002, MATH 140 2008 09 2028

π π π π 2π 3π 5π 3π
θ 0 6 4 π 2π
√3 2 √3 4 6 2
1 √1 3 3 √1 1
sin θ 0 1 0 −1 0
√2 2 2 2 2 2√
3 √1 1 3
cos θ 1 2 2 2 0 − 12 − 12
√ − 2 −1 0 1
tan θ undefined undefined .
cot θ undefined undefined undefined
sec θ undefined undefined
csc θ undefined undefined undefined

Table 5: Values of trigonometric functions at some “convenient” points (cf. Exercise 5.4)

Trigonometric Identities There are many ways in which the 6 trigonometric func-
tions are related. These relationships usually take the form of equations that are true for
all values of the independent variable. We call a relationship of this type an identity. As
I remind you of identities, I will sometimes offer a proof, sometimes not. We don’t expect
you to supply proofs; but sometimes understanding a proof may help you remember the
identity, or help you understand another identity that isn’t in the present list.

Reciprocal Trigonometric Functions We can see the following relationships by


examining the definitions (26):

1
csc x = sin x sin x = csc1 x

sec x = cos1 x cos x = sec1 x (27)


1 1
cot x = tan x tan x = cot x

(The identities are claimed to hold whenever both members of a pair of reciprocals are
defined.)

Expression of the Trigonometric Functions in Terms of Sines and Cosines


All 6 functions can be expressed in terms of certain pairs of two of the functions. For
Notes for Lecture Section 002, MATH 140 2008 09 2029

example:
sin x
tan x = cos x
cot x = cos x
sin x
(28)
sec x = cos1 x

csc x = sin1 x

Parity: Evenness and Oddness of Trigonometric Functions Examination of


definitions (26) shows that

sin(−θ) = − sin θ (29)


cos(−θ) = cos θ (30)

which, by identities (28), imply that

tan(−θ) = − tan θ (31)


cot(−θ) = − cot θ (32)
sec(−θ) = sec θ (33)
csc(−θ) = − csc θ (34)

In other words, we have


Theorem 5.5 The sine, tangent, cotangent, and cosecant functions are odd; the cosine
and secant functions are even.
In my definitions of the trigonometric functions I used the symbol θ for the variable, and
x and y to denote the coordinates of any point except the origin chosen on the radius
vector emanating from the origin. While the definitions of the functions involved an
angle at the origin, we now wish to view these functions simply as real valued functions
of a real variable. I will express this idea henceforth by usually using for the variable
the same symbols that we typically use for a real variable — usually lower-case letters
of the Latin alphabet.

Addition Formulas Many of the other identities that will interest us can be de-
rived from the following two identities:

sin(x + y) = sin x cos y + cos x sin y (35)


cos(x + y) = cos x cos y − sin x sin y (36)
Notes for Lecture Section 002, MATH 140 2008 09 2030

These identities are true for all real numbers x, y. If we replace y by −y wherever it
appears, we obtain an equivalent pair of identities:
sin(x − y) = sin x cos(−y) + cos x sin(−y)
cos(x − y) = cos x cos(−y) − sin x sin(−y)
which we can simplify by remembering that the sine is odd and the cosine is even:
sin(x − y) = sin x cos y + cos x(− sin y) = sin x cos y − cos x sin y (37)
cos(x − y) = cos x cos y − sin x(− sin y) = cos x cos y + sin x sin y (38)
We can derive from these formulas addition and subtraction formulas for the other four
functions; for example
tan x + tan y
tan(x + y) = (39)
1 − tan x tan y
tan x − tan y
tan(x − y) = (40)
1 + tan x tan y
which are valid whenever the denominators are non-zero.

Sum of Squares Formulas If, in formula (38), we take x = y, we obtain


cos 0 = cos2 x + sin2 x .
Recalling that cos 0 = 1, we have
sin2 x + cos2 x = 1 (41)
By dividing by cos2 or by sin2 x we obtain two other identities of the same type:
tan2 x + 1 = sec2 x (42)
1 + cot2 x = csc2 x (43)

D Exercises
[1, Exercise 4, p. A32] Convert from degrees to radians: −315◦ .
Solution:
360◦ = 2π radians

⇒ 1◦ = radians
360
2π 7π
⇒ −315◦ = (−315) × or − radians.
360 4
Notes for Lecture Section 002, MATH 140 2008 09 2031


[1, Exercise 10, p. A32] Convert from radians to degrees: 3
radians.
Solution:

π radians = 180◦
180 ◦
⇒ 1 radian =
π
8π 8π 180 ◦
⇒ radians = × or 480◦ .
3 3 π

[1, Exercise 14, p. A32] If a circle has radius 10 cm., find the length of the arc sub-
tended by a central angle of 72◦ .
72
Solution: arc length = rθ = 10 × 180 × π = 4π. Note that, to apply this formula,
we must first convert the angle to radian measure.

[1, Exercise 28, p. A32] Find the values of the 6 trigonometric functions for an angle
of 11π
4
radians.
j 11π k ¥ ¦
Solution: Since 2π 4
= 11
8
= 1, the angle lies between 1 and 2 revolutions. The
trigonometric functions of this angle coincide with those of 11π
4
−1(2π) = 34 π, which
bisects the 2nd quadrant. One possible pair of values for the point P is (−1, 1),
yielding the values
1 1 √ √
sin θ = √ , cos θ = − √ , tan θ = −1 = cot θ, sec θ = − 2, csc θ = 2.
2 2
(Students may have been taught that writing square roots in the denominator is
“bad form.” This was indeed the case in the days when computations had to be
done using tables. Nowadays there is less reason to avoid writing surds in the
denominator in simple expressions, although it is still easier to carry out simplifi-
cations if the denominators are integers.)

[1, Exercise 32, p. A32] Find the remaining trigonometric ratios: cos θ = − 31 , π <
θ < 3π
2
.
Solution: The textbook shows, in [1, Example 4, p. A27], a graphical method for
solving this problem. This method should be avoided, except for angles in the first
quadrant! The reason is that it ignores the signs of the coordinates of the point
P (x, y), and can lead to sign errors in the trigonometric ratios. If, however, the
student works with signed lengths, as in [1, Figure 9 accompanying Example 3,
p. A27], the method is quite acceptable. Instead, I will solve this problem in a
non-graphical way.
Notes for Lecture Section 002, MATH 140 2008 09 2032

We can find the values of all of the trigonometric functions if we know the sine
and cosine. We are given that cos θ = − 13 , so, since sin2 θ + cos2 θ = 1, sin θ =
q q
± 1 − 91 = ± 89 . But which sign is applicable here? The information that
π < θ < 3π2
tells us that the angle is in the 3rd quadrant, where we know sines
toqbe negative (since the ordinate y of P (x, y) will be negative). Hence sin θ =

− 89 = − 2 3 2 . We can compute the values of the other functions from the sine
and cosine:
sin θ √
tan θ = =2 2
cos θ
1 1
cot θ = = √
tan θ 2 2
1
sec θ = = −3
cos θ
1 3
csc θ = =− √
sin θ 2 2

[1, Exercise 52, p. A32] Prove the identity


1 1
+ = 2 sec2 θ
1 − sin θ 1 + sin θ

Solution: When a question asks for a proof, the student needs to know what meth-
ods are acceptable, and what other results can be assumed. Problems involving
identities are standard in any good trigonometry course, and you should have seen
this type of problem before. The intention here would be to make use of either
basic properties of the functions, or of standard identities like those listed above.
While the textbook has not explicitly mentioned it, you will have to assume that θ
is not one of the numbers which will render any of the functions undefined — i.e.,
that θ is not an odd integer multiple of π2 .
One way to prove an identity is to transform both sides of the claimed equation
into the same function, and this is probably the way in which most rough proofs
begin. But a more elegant way of expressing a proof is to start with either side of
the claimed equation and, by successive, reversible operations, transform it to the
other side. The following is just one possible proof:

1 1 (1 + sin θ) + (1 − sin θ)
+ =
1 − sin θ 1 + sin θ (1 − sin θ)(1 + sin θ)
taking the fractions to a common denominator
Notes for Lecture Section 002, MATH 140 2008 09 2033

2
=
1 − sin2 θ
2
= by (41)
cos2 θ
= 2 sec2 θ by (27) .

(In this course we don’t plan to spend time on the proofs of trigonometric identities
— but you need to know how to carry out such proofs, since the identity might be
needed in the course of a calculus problem; also we may be proving other types of
identities that use the same kinds of methods as those you should have learned in
a trigonometry course.)
Notes for Lecture Section 002, MATH 140 2008 09 2034

5.3 Supplementary Notes for the Lecture of September 10th,


2008
Release Date: Wednesday, September 10th, 2008

(I plan to discuss [1, §1.3] before continuing the review of trigonometry.)

5.3.1 §1.3 New Functions from Old Functions.


Transformations of Functions Read this material. You will not be expected to
carry out “shifts”, “stretching”, “compression”, “reflection” on a given function, but you
are certain to meet functions that have been obtained from others in these ways.

Combinations of Functions.

Algebraic sum, difference, product, quotient of functions. Given functions


f and g, we can define the functions f + g, f − g, f g, f /g by a precise definition of the
value of each of these functions at a general point x of its domain. While I am not likely
to use these notations in the course, we will have occasion to combine functions in these
ways — it’s only the notation that we may not be using. The domains of these functions
can never be greater than the set of points where both of the combinant functions are
defined, i.e., the intersection of the domains of the functions f and g; except that, in the
case of f /g, we have to exclude points where g is 0.

Example 5.6 [7, Exercise 1.3.32, p. 47] Find f + g, f − g, f g, and f /g, and state their
domains: √ √
f (x) = 1 + x g(x) = 1 − x .
Remember the textbook’s definitions:

(f ± g)(x) = f (x) ± g(x) (44)


(f g)(x) = f (x)g(x) (45)
µ ¶
f f (x)
(x) = (46)
g g(x)

Solution: When the domains of the functions have not been stated explicitly, our con-
vention is that the domains are to be the largest possible in each case. The function
f is the composition of two functions: first x is mapped on to 1 + x, a function whose
(maximal) domain is the whole real line R. Then the result of that first operation is
subjected to the square root function, which is defined for all non-negative real numbers;
Notes for Lecture Section 002, MATH 140 2008 09 2035

thus that phase is well defined if and only if 1 + x ≥ 0, i.e., iff61 x ≥ −1. Thus the
domain of f is x ≥ −1. In a similar way, we can show that the domain of g is the set of
all x such that 1 − x ≥ 0, i.e., x ≤ 1. When we combine these two functions f and g,
the domain will be the intersection of these two domains — from which we will have to
further delete the point x = −1 — the point where the g(x) = 0 — in the case of f /g.
The intersection is the closed interval −1 ≤ x ≤ 1. Thus we have

domain of f ± g, f g = [−1, 1]
domain of f /g = [−1, 1)

(As an exercise you might try to determine the range (or image) of each of the new
functions.)

Composition of Functions Sometimes we wish to follow one function g by an-


other function f . For that to be defined, we require that the points in the image (range)
of g be contained in the domain of f . The combined action of g followed by f is called
the composition or composite, and is denoted by f ◦ g, formally defined as follows:

Definition 5.2
(f ◦ g)(x) = f (g(x))

Remember that the definition of f ◦ g takes g as the function applied first, and f second
[1, p. 41]. This definition is a practical consequence of the notation we are using for
functions, wherein we place the name of the function to the left of the name of the point
on which it acts. This means that the actions of the functions can be represented by a
diagram like
g f
x 7→ g(x) 7→ f (g(x)) = (f ◦ g)(x) .
It is possible to compose a sequence of functions, provided the image of each is contained in
the domain of the next; in an algebra course we would show that there is no need to write
parentheses, since f ◦ (g ◦ h) = (f ◦ g) ◦ h.

Example 5.7 [7, Exercise 1.3.48, p. 47] Express the function G in the form f ◦ g:
1
G(x) =
x+3
Solution: The last operation performed in the evaluation of G is the taking of a reciprocal.
1
Define f (x) = . Prior to that the first step was x 7→ x + 3, which we can define to be
x
the function g. Then G = f ◦ g.
61
a mathematicians’ abbreviation for “if and only if”; in French we write ssi
Notes for Lecture Section 002, MATH 140 2008 09 2036

(The domain of g is R; the domain of f is R − {0}. Hence the domain of G is62 “the
set of all x in the domain of g such that g(x) is in the domain of f ”, i.e., the set of all x
such that x + 3 6= 0, i.e., R − {−3}.)
The preceding is probably the solution that the textbook was seeking; but it not
the only solution. For example, if we define h(x) = x for all x; then G = G ◦ h; also,
G = h ◦ G. Less trivially, if we define k(x) = − x1 , and `(x) = −x − 3, then G = k ◦ `.

1.3 Exercises

[1, Exercise 1.3.32, p. 44] Find the functions (a) f ◦ g, (b) g ◦ f , (c) f ◦ f , (d) g ◦ g
and their domains. If you can, comment about the images of the compositions.63 :
f (x) = x − 2, g(x) = x2 + 3x + 4.
Solution: The given functions are polynomials, so their domains are both all of
R. Their compositions are defined for all real numbers, so the domains of the four
compositions are also all equal to R.

(a)

(f ◦ g)(x) = f (g(x))
= f (x2 + 3x + 4)
= (x2 + 3x + 4) − 2 = x2 + 3x + 2

But what is the image of this function? It is the set of values that the function
assumes. Later in the course we will see how to solve this problem using the
calculus. But it can be solved without calculus, if we observe (by completing
the square) that
µ ¶2
2 3 1
x + 3x + 2 = x + − .
2 4
The square can assume
£ all real values ≥ 0; hence the composition has as its
image the interval − 14 , ∞ ).
(b)

(g ◦ f )(x) = g(x − 2)
= (x − 2)2 + 3(x − 2) + 4
= x2 − x + 2
62
[1, p. 41]
63
This extension was added by your instructor. You don’t yet have the machinery to solve this part
of the problem systematically.
Notes for Lecture Section 002, MATH 140 2008 09 2037

As before, we can complete the square, to show that


µ ¶2
2 1 7
x −x+2= x− +
2 4
£7
and conclude that the image is 4 , ∞ ).
(c)
(f ◦ f )(x) = f (x − 2)
= (x − 2) − 2
= x−4
Here the image of the composite function is R.
(d)
(g ◦ g)(x) = g(x2 + 3x + 4)
¡ ¢2
= x2 + 3x + 4 + 3(x2 + 3x + 4) + 4
= x4 + 6x3 + 20x2 + 33x + 32
¡ ¢2
What about the image? We can observe that x2 + 3x + 4 = x + 32 + 74 ≥ 74 :
the function g is minimized (with minimum value 47 ) when x = − 32 , and
is increasing for x ≥ − 32 . So, if we evaluate the function for x ≥ 74 , it
will be minimized when x = 74 — but that’s what we are doing when we
evaluate g ◦¡ g.¢ Thus the composite function g ◦ g can’t assume any value
less than g 74 = 197 16
. We will see later in the course that this composite
function assumes arbitrarily large values, and also that
£ it must take on all
values between any two values. Hence its image is 197 16
, ∞ ). We will have
other ways of systematically determining the minimum value of g ◦ g.
[1, Exercise 1.3.36, p. 44] Find the functions (a) f ◦ g, (b) g ◦ f , (c) f ◦ f , (d) g ◦ g
x
and their domains: f (x) = 1+x , g(x) = sin 2x.
Solution: First observe that g is defined for all real numbers x. But f is defined only
if its denominator is non-zero, i.e., it has domain R−{−1} = (−∞, −1)∪(−1, +∞).
(a)
sin 2x
(f ◦ g)(x) = .
1 + sin 2x
This ratio is meaningful only if sin 2x 6= −1, i.e., provided 2x is distinct from

2
+ 2nπ where n is any integer. Thus the domain of f ◦ g is
½µ ¶ ¯ ¾
3 ¯
R− ¯
+ n π¯ n ∈ Z .
4
Notes for Lecture Section 002, MATH 140 2008 09 2038

(b) The function µ ¶


2x
(g ◦ f )(x) = sin
1+x
is defined for all x distinct from −1.
(c) The function µ ¶ x
x 1+x x
(f ◦ f )(x) = f = 1+2x =
1+x 1+x
1 + 2x
provided x 6= −1. Here the domain cannot include −1, since the first applica-
1
tion of f is not defined at x = −1; but it also cannot include
© − 2 1, ªwhich would
cause the denominator to be 0. Thus the domain is R − −1, − 2 . Note that
the point x = −1 must be excluded from the domain even though the reduced
fraction we last computed with is fully defined at x = −1; perhaps, instead
x
of writing the fraction as 1+2x together with the restriction that x 6= −1, I
x(1+x)
should have written (1+2x)(1+x) — then the restriction would not have had to
be explicitly stated, as the fraction is not defined when x = −1.
(d) (g ◦ g)(x) = sin (2 sin 2x). This function is defined for all real numbers x.
[1, Exercise 1.3.44,rp. 44] Express the following function as a composition of the form
x
f ◦ g: G(x) = 3 .
1+x
Solution: It is convenient to take theµlast function
¶ applied to be the cube root
√ x x
function: f (x) = 3 x. Thus G(x) = f , and we can define g(x) = .
1+x 1+x
[1, Exercise 1.3.56, p. 44] An airplane is flying at a speed of 350 mi/h at an altitude
of one mile, and passes directly over a radar station at time t = 0.
(a) Express the horizontal distance d (in miles) that the plane has flown as a
function of t.
(b) Express the distance s between the plane and the radar station as a function
of d.
(c) Use composition to express s as a function of t.
Solution:
(a)
Distance traversed in 1 hour = 350 miles
⇒ Distance d traversed in t hours = 350t miles.
In function notation, this could be written as d(t) = 350t.
Notes for Lecture Section 002, MATH 140 2008 09 2039

(b) A right-angled triangle is formed by the radar station, the point 1 mile directly
above it (which is the right-angle), and the point where the plane is located at
time t. By the Theorem of Pythagoras, the length of the hypotenuse √ — which
measures the distance from the plane to the radar station — is s = 1 + d2 .

(c) If we define f (x) = x, g(x) = 1 + (350x)2 , then s(t) = f (g(t)), so we can
write s = f ◦ g.

5.3.2 Appendix D. Trigonometry (continued)


Double Angle Formulas; Half-Angle Formulas If, in the addition formulas,
we take x = y, we obtain
sin 2x = 2 sin x cos x (47)
cos 2x = cos2 x − sin2 x (48)
The second formula can be written in other ways, by applying identity (41):
cos 2x = cos2 x − sin2 x = 2 cos2 x − 1 = 1 − 2 sin2 x (49)
If, in identity (49), we solve for the functions whose argument is x, we obtain
1 − cos 2x
sin2 x = (50)
2
1 + cos 2x
cos2 x = (51)
2
Note that it is customary to write powers of trigonometric functions by simply placing
an exponent above the upper right-hand corner of the function name. This custom is
restricted to trigonometric and hyperbolic functions64 . You should not use this notation
for the (−1)st power, i.e., for the reciprocal, as that particular symbol will have another
meaning. These three identities can all be interpreted as being equivalent to the Theorem
of Pythagoras, that the square on the hypotenuse of a right-angled triangle is equal to
the sum of the squares on the other two sides.

Trigonometric Cofunctions From the original definitions we see that sin π2 = 1,


cos π2 = 0. Applying the subtraction formulas we obtain
³π ´ π π
sin − x = sin cos x − cos sin x
2 2 2
= cos x − 0 = cos x (52)
³π ´ π π
cos − x = cos cos x + sin sin x
2 2 2
= 0 + sin x = sin x (53)
64
not to be met until [1, §3.11]
Notes for Lecture Section 002, MATH 140 2008 09 2040

We can prove analogous relationships between the other two pairs of trigonometric func-
tions:
³π ´
tan − x = cot x (54)
2
³π ´
cot − x = tan x (55)
2
³π ´
sec − x = csc x (56)
2
³π ´
csc − x = sec x (57)
2

Conversion of Products into Sums and Differences By taking half the sum
of identities (35) and (36) we can obtain
sin(x + y) + sin(x − y)
sin x cos y = (58)
2
and, analogously, using (37) and (38),
cos(x + y) + cos(x − y)
cos x cos y = (59)
2
cos(x − y) − cos(x + y)
sin x sin y = (60)
2

Conversion of Sums and Differences into Products It is also possible to


develop identities that convert sums of sines and sums of cosines into products. These
identities, which should have been met in a good trigonometry course, will not be required
in MATH 140.

Law of Sines and Law of Cosines for the 3 angles of a Triangle I list without
proof several properties that pertain to general — not only right-angled — triangles.
1. Law of Sines: If the (lengths of the) sides of a triangle are a, b, c, and the angles
opposite them are A, B, C, then
sin A sin B sin C
= = (61)
a b c

2. Law of Cosines: [1, Exercise 83, p. A33] If the (lengths of the) sides of a triangle
are a, b, c, and the angle between a, b is denoted by C, then

c2 = a2 + b2 − 2ab cos C (62)


Notes for Lecture Section 002, MATH 140 2008 09 2041

3. Area of a triangle: [1, Exercise 88, p. A33] The area of a triangle can be shown
to be
1
× length of base × height ,
2
(known to Euclid over two thousand years ago). By the Law of Sines this is equal
to
1 1 1
ab sin C = bc sin A = ca sin B (63)
2 2 2
where we again follow the convention that the lengths of the sides opposite angles
A, B, C are denoted by a, b, c.

You might be called upon to use these properties in some Optimization Problems in the
textbook, These are simply part of the general “precalculus” knowledge that you should
be bringing into a calculus course.

Graphs of the trigonometric functions. Portions of the graphs of the 6 trigonomet-


ric functions for −4π ≤ x ≤ 4π are shown in Figures 2, 3, 4, 5, 6, 7, respectively on pages
2041, 2041, 2042, 2043, 2044, 2045. You should be familiar with the approximate

0
-10 -5 0 5 10

Figure 2: Graph of the Function sin x

0
-10 -5 0 5 10

Figure 3: Graph of the Function cos x

shapes of the graphs of the trigonometric functions [1, Figures 13, 14, pp. A30-A31]. No
one expects you to draw a precise graph — just a crude sketch that indicates things like
the periodicity of the function, its domain and “range”, etc.
Notes for Lecture Section 002, MATH 140 2008 09 2042

10

0
-10 -5 0 5 10

-5

-10

Figure 4: Graph of the Function tan x

Some types of trigonometry problems that you should be able to solve.


Trigonometry is among the prerequisites to this course, so you should be able to solve
any types of problems that normally appear in a precalculus course. Some of these are
more likely than others to appear in MATH 140. Usually a calculus problem will not
be primarily trigonometric; but, if you are unable to deal with the trigonometric issues
that arise, you will be unable to solve the calculus problem. For example,
1. To solve an equation relating trigonometric functions of one or more variables.
For example, [1, Example 6, p. A30] asks you to determine where the graphs of
y = sin x and y = sin 2x intersect.
2. Given the value of, for example, sec x, to determine the value of, for example,
Notes for Lecture Section 002, MATH 140 2008 09 2043

10

0
-10 -5 0 5 10

-5

-10

Figure 5: Graph of the Function cot x

sin x. Usually this should not be solved by first attempting to determine x; rather,
you need to be able to express one trigonometric function in terms of the others.
This could involve a possible choice of two solutions; sometimes there is additional
information available to enable you to exclude one of the possible solutions. In the
specific problem mentioned, one could proceed as follows:

sin x = ± 1 − cos2 x
r
1
= ± 1−
sec2 x
√ √
sec2 x − 1 sec2 x − 1
= =±
± sec x sec x
Notes for Lecture Section 002, MATH 140 2008 09 2044

10

0
-10 -5 0 5 10

-5

-10

Figure 6: Graph of the Function sec x

(where the last two steps are simply to improve the appearance of the answer).

3. Determination of some facts about side lengths and angle magnitudes of a given
triangle from other given facts; this is sometimes called “Solution of a triangle”.
Where the triangle is right-angled, this is usually not complicated. Where the
triangle is not right-angled, you could need to “drop” a perpendicular from a
vertex to the opposite side; or to use the Law of Sines or Law of Cosines.

4. More than being able to solve trigonometry problems, you need to be able to
work efficiently with the functions. Sometimes this is best done by applying some
identities involving the functions. For example, the formulæ which express sin2 x
Notes for Lecture Section 002, MATH 140 2008 09 2045

10

0
-10 -5 0 5 10

-5

-10

Figure 7: Graph of the Function csc x

and cos2 x in terms of cos 2x can be used to reduce the degree of a product of
sines and cosines, a simplification which may make a great difference in solving a
problem where trigonometric functions happen to appear.

5. While the derivation of trigonometric identities is not central to this course, the
skills learned from that part of a trigonometry course will be needed in this first
calculus course. Remember the procedures to follow in proving an identity of the
form
f (x) = g(x)
where f and g are functions that are known.
Notes for Lecture Section 002, MATH 140 2008 09 2046

• Your proof must be completely general: you must not restrict the variable x
in any way beyond the restrictions that are stated in the problem.
• One method of solution is to transform the two sides of the equation until
each of them can be shown to be equal to the same convenient function of x.
• A more elegant solution would start on one side of the alleged equation and,
through a sequence of reversible transformations, evolve it into the other side
of the equation. This type of solution is more pleasing than the preceding,
but does not always suggest itself immediately.

D Exercises (continued)

[1, Exercise 72, p. A33] Find all values of x in the interval [0, 2π] that satisfy the
equation 2 + cos 2x = 3 cos x.
Solution:

2 + cos 2x = 3 cos x ⇔ 2 + (2 cos2 x − 1) = 3 cos x


⇔ 2 (cos x)2 − 3 cos x + 1 = 0
a quadratic polynomial in cos x
⇔ (2 cos x − 1)(cos x − 1) = 0
1
⇔ cos x = or cos x = 1
2
π 5π
⇔ x= , or x = 0, 2π.
3 3
Notes for Lecture Section 002, MATH 140 2008 09 2047

5.4 Supplementary Notes for the Lecture of September 15th,


2008
Release Date: Monday, September 15th, 2008

5.4.1 Appendix D. Trigonometry (conclusion)


D Exercises (conclusion)

[1, Exercise 74, p. A33] Find all values of x in the interval [0, 2π] that satisfy the
inequality 2 cos x + 1 > 0.
Solution: The inequality is equivalent to cos x > − 21 = cos 2π3
= cos 4π
3
. The points
2π 4π 1
3
and 3
are the only values where the cosine is equal to − 2
in the interval [0, 2π].
One way to solve the problem is to make use of the fact that the cosine function
is decreasing in the first and second quadrants, and increasing in the third and
fourth. It follows that either 0 ≤ x < 2π
3
or 4π
3
< x ≤ 2π.

[1, Exercise 76, p. A33] Find all values of x in the interval [0, 2π] that satisfy the
inequality sin x > cos x.
Solution:

First Solution: One way to solve this problem is to divide both sides by the
cosine. But, to do that, we need to know the sign of the cosine, since the
inequality will be reversed when the cosine is negative. A foolproof way of
solving the problem is to divide the interval into several parts. Another snag
is that we can’t divide by 0:
Case 1: cos x = 0. In the interval [0, 2π] the cosine is equal to 0 only when
x = π2 , 3π
2
. At x = π2 , sin x = 1 > 0 = cos x, so x = π2 is one solution. At

x = 2 sin x = −1 < 0 = cos x, so this point does not yield a solution to
the inequality.
Case 2: cos x > 0. This situation occurs in the intervals 0 < x < π2 and 3π 2
<
x < 2π. Division by cos x preserves the inequality, yielding tan x > 1.
The tangent function is an increasing function, and attains the value 1
at π4 and 5π 4
. In the first quadrant this implies that all points in the
subinterval 4 < x < π2 are solutions. In the 4th quadrant the tangent is
π

negative, so we obtain no solutions.


Case 3: cos x < 0. This situation occurs in the interval π2 < x < 3π 2
. Division
by cos x reverses the inequality, yielding tan x < 1. The tangent function
is an increasing function, and attains the value 1 at 5π 4
. It will be less
than 1 for π2 < x < 5π 4
.
Notes for Lecture Section 002, MATH 140 2008 09 2048

We have shown that the solutions are all points in


n ¯π ½ ¯ ¾
¯ π o nπ o ¯π
¯ 5π
x¯ < x < ∪ ∪ x¯ < x < ,
4 2 2 2 4
¡ ¢
i.e., the open interval (without its end-points) π4 , 5π
4
.
Another Solution: If we subtract from identity (36) on page 2029 of these notes
identity (37) on page 2030, we obtain the identity

sin(x + y) − sin(x − y) = 2 cos x · sin y .


u+v u−v
If we define u = x + y, v = x − y, we obtain x = 2
, y= 2
, and the new
identity becomes
u+v u−v
sin u − sin v = 2 cos · sin .
2 2
Now we can proceed as follows to apply this identity:

sin x > cos x ⇔ sin x − cos ³x > 0 ´


π
⇔ sin x − sin −x >0
µ2 ¶
π 2x − π2
⇔ 2 cos · sin >0
4 2
1 ³ π´
⇔ 2 √ · sin x − >0
2 4
³ π´
⇔ sin x − > 0.
4
But we know where the sine function is positive — it is in the first or second
quadrants. So the given inequality is equivalent to all of the inequalities

...
−2π < x − π4 < −π
0 < x − π4 < π
2π < x − π4 < 3π
...

and the only interval which is contained in the given interval [0, 2π] is the
π π 5π
first, 0 < x − < π, which is equivalent to < x < , which is the same
4 4 4
interval found earlier, in a solution by cases.
Notes for Lecture Section 002, MATH 140 2008 09 2049

Third Solution: This solution is very short, but would need a result from [1,
§2.5]. We can first determine the points where sin x − cos x = 0. We know
that sin x and cos x can’t both be 0, as the sum of their squares is 1; so we can
divide by cos x, and obtain an equivalent equation, tan x = 1, whose solutions
we know to be x = π4 , 5π
4
and no others in the interval [0, 2π]. Between the ends
of this interval and the two points where the difference vanishes, the function
can be shown to always have the same sign — this is an application of the
“Intermediate Value Theorem”, which we will meet in [1, §2.5]. Thus we need
only take some sample point in each of the subintervals to determine whether
sin x − cos x is positive or negative in the interval, to determine where the
sine exceeds the cosine. (Normally it is not justified to draw conclusions from
working with “sample” points. The conclusion is justified in this case because
we know that the difference changes sign only at the two points we found,
and that between any two points where the continuous function sin x − cos x
is of opposite signs there must be a point where it is equal to 0.)

[1, Exercise 83, p. A33] This problem asks you to prove the Law of Cosines. You are
not expected to know how to prove the Law of Cosines; but you do need to be able
to use it, as in [1, Exercise 84, p. A33].

[1, Exercise 85, p. A33] This problem asks you to prove identity (38) which was
quoted above. You are not expected to be able to prove this identity in this
course; but you do need to remember it, and to be able to use it.

[1, Exercise 88, p. A33] This problem asks you to prove (63) above. You are not
expected to be able to reproduce this proof, but you need to remember the result.

Review exercises from the Trigonometry Supplement used in this course in


2005/2006.

[21, Exercise 62, p. 558] Prove the identity,

4(sin6 x + cos6 x) = 4 − 3 sin2 2x .

Solution: By an identity we mean an equation involving variables, which is true for


all values of those variables in a prescribed domain. Thus it’s not enough to know
the equation is true sometimes: it must be true for every value, and its failure for
just one value of x would be enough to prevent us from calling it an identity. As
mentioned above, the most elegant solutions of problems of this type will begin
with one side of the alleged equation, and, by applying various results that we
know, progressively transform it into the other side. This strategy is not always
Notes for Lecture Section 002, MATH 140 2008 09 2050

easy to see at first. It is best to begin with what you see as the “more complicated”
of the two members of the equation.
I plan to base my argument on the factorization of the left side of the equation.
You may not know how to factorize a sum of 6th powers, but you should know how
to factorize a sum of 3rd powers:
a3 + b3 = (a + b)(a2 − ab + b2 ) .
So let’s interpret the 6th powers as cubes of squares.
³¡ ¢3 ¡ ¢3 ´
4(sin6 x + cos6 x) = 4 sin2 x + cos2 x
= 4(sin2 x + cos2 x)(sin4 x − sin2 x cos2 x + cos4 x)
= 4 · 1 · (sin4 x − sin2 x cos2 x + cos4 x) by (41).
At this point we examine the expression within parentheses on the right, and
observe that it looks something like a square. If we compare it with
(sin2 x + cos2 x)2 = sin4 x + 2 sin2 x cos2 x + cos4 x ,
we see that we are short 3 sin2 x cos2 x. So we have
4(sin6 x + cos6 x) = 4(sin4 x − sin2 x cos2 x + cos4 x)
³¡ ¢2 ´
= 4 sin2 x + cos2 x − 3 sin2 x cos2 x
¡ ¢
= 4 12 − 3 sin2 x cos2 x
= 4 − 3 (2 sin x cos x)2 .
And why have I expressed the subtracted term in this way? Because I wish to
make use of the double angle formula (47):
sin 2x = 2 sin x · cos x .
We obtain
4(sin6 x + cos6 x) = 4 − 3 sin2 2x
as required.
Another approach would be
¡ ¢3
4(sin6 x + cos6 x) = 4 sin6 x + 4 cos2 x
¡ ¢3
= 4 sin6 x + 4 1 − sin2 x
¡ ¢
= 4 sin6 x + 4 1 − 3 sin2 x + 3 sin4 x − sin6 x
= 4 − 12 sin2 x + 12 sin4 x
¡ ¢
= 4 − 12 sin2 x 1 − sin2 x
= 4 − 12 sin2 x cos2 x = 4 − 3(2 sin x cos x)2 = 4 − 3 sin2 2x .
Notes for Lecture Section 002, MATH 140 2008 09 2051

[21, Exercise 48, p. 558] Find the value of the product 3 cos 37.5◦ · cos 7.5◦ .
Solution: First note that does not contradict my earlier statement that I wish the
argument of trigonometric functions to be expressed only in radians. We must
interpret these expressions as referring to the radian equivalent of the angle given
in angles. There will be no need to actually compute that equivalent.
What is the point of this problem? The intention is that you should be able
to “simplify” products into sums. It is often — but not always — preferable to
work with a sum of like trigonometric functions rather than a product. So the
intention here is that you should convert this product of cosines into a sum of
sines or cosines. The tool we use is a family of identities which your textbook calls
“product formulas”. These formulæ are consequences of the Addition formulæ. If
we add the expansions of cos(x + y) and cos(x − y), we obtain

cos(x + y) + cos(x − y) = 2 cos x · cos y (64)

Taking x = 37.5◦ and y = 7.5◦ , we obtain


3
3 cos 37.5◦ · cos 7.5◦ = (cos 45◦ + cos 30◦ )
2
3³ π π´
= cos + cos
2Ã 4 3
√ !
3 1 3
= √ +
2 2 2

Why, you might ask, is this answer an improvement over the original product —
since one will still need to use a calculator to evaluate it? One needs to recognize
that numbers like square roots can be evaluated, albeit laboriously, by hand; while
the evaluation of trigonometric functions is much more complicated65 . The uses
of the procedures to convert from products to sums are more convincing when
one considers, for example, a product of the cosines of variables rather than of
constants, which is the case in the present, easy example.
π
[21, Exercise 52, p. 558] Find the value of the sum cos 12 + cos 5π
12
.
Solution: This is an example of the reverse of the operation considered in [21,
Exercise 48, p. 558], solved above. There are situations where one finds a product
more amenable than a sum. In order to make this more convincing, I am going
make the question more difficult. So let’s change it to
¡ π
¢ ¡ ¢
“Find the value of the sum cos x + 12 + cos x + 5π
12
.”
65
although it can still be done by hand, as you will see if you take Calculus 3
Notes for Lecture Section 002, MATH 140 2008 09 2052

(So the case that we are being asked about is x = 0.) To convert from a sum to
a product we again use the identity (64) proved above. This is an identity — true
for all real numbers x and y. Suppose that we choose x and y so that

x+y = X
x−y = Y

These two equations are equivalent to those obtained by solving the system for X
and Y in terms of x, y:
X +Y
x =
2
X −Y
y =
2
so (64) can be rewritten as
X +Y X −Y
cos X + cos Y = 2 cos · cos . (65)
2 2
π 5π
For the general problem stated above take X = x + 12
, Y =x+ 12
: thus
X +Y π
= x+
2 4
X −Y π
= −
µ 2 ¶ 6
³ π´ 5π ³ π´ ³ π´
so cos x + + cos x + = 2 cos x + · cos −
12 12 4 6
³ π ´ π
= 2 cos x + · cos
³ 4 ´ 6
√ π
= 3 cos x + .
4
The identity shows us that this sum of two cosines has a graph which is a scaled
and translated sine curve. As for the special case, setting x = 0 yields
r
π 5π √ 1 3
cos + cos = 3· √ = .
12 12 2 2

Further Review Exercises in Trigonometry


Exercise 5.5 The following problems are taken from a textbook which was once a stan-
dard high school source [36]. While the edition is over a century old, the problems are
still appropriate trigonometric background for a calculus course.
Notes for Lecture Section 002, MATH 140 2008 09 2053

1. [36, Problem 13, p. 62] Show that tan2 x − sin2 x = sin4 x · sec2 x.

2. [36, Problem 26(2), p. 63] Solve the equation tan θ + sec π6 = cot θ.
5 sin x − 3 cos x
3. [36, Problem 27, p. 63] If 5 tan x = 4, determine the value of
sin x + 2 cos x
without using inverse trigonometric functions.
sin 3x
4. [36, Problem 10, p. 121] Determine the domain of the function , and
sin 2x − sin x
express the function in terms of cos x.
5
5. [36, Problem 4, p. 337] If sin x = 13
, find the value of tan x + sec x.

6. [36, Problem 45, p. 341] Find all x such that 0 ≤ x ≤ 2π and 2 cos2 x = 1 + sin x.

7. [36, Problem 63, p. 344] Let α and β be fixed real numbers, where β is not an
integer multiple of π. Determine the domain and image of the function

sin(α + x) − sin(α − x)
f (x) = .
cos(β − x) − cos(β + x)

Remedying deficiencies in your precalculus background Some of the topics


I have been discussing should be part of any reasonable high school precalculus or
trigonometry course — but they may not have been part of yours. Don’t despair! Possi-
bly you were short-changed by your high school, or by the bureaucracy that prescribed
the syllabus for your high school — or you may simply not have paid attention when the
topics were covered. Whichever was the case, you can probably remedy deficiencies, as
you may have to remedy other deficiencies in your training, but it will take some effort
on your part, and you will have to manage that remediation on your own. Provided the
gaps are not too extensive, if you were good enough to be admitted to McGill, you should
be able to accomplish this painlessly.
However if you never had a precalculus course, or have never studied any trigonom-
etry, you might be advised to drop MATH 140 2008 09, and to take MATH 112 2008 09
— a precalculus course that we offer only in the fall term.

5.4.2 §1.4 Graphing Calculators and Computers (OMIT)


This section may be safely omitted; we will not be using calculators or computers in this
course.
Notes for Lecture Section 002, MATH 140 2008 09 2054

5.4.3 §1.5 Exponential Functions


(cf., these notes, §5.2.1, p. 2022) In this “early transcendental” treatment of exponentials,
the author’s introduction is intuitive, without proofs. He states the

Theorem 5.8 (Laws of Exponents [1, p. 54]) Let a and b be any positive real num-
bers, and x and y be any real numbers. Then

1. ax+y = ax ay .
ax
2. ax−y =
ay
3. (ax )y = axy

4. (ab)x = ax bx

Note that one consequence of these “Laws” — take x = y = 0 in Theorem 5.8.3 — is


that a0 = 1, and that is true even when a = 0: 00 = 1. We will not attempt to prove
Theorem 5.8 in this course, and you will not be expected even to cite the theorem by
name whenever you need to use it. At this point in this course you are asked to assume
these laws without proof, and to be comfortable using them. These are skills you should
be bringing with you from your pre-calculus and earlier courses.

Applications of Exponential Functions This is motivational material. Read it.

The Number e In MATH 141 you will see ways in which the constant e can be defined
formally, and can be evaluated to any desired accuracy:

e = 2.71828182845904523536028747135266249775724709369995957... (66)

For the present it is satisfactory to think of e as that constant c — between 2 and 3 — for
which the tangent to the graph of the exponential function cx makes an angle of π4 radians
with the y-axis where it crosses that axis (at the point (x, y) = (0, 1)). An alternative
definition will be found on [1, p. 179, §3.1]. The “early transcendental” presentation of
the calculus makes do with these crude definitions as an expedient, in order that you can
work earlier with the exponential and logarithm functions. Historically, the traditional
definition would be based on theory in [1, §11.9 et seq.], which is not even in the syllabus
of MATH 141. Most mathematicians today prefer to define the logarithm function first,
1
where ln t is defined to be the area under between the graph of y = and the x-axis,
x
the line y = 1 and the line y = t. This type of definition is not available to us yet, as we
haven’t defined areas.
Notes for Lecture Section 002, MATH 140 2008 09 2055

1.5¡Exercises
¢ [1, Exercise
√ 1.5.16, p. 58] “Find the domain of each function: (a) g(t) =
−t t
sin e , (b) h(t) = 1 − 2 .”
Solution:

(a) We first examine the “innermost” function in the composition. The function t 7→
e−t is defined for all real numbers t, i.e., its domain is R. Likewise, the second
function in the composition, the sine function, is also defined for all values of its
argument. Thus the definition of the composite function “makes sense” for all real
numbers: the domain of g is R.

(b) While the first function applied, t 7→ 1 − 2t , is defined for all real numbers t, we
have to confine ourselves to those points in its domain which yield a non-negative
value, in order that the square root may be taken. The inequality 1 − 2t ≥ 0 is
equivalent to 2t ≤ 1, hence to t ≤ log2 1 = 0. Thus the domain of h is the half-line66
(−∞, 0].

5.4.4 §1.6 Inverse Functions and Logarithms


What do we mean by an “inverse” function? We might wish to call a function g
an inverse of a function f if application of g can reverse the action of f ; that is, if

g(f (x)) = x (67)

for all x in the domain of f , which concerns the application of the inverse after the
application of the original function; and if

f (g(y)) = y (68)

for all y in the image of f , which concerns the application of the inverse before the
application of the original function. If we apply the function f to both sides of that
equation, we obtain f (g(f (x)) = f (x) , which says that f (g(y)) = y for all y in the
image of f . If we wish to “reverse” the action of f , we can’t permit a situation where
f maps two distinct points x1 , x2 to the same value y. The “inverse” will need to be
defined for all points in the image of f .
The graph of a function f can be interpreted as describing the action of the function:
given any point a in the R, there is, by the “Vertical Line Test”, at most one point on
the line x = a on the graph; that unique point will exist precisely if x is in the domain
of f . If there exists such a point, say (a, b), then f (a) = b. Thus the graph can be
used to reconstitute the function. Under these conditions the graph can be interpreted
as describing a function that maps points in the image of f , on the y-axis, on to points
66
We sometimes call a half-line a ray.
Notes for Lecture Section 002, MATH 140 2008 09 2056

in the domain of f . The property needed to ensure that the graph can be interpreted
in this way is called by your textbook the “Horizontal Line Test”: any line y = b may
intersect the graph in at most one point:

Horizontal Line Test (preliminary version): A


function has an inverse if and only if no horizontal line
intersects its graph more than once.

To formalize this idea we need the following definition:

Definition 5.3 [1, Definition 1, p. 60] A function f is one-to-one or injective if distinct


points are mapped on to distinct points; symbolically, if

x1 6= x2 implies that f (x1 ) 6= f (x2 ) .

We can now reformulate the Horizontal Line Test:


Horizontal Line Test: A function is one-to-one if and
only if no horizontal line intersects its graph more than
once.
Once a function f is known to be injective, or one-to-one, we know that there exists an
inverse function which acts on the image or range of f and takes its values in the domain
of f . But, even if we have a simple formula for the action of f , this does not mean that
it will be easy or even possible to find a simple formula for f −1 . When it is possible to
find a formula for the inverse, this can be done by following the boxed instructions on
[1, p. 62]:

STEP 1. Write y = f (x).

STEP 2. Solve the equation y = f (x) for x in terms of y. This


gives a formula expressing y in terms of x, i.e., a formula for
x = f −1 (y). This determines the function f −1 in a “non-
standard” way, in that the action of the function is expressed
in terms of a variable named y.

STEP 3. If you wish to express your formula using the symbol x


for the independent variable, simply replace the symbol y by
x throughout your solution.

Instead of writing the words implies that we will often use the symbol ⇒. To be precise,
we should explain that the implication must be true whenever x1 and x2 are any points
of the domain of the function f . We can write this using mathematical logic symbols as
Notes for Lecture Section 002, MATH 140 2008 09 2057

∀x1 ∀x2 , or (∀x1 )(∀x2 ); ∀ is called the “universal quantifier”, because it tells you that a
statement must be true for all points in the “universe”, which, in this case, is understood
to be the domain of f . You won’t be expected to use either of these symbols, but you
may see them on the chalkboard or in the notes.
There is an equivalent way of defining one-to-one or injective which is often easier to
work with:

Definition 5.4 A function f is one-to-one or injective 67 if

f (x1 ) = f (x2 ) ⇒ x1 = x2

or, more precisely,


∀x1 ∀x2 (f (x1 ) = f (x2 ) ⇒ x1 = x2 ) (69)

Logicians call
f (x1 ) = f (x2 ) ⇒ x1 = x2
the contrapositive of
x1 6= x2 ⇒ f (x1 ) 6= f (x2 ) :
it is obtained by negating the statements connected by the ⇒ and reversing the direction
of the implication arrow. It can be shown that any implication is true precisely when its
contrapositive is true.
One way of inferring that a function is one-to-one is from the fact that a function
is monotone, i.e., it is always increasing or always decreasing. For, if the function is
increasing or decreasing, the graph can cross a horizontal line no more than once.

Exercise 5.6 Can you construct an example of a function that is one-to-one but is not
monotone?

In the important cases below we will be naming and developing the properties of the
inverse functions of some of the important functions that we need to have available to
us.

Theorem 5.9 Suppose that f : A → B has an inverse function f −1 : B → A. Then

1. The domain of f −1 is the image of f .

2. The image of f −1 is the domain of f .


67
The term one-to-one is older than injective, but is still in current use. A function that is not one-
to-one might be described as “many-to-one”. In the current use of the word function we never permit
a function to be “one-to-many”.
Notes for Lecture Section 002, MATH 140 2008 09 2058

3. (Cancellation Equation, cf. [1, p. 62])

f −1 (f (x)) = x for all x ∈ A . (70)

4. (Cancellation Equation, cf. [1, p. 62])

f (f −1 (y)) = y for all y ∈ B (71)

We can write the “Cancellation Equations” more compactly. For any set X, define a
function 1X : X → X (called the identity function) by x 7→ x. Then we may rewrite
equations (70), (71) as
¡ −1 ¢
f ◦ f (x) = 1A (x) for all x ∈ A (72)
¡ −1
¢
f ◦f (y) = 1B (y) for all y ∈ B (73)

or, even more compactly, as

f −1 ◦ f = 1A (74)
f ◦ f −1 = 1B . (75)

(Remember that two functions are equal when they have the same domain and the same
action on every point of that domain.) This pair of equations will always hold when
there exists a “true” inverse to a function, in particular, for logarithms and exponentials.
But, when we come to trigonometric functions, the situation becomes murkier.68

Example 5.10 [7, Exercise 1.6.28, p. 75] “Find a formula for the inverse of the function
x
y = 1 + ex .”
1−e
Solution: Here the “boxed” procedure in the textbook [1, p. 62], given on page 2056 of
these notes, is applicable. Solving the given equation algebraically yields
y−1
ex = .
y+1
We next apply the inverse function of ex to both sides of the equation, to obtain
y−1
ln (ex ) = ln
y+1
y−1
⇒ x = ln
y+1
68
We shall see that all of the 6 basic trigonometric functions fail the horizontal line test, and so none
of them possesses an inverse!
Notes for Lecture Section 002, MATH 140 2008 09 2059

If we wish to express the inverse function in terms of an independent variable named x,


we have to interchange the symbols: the inverse function is y = ln x−1
x+1
or

x−1
x 7→ ln .
x+1
x−1
The domain of the inverse function consists of those x for which the ratio is
x+1
positive, which can be shown to be (−∞, −1) ∪ (1, +∞), or R − [−1, 1], or |x| > 1 (short
for {x : |x| > 1}).
Notes for Lecture Section 002, MATH 140 2008 09 2060

5.5 Supplementary Notes for the Lecture of September 17th,


2008
Release Date: Wednesday, September 17th, 2008

5.5.1 §1.6 Inverse Functions and Logarithms (conclusion)


Logarithmic Functions Let a be a positive fixed real number (=positive constant).
Once we have proved that the function x 7→ ax has an inverse, we assign to the inverse
function the name loga . We can then apply the preceding general theory of inverses to
the pair of functions ax , loga x. Equations (70) and (71) become
loga (ax ) = x (76)
alog y = y (77)
where the first equation is valid over the domain of the function ax , i.e., over the entire
line R; and the second equation is valid over the domain of loga , i.e., over (0, ∞). We
can solve equations by applying either exponentiation or the taking of the logarithm (to
any positive base) to both sides of the equation.
The basic property of the logarithm is a consequence of its being the inverse of the
exponential:69
loga (xy) = loga x + loga y . (78)
This property can be shown to imply the other properties stated in the textbook:
µ ¶
x
loga = loga x − loga y (79)
y
loga (xr ) = r loga x (80)
first for integers r; then for rational numbers r; then for any real number r; these
proofs will not be given in the course, but you should remember the facts, and use them
whenever necessary.

Natural Logarithms Here we specialize the constant a of the previous item to a = e;


instead of writing loge , the custom of calculus textbooks is to denote the function by ln.
Mathematicians often persist in writing loge , or more likely just log, where the base of
the logarithms is understood from the context70 . The “change of base formula”
loga x loga x ln x
loga x = = =
1 loga a ln a
69
One can prove that this property is a consequence of properties (76), (77); I do not expect you to
be able to supply such a proof.
70
In some situations mathematicians write log to denote a logarithm to a base other than e; you will
not likely encounter such situations in your calculus courses.
Notes for Lecture Section 002, MATH 140 2008 09 2061

or, more generally,


loga x logb x
= (81)
loga y logb y
for any positive real numbers a, b, x, y, permits statements in terms of logs to one base
to be converted to another base.
Example 5.11 Use the “change of base formula” to express

log6 10 + log6 20 − 3 log6 2

in terms of logarithms to base 7 (cf. [7, Exercise 1.6.37(b), p. 76]). Simplify your answer.
Solution:
µ ¶
10 × 20
log6 10 + log6 20 − 3 log6 2 = log6
2×2×2
= log6 25
log6 25
=
log6 6
logb 25
= to any base b
logb 6
log7 25
= to the required base 7
log7 6

Inverse Trigonometric Functions Since all of them are periodic (with periods either
2π or π), none of the six trigonometric functions is one-to-one (injective), so none of
them has an inverse! What we will be calling the inverses of trigonometric functions will
actually be inverses of restrictions of each of these functions to a smaller subdomain. In
the case of sin, cos, tan, cot, there is a “natural” choice for this restricted domain; these
are not the only possible choices, but they are the ones that are made in practice around
the world at the present time.
The functions sec and csc also fail to be invertible, since their graphs cross some
horizontal lines more than once; i.e. they violate the “Horizontal Line Test” [1, p. 60].
And, as with the other four trigonometric functions, we can find an inverse for a restric-
tion of either function to a smaller domain. Unlike the situations for the functions sin,
cos, tan, cot, the author of the textbook can argue that there is no “natural” choice
for which restricted domains one should use. Some authors select as the domains the
same domains they used for the reciprocals of these functions, except that they delete
the points where ©cosª and sin were zero. Such£ a ¤selection would give us the restricted
domains [0, π] − π2 for the cosine, and − π2 , π2 − {0} for the sine; within these re-
spective domains the functions are one-to-one, and can be inverted. The author of your
Notes for Lecture Section 002, MATH 140 2008 09 2062

textbook finds these selections to be problematical. One reason is that later, when we
wish to determine the derivatives of the inverse functions, these choices would lead to
slightly unpleasant formulæ which involve the absolute value function. Possibly for this
reason, the author selects a different part of the domain of the cos and sin functions.
The resulting functions have graphs consisting of two branches which are separated by an
empty horizontal band; the possible choices mentioned earlier would also have produced
graphs with two branches, but the graphs would have been separated only by an empty
horizontal line of 0 height. Since we have chosen to use the Stewart textbook, we shall
adopt the Stewart choice.71 For trigonometric functions, (and later for hyperbolic func-
tions) the inverse functions are denoted either with a superscripted −1, or by prefixing
the function name with the particle arc; thus the inverse function associated with the
portion of the sine function defined for the variable between − π2 and + π2 is often denoted
by arcsin, and called the arcsine function.
The discussion of inverse trigonometric functions in [1, §1.6] will not be completed
until we reach [1, §3.5]. At that time we will explain further why the author of the
textbook has chosen to define the inverse secant and inverse cosecant functions in a
way that appears to be non-intuitive. Because these two definitions are difficult, I may
minimize using the two functions in question, except to illustrate the essential conditions
needed for an inverse function to exist.
The basic fact remains that because all six trigonometric functions have graphs
which fail to satisfy the Horizontal Line Rule, none of them has an inverse!
We cope with this difficulty in every case by restricting ourselves to a subset of the
domain of the original trigonometric function. There are infinitely many ways in which
such a restriction could be carried out. In the case of the sine, cosine, tangent, and
cotangent we select a subdomain which one could argue is “natural”. Your textbook’s
selected subdomain for the secant and cosecant is “unusual”, and I shall discuss the
reasons for the author’s choice later in the term. In the meantime I will not expect you
to be able to work with inverse secants and cosecants. Here are the restricted domains
that we select for inversion of the sine, cosine, tangent, and cotangent:
71
Other choices would have been possible, even for the inverses of the other trigonometric functions.
For example, we could have chosen, for sin, to invert the portion of the graph of sin that is above the
following union of intervals:
h π π ´ µ 5π 3π ¶ µ 11π 13π ¸
, ∪ , ∪ , .
4 2 4 2 4 4
An inverse sine function determined by the restriction of the sine function to this union of intervals
would be just as useful as the function we usually denote by sin−1 or arcsin. The computations would,
however, be hideous and nonintuitive.
Notes for Lecture Section 002, MATH 140 2008 09 2063

function restricted domain for inversion


sin x − π2 ≤ x ≤ + π2
cos x 0 ≤ x ≤ +π
tan x − π2 < x < + π2
cot x 0<x<π
The intervals shown in this table are also the images or “ranges” of the 4 respective
inverse functions. I tabulate domains and ranges of the inverse functions, using the
“arc-” names for these functions, instead of the “function−1 ” names:

function domain image=range


y = arcsin x −1 ≤ x ≤ 1 − π2 ≤ y ≤ + π2
y = arccos x −1 ≤ x ≤ 1 0 ≤ y ≤ +π
y = arctan x −∞ < x < +∞ − π2 < y < + π2
y = arccot x −∞ < x < +∞ 0<y<π
(Note that in this last chart I have called the independent variable x and the dependent
variable y.) You should verify that the “cancellation equations”:

sin(arcsin x) = x for −1 ≤ x ≤ 1
cos(arccos x) = x for −1 ≤ x ≤ 1
tan(arctan x) = x for −∞ < x < ∞
cot(arccot x) = x for −∞ < x < ∞
arcsin(sin x) = x for − π2 ≤ x ≤ π2
arccos(cos x) = x for 0≤x≤π
arctan(tan x) = x for − π2 < x < π2
arccot(cot x) = x for 0<x<π

In the cases of the first four lines of the preceding table, the compositions have the
desired property wherever they “make sense”. But, in the case of the last four lines, the
compositions “make sense” for real numbers x outside of the designated intervals. Then
the compositions do not have the stated properties!
You need to try to understand how the inverse trigonometric functions are con-
structed. The construction always involves selecting a portion of the latter and reversing
the roles of the x- and the y-axes. We will have frequent occasions to need to remember
the images of the inverse functions in the course of other calculations: typically what
will be involved will be the choice of signs in some square root calculation.

Example 5.12 Construction of the Inverse Sine Function, arcsinx or sin−1 x.


Notes for Lecture Section 002, MATH 140 2008 09 2064

1. Start with the sine function, (cf. Figure 2, page 2041).

2. Select a (maximal) portion of the domain for which the Horizontal Line Test is
satisfied; usually we select the portion of the domain − π2 ≤ x ≤ + π2 (cf. Figure 8).
The graph of the inverse function, y = arcsin x = sin−1 x (cf. Figure 9, page 2065)

1
-10 -5 -10 5 10

Figure 8: Invertible restriction of the Function sin x

can be obtained reflecting the selected portion of the graph of the sine function in
the line y = x.

[1, Exercise 1.6.66, p. 72] Simplify the expression tan(arcsin x).


Solution:

Textbook SOLUTION 1: Let y = arcsin x. The first step is to try to express


the tangent of y in terms of its sine:
sin y
tan y = .
cos y
We know an identity that relates sines and cosines:

sin2 y + cos2 y = 1 ,

which could be solved to yield


¡ ¢1 ¡ ¢1
cos y = ± 1 − sin2 y 2 = ± 1 − x2 2 .

But which sign is correct? Are they both right? The answer is that they are
not both right! To resolve this dilemma we have to remember how the inverse
sine function was defined; and the resolution depends on the specific choices
that we made when we constructed the inverse function. We decided to invert
the portion of the sine function defined for a variable between − π2 and + π2
inclusive. So we know that y is in either the fourth or the first quadrant; and
Notes for Lecture Section 002, MATH 140 2008 09 2065

0
-1 -0.5 0 0.5 1

-1

Figure 9: Graph of the Inverse Sine Function

the cosine function is positive in these quadrants. Thus the correct sign here
is +. Now we can complete the calculations:

tan(arcsin x) = tan y
sin y
=
cos y
sin y
= ¡ ¢1
+ 1 − sin2 y 2
sin(arcsin x)
= ¡ ¢1
+ 1 − sin2 (arcsin x) 2
x
= √ .
1 − x2
Notes for Lecture Section 002, MATH 140 2008 09 2066

-1.5 -1 -0.5 0 0.5 1 1.5

-1

Figure 10: Reflection of the restriction of sin x in the line y = x

NOTE: Your proof could be considered incomplete unless you explain clearly
why you choose a particular sign when there was a choice; it may not be
enough to make the right choice without a clear explanation!
Textbook SOLUTION 2: (cf. [1, p. 68]) The textbook gives a second method
that uses a diagram. As usual, I urge you to avoid proofs using diagrams, as
they tend to be incomplete. Here again it is simplest if we begin by defining
y = arcsin x, and conclude that
sin y = x .
The author then constructs a right-angled triangle in which one angle is to
represent y. For its sine to be equal to x he denotes the side opposite y by
Notes for Lecture Section 002, MATH 140 2008 09 2067

x, and the hypotenuse of the triangle as 1. Then, this being a right-angled


triangle, he applies the Theorem√ of Pythagoras to conclude that the length of
the 3rd side of the triangle is 1 − x2 . The tangent of y can then be read off
from the triangle.
What’s wrong with this proof? The essential difficulty is that the proof is
implicitly assuming that y ≥ 0; but the property we proved above holds
for negative y as well — so this proof covers only “half” of the cases. This
objection could be removed by producing another diagram that holds for the
negative cases; this is delicate, since we would have to draw the triangle in a
coordinatized plane.
In practice the diagram method “works”, but only because of the careful
way in which we have decided to invert our functions: it has its place in a
high school, where the main interest is in positive, acute angles. I would
recommend that students in a science course like MATH 140 avoid this type
of solution.

1.6 Exercises

[1, Exercise 1.6.9, p. 70] Determine whether the following function is one-to-one:

f (x) = x2 − 2x .

Solution: Let’s assume that the function takes on the same value at two distinct
points x = a and x = b. Then

f (a) = f (b) ⇔ a2 − 2a = b2 − 2b
⇔ (a − b)(a + b − 2) = 0

I assumed that a 6= b; but the last equation can still be satisfied if a + b = 2. For
example, a = 0, b = 2 are two points where the function takes the same value.
That shows that f is not one-to-one. (This problem could have been approached
geometrically: the graph y = f (x) is a parabola, which could be shown to not
satisfy the Horizontal Line Law. One example is the intersection of the graph with
the x-axis, which consists of two points, (x, y) = (0, 0), and (x, y) = (2, 0).)
³ πx ´
[1, Exercise 1.6.16, p. 70] “Let f (x) = 3 + x2 + tan , where −1 < x < 1.
2
(a) Find f −1 (3).
(b) Find f (f −1 (5)).”
Notes for Lecture Section 002, MATH 140 2008 09 2068

Solution: The first step in solving this problem is to show that there is an inverse.
We cannot do so by following the boxed instructions given earlier and producing
an explicit formula for the inverse, since we have no way of solving the defining
equation for x in terms of f (x). However, we know that, for 0 ≤ x < 1, the function
tan πx
2
is the sum of three functions, one of which is constant, and the other two are
increasing; hence, the sum is increasing, and, for x in that portion of the domain,
the function possesses an inverse. For −1 < x ≤ 0 it is more difficult to prove that
the function is increasing, although we will be able to do this using the methods of
[1, §4.3, p. 287]. Thus this problem is premature, unless the author were to restrict
the domain to 0 ≤ x < 1.

(a) Once we know the inverse exists, we do not need to have an explicit formula,
since we can see by inspection that the specific value of 3 for f is attained.
Evidently f (0) = 3 + 02 + tan 0 = 3. Thus f −1 (3) = 0.
(b) But we can’t use this approach for the second part of the problem, since
inspection does not yield a convenient value for f −1 (5). Here we may apply
the “cancellation equation” (75) to conclude that
¡ ¢
f f −1 (5) = 5 .

Since we can find the “answers” in both cases, does my objection about the proof
of invertibility apply? Without having proved invertibility, we can’t be sure that
the graph of f doesn’t cross the line y = 3 more than once: so the invertibility is
needed to prove the uniqueness of the answer.

[1, Exercise 1.6.22, p. 71] Find a formula for the inverse of the function
4x − 1
f (x) = .
2x + 3

Solution:

1. Denote the “dependent variable” by y:


4x − 1
y= .
2x + 3

2. Solve for x in terms of y: y(2x + 3) = 4x − 1 implies that (2y − 4)x = −1 − 3y;


hence
−1 − 3y
x= .
2y − 4
Notes for Lecture Section 002, MATH 140 2008 09 2069

3. What we have found is the point in the target, i.e., on the x-axis, which is
the image of a point y on the y-axis. We can denote that fact by
−1 − 3y
f −1 (y) = . (82)
2y − 4
4. If we wish to represent the function f −1 as mapping part of the x-axis on to
part of the y-axis, i.e., if we wish to name the “independent” variable x, then
we can simply replace y by x in equation (82):
−1 − 3x
f −1 (x) = . (83)
2x − 4
You can verify your work in the problem by determining the values of (f ◦ f −1 ) (x)
and of (f −1 ◦ f ) (x); both of the composite functions should take the value x,
wherever they are defined .
3
The graph of f is a hyperbola with a vertical asymptote
© 3 ª along x = − 2 , with
horizontal asymptote along y = 2: the domain is R − 2 . The image of f , which
is the domain of f −1 , is R − {2}. These facts are easy to see if you express the
formulæ for the functions in the forms
7
f (x) = 2 −
2x + 3
3 7
f −1 (x) = − −
2 2x − 4

[1, Exercise
³ 1.6.36,
´ p. 71] Find the exact value of each expression: (a) e−2 ln 5 (b)
10
ln ln ee
Solution:
(a)
e−2 ln 5 = e(ln 5)(−2)
¡ ¢−2
= eln 5 by Exponent Law Theorem 5.8.3
= (5)−2 since logarithm and exponential are inverses
1
=
25
(b)
³ ´ ¡ ¢
ln ln e(e ) = ln e10
10
2nd ln and 1st exponential are inverses
= 10 1st ln reverses the action of the last exponential
Notes for Lecture Section 002, MATH 140 2008 09 2070

[1, Exercise 1.6.52, p. 71] Solve each inequality for x: (a) 2 < ln x < 9 (b) e2−3x > 4
Solution:
(a) We know that the exponential to base e is the inverse of the natural logarithm.
That would permit us to solve an equation, by applying the inverse function
to both sides. In this case we have inequalities, not equations. We are actually
using more than just the inverse property — we are using the fact that the
exponential function preserves order ; this will be seen later by the fact that
its graph is always sloping upwards. This enables us to conclude that, when
we apply the exponential to the three members of this pair of inequalities, the
relationship of order is preserved. We obtain
2 < ln x < 9 ⇒ e2 < eln x < e9
⇔ e2 < x < e9
which is the desired solution: the given inequalities are satisfied if and only if
x is in the interval (e2 , e9 ).
(b) In this case we are using the fact that the logarithm function is also “mono-
tone”, and preserves order. We have
¡ ¢
e2−3x > 4 ⇒ ln e2−3x > ln 4
⇔ 2 − 3x > ln 4
⇔ 3x < 2 − ln 4
2 − ln 4
⇔ x<
3
¡ ¢
Here the points which satisfy the inequality are those in the interval −∞, 2−ln3
4
.
[1, Exercise 1.6.54, p. 71] For the function f (x) = ln(2 + ln x), find (a) the domain
of f , and (b) f −1 .
Solution:
(a) This first function applied is given by x → 2 + ln x. This is defined whenever
x > 0. But the second function will map 2 + ln x on to its logarithm. For
that to be well defined we need to have 2 + ln x > 0; we proceed to solve this
inequality:
2 + ln x > 0 ⇔ ln x > −2
⇔ eln x > e−2
⇔ x > e−2 .
Thus the domain is all positive points in the interval (e−2 , +∞); but this
interval consists only of positive numbers, so this is the domain of f .
Notes for Lecture Section 002, MATH 140 2008 09 2071

(b)

y = ln(2 + ln x) ⇔ ey = eln(2+ln x) = 2 + ln x
⇔ ln x = ey − 2
y
⇔ eln x = ee −2
y
⇔ x = ee −2 .
y
It follows that the inverse function to f is given by y 7→ ee −2 . As we usually
describe functions using the symbol x for the independent variable, we can
x
write f −1 (x) = ee −2 .

[1, Exercise 1.6.68, p. 72] Simplify the expression cos (2 tan−1 x).
Solution: cos (2 tan−1 x) = 2 cos2 (tan−1 x) − 1 by identity (49) on page 2039 of
these notes. It will be easier to complete the solution to this problem if we assign
a name u = tan−1 x. Then tan u = x, so sec2 u = 1 + tan2 u = 1 + x2 , and
1
cos2 u = . It follows that
1 + x2
¡ ¢ ¡ ¢ 2 1 − x2
cos 2 tan−1 x = 2 cos2 tan−1 x − 1 = − 1 = .
1 + x2 1 + x2
This equation is valid wherever the composition is defined; since the cosine is
defined everywhere, this is equivalent to requiring that the inverse tangent arctanx
be defined, and that function is defined for all x. If we write the formula in the form
2
− 1, we see that the composition takes values between +1 and -1, although
1 + x2
it attains the value +1 (when x = 0), but never attains the value −1. The graph of
the composition is a bell-shaped curve, with maximum point at (0, 1), and which
approaches from above, but never touches the line x = −1; the composition is an
even function, so the graph is symmetric about the y-axis.

[1, Exercise 1.6.72, p. 72] (This problem is marked by the author as an exercise that
“definitely requires the use of..technology”. But, in fact, no technology at all is
needed. That doesn’t mean that the problem is easy!)

(a) Graph the function f (x) = sin(arcsin x), and explain the appearance of the
graph.
(b) Graph the function g(x) = arcsin(sin x). How do you explain the appearance
of this graph?

Solution: The functions sin and arcsin are mutual inverses.


Notes for Lecture Section 002, MATH 140 2008 09 2072

(a) The function arcsin is defined only on the interval [−1, 1]. The graph of
the composition sin ◦ arcsin will be that portion of the line y = x that lies
above/below this interval on the x-axis, i.e., it is the line segment joining the
point (−1, −1) to the point (1, 1), including the two end-points.
(b) The sine function is defined for all real numbers.
i. Suppose first that − π2 ≤ x ≤ π2 . This is the interval for which the
arcsine function was defined: £ πthe graph ¤is the portion of the line y = x
π
above/below the interval
¡ ¢− 2 ≤¡x ≤ ¢2 on the x-axis, a line segment
joining the points − π2 , − π2 and π2 , π2 .
ii. Next consider x in the interval π2 ≤ x ≤ 3π 2
. By the addition formula for
the sine function,

sin (π − x) = sin π · cos x − cos π · sin x


= 0 · cos x − (−1) sin x = sin x

Since π2 ≤ x ≤ 3π 2
, − π2 < π − x < π2 , so π − x lies in the interval where
the arcsine function reverses the action of the sine. Accordingly it follows
that, for such x,

g(x) = arcsin sin x = arcsin sin(π − x)


= π − x.

Here the graph£ will¤ be the portion of the line y = π−x, lying above/below
the interval π2 , 3π
2
on the x-axis.
iii. The preceding discussions can be generalized. We can show that the graph
of arcsin ◦ sin is a zigzag which crosses the x-axis at the points which are
integer multiples of π, always with slopes ±1, between the horizontal lines
y = ± π2 . No calculator or computer is required.
Notes for Lecture Section 002, MATH 140 2008 09 2073

5.6 Supplementary Notes for the Lecture of September 22th,


2008
Release Date: Monday, September 22nd, 2008

At the end of the last lecture one student observed that I may have reversed the axes
in a rough chalkboard sketch of the inverse cosine function. The correct sketch should
have looked like the following:

2.5

1.5

0.5

0
-1 -0.5 0 0.5 1
x

Figure 11: Graph of the Function arccos x

Textbook Chapter 2. LIMITS AND DERIVATIVES.


The essence of this chapter of the textbook is contained in [1, §2.4]. Because the concepts
in that section are difficult to grasp, and students might not expect to be comfortable
with them until the end of the term or later, that section is not on the syllabus of the
course — even though I will be talking about it. In the preceding section, [1, §2.3], the
abstract definitions of [1, §2.4] are applied to create workable “Laws” for working with
limits; the concept of limits will be seen in applications beyond those mentioned in [1,
§2.1].
Notes for Lecture Section 002, MATH 140 2008 09 2074

5.6.1 §2.1 The Tangent and Velocity Problems.


We will not spend much time on this section, which is present at this point mainly for
motivational purposes. Here, and again in [1, §2.7], the author discusses two applications
which motivate the definition of the derivative, to appear in [1, §2.8]. That definition
will apply the concept of limit, which we shall study in [1, §§2.2-2.4].

The Tangent Problem The determination of the slope of a tangent is, indeed, one
motivation for the development of the calculus. But you should be cautioned that the
suggestion that one can infer the slope from a table of values is unwise, and even objec-
tionable. It is always possible to construct examples where a finite table of values can
give totally misleading information.

The Velocity Problem The textbook discusses two concepts:


distance travelled (or change in position)
average velocity =
time elapsed
and
instantaneous velocity .
It is the latter that we will eventually call, simply, velocity. As for the former, you
are free to use the 2-word name for the concept. But we cannot formally define what
we mean by an average until [1, §6.5, p. 442], which we meet in MATH 141.72 Here
you have an instance where mathematicians have appropriated words from the general
vocabulary, and given them “reasonable” meanings, but where the precise meaning still
requires an explicit definition.

2.1 Exercises Some of the problems are calculator based, and hence excluded from
this course. Others involve calculation of the average velocity, and making inferences
from it about the instantaneous velocity.

5.6.2 §2.2 The Limit of a Function.


The discussion in this section is useful, motivational, and intuitive, but there are no
mathematically valid proofs. However, many terms are defined here, and you should
remember their definitions; those definitions are necessarily “fuzzy”, since they all refer
to the concept of a limit, and that concept is formally defined in [1, §2.4], which we are
omitting from the formal syllabus.
72
This sounds unreasonable — after all, every school child knows what is meant by an average. The
problem is that the concept is not being used for a finite list of numbers, but for a function defined at
an infinite continuum of points over an interval.
Notes for Lecture Section 002, MATH 140 2008 09 2075

One-Sided and Two-Sided Limits; Infinite Limits Based on the ideas in this
section we will justify results in [1, §2.3], which will then be used to evaluate limits. We
will be discussing the meaning of the following statements (where a, L are real numbers):

lim f (x) = L lim f (x) does not exist


x→a x→a
lim f (x) = L lim f (x) does not exist
x→a+ x→a+
lim f (x) = L lim f (x) does not exist
x→a− x→a−
lim f (x) = +∞ lim f (x) = −∞
x→a x→a
lim f (x) = +∞ lim f (x) = −∞
x→a+ x→a+
lim f (x) = +∞ lim f (x) = −∞
x→a− x→a−
The limits that involve a notation x → a+ are called limits from the right; those that
involve a notation x → a− are called limits from the left; the + and − signs are normally
written as superscripts, although some authors will write them in line with the constant
a. The limits without those signs are called two-sided limits (as opposed to the preceding,
which are one-sided limits). For all of these definitions there are equivalent, more verbal
ways of writing; for example

One notation An alternative notation


lim f (x) = L f (x) → L as x → a
x→a
lim f (x) = L f (x) → L as x → a from the right
x→a+
lim f (x) = L f (x) → L as x → a from the left
x→a−
The arrow → is read as “approaches”, and we may write the word instead of the
symbol.
While the formal definitions of the one-sided limits require the theory of [1, §2.4], we
can define
Definition 5.5
Notes for Lecture Section 002, MATH 140 2008 09 2076

1. [1, p. 93] lim f (x) = L if and only if lim f (x) = L and


x→a x→a+
lim f (x) = L .
x→a−

2. lim f (x) = +∞ if and only if lim+ f (x) = +∞ and lim− f (x) = +∞ .


x→a x→a x→a

3. lim f (x) = −∞ if and only if lim+ f (x) = −∞ and lim− f (x) = −∞ .


x→a x→a x→a

It looks as though the 2nd and 3rd parts of these definitions can be obtained from the
first simply by substituting ±∞ for L, but remember that ∞ and −∞ are not real
numbers; our definitions of these concepts are “reasonable” in that the statements look
plausible, but can’t be obtained by such substitutions. Your textbook presents these
statements not as definitions, but as theorems. That is because the author thinks of
both the one-sided and two-sided limit concepts as being defined formally in [1, §2.4];
I am, instead, thinking of only the one-sided limits as being defined there, and of the
two-sided limit as being a simple consequence that we can define here in terms of those
one-sided limits. This means there is a bit less work to do if we do choose to work
through the theoretical definitions.

Vertical Asymptotes to a Graph of a Function An asymptote to a graph will be


a curve which is, in some sense to be made more precise “arbitrarily close” to the given
curve. The only asymptotes we will be considering in this course will be lines, and the
only cases we will consider will be where those lines are either horizontal or vertical.
The concept of a horizontal asymptote will be considered in [1, S2.6]. In this section we
define what is meant by a vertical asymptote:

Definition 5.6 Let a be a real number, and f a given function. We say that

“The line x = a is a vertical asymptote of the graph of the function f ”,

if at least one of the following 6 statements is true:

lim f (x) = +∞ lim f (x) = +∞ lim f (x) = +∞


x→a− x→a+ x→a

lim f (x) = −∞ lim f (x) = −∞ lim f (x) = −∞


x→a− x→a+ x→a

(Of course, this definition is redundant, in that the 3rd alternative in each row need
not be included in each of these cases, since its truth implies the truth of the first two
alternatives.)

Since these statements all involve limits, none of them is concerned with the value of f
at x = a, and it may happen that f is not even defined there.
Notes for Lecture Section 002, MATH 140 2008 09 2077

Tabulation of Function Values, and the Determination of Limits. In a number


of examples the textbook produces tables of function values. These tables are intended
to suggest that the function in question has a particular limit. But students should not
infer that we can evaluate lim f (x) by computing and tabulating values of f near a, or
x→a
by using a graphing calculator! The tables might be helpful if we wish to guess the value
of the limit, but are of no use if we wish to prove that our guess is correct. I reiterate:
A table of values of a function has no place in a proof that the limit of the function is
equal to a particular number as the independent variable approaches a particular value.

Limits “at ±∞” There is another way in which we will want to generalize the limit
concept to “infinite values”; in that case we will want to permit consider limits as x → ∞
and as x → −∞. This concept will be introduced in [1, §2.6]. We will also wish to
consider situations where such limits “have infinite values”.

2.2 Exercises Many of the problems in this set of exercises suggest the use of argu-
ments which would not be acceptable once we have studied [1, §§2.3 – 2.5]. We will, in
connection with [1, §2.5], discuss a theorem which will permit us to determine lim f (x)
x→a
for many large families of functions by simply substituting the value a for the variable
x; once that result has been discussed, you will be in a position to evaluate many of the
limits given in these exercises; of course, a formal proof would require the use of the
omitted [1, §2.4].
In the following problems I will usually be applying the “Laws” that we will be
developing in the coming section. I would have preferred to wait until those Laws had
been discussed thoroughly, but will discuss some of the applications here ”under protest”.
x+2
[1, Exercise 26, p. 98] Determine the infinite limit lim − .
x→−3 x + 3

Solution: As x → −3 from the left, the denominator, x + 3, approaches 0, from


the left, i.e., remaining negative; the numerator approaches −3 + 2 = −1, which
is negative. The quotient is always positive (negative ÷ negative), but becomes
arbitrarily large in magnitude. Thus the limit will be +∞. When we study [1, §2.3]
we will justify this reasoning as an application of the Quotient Law , generalized to
the case where the limit is infinite.
x2 − 2x
[1, Exercise 32, p. 98] Determine the infinite limit lim− .
x→2 x2 − 4x + 4
Solution: Both the numerator and the denominator approach 0 as x → 2 from the
left. We can’t give a meaning to a fraction 00 . Fortunately we are not interested in
the ratio of the limits — it is the limit of the ratio that interests us. So long as x
Notes for Lecture Section 002, MATH 140 2008 09 2078

is different from 2,
x2 − 2x x(x − 2) x
2
= 2
= ,
x − 4x + 4 (x − 2) x−2
x
so we may work instead with the simplified ratio . In this ratio the numerator
x−2
approaches 2, while the denominator approaches 0 from the left — i.e., always
remaining negative. When x is “close enough to 2, i.e., within 2 units of 2, the
ratio will be negative. And it will grow in magnitude as x approaches 2; since x
is approaching 2 from the left, the ratio will approach −∞. This will be seen as a
nother application of the Quotient Law , generalized to infinite limits.

[1, Exercise 34(a), p. 98] (not discussed in the lecture) Find the vertical asymptotes
x2 + 1
of the function y = .
3x − 2x2
Solution:

1. By definition, the vertical asymptotes occur at those real numbers a where the
function has a one-sided or 2-sided infinite limit. In this fraction the numerator
will, as x → a, where a is any real number, approach a2 +1. And the ratio will
approach a finite limit provided the denominator also approaches a non-zero
limit. As we shall see, that will occur at every real number a different from
the 2 values that make the denominator equal to 0, i.e., a = 0 and a = 32 . The
only candidates for vertical asymptotes are, therefore x = 0, 32 .
2. As x → 0− the denominator approaches 0 because it is the product of x,
which approaches 0 through negative numbers, and 3 − 2x, which eventu-
ally approaches 3 through positive numbers; thus the denominator is even-
tually negative, while the numerator is always positive. We can say that
x2 + 1
lim− = −∞. This one-sided infinite limit as x → 0− is sufficient
x→0 x(3 − 2x)
x2 + 1
for us to declare that x = 0 is a vertical asymptote of y = . How-
3x − 2x2
ever, we could equally well base our argument on the behavior of the ratio as
x → 0+ ; in that case the one-sided limit would be +∞. Either of these facts
justifies our saying that x = 0 is a vertical asymptote.
¡ ¢− ¡ ¢+
3. As x → 23 , the ratio approaches +∞; as x → 32 , the ratio approaches
−∞; either of these facts justifies the conclusion that x = 32 is a vertical
asymptote to the graph.
4. Since the denominator changes sign at each of x = 0 and x = 32 , we can’t
justify a conclusion about vertical asymptotes from the value of the (two-
Notes for Lecture Section 002, MATH 140 2008 09 2079

sided) limit of the fraction — since it does not possess a (2-sided) limit at
either x = 0, x = 32 .

1
Example 5.13 Find the vertical asymptotes of the functions tan x, arctan x, |x|, tan ,
x
x2 − 2
.
x2 − x − 2
tan x: The only places where the tangent function is unbounded are the odd integer
multiples of π2 . From the left of such a point the function approaches +∞; from
the right of an odd multiple of π2 the tangent approaches −∞. For either of these
reasons the vertical asymptotes to the graph of tan x are all lines x = (2m+1)π 2
,
where m is any integer. As we shall see, the function has (finite) limits at all other
points.

arctan x: The arctangent function never approaches ±infinity. Its graph has no vertical
asymptotes. (We shall see later that this graph has horizontal asymptotes at
y = ± π2 .)

|x|: There is no real number a such that lim+ |x| = ±∞ or lim− |x| = ±∞. Thus the
x→a x→a
graph of |x| has no vertical asymptotes. (We will eventually say that lim x → ±∞|x| =
∞. However, this is not a limit at a real number — it is a statement about the
growth of a function as the independent variable becomes arbitrarily large. Al-
though this statement looks like the statements that are equivalent to the existence
of a vertical asymptote, that similarity of appearance is contrived by our decision
when we extended the terminology beyond the (finite) real numbers.)
1
tan : (This example is more difficult than the others.) Since the tangent function
x
approaches infinity on the sides of the odd multiples of π2 , this function approaches
2
infinity on the sides of the reciprocals of those numbers, i.e., at points .
(2m + 1)π
There will be infinitely many vertical asymptotes between x = ± π2 .

x2 − 2
: (This problem is related to [1, Exercise 17, p. 98]; however that exercise
x2 − x − 2
asked students to use a calculator, which I will not request in this course.) Let’s
factorize the denominator: x2 − x − 2 = (x − 2)(x + 1); thus the only candidates
for vertical asymptotes are the lines x = 2, x = −1, as the ratio cannot approach
infinity at any other real numbers, even from only one side. However, if we factorize
the numerator also, as x2 − 2x = x(x − 2), we see that the factor x − 2 is common
x
to numerator and denominator: the function is equal to when x 6= 2, and is
x+1
Notes for Lecture Section 002, MATH 140 2008 09 2080

not defined when x = 2 (since we have given no meaning to the ratio 00 ). The fact
that it is not defined at x = 2 does not affect its behavior “in the limit” as x → 2−
or x → 2+ . I will explain in the coming sections that these limits are both equal to
2
3
; since the function has a limit at these values, the lines x = ±2 are not vertical
asymptotes. However, as x → −1− , the function becomes negatively infinite, which
x
we write as lim − = −∞, and we say that x = −1 is a vertical asymptote
x→−1 x + 1
to the graph of the function. We could have drawn this conclusion also from the
x
fact that lim + = ∞.
x→−1 x + 1

Wednesday’s Lecture Ends at 17:45

On Wednesday the Adams Auditorium must be prepared for the 6


p.m. “Science Education Lecture”, to be given by Nobel Prize winner
Carl E. Wieman; I ask you to arrive for the MATH 140 lecture to
begin promptly at 16:35, and will stop lecturing at 17:45 to permit
the A/V technicians to prepare the room for Dr. Wieman’s lecture.
(http://www.mcgill.ca/channels/events/item/?item id=101504).
Please refer to the WebCT site for this course in case there are other
announcements about Wednesday’s class.
Notes for Lecture Section 002, MATH 140 2008 09 2081

5.7 Supplementary Notes for the Lecture of September 24th,


2008
Release Date: Wednesday, September 24th, 2008
(subject to correction)

5.7.1 §2.4 The Precise Definition of a Limit (OMIT)


While this section is not examination material, it underlies all of the theory of the course;
I will be discussing some of the concepts at the beginning of this lecture, and you should
try to understand the spirit of the definition, without the pressure of fearing that you will
be examined on it. While I will not include the formal definitions in these notes, one fact
that should be remembered is that the existence or non-existence of a limit as x → a, do
not depend on the value of the function at x = a.73 The spirit of the definitions is that
the limit of a function “at” a point — i.e., as one approaches that point (possibly only
from one side) — is the value that could be inferred from the behavior of the function
“near” the point. But these English words need to be replaced by precise, mathematical
statements, which will involve distances measured horizontally and vertically.

The order of doing mathematics For thousands of years mathematics has followed
a logical order:

• First we define the terms that are to be used.

• Then we state the results we plan to prove.

• Then we prove what we claim.

• Finally, we may apply the new tools that we have forged.

Often there will be interruptions to this sequence where we may experiment with ideas, to
explore new possibilities, and to motivate the subsequent steps. Your textbook frequently
includes such motivational sections; §§2.1 and 2.2 are largely of this type. Where you
see, in these sections, statements that appear to be results you should interpret them
as being advance hints of results that will become available after the formal definitions
are announced and theorems have been proved. That is the nature of, for example, [1,
Theorem 2.2.3, p. 93]; the result is stated again as [1, Theorem 2.3.1, p. 104]; in this
particular case I can dispense with the theorems, as I have made the one-sided limits
my primary concepts, and defined the two-sided limit in terms of them. Some of the
73
But functions f that have the property that they are continuous at x = a will have the property
that the value of the lim f (x) = f (a).
x→a
Notes for Lecture Section 002, MATH 140 2008 09 2082

“definitions” in [1, §2.2] will become perfectly valid once the basic concept of limit is
defined. But [1, Definition 2.2.1, p. 88] is purely motivational, and will be replaced
by [1, Definition 2.4.2, p. 110]. In your first reading of the subject, these motivational
statements help you to understand the purpose of the formal definitions.
The main formal definitions appear in [1, §2.4]. When we discuss [1, §2.3], students
should understand that we are actually assuming [1, §2.4]. Your responsibility will be
to be able to be able to use the results in [1, §2.3] intelligently, without necessarily
understanding how they are proved. Most of our demands on the limit concept can
be satisfied with these results; we will return to it in [1, §2.6], where we generalize
the concept to lim f (x) and lim f (x). In MATH 141 we will have several occasions
x→∞ x→−∞
to return to the definition for further generalizations; but you will not be expected to
work extensively with the formal definitions in either of these courses: we are spending
some time on the definition because we believe that, without exposure to it, you cannot
understand how the calculus “works”. A logical order for reading these sections is
• Sections 2.1, 2.2 for motivation
• Section 2.4 — the formal definitions
• Section 2.3 — the basic theorems about limits
• Section 2.5, etc.

A game between x and y In the last lecture I briefly discussed the formal definition
of the statement lim f (x) = L, where a and L are real numbers [1, Definition 2.4.2, p.
x→a
110]. One way of understanding this definition is to think of a 2-player game. The game
is played with a function f . Player A chooses a real number, a, and player L responds
with a choice of a real number L, the number that he suspects to be the “limit” of f
as x approaches a. Play proceeds as follows: A prescribes a tolerance, representing half
of the width of a horizontal band that is centred at the line y = L; this half-width is
traditionally denoted by the lower case Greek letter epsilon, ² (also written as ε). Then
L has to produce a positive real number, traditionally denoted by the lower case Greek
letter delta, δ, with the property that all the points in the interval a − δ < x < a + δ
(except possibly the point a) must have function value within a distance of ² from L —
equivalently, that the graph of the function for the points in the given punctured interval
surrounding x = a lies entirely in the vertical band of width 2² centred at the line y = L;
symbolically, L needs to produce δ such that
0 < |x − a| < δ implies that |f (x) − L| < ² .
Such a game would never end, since there are infinitely many values of ² that can be
chosen. L wins the game if she can produce a procedure or formula that will always
Notes for Lecture Section 002, MATH 140 2008 09 2083

yield a value of δ, for any ² that A tries.74 When L wins we say that lim f (x) exists and
x→a
is equal to L. A wins if she can show that there exists no procedure that can accomplish
the task; then we say that either lim does not exist, or that it exists but is not equal to
x→a
L.
How can A win? She has to show that there is some real number ² such that it is
impossible to find a real number δ with the property described above; that is, a positive
real number ² so that, no matter how close we get to the point a, there are points x for
which f (x) is at least a distance of ² from L. We may discuss briefly examples of this
situation.

Limits from the left and right. The preceding definition is modified in the obvious
way to permit us to speak of “one-sided” limits. We say that lim− f (x) = L if, for every
x→a
positive real number ², there exists a positive real number δ such that

a − δ < x < a implies that |f (x) − L| < ² ;

and that lim+ f (x) = L if, for every positive real number ², there exists a positive real
x→a
number δ such that

a < x < a + δ implies that |f (x) − L| < ² .



 3x + 1 when x < 0
Example 5.14 Let f (x) = 2 when x = 0 .
 2
x when x > 0

1. Find lim− f (x), or show that it does not exist.


x→0

2. Find lim+ f (x), or show that it does not exist.


x→0

3. Find lim f (x), or show that it does not exist.


x→0

Solution:

1. Examination of the function f for x < 0 (using a graph or a table of values) suggests
that, as x moves close to 0, the function value will also be close to 1; this suggests
that the limit from the left is 1 — but this suggestion is not a proof. Given any
74
This overstates the requirement. All L needs is to demonstrate that a value of δ exists — it isn’t
even necessary to show how to find it.
Notes for Lecture Section 002, MATH 140 2008 09 2084

positive real number ² > 0, we find that we can confine the graph in a horizontal
band of width 2² centred at the line y = 1 by taking δ = 13 ²: when
²
− < x < 0,
3
1 − ² < 3x + 1 < 1, so
−² < f (x) − 1 < 0 < ² .
As this argument holds for any positive real number ², we conclude that

lim f (x) = 1 .
x→0−

2. Examination of the function f for x > 0 (using a graph or a table of values) suggests
that, as x gets close to 0, the function value gets close to 0; this suggests that the
limit from the right is 0 — but this suggestion also is not a proof. Given any
positive real number ² > 0, we find that we can confine the √ graph in a horizontal

band of width 2² centred at the line y = 0 by taking δ = ²: when 0 < x < ²,
0 < x2 < ², so
−² < f (x) − 0 < ² .
As this argument holds for any positive real number ², we conclude that

lim f (x) = 0 .
x→0+

3. The limits from the left and right are different. To show that lim f (x) does not
x→0
exist, we can consider the specific value ² = 41 (any positive number less than 12
will do here). To the right of x = 0 the graph of f is close to the x-axis; to the
left of x = 0 the graph is above the line y = 1. No horizontal line y = L is within
1
4
of both of these parts of the graph, so there is no number L which could be the
value of the limit. This argument could be made more precise by giving precise
inequalities. This proof needed the fact that we were working with ² “sufficiently
small”: if we tried to give the same argument when ² = 1, it would not be correct;
but any value less than 14 would also work.
In future we will simply refer to the Definition 5.5 of these notes: a limit exists if
and only if the limits from the left and right exist and are equal.

5.7.2 §2.3 Calculating Limits Using the Limit Laws.


If we were to study [1, §2.4] (whose position is misplaced — the definitions should precede
their application), we would now know precisely what we mean by the limit. But the
Notes for Lecture Section 002, MATH 140 2008 09 2085

calculations of limits using those definitions would appear to be very complicated: is this
what has to be done whenever we need to find a limit or prove that one does not exist?
Fortunately not! Using the theory of [1, §2.4] we could prove some theorems that enable
us to evaluate limits routinely for the types of functions we meet most often in practice.
The author calls these theorems “Limit Laws”. You should remember the results and
be able to use them; you should also be able to identify situations where they cannot
be applied, i.e., where some condition of the theorem is not satisfied. Because we aren’t
studying [1, 4.2], your calculations of limits will usually be based on these laws, and you
could be asked to be very specific about how you are applying these laws, although you
will not be asked to remember the numbers given below for the laws, or any names for
specific laws that I may use in the lectures.

Theorem 5.15 (Limit Laws [1, §2.3]) Let a, c be constant real numbers, n be a
positive integer, f and g be functions such that lim f (x) and lim g(x) exist. Then
x→a x→a

1.

lim c = c for any constant c (84)


x→a
lim x = a (85)
x→a
lim [f (x) ± g(x)] = lim f (x) ± lim g(x) (86)
x→a x→a x→a
lim c · f (x) = c · lim f (x) for any constant c (87)
x→a x→a
lim (f (x) · g(x)) = lim f (x) · lim g(x) (88)
x→a x→a x→a

f (x) lim f (x)


lim = x→a if lim g(x) 6= 0 (89)
x→a g(x) lim g(x) x→a
hx→a in
lim [f (x)]n = lim f (x) for any positive integer n (90)
x→a x→a
lim xn = a n
for any positive integer n (91)
x→a
√ √
lim n x = n a for any positive integer n ;
x→a
if n is even, we assume a > 0; (92)
p q
n
lim f (x) = n lim f (x) for any positive integer n ;
x→a x→a
if n is even, we assume lim f (x) ≥ 0. (93)
x→a

2. All of the preceding equations hold when we are considering only limits from the
left, x → a− or limits from the right, x → a+ .
Notes for Lecture Section 002, MATH 140 2008 09 2086

There are no “standard” names for these laws; I may, when it is convenient, refer to (86)
as the “Sum” Law, to (88) as the “Product” Law, to (89) as the “Quotient” Law, etc.
As for the actual values of limits, we have in the foregoing only (84) and (85). We will
have, as a consequence of [1, Theorem 2.5.7, p. 124] the following

Theorem 5.16 (Direct Substitution Property (generalizing [1, p. 102])) If f is


any one of the following types of functions,

• polynomials

• rational functions

• root functions

• trigonometric functions

• inverse trigonometric functions

• exponential functions

• logarithmic functions

• hyperbolic functions75

and a is a point in its domain, then lim f (x) = f (a).


x→a

Limits of functions related by inequalities. If functions are so related that the


values of the one are never greater than the value of the other, then that property can
be expected to be inherited by the limits, if they exist. More precisely, we have the

Theorem 5.17 If, for all x (except possibly at x = a),

f (x) ≤ g(x) , (94)

then, whenever the limits mentioned below exist, they will have the following properties:

1. lim− f (x) ≤ lim− g(x)


x→a x→a

2. lim+ f (x) ≤ lim+ g(x)


x→a x→a

3. lim f (x) ≤ lim g(x)


x→a x→a

75
not defined yet
Notes for Lecture Section 002, MATH 140 2008 09 2087

The condition “for all x” can be weakened to “for all x ‘near’ a”, where, by near we mean
within a given positive distance from — where the specific value of that positive distance
does not matter, as long as the distance is, indeed, positive. Where the statement refers
to one-sided limits, then the hypothesis refers only to x on that side of a.

Exercise 5.7 If, in inequality (94) of Theorem 5.17, we replace ≤ by a strict inequality,
<, do the conclusions also hold with the same strict inequality?
Solution: NO! Try to find an example of two functions f, g with the property that
f (x) < g(x) for all x 6= a, but where lim f (x) = lim g(x).
x→a x→a

As a “corollary” of Theorem 5.17 we have the following result (whose name is not stan-
dard), where a function is “trapped” between two other functions having the same lim-
iting value; here again the behavior of the functions at the point x = a is irrelevant.

Theorem 5.18 (“Squeeze” theorem) If, for all x “near a” (except possibly at x = a),
f (x) ≤ g(x) ≤ h(x), and if
lim f (x) = L = lim h(x)
x→a x→a

then lim g(x) = L. Analogous results hold for one-sided limits.


x→a

The “greatest integer function” [[x]]. The function that your textbook denotes
by [[x]] was, in the past, often denoted simply by [x]. Mathematicians, and computer
scientists nowadays usually denote this function by the symbol, bxc, and call it the “floor”
function; an analogous function is denoted by dxe, and called the “ceiling” function. The
value is defined to be the largest integer n whose value does not exceed x; for x ≥ 0
[[x]] can be obtained by truncating the decimal expansion of x at the decimal point. The
graph of [[x]] is a rising sequence of horizontal line segments of length 1, each containing
the left end-point but not the right. The function does not have a limit at any integer
point, because the limits from the left and right are different; but, at any point a other
than an integer point, the limit exists and is equal to [[a]].

Limits that “do not exist”. When, for a function f and a point a, there does not
exist a real number L such that lim f (x) = L or lim− f (x) = L or lim+ f (x) = L,
x→a x→a x→a
we say that the respective limit, limit from the left, or limit from the right does not
exist. We use this terminology even if we know that lim f (x) = −∞ or lim f (x) = +∞
x→a x→a
or lim− f (x) = −∞ or lim− f (x) = +∞ or lim+ f (x) = −∞ or lim+ f (x) = +∞, as
x→a x→a x→a x→a
defined informally in [1, §2.2]. Those statements with ±∞ are simply convenient ways
of describing the growth of the function at a, but do not invalidate the nonexistence
statement.
Notes for Lecture Section 002, MATH 140 2008 09 2088

2.3 Exercises

[1, Exercise 10, p. 106] (a) What is wrong with the following equation?

x2 + x − 6
= x + 3? (95)
x−2

(b) In view of part (a), explain why the equation

x2 + x − 6
lim = lim (x + 3)
x→2 x−2 x→2

is correct.
Solution:

(a) There is nothing wrong with equation (95), unless one wishes to interpret it as
implicitly suggesting that the condition is to be true for all x. An equation is
a statement — a sentence. This sentence is, in fact, true for all real numbers
x except x = 2 (since x2 + x − 6 = (x + 3)(x − 2) for all x). But, for the
value x = 2, the left side of the equation is undefined, while the right side
is still meaningful. It’s not that the equation is false for x = 2 — it is that
the statement does not even make sense there, since the left side refers to a
division operation that is forbidden, as we have no meaning for the quotient
0
. This is what the textbook intends by “wrong”.
0
(b) When we consider a limit as x → 2, we expressly avoid any consideration of
the behavior of the function at x = 2. While the function x + 3 is defined at
x = 2, that is completely irrelevant, as we don’t need to consider that value in
finding the limit; the function on the left is undefined at x = 2, but that does
not prevent us from showing that it has a limit as x → 2. Thus the equation
between limits is meaningful. It is also true, since the value of the limit is, in
both cases, 5.
x2 + 5x + 4
[1, Exercise 12, p. 107] Evaluate the limit, if it exists, of lim .
x→−4 x2 + 3x − 4

Solution: Since the limit of the denominator is 0, we cannot apply the Quotient
Law here. But both numerator and denominator factorize:
x2 + 5x + 4 (x + 1)(x + 4)
lim = lim
x→−4 x2 + 3x − 4 x→−4 (x − 1)(x + 4)
x+1
= lim
x→−4 x − 1
Notes for Lecture Section 002, MATH 140 2008 09 2089

since x + 4 is a non-zero common factor away from x = 4


lim (x + 1)
x→−4
=
lim (x − 1)
x→−4
for the Quotient Law is applicable now
−4 + 1 3
= = .
−4 − 1 5

[1, Exercise 22, p. 107] Evaluate, if it exists,



1+h−1
lim .
h→0 h

Solution: Because the limit of the denominator is 0, we cannot apply the Quotient
Law...yet. Instead, we first perform an operation, sometimes called rationalization
which transforms the difference of unpleasant expressions in the numerator into
a sum in the denominator. This will permit a simplification, after which we can
apply the Quotient Law.
The logic is one we will use repeatedly: we will multiply by a factor, thereby
distorting the function, and then divide it out, thereby restoring the function to
its original value.
√ µ√ √ ¶
1+h−1 1+h−1 1+h+1
lim = lim ·√
h→0 h h→0 h 1+h+1
(1 + h) − 1
= lim ¡√ ¢
h→0 h 1+h+1
h
= lim ¡√ ¢
h→0 h 1+h+1
1
= lim ¡√ ¢
h→0 1 1+h+1
dividing numerator and denominator by h,
which we may assume is non-zero
1
= lim √
h→0 1+h+1
lim 1
= √ h→0
lim 1 + h + lim 1
h→0 h→0
by the Quotient and Sum Laws
Notes for Lecture Section 002, MATH 140 2008 09 2090

1
= √
lim 1+h+1
h→0
since we know the limit of a constant
1
= q
lim (1 + h) + 1
h→0
1 1
= √ = .
1+1 2
µ ¶
1 1
[1, Exercise 26, p. 107] Evaluate the limit if it exists: lim − .
t→0 t t2 + t
Solution: This limit cannot be evaluated by the Difference Law, because neither of
the two fractions has a limit as t → 0. However
µ ¶ µ ¶
1 1 1 1
lim − = lim −
t→0 t t2 + t t→0 t t(t + 1)
(t + 1) − 1
= lim
t→0 t(t + 1)
t
= lim
t→0 t(t + 1)
1
= lim
t→0 t + 1
lim 1
t→0
= by the Quotient Law
lim(t + 1)
t→0
1
= by the Direct Substitution Property
0+1
= 1

[1, Exercise 36, p. 107] If 2x ≤ g(x) ≤ x4 − x2 + 2 for all x, evaluate lim g(x).
x→1

Solution: The function 2x is a polynomial; as x → 1, 2x → 2(1) = 2; similarly, as


x4 − x2 + 2 is a polynomial, its limits as x → 1 is 14 − 12 + 2 − 2. The function g
is thus “trapped” between two functions having the same limit as x → 1. By the
“Squeeze Theorem”, lim g(x) must be equal to this common value of the limits of
x→1
the two functions which are above and below, i.e., to 2.
√ π
[1, Exercise 38, p. 107] Prove that lim+ x · esin x = 0.
x→0
Notes for Lecture Section 002, MATH 140 2008 09 2091

Solution: 76 The only tool we have available for a problem like this is the “Squeeze
Theorem”. But that requires two functions “trapping” the given function. The
clue is in the exponent of the exponential, which is a sine. While that exponent
behaves wildly as x → 0+ and x → 0− , it never exceeds 1 in magnitude. Thus we
have
π
−1 ≤ sin ≤ +1
x
which implies that
π
e−1 ≤ esin x ≤ e+1 .

If we multiply all members of these inequalities by the positive quantity x, we
obtain
√ 1 √ π √
x · ≤ x · esin x ≤ x · e .
e
The extreme members of these inequalities are polynomials; as x → 0+ they both
approach the value obtained by substituting x = 0, i.e., the value 0. This “traps”
the function in the middle to also approach 0 as x → 0+ .
µ ¶
1 1
[1, cf. Exercises 43, 44, p. 107] Find lim − if it exists. If the limit does
x→0 x |x|
not exist, explain why.
Solution: The safest policy for students who are intimidated by an absolute value
sign is to rewrite the function without absolute values. We obtain
 

 1 1 2 

 + = if x < 0 


1 1 x x x
− = undefined if x = 0 .
x |x|   1 1 


 − = 0 if x > 0  
x x
The fact that the function is undefined at x = 0 is irrelevant, since we don’t care
about the value, if any, of a function at x = 0 when we are investigating the limit
as x → 0. For x ≥ 0 the function is equal to the constant 0, and its limit from the
right will also be 0, by the Constant Law. But, for < 0, the µ function
¶ is equal to
2 1 1
, which approaches −∞ as x → 0− . While we write lim− − = −∞, we
x x→0 x |x|
still say that the limit from the left does not exist. Since the limit from one side
does not exist, the 2-sided limit also does not exist: the 2-sided limit exists only if
each of the 1-sided limits exist, and they are equal, in which case the 2-sided limit
is the common value of the two 1-sided limits.
76

Don’t be put off by the fact that this involves only a one-sided limit; that’s because√ x has
p no
meaning for x < 0 — the problem could have been stated for two-sided limits by replacing x by |x|.
Notes for Lecture Section 002, MATH 140 2008 09 2092

f (x)
[1, Exercise 56, p. 108] If lim = 5, find the following limits:
x→0 x2

(a) lim f (x)


x→0

f (x)
(b) lim
x→0 x

Solution:

(a)
µ ¶
f (x) 2
lim f (x) = lim ·x
x→0 x→0 x2
f (x)
= lim · lim x2 by the “Product” Rule
x→0 x2 x→0
= 5·0 since x2 is a polynomial
= 0

(b)
µ ¶
f (x) f (x)
lim = lim ·x
x→0 x x→0 x2
f (x)
= lim 2 · lim x by the “Product” Rule
x→0 x x→0
= 5·0 since x is a polynomial
= 0

6−x−2
[1, Exercise 60, p. 108] Evaluate lim √ .
x→2 3−x−1
Solution:
√ µ√ √ √ ¶
6−x−2 6−x−2 6−x+2 3−x+1
lim √ = lim √ ·√ ·√
x→2 3−x−1 x→2 3−x−1 6−x+2 3−x+1
√ √ √
( 6 − x − 2)( 6 − x + 2)( 3 − x + 1)
= lim √ √ √
x→2 ( 3 − x − 1)( 3 − x + 1)( 6 − x + 2)

(6 − x − 4)( 3 − x + 1)
= lim √
x→2 (3 − x − 1)( 6 − x + 2)

(2 − x)( 3 − x + 1)
= lim √
x→2 (2 − x)( 6 − x + 2)
Notes for Lecture Section 002, MATH 140 2008 09 2093


3−x+1
= lim √
x→2 6−x+2
since we can assume x 6= 2

lim ( 3 − x + 1)
= x→2 √
lim ( 6 − x + 2)
x→2
by the “Quotient” Rule

lim 3 − x + lim 1
= x→2 √ x→2
lim 6 − x + lim 2
x→2 x→2
by the “Sum” Rule

3−2+1
= √
6−2+2
substituting in root functions and polynomials
1+1 1
= =
2+2 2
Notes for Lecture Section 002, MATH 140 2008 09 2094

5.8 Supplementary Notes for the Lecture of September 29th,


2008
Release Date: Monday, September 29th, 2008

On Wednesday, October 1st, in place of the regular lecture, we will


be administering a multiple-choice quiz to evaluate your preparation in
trigonometry. This quiz — Q2 — does not count in your term mark,
but the information it provides should help you locate those ares in
which your trigonometry background should be strengthened. The quiz
will be 25 minutes long, but some time will be required for distribution
and collection of materials.

5.8.1 §2.5 Continuity.


Definition of continuity. The definition,

the function f is continuous at the point x = a

looks like a single condition:


lim f (x) = f (a) (96)
x→a

but is better remembered as three conditions:

1. f is defined at a, i.e. a is in the domain of f ;

2. lim f (x) exists as x → a; and

3. lim f (x) = f (a).


x→a

Our limit notation includes all of this information in the single statement (96). Similar
interpretations can be made for the statements

lim f (x) = f (a) (97)


x→a−
lim f (x) = f (a), (98)
x→a+

which we read, respectively as

f is continuous from the left at a


and
f is continuous from the right at a
Notes for Lecture Section 002, MATH 140 2008 09 2095

Classification of Discontinuities. We call a point where a function fails to be con-


tinuous a discontinuity. If f has a discontinuity at x = a, but it is possible to extend the
definition of the function so as to assign a value to the function at the offending point,
or to change a value already assigned in such a way that the new function is continuous
there, then we say that the discontinuity is removable. This can happen in various ways:
* Sometimes a point a is not in the domain only because the inventor of the function
did not choose to define the function there. For example, if we were to define
f1 (x) = 6 for all x different from 1
and say nothing about the value of f1 at x = 1, that discontinuity could be removed
by extending the definition to give the function the value 6 at x = 1. (Technically
we should77 also change the name of the function now, since the extended function,
having a changed domain, is not the same function as before.)
** Sometimes a point a is not in the domain because some operation on the expressions
that were used to define the function may not be defined. For example, if we define
2(x − 1)
f (x) to be , then f is not defined at x = 1. Here the limit as x → 1 does
x−1
exist, but the graph of f has a hole in it over the point x = 1 on the x-axis. We
can remove this discontinuity by assigning to the function the value 2 at x = 1.
*** Sometimes a point a is in the domain, but the inventor has assigned a function
value there that causes the discontinuity; it may be possible to change the function
value at x = a and thereby remove the discontinuity, as in the function
½
2 if x 6= 1
f2 (x) =
7 if x = 1
where we can shift the function value at x = 1 from 7 to 2 and thereby remove the
discontinuity. Here again, the change in the function should remind us to change
its name.
Some discontinuities cannot be removed : there may be no way of repairing the function
at x = a to make it continuous. This can happen, for example
• if lim− and lim+ both exist, but are different — we call this a jump discontinuity;
x→a x→a

• if at least one of lim− or lim+ is ∞ or −∞, i.e., if the line x = a is a vertical


x→a x→a
asymptote to the graph — we call a an infinite discontinuity;
• more generally, if at least one of lim− or lim+ does not exist, since continuity at a
x→a x→a
requires the existence and equality of both one-sided limits.
77
but usually don’t
Notes for Lecture Section 002, MATH 140 2008 09 2096

Continuity on an interval. A function is continuous on an open interval (c, d) if it is


continuous at every point in the interval. We extend this concept to intervals including
either or both of their left and right end-points, i.e. to [c, d), (c, d], [c, d], by requiring
that the appropriate one-sided continuity hold when an end-point is included. So, for
example, f is continuous on [a, b] if it is continuous at every point in (a, b) and if, in
addition, it is continuous from the right at a and from the left at b.
Intuitively, f is continuous on [a, b] if its graph on that interval can be drawn “without
lifting your pencil”.

Consequences of the Limit Laws [1, Theorem 3, p. 122]. Functions that are
continuous at the same point can be added, subtracted, multiplied, and divided (provided
the divisor has non-zero value). There are no surprises here.

Large families of continuous functions. In [1, Theorem 7, p. 124] the author sum-
marizes his comments about various classes of functions, observing that all of the follow-
ing types of functions are continuous wherever they are defined :

• polynomials; more generally,

• rational functions

• “root” functions

• trigonometric functions

• inverse trigonometric functions

• exponential functions

• logarithmic functions

(to be continued)

We can apply [1, Theorem 7, etc., §2.5] to evaluate limits at a point in the domain of a
function. Since continuity at x = a implies that the limit of f (x) as x → a is equal to
the function value f (a), we need only evaluate f at a to find the limit.78

Theorem 5.19 1. If a function f (x) is continuous from the right at x = a, then


lim+ f (x) = f (a).
x→a

78
The fact that the limits of continuous functions can be found by substitution was seen earlier in the
“Direct Substitution Property” stated for polynomials and rational functions in [1, p. 102].
Notes for Lecture Section 002, MATH 140 2008 09 2097

2. If a function f (x) is continuous from the left at x = a, then lim− f (x) = f (a).
x→a

3. If a function f (x) is continuous at x = a, then lim f (x) = f (a).


x→a

Thus the only difficulties in finding limits of these functions will be at points not in their
domains, or as x → ±∞.

Continuity of the Composition of Two Functions The following theorem can be


proved:
Theorem 5.20 ([1, Theorem 2.5.9]) If g is continuous at a, and f is continuous at
g(a), then f ◦g is continuous at a. In other words, “A continuous function of a continuous
function is a continuous function.”
This theorem can be generalized:
Theorem 5.21 ([1, Theorem 2.5.8]) If lim g(x) = b, and f is continuous at b, then
x→a
³ ´
lim (f ◦ g)(x) = f lim g(x) = f (b) .
x→a x→a

Example 5.22 [7, Exercise 42, p. 134] Find the constant(s) c that make g continuous
on (−∞, ∞): ½ 2
x − c2 if x < 4
g(x) =
cx + 20 if x ≥ 4
Solution: First observe that, for x < 4, the function is a polynomial, and we know that
polynomials are continuous everywhere; thus g is continuous for x < 4. Similarly, for
x > 4, the function is again a polynomial, and is again continuous. I have carefully
avoided saying anything about the point x = 4; for all other points the function is a
polynomial in an interval surrounding the point, so the limits from left and right will
exist and be equal. But, for the point x = 4, the description of the polynomial is different
on the two sides of the point, and we cannot use this argument in this simple a form.
To the left of x = 4 the function is a polynomial; as we know that polynomials are
continuous everywhere, we can conclude that the limit of x2 − c2 as x → 4− will be the
value of this polynomial at x = 4, i.e., 42 − c2 . Similarly
lim g(x) = lim+ (cx + 20) = c(4) + 20 .
x→4+ x→4

For g to be continuous at x = 4 we need to know, in particular, that the limit exists.


We have just seen that the limits from the left and right exist, so we need only impose
the condition that these two one-sided limits are equal:
42 − c2 = 4c + 20 ,
Notes for Lecture Section 002, MATH 140 2008 09 2098

which is equivalent to the equation

c2 + 4c + 4 = 0 , (99)

which has the single solution, c = −2. But we are not done yet. For continuity we need
to know that
• the limit exists as x → 4

• the function is defined at x = 4

• the limit is equal to the function value.


Now we return to the definition, and apply information that we haven’t used yet. That
is the specific value of g(4), which is given in the second line of the definition as c(4) +20,
which we now know must be (−2)(4) + 20 = 12. When c = 2 the common value of the
limits from left and right is
42 − c2 = 42 − 4 = 12
which is equal to the function value at the point. Now we may conclude that the function
is continuous everywhere when c = 2, and only for this value of c.

The Intermediate Value Theorem. This is an important result, enabling us to


determine with any desired accuracy the points where graphs of continuous functions
cross a horizontal line, in particular, the x-axis. As stated, the result is an existence
theorem, in that it asserts that a number exists, without telling you how to find it.
However it may be shown in class that one can apply the result to obtain approximations
to the desired numbers. These approximations do not, in theory, require the use of a
calculator; but, as the arithmetic becomes more and more complicated, the algorithm
we will describe is not practical for hand calculation. For that reason, we will not expect
you to carry out extended calculations at quizzes or the final examination.
The proof of this theorem is beyond the course, but you are expected to understand
the statement [1, Theorem 10, p. 126], and how to use it:
Theorem 5.23 (Intermediate Value Theorem) Let f be continuous on the closed
interval [a, b], and let N be any number such that f (a) ≤ N ≤ f (b) or that f (b) ≤ N ≤
f (a). Then there exists a number c in [a, b] such that f (c) = N .

Example 5.24 While the Intermediate Value Theorem is an existence theorem, we can
derive from its statement, a constructive procedure or algorithm to determine where a
continuous √function assumes a particular value. Suppose, for example, we wished to
determine 3 to any desired accuracy. We can consider the equation f (x) = x2 . By
trial and error we observe two values of the function above and below 3: f (0) = 0 < 3,
Notes for Lecture Section 002, MATH 140 2008 09 2099

f (2) = 4 > 3. The theorem tells us that, since x2 is a polynomial — and therefore
continuous — there will be a point in the interval [0, 2] at which f (x) = 3. If we cut
the interval in half, and determine the value of f at the mid-point x = 1, we find that
f (1) = 12 = 1. As this is less than 3, we can now confine our attention to the half-interval
3
[1,¡2],¢ and repeat the operation. We test the function at the £ mid-point,
¤ 2
, and find that
3 9 3
f 2 = 4 < 3: thus we can now work with the interval 2 , 2 . The mid-point of this
3
+2 ¡ ¢ 49 £ ¤
interval is 2 2 = 74 , at which point f 74 = 16 > 3. Now we work with the interval 32 , 74 .
3
+ 47 13
¡ 13 ¢ 169
We evaluate f at the mid-point
£ 7¤ of this interval, 2
2
= 8
: f 8
= 64 < 3, so the next
interval we work with is 13 ,
8 4
. The mid-point is 27
16
, etc. If we carry out this procedure
10 times, we reduce the size of the interval considered by a factor of 210 = 1024 > 1000,
so every 10 applications improve our approximation by 3 decimal places.
Several comments are needed:
• There are other, better ways of determining square roots — this example is pre-
sented only to illustrate a consequence of the Intermediate Value Theorem.
• We can’t use an approach like this to approximate solutions to an equation where
the functions involved are not continuous. For such functions there might not even
be a solution. So, for example, the function [[x]] never takes the value 12 , even
though it does have values 0 = [[0]] and 1 = [[1]] and 0 < 12 < 1. This example
shows that failure of the hypotheses to hold, even at just one point, can cause the
theorem to be inapplicable. (The interval is [0, 2], and the point of discontinuity is
1.)
We will return to this topic in connection with [1, §4.2 The Mean Value Theorem], when
we will refine these investigations to determine maximum numbers of solutions in a given
interval.

2.5 Exercises

1 
if x 6= 1
[1, Exercise 2.5.16, p. 128] Explain why the function f (x) = x−1 is

2 if x = 1
discontinuous at the point x = 1.
Solution: One part of the definition of continuity at x = 1 requires that the limit
of the function exist at x = 1. But lim− f (x) = −∞, and lim+ f (x) = +∞: for
x→1 x→1
either of these reasons the limit does not exist.
½ 2x2 −5x−3
if x 6= 3
[1, Exercise 2.5.20, p. 128] Explain why the function f (x) = x−3
6 if x = 3
is discontinuous at the point x = 3.
Notes for Lecture Section 002, MATH 140 2008 09 2100

Solution:

2x2 − 5x − 3
lim f (x) = lim
x→3 x→3 x−3
where, the value assigned to f (3) is irrelevant
(2x + 1)(x − 3)
= lim = lim (2x + 1) = 2(3) + 1 = 7
x→3 x−3 x→3
6= 6 = f (3).

While the limit does exist, and the function is defined at the point x = 3, the limit
is not equal to the defined function value. It follows that f is not continuous at
x = 3. (This discontinuity is removable: if we were to “repair” the function by
changing the value at x = 3 to 6, we would obtain a continuous function.)

[1, Exercise 2.5.26, p. 129] Explain, by referring to specific theorems, why the func-
tion F (x) = arcsin(x2 − 1) is continuous at every number in its domain; state the
domain.
Solution: The domain consists of all x such that x2 − 1 ∈ [−1, 1]. Solving
√ the
2 2
inequalities
√ −1 ≤ x − 1 ≤ 1 we obtain
√ √ 0 ≤ x ≤ 2, equivalently, − 2 ≤ x ≤
2: the domain is the interval [− 2, 2]. This function is the composition of
a polynomial and an inverse trigonometric function; since both of these types of
functions are continuous [1, Theorem 2.5.7, p. 124] everywhere in its domain, the
composition of such functions is also continuous everywhere [1, Theorem 2.5.9, p.
125].

[1, Exercise 2.5.34, p. 129] “Use continuity to evaluate

x2 − 4
lim arctan .”
x→2 3x2 − 6x

Solution: The function in question may be interpreted as a composition (f ◦ g)(x),


where
x2 − 4
g(x) = 2
3x − 6x
x2 − 4 x+2
and f (x) = arctan x. Since 2
= when x 6= 2,
3x − 6x 3x
x2 − 4
lim g(x) = lim
x→2 x→2 3x2 − 6x
x+2
= lim
x→2 3x
Notes for Lecture Section 002, MATH 140 2008 09 2101

lim (x + 2)
x→2
=
lim 3x
x→2
by the Quotient Law
2+2
=
3·2
by the continuity of polynomials
2
=
3
Then
³ ´
lim (f ◦ g)(x) = f lim g(x)
x→2 x→2
2
by continuity of f at 3
2
= arctan
3
2
The answer need not be reduced further, as 3
is not the tangent of a familiar angle,
and is approximately 0.59 radians.

[1, Exercise 2.5.38, p. 129] Find the numbers at which the function

 x+1 if x≤1
 1
f (x) = if 1<x<3

 √x
x − 3 if x≥3
is discontinuous. At which of these numbers if f continuous from the right, from
the left, or neither?
Solution: This function is defined “piecewise”. The three functions pieced together
are, respectively, a polynomial, a rational function, and the square root of a poly-
nomial. Being either functions “on the list”, or a composition of such functions,
we know that these “pieces” are continuous. Indeed, even at the points where
the functions are “pasted together”, the functions are continuous from the right
or from the left. Those points where the “pasting” occurs are x = 1, 3: they are
the only candidates where the function could be discontinuous. Since the values
assigned to the new function are always inherited from one side or the other, the
function value will always equal one of the one-sided limits at these points. For
continuity we need that both of the limits at the point x = 1 be equal, and that
both of the limits at the point x = 3 be equal. I compute the one-sided limits:

lim (x + 1) = 1 + 1 = 2
x→1−
Notes for Lecture Section 002, MATH 140 2008 09 2102

1
lim+ = 1 6= 2 = lim− f (x)
x→1 1 x→1
1 1
lim =
x→3− x 3
√ √ 1
lim+ x − 3 = 0 = 0 6= = lim− f (x)
x→3 3 x→3
This function is discontinuous at both of the points x = 1, 3. At x = 1 the function
is continuous from the left only, since the limit from the left is equal to the function
value; at x = 3 the function is continuous from the right only, since the limit from
the right is equal to the function value.

[1, Exercise 50, p. 129] “Use the Intermediate Value Theorem to show that there is
a root79 of the given equation in the specified interval: ln x = e−x , on the interval
(1, 2).”
Solution: Because students do not have access to calculators, I will provide80 the
following additional information:

2.7 < e < 2.8 (100)


0.69 < ln 2 < 0.7 . (101)

In order to apply the theorem we need to work with a single function. I will define
f (x) = ln x − e−x ; the same sort of reasoning would work if we took the function
2
to be, for example, − ln x + e−x , 5(ln x − e−x ), (ln x − e−x ) , ln x − e−x + 17.
We calculate
1 1
f (1) = ln 1 − =0− <0
e e
1 1 1
f (2) = ln 2 − 2 > 0.69 − = 0.69 − >0
e (2.7)2 7.29

It is essential that we also observe that f is continuous on the interval [1, 2], since it
is the sum of a logarithm and an exponential function, both known to be continuous
79

Definition 5.7 A root or zero of a polynomial f (x) is a number a such that f (a) = 0; as solution to
an equation f (x) = 0 is a number a such that f (a) = 0. The textbook sometimes uses the word root to
describe what I call a solution, but there is no danger of confusion.

80
You could have determined (101) easily yourself by experimentally squaring numbers of the form
k
1+ 10 .
Notes for Lecture Section 002, MATH 140 2008 09 2103

by our observation on page 2096 of these notes, and since the sum of two continuous
functions is also continuous. Then, as we have proved that

f (1) ≤ 0 ≤ f (2) ,

we may conclude that f assumes all values between f (1) and f (2), in particular,
the value 0, say at a point c such that 1 < c < 2. The proof may now be completed
by observing that f (c) = 0 is equivalent to ln c = e−c .

[1, Exercise 64, p. 130] (a) Show that the absolute value function F (x) = |x| is
continuous everywhere.
Solution: The function |x| is constructed “piecewise” by combining linear
functions which are defined, respectively, on the intervals (−∞, 0), and on
[0, ∞). Linear functions are continuous at every point in their domain. The
only point that needs to be checked is the point 0, where we know only that
the function defined on (−∞, 0) is continuous from the left at x = 0. Since
lim− (−x) = 0, we have lim− |x| = 0; since lim+ x = 0, we have lim+ |x| = 0.
x→0 x→0 x→0 x→0
Thus the limits of |x| from left and right both exist at x = 0, and coincide
with 0, the defined value of |0|; by definition, this means that |x| is continuous
at x = 0 also — so it is continuous over all of R.
(b) Prove that if f is a continuous function on an interval, then so is |f |.
Solution: |f | is the composition of two functions — the function f , followed
by the absolute value function. We know by [1, Theorem 9, p. 125] that the
composition of continuous functions is continuous.
(c) Is the converse of the statement in part (b) also true? In other words, if |f |
is continuous, does it follow that f is continuous? If so, prove it. If not, find
a counterexample.
Solution: The converse of an implication If P then Q is the implication If
Q then P ; here the textbook is asking if, for any function f , the continuity
of g(x) = |f (x)| implies the continuity of f itself. The claim is false, as the
following example will show.
½
2 if x 6= 1 ,
Example 5.25 Define f (x) = The composition function
−2 if x = 1 .
g = |f | is the constant function x 7→ 2, and constant functions are known to
be continuous. But the function f is discontinuous at x = 1, since the limit
at x = 1 is +2, which differs from the function value, which is defined to be
−2.
Notes for Lecture Section 002, MATH 140 2008 09 2104

5.9 Supplementary Notes for the Lecture of October 3rd, 2007


Release Date: Wednesay, October 3rd, 2007
subject to correction

5.9.1 §2.6 Limits at Infinity; Horizontal Asymptotes.


As soon as he has discovered a useful concept and proved a few theorems, a mathemati-
cian tries to generalize it. After defining what is meant by lim f (x) = L, we generalized
x→a
this definition by defining what we mean by lim f (x) = ±∞. The generalization is a
x→a
new definition, but we adopt it because the newly defined “limits” behave similarly to
finite real limits. We can generalize the concept “to infinity” in another way also — by
defining what we mean by lim f (x) = L and lim f (x) = L. When either or both of
x→∞ x→−∞
these limits is equal to L, we say that the line y = L is a horizontal asymptote of the
graph y = f (x). This is intended to say that the vertical distance between the graph and
the line approaches 0 as one moves arbitrarily far to the right or arbitrarily far to the
left on the x-axis. The either or both part of the definition is analogous to our definition
of vertical asymptotes, where we demanded that either a limit from the left or from the
right or both be ±∞.

Can a graph cross its asymptote? Some students have the impression that an
asymptote is a line that is approached but never met. While this will be the case
for vertical asymptotes, because of the “Vertical Line Test”, it is not a restriction for
horizontal asymptotes: a graph can cross its horizontal asymptotes. For example, the
sin x
graph of the function has the x-axis as a horizontal asymptote, and it crosses this
1 + x2
asymptote infinitely often, at every integer multiple of π.

1
Limits of powers. Consider the behavior of the function f (x) = as x becomes
x
large. If we want to ensure that the graph of this function lies in a narrow horizontal
band between the lines y = 0 and y = ² — where ² is any small, positive number — all
1
we need to do is to take x greater than ; what this argument shows, using a definition
²
that has not been stated explicitly, is that
1
lim= 0.
x→∞ x

Similar reasoning can be applied to any positive power of x: if r is any positive real
number,
1
lim r = 0 ,
x→∞ x
Notes for Lecture Section 002, MATH 140 2008 09 2105

or, equivalently,
lim x−r = 0 .
x→∞

(Your textbook restricts these results to rational real numbers r, since the author has
not yet defined what he means by an irrational exponent.)

Infinite Limits at Infinity. We can also combine the two generalizations of the orig-
inal limit definition, and define what we mean by lim f (x) = ±∞, and by lim f (x) =
x→∞ x→−∞
±∞. The precise definitions are given in the subsection called “Precise Definitions” [1,
pp. 138-140] (which is omitted from the syllabus of this course).

Limits of polynomials and reciprocals of polynomials. I will illustrate the general


situation by considering some specific examples.

Example 5.26 In each of the following cases evaluate the limit, or show that it does
not exist:

1. lim (3x5 + 4x − 6)
x→∞

2. lim (3x5 + 4x − 6)
x→−∞

1
3. lim
x→∞ 3x5 + 4x − 6
2
4. lim (3x5 + 4x − 6)
x→−∞

3x5 + 4x − 6
5. lim
x→∞ 4x7 − 2x5 + 3x

3x5 + 4x − 6
6. lim
x→−∞ 4x7 − 2x5 + 3x

4x7 − 2x5 + 3x
7. lim
x→∞ 3x5 + 4x − 6

4x7 − 2x5 + 3x
8. lim
x→∞ 3x7 + 4x3 − 6x2

Solution:
Notes for Lecture Section 002, MATH 140 2008 09 2106

1. One recommended way of dealing with problems of this type is to factor out from
the polynomial the leading power of x. Here
µ ¶
5 5 4 6
3x + 4x − 6 = x 3 + 4 − 5 .
x x
4 6
As x → ∞ or x → −∞, both 4
and 5 approach 0, so
x x
µ ¶
4 6 1 1
lim 3 + 4 − 5 = lim 3 + 4 lim 4 − 6 lim 5
x→∞ x x x→∞ x→∞ x x→∞ x

by the Sum Law


= 3+4×0−6×0=3+0+0=3

We see that as the variable becomes infinite, a polynomial behaves like its leading
term: it is as though we simply disregarded the terms of lower powers. It now
follows that
µ ¶
¡ 5 ¢ 5 4 6
lim 3x + 4x − 6 = lim x · lim 3 + 4 − 5
x→∞ x→∞ x→∞ x x
by a variant of the Product Law
= ∞ · 3 = ∞.

It looks as though we have violated the instructions about ∞ stated earlier. But, in
fact, we can show that the various limit laws continue to hold with infinite limits,
provided the various operations are defined. In the case of multiplication, a Product
Law can be proved that would justify taking a product, even if one “factor” is ∞,
provided the finite factor is not 0: we cannot give a meaning to a product ∞ · 0 or
−∞ · 0. Notwithstanding this comment, we still say that lim (3x5 + 4x − 6) does
x→∞
not exist.

2. A result analogous to the preceding holds as x → −∞:


¡ ¢
lim 3x5 + 4x − 6 = −∞ .
x→−∞

Again, we still say that the limit does not exist!

3. We may apply the Quotient Law: since the denominator approaches ∞, the recip-
rocal of the polynomial approaches 0.

4. = ∞, left to the student.


Notes for Lecture Section 002, MATH 140 2008 09 2107

5. We again factor out leading powers;


3x5 + 4x − 6 x5 (3 + 4x−4 − 6x−5 )
lim = lim
x→∞ 4x7 − 2x5 + 3x x→∞ x7 (4 − 2x−2 + 3x−6 )

x5 3 + 4x−4 − 6x−5
= lim 7 ·
x→∞ x 4 − 2x−2 + 3x−6
x5 3 + 4x−4 − 6x−5
= lim 7 · lim
x→∞ x x→∞ 4 − 2x−2 + 3x−6
1 3 + 4x−4 − 6x−5
= lim 2 · lim
x→∞ x x→∞ 4 − 2x−2 + 3x−6
3
= 0· =0
4

6. Left to student.

7. This time the first factor has “limit” ∞, while the second has limit 34 . The Limit
Law can be shown to applicable, and the limit is ∞.

8. Show that the limit in this case is 34 .

Example 5.27 (cf. [7, Exercise 2.6.38, p. 147]) Find the horizontal and vertical asymp-
5x2 + 4
totes of the graph of 2 .
x −1
Solution: Observe that numerator and denominator are polynomials of the same degree.
First observe that the denominator is the product of x − 1 and x + 1: it will be undefined
at x = 1 and x = −1, and will approach ±∞ as x → −1− , x → −1+ , x → +1− ,
x → +1+ , so each of the lines x = −1 and x = +1 is a vertical asymptote (in each case
for more than one reason, because 2 of these one-sided limits are infinite).
If we divide out from numerator and denominator the “leading” power of x, we obtain
¡ ¢
5x2 + 4 x2 5 + x42
= ¡ ¢
x2 − 1 x2 1 − x12
5 + x42
= for x 6= ±1
1 − x12
5+0
→ = 5 as x → ∞ and as x → −∞ .
1−0
Thus y = 5 is the only horizontal asymptote.

Precise Definitions (OMIT). We are not expecting you to work with these precise
definitions in this course.
Notes for Lecture Section 002, MATH 140 2008 09 2108

2.6 Exercises
2 − 3y 2
[1, Exercise 2.6.18, p. 141] Find lim .
y→∞ 5y 2 + 4y

Solution: Dividing numerator and denominator by the leading power, y 2 , we obtain


¡ ¢
2 −2 lim 2y −2 − 3
2 − 3y 2y − 3 y→∞ 0−3 3
lim = lim = ¡ ¢
−1 = =− .
y→∞ 5y 2 + 4y y→∞ 5 + 4y −1 lim 5 + 4y 5+0 5
y→∞

x+2
[1, Exercise 2.6.22, p. 141] Find lim √ .
x→∞ 9x2 + 1
Solution: We can’t apply the Quotient Law immediately, as it would yield a ratio
of the type ∞∞
, which we can’t evaluate. So we transform the fraction first before
taking the limit:
¡ ¢
x+2 x 1 + x2
lim √ = lim q ¡ ¢
x→∞ 9x2 + 1 x→∞
x2 9 + x12
¡ ¢
x 1 + x2
= lim √ q
x→∞
x2 · 9 + x12
¡ ¢
x 1 + x2
= lim q
x→∞
|x| · 9 + x12
¡ ¢
x 1 + x2
= lim q since x is eventually positive
x→∞
x · 9 + x12
1+ 2
= lim q x .
x→∞
9 + x12

1+0 1
At this point we can apply the Quotient Law, obtaining as our limit √ = .
9+0 3
³ √ ´
[1, Exercise 2.6.26, p. 141] “Find lim x + x2 + 2x .”
x→−∞

Solution: Since the limits of the two summands are, respectively −∞ and +∞, we
cannot apply the Sum Law here; while we extended this and other laws to situations
where limits are infinite, we cannot give a meaning to −∞ + ∞. Accordingly we
Notes for Lecture Section 002, MATH 140 2008 09 2109

modify the function using a technique related to one we saw earlier:


à !
³ √ ´ ³ √ ´ x − √x2 + 2x
lim x + x2 + 2x = lim x + x2 + 2x · √
x→−∞ x→−∞ x − x2 + 2x
x2 − (x2 + 2x)
= lim √
x→−∞ x − x2 + 2x
−2x
= lim √ .
x→−∞ x − x2 + 2x
Note that, superficially, this last function appears “more complicated” than the one
we started with. But there is an important difference: in the earlier function there
was a sum whose value was indeterminate, even if we were prepared to work with
±∞; but, in the last quotient, both numerator and denominator are meaningful,
provided we are still willing to work with ±∞. Unfortunately there is still a snag:
we cannot evaluate the ratio of two functions that are both approaching ±∞.
Fortunately, we can cope with that by dividing numerator and denominator by x:
−2x −2
lim √ = lim √
x→−∞ x − x2 + 2x x→−∞ 1 − 1 x2 + 2x
x
−2
= lim 1

x→−∞ 1 −
−|x|
x2 + 2x
−2
= lim 1

x→−∞ 1 − √
− x2
x2 + 2x
−2
= lim q
x→−∞ x2 +2x
1+ x2
−2
= lim q
x→−∞ 2
1+ 1+ x
−2
= Ã r !
2
lim 1+ 1+
x→−∞ x
by the Quotient Law
−2
= r
2
1 + lim 1+
x→−∞ x
by the Sum Law
Notes for Lecture Section 002, MATH 140 2008 09 2110

−2
= s µ ¶
2
1+ lim 1 +
x→−∞ x

by the continuity of
−2
= √ = −1 .
1+ 1+0
Suppose
√ that we wished to determine the horizontal asymptotes to the graph of
x + x2 + 2x. The preceding shows that the line y = −1 is³one asymptote. ´

To complete the investigation we would need to determine lim x + x2 + 2x .
√ x→+∞
Since ³each of x and ´ x2 + 2x approaches +∞ as x → +∞, we conclude that

lim x + x2 + 2x = +∞ and so there are no horizontal asymptotes other
x→+∞
than the one already found.

[1, Exercise 2.6.28, p. 141] Find lim cos x.


x→∞
Solution: This limit does not exist: as x ranges over any interval of length 2π
the value of the cosine ranges between −1 and +1. For a limit to exist we would
need to show that, for x sufficiently large, the value of the cosine would remain
arbitrarily close to that proposed limit value — but there is no number with that
property, because the function oscillates over an interval of length 1 − (−1) = 2.

[1, Exercise 2.6.34, p. 141] Find lim arctan(x2 − x4 ).


x→∞
Solution: The first problem is that we have a difference of two functions, both of
which approach ∞. This is one of the combinations to which we cannot extend
the algebra of real numbers. If we write x2 − x4 = x2 (1 − x2 ), we can then
express the function as a product of two factors, one approaching +∞, and the
other approaching −∞. Here we can comfortably assert that lim (x2 − x4 ) = −∞,
x→∞
since the product will be negative for x > 1, and will become arbitrarily large in
magnitude. As y → −∞ arctan y → − π2 . Hence
π
lim arctan(x2 − x4 ) = .
x→∞ 2

[1, Exercise 2.6.36, p. 141] Find lim + etan x .


x→( π2 )
¡ ¢+
Solution: As x → π2 , tan x → −∞. Since the exponent of etan x is becoming
arbitrarily large negatively, the exponential itself will approach 0.
Notes for Lecture Section 002, MATH 140 2008 09 2111

[1, Exercise 2.6.48, p. 142] “Find a formula for a function (whose graph) has vertical
asymptotes x = 1 and x = 3 and one horizontal asymptote, at y = 1.”
Solution: There are infinitely many solutions to problems of this type. We can
construct a function by adding together functions having the properties indepen-
1
dently. One function whose graph has a vertical asymptote at x = 1 is ;
x−1
1
similarly, one function whose graph has a vertical asymptote at x = 3 is .
x−3
Adding or subtracting these we obtain a function like
1 1

x−1 x−3
whose graph has vertical asymptotes at precisely the designated places. The graph
of the function we have found has a horizontal asymptote at y = 0. We can shift
this asymptote to y = 1 by adding a constant:

1 1 2 x2 − 4x + 1
1+ − =1− = .
x−1 x−3 (x − 1)(x − 3) (x − 1)(x − 3)

x2
A “simpler” example would be .
(x − 1)(x − 3)

5.9.2 §2.7 Derivatives and Rates of Change.


§2.7 continues the discussion begun in [1, §2.1], where the author proposed two applica-
tions which would require the concept of limit.

Tangents. For a given point P0 = (x0 , y0 ) on the graph of the function f , a secant
through P0 is a line passing through P0 and another point, P1 = (x1 , y1 ), on the graph;
since the points are on the graph, y0 = f (x0 ), y1 = f (x1 ). The slope of such a secant
y1 − y0 f (x1 ) − f (x0 )
is = . As we allow the point P1 to move along the curve towards
x1 − x0 x1 − x0
P0 , the secant 81 becomes a tangent 82 ; its slope is

f (x1 ) − f (x0 )
lim .
x1 →x0 x1 − x0
81
from the Latin, secare, to cut; do not confuse with the trigonometric function secant whose name
has a similar etymology.
82
from the Latin tangere, to touch; do not confuse with the trigonometric function tangent whose
name has a similar etymology.
Notes for Lecture Section 002, MATH 140 2008 09 2112

Note that, in the foregoing, x0 is the abscissa83 of the fixed point, and x1 is the abscissa
of the variable point; I have avoided using the letter x here, as I may wish to write the
equation of a line, and wish to have the usual symbols free for that purpose. When we
reach [1, Chapter 3], we will be developing methods for routine computation of limits of
the preceding type; at present we will have to compute such limits “from first principles”,
without the use of the “Differentiation Rules”. Please remember that, even if you know
the “Rules”, you are expected to be able to carry out computations “from first principles”
when asked to do so.

Velocities. Suppose a particle is moving along a straight line — which we conveniently


coordinatize as the x-axis — so that its position at time t is f (t). Between times t = t0
and t = t1 , the position of the particle has changed by a distance f (t1 ) − f (t0 ); this
is a signed distance. We often use the word displacement to describe the position of a
particle measured relative to a fixed point, which we often take to be the origin of the
coordinate system. The “average distance traversed per unit time between t = t0 and
f (t1 ) − f (t0 )
t = t1 ” is defined to be the quotient , often called the average velocity.84
t1 − t0
If, as we allow t1 to approach t0 , we obtain a limit; we call that limit the velocity or
instantaneous velocity of the moving particle at time t0 . We may denote the velocity at
time t = t0 by v(t), and define

f (t1 ) − f (t0 )
v(t0 ) = lim .
t1 →t0 t1 − t0
If we change our notation, and denote t1 by t0 + h, or by t0 + ∆t (so h or ∆t denote
t1 − t0 , the change in time t from t0 to t1 ), the equation becomes

f (t1 ) − f (t0 ) f (t1 ) − f (t0 )


v(t0 ) = lim or v(t0 ) = lim .
h→0 h ∆t→0 ∆t
Definition 5.8 The speed of a moving body is defined to be the magnitude of its velocity.

Derivatives. In both of the preceding applications we were interested in a limit of the


f (x) − f (a)
form lim . We give a name to this type of limit:
x→a x−a
83
x-coordinate
84
Later, in MATH 141, when we define the concept of average, we will be able to show that, in fact,
this is the average of the velocity function we are about to define. (The problem is that, at this stage,
we haven’t defined what we mean by average of a function over an interval. We do know what we mean
by the average of a finite list of numbers, and our eventual definition will generalize that definition.
But being responsible mathematicians, we know that we can’t use a concept until it has been formally
defined. So, for the present, we can simply work with a concept having a 2-word name, average velocity.)
Notes for Lecture Section 002, MATH 140 2008 09 2113

Definition 5.9 For a given function f , and a point a in its domain, we define
f (x) − f (a)
f 0 (a) = lim
x→a x−a
if the limit exists. When the limit exists, we say that f is differentiable at x = a;
otherwise f fails to be differentiable 85 at x = a.
As observed earlier, we can reformulate this definition, as, for example, a limit as a
variable ∆x approaches 0. The notation, with the symbol for the function bearing a
superscript 0 , suggests an operation on the function which yields another function. This
interpretation will be amplified in the next section.

Rates of Change. In the preceding discussion the symbol ∆t denotes a single variable
with a two-letter name; it is not a product, and you must not pull the two characters
∆ and t apart. Historically, it was intended to denote the “increment” in t. We also
use the “operator” ∆ to describe the change in the function f , writing ∆y or ∆f (t0 ) to
denote f (t1 ) − f (t0 ), so that the instantaneous velocity may also be expressed as
∆f (t0 ) ∆y
v(t0 ) = lim = lim .
∆t→0 ∆t ∆t→0 ∆t

Students may observe, quite correctly, that the meaning of the symbol ∆f is not always
completely unambiguous, even when we write the variable, as ∆f (t0 ): how does one
know what the second value of the argument86 is? The answer is that, in practice, that
information can be inferred from the context.
The author of the textbook again uses the word average in this section to describe
the ratio of the increment of the function to the increment of the independent variable.
We might call the first limit above the limit as h → 0 of the “average value” of f over
the interval [x, x + h] or the “average value” of f over the interval [x + h, x], depending
on whether h is taken to be positive or negative; but, as before, this is simply a 2-word
name for a concept, as we haven’t yet defined what we mean by an “average”.

2.7 Exercises Where a derivative is required, it is expected that students evaluate it


“from first principles”, not by using the Differentiation Rules, which will not be intro-
duced until the next chapter.
[1, Exercise 2.7.8, p. 150] Find an equation of the tangent line to the curve y =
2x
at the point (x, y) = (0, 0).
(x + 1)2
85
or some similar wording
86
Argument is a synonym for variable; the argument of f (t) is t.
Notes for Lecture Section 002, MATH 140 2008 09 2114

³ ´
2x
Solution: The secant to the curve passing through the points (0, 0) and x, (x+1)2

has slope
2x
(x+1)2
−0 2
lim = lim
x→0 x−0 x→0 (x + 1)2

lim 2
= ¡x→0 ¢
lim (x + 1)2
x→0
2
= =2
12
Hence the tangent is the line through (0, 0) with slope 2, i.e., y = 2x.

[1, Exercise 2.7.16, p. 151] “The displacement (in meters) of a particle moving in a
straight line is given by s = s(t) = t2 − 8t + 18, where t is measured in seconds.

(a) Find the average velocity over each time interval:


(i) [3, 4]
(ii) [3.5, 4]
(iii) [4, 5]
(iv) [4, 4.5]
(b) Find the instantaneous velocity when t = 4.

Solution:

(a) Each of the average velocities is defined to be the ratio of the displacement
over the elapsed time.

s(3) = 32 − 8 · 3 + 18 = 3
s(3.5) = (3.5)2 − 8 · (3.5) + 18 = 2.25
s(4) = 42 − 8 · 4 + 18 = 2
s(4.5) = (4.5)2 − 8 · (4.5) + 18 = 2.25
s(5) = 52 − 8 · 5 + 18 = 3

It follows that

s(4) − s(3) = −1
s(4) − s(3.5) = −0.25
s(5) − s(4) = 1
s(4.5) − s(4) = 0.25
Notes for Lecture Section 002, MATH 140 2008 09 2115

and
s(4) − s(3)
= −1
4−3
s(4) − s(3.5) 1
= −
4 − 3.5 2
s(5) − s(4)
= 1
5−4
s(4.5) − s(4) 1
=
4.5 − 4 2
(b)
∆s(t) s(t + ∆t) − s(t)
lim = lim
∆t→0 ∆t ∆t→0 ∆t
((t + ∆t)2 − 8(t + ∆t) + 18) − (t2 − 8t + 18)
= lim
∆t→0 ∆t
2
2∆t · t + (∆t) − 8∆t
= lim
∆t→0 ∆t
= lim (2t + ∆t − 8)
∆t→0
= 2t − 8

for any value of t, in particular for t = 4, in which case the value of the limit
is 0. Thus the particle is instantaneously at rest when t = 4.

[1, Exercise 2.7.30, p. 151] Find f 0 (a) “from first principles” if f (x) = 3x + 1.
Solution:
f (x) − f (a)
f 0 (a) = lim
√ x−a √
x→a

3x + 1 − 3a + 1
= lim
x→a x−a√
µ√ √ √ ¶
3x + 1 − 3a + 1 3x + 1 + 3a + 1
= lim ·√ √
x→a x−a 3x + 1 + 3a + 1
√ √ √ √
( 3x + 1 − 3a + 1)( 3x + 1 + 3a + 1)
= lim √ √
x→a (x − a)( 3x + 1 + 3a + 1)
(3x + 1) − (3a + 1)
= lim √ √
x→a (x − a)( 3x + 1 + 3a + 1)
3(x − a)
= lim √ √
x→a (x − a)( 3x + 1 + 3a + 1)
Notes for Lecture Section 002, MATH 140 2008 09 2116

3
= lim √ √
x→a 3x + 1 + 3a + 1
lim 3
x→a
= √ √
lim ( 3x + 1 + 3a + 1)
x→a
lim 3
x→a
= √ √
lim 3x + 1 + lim 3a + 1
x→a x→a
3 3
= √ √ = √
3a + 1 + 3a + 1 2 3a + 1

[1, Exercise 2.7.38(a), p. 152] A particle moves along a straight line with equation
of motion s = f (t) = t−1 − t, where s is measured in meters, and t in seconds.
Find – from first principles — the velocity and the speed when t = 5.
Solution: The velocity is

f (t) − f (5) (t−1 − t) − (5−1 − 5)


f 0 (5) = lim = lim
t→5 t−5 t→5
à 2 t −! 5
1−t 24
t
+ 5
= lim
t→5 t−5
5 (1 − t2 ) + 24t
= lim
t→5 5t(t − 5)
−(5t + 1)(t − 5)
= lim
t→5 5t(t − 5)
5t + 1 26
= − lim =−
t→5 5t 25
26
whose units are meters/sec. The speed is the magnitude of the velocity, i.e., +
25
meters/sec.

[1, Exercise 2.7.34, p. 151] The following limit represents the derivative of some func-
tan x − 1
tion f at some number a. Find such an f and a: limπ .
x→ 4 x − π4
Solution:
tan x − 1 tan x − tan π4
limπ = lim ,
x→ 4 x − π4 x→ π4 x − π4
which is the derivative of tan x at the point x = π4 . This isn’t the only solution:
Notes for Lecture Section 002, MATH 140 2008 09 2117

another solution would be the derivative of the function tan x + 17, since
¡ ¢
tan x − tan π4 (tan x + 17) − tan π4 + 17
lim = limπ .
x→ π4 x − π4 x→ 4 x − π4

5.9.3 §2.8 The Derivative as a Function.


Other Notations. I have defined, in Definition 5.9 the value of the derivative of a
function f at a point x = a of its domain. Let’s now restate that definition, with the
point of view that the point a may vary over the domain of f . To do that, I would like
to use the symbol x for the variable point. But x is busy at the moment — I have used
it in my earlier definition. So I can change
f (x) − f (a)
f 0 (a) = lim
x→a x−a
into
f (u) − f (a)
f 0 (a) = lim ,
u→a u−a
thereby freeing the symbol x for other uses. The use I want is
f (u) − f (x)
f 0 (x) = lim ,
u→x u−x
for any point x in the domain of f . There has been no significant change to the definition.
There are some notations that are traditional. One change is to take the variable in the
limit to be u − x, and thus to allow this variable to approach x − x, i.e., 0. That
difference is traditionally denoted by ∆x — a 2-letter name for a variable that is called
the increment in x; but it’s a 2-letter name, and you must not attempt to factor out
the ∆ or the x; sometimes the increment of f (x) is denoted by ∆f (x). Since the use of
Greek symbols is confusing to some readers, some authors have been replacing ∆x by h.
So we have
f (u) − f (x)
f 0 (x) = lim = f 0 (x)
u→x u−x
f (x + ∆x) − f (x)
= lim
∆x→0 ∆x
∆f (x)
= lim
∆x→0 ∆x
f (x + h) − f (x)
= lim ,
h→0 h
wherever the limit exists.
Notes for Lecture Section 002, MATH 140 2008 09 2118

A number of different notations are used for the value of the derivative at a; I may,
from time to time, use any¯ one of the following symbols,
· ¸ all of which have the same
df df ¯
meaning: f 0 (a), (a), ¯ , Df (a), Df (x)|x=a , df , Dx f (a). Other notations
dx dx ¯x=a dx x=a
may be presented later. I will call the symbols that have the format of fractions, such as
df
, “Leibniz notation”.
dx
Definition 5.10 If a function is differentiable at every point in an open interval (a, b),
i.e., at all x such that a < x < b, f is said to be differentiable on (a, b).

1
Example 5.28 (cf. [7, Exercises 2.8.17-18, p. 163]) Find f 0 (5) if f (x) = ¡√ ¢3 .
x+4
1 1
√ 3 − √ 3
( x+4) ( 5+4)
f 0 (5) = lim
x→5 x−5
¡√ ¢3
33 − x+4
= lim ¡√ ¢3
x→5 (x − 5) · 33 · x+4

We can rationalize the numerator:


¡√ ¢3 ¡√ ¢3 ¡√ ¢3
33 − x+4 33 − x+4 33 + x+4
lim ¡√ ¢3 = x→5
lim ¡√ ¢3 · ¡√ ¢3
x→5 (x − 5) · 33 · x+4 (x − 5) · 33 · x+4 33 + x+4
272 − (x + 4)3
= lim
x→5
¡√ ¢3 ³ ¡√ ¢3 ´
(x − 5) · 33 · x + 4 · 33 + x+4
1
= lim ³ ¡√ ¢3 ´ ¡√ ¢3
x→5
27 27 + x+4 x+4
272 − (x + 4)3
× lim
x→5 x−5
1
by the Product Law, in which the first limit is . For the second limit we see that
2 · 273
the numerator is a polynomial which has value 0 when x = 5, which tells us that x − 5
must divide the polynomial. By long division or otherwise we can see that

272 − (x + 4)3 = −x3 − 12x2 − 48x + 665


¡ ¢
= (x − 5) −x2 − 17x − 133 .
Notes for Lecture Section 002, MATH 140 2008 09 2119

Thus the second limit factor is


272 − (x + 4)3 ¡ ¢
lim = lim −x2 − 17x − 133 = −25 − 85 − 133 = −243 .
x→5 x−5 x→5

1
We conclude that f 0 (5) = − .
162

Intuitive meaning of differentiability. Intuitively a function f is differentiable at


x = a when the graph of f does not have a “corner” at (x, y) = (a, f (a)).

Differentiability implies Continuity. This result is proved in [1, Theorem 4, pp.


158-159]. You will not be responsible for proving the result, but a very short proof is
possible. First observe that the definition of f 0 (a) involves f (a); that is, we can’t even
write down the quotient whose limit we need unless we know that f (a) exists: this gives
one of the conditions we need for continuity of f at x = a. Thus

lim f (x) = lim ((f (x) − f (a)) + f (a))


x→a x→a
= lim ((f (x) − f (a)) + lim f (a) (by Sum Law)
x→a x→a
µ ¶
f (x) − f (a)
= lim · (x − a) + lim f (a)
x→a x−a x→a

(dividing and multiplying by x − a)


µ ¶
f (x) − f (a)
= lim · lim (x − a) + lim f (a) (by Product Law)
x→a x−a x→a x→a
0
= f (a) · lim (x − a) + lim f (a) (since f is differentiable at a)
x→a x→a
= f 0 (a) · 0 + lim f (a) (as x → a, (x − a) → 0)
x→a
= 0 + lim f (a)
x→a
= 0 + f (a) (limit of a constant, f (a))
= f (a)

Note the author’s warning [1, p. 159] that the converse87 of the preceding theorem is
not true! You should be able to supply examples of a function f and a point a in its
domain such that f is continuous at a, but fails to be differentiable there. Graphs of
the simplest examples are made by piecing together at a point two line segments with
different slopes; for example, the function |x| is of that type. You should be able to
construct other examples, with additional constraints on their construction.
87
The converse of a statement of the form “If A then B” is the statement “If B then A.” Can you
think of other implications which are true in one direction but not in the other?
Notes for Lecture Section 002, MATH 140 2008 09 2120

5.10 Supplementary Notes for the Lecture of October 08th,


2008
Release Date: corrected Friday, October 10th, 2008

The next lecture in this section of the course will be on Wednesday, October 15th, 2008.
Students registered in tutorial sections 003 — 008, whose session for October 13th has
had to be cancelled, should read the following message, also posted on myCourses:

Provisions for the Loss of the Monday Tutorial on Thanksgiving, in Sections 003–008
The University’s plan to replace the classes lost on Thanksgiving Monday by classes on Tuesday, December 02nd does not offer a
practical solution for a tutorial lost early in the academic year in a course with tutorials on 3 other weekdays; the plan that was
published — apparently in error — in the print version of the 2008-09 Undergraduate Calendar, would have considered October 14th
a Monday for timetable purposes; but that plan was not adopted.

Students in the 6 tutorials that have been cancelled for October 13th, 2008, are being invited
to either

1. attend a session of another tutorial section meeting regularly on Wednesday, Thurs-


day, or Friday, October 15th, 16th, or 17th, 2008, provided there is space in the tutorial
classroom; or

2. attend one of the special tutorials that will be offered on two late afternoons/evenings
between October 14th and 17th.
Since some students may be unable to avail themselves of either opportunity, I am summa-
rizing the suggestions given to TA’s for the tutorials of October 14–17:
The 6th tutorial is based on textbook sections 2.7, 2.8 (differentiation) and Review of Chapter 2. Since the Differentiation Rules are
not introduced until Chapter 3, all differentiation here should be from first principles, by evaluating limits.

1. Do a problem like 50(a,b,c) on page 165, finding — FROM FIRST PRINCIPLES — the derivative of a fractional power
of x. For the problem in the textbook the method would be to factorize the ratio (x2/3 − a2/3 )/(x3/3 − a3/3 ) into
(x1/3 − a1/3 ).(x1/3 + a1/3 )/(x1/3 − a1/3 ).(x2/3 + x1/3 a1/3 + a2/3 ) = x1/3 + a1/3 /x2/3 + x1/3 a1/3 + a2/3 where x is
not equal to a, and thus obtain, in the limit, 2a1/3 /3a2/3 , etc. Our students are very poorly prepared in routine alge-
bra, so this review of the use of fractional powers is important for them. While, in most situations, they will be able to use
the differentiation rules, they must be prepared to find derivatives from first principles, and problems of this type have often
appeared on quizzes in the past. (After solving the problem from first principles, you could then verify your work by using
the power rule, since most of them have seen the rules in a high school calculus course.) Before leaving this problem, point
out to students that the y-axis is NOT a vertical asymptote, since the limit of the function is not infinite on one side of that
line. In fact, the curve is tangent to the y-axis.

2. Do a problem like the following generalization of Problem 35, p. 168: For ANY POINT on the curve y = 9 − 2x2 , (not only
(2, 1)), find the slope and an equation for the tangent line. Begin by assuming the general point is (u, 9 − 2u2 ), u ∈ R.

3. After reminding students that |x| is differentiable at all points except x = 0, discuss Problem 53, p. 165.

4. Consider the following review problems: p. 167 ##10, 14, 23. (In the case of #23, after solving with the Squeeze Theorem,
show how to interpret the inequalities geometrically: the graph of f is trapped between the line y = 2x − 1 and the parabola
y = x2 , to which it is tangent at (1, 1).)

Squeeze Theorem applied to a limit “at infinity” [1, Exercise 2.6.53(a), p. 142]
sin x
“Use the Squeeze Theorem to evaluate lim .”
x→∞ x
Solution: This limit should not be confused with the limits that we will be determining
in connection with [1, §3.4].
Notes for Lecture Section 002, MATH 140 2008 09 2121

The sine function never exceeds 1 in absolute value; thus

−1 ≤ sin x ≤ 1 for all x .


1
We multiply these inequalities by obtaining
x
1 sin x 1
− ≤ ≤ for x > 0 . (102)
x x x
Similarly — although the question did not ask about the limit as x → −∞, for x < 0,
we obtain
1 sin x 1
≤ ≤− for x < 0 , (103)
x x x
and the two inequalities can be combined into one:
1 sin x 1
− ≤ ≤ for x 6= 0 . (104)
|x| x |x|

(If we are interested in only one of the limits lim , lim , then only one of the two
x→+∞ x→−∞
equations (102), (103) is needed.) The extreme members of these inequalities approach 0
sin x
as x approaches ∞, trapping the middle member; thus we may conclude that lim =
x→∞ x
0 . This is, as the author suggests, another example of a function whose graph crosses a
horizontal asymptote infinitely often.

5.10.1 §2.8 The Derivative as a Function (continued).


How Can a Function Fail To Be Differentiable? The textbook describes several
situations in which differentiability of a function f can fail at a point a.
f (x)−f (a) f (x)−f (a)
• If lim− x−a
and lim+ x−a
both exist, but do not have the same value, then,
x→a x→a
by definition lim f (x)−f (a)
does not exist. The graph of the function will appear to
x−a
x→a
have a “corner” at x = a.

• If f is not continuous at x = a, then, a fortiori , by the result above88 , it cannot


be differentiable.89
88
Differentiability ⇒ continuity2119
89
In the language of logic we are using the contrapositive of the result we proved: the contrapositive
is true if and only if the original statement is true. The contrapositive is obtained by reversing the
implication direction and also negating both statements. So the contrapositive of “A implies B” is “if
B is false, A is false”.
Notes for Lecture Section 002, MATH 140 2008 09 2122

• If f is continuous, but has a vertical tangent line, we say that lim f (x)−f
x−a
(a)
does
x→a
not exist, so f is not differentiable.
p An example of such a situation is at the point
x = 0 for the function |x|. Note that the function is continuous at 0. We say
that its graph has a cusp at x = 0.
• Differentiability can fail for other reasons. For example, the function
½
x sin x1 if x 6= 0
f (x) =
0 if x = 0
defined in [1, Problem 51, §2.7, p. 153] fails to have a limit because the slopes of
the secants through the origin oscillate between ±1 arbitrarily close to x = 0. In
this course you are not expected to be able to construct “pathological” functions
of this type.
Example 5.29 The function f (x) = |x| is not differentiable at the point x = 0; it is
differentiable at all other points in R.
Solution:
1. Suppose that a > 0. Then the derivative of f at a is
f (a + h) − f (a) |a + h| − |a|
lim = lim
h→0 h h→0 h
(a + h) − a
= lim when a + h ≥ 0
h→0 h
h
= lim = lim 1 = 1
h→0 h h→0

2. Suppose that a < 0. Then the derivative of f at a is


f (a + h) − f (a) |a + h| − |a|
lim = lim
h→0 h h→0 h
−(a + h) + a
= lim when a + h ≤ 0
h→0 h
−h
= lim = lim (−1) = −1
h→0 h h→0

3. Suppose a = 0. Then the limit of the “difference quotient” from the right is
f (0 + h) − f (0) |0 + h| − |0|
lim+ = lim+
h→0 h h→0 h
h−0
= lim+
h→0 h
h
= lim+ = lim 1 = 1
h→0 h h→0
Notes for Lecture Section 002, MATH 140 2008 09 2123

And the limit from the left is


f (0 + h) − f (0) |0 + h| − |0|)
lim− = lim−
h→0 h h→0 h
−h − 0)
= lim−
h→0 h
−h
= lim− = lim− (−1) = −1
h→0 h h→0

f (0+h)−f (0)
Since the limit from the left differs from the limit from the right, lim h
h→0
does not exist.

Higher Derivatives. The operation of differentiation produces from a function f an-


other function, f 0 , whose domain is contained in the domain of f , but may be smaller!
We can apply the same operation on f 0 , and thereby obtain the function (f 0 )0 , which we
call the Second Derivative, usually denoted by f 00 . This procedure can be repeated indef-
initely, to obtain the Third , Fourth, . . . , nth Derivatives; we call the derivatives after the
first the higher derivatives. We use variants of the ¯ symbols introduced earlier, as f 00 (a),
d2 f dn f dn f ¯¯
f 000 (a), . . . , f (n) (a); 2
(a), . . . , n
(a); n¯
; Dn f (a), Dn f (x)|x=a ; Dxx...x f (a);
· n ¸ dx dx dx x=a
d f
. Note that in the “Leibniz” (fractional) notation the symbol indicating the
dxn x=a
number of differentiations is placed at a location in the “numerator” different from that
in the “denominator” of the symbol. Students studying physics or dynamics will be
exposed to physical significance of the 2nd and possibly higher derivatives; for example,
the 2nd derivative of displacement with respect to time is called the acceleration.
The second derivative represents the rate of change of the first derivative. In [1, §4.3]
we shall see that, for a doubly differentiable function f (x), f 00 (x) has a significance in
connection with the “concavity” of the graph of f .
When f (t) represents the position at time t of a particle moving on a line, the first
derivative represents the velocity of the moving particle, and y 00 (t) represents the rate of
change of velocity, which is called the acceleration of the moving particle. Many of the
common equations of mathematical physics involve the second derivative, since Newton’s
2nd Law of Motion [1, §6.4, p. 438] relates the time derivative of the product of mass
and f 0 (t) to the applied force. Where mass is constant, this reduces to the equation

F = mass × f 00 (t) ,

which leads to a differential equation that must be solved to determine the equation of
motion of the moving particle. Even where mass is not constant, as, for example, in a
Notes for Lecture Section 002, MATH 140 2008 09 2124

rocket rising vertically under gravity, and gradually expending its fuel (and so reducing
its mass), an equation can be obtained involving the second derivative:

d mass 0
F = · f (t) + mass × f 00 (t) .
dt

2.8 Exercises The intention is that all derivatives be computed “from first principles”,
since we have not yet derived “Rules” for calculating derivatives of familiar functions.
You are not permitted to use standard formulæ for calculating derivatives, even though
you may have learned them prior to taking this course. Those formulæ will be developed
in [1, Chapter 3].

[1, Exercises 2.8.22, p. 151] “If g(x) = 1−x3 , find g 0 (0) and use it to find an equation
of the tangent line to the curve y = 1 − x3 at the point (0, 1).”
Solution:

g(x) − g(0)
g 0 (0) = lim
x→0 x−0
(1 − x3 ) − (1 − 03 )
= lim
x→0 x
−x3 ¡ ¢
= lim = lim −x2 = 0
x→0 x x→0

so the tangent at (x, y) = (0, 1) has slope 0, i.e., is horizontal. Its equation is
y − 1 = 0(x − 0), i.e., y = 1.

[1, Exercise 24, p. 163] Find the derivative of the function f (x) = x + x using the
definition of derivative. State the domain of the function and the domain of its
derivative.
Solution: The square root is defined for all non-negative x; the polynomial x is
defined for all real numbers. The intersection of these sets, [0, ∞), is the domain
of the function.
√ √
0 (x + h) + x + h − x − x
f (x) = lim
h→0
√ h
µ √ ¶
x+h− x
= lim 1 +
h→0 h
µ√ √ √ √ ¶
x+h− x x+h+ x
= 1 + lim ·√ √
h→0 h x+h+ x
(rationalizing the numerator)
Notes for Lecture Section 002, MATH 140 2008 09 2125

à √ √ !
( x + h)2 − ( x)2
= 1 + lim ¡√ √ ¢
h→0 h x+h+ x
à !
|x + h| − |x|
= 1 + lim ¡√ √ ¢
h→0 h x+h+ x
à !
(x + h) − x
= 1 + lim ¡√ √ ¢
h→0 h x+h+ x
since the domain of f is [0, ∞)
à !
h
= 1 + lim ¡√ √ ¢
h→0 h x+h+ x
µ ¶
1
= 1 + lim √ √
h→0 x+h+ x
lim 1
h→0
= 1+ √ √
lim x + h + lim x
h→0 h→0
1
= 1+ q q
lim (x + h) + lim x
h→0 h→0
by continuity of the square root function
1 1
= 1+ √ √ =1+ √ .
x+ x 2 x
The domain of the derivative is (0, ∞): the point 0, which is in the domain of
the function, is not in the domain of the derivative. (As x → 0+ the slope of the
tangent approaches +∞, so the limit from the right does not exist; the function is
not defined for x < 0, so the limit from the left does not exist either.)
Suggested exercises:
1. Try to extend the method of rationalization shown above to find the derivative
of the derivative.
√ p
2. Replace x by |x|, so that the function is now defined over all of R, and
determine the derivative, wherever the new function is differentiable.
[1, Exercises 2.7.51–5, p. 153] are interesting, but it would be beyond the expecta-
tions of this course to expect you to be able to solve them without assistance.
However, I may discuss them in class, perhaps together with a third function:
½
sin x1 if x 6= 0
f1 (x) =
0 if x = 0
Notes for Lecture Section 002, MATH 140 2008 09 2126

½
x sin x1 if x 6= 0
f2 (x) =
0 if x = 0
½
x2 sin x1 if x 6= 0
f3 (x) =
0 if x = 0

Here f1 is continuous everywhere except at x = 0, and has a derivative at every


point; f2 is continuous everywhere, and has a derivative at every point except
x = 0; and f3 is continuous everywhere and has a derivative everywhere.

5.10.2 2 Review
True-False Quiz These are valuable exercises for determining whether you understand
the material in the chapter. We can’t use problems like this on assignments, since, as
True/False problems, you can guess the answer with an expectation of 50% correctness.
But you should try them yourselves, and ask me or a TA if you are unsure of the
correctness of your answer. Some of them could be modified into good examination
questions.
based on [1, True-False Question 6, p. 166] “Prove or disprove the following state-
ment: ‘If lim f (x) · g(x) exists, then the limit must be f (6) · g(6).’ ”
x→6
Solution: The statement is false. A very simple example is
½
1 if x 6= 6
f (x) = g(x) = ,
undefined if x = 6
where f and g are not even defined at x = 6, even though the limit of their product
exists. In this case the discontinuity of the functions is removable, but we could
1
modify the example to make it unremovable, for example, by taking f (x) =
x−6
and g(x) = x−6. Here g has limit 0, but f (6) is undefined and has an unremovable
discontinuity. Other variations can be constructed where f and/or g are more “ill
behaved”, and one is the reciprocal of the other, so their product is “well behaved”.
We can use the same device to construct functions whose sum is “well behaved”
but where the individual functions are not, if we design the functions so that the
sum cancels out the poor behavior of the individual summands.

based on [1, True-False Question 10, p. 166] “Prove or disprove the statement ‘If
f has domain [0, ∞), and has no horizontal asymptote, then lim f (x) = ∞ or
x→∞
lim f (x) = −∞.’ ”
x→∞
Solution: The statement is false. To disprove it I supply a counterexample. To
have a horizontal asymptote we need the limit to exist (i.e., as a real number). But
Notes for Lecture Section 002, MATH 140 2008 09 2127

the non-existence of the limit does not imply that the limit is ±∞. For example,
the function f (x) = [[x]]−x has no limit as x → ∞ or as x → −∞; another example
is sin x.

based on [1, True-False Question 11, p. 166] “Prove or disprove the following state-
ment. If you claim it is false, you are expected to supply a counterexample:

‘If the line x = 1 is a vertical asymptote of y = f (x), then f is not


defined at x = 1.’ ”

Solution: The statement is false. To be a vertical asymptote we need only that


one of lim− f (x), lim+ f (x), is either −∞ or +∞; thus there are 4 possible reasons
x→1 x→1
why x = 1 might be a vertical asymptote, and we need at least one of them to
hold; it could happen that 2 of them hold, but not more. These conditions are
independent of the question of whether f is defined at x = 1. For example, the
function f defined by 
 1
 if x 6= 1
f (x) = x−1

 5 if x = 1
is defined at x = 1 and has the line x = 1 as a vertical asymptote (for two different
reasons).

based on [1, True-False Question 17, p. 166] We could ask “Is the following state-
ment true or false? If it is true simply state that. If it is false, you are expected to
provide an example — called a counterexample — to substantiate your claim:

‘If f is continuous at a, then f is differentiable at a.’ ”

Solution: The statement is false, as we have seen above. One counterexample is


the function f (x) = |x|, which is continuous at all points — you could be asked to
prove that — but which fails to have a derivative at x = 0.

Review Exercises

[1, Review Exercise 22,√p. 167] Determine


√ all horizontal and vertical asymptotes to
the graph of f (x) = x2 + x + 1 − x2 − x.
Solution: By “completion of the square” we see that
µ ¶2
2 1 3
x +x+1= x+ + >0
2 4
Notes for Lecture Section 002, MATH 140 2008 09 2128


so the domain of x2 + x + 1 is all of R. However,
√ p
x2 − x = x(x − 1)

is defined only for x ≤ 0 and for x ≥ 1. The domain of the difference f (x) is the
intersection of these two domains, i.e., again the union of the intervals (−∞, 0] and
[1, +∞). But, at every point of this domain, the one-sided limits are finite; hence
there are no vertical asymptotes.
√ √Note that x = 0 is not a vertical asymptote, since
the limit from the left is 1 − 0 = 1, not ±∞; the graph is, in fact, tangent to
the y-axis, and touches the axis, at the point (x, y) = (0, 1).
As x → ±∞ the two square roots both become infinite, so we cannot evaluate the
limit by using the Difference Law. We will use the usual device of rationalizing the
square roots:
√ √
x2 + x + 1 − x2 − x
³√ √ ´ √ x2 + x + 1 + √ x2 − x
= x2 + x + 1 − x2 − x · √ √
x2 + x + 1 + x2 − x
(x2 + x + 1) − (x2 − x)
= √ √
x2 + x + 1 + x2 − x
2x + 1
= √ √ ,
x + x + 1 − x2 − x
2

provided x is not in the interval 0 < x < 1. Now what? We are planning to have
x → ∞ and x → −∞. Both the numerator and the denominator approach ±∞,
so we can’t use the Quotient Law.
¡ ¢
2x + 1 x 2 + x1
√ √ = q ¡ ¢ q ¡ ¢
x2 + x + 1 + x2 − x x2 1 + x1 + x12 + x2 1 − x1
¡ ¢
x 2 + x1
= √ q √ q
x 1 + x + x2 + x2 1 − x1
2 1 1

√ √ √
since ab = a · b where everything is defined
¡ ¢
x 2 + x1
= q q
|x| 1 + x1 + x12 + |x| 1 − x1
by definition of |x|
x 2 + x1
= ·q q
|x| 1+ 1 + 1 + 1− 1
x x2 x
Notes for Lecture Section 002, MATH 140 2008 09 2129

2
As x → +∞ this product approaches 1 · 1+1 = 1, and as x → −∞ the product
2
approaches (−1) · 1+1 = −1. There are thus two horizontal asymptotes: y = ±1,
(see Figure 12).

y
0
-10 -5 0 5 10
x

-1

y=f(x)
upper bound, y=f(1)
horizontal asymptote y=-1
horizontal asymptote y=1

√ √
Figure 12: Graph of the Function f (x) = x2 + x + 1 − x2 − x

[1, Review Exercise 34, p. 168] “Use the Intermediate Value Theorem to show that
2
there is a solution of the equation e−x = x in the interval (0, 1).”
Solution: To apply the given theorem we need to designate a continuous function.
But the equation relates two functions! There are many ways to combine them,
2
but the easiest is something like this. Define f (x) = e−x − x. We know that
the polynomials −x2 and x are continuous everywhere; and we know that the
exponential function ex is continuous everywhere. Hence the composition of the
2
polynomial function −x2 followed by the exponential function, i.e., e−x , is also
continuous everywhere. We also know that sums and differences of continuous
functions are continuous where the summands and “differands” are continuous.
That tells us that the function we have called f is continuous everywhere. The
theorems we have applied are [1, Theorems 2.5.4, 2.5.5, 2.5.7, 2.5.8 in §2.5].
Now we have to show that the value of interest lies between the values of the
2
function at the end points of the given interval. f (0) = e−0 −0 = e0 −0 = 1−0 = 1,
and
2 1
f (1) = e−1 − 1 = − 1 < 1 − 1 = 0 .
e
Thus f (0) > 0 > f (1). By the Intermediate Value Theorem [1, Theorem 2.5.10,
p. 126] there exists a point c such that 0 < c < 1 and f (c) = 0, i.e., such that
2
e−c = c.
(One application of the theorem simply proves the existence of the point. But,
if we were to bisect the interval (0, 1), and evaluate the function at the midpoint
Notes for Lecture Section 002, MATH 140 2008 09 2130

1
x = , we could trap a root in an interval of half the length; every 10 repetitions
2
of this bisecting reduces the size of the interval by a factor of 210 > 1000 = 103 , so
we could determine a solution to any desired number of decimal places, provided
we had a way of evaluating the exponential function whenever we needed it.)

[1, Review Exercise 36, p. 168] “Find equations of the tangent lines to the curve
2
y= at the points with x-coordinates 0 and −1.”
1 − 3x
Solution: We will first determine the derivative in general from first principles,
then take the specific values for the given points.
y(x + h) − y(x)
y 0 (x) = lim
h→0 h
2 2
1−3(x+h)
− 1−3x
= lim
h→0 h
(1 − 3x) − (1 − 3x − 3h)
= lim 2 ·
h→0 h(1 − 3x)(1 − 3(x + h))
3h
= 2 lim
h→0 h(1 − 3x)(1 − 3(x + h))
3
= 2 lim
h→0 (1 − 3x)(1 − 3(x + h))
3
= 2 lim = 6(1 − 3x)−2
h→0 (1 − 3x)2

¡ ¢
Then y 0 (0) = 6, and y 0 (−1) = 38 . The tangents at the points (0, 2) and −1, 21
respectively have equations

y − 2 = 6(x − 0) or 6x − y = −2 and
1 3
y − = (x + 1) or 3x − 8y = −7.
2 8

[1, Problems Plus Exercise 12, p. 171] Suppose that f is a differentiable function,
and g(x) = xf (x). Use the definition of a derivative to show that g 0 (x) = xf 0 (x) +
f (x).
Solution:
g(x + h) − g(x)
g 0 (x) = lim
h→0 h
(x + h)f (x + h) − xf (x)
= lim
h→0 h
Notes for Lecture Section 002, MATH 140 2008 09 2131

µ ¶
f (x + h) − f (x)
= lim f (x + h) + x ·
h→0 h
µ ¶
f (x + h) − f (x)
= lim f (x + h) + lim x ·
h→0 h→0 h
by the Sum Law
f (x + h) − f (x)
= lim f (x + h) + x lim
h→0 h→0 h
since x is constant with respect to lim
h→0
0
= lim f (x + h) + xf (x)
h→0
by definition of f 0
= f (x) + xf 0 (x) since f is continuous

[1, Problems Plus Exercise 14, p. 171] “Suppose that f is a function with the prop-
erty that
|f (x)| ≤ x2 (105)
for all x. Show that f (0) = 0. Then show that f 0 (0) = 0.
Solution: One reason I am discussing this problem is to remind you of how to
interpret an inequality where the left member is an absolute value.
First observe that when we set x = 0 in (105) we obtain |f (0)| ≤ 0; but, as we
know that |f (0)| ≥ 0 (being an absolute value), we conclude that

f (0) = 0 . (106)
f (x) − f (0)
The derivative of f at x = 0 is the limit of the quotient as x → 0.
x−0
From (105) we may conclude that

−x2 ≤ f (x) ≤ x2 (107)

for all x; hence, for x 6= 0, we may divide all members by x to obtain


f (x) − f (0)
−x ≤ ≤ x. (108)
x
Since
lim (−x) = 0 = lim x ,
x→0 x→0

f (x) − f (0)
we conclude from the Squeeze Principle that lim = 0, i.e., that f 0 (0) =
x→0 x
0.
Notes for Lecture Section 002, MATH 140 2008 09 2132

5.11 Supplementary Notes for the Lecture of October 15th,


2008
Release Date: Wednesday, October 15th, 2008
subject to correction

Textbook Chapter 3. DIFFERENTIATION RULES.


(I began the lecture by reviewing the definition of the derivative, and illustrated
√ the
computation of a derivative “from first principles”, using the function f (x) = x + 5. I
reminded students that, while the “rules” we will be discussing will be used in practice
to calculate derivatives, they are expected to determine derivatives using limits, when
asked to do so for reasonable functions.)

5.11.1 §3.1 Derivatives of Polynomials and Exponential Functions


Beginning in this section we develop properties of the differentiation operation that will
permit the systematic computation of derivatives without the need for computations
“from first principles”. While students are expected to master these “Rules”, you are
also expected to be able to carry out computations “from first principles”. Except for
problem functions that don’t fit into the categories to which the “Rules” apply, you will
normally not use “first principles” unless asked to do so.

Derivative of a constant function. For any constant c the function x 7→ c has, at


any point x, the derivative
d f (x + h) − f (x) c−c 0
c = lim = lim = lim = lim 0 = 0
dx h→0 h h→0 h h→0 h h→0

by virtue of the Law of Limits that states that the limit of a constant c is that constant
c.

Power Rules. We can easily prove from first principles that


d
x = 1.
dx
If we ask what is the derivative of the nth power of x, we have two facts available, as we
have determined the derivatives of x = x1 and 1 = x0 . It would be prudent to experiment
before guessing about a general rule. So we could, from first principles, determine the
derivatives of x2 and x3 , and then we could conjecture (=guess) that
d n
x = nxn−1 ;
dx
Notes for Lecture Section 002, MATH 140 2008 09 2133

this can be proved in various ways. The textbook is careful to assert, at first, that this
generalized “Power Rule” is true for positive integers. But, in the course of several steps,
it will eventually be “proved” for all real number n. You are not expected to be able to
reproduce the proofs. Both of the proofs that the textbook gives of the Power Rule for
integer exponents rely on this Principle.90

New Derivatives from Old. The textbook proves “rules” the author calls the Con-
stant Multiple Rule, the Sum Rule, the Difference Rule (a consequence of the preceding
two rules). You are not expected to memorize these proofs, but the level of difficulty of
the proofs is no greater than could arise in a reasonable problem that could be set on the
material of this section and some of the preceding sections. The several rules presented,
d d
(cf (x)) = c f (x) (109)
dx dx
d d d
(f (x) ± g(x)) = f (x) ± g(x) (110)
dx dx dx
could all be compressed into one general rule:
d d d
(f (x) + `g(x)) = f (x) + ` g(x) (111)
dx dx dx

or, more symmetrically,


d d d
(kf (x) + `g(x)) = k f (x) + ` g(x) (112)
dx dx dx

where k and ` are any real constants (positive, negative, or zero), and f and g are
differentiable functions.

Exponential Functions. The textbook considers, for any positive real number a, the
value of the derivative of the function ax at any point x. The author shows that
d x ah − 1
a = ax · lim .
dx h→0 h
Thus, if the limit
ah − 1
lim (113)
h→0 h
90 d
Yet another proof could be based on applying to the result (x) = 1 the Product Rule (proved
dx
below in the discussion of [1, §3.2]). A proof would require using Mathematical Induction or an equivalent
tool, [1, Exercise 3.2.55, p. 189].
Notes for Lecture Section 002, MATH 140 2008 09 2134

exists — which the author assumes without proof91 — the derivative is a constant mul-
tiple of the function; this can be shown to be a property that actually characterizes
exponential functions — they are the only functions that have this property. The text-
book reports, without proof, that the value of limit (113) ranges continuously over the
positive real numbers. Then Napier’s constant, denoted by e, is “defined” to be the
eh − 1
unique constant, lying between 2 and 3, with the property that lim = 1, and
h→0 h
d x
consequently e = ex . This is the same constant e for which it was mentioned earlier
dx
(66) that

e = 2.71828182845904523536028747135266249775724709369995957...

The differentiation rules developed in this section are summarized in Table 5.11.1.92

3.1 Exercises Remember that the textbook’s intention is that you use only tools that
are mentioned, or, in any case, do not use tools that will be developed in subsequent
sections. So, for example, [1, Exercise 3.1.22, p. 181] should not be solved using the
product rule.

[1, Exercise
√ √ 3.1.46, p. 181] Find the first and second derivatives of the function G(r) =
r + 3 r.
Solution:
d d ¡√ √ ¢
G(r) = r+ 3r
dr dr
d ³ 1 1
´
= r2 + r3
dr
d ³ 1´ d ³ 1´
= r2 + r3 by the Sum Rule
dr dr
1 1 −1 1 1 −1
= r2 + r3 by the Power Rule
2 3
1 −1 1 −2
= r 2+ r 3
2 µ 3 ¶
d2 d d
2
G(r) = G(r)
dr dr dr
by definition of the Second Derivative
91
Remember, this is the “Early Transcendentals” version of the textbook: the study of exponential
functions has been advanced to a point where some steps cannot be proved.
92
There remains some unfinished business: we are not ready to determine the derivative of ax , where
a is a positive constant different from e.
Notes for Lecture Section 002, MATH 140 2008 09 2135

Derivative of a Constant:
d
(c) = 0
dx
Power Rule:
d n
(x ) = nxn−1
dx
for any real number n

Constant Multiple Rule:


d d
(cf (x)) = c f (x)
dx dx
for any constant c
and any differentiable function f

Sum and Difference Rules:


d d d
(f (x) ± g(x)) = f (x) ± g(x)
dx dx dx
for any differentiable functions f , g

Derivative of Natural Exponential Function:


d x
(e ) = ex
dx

Table 6: Differentiation Rules from [1, §3.1]


µ ¶
d 1 −1 1 −2
= r + r
2 3
dr 2 3
µ ¶ µ ¶
d 1 −1 d 1 −2
= r 2 + r 3 by the Sum Rule
dr 2 dr 3
1 d ³ −1 ´ 1 d ³ −2 ´
= r 2 + r 3
2 dr 3 dr
by the Constant Multiple Rule
µ ¶ µ ¶
1 1 − 1 −1 1 2 − 2 −1
= · − r 2 + · − r 3
2 2 3 3
µ ¶ µ ¶
1 1 −3 1 2 −5
= · − r 2+ · − r 3
2 2 3 3
1 3 2 5
= − r− 2 − r− 3
4 9
Notes for Lecture Section 002, MATH 140 2008 09 2136

[1, Exercise 3.1.52, p. 181] “For what values of x does the graph of f (x) = x3 +3x2 +
x + 3 have a horizontal tangent?”
Solution: To determine the points on a curve y = f (x) having a horizontal tangent
one solves the equation f 0 (x) = 0. In this problem that equation is quadratic, and
leads to two points (with unpleasant coordinates). By the Rules developed thus
far
d ¡ 3 ¢ d ¡ 3¢ d ¡ 2¢ d d
x + 3x2 + x + 3 = x + 3x + (x) + (3)
dx dx dx dx dx
by the Sum Rule
d ¡ 3¢ d ¡ 2¢ d d
= x +3 x + (x) + (3)
dx dx dx dx
by the Constant Multiple Rule
d ¡ 3¢ d ¡ 2¢ d
= x +3 x + (x) + 0
dx dx dx
by the Derivative of a Constant Function Rule
¡ ¢ ¡ ¢
= 3x2 + 3 2x1 + 1 x0 + 0
by the Power Rule
= 3x2 + 3 (2x) + 1 (1) + 0 = 3x2 + 6x + 1 .

Tangents are horizontal at points whose abscissa x is a root of the polynomial


3x2 + 6x + 1. Students should be able to find the roots by using the “Quadratic
Formula”. I prefer to use “Completion of the Square”, because it is a technique
that
¡ we will find
¢ useful in¡ the next course:
¢ 3x2 + 6x + 1 = 3(x2 + 2x) + 1 =
2 2 2
3 (x + 1) − 1 + 1 = 3 (x + 1) − 3 . The expression in parentheses may be
interpreted as a difference of squares, and we know how to factor such differences:
Ãr !2 Ã r !Ã r !
2 2 2 2
(x + 1)2 − = (x + 1)2 − = x+1+ x+1−
3 3 3 3

so the derivative is 0 when either of these linear factors is 0, i.e., when


r
2
x = −1 ± .
3
Some students may have been “taught” to avoid placing a surd in the denominator;
if you prefer, we can write the solution as
1√
x = −1 ± 6.
3
Notes for Lecture Section 002, MATH 140 2008 09 2137

(Placing a surd in the denominator is not incorrect; in the days before calculators
were commonly available, computation could be greatly simplified when denomi-
nators were integers. Avoiding surds in the denominator may still be advisable,
but it is not mandatory.)

[1, Exercise 3.1.58, p. 182] “Where does the normal line to the parabola y = x − x2
at the point (1, 0) intersect the parabola a second time?”
Solution: In this context the word normal means perpendicular : thus its slope will
be not the derivative but the negative reciprocal of the derivative:
1
− .
dy
dx
This is by virtue of the property that
³ π´ 1
tan θ + = , (114)
2 − tan θ
which is a consequence of the property that
tan a + tan b
tan(a + b) = . (115)
1 − tan a · tan b
d
(x − x2 ) = 1 − 2x, so the tangent at the point (1, 0) has slope 1 − 2 = −1, and
dx
1
the normal (line) has slope − = 1; thus the equation of that normal is
−1
y − 0 = 1(x − 1) .

If we solve this equation with the equation of the curve, we obtain two values for the
x-coordinate of a point of intersection, x = −1 and x = 1, so there are two points
where the line and curve intersect: (1, 1 − 12 ) = (1, 0) and (−1, −1 − (−1)2 ) =
(−1, −2). The first is the point of tangency; the second is where the tangent at
(x, y) = (1, 0) meets the curve a second time.

[1, Exercise 3.1.60(a), p. 182] “Find equations of both lines through the point (2, −3)
that are tangent to the parabola y = x2 + x.”
Solution: The point (x, x2 + x) on the curve has a tangent with slope

d 2
(x + x) = 2x + 1 .
dx
Notes for Lecture Section 002, MATH 140 2008 09 2138

As we wish to find an equation, and require the symbol x to denote the abscissa
of the general point on that line, we will rephrase the preceding result: the point
(a, a2 + a) on the curve has a tangent with slope 2a + 1. The tangent at the point
(a, a2 + a) has equation
y − (a2 + a) = (2a + 1)(x − a) or y = (2a + 1)x − a2 .
We impose the condition that these tangent lines pass through the point with
coordinates (2, −3), i.e., that (x, y) = (2, −3) is a solution of the equation:
−3 = (2a + 1)2 − a2 or a2 − 4a − 5 = 0 .
Solving this quadratic equation yields a = −1, 5, so the two tangents through
(2, −3) are
y = −x − 1 and y = 11x − 25 .
(There is a “classical” way of solving this problem without apparent use of the
calculus, but it will not be studied in this course.)
[1, Exercise 3.1.70, p. 182] Where is the function h(x) = |x − 1| + |x + 2| differen-
tiable? Give a formula for h0 .
Solution: This function is piecewise polynomial, but it changes its description at
the points where the absolute value functions change theirs. We know that
½
−(x − 1) if x − 1 < 0
|x − 1| = (116)
+(x − 1) if x − 1 ≥ 0
½
−(x + 2) if x + 2 < 0
|x + 2| = (117)
+(x + 2) if x + 2 ≥ 0
It follows that the sum of the functions behaves as follows:

 −(x − 1) − (x + 2) = −2x − 1 if x < −2
|x − 1| + |x + 2| = −(x − 1) + (x + 2) = 3 if −2 ≤ x < 1

+(x − 1) + (x + 2) = 2x + 1 if 1 ≤ x .
The derivative when x is different from −1, 2 is that of the appropriate polynomial:

 −2 if x < −2
h0 (x) = 0 if −2 < x < 1 .

+2 if 1 < x
But the function is not differentiable at either x = −2 or x = 1. This is because the
one-sided limits of the difference quotients are not equal, so the difference quotient
does not have a limit at either of these points. For example, at x = −2, where
|x + 2| = 0,
−h − 0 +h − 0
lim− = −1 6= 1 = lim+ .
h→0 h h→0 h
Notes for Lecture Section 002, MATH 140 2008 09 2139

5.11.2 §3.2 The Product and Quotient Rules


The textbook continues the derivation of “Rules” which will permit the mechanical
computation of derivatives for common functions without the need to evaluate limits. In
practice you will usually use these rules; but, as with the Rules in the preceding section,
you need to be able to find limits “from first principles” when so requested.

The Product Rule. The proof of the Product Rule requires a familiar “trick”, similar
to one we saw in the proof that differentiability implies continuity: we add a term, to
transform a function to a more convenient form, then subtract the offending term so
that the value has not been changed. One point in the proof is delicate — we use the
continuity of one of the functions at one point: we know this to be true since we are
assuming that both functions are differentiable.
While we will not ask you to memorize this proof — or any proofs — we expect
you to understand the use of continuity, and could devise a way of calling on you to
demonstrate that in an examination — without having to reproduce a memorized proof.

The Quotient Rule; the Reciprocal Rule. The “Quotient Rule” could also be
proved by first proving a Reciprocal Rule [1, Exercise 3.2.58, p. 189]:

µ ¶ d
d 1 g(x)
= − dx 2 ,
dx g(x) (g(x))
1
and then applying the product rule to the product f (x) · . The differentiation rules
g(x)
developed in this section are summarized in Table 5.11.2.
Example 5.30 (cf. [7, Exercise 3.2.32, p. 197]) “If f (3) = 4, g(3) = 2, f 0 (3) = −6, and
g 0 (3) = 5, find the following numbers:
(a) (f + g)0 (3)
(b) (f g)0 (3)
µ ¶0
f
(c) (3)
g
µ ¶0
f
(d) (3)”
f −g
These problems refer to the conventions that the textbook defined in [1, Combinations
of Functions, p. 41].
Solution:
Notes for Lecture Section 002, MATH 140 2008 09 2140

Product Rule:
d d d
(f (x) · g(x) = f (x) · g(x) + f (x) · g(x)
dx dx dx
for any differentiable functions f , g

Quotient Rule:
µ ¶ d d
d f (x) f (x) · g(x) − f (x) · g(x)
= dx 2
dx
dx g(x) (g(x))
for any differentiable functions f , g,
(where g(x) 6= 0)

Reciprocal Rule: [1, Exercise 58, p. 189]


µ ¶ d
d 1 f (x)
=− dx
dx f (x) (f (x))2
for any differentiable function f , where f (x) 6= 0

Table 7: Differentiation Rules from [1, §3.2]

(a) By the Sum Rule, (f + g)0 (x) = f 0 (x) + g 0 (x) for all x. Here (f + g)0 (3) = f 0 (3) +
g 0 (3) = (−6) + 5 = −1.

(b) By the Product Rule (f g)0 (3) = f (3) · g 0 (3) + f 0 (3) · g(3) = 4 · 5 + (−6) · 2 = 8.
µ ¶0
f g(3) · f 0 (3) − f (3) · g 0 (3) 2 · (−6) − 4 · 5
(c) By the Quotient Rule (3) = 2 = =
g (g(3)) 22
−8.

(d)
µ ¶0
f (f − g)0 (3) · f (3) − f (3) · (f − g)0 (3)
(3) =
f −g ((f − g)(3))2
(f (3) − g(3)) · f 0 (3) − f (3) · (f 0 (3) − g 0 (3))
=
((f − g)(3))2
(4 − 2) · (−6) − 4 · (−6 − 5)
= = 8.
(4 − 2)2

3.2 Exercises
Notes for Lecture Section 002, MATH 140 2008 09 2141

x
[1, Exercise 3.2.42, p. 188] If g(x) = , find g (n) (x).
ex
Solution: Since this is a formula for general, integer n, a formal proof would require
the use of the Principal of Induction, which has been omitted from this year’s
syllabus. Even with the use of that principle, there are still different ways of
attacking the problem. The textbook probably intends at this point that you
differentiate the function several times, examine the result, and then conjecture
(=guess) what the general formula should be; a proof would require Mathematical
Induction.
A simpler way of arriving at a guess would be to delay this problem until we have
reached the Chain Rule; that would permit us to treat the function as a product,
xe−x , rather than as a quotient; it’s usually easier to differentiate products than
quotients. We would still have to do some experimentation to obtain evidence
on which to base a guess; and a formal proof would still require Mathematical
Induction or some equivalent tool.
[1, Exercise 3.2.52, p. 189] “Find equations of the tangent lines to the curve y =
x−1
that are parallel to the line x − 2y = 2.”
x+1
Solution: One way of solving this problem would be similar to that given earlier for
[1, Exercise 3.1.60(a), p. 182] above: one finds the slope of the tangent at any point
on the curve y = f (x), say with coordinates (a, f (a)), and imposes the condition
that these tangents have the slope of a given line, thereby locating the points at
which the tangents are parallel to the given line. There are two solutions. Problems
of these types, involved specific techniques of using the calculus to solve geometric
problems, specifically to find tangents to a curve where the given information does
not include the point(s) of contact of the tangents, are well within the range of
problems that students in Math 140 are expected to be able to solve.
x−1
Here is a slightly different approach. The derivative of can be shown to be
x+1
2(x + 1)−2 . We wish to find the tangent lines that are parallel to x − 2y = 2, i.e.,
those whose slope is 12 . Imposing that condition on the derivative yields
2 1
2
=
(x + 1) 2
which reduces to x2 + 2x − 3 = 0 or (x + 3)(x − 1) = 0, satisfied by x = −3 and
x = 1. We have thus µ shown that¶the tangents
µ we¶seek are those which touch the
−3 − 1 1−1
curve at the points −3, and 1, , i.e., at (−3, 2) and (1, 0). The
−3 + 1 1+1
lines through these points with slope 21 are x − 2y = −7 and x − 2y = 1.
Notes for Lecture Section 002, MATH 140 2008 09 2142

5.12 Supplementary Notes for the Lecture of October 20th,


2008
Release Date: Monday, October 20th, 2008
subject to correction

5.12.1 §3.3 Derivatives of Trigonometric Functions.


sin x
All the results in this section depend on the value of lim , through the application
x→0 x
of simple trigonometric identities.

t
½ D
½
½
½
½
½
½
½
½
½
½
½
B ½
½
½t
½ S
S
½ S
½
½ S
½ S
½ S
½
½ S
½ S
½ S
½
½ S
½ S
½ S
½
½ S
½ S
½ S
½
½ S
½ S
½ S
½ π S

t
½∠ = θ
C∠=
t 2 St A

sin θ
Figure 13: Computing bounds for
θ
sin θ
lim .
θ→0 θ
Notes for Lecture Section 002, MATH 140 2008 09 2143

Warning! The results of this section apply only if the variable of the trigonometric
functions is considered to be in radians; if we wish to consider, for example, a function
f (θ) = the sine of an angle of θ degrees, the derivative will not be the cosine function of
the angle measured in degrees. We can reconsider this question after we have completed
the next section of the textbook.
sin θ
The determination of lim shown in the lectures differs from that in the textbook:
θ→0 θ
we base our inequalities on comparisons of areas, rather than of lengths. While students
are not expected to memorize this proof, it is a valuable example of uses of the “Squeeze”
Theorem, which student often find difficult. We first prove that
tan θ θ sin θ
≥ ≥
2 2 2
for θ > 0, (cf. Figure 13). We begin with a circle (not shown in the figure) with centre O
(taken as the origin), passing through a point A on the positive x-axis, and draw a radius
vector from O which meets the circle in B (so |OA| = |OB|), and extends to intersect
the perpendicular to the x-axis through A at the point D; θ denotes angle AOB. We
also drop a perpendicular from B to the x-axis, meeting it in C. The inequalities can be
obtained by ordering from greatest to least the areas of three regions:
• 4OAD has base |OA| and height |AD| = |OA| tan θ, so its area is 12 |OA|2 tan θ.
θ
• The sector of the circle centred at O and passing through A, B has area ·π|OA|2

= 21 θ|OA|2 .
• 4OBA has height |BC| = |OB| sin θ, and base of length |OA|. The area will be
1
2
|OA| · |BC| = 12 |OA|2 sin θ.
Thus
1 1 1
|OA|2 · sin θ < |OA|2 · θ < |OA|2 · tan θ .
2 2 2
sin θ
If we we divide through by |OA|2 · , we obtain
2
θ
sec θ ≥ ≥ 1,
sin θ
and then take reciprocals of the three members, which reverses both inequalities, yielding
sin θ
cos θ ≤ ≤ 1, (θ > 0) .
θ
Our objective is to apply the Squeeze Theorem, taking the limit as θ → 0. For that
purpose we need to have the pair of inequalities holding in a (possibly “punctured”)
Notes for Lecture Section 002, MATH 140 2008 09 2144

interval surrounding 0, not only on one side. We obtain a corresponding result for θ < 0
simply by renaming θ as −λ, and now requiring λ < 0; the inequalities become

sin(−λ)
cos(−λ) ≤ ≤ 1 (λ < 0) .
−λ
sin(−λ) sin λ
We observe that, since the sine function is odd , sin(−λ) = − sin λ, so =
−λ λ
sin λ
even when λ < 0; and cos(−λ) = cos λ: indeed, both cos λ and are even functions.
λ
Thus we have shown that the same pair of inequalities hold on both sides of 0; of course,
they fail to hold at 0 because the middle member is not defined there. There is no longer
any reason to use two different symbols for the variable on the two sides of 0, so we
combine them into
sin θ
cos θ ≤ ≤ 1 , (θ 6= 0) .
θ
We may now apply the Squeeze Theorem, which does not require that the inequalities
hold at θ = 0, to obtain
sin θ
lim cos θ ≤ lim ≤ lim 1 ,
θ→0 θ→0 θ θ→0

sin θ
i.e., 1 ≤ lim ≤1
θ→0 θ

from which it follows that


sin θ
lim = 1. (118)
θ→0 θ

Note that what we have found is the derivative of sin θ evaluated when θ = 0. We will
d
apply this result to find the value of sin θ at any θ.

lim cos θ − 1 . There are a number of easy ways to extract this limit from (118). The
θ→0 θ
proof in the textbook proceeds as follows:
µ ¶
cos θ − 1 cos θ − 1 cos θ + 1
lim = lim ·
θ→0 θ θ→0 θ cos θ + 1
2
cos θ − 1
= lim
θ→0 θ(cos θ + 1)

− sin2 θ
= lim by a standard identity
θ→0 θ(cos θ + 1)
sin θ − sin θ
= lim · lim
θ→0 θ θ→0 cos θ + 1
Notes for Lecture Section 002, MATH 140 2008 09 2145

sin θ − lim sin θ


θ→0
= lim ·
θ→0 θ lim cos θ + lim 1
θ→0 θ→0
0
= 1· =0 (119)
2
This shows that the value of the derivative of cos θ at θ = 0 is 0.
A shorter proof, using a “double angle formula” is as follows:

cos θ − 1 1 − 2 sin2 2θ − 1
lim = lim
θ→0 θ θ→0 θ
sin θ2 θ
= lim θ · lim sin
θ→0
2
θ→0 2
= 1 · 0 = 0.

Derivative of cos x. As an example, here I determine the derivative of cos x for general
x.
d cos(x + h) − cos x
cos x = lim
dx h→0 h
(cos x · cos h − sin x · sin h) − cos x
= lim
h→0 h
(cos x)(cos h − 1) − (sin x)(sin h)
= lim
h→0 h
(cos x)(cos h − 1) sin x · sin h
= lim − lim
h→0 h h→0 h
by the Sum/Difference Rule
cos h − 1 sin h
= (cos x) · lim − (sin x) · lim
h→0 h h→0 h
by the Constant Multiplier Rule
sin h
= (cos x) · 0 − (sin x) · lim by (119)
h→0 h
= (cos x) · 0 − (sin x) · 1 by (118)
= − sin x.

Derivatives of trigonometric functions Thus far we have been able to determine


the values at 0 of the derivatives of the sine and cosine function, and, just above, the
derivative of the general cosine function. As the other four trigonometric functions can
be expressed in terms of sines and cosines, it should not be surprising that we can now
evaluate all of their derivatives at 0. But, in fact, we now have enough information to
Notes for Lecture Section 002, MATH 140 2008 09 2146

evaluate the derivatives of the these six functions at all points in their domains. (The
functions are differentiable wherever they are defined, but all of them except the sine
and cosine have points on R where they are not defined, and a fortiori there are no
derivatives at those points.) Here are the derivatives:

d
sin θ = cos θ (120)

d
cos θ = − sin θ (121)

d
tan θ = sec2 θ (122)

d
cot θ = − csc2 θ (123)

d
sec θ = sec θ · tan θ (124)

d
csc θ = − csc θ · cot θ (125)

I do not plan to prove most of these results in the lecture, but you should be able to
prove all of them from the derivative of the cosine (or, equivalently, of the sine) by
using standard identities [1, pp. A28-A30] involving the trigonometric functions. Don’t
memorize proofs! While you should remember how we proved these formulæ, the first
priority is that you have the formulæ committed to memory, as several of them are in very
frequent use. (This is one of the few times in this course when I suggest memorization.)

Other limit problems involving trigonometric functions We can also apply the
sin 4x
limit results above to other limit problems. For example, to evaluate lim , we
x→0 tan 7x
sin y
would rewrite the given fraction as a product of two fractions of the form where
y
y → 0:
µ ¶
sin 4x sin 4x 7x 4
lim = lim · ·
x→0 tan 7x x→0 4x tan 7x 7
Notes for Lecture Section 002, MATH 140 2008 09 2147

sin 4x 7x 4
= lim · lim · lim by the Product Law
x→0 4x x→0 tan 7x x→0 7
sin 4x cos 7x 4
= lim · lim sin 7x · lim
x→0 4x x→0 x→0 7
7x

sin 4x x→0lim cos 7x 4


= lim · · lim by the Quotient Law
x→0 4x sin 7x x→0 7
lim
x→0 7x
1 4 4
= 1· · =
1 7 7
In the last line I am, on two occasions, using the result that lim f (g(x)) = f (lim g(x))
x→a x→a
under assumptions of continuity, [1, p. 124]. I usually don’t expect students in Math 140
to give this explanation, but it is a justification that is missing above.
sin x − cos x
Example 5.31 (cf. [7, Exercise 3.4.42, p. 217]) “Find limπ ”.
x→ 4 cos 2x
Solution:
sin x − cos x sin x − cos x
limπ = limπ 2
x→ 4 cos 2x x→ 4 cos2 x − sin x

by a “double angle” formula


sin x − cos x
= limπ
x→ 4 (cos x − sin x)(cos x + sin x)

1
= limπ
x→ 4 (−1)(cos x + sin x)
π
since sin x − cos x 6= 0 near (but not at) x = 4
1 1
= ³ ´ = −2− 2 .
− √12 + √12

3.3 Exercises
tan x − 1
[1, Exercise 3.3.31, p. 196] “Differentiate y = .”
sec x
Solution: While it is possible to solve this problem correctly by applying the Quo-
tient Rule naively, it is easier if the numerator and denominator are first both
multiplied by cos x.
µ ¶
d tan x − 1 d
= (sin x − cos x)
dx sec x dx
= cos x + sin x
Notes for Lecture Section 002, MATH 140 2008 09 2148

However, we must not forget that, in the original, the denominator is not defined at
any odd multiple of π2 , where it has removable discontinuities. Thus the derivative
obtained is valid only when x is not an odd integer multiple of π2 (at which points
the function is undefined).
cos x
[1, Exercise 3.3.34, p. 196] Find the points on the curve y = at which the
2 + sin x
tangent is horizontal.
Solution:

dy (− sin x)(2 + sin x) − (cos x)(0 + cos x)


= by the Quotient Rule
dx (2 + sin x)2
−1 − 2 sin x
= by a standard identity.
(2 + sin x)2

The derivative equals 0 precisely at the points whose abscissa x satisfies the equa-
1
tion −1 − 2 sin x = 0, equivalently sin x = − . In the interval 0 ≤ x ≤ 2π this
2
7π 11π
occurs at x = , . Solving the equation for x ∈ R we obtain
6 6
7π 11π
x= + 2nπ, + 2nπ
6 6
where n is any integer — positive, negative, or 0. More elegantly, the points with
horizontal tangents have abscissa
µ ¶ µ ¶
7 + 12n 11 + 12n
x= π, π (n ∈ Z).
6 6

cos θ − 1
[1, Exercise 3.3.42, p. 196] “Find lim .”
θ→0 sin θ
Solution: When you remember the simple trigonometric identities, there are often
many methods of solving problems of this type that will suggest themselves. One
simple method that does not require such identities would involve first dividing
numerator and denominator by θ, in order to transform both into functions that
have been studied in this section:
 
cos θ − 1
cos θ − 1  θ 
lim = lim  
θ→0 sin θ θ→0 sin θ
θ
Notes for Lecture Section 002, MATH 140 2008 09 2149

µ ¶
cos θ − 1
lim
θ→0 θ
= µ ¶
sin θ
lim
θ→0 θ
0
= = 0.
1
Another attack could be as follows:
¡ ¢
cos θ − 1 1 − 2 sin2 2θ − 1
lim = lim
θ→0 sin θ θ→0 2 sin 2θ · cos 2θ
θ 0
= − lim tan = − tan = − tan 0 = 0
θ→0 2 2

sin(x − 1)
[1, Exercise 3.3.48, p. 196] Find lim .
x→1 x2 + x − 2

Solution:
sin(x − 1) sin(x − 1)
lim 2
= lim
x→1 x + x − 2 x→1 (x − 1)(x + 2)
µ ¶
sin(x − 1) 1
= lim ·
x→1 (x − 1) x+2
by the Product Rule
sin(x − 1) 1
= lim · lim
x→1 (x − 1) x→1 x + 2
sin t 1
= lim · lim
t→0 t x→1 x + 2
changing the variable to t = x − 1
1 1
= 1· since is continuous
1+2 x+2
1
= .
3
The changing of the variable referred to above could also be interpreted in terms
of the continuity of the limit of the composition of two continuous functions. If
sin x sin(x − 1)
one defines f (x) = , and g(x) = x − 1, then lim = lim f (g(x)).93
x x→1 (x − 1) x→1

93
There appears to be a defect in this reasoning, since [2, Theorem 8, p. 125] requires the functions f
and g to be continuous, and f fails to be continuous at x = 0. However, the discontinuity in f at x = 0
is removable: if we extend the definition to include f (0) = 1, then both functions are continuous, and
the theorem is applicable.
Notes for Lecture Section 002, MATH 140 2008 09 2150

5.12.2 §3.4 The Chain Rule.


The “Chain Rule” enables us to determine the derivative of the composition of several
differentiable functions. It can be described briefly using the “Leibniz notation”: If
y = f (u) and u = g(x), where f and g are both differentiable, then the derivative of the
composite function f ◦ g is given by
dy dy du
= · , (126)
dx du dx
which is easy to remember because it appears to be a statement about the product of
two fractions. If we define the derivative as the limit of a quotient of increments, then
the Chain Rule appears to be a simple consequence upon the passage to the limit; but
it’s not as simple as it appears — however, we won’t be discussing the proof of the Chain
Rule in this course. The Chain Rule may also be written in the form
d
f (g(x)) = f 0 (g(x)) · g 0 (x) . (127)
dx
The Chain Rule may need to be applied more than once when we differentiate a function
that has been constructed by composing many functions. In general, it involves “peeling
off one layer after another” of the composed functions, at every stage multiplying by
a factor which is a derivative evaluated at the point where the original function was
evaluated. When you are first learning the Chain Rule, you may find it convenient
to name the various functions that compose. Eventually this intermediate step should
become unnecessary.

The “Power Rule” combined with the Chain Rule. Let u = g(x) be a differen-
tiable function, and let a be any non-zero real number. By applying the Chain Rule, we
can show that
d
(g(x)a ) = a(g(x))a−1 · g 0 (x) . (128)
dx
Note the presence of the final factor g 0 (x). If we apply these combined rules to a negative
power of a function µ ¶n
1 1
= = (g(x))−n
g(x) (g(x))n
we obtain a generalization of the “Reciprocal Rule” [1, Exercise 3.2.58, p.189]:
µµ ¶n ¶ d
d 1 g(x)
= (−n)(g(x))−n−1 · g 0 (x) = (−n) · dx n+1
dx g(x) (g(x))
g 0 (x)
= (−n) · .
(g(x))n+1
A special case is
Notes for Lecture Section 002, MATH 140 2008 09 2151

The “Reciprocal Rule”.


µ ¶
d 1 d ¡ ¢ ¡ ¢
= g(x)−1 = (−1) g(x)−1−1 · g 0 (x)
dx g(x) dx
g 0 (x)
= − ,
(g(x))2
which can also be proved by94 the Quotient Rule.

Derivative of ax , where a is constant


d x d ¡ ln a ¢x
a = e
dx dx
d (ln a)x
= e
dx
= e(ln a)x · ln a
by the Chain Rule, since ln a is constant
¡ ¢x
= eln a · ln a
= ax · ln a

Example 5.32 ([7, Exercise 3.5.30, p. 224]) “Find the derivative of the function y =
sin2 x
.”
cos x
Solution: This problem could be solved by using the Quotient Rule, and by differentiating
(sin x)2 by the Product Rule and the Chain Rule:
µ ¶
d sin2 x (2 sin x · cos x) · cos x − sin2 x(− sin x)
=
dx cos x cos2 x
sin x · (2 cos2 x + sin2 x)
= ,
cos2 x
sin x · (cos2 x + 1)
which, while correct, can certainly be simplified, since cos2 x+sin2 x = 1, to ;
cos2 x
some users would want to reduce it further to sin x · (1 + sec2 x). However, it is easier to
use a trigonometric identity earlier:
µ ¶ µ ¶
d sin2 x d 1 − cos2 x
=
dx cos x dx cos x
d
= (sec x − cos x)
dx
94
and implies
Notes for Lecture Section 002, MATH 140 2008 09 2152

d d
= sec x − cos x
dx dx
= sec x · tan x − (− sin x)
= sec x · tan x + sin x .
sin x
Another approach would be to replace by tan x, and write the function as sin x ·
cos x
tan x, whose derivative may be found by the Product Rule: cos x · tan x + sin x · sec2 x,
which again admits simplification:

cos x · tan x + sin x · sec2 x = sin x + sin x · sec2 x = sin x · (1 + sec2 x) .

How to Prove the Chain Rule. You are not expected to be able to prove this
theorem. The author includes this discussion because the earlier sketch of a proposed
proof [1, p. 198], while intuitively satisfying, is flawed.

3.4 Exercises

[1, Exercise 3.4.4, p. 203] “Write the composite function in the form y = f (g(x)). ...
dy
Then find the derivative : y = tan(sin x).”
dx
Solution: The function that is applied first is sin: let’s call it g, so g(x) = sin x.
The function applied next is tan; I’ll call it f , so f (x) = tan x. Then y = f (g(x)).
By the Chain Rule,
dy
= f 0 (g(x)) · g 0 (x)
dx
= sec2 (sin x) · cos x .

Alternatively, denoting g(x) by the single letter u, we could have written

dy d
= f (u)
dx dx
du
= f 0 (u) ·
dx
= f (sin x) · g 0 (x)
0

= sec2 (sin x) · cos x .

2
[1, Exercise 3.4.24, p. 204] “Find the derivative of the function y = 101−x .”
Notes for Lecture Section 002, MATH 140 2008 09 2153

Solution: Let u = 1 − x2 . Then


d ³ 1−x2 ´ d u
10 = 10
dx dx
d u du
= 10 ·
du dx
d
= 10u · ln 10 · (1 − x2 )
dx
by (133)
2
= 101−x · ln 10 · (−2x)

eu − e−u
[1, Exercise 3.4.28, p. 204] Find the derivative of y = . This function is
eu + e−u
what we will eventually call the hyperbolic function tanh u. The differentiation can
be carried out as in Example 5.34 above.
q p √
[1, Exercise 3.4.42, p. 204] Find the derivative of x + x + x.
√ 1 d√ 1
Solution: Since y = y2, y = √ . Hence
dy 2 y
r q µ q ¶
d √ 1 d √
x+ x+ x = q p · 1+ x+ x
dx √ dx
2 x+ x+ x
à µ ¶!
1 1 d√
= q 1+ p
p √ · 2 x+ x
√ 1+
dx
x
2 x+ x+ x
à µ ¶!
1 1 1
= q p · 1+ p √ 1+ √
√ 2 x + x 2 x
2 x+ x+ x

[1, Exercise 3.4.59, p. 204] “Find the x-coordinates of all points on the curve y =
sin 2x − 2 sin x at which the tangent line is horizontal.”
Solution: We have to find all solutions x of the equation y 0 = 0, i.e.,
y 0 = cos 2x · 2 − 2 cos x . (129)
We can solve this if we replace cos 2x by 2 cos2 x − 1, an identity that you are
expected to have available:
¡ ¢
2 2 cos2 x − 1 − 2 cos x = 0 ⇔ 2 cos2 x − cos x − 1 = 0
⇔ (2 cos x + 1)(cos x − 1) = 0
Notes for Lecture Section 002, MATH 140 2008 09 2154

1
which is satisfied when (and only when) cos x is equal to either 1 or − .
2
cos x = 1 ⇔ x = 2nπ
1 2π
cos x = − ⇔ x=± + 2mπ
2 µ 3 ¶
2
⇔ x = ± + 2m π
3
where m and n are any integers. We find that the points with horizontal tangents
are evenly spaced along the x-axis, at intervals of length 2π3
. (We could also have
solved equation (129) in a simpler way, by replacing the difference of cosines by a
product of sines:
2x + x 2x − x
2(cos 2x − cos x) = −4 sin · sin
2 2
3x x
= −4 sin · sin .
2 2
The factor sin 2 vanishes precisely where x = 2nπ; the factor sin 3x
x
2
vanishes pre-
cisely where x = 2n

.)
[1, Exercise 3.4.94(a), p. 206] Write

|x| = x2 , (130)
d x
and use the Chain Rule to show that |x| = .
dx |x|
Solution: Students should understand that equation(130) is not simply a trick that
the author is proposing
√ — it could be taken as the definition of what we always
1
mean when we write x or x . If we differentiate both sides with respect to x,
2

using the Chain Rule, we obtain


d d ³¡ 2 ¢ 12 ´ 1 ³¡ 2 ¢ 12 −1 ´ d ¡ 2 ¢
|x| = x = x · x
dx dx 2 dx
³
1 ¡ 2 ¢− 12 ´
= x · 2x
2
1 1
= · · 2x
2 (x2 ) 21
1 1 x
= · · 2x =
2 |x| |x|
½
x if x ≥ 0
for x 6= 0. Alternatively, we could use a “piecewise” definition |x| =
½ −x if x < 0
1 if x > 0
to see that the derivative is .
−1 if x < 0
Notes for Lecture Section 002, MATH 140 2008 09 2155

Example 5.33 Prove that the function y = e4x + 2e−x has the property that

y 000 − 13y 0 − 12y = 0 (131)

for all x. We say that y = e4x + 2e−x is a solution of the differential equation (131).
Solution: We determine the derivatives by repeated applications of the rules of differen-
tiation, including the Chain Rule:

y = e4x + 2e−x
y0 = 4e4x − 2e−x
y 00 = 16e4x + 2e−x
y 000 = 64e4x − 2e−x

Hence
¡ ¢ ¡ ¢ ¡ ¢
y 000 − 13y 0 − 12y = 64e4x − 2e−x − 13 4e4x − 2e−x − 12 e4x + 2e−x
= 0 · e4x + 0 · e−x = 0
Notes for Lecture Section 002, MATH 140 2008 09 2156

5.13 Supplementary Notes for the Lecture of October 22nd,


2008
Release Date: Wednesday, October 22nd, 2008
corrected 12 November, 2008; subject to further correction

5.13.1 §3.4 The Chain Rule (conclusion).


Derivative of an exponential whose base is not e. Let a(x) be any positive,
differentiable function of x. Then
d d ³¡ ln a(x) ¢x ´
((a(x))x ) = e
dx dx
since exponential and logarithm are inverses
d ¡ x ln a(x) ¢
= e
dx
by the exponent rules
d
= ex ln a(x) · (x ln a(x))
dx
µ ¶
x ln a(x) d
= e · 1 · ln a(x) + x · ln a(x)
dx
by the Product Rule
µ ¶
x ln a(x) d
= e · ln a(x) + x · ln a(x)
dx
µ ¶
x d
= a(x) · ln a(x) + x · ln a(x) (132)
dx
which we cannot complete yet, because we don’t know the derivative of the logarithm.
However, when a(x) is a constant, a, then ln a(x) is also constant, and its derivative is
0; we obtain
d x
(a ) = ax · ln a . (133)
dx
In the next sections we shall show how our tools are now strong enough to determine
the derivative of the logarithm function; indeed, we are now equipped to differentiate all
of the functions we are likely to meet in this course. We will also see another way of
determining the derivative of (a(x))x , and, more generally, of (a(x))b(x) , where b is any
differentiable function; the other method, which is no stronger than the method we have
shown here, is called logarithmic differentiation, [1, p. 217-219].
Example 5.34 ([7, Exercise 3.5.28, p. 224]) “Find the derivative of the function y =
e2u
.”
eu + e−u
Notes for Lecture Section 002, MATH 140 2008 09 2157

Solution: We could apply the Quotient Rule, using the Chain Rule where necessary:
µ ¶
d e2u (e2u · 2) · (eu + e−u ) − e2u · (eu + e−u · (−1))
=
du eu + e−u (eu + e−u )2
eu + 3e−u
= e2u ·
(eu + e−u )2

Another approach would be to first divide top and bottom by e2u , thereby making the
Reciprocal Rule applicable:
µ ¶ µ ¶
d e2u d 1
=
du eu + e−u du e−u + e−3u
µ ¶2
1 ¡ −u −3u
¢
= − −u · e · (−1) + e · (−3)
e + e−3u
e−u + 3e−3u
= .
e−u + e−3u
Yet another approach would be to introduce an intermediate variable, t = eu :
 
µ 2u
¶ 2
d e d  t  dt
=  ·
u
du e + e −u dt 1  du
t+
µ 3t¶
d t dt
= 2
·
dt t + 1 du
(to simplify the rational function before differentiation)
3t2 (t2 + 1) − t3 (2t) dt
= ·
(t2 + 1)2 du
4 2
t + 3t dt
= 2 2
·
(t + 1) du
e4u + 3e2u u
= ·e .
(e2u + 1)2

Additional Problems.
x
1. Differentiate y = sin · cos 2x.
2
Solution:
Notes for Lecture Section 002, MATH 140 2008 09 2158

(a) First Solution: There are no tricks. We apply the Product Rule, and, within
it, the Chain Rule twice:
d ³ x ´ x 1 x
sin · cos 2x = cos · · cos 2x + sin · (− sin 2x) · 2
dx 2 2 2 2
x 1 x
= cos · · cos 2x − sin · (sin 2x) · 2 (134)
2 2 2
(b) Second Solution: We can apply the identity [1, 18.a p. A30] which the text-
book calls a “Product Formula”,
1
sin x · cos y = [sin(x + y) + sin(x − y)].
2
(This is an identity you will require in MATH 141.)
µ µ µ ¶ µ ¶¶¶
d ³ x ´ d 1 5 3
sin · cos 2x = sin · x + sin − · x
dx 2 dx 2 2 2
µ µ ¶ µ ¶¶
1 d 5 3
= sin · x + sin − · x
2 dx 2 2
µ µ ¶ µ ¶ µ ¶¶
1 5 5 3 3
= cos · x · + cos − · x · −
2 2 2 2 2
µ ¶ µ ¶
5 5 3 3
= cos · x − cos − · x
4 2 4 2
µ ¶ µ ¶
5 5 3 3
= cos · x − cos ·x .
4 2 4 2
The answer obtained first could also be simplified by applying the same iden-
tity. The advantage of the second method is that the functions which are even-
tually differentiated are “simpler” than those given, in that the trigonometric
functions appear only multiplied by a constant, rather than in products.
r r
x+1 x2 + 1
2. If y(x) = , and z(x) = , find y 0 (2) and z 0 (2).
x−1 x2 − 1
Solution:
(a)
r
dy d x+1
=
dx dx x−1
õ ¶1 !
d x+1 2
=
dx x−1
Notes for Lecture Section 002, MATH 140 2008 09 2159

µ ¶ 12 −1 µ ¶
1 x+1 d x+1
= · ·
2 x−1 dx x−1
µ ¶− 1
1 x + 1 2 1 · (x − 1) − (x + 1) · 1
= · ·
2 x−1 (x − 1)2
1
= − 1 3
(x + 1) 2 (x − 1) 2

0 1 3
When x = 2, y = − √ = − .
3 3
(b) Let u = x2 . Then z(x) = y(u), so
d d d
z(x) = y(u) · u
dx du dx
0 0
= y (u) · u
1
= − 1 3 · 2x
(u + 1) 2 (u − 1) 2
1
= − 1 3 · 2x
(x + 1) 2 (x2 − 1) 2
2

4 4√
so z 0 (2) = − 1 3 =− 15.
5 ·3
2 2 45

5.13.2 §3.5 Implicit Differentiation.


The techniques we have developed in the preceding sections are sufficient for determining
the derivatives of functions formed from the familiar classes of functions (polynomials,
roots, rational functions, trigonometric functions, exponential functions, exponential and
logarithmic functions, etc.) by the methods seen in [1, §1.3, New Functions From Old].
But sometimes we have to work with functions whose definitions are not explicit, that
is, where we do not have a precise formula for the function, and yet the function is still
fully determined. One way in which this happens is where the values of a function are
known to satisfy an equation, which we may not be able to solve explicitly. In such cases
we can think of the function as being implicitly defined by the equation. The Chain
Rule can often be used to find the derivative of such an implicitly defined function. It is
something of a misnomer to call the operation of differentiation in such a case implicit
differentiation: there is nothing implicit in the differentiation — it is the definition that
is implicit. When we apply differentiation in cases like these, the result is often a formula
which gives the values of the derivative of the function in terms of both the independent
variable and the function values; that is, we may have to express f 0 (x) in terms of x and
f (x), and may be unable to find an explicit formula for f 0 (x) in terms of x alone.
Notes for Lecture Section 002, MATH 140 2008 09 2160

Here’s how it works. We have already developed the Chain Rule to differentiate
a composition of functions. Where necessary we will apply the Chain Rule even if we
don’t have an explicit formula for one of the factors in the “chain”. The result will be
an equation that can be solved for an unknown derivative. I illustrate the method in the
following example.

Example 5.35 (cf. [1, Example 2, pp. 209-210]) The Folium of Descartes has equation

x3 + y 3 = 6xy . (135)

1. Find a formula for y 0 .

2. Find the tangent to the folium at the point (x, y) = (3, 3) on the curve.

3. Determine where the (tangent to the) curve is horizontal.


d2 y
4. Determine the value of when (x, y) = (3, 3).
dx2
Solution:

1. Equation (135) constrains y as a function of x, even though it does not explicitly


express y as a function of x. We will proceed to differentiate the functions of x
on both sides of the equation, and thereby obtain a constraint on y 0 . We call
this implicit differentiation with respect to y; there’s really nothing different in
the operation of differention — what’s different is that the function is known only
through the information that is implicit in equation (135). We see that

3x2 + 3y 2 · y 0 = 6(1 · y + x · y 0 ) ,

which we can solve for y 0 :


2y − x2
y0 = . (136)
y 2 − 2x
2. Note that the denominator of this fraction is non-zero at (x, y) = (3, 3), so the
solution is certainly valid at that point. We evaluate the ratio, to obtain y 0 = −1;
knowing a point on the tangent line and its slope, we can find an equation for the
line to be
y − 3 = −1(x − 3)
or x + y = 6.
Notes for Lecture Section 002, MATH 140 2008 09 2161

3. To determine the locations of the horizontal tangents, we solve the equation y 0 = 0,


which is equivalent to
2y = x2 , (137)
with the equation (135) of the curve, since the points (x, y) that we seek satisfy
conditions. We obtain, after reduction, x3 = 0, 16, which imply that
both of these √
x = 0 or x = 3 16.
√ 4
2
5 10 7
(a) When x = 3 16 = 2 3 , y = 1623 = 2 3 . Since, at that point, y 2 −2x = 2 3 −2 3 6=
0, equation (136) is applicable.
2
(b) But, when x = 0, y = 02 = 0, so the point is the origin. Determining the
behavior of tangents at (x, y) = (0, 0) is more delicate. Equation (136) is
not applicable at this point, since it was obtained on the assumption that
y 2 − x2 6= 0, and that is not the case at the origin. A more careful evaluation
of the tangents at (x, y) = (0, 0) requires considering the limit of y 0 in equation
(136) as x approaches 0, and is beyond this course; we could show that the
curve has two tangents at (x, y) = (0, 0): one is horizontal, and the other is
vertical.

4. Differentiating equation (136) again with respect to x — “implicitly”, where re-


quired — we obtain
µ ¶
00 d 2y − x2
y =
dx y 2 − 2x
(2y 0 − 2x)(y 2 − 2x) − (2y − x2 )(2yy 0 − 2x)
=
(y 2 − 2x)2

24y 0 (3) − 24
Substituting x = y = 3 yields y 00 = , where the argument of y 0 refers
9
to x = 3. Since substitution of (x, y) = (3, 3) in (136) yields y 0 (3) = −1, we have
−24 − 24 16
y 00 = − =− .
9 3

Derivatives of Inverse Trigonometric Functions Earlier in the term we investi-


gated the definitions of inverse trigonometric functions, and found that, for each of the
6 functions, we needed to restrict the domain in order that the graph would satisfy the
“Horizontal Line Rule”, and that the function would have an inverse.
We can obtain, for example, the derivative of the inverse cosine function, as follows.
We recall that the custom is to restrict the domain of the cosine to

0 ≤ x ≤ π, (138)
Notes for Lecture Section 002, MATH 140 2008 09 2162

so that
arccos(cos x) = x (139)
for all x satisfying (138), and
cos(arccos x) = x (140)
for all x in the domain of the arccosine function, i.e. for −1 ≤ x ≤ 1. Differentiating
(140) “implicitly” with respect to x, we obtain
d
− sin(arccos x) · arccos x = 1 ,
dx
so
d 1 1
arccos x = − =− 1 . (141)
dx sin(arccos x) ± (1 − cos2 (arccos x)) 2
But the values of the arccosine function are in the restricted domain of the cosine, i.e., in
(138), where the sine function is positive. Thus the ambiguous sign ± may be replaced
by +, and
d 1 1
arccos x = − 1 = −√
dx (1 − cos2 (arccos x)) 2 1 − x2
by (140). The derivatives of the other 5 inverse trigonometric functions may be deter-
mined in a similar way. Here are the derivatives:

d 1
sin−1 x = √ (142)
dx 1 − x2
d 1
cos−1 x = −√ (143)
dx 1 − x2
d 1
tan−1 x = (144)
dx 1 + x2
d 1
cot−1 x = − (145)
dx 1 + x2
d 1
sec−1 x = √ (146)
dx x x2 − 1
d 1
csc−1 x = − √ (147)
dx x x2 − 1
Notes for Lecture Section 002, MATH 140 2008 09 2163

However, the textbook’s definitions of the inverse secant and inverse cosecant functions
are non-standard, and the resulting derivatives are also non-standard. If you need to
work with either of these functions outside of this course, you must first check what
definition is in force.

Remember that the “inverse trigonometric functions” are not true inverses!
These functions are inverses of restrictions of the trigonometric functions to a portion
of their domain where the function will be invertible. When an inverse trigonometric
function is followed by the corresponding trigonometric function, the composition is
always the identity:

sin(arcsin x) = x
cos(arccos x) = x
tan(arctan x) = x
cot(arccot x) = x
sec(arcsec x) = x
csc(arccsc x) = x

wherever the inverse function is defined; i.e., for the inverse sine and cosine, for −1 ≤
x ≤ 1, for the inverse tangent and cotangent, for all real numbers, and, for the inverse
secant and cosecant, for all x such that |x| ≥ 1. But, when the functions are applied
in the reverse order, the composition maps x on to a point in the interval to which the
original function was restricted. For example,

arcsin sin x = x

only if − π2£≤ x ≤ +¤ π2 . While arcsin sin x has a meaning for all x ∈ R, its value is x only
when x ∈ − π2 , + π2 .

Derivatives of inverse functions in general. (cf. [1, Exercise 67, §3.5, p. 215])
Suppose that we know that a function f has an inverse, so that we have

y = f (x) (148)
x = f −1 (y) (149)
¡ −1 ¢
f f (y) = y (150)
f −1 (f (x)) = x (151)
Notes for Lecture Section 002, MATH 140 2008 09 2164

then we can differentiate implicitly to show that


dx 1
= (152)
dy dy
dx
µ ¶
dy
where 6= 0 , so that the derivatives appear to behave like fractions. Sometimes
dx
dy dx
we need to determine the slope of a curve, , but it is much easier to determine .
dx dy
We can determine the latter first, then simply take its reciprocal.
Example 5.36 ([7, Exercise 3.6.50, p. 234]) “Find the derivative of the function
y = arctan cos θ . (153)
Simplify where possible.”
Solution:
1. First solution, using the Chain Rule:
d 1
arctan(cos θ) = · (− sin θ) .
dθ 1 + cos2 θ
2. Second solution, using implicit differentiation: Applying the tangent function to
both sides of (153) yields
tan y = cos θ .
We differentiate both sides with respect to θ:
dy
sec2 y · = − sin θ

dy
and solve for :

dy sin θ sin θ sin θ
=− 2 =− 2
=− .
dθ sec y 1 + tan y 1 + cos2 θ
Example 5.37 ([7, Exercise 3.7.20, p. 240]) “Find the first and second derivatives of
the function y = tan−1 (x2 ).”
Solution: By the Chain Rule,
d 1 2x
tan−1 (x2 ) = 2 2
· 2x = .
dx 1 + (x ) 1 + x4
We then apply the Quotient Rule:
2(1 + x4 ) − 2x · 4x3 2 (1 − 3x4 )
y 00 = = .
(1 + x4 )2 (1 + x4 )2
Notes for Lecture Section 002, MATH 140 2008 09 2165

3.5 Exercises
dy
[1, Exercise 3.5.10, p. 213] Suppose we need to find y 0 , i.e. , where all we know is
dx
that
2
y 5 + x2 y 3 = 1 + yex . (154)
We do not have practical methods for solving this equation for y explicitly in terms
of x. But we can apply the Chain Rule to the two sides of the defining equation,
treating each term on each side as a function of x:

y 5 : We differentiate by the Power Rule combined with the Chain Rule, to obtain
d 5 dy
y = 5y 4 · ,
dx dx
dy
and make no attempt to determine at this time; indeed, this derivative
dx
is what we are trying to find, and we are in the process of determining a
constraint on it.
x2 y 3 : This term is differentiated by the Product Rule, and then one of the resulting
terms by the Power Rule combined with the Chain Rule:
d 2 3 dy
(x y ) = 2x · y 3 + x2 · 3y 2 · .
dx dx
1:
d
1=0
dx
In this case the definition is “explicit”.
2
yex :
d ³ x2 ´ dy x2 2
ye = · e + y · ex · 2x .
dx dx
Collecting the differentiated terms together, and equating the two expressions we
have found for the derivative of the function given by each of the sides of the
equation, we obtain
dy dy dy x2 2
5y 4 · + 2x · y 3 + x2 · 3y 2 · =0+ · e + y · ex · 2x .
dx dx dx
dy
We shift all terms having a factor to one side of the equation:
dx
dy dy dy x2 2
5y 4 · + x2 · 3y 2 · − · e = −2x · y 3 + y · ex · 2x .
dx dx dx
Notes for Lecture Section 002, MATH 140 2008 09 2166

dy
and solve for :
dx
³ ´
x2 2
dy −2xy 3 + ye 2xx2 2xy e − y
= 4 = .
dx 5y + x2 3y 2 − ex2 5y 4 + x2 3y 2 − ex2

[1, Exercise 3.5.50,


√ p. 214] “Find and simplify the derivative of the function f (x) =
arctan(x − 1 + x2 ).”
Solution:
µ ¶
0 1 1 2 − 12
f (x) = √ · 1 − · (1 + x ) · 2x
1 + (x − 1 + x2 )2 2
µ ¶
1 x
= √ · 1− √
1 + x2 + (1 + x2 ) − 2x 1 + x2 1 + x2
Ã√ !
1 1 1 + x2 − x
= · √ · √
2 1 + x2 − x 1 + x2 1 + x2
Ã√ !
1 1 1 + x2 − x
= ·√ ¡√ ¢· √
2 1 + x2 1 + x2 − x 1 + x2
1 1
= ·
2 1 + x2
r
1−x
[1, Exercise 3.5.54, p. 214] Find the derivative of f (x) = arctan .
1+x
Solution: Before undertaking the solution of the given problem, let’s investigate
the domain of ¯this¯ function. We can proceed in the traditional way, by solving
the inequality ¯ x−1
x+1
¯ ≥ 0, since the argument of the square root function must be
non-negative. This inequality can be multiplied by a positive number, and thereby
preserved; or by a negative number, and is thereby reversed. We obtain
½
1−x 1 − x ≥ 0 if 1 + x > 0
≥0 ⇒
1+x 1 − x ≤ 0 if 1 + x < 0
½
x ≤ 1 and x > −1 ; or
⇔ .
x ≥ 1 and x < −1

The second pair of conditions are inconsistent, but the first are equivalent to −1 <
x−1
x ≤ +1. Of course, we have to exclude the point x = −1, since the fraction
x+1

UPDATED TO December 2, 2008


Notes for Lecture Section 002, MATH 140 2008 09 2167

is undefined there. Thus the domain of f is the interval (−1, +1].


1 1 −(1 + x) − (1 − x)
f 0 (x) = ¯ ¯ r
¯1 − x¯ · ·
¯ 2 1−x (1 + x)2
1 + ¯¯
1 + x¯ 1+x
1 1 −2
= · r ·
1−x 1 − x (1 + x)2
1+ 2
1+x 1+x
1
= − √
2 1 − x2

[1, Exercise 3.5.65, p. 215] Find all points on the curve

x2 y 2 + xy = 2 (155)

where the slope of the tangent line is −1.


Solution: Implicit differentiation of (155) yields

2x · y 2 + x2 · 2y · y 0 + y + x · y 0 = 0 . (156)

While we could solve for y 0 , we need only set y 0 = −1 directly in (156), obtaining

2x · y 2 − x2 · 2y + y − x = 0 , (157)

which factorizes as
(1 + 2xy)(x − y) = 0 .
Thus the points we seek lie on at least one of the curves
1
y = − (158)
2x
y = x. (159)

They also lie on the given curve (155); we can find the points by solving each of the
preceding equations with (155). We see immediately that there are no points of
intersection on (158); but the coordinates of points of intersection on (159) satisfy
x4 + x2 = 2, i.e. (x2 + 2)(x2 − 1) = 0, so x = ±1, and y = ±1.
(The equation of the given “curve” factorizes into

(xy + 2)(xy − 1) = 0

so the “curve” actually is the union of two curves, xy = −2 and xy = 1, indeed it


has 4 branches! We could have attacked the problem by explicit differentiation of
Notes for Lecture Section 002, MATH 140 2008 09 2168

2 1
− and − , setting the derivatives equal to the given value of 1, etc. This is not
x x
always the case, but often happens in textbook examples that are chosen to have
simplified algebra. Try to solve problems for the purpose they are intended, but
then look for facts like this one to verify your work or to permit more insight into
the particular example.)

[1, Exercise 3.5.68, p. 215] 1. “Show that f (x) = 2x + cos x is one-to-one.


2. “What is the value of f −1 (1)?
0
3. “... find (f −1 ) (1)?”
Solution:

1. This question is actually premature, since a rigorous proof would need to use
the theory we will meet in [1, §4.2]. So let us look at the question intuitively.
The slope of the graph is y 0 = 2 − sin x, which can never be less than 2 − 1 =
1 > 0; thus the tangents are always sloping upwards, so the graph is always
rising as one proceeds from left to right, and there cannot be two points on
the curve having the same y-value; what we have “proved” is that the graph
satisfies the Horizontal Line Test [1, p. 60]. This proof is a bit “fuzzy”, and
we will have a better way of expressing these ideas when we reach [1, §4.2]
and consider the Mean Value Theorem.
2. We have to solve, for x, the equation 1 = y = 2x − cos x. While this would
be difficult if y were a general number, we can do it easily by inspection when
y = 1, as we can see that x = 0 has the desired property. Since the function
has an inverse, x = 0 is the only solution.
3. The equation of the graph of the given function is

y = 2x + cos x,

and, as seen above, differentiation yields


dy
= 2 − sin x
dx
dy
We just saw that, when y = 1, x = 0. We know that the value of is
dx
dx 1
2 − sin 0 = 2 at this point; it follows from (152) that = .
dy 2

Example 5.38 (cf. [1, Exercise 112, p. 264]) Show that the length of the portion of any
2 2 2
tangent line to the astroid x 3 + y 3 = a 3 cut off by the coordinate axes is constant.
Notes for Lecture Section 002, MATH 140 2008 09 2169

Solution: Let’s differentiate all terms in the given equation with respect to x. We
don’t have a formula for y as a function of x (although, in this case, we could find
2
one by solving the equation), so let’s just differentiate y 3 by the Chain Rule without
attempting to obtain a formula. (We call this implicit differentiation, since we are
defining the function y(x) “implicitly” in terms of x, rather than attempting to obtain
an explicit formula.) We obtain, as a result of that differentiation,
2 −1 2 −1 0
x 3 + y 3 ·y =0
3 3
so r
0 y
y =−3 .
x
Suppose that we are considering the tangent torthe curve at the point (x, y) = (u, v) on
v
the curve. Then the slope of the tangent is − 3 , and the equation of that tangent is
u
r
v
y − v = − 3 · (x − u) .
u
This line meets the x-axis in the point
µ r ¶
u
u+v 3
,0
v
and the y-axis in the point µ r ¶
v
0, v + u 3
u
The square of the distance between these two intercepts is
µ r ¶2 µ r ¶2
u v
u+v 3
+ v+u3
v u
³ 1 2
´ 2 ³ 1
´
2 2
= u + u3 v 3 + v + v 3 u3
2
³ 2 ´
2 2 2
³ 2 ´
2 2
= u3 u3 + v 3 + v 3 v 3 + u3
³ 2 ´
2 3
= u +v 3 3 .

But we assumed that (u, v) is a point on the curve, so


2 2 2
u3 + v 3 = a3 (160)
Notes for Lecture Section 002, MATH 140 2008 09 2170

³ 2 ´3
and the square of the distance between the intercepts is a 3 = a2 , which is, indeed,
a constant. Hence the length of the portion of the tangent line intercepted by the
coordinate axes is the constant |a|.
dy
(Sometimes it may happen that the formula we obtain for may simplify further
dx
if we use the constraint we know to hold between x and y: this is what we saw above in
equation (160).)

Orthogonality of curves; orthogonal trajectories. As defined in [1, Exercises 59-


62], two curves are orthogonal if, at any point of intersection, their tangent lines are
perpendicular (=orthogonal) — i.e., the product of their slopes is −1.

Example 5.39 ([7, Exercise 3.6.56, p. 235]) “Show that the given curves are orthogonal:

x2 − y 2 = 5 (161)
4x2 + 9y 2 = 72 .” (162)

Solution: Differentiating the first equation “implicitly” with respect to x yields


dy dy x
2x − 2y · =0⇒ = ;
dx dx y
while the second equation yields
dy dy 4x
8x + 18y · =0⇒ =− .
dx dx 9y
dy
In the preceding, the derivatives are not the same; they denote the slopes of the two
dx
curves, even if we are considering a point which is simultaneously on both curves. That
is precisely what we must do if we wish to test orthogonality, i.e., perpendicularity: we
must show that, at the points where the curves meet, their tangents are perpendicular;
or, equivalently, we must show that the products of the slopes of the two curves at a
point where they meet is −1. The product is
µ ¶µ ¶
x 4x 4x2
− = − 2.
y 9y 9y

But this does not seem to be equal to −1; where have we gone wrong? Remember that
we are considering this product of derivatives only at points that are on both curves. So
the only points we need to consider are those satisfying both of the equations. Consider
Notes for Lecture Section 002, MATH 140 2008 09 2171

a point (x, y) which is on both curves. If we solve equations (161), (162), interpreting
them as equations in the variables x2 and y 2 , we obtain

(x2 , y 2 ) = (9, 4)

from which it follows that the product of the slopes of the tangents at the points (x, y)
of intersection is
4x2 4·9
− 2 =− = −1
9y 9·4
so the tangents at points of intersection are, indeed, perpendicular.

Does the second derivative behave “like a fraction”? We have seen in (152) that
the first derivative “behaves like a fraction”, in the sense that, where a function y(x) has
an inverse,
dx 1
= .
dy dy
dx
This symbolically simple situation does not hold for higher derivatives. Let’s attempt to
d2 x
express 2 in terms of the derivatives of y with respect to x. In the equation
dy
dx dy
· =1
dy dx
we interpret each of the factors on the left side as a function of x; but, in differentiating
dx
the first factor we interpret as a function of an intermediate variable y which is a
dy
function of x; we obtain µ 2 ¶
dx dy dx d2 y
· + · = 0,
dy 2 dx dy dx2
d2 x
which we may solve for :
dy 2
d2 x dx
·
d2 x dy 2 dy
= − µ ¶2
dy 2 dy
dx
d2 y
2
= − µ dx ¶3 ,
dy
dx
Notes for Lecture Section 002, MATH 140 2008 09 2172

so the 2nd derivative does not appear to have a “reciprocal” property similar to that
possessed by the first derivative. (You are not expected to memorize this formula!)

Example 5.40 Suppose that x and y are related by the equation ex+y = xy. Determine
d2 y
.
dx2
Solution: Implicit differentiation of the given equation with respect to x yields

ex+y · (1 + y 0 ) = 1 · y + x · y 0 .

We can now proceed in two ways.

Solve for y 0 : Solving for y 0 yields


y − ex+y
y0 =
. (163)
ex+y − x
However, we could have simplified the equation before solving, since ex+y = xy:

xy · (1 + y 0 ) = 1 · y + x · y 0 . (164)

Now, when we solve, we obtain

y(1 − x)
y0 = − .
x(1 − y)

We then differentiate using the Quotient Rule:

(y 0 (1 − x) + y(−1)) · x(y − 1) + y(1 − x) · (1(1 − y) + x(−y 0 ))


y 00 =
x2 (1 − y)2

Differentiate implicitly first: As in the preceding method, first simplify the equation
to (164); then differentiate implicitly without solving for y 0 :

1 · y · (1 + y 0 ) + xy 0 · (1 + y 0 ) + xy · (0 + y 00 ) = y 0 + 1y 0 + xy 00 (165)

At this point we could solve for y 00 .

Neither of these solutions is fully simplified. For the 1st method we can further simplify
by using the information in (163):
y ¡ ¢
y 00 = · (x − 1) 2
+ (y − 1)2
.
x2 (y − 1)3
Notes for Lecture Section 002, MATH 140 2008 09 2173

3.5 Exercises (conclusion)

[1, Exercise 3.5.28, p. 213] A generalization of this problem ([1, Exercise 112, p.
264]) is solved on page 2168 above in Example 5.38.

[1, Exercise 3.5.40, p. 214] 1. “Show by implicit differentiation that the tangent
x2 y 2
to the ellipse 2 + 2 = 1 (where a and b are non-zero constants) at the point
a b
(x0 , y0 ) on the curve is
x0 x y 0 y
+ 2 = 1 .”
a2 b
µ ¶
3a 4b
2. Find an equation for the normal to the curve at the point , . (In this
5 5
context the word normal means perpendicular ; the normal to the curve at a
point is the line through the point which is perpendicular to the tangent line.)
Solution:

1. Differentiating the equation of the curve implicitly with respect to x yields


2x 2y 0
+ 2 · y = 0,
a2 b
so the slope of the tangent to the curve at the point on the curve with coor-
0 b2 x
dinates (x, y) is y = − 2 . At the point with coordinates (x0 , y0 ), the slope
ay
b 2 x0
of the tangent is, therefore, − 2 , and the equation of the tangent line is,
a y0
therefore,
b2 x0
y − y0 = − 2 · (x − x0 )
a y0
which may be rewritten as

x0 x y0 y x20 y02
+ = + 2. (166)
a2 b2 a2 b
We haven’t yet used the fact that (x0 , y0 ) lies on the curve; this implies that
these coordinates satisfy the equation of the curve, so

x20 y02
+ 2 = 1.
a2 b
When we substitute this information in equation (166), we obtain the desired
equation for the tangent line.
Notes for Lecture Section 002, MATH 140 2008 09 2174

µ ¶ 3a
3a 4b b2 ·
2. The slope of the tangent at , is y 0 = − 5 = − 3b . The slope of
5 5 4b 4a
a2 ·
5
4a
the normal will, therefore, be the negative reciprocal of this number, i.e., .
3b
The equation of the line through the given point, with this slope, will be
µ ¶
4b 4a 3a
y− = · x−
5 3b 5
i.e. 5(4ax − 3by) = 12(a2 + b2 ).
[1, Exercise 3.5.66, p. 215] Find equations of both the tangent lines to the ellipse
x2 + 4y 2 = 36 that pass through the point (12, 3).
Solution: (cf. solution to [1, Exercise 3.5.40, p. 214], above on page 2173 of these
notes) At a point (x0 , y0 ) on the curve the slope of the tangent is given by the
following relation, found by implicit differentiation:
µ ¶
dy
2x + 8y = 0,
dx
x0
so the slope of the tangent at a point (x0 , y0 ) is − . The equation of the tangent
4y0
to the curve at that point is, therefore,
x0
y = y0 − (x − x0 ) ,
4y0
which is equivalent to
x0 x + 4y0 y = x20 + 4y02 .
But, since the point (x0 , y0 )) lies on the curve x2 + 4y 2 = 36, x20 + 4y02 = 36, so the
equation found for the tangent can be simplified to read
x0 x + 4y0 y = 36 . (167)
This is the general tangent line: but its formulation depends on knowing the point
of contact of the tangent. If we impose the condition that this line pass through
the point (12, 3), we obtain the constraint 12x0 + 12y0 = 36, or simply x0 + y0 = 3.
What we have found is that the points of contact of the tangent lines through the
point (12, 3) are at the intersections given by the simultaneous solutions of the
following equations:
x20 + 4y02 = 36
x0 + y 0 = 3
Notes for Lecture Section 002, MATH 140 2008 09 2175

¡ ¢
There are two solutions: (x0 , y0 ) = (0, 3) and (x0 , y0 ) = 24
5
, − 59 ; these are the
two points of contact of the tangents to the ellipse from the point (12, 3) off the
curve. We can, therefore, find the tangents to be, respectively, (by substitution in
(167),) y = 3 and 2x − 3y = 15.
The preceding is a direct, naive solution to the problem. There exists a much more
elegant way of solving this type of problem, but I will not discuss it in this course; it was
discovered by a German mathematician named F. J. Joachimsthal, who lived between
1818 and 1861.

[1, Exercise 3.5.67, p. 215] This result is much easier to remember if one differenti-
ates the equation x(y(x)) = x implicitly with respect to x. Here I am using the
parentheses notation to simply indicate that one variable can be interpreted as a
function of the other, without giving the function and the inverse function names.
dx
Then the derivative of the right side is = 1. The left side can be differentiated
dx
(implicitly) by the Chain Rule:

d dx dy
(x(y(x)) = · .
dx dy dx
Equating these two results gives
dx 1
=
dy dy
dx
provided the denominator on the right is not zero.
(Using the other notation we have
¡ ¢ ¡ ¢ ¡ ¢0
f f −1 (x) = x ⇒ f 0 f −1 (x) · f −1 (x) = 1 etc.
Notes for Lecture Section 002, MATH 140 2008 09 2176

5.14 Supplementary Notes for the Lecture of October 27th,


2008
Release Date: Monday, October 27th, 2008
subject to correction

5.14.1 §3.6 Derivatives of logarithm functions (additional note)


3.6 Exercises
(2t + 1)3
[1, Exercise 3.6.11, p. 220] Differentiate the function F (t) = ln .
(3t − 1)4
Solution: (Added comments) In the notes for the last lecture I discussed 2 methods
for solving this problem: first by using the Chain Rule directly; second (following
the suggestions of the Student Solutions Manual) by first simplifying the logarithm.
I pointed out that the second method, as shown in the Manual, contained a techni-
cal error, in that, for certain values of t, it appeared to be applying the logarithm
function to negative numbers. As a “fix” for that, I showed that the solution could
be divided into two cases. But then some of the advantages of the second method
are lost, as the method is no longer as short as it might have been.

5.14.2 §3.6 Derivatives of logarithm functions


Since the logarithm and exponential functions are inverses of each other, we know that

eln x = x for all x > 0 (168)


and ln ex = x for allx . (169)

Analogously to our earlier determination of the derivatives of the inverses of the trigono-
metric functions, let us differentiate implicitly in (168) with respect to x:

d
eln x · ln x = 1 ,
dx
which equation we may solve for the derivative of ln x:
d 1 1
ln x = ln x = .
dx e x
More generally, if a is any positive real number, and we differentiate implicitly in

aloga x = x ,
Notes for Lecture Section 002, MATH 140 2008 09 2177

we obtain
d
aloga x ln a · loga x = 1 ,
dx
which we may solve to obtain
d 1 1
loga x = log x = .
dx a a ln a x ln a

The derivative of ln |x|. If we consider separately the domains 0 < x and x < 0, we
d 1
can show that ln |x| = for x 6= 0. Read the proof in [1, Example 3.6.6, p. 217].
dx x

Fixed base and exponent vs. variable base and exponent. Let’s consider how
we differentiate functions of the form a(x)b(x) .

1. When a(x) and b(x) are both constants, a(x)b(x) is also constant, and its derivative
is 0.

2. When b(x) is a constant b 6= 0, use the Chain Rule:

d ¡ ¢ da(x)
a(x)b = b · a(x)b−1 · ;
dx dx
d ¡ ¢ d
of course a(x)0 = 1 = 0.
dx dx
3. When a(x) is a positive constant a, and b(x) = x, we saw in equation (133), that

d x
a = ax ln a .
dx

4. When a(x) is a positive constant a, and b(x) is unrestricted, we may apply the
result just preceding to obtain (by the Chain Rule with u = b(x)),
¯
d ¡ b(x) ¢ d u ¯¯ d
a = a ¯ · u
dx du u=b(x) dx
d
= ab(x) · ln a · b(x)
dx

5. But we are even able to differentiate the most general case, if a(x) > 0:

d ¡ ¢ d ³¡ ln a(x) ¢b(x) ´
a(x)b(x) = e
dx dx
Notes for Lecture Section 002, MATH 140 2008 09 2178

d ¡ (ln a(x))·b(x) ¢
= e
dx
d
= e(ln a(x))·b(x) · ln e · ((ln a(x)) · b(x))
dx
d
= a(x)b(x) · ln e · ((ln a(x)) · b(x))
dx
µ ¶
b(x) d d
= a(x) · ln e · (ln a(x)) · b(x) + ln a(x) · b(x)
dx dx
µ ¶
b(x) 1 d d
= a(x) · ln e · · a(x) · b(x) + ln a(x) · b(x)
a(x) dx dx
µ ¶
b(x) 1 d d
= a(x) · · a(x) · b(x) + ln a(x) · b(x)
a(x) dx dx
since ln e = 1.
Warning! Do not attempt to memorize these formulæ. If you are asked to
differentiate functions of these types, we will always expect you to show your work, so it
will not be acceptable to substitute in a memorized formula, even if you could remember
it perfectly.

Logarithmic Differentiation. The last case discussed above can also be approached
as follows. First give a name to a(x)b(x) , say y = a(x)b(x) . Then take logarithms of both
sides of the equation:
ln y = b(x) · ln a(x) ,
and differentiate the resulting equation implicitly with respect to x:
1 dy d 1 d
· = b(x) · ln a(x) + b(x) · · a(x) .
y dx dx a(x) dx
dy
Now solve the equation for and express in terms of a(x) and b(x):
dx
µ ¶
dy d 1 d
=y b(x) · ln a(x) + b(x) · · a(x) .
dx dx a(x) dx
Then express this result in terms of the original function:
µ ¶
d ¡ b(x)
¢ b(x) d 1 d
a(x) = a(x) b(x) · ln a(x) + b(x) · · a(x) ,
dx dx a(x) dx
as before. You may use either the method given before, or this latter one, (which is often
called “logarithmic differentiation”).
The preceding formula should not be memorized! What you should remember
is the procedure:
Notes for Lecture Section 002, MATH 140 2008 09 2179

• Name the function.


• Take logarithms of both sides of the naming equation — provided both sides are
positive.
• Differentiate both sides implicitly with respect to the original independent variable.
• Solve for the derivative.
• Simplify, and remove reference to the name given to the function.

The number e as a limit. Let a be a real number. Then


1 ¡ ¢1
lim (1 + ax) x = lim eln(1+ax) x
x→0 x→0
³ ln(1+ax) ´
= lim e x
x→0
ln(1+ax)
lim
= ex→0 x
by the continuity of the exponential function
ln(1+ax)−ln(1+0(x))
lim
= ex→0 x−0

e dx (ln(1+ax))|x=0
d
=
e 1+ax |x=0
a
=
= ea .
In particular,
1
lim (1 + x) x = e . (170)
x→0

We may wish to express this as a limit to +∞, by defining y = x1 :


µ ¶y
a
lim 1 + = ea . (171)
y→+∞ y
Example 5.41 ([7, Exercise 3.8.18, p. 249]) “Differentiate the function
r
3u + 2
G(u) = ln .” (172)
3u − 2
Solution: In the Solutions Manual that the publisher provided instructors, the authors
wrote
1
G(u) = [ln(3u + 2) − ln(3u − 2)] (173)
2
⇒ µ ¶
0 1 3 3 −6
G (u) = − = 2 (174)
2 3u + 2 3u − 2 9u − 4
Notes for Lecture Section 002, MATH 140 2008 09 2180

The value given for the derivative was correct; but the full solution is not always correct.
3u + 2
Suppose that u < − 23 . Then both the numerator and the denominator of are
3u − 2
negative: while the fraction is positive, and the square root function is defined there,
the logarithm function is not defined at either the numerator or the denominator. One
could modify the preceding incorrect derivation as follows:
2 1
When u > , G(u) = [ln(3u + 2) − ln(3u − 2)] (175)
3 2
⇒ µ ¶
0 1 3 3 −6
G (u) = − = 2 (176)
2 3u + 2 3u − 2 9u − 4
2 1
When u < − , G(u) = [ln(−3u − 2) − ln(−3u + 2)] (177)
3 2
⇒ µ ¶
0 1 −3 −3 −6
G (u) = − = 2 . (178)
2 −3u − 2 −3u + 2 9u − 4

Thus the expression given for the derivative was correct, even though the derivation was
incorrect when u < − 23 .
1
Example 5.42 ([7, Exercise 3.8.40, p. 249]) To differentiate y = x x .
Solution:

Using logarithmic differentiation:


1 1 ln x
y = x x ⇒ ln y = · ln x =
x x
1
1 0 · x − (ln x) · 1
⇒ ·y = x
y x2
1
0 1 − ln x 1 − ln x
⇒ y =y· 2
= xx ·
x x2

Without using logarithmic differentiation:


d ³¡ ln x ¢ x1 ´
y0 = e
dx
d ³ ln x ´
= e x
dx
³ ln x ´ d µ ln x ¶
= e x ·
dx x
Notes for Lecture Section 002, MATH 140 2008 09 2181

³ ´ 1
· x − (ln x) · 1
ln x
x
= e x ·
x2
¡ ¢1 1
· x − (ln x) · 1
= eln x x · x
x2
1
· x − (ln x) · 1
1
= x ·
x x
x2
etc.

3.6 Exercises
1
[1, Exercise 3.6.28, p. 220] To differentiate f (x) = , and to find the domain
1 + ln x
of f .
Solution: The domain of f consists of all x where the denominator is defined, and
where that denominator is non-zero. The denominator is defined wherever ln x is
defined, i.e. where x > 0. However, it is 0 when ln x = −1, i.e. when x = 1e . So the
domain is µ ¶ µ ¶
1 1
0, ∪ ,∞ .
e e
µ ¶ µ ¶
d 1 −1 1
= · 0+
dx 1 + ln x (1 + ln x)2 x
−1
=
x(1 + ln x)2
Problems involving logarithms can often lead to several solutions that appear to
be different. For example, in this problem one can use the fact that 1 = ln e to
rewrite 1 + ln x as ln e + ln x, which is ln(ex).

[1, Exercise 3.6.40, p. 220] “Use logarithmic differentiation to find the derivative of
the function r
2
4 x + 1
y= .” (179)
x2 − 1
Solution: Taking logarithms in equation (179) yields

1 x2 + 1
ln y = ln .
4 x2 − 1
We observe that the numerator of the fraction is always positive; as we are taking
the 4th root of the fraction, it must be positive, so the denominator also must be
Notes for Lecture Section 002, MATH 140 2008 09 2182

positive. We may, therefore, invoke the property of logarithms [1, 2nd Law, p. 64]
that the log of a quotient is the difference of the logs:
1¡ ¡ 2 ¢ ¡ ¢¢
ln y = ln x + 1 − ln x2 − 1 .
4
Differentiating implicitly with respect to x in the last equation yields
µ ¶
1 0 1 1 1
·y = · 2x − 2 · 2x
y 4 x2 + 1 x −1
−x
=
(x + 1)(x2 − 1)
2

implying that
−x
y0 = y · 2
(x + 1)(x2 − 1)
3 5
= −x(x2 + 1)− 4 (x2 − 1)− 4 .

3.6 Exercises
(2t + 1)3
[1, Exercise 3.6.11, p. 220] Differentiate the function F (t) = ln .
(3t − 1)4
Solution:

1. Method 1: Using the Chain Rule immediately

1 3(2t + 1)2 · 2 · (3t − 1)4 − (2t + 1)3 · 4(3t − 1)3 · 3


F 0 (t) = ·
(2t+1)3
(3t−1)4
((3t − 1)4 )2
1 6(3t − 1) − 12(2t + 1) 6(t + 3)
= · =−
2t + 1 3t − 1 (2t + 1)(3t − 1)

2. Method 2: Simplifying the logarithm before differentiating. (The solution in


the Student Solutions Manual has an error, which does not affect the final
answer.)
Since the function is a logarithm of a quotient, we can use the fact that
(2t + 1)3
F (t) = ln
(3t − 1)4
= ln (2t + 1)3 − ln (3t − 1)4 .

At this point we could differentiate by the Chain Rule. However, as the


logarithm is applied to a power of a linear function, there is a temptation
Notes for Lecture Section 002, MATH 140 2008 09 2183

to use a simple property of the logarithm. The denominator is a 4th power,


so it is always positive. The fraction must be positive in order that we may
find its derivative; hence the numerator must also be positive. The function
(2t + 1)3 is a cube; in order that it be positive, 2t + 1 must be positive, so we
must assume that t > − 21 for the problem to make sense. There is no harm in
replacing ln (2t + 1)3 by 3 ln(2t + 1). However, the situation is more delicate
with the denominator. While (3t−1)4 is positive, we cannot be sure that 3t−1
is positive. If t > 13 , we may replace ln (3t − 1)4 by 4 ln(3t − 1); but, if t < 13 ,
we should replace ln (3t − 1)4 = ln (1 − 3t)4 by 4 ln(3t − 1) — the logarithm
function is not defined when the argument is negative! Differentiating, I obtain


 d
 (3 ln(2t + 1) − 4 ln(3t − 1)) if t > 13
0 dt
F (t) =

 d
 (3 ln(2t + 1) − 4 ln(1 − 3t)) if t < 13
dt
 µ ¶
 2 3

 3 · 2t + 1 − 4 · 3t − 1 if t > 13
= µ ¶

 2 −3
 3· −4· if t > 13
2t + 1 1 − 3t
2 3 1
= 3· −4· whenever t 6=
2t + 1 3t − 1 3
t+3
= −6 · .
(2t + 1)(3t − 1)
Thus both of the alternatives yield the same final answer. (But technically it
would be incorrect to apply the logarithm function to a negative number.)
There is another way to “fix” the solution in the Manual. That is to use the
d 1
fact that ln |x| = . With this device the solution becomes
dx x
d
F 0 (t) = (3 ln(2t + 1) − 4 ln |3t − 1|)
dt
µ ¶
2 3
= 3· −4·
2t + 1 3t − 1
t+3
= −6 · .
(2t + 1)(3t − 1)

5.14.3 §3.7 Rates of Change in the Natural and Social Sciences


Because MATH 140 is populated by students from many different specialties, it is not
practical to consider specific applications here. You should read the pages that pertain to
Notes for Lecture Section 002, MATH 140 2008 09 2184

your own area, but the jargon of your subject will not be considered part of this course.
In fact, the applications are mostly of the same mathematical types. Because there are
some physical problems in the problem sets supplied to this course from WeBWorK, I
may consider some simple dynamical problems involving the motion of particles, e.g.,
the first example below.

Physics.
Example 5.43 ([7, Exercise 3.7.50, p. 241]) This problem asks that you consider a
t
particle moving along the x-axis, with position x at time t ≥ 0 given by x(t) = .
1 + t2
You are asked to find a formula for the acceleration, and to determine when the particle
is speeding up and slowing down.
Solution:
1 · (1 + t2 ) − t · (0 + 2t)
x0 (t) =
(1 + t2 )2
1 − t2
= (180)
(1 + t2 )2
−2t · (1 + t2 )2 − (1 − t2 ) · 2(1 + t2 )2t
x00 (t) =
(1 + t2 )4
−2t(3 − t2 )
= (181)
(1 + t2 )3
To analyze when the particle is “speeding up”,¯we first¯ need to ask what is meant by speed .
¯d ¯
This concept was defined on [1, p. 148] to be ¯¯ x(t)¯¯, the absolute value of the velocity.
dt
The particle is “speeding up” when the first derivative of this function is increasing;
equivalently, when the derivative of the first derivative is positive; equivalently, when
(t + 1)(t − 1)
the second derivative is positive. We know from (180) that x0 = ; as the
(1 + t2 )2
denominator is always positive, the fraction is positive when either 0 or 2 of the factors
in the numerator are positive. This occurs either when t < −1 or when t > 1. As we are
told to consider only non-negative t, this yields


 −(t + 1)(t − 1)
 2 )2
= x0 for 0 < t < 1
|x0 | = (1 + t (182)

 (t + 1)(t − 1) 0
 = −x for 1 < t
(1 + t2 )2
√ √
00 −2(t + 3)(t − 3)
We know from (181) that x = ; knowing where x0 (t) is positive and
(1 + t2 )3
Notes for Lecture Section 002, MATH 140 2008 09 2185

negative, we can infer from this statement that


 √ √

 00 2t(t + 3)(t − 3)
 x = for 0<t<1
d 0 (1 + t2 )3
|x | = √ √ (183)
dt 
 00 2t(t + 3)(t − 3)
 −x = − for 1<t
(1 + t2 )3

(We would have to examine the formulæ more carefully to show that the function has
no derivative
√ at t =
√ 1.) The functions have a numerator which is a constant multiple of
t, t − 3, and t + 3. At time t = 0 the particle starts from x(0) = 0 and moves to the
right (i.e., in the positive direction). The initial speed is 1, and this speed decreases as
t → 1− . When t = 1, the particle has arrived at x(1) = 21 , its speed has decreased to

0. As t increases from 1 to 3, the velocity becomes √ negative, and the particle
√ moves
to the left at an increasing speed, until time t = 3, when it is at position 43 , and it is
moving to the left at the rate of 18 . At that point in time the speed stops increasing and
starts to decrease again, although the particle does not change its direction of motion.
It continues to move to the left, now at decreasing speed, approaching — but never
reaching — the origin, which is its limiting position as t → ∞.

Chemistry. (OMIT)

Biology. (OMIT)

Economics. (OMIT)

A Single Idea, Many Interpretations. (OMIT)

3.7 Exercises

[1, Exercise 3.7.10, p. 231] If a ball is thrown vertically upward with a velocity of 80
feet per second, then its height after t seconds is

s(t) = 80t − 16t2 . (184)

1. What is the maximum height reached by the ball?


2. What is the velocity of the ball when it is 96 feet above the ground, on its
way up? ...on its way down?
Notes for Lecture Section 002, MATH 140 2008 09 2186

95
Solution:

1. To determine the maximum height. One method would be to examine the


function s(t). If we express the given formula
³ as s(t) ´ = −16(−5t + t2 ), we
¡ ¢ 2
can complete the square: s(t) = −16 t2 − 5t + 25 + 16 · 25
4
= 100 −
¡ ¡ ¢¢2
4 t − 25 = 100 − (4t − 10)2 . The subtracted term is a square, so it cannot
be negative; s(t) will be greatest when the subtracted term is as small as
possible, i.e. 0, i.e. where t = 52 . When t = 52 , s(t) = 100: this is the
maximum height. No calculus is needed!
2. The velocity is dtd s(t) = 80 − 32t; but this is given in terms of t, not in terms
of position s. When s = 96, t satisfies the equation 96 = 80t − 16t2 , so t = 2
or t = 3.

s0 (2) = 80 − 64 = 16
s0 (3) = 80 − 96 = −16

As the positive direction has been taken to be upward, height s = 96 is


attained when t = 2, when the velocity is 16 ft/sec upward; and when t = 3,
on the downward trip, when the velocity is again 16, but this time directed
in the negative, i.e. downward, direction.

5.14.4 §3.8 Exponential Growth and Decay (OMIT)


5.14.5 §3.9 Related Rates.
In this section we consider problems where there are multiple related variables, but
usually just one constraint holding between them. Typically we need to relate the rates
of change with respect to one of these variables, or with respect to a new variable that
isn’t directly involved in the constraint (e.g., with respect to time). The problems are
usually presented in a verbal form, and so the first step of solution requires translation
into a mathematical statement to which we may apply the tools we have available from
the calculus. When a function is spoken about, it may be advisable to attempt to specify
the domain; which will often — but not always — be the “maximal” domain. There are a
number of “typical” questions that calculus books like to ask; of course, you can prepare
yourself for those types by working extensive examples. These “typical” questions involve
95
This problem could have been made more difficult by requiring that you determine the “equation
of motion” from the initial velocity and the initial position. But the equation of motion is given to
you in (184). The determination of this equation would be premature, as it would depend on a result
in [1, §4.2] not yet proved; but the textbook occasionally does ask you to proved results of this type
prematurely.
Notes for Lecture Section 002, MATH 140 2008 09 2187

the basic issues, but have become typical over the years because they entail a practical
level of difficulty, or involve concepts that students like you are able to identify with;
however, the equations that one obtains using the Chain Rule are normally “linear” —
of a form where the various derivatives are related by a sum of the form
function × derivative + function × derivative +...=0
and that means that it is normally possible to solve for the derivative that interests us in
terms of the other derivatives. Thus there is potentially a very wide range of problems
that can be formulated, and, in theory, you should be able to solve all of them! It is
possible that you will be asked a problem different from any you have seen, so you should
try to understand the underlying issues. The textbook contains 5 worked examples of
different types, and I will not have time to work all of these types in the lectures. Do
not assume that, because a particular type does not appear in the worked examples, or
does not appear in a WeBWorK assignment, that it is not a reasonable question for
Math 140.
Some of the problems in your textbook require familiarity with basic formulæ of
mensuration; e.g., with the formulæ for areas and volumes of familiar geometric figures.
While we will be developing some of these formulæ in MATH 141, you are expected to
know them. Unless the textbook routinely states such a formula in its problems, then
that’s a formula that you should know.

3.9 Exercises
[1, Exercise 3.9.16, p. 245] “A spotlight on the ground shines on a wall 12 m away.
If a man 2 m tall walks from the spotlight toward the wall, at a speed of 1.6 m/s,
how fast is the length of his shadow on the building decreasing when he is 4 m
from the building?”
Solution: It is usually good practice to draw a sketch of whatever situation is being
described; (I shall not do that in these notes because the software I am using is
very cumbersome and time-consuming to create even simple figures).
Since I don’t have a figure to rely on, I am forced to do what you should always
do: to define precisely every symbol that is used.96 I will place the spotlight at the
origin, and denote the foot of the wall by W ; I take the ground to be along the
positive x-axis, and so the coordinates of W are (12, 0). The top of the shadow
will be the point S, and I define its height as h, so its coordinates are (12, h). Let
the feet of the walking man be at A(x, 0), and the top of the man’s head B(x, 2).
dh dx
The problem is to determine − , when we know .
dt dt
96
In these notes I have used different labels from those that I used during the lecture.
Notes for Lecture Section 002, MATH 140 2008 09 2188

Triangles SOW and BOA are similar, and so the lengths of their sides must be
proportional. This implies that

h 12 |OS|
= = .
2 x |OB|

From these equations, of which we are using only one, we infer that
24
h= .
x
Differentiating all members of this equation with respect to t, we obtain
µ ¶
dh d 24 24 dx
= =− 2 · .
dt dt x x dt

We now have the relationship we need in general; we apply it when |AW | = 4, i.e.,
when x = 12 − 4 = 8.
dh 24
= − 2 · (1.6) = −0.6 m/s .
dt 8
The negative sign of the answer indicates that the top of the shadow is moving
in the negative direction, i.e., downwards. As the problem requested the rate at
which the length is decreasing, that will be +0.6 m/s.
Notes for Lecture Section 002, MATH 140 2008 09 2189

5.15 Supplementary Notes for the Lecture of October 29th,


2008
Release Date: Wednesday, October 29th, 2008
subject to correction

5.15.1 §3.7 Rates of Change in the Natural and Social Sciences


Because MATH 140 is populated by students from many different specialties, it is not
practical to consider specific applications here. You should read the pages that pertain to
your own area, but the jargon of your subject will not be considered part of this course.
In fact, the applications are mostly of the same mathematical types. Because there are
some physical problems in the problem sets supplied to this course from WeBWorK, I
may consider some simple dynamical problems involving the motion of particles, e.g.,
the first example below.

Physics.

Example 5.44 ([7, Exercise 3.7.50, p. 241]) This problem asks that you consider a
t
particle moving along the x-axis, with position x at time t ≥ 0 given by x(t) = .
1 + t2
You are asked to find a formula for the acceleration, and to determine when the particle
is speeding up and slowing down.
Solution:

1 · (1 + t2 ) − t · (0 + 2t)
x0 (t) =
(1 + t2 )2
2
1−t
= (185)
(1 + t2 )2
−2t · (1 + t2 )2 − (1 − t2 ) · 2(1 + t2 )2t
x00 (t) =
(1 + t2 )4
−2t(3 − t2 )
= (186)
(1 + t2 )3

To analyze when the particle is “speeding up”,¯we first¯ need to ask what is meant by speed .
¯d ¯
This concept was defined on [1, p. 148] to be ¯¯ x(t)¯¯, the absolute value of the velocity.
dt
The particle is “speeding up” when the first derivative of this function is increasing;
equivalently, when the derivative of the first derivative is positive; equivalently, when
(t + 1)(t − 1)
the second derivative is positive. We know from (185) that x0 = ; as the
(1 + t2 )2
Notes for Lecture Section 002, MATH 140 2008 09 2190

denominator is always positive, the fraction is positive when either 0 or 2 of the factors
in the numerator are positive. This occurs either when t < −1 or when t > 1. As we are
told to consider only non-negative t, this yields

 −(t + 1)(t − 1) = x0 for 0 < t < 1


|x0 | = (1 + t2 )2 (187)

 (t + 1)(t − 1) 0
 = −x for 1 < t
(1 + t2 )2
√ √
00 −2(t + 3)(t − 3)
We know from (186) that x = ; knowing where x0 (t) is positive and
(1 + t2 )3
negative, we can infer from this statement that
 √ √

 2t(t + 3)(t − 3)
d 0  x00 = 2 3
for 0 < t < 1
|x | = (1 +
√ t ) √ (188)
dt 
 2t(t + 3)(t − 3)
 −x00 = − for 1 < t
(1 + t2 )3

(We would have to examine the formulæ more carefully to show that the function has
no derivative
√ at t =
√ 1.) The functions have a numerator which is a constant multiple of
t, t − 3, and t + 3. At time t = 0 the particle starts from x(0) = 0 and moves to the
right (i.e., in the positive direction). The initial speed is 1, and this speed decreases as
t → 1− . When t = 1, the particle has arrived at x(1) = 21 , its speed has decreased to

0. As t increases from 1 to 3, the velocity becomes √ negative, and the particle
√ moves
3
to the left at an increasing speed, until time t = 3, when it is at position 4 , and it is
moving to the left at the rate of 18 . At that point in time the speed stops increasing and
starts to decrease again, although the particle does not change its direction of motion.
It continues to move to the left, now at decreasing speed, approaching — but never
reaching — the origin, which is its limiting position as t → ∞.

Chemistry. (OMIT)

Biology. (OMIT)

Economics. (OMIT)

A Single Idea, Many Interpretations. (OMIT)


Notes for Lecture Section 002, MATH 140 2008 09 2191

3.7 Exercises

[1, Exercise 3.7.10, p. 231] If a ball is thrown vertically upward with a velocity of 80
feet per second, then its height after t seconds is

s(t) = 80t − 16t2 . (189)

1. What is the maximum height reached by the ball?


2. What is the velocity of the ball when it is 96 feet above the ground, on its
way up? ...on its way down?
97
Solution:

1. To determine the maximum height. One method would be to examine the


function s(t). If we express the given formula
³ as s(t) ´ = −16(−5t + t2 ), we
¡ ¢ 2
can complete the square: s(t) = −16 t2 − 5t + 25 + 16 · 25
4
= 100 −
¡ ¡ ¢¢2
4 t − 25 = 100 − (4t − 10)2 . The subtracted term is a square, so it cannot
be negative; s(t) will be greatest when the subtracted term is as small as
possible, i.e. 0, i.e. where t = 52 . When t = 52 , s(t) = 100: this is the
maximum height. No calculus is needed!
2. The velocity is dtd s(t) = 80 − 32t; but this is given in terms of t, not in terms
of position s. When s = 96, t satisfies the equation 96 = 80t − 16t2 , so t = 2
or t = 3.

s0 (2) = 80 − 64 = 16
s0 (3) = 80 − 96 = −16

As the positive direction has been taken to be upward, height s = 96 is


attained when t = 2, when the velocity is 16 ft/sec upward; and when t = 3,
on the downward trip, when the velocity is again 16, but this time directed
in the negative, i.e. downward, direction.

5.15.2 §3.9 Related Rates (conclusion).


3.9 Exercises (continued)
97
This problem could have been made more difficult by requiring that you determine the “equation
of motion” from the initial velocity and the initial position. But the equation of motion is given to
you in (189). The determination of this equation would be premature, as it would depend on a result
in [1, §4.2] not yet proved; but the textbook occasionally does ask you to proved results of this type
prematurely.
Notes for Lecture Section 002, MATH 140 2008 09 2192

dy dx
[1, Exercises 3.9.8, p. 245] If x2 + y 2 = 25 and = 6, find when y = 4.
dt dt
Solution: Differentiating implicitly with respect to t in the equation of the circle
gives
dx dy
2x · + 2y · = 0.
dt dt
dy
Substituting y = 4 and = 6 yields
dt
dx
2x · + 2(4)6 = 0 ,
dt
dx
so x · = −24. However, for the given value of y, x2 = 25 − 16 = 9, so x = ±3.
dt
dx
If follows that there are 2 different values of the derivative we seek: = ∓8,
dt
according as x = ±3.

[1, Exercises 3.9.10, p. 245] A particle moves along the curve y = 1 + x3 . As it
reaches the point (2, 3), the y-coordinate is increasing at a rate of 4 cm/s. How
fast is the x-coordinate of the point changing at that instant?
Solution: Differentiating the equation of the curve implicitly with respect to time
dx dy
t yields a relationship between and valid at any instant:
dt dt
dy 1 ¡ ¢ 1 −1 dx
= · 1 + x3 2 · 3x2 · .
dt 2 dt
√ √ dy
When x = 2 (and y = 1 + 23 = 9 = 3), and = 4, this equation becomes
dt
1 1 dx 1 1 dx
4= ·√ · 3(22 ) · = · · 3(4) ·
2 1 + x3 dt 2 3 dt

dx
which implies that = 2 cm/s.
dt
[1, Exercise 35, p. 247] “Two sides of a triangle have lengths 12 m and 15 m. The
angle between them is increasing at a rate of 2 degrees/minute. How fast is the
length of the third side increasing when the angle between the sides of fixed length
is 60 degrees.”
Solution: To solve this problem you need to know a relationship between the angle
and the lengths of the sides. This relationship has not been stated in the problem,
Notes for Lecture Section 002, MATH 140 2008 09 2193

but is in the elementary trigonometry that you are assumed to have brought to the
course from your precalculus or earlier background. The simplest way to express
this relationship is by the Law of Cosines, which is given in the textbook [1,
Exercise 83, Appendix D, p. A33]:

If a triangle has sides with lengths a, b, c, and C is the angle between the
sides with lengths a and b, then

c2 = a2 + b2 − 2ab cos C .

In the present problem, the relationship is given by

c2 = 122 + 152 − 2 · 12 · 15 cos C

so
c2 − 144 − 225
cos C = − ,
360
but we should precede the invocation of this equation by a statement defining
what c and C are in our discussion, as we should never use a symbol that isn’t
mentioned in the problem without first defining it. (This is a rule that we don’t
have a chance to remind you about when you work problems on WeBWorK.)
When we differentiate either of the last 2 equations with respect to time, we will
obtain the relationship we need. Prior to doing that, if we wish to denote time by
a symbol, we need to define it. So we begin with

Let t denote time, measured in seconds.98

Now, differentiating, say, the latter of the two equations, we obtain


dC 2c dc
− sin C · =− · ,
dt 360 dt
which we may solve to yield
dc sin C dC
= 2c · ,
dt 360
dt

dC
which expresses the desired rate of change in terms of sin C, , and c. But we
dt
don’t know c yet; so we will have to apply the law of cosines to express c in terms
98
Strictly speaking, we should always indicate the orientation of the coordinate axis when we describe
a new variable; as time is normally measured from the past to the future, we will dispense with that
pedantic statement.
Notes for Lecture Section 002, MATH 140 2008 09 2194

of things we know, and substitute that formula to obtain


dc sin C dC
= √ ·
dt 2 122 + 152 − 2 · 12 · 15 cos C dt
360
sin π3 dC
= p π
·
2 12 + 15 − 2 · 12 · 15 cos 3 dt
2 2

360
30 dC
= √ ·
7 dt
But there is still a snag! We are given that “The angle between (the sides) is
increasing at a rate of

2 degrees/minute.”

dC
Why can’t we simply substitute 2 as the value of ? Because our convention is
dt
that angles are always measured in radians! We have chosen to be consistent about
this, because the alternative would be to have 2 sets of trigonometric functions,
with conversion tables. So we have to recall that
2
2◦ = · 2π radians
360
dC π
so = radians/minute; substituting this value, we obtain
dt 90
dc π
= √ metres/minute.
dt 3 7
While it is always possible for a nasty examiner to mischievously insert a trap in a
verbally stated problem, by an inconspicuous unit change, the distinction between
degrees and radians is more serious than that, because the basic differentiation
rules that we have developed depend on the use of radian measure. Be careful!
A word about notation. We often use letters like c and C to denote constants; but, when
you are in control, you are free to use any symbol for any purpose, and here both of these
letters represent variables, following an old convention in trigonometry that the angle is
named with a capital letter and the opposite side with the corresponding small letter.
Of course, it might be unwise to use a letter like d for a variable, since we might wish to
use it in representing a derivative.
Notes for Lecture Section 002, MATH 140 2008 09 2195

[1, Exercise 3.9.44, p. 247] “The minute hand on a watch is 8 mm long, and the hour
hand is 4 mm long. How fast is the distance between the tips of the two hands
changing at 1 o’clock?”
Solution: This is the last problem in the set of exercises, and might be assumed
from its location to be more challenging than the earlier problems. But, once
all the facts are expressed symbolically, it is only moderately difficult. It does,
however, have one distinction: there are two different rates that are known, in
terms of which a third needs to be determined. There is quite a bit of work that
needs to be done in managing definitions, etc., before the relevant mathematical
formulæ are available; once that has been done, the computations are simple and
straightforward.
Let’s first set up a coordinate system. It appears natural to locate the origin at
the centre of the watch, and to orient the watch so that 12 o’clock is on the y-axis,
that 3 o’clock is on the x-axis, and that the units are chosen to be mm, so that the
tip of the minute hand rotates on the circle x2 + y 2 = 82 , and the hour hand on the
circle x2 + y 2 = 42 . The problem involves a rate with respect to time. I will denote
time by t, measured in hours. The hour hand on the clocks we use completes a
cycle in 12 hours; the minute hand in 1 hour. The problem is to determine the
rate of change of distance between the tips of the hands: let’s denote the position
of the tip of the hour hand by H, and of the minute hand by M ; since the motion
of these hands is linked to a constant rate of rotation, it is convenient to name
the angles between the hands and the positive x-axis: call the angle made by HO
h, measured in radians, and the angle made by M O m, also measured in radians;
both h and m are functions of time — the hour hand moves at a constant rate,
completing a revolution in 12 hours, while the minute hand, moving at a faster but
constant rate, completes a revolution in 1 hour. Note that we are not following the
usual convention for clocks, where an angle is measured from the y-axis in what
we would consider the negative direction — clockwise. Then we know that the
coordinates of H are (4 cos h, 4 sin h), and those of M are (8 cos m, 8 sin m). The
distance between H and M is given by
p
|HM | = (4 cos h − 8 cos m)2 + (4 sin h − 8 sin m)2
However, we can use the Law of Cosines to express the distance in terms of the
difference of the respective angles:
p
|HM | = 42 + 82 − 2 · 4 · 8 cos(h − m) .
It follows that
d|HM | 1 1 d ¡ 2 ¢
= ·p · 4 + 82 − 2 · 4 · 8 cos(h − m)
dt 2 42 + 82 − 2 · 4 · 8 cos(h − m) dt
Notes for Lecture Section 002, MATH 140 2008 09 2196

µ ¶
1 1 d
= ·p · −2 · 4 · 8 (cos(h − m))
2 42 + 82 − 2 · 4 · 8 cos(h − m) dt
µ ¶
1 1 d
= ·p · 2 · 4 · 8 sin(h − m) · (h − m)
2 42 + 82 − 2 · 4 · 8 cos(h − m) dt
µ µ ¶¶
1 1 dh dm
= ·p · 2 · 4 · 8 sin(h − m) · − .
2 42 + 82 − 2 · 4 · 8 cos(h − m) dt dt

And what do we know? We know that


dh 2π
= − ,
dt 12
dm
= −2π ,
dt
π
h = ,
3
π
m = .
2
Hence
d|HM | 22π
=− p √ = −18.5896...
dt 3 5−2 3
approximately. At 1 o’clock the distance between the tips of the hands is decreasing
at the rate of 18.5896... mm/hour.

5.15.3 §3.10 Linear Approximations and Differentials


In this section we are interested in determining convenient approximations to a function
f . Our approximations will always be associated with a specific point in the domain of
f . For an approximation to be useful we need to know how close it is to the correct
value; or, what is the worst that the error can be within a given interval around the
point where the approximation is made. Usually the quality of the approximation —
measured by the relative size of the maximum error — deteriorates as we move farther
from the point which is the “centre” of the approximation.

0th degree approximations. Suppose that we take, as an approximation to the


function f near the point x = a in its domain, the constant function f (a). Then the
error at x = a is 0; but, without more information about the function, we can say little
about the size of the error away from x = a. Nevertheless, approximation by a constant
is often a useful, crude approximation.
Notes for Lecture Section 002, MATH 140 2008 09 2197

1st degree approximations. A next stage of approximation is to approximate a


function f by one whose graph is still a straight line, but where we choose the line to
better match the graph of f . One way of doing this is by using the tangent line to the
graph of f at x = a. This type of approximation is called a linear approximation, or
a tangent line approximation, or the linearization of f . A linear function is one of the
form kx + `,99 and what we are doing is to choose the constants k and ` to “fit” the curve
best. To make the approximating curve pass through the point (a, f (a)), we impose the
condition that
ka + ` = f (a) ,
but this single equation is not enough to determine both of the constants. For a tangent
line approximation the second constraint we impose is that the approximating line should
“touch” the curve at the point (a, f (a)); we require that its slope be the same as the
slope of the tangent line; in fact, we are requiring that the approximating line be the
tangent line. The constraint is *
k = f 0 (a) ,
which implies that ` = f (a) − af 0 (a), and that the approximating function, which we
may denote by L(x), is
L(x) = f (a) + f 0 (a) · (x − a) .

Higher order approximations. Could we improve the approximation by finding a


curve that fits the curve better? If f is differentiable more than once, i.e., if f 00 exists,
we can approximate better with a polynomial of degree 2, i.e., a function of the form
kx2 + `x + m. We will not pursue this possibility at this time, but may return to it at
the end of Math 141.

Why do we need approximations, anyhow? Sometimes an approximation is such


that, when we replace the actual function by the approximation in an important equation,
the problem changes from one that is very difficult to solve to one that is amenable. Your
textbook gives examples of this.

Applications to Physics (OMIT). One interesting example is in studying the mo-


tion of a “simple” pendulum. After determining the differential equation that describes
the motion of the pendulum, we simplify it by replacing a term sin θ by θ, where
θ is a small angle, measured in radians; this seems reasonable, since we know that
sin x
lim = 0. In terms of linear approximation we are replacing the graph of the sine
x→0 x

99
The word linear is sometimes used in a more restrictive way, to describe a function of the form kx,
i.e., where ` = 0; when an author uses the word linear in that sense, then she usually calls a function
kx + ` affine. You could meet this version of the terminology in your Linear Algebra course.
Notes for Lecture Section 002, MATH 140 2008 09 2198

curve near θ = 0 by the graph of its tangent line: the slope is cos 0 = 1, so the lineariza-
tion is
sin θ ≈ sin 0 + cos 0 · θ = θ .
(Remember that the angle θ must be given in radians; if we wish to work in degrees,
sin x
then the limit of will not be 1, and L(x) will take a different form.)
x

Differentials.

Increments. The notation here can be a little confusing. The prefixing of the
symbol for a variable with a Greek upper case (capital) Delta (∆), normally represents
the increment in the variable; when x is the independent variable — the variable over
which we have control, the variable which is restricted only by the domain of the function
— we may denote any change in its value by ∆x. In the days before the language of
functions was formalized, mathematicians used to talk of a dependent variable y, where
we today would speak of a function y(x). Then the change in y(x) corresponding to a
change ∆x in x might be denoted by ∆y; in our notation, we would be defining
∆(y) = y(x + ∆x) − y(x) .
dy
We give meanings to dx and dy, so that, with these definitions, the fraction will
0
dx
actually be equal to f (x). We define dx to simply be any real number, although we are
not going to use this formalism for “large” numbers; the interpretation we wish to make
is that dx is to denote the increment: thus we are taking dx to be ∆x. Then we define
dy to be a function of x and of dx also; more precisely, we define
dy = f 0 · dx
which depends on two variables — on x, and also on the value we assign to dx. We will
not dwell on this issue in Math 140, because the concept of functions of more than one
variable is not introduced in our calculus sequence until Calculus 3 (Math 222). Hitherto
I have been saying that the derivative “behaves like a fraction”; with these definitions
you are free to actually consider it a fraction; it is not simply a convenient notation.

Errors. If, as an approximation to the true value of a function f at a point x = a, we


use a number A, then
error = A − f (a)
A − f (a)
relative error =
f (a)
percentage error = relative error × 100%
Notes for Lecture Section 002, MATH 140 2008 09 2199

5.16 Supplementary Notes for the Lecture of November 03rd,


2008
Release Date: Monday, November 03rd, 2008
subject to correction

5.16.1 §3.10 Linear Approximations and Differentials (conclusion)


Be careful when you choose variables!

Example 5.45 ( [1, Review Exercise 105, p. 264]) “A window has the shape of a square
surmounted by a semicircle. The base of the window is measured as having width 60
cm, with a possible error in measurement of 0.1 cm. Use differentials to estimate the
maximum error possible in computing the error of the window.”
Solution: We need some named variables. Let’s denote the side of the square by x. Then
x
the semicircle has radius . This makes the whole exercise involve fractions. So let’s
2
go back to the drawing board, and name the side of the square 2X; the radius of the
semicircle is X, and the area of the entire window, which we will denote by A(X) is
1
A(X) = (2X)2 + · πX 2 .
2
(Note that the remedy to eliminate fractions didn’t work — we would have to rename
the variable again if we wish to do that.) Differentiating the last formula yields

dA = 2(2X)2 dX + πX dX = (8 + π)X dX .

When dX = ±0.1, and X = 30, dA = . . .. WAIT!


¡ It was
¢ dx that was equal to ±0.1,
dX = ±0.05; then dA = ±(8 + π)30(0.05) = ± 12 + 3π
2
.

Using ingenuity to improve an approximation. Sometimes one can get “better”


results with a mathematical tool by applying it more carefully.

Example 5.46 Suppose we wish to approximate cos x for − π3 ≤ x ≤ π3 . A naive


application of the present theory would approximate with the linear function

cos 0 + (− sin 0)(x − 0) = cos 0 = 1

so the linear approximation is the same as the 0th degree approximation: we would be
approximating the graph of the cosine function by the horizontal line which is tangent to
cos x − 1
the cosine graph at (0, 1). Now recall that we have proved that lim = 0. Let’s
x→0 x
Notes for Lecture Section 002, MATH 140 2008 09 2200

cos x − 1
try to approximate the function linearly. Since 0 is not in the domain of this
x
function, but since its limit as x → 0 exists, the discontinuity is removable. We define

 cos x − 1 if x 6= 0
f (x) = x .

0 if x = 0

This function is continuous at all points x. In order to find a first order approximation,
we will need its derivative. This is no problem for x 6= 0, since
−(sin x)x − (cos x − 1)1 sin x 1 − cos x
f 0 (x) = = − + .
x2 x x2
But, to approximate around x = 0, we will need to know f 0 (0), and we don’t even know
whether f is differentiable there. Consider the ratio
µ ¶
f (x) − f (0) cos x − 1 1 − 2 sin2 x2 − 1 1 sin x2 2 1
= 2
= 2
=− · x →−
x−0 x x 2 2
2

as x → 0. Thus a first approximation of f around x = 0 is


µ ¶
1
f (x) ≈ 0 + x · −
2
which implies that
x2
cos x ≈ 1 −
2
This is a much better approximation to the cosine function. In fact, we have found a
2nd degree approximation. Indeed, we could continue this procedure through as many
steps as we like, and obtain the Taylor polynomial which approximate the cosine function
around x = 0. This topic appears in Calculus 3, unless we succeed in introducing it at
the very end of Calculus 2.

3.11 Exercises
[1, Exercise 28,√ p. 252] Use differentials (or, equivalently, a linear approximation) to
estimate 99.8.
√ 1
Solution: Let f (x) = x; then f 0 (x) = √ . We are going to approximate by a
2 x
tangent line. What should be the contact point of the tangent? If we take the point
(a, f (a)) = (0, 0), the tangent line is vertical: it never crosses the line x = 99.8,
and cannot be used to approximate this function at this point; or, alternatively, we
Notes for Lecture Section 002, MATH 140 2008 09 2201

can say that the approximation is infinite. If we take the point (1, 1) on the graph
of the function, we obtain a very, very bad approximation:

L(x) = f (1) + f 0 (1) · (99.8 − 1) = 1 + 49.9 = 50.9 .


√ √
But, if we take as the contact point the point (a, a) on the curve y = x which
is the closest point “convenient” to 99.8, which is (a, f (a)) = (100, 10), then the
linearization is
1
L(99.8) = f (100) + f 0 (100) · (−0.02) = 10 + (−0.02) = 9.99 .
20
Of course, there are points that are closer — in the sense of distance along the
x-axis; but they wouldn’t be convenient in the sense that there would be computa-
tional difficulties if we were to use them. For example, we could take a = (9.99)2 =
99.8001. Then the linearization would yield
1
L(99.8) = f (99.8001) + f 0 (99.8001) · (−0.0001) = 10 + (−0.0001) = 9.99 .
2(9.99)

Here we wouldn’t be able to carry out the division as easily. But long division
would yield an approximation of approximately 9.989995, which is a much better
approximation than 9.99 obtained earlier. The correct square root is approximately

99.8 = 9.98999499499374123685434142842049975151273408246 . . . ,

so the absolute errors are, approximately −5.0 × 10−5 in the case of the approxi-
mation 9.99, and −5.0 × 10−8 in the case of the approximation 9.989995 — about
1000 times smaller! (Don’t panic! It was the approximation at (100, 10) that the
textbook expected.)

[1, Exercise 38, p. 252] One side of a right triangle is known to be 20 cm long, and
the opposite angle is measured as 30◦ , with a possible error of ±1◦ .

(a) Use differentials to estimate the error in computing the length of the hy-
potenuse.
(b) What is the percentage error?

Solution:

(a) If the angle opposite the known side is called θ, then the length of the hy-
potenuse will be
f (θ) = 20 csc θ .
Notes for Lecture Section 002, MATH 140 2008 09 2202

We are told that the measured value, 30◦ , of θ could differ from the true value
π
by up to ±1◦ . We can take ∆θ = ± 180 radians, and the approximation can
be centred at a = 30π
180
= π
6
radians. The magnitude of the error will then be
approximately within
¯ √
0 ¯ π π ³ π ´¯¯ √ π 2 3
|f (a) · ∆θ| = ¯−20 csc · cot · ± ¯ = 20 × 2 3 × = π.
6 6 180 180 9
(b) The approximate length of the √ side being 20 csc θ ≈ 20(2) = 40, the rela-
2 3 √
π π 3
tive error is approximately 9 = ; the percentage error will be this
40 180√ ¡ √ ¢
π 3 5 3 π
fraction expressed as a percentage, i.e., × 100% = %.
180 9

Laboratory Project: Taylor Polynomials (This is a topic which may be reached


at the end of Math 141; it is not in the syllabus of Math 140.)

5.16.2 §3.11 Hyperbolic Functions


In this section we define functions that have a strong relationship to the 6 trigonometric
functions; they are called the hyperbolic functions, and their names resemble those of
the trigonometric functions, except for the letter “h” added at the end. Most of the
reasons for this relationship cannot be proved in this course, however; in MATH 141,
when we study [1, §10.1]100 , we shall see that there is a certain analogy between two of
these functions and the corresponding trigonometric functions.
Earlier in these notes I stated a theorem (5.3, p. 2015) which asserted that any
function can be decomposed into an “even” part and an “odd” part. The hyperbolic
cosine function, cosh, is defined to be the “even” part of ex ; formally, its definition is
1¡ x ¢
cosh x = e + e−x .
2
By definition, it is even, (just like the cosine function), because
1¡ x ¢
cosh x = e + e−x
2
1 ¡ −x ¢
= e + ex
2
1 ¡ −x ¢
= e + e−(−x)
2
= cosh(−x) .
100
see, for example [1, Exercise 17, p. 626]
Notes for Lecture Section 002, MATH 140 2008 09 2203

In a similar way, one can prove that the hyperbolic sine, defined by
1¡ x ¢
sinh x = e − e−x .
2
has the property that
sinh(−x) = − sinh x
which, by definition of odd , tells us that the function is odd, just like the sine function.
The other four hyperbolic functions are defined by the same ratios used to define the
analogous trigonometric functions:
sinh x ex − e−x
tanh x = = x
cosh x e + e−x
cosh x ex + e−x
coth x = = x
sinh x e − e−x
1 2
sech x = = x
cosh x e + e−x
1 2
csch x = = x
sinh x e − e−x
The similarities with the trigonometric functions do not end here. We find that the hy-
perbolic functions satisfy identities that resemble trigonometric identities. For example,
you should be able to prove that

cosh2 x − sinh2 x = 1
sinh(x + y) = sinh x cosh y + cosh x sinh y
cosh(x + y) = cosh x cosh y + sinh x sinh y .

The similarities continue to the derivatives:


d
sinh x = cosh x
dx
d
cosh x = sinh x
dx
d
tanh x = sech2 x
dx
d
coth x = −csch2 x
dx
d
sech x = −sech x · tanh x
dx
d
csch x = −csch x · coth x .
dx
Notes for Lecture Section 002, MATH 140 2008 09 2204

The Catenary Curve. The graph of an arc of the graph of the hyperbolic cosine
symmetric about the minimum point is sometimes called the catenary because it is the
shape taken by a heavy, uniform chain, rope, or cable hanging under its own weight; the
proof of this fact is beyond MATH 140.

Inverse Hyperbolic Functions. Hyperbolic functions sinh and tanh are invertible;
as are csch and coth, but the domains of the latter two functions exclude the origin.
Functions cosh and sech are invertible only if we confine ourselves to a portion of their
domain, usually taken to be the non-negative real numbers. The properties of the in-
verse functions can be discovered using similar methods to those we used to study the
inverse trigonometric functions. Unlike the situation with the inverse trigonometric func-
tions, these inverse functions don’t enable us to express any new ideas, in that they are
expressible in terms of known functions like the natural logarithm.

Example 5.47 To determine the inverse hyperbolic tangent.

y = tanh x
ex − e−x
⇒ y= x
e + e−x
1+y
⇒ e2x =
1−y
r
x 1+y
⇒ e =±
1−y
But only the positive
rsign is acceptable,rsince an exponential cannot be negative. We
1+y 1+y
conclude that ex = , so x = ln . Since we usually use the symbol x to
1−y 1−y
denote the independent variable in a function, we may rewrite this last result as
r
1+x
tanh−1 x = ln .
1−x
Example 5.48 Express the inverse hyperbolic cosecant in terms of familiar functions.
Solution: Suppose that y = csch x; we wish to solve for x in terms of y.
1
y = csch x ⇔ sinh x =
y
2
⇔ ex − e−x =
y
2 x
⇔ e2x − 1 = ·e
y
Notes for Lecture Section 002, MATH 140 2008 09 2205

2 x
⇔ (ex )2 − ·e −1=0
y
1³ p ´
⇒ ex = 1 ± 1 + y2
y
Case 1: Suppose y > 0. As exponentials are never negative, we can replace ± by +; then
we take logarithms of both sides:
à p !
1 + 1 + y2 p
x = ln = ln(1 + y 2 + 1) − ln y .
y

Since we are in the custom of calling the independent variable x, we will change the
variable’s name, to obtain
p
csch−1 x = ln(1 + x2 + 1) − ln x .

Case 2: Suppose y < 0. Here the positivity of ex leads us to choosing the minus sign; we
obtain, after reduction and the taking of logarithms,
à !
−y p
x = ln p = − ln(1 + y 2 + 1) + ln(−y) .
1 + 1 + y2

Since we are in the custom of calling the independent variable x, we will change the
variable’s name, to obtain
p
csch−1 x = − ln(1 + x2 + 1) + ln(−x) .

This example demonstrates that anything we wish to “say” using the inverse hy-
perbolic cosecant could be expressed in terms of polynomials and logarithms; the same
phenomenon holds for the other inverse hyperbolic functions.
We sometimes use the alternative “arc” names: arcsinh, arccosh, . . .. The derivatives
of these functions can be determined using implicit differentiation, similarly to what we
did for inverse trigonometric functions. The derivatives of the inverses may also be found
by explicit differentiation after we have found a formula for the inverse functions.
The formulas which express these inverse functions in terms of the natural logarithm
are not intended to be memorized: however, we could expect students to be able to
explore properties of the inversion of the hyperbolic functions, in the same way as we
could expect you to be able to explore the properties of any other reasonable invertible
function. In the same way, we don’t expect you to memorize the formulas for the deriva-
tives of these inverse functions; but you could be expected to explore the properties of
the derivatives by using implicit differentiation, etc.
Notes for Lecture Section 002, MATH 140 2008 09 2206

Limits involving the hyperbolic functions; asymptotic behavior. As these func-


tions can be expressed in terms of exponentials, you should be able to find the limits
as one approaches ±∞ or, in the cases of those functions having discontinuities, either
side of the points of discontinuity. Such limits can be explored without using L’Hospital’s
Rule (which we haven’t met yet), using standard methods you should already be familiar
with.
Example 5.49 Determine the asymptotes of the graph of tanh x.
Solution:
sinh x
lim tanh x − = lim
x→∞ x→∞ cosh x
ex − e−x
= lim 2
x→∞ ex + e−x

2
ex − e−x
= lim
x→∞ ex + e−x
ex (1 − e−2x )
= lim x
x→∞ e (1 + e−2x )

1 − e−2x
= lim
x→∞ 1 + e−2x

1 − lim e−2x
x→∞
=
1 + lim e−2x
x→∞
1−0
= = 1.
1+0
In a similar way it can be proved that lim tanh x = −1. Thus the horizontal asymptotes
x→−∞
of the graph of tanh x are y = ±1. This function has no discontinuities, so there are no
vertical asymptotes.
Example 5.50 ([7, Exercise 3.9.20, p. 255]) “If sinh x = 34 , find the values of the other
hyperbolic functions at x.”
Solution: Since we know that the hyperbolic sine has an inverse, we could use that
function to determine x, then evaluate the other functions there.
3
x = arcsinh
 4s 
µ ¶2
3 3
= ln  + + 1
4 4
Notes for Lecture Section 002, MATH 140 2008 09 2207

µ ¶
3 5
= ln + = ln 2
4 4
⇒ ex = 2
1
⇒ e−x = .
2
It follows that cosh x = 45 , tanh x = 53 , etc.
Instead, we will proceed from first principles; in effect we are repeating some of the
work that was done to determine a formula for the inverse function (which you are not
expected to memorize).

3 ex − e−x 3
sinh x = ⇒ =
4 2 4
x −x
⇒ 2(e − e ) = 3
⇒ 2 (ex )2 − 3ex − 2 = 0

x 3 ± 9 + 16 3±5 1
⇒ e = = = 2 or − ,
4 4 2
But, of these 2 values, the second is extraneous, since an exponential cannot be negative.
From the value of ex we can proceed as above to determine the values of the other
hyperbolic functions.

What should I remember? It is essential to remember the definitions of the functions


in terms of exponential functions. Then, even if you don’t remember some identities,
you can reconstruct them by expressing the hyperbolic functions in those terms.
We will not expect students in this course to remember the explicit formulæ for the
inverse hyperbolic functions or the derivatives of those functions, although you should
know how to find those formulæ if asked, using the method of the preceding example.
You should also be comfortable with the general question of finding the inverse function
of any function whose graph satisfies the “Horizontal Line Test”, and know how to find
its derivative.

Whence the name “hyperbolic”? The trigonometric functions can be used to de-
scribe circles, and, more generally, ellipses. We will see this in MATH 141 in connection
with [1, Chapter 10]; that’s why the trigonometric functions are sometimes called cir-
cular functions. In an analogous way, hyperbolic functions can be used to describe a
well known family of curves called hyperbolæ. The names are not completely logical;
for example, the term elliptic function is not analogous, but has a completely different
meaning.
Notes for Lecture Section 002, MATH 140 2008 09 2208

3.11 Exercises
[1, Exercise 3.11.9, p. 255] “Prove that ex = cosh x+sinh x.” This is a decomposition
of ex as a sum of an even function and an odd function.
[1, Exercise 3.11.33, p. 260] “Find the derivative: f (t) = ln(cosh t).”
Solution: We apply the Chain Rule:
1
f 0 (t) = · sinh t
cosh t
= tanh t .
sinh x
[1, Exercise 3.11.54, p. 261] Evaluate lim .
x→∞ ex
Solution:
ex − e−x
sinh x 2
lim = lim
x→∞ ex x→∞ ex
1 − e−2x 1
= lim = .
x→∞ 2 2
[1, Exercise 3.11.56, p. 261] “ If x = ln(sec θ + tan θ), show that sec θ = cosh x.”
Solution:
cosh x = cosh(ln(sec θ + tan θ))
1 ¡ ln(sec θ+tan θ) ¢
= e + e− ln(sec θ+tan θ)
2µ ¶
1 ln(sec θ+tan θ) 1
= e + ln(sec θ+tan θ)
2 e
µ ¶
1 1
= sec θ + tan θ +
2 sec θ + tan θ
2 2
sec θ + tan θ + 2 sec θ · tan θ + 1
=
2(sec θ + tan θ)
2
2 sec θ + 2 sec θ tan θ
=
2(sec θ + tan θ)
= sec θ

5.16.3 3 Review
True-False Quiz
[1, True/False Exercise 10, p. 261] A discussion of this problem appears in these
notes on page 2211 et seq.
Notes for Lecture Section 002, MATH 140 2008 09 2209

Exercises

[1, Review Exercise 102(a), p. 264] Find the linear approximation to f (x) = 25 − x2
near 3.
1 x
Solution: f 0 (x) = 21 (25 − x2 ) 2 · (−2x) = − √ ; thus f 0 (3) = − 43 . Since
25 − x 2

f (3) = 16 = 4, the linearization of f near x = 3 is

L(x) = f (3) + f 0 (3) · (x − 3)


3 25 − 3x
= 4 − · (x − 3) =
4 4

[7, True/False Exercise 12, p. 272] “Determine whether the statement is true or
false. If it is true, explain why. If it is false, explain why, or give an example
that disproves the statement:
µ ¶2
d2 y ? dy
= .” (190)
dx2 dx

More precisely, the statement is asserting that the equality holds for all functions
f having derivatives of the given orders.
Solution: This statement is false, and most functions you know could serve as
counterexamples. For example, if f (x) = a + bx, then the second derivative is
0, but the square of the first derivative is b2 ; so any linear function of the form
f (x) = a + bx (b 6= 0) is a counterexample. We couldn’t take a constant function,
as such functions do have the given property.
A more interesting question might be to characterize all counterexamples. And
it’s not hard to do. With a property we will meet in the next chapter, we can do
dy
this! For convenience let’s denote by v. Then (190) could be rewritten as
dx
dv
= v2 .
dx
If v 6= 0, this can be rewritten as
1 dv
· = 1,
v 2 dx
d 1
or − = 1.
dx v
Notes for Lecture Section 002, MATH 140 2008 09 2210

We shall see in the next chapter that the most general function having 1 as a
derivative is a linear function x + C, where C is any constant (real number). Thus
the most general solution of the “differential equation” is
1 dy 1
v=− ⇔ =− .
x+C dx x+C
But, again by the theory of the next chapter, the most general function having
1
− as its derivative is − ln |x + C| + K, where K is another constant. So we
x+C
see that the most general non-zero function that possesses property (190) is

y = − ln |x + C| + K

where C and K are constants. We have seen earlier that the function 0 also has
the property.
You are not expected to solve general differential equations per se in this course;
this last paragraph is presented to show you an application of the theory we will
be meeting in [1, §4.2]. If you wish to read more about Differential Equations, you
could look at [1, Chapter 9, Differential Equations], which is not on the syllabus of
MATH 140 or MATH 141. The present application could, however, be formulated
as looking for an “antiderivative” of certain functions, and we shall be meeting
that topic later in this course in [1, §4.9].

Operations that “commute”. We say that two operations commute if the order in
which they are performed does not affect the result. For example, on the set of functions
of a real variable x
• the operations of multiplication by a constant c and taking the limit as x → a
commute (provided all the limits exist); this is the Constant Multiple Law;

• the operations of addition and taking the limit commute provided the limits exist;
this is the Sum Law;

• the operations of multiplication and taking the limit commute provided the limits
exist; this is the Product Law;

• the operations of multiplication by a constant and differentiation commute (pro-


vided all derivatives exist); this could be called the “Constant Multiple Rule” of
Differentiation;

• the operations of addition and differentiation commute provided all derivatives


exist; this is the Sum Rule of Differentiation.
Notes for Lecture Section 002, MATH 140 2008 09 2211

But there are situations where operations do not commute; one that we have just met
is the Product Rule of Differentiation, which gives a formula for the derivative of the
product of two functions, but where you can see that the derivative of the product is
not usually equal to the product of the derivatives. The following problem investigates
a question about whether two given operations commute.
[1, True/False Exercise 10, p. 261]
“Determine whether the statement is true or false. If it is true, explain why. If it is false,
explain why or give an example that disproves the statement:
d ¯¯ 2 ¯
x + x¯ = |2x + 1| .”
dx
More generally, the question could have asked: “Do the operations of taking the absolute
value and differentiation commute; i.e., is it always true that
¯ ¯
d ¯d ¯
|f (x)| = ¯¯ f (x)¯¯ ?”
dx dx
Solution: Your first reaction to the statement should be one of suspicion: while we have
seen that some operations “commute” with differentiation — operations like addition,
multiplication by a constant — there is no good reason why a function that we already
know to be “unruly” should commute with differentiation. But, until we investigate a
possible proof, we can’t dismiss the possibility. The best way to approach this problem
is to determine the value of the left and right sides of the proposed equation, and to
compare them carefully.
½
¯ 2 ¯ x(x + 1) if x(x + 1) ≥ 0
¯x + x¯ =
−x(x + 1) if x(x + 1) < 0
½
x(x + 1) if x ≤ −1 or x ≥ 0
=
−x(x + 1) if −1 < x < 0
Now we differentiate:


 d
d ¯¯ 2 ¯ [x(x + 1)] if x < −1 or x > 0
x + x¯ = dx
d
dx 
 [−x(x + 1)] if −1 < x < 0
 dx

 d 2
[x + x] if x < −1 or x > 0
= dx
 d
 [−x2 − x] if −1 < x < 0
½ dx
2x + 1 if x < −1 or x > 0
=
−2x − 1 if −1 < x < 0
Notes for Lecture Section 002, MATH 140 2008 09 2212

(We had to exclude the points −1 and 0 because the function fails to be differentiable
at those points.) We see both 2x + 1 and −(2x + 1) appearing in our formula, and it is
tempting to jump to the conclusion that the claimed equality holds. But let’s determine
the value of the right side of the proposed equation without using absolute value signs:
½
2x + 1 if 2x + 1 ≥ 0
|2x + 1| =
−2x − 1 if 2x + 1 < 0
½
2x + 1 if x ≥ − 21
=
−2x − 1 if x < − 12

Now we see that significant points for analyzing this problem are −1, − 21 , and 0. These
divide the real line into 4 intervals. By examining the values of the two sides of the equa-
tion for points in these four intervals, we should be able to find a counterexample to the
claimed equation. It’s hard work! Note first that, if x ≥ 0, the claimed equation holds;
similarly, it can be seen ¡to hold¢ for −1 < x < − 12 . But we claim it fails throughout the
intervals (−∞, −1) and − 21 , 0 . We don’t need to prove that much for a counterexample
— we just need to provide one example. For example, take the point x = − 14 . Around
that point on the real line |x2 + x| = −x2 − x, so its derivative is −2x − 1: at x = − 14
this is 21 − 1 = − 12 . But, at x = − 14 , |2x + 1| is 12 . (In fact, it is enough to observe that,
being an absolute value, it cannot be negative.) Thus we have disproved the claim.
Most of this “solution” is superfluous, as I was showing you how I arrived at a
counterexample. All that is needed in the proof is something like the following:
2 1
¡ 1¢
The claim is false, because the
¯ derivative
¯ of x +x at x = − 4
is −2 − 4 −1 =
1 ¯ 1 ¯
− 2 , which is negative, but 2(− 4 ) + 1 is positive.
Another way to “discover” a counterexample in this case would be to sketch a graph of
left and right sides. Of course, the graph has no place in the proof itself, but it could be
used to discover the problem. The graph of y = |x2 + x| can be obtained from the graph
of y = x2 + x by “folding” the portion that lies below the x-axis upward so that it now
appears above the x-axis. You can then see that there are 2 intervals where the function
is increasing, and two intervals where it is decreasing; but the graph of |2x + 1| has just
one interval where the function is decreasing, and one interval where it is increasing.
This shows that something is wrong, etc.

For the final examination, do we need to know how to prove theorems? Math
140 is mainly a “problem-oriented” course — we usually don’t expect students to mem-
orize proofs of theorems; however if a theorem can be formulated as a “reasonable”
problem, then it could be posed as such, and you could be expected to be able to solve
it.
While you aren’t expected to memorize proofs, you are certainly expected to know
Notes for Lecture Section 002, MATH 140 2008 09 2213

• the statements of important theorems

• precisely what the theorem assumes, and what it applies to

• how and when to use the theorem

You could certainly be presented with questions which test that knowledge. So, for
the Mean Value Theorem, discussed below, [1, Exercise 4.2.15, p. 286] is a perfectly
reasonable question for students in Math 140. Few, if any, questions that probe your un-
derstanding of a theorem are to be found on WeBWorK. We have adopted WeBWorK
because of its great value in monitoring your progress in solving numerical problems, but
it does not “drive” the course: the best aid for learning the material in the course is the
exercises in the book, monitored with the Student Solutions Manual. Excellent problems
for testing your understanding of the concepts are in the Review exercises entitled “Con-
cept Check” and “True-False Quiz” at the end of each chapter of the textbook. However,
these problems are not necessarily formulated in a form that would be practical for an
examination.
Notes for Lecture Section 002, MATH 140 2008 09 2214

5.17 Supplementary Notes for the Lecture of November 05th,


2008
Release Date: Wednesday, November 05th, 2008
subject to correction

Textbook Chapter 4. APPLICATIONS OF


DIFFERENTIATION.

5.17.1 §4.1 Maximum and Minimum Values


Definitions The terminology of this section uses English words that have meanings
outside of mathematics. Some of the terms we use may mean precisely what you would
expect from the words; in other cases there could be some doubt. So we have to begin as
does any mathematics text in an area new to the reader — by being extremely precise
about our terminology.
Definition 5.11 A neighbourhood of a point c in a set S in R is an open interval of the
form (a − h, a + h) contained entirely in S. The distance h could be called the radius of
the open interval, and c could be called its centre; mathematicians often use the symbol
ε or ² (read “epsilon”) instead of h to denote the radius of a neighbourhood.101 .
Definition 5.12 1. Whenever, in reference to a function, we use the terms maximum
or minimum, or any expressions defined in terms of them, we must specify a region
where we are considering the function. This region could be the entire domain of
the function, but it might be a smaller subset of the domain — where, in effect,
we are restricting the function to a smaller domain than the domain originally
announced for the function, or to the “natural” domain of the function (the set
where the function “makes sense”).
2. A function f is said to have a maximum for (or “on”) a set S of points in its
domain at a point c if f (x) ≤ f (c) for all x in S; f (c) is said to be the maximum
value of f on S. The word maximum may be modified by the adjectives absolute
or global 102 , without changing the meaning of the term.
3. A function f is said to have a minimum for (or “on”) a set S of points in its domain
at a point c if f (x) ≥ f (c) for all x in S. f (c) is said to be the minimum value of
f on S. The word minimum may be modified by the adjectives absolute or global .
101
This is the usage in the formal definition of limit.
102
The descriptor global could be considered the newer usage more favoured by mathematicians.
Notes for Lecture Section 002, MATH 140 2008 09 2215

4. A function f is said to have a local maximum or relative maximum 103 for (or “on”)
a set S of points in its domain at a point c if there exists a positive number h such
that c is a maximum for f on a neighbourhood (c − h, c + h) contained in S.104

5. A function f is said to have a local minimum or relative minimum for (or “on”) a
set S of points in its domain at a point c if there exists a positive number h such
that c is a minimum for f on a neighbourhood (c − h, c + h) contained in S.

6. By extremum we mean either a maximum or a minimum. All the modifying terms


described above can be applied to extremum, but we would usually say extremal
value or extreme value when we wish to group maximum value and minimum value
together.

7. In mathematical English the words maximum, minimum, extremum are pluralized


as Latin neuter nouns: maxima, minima, extrema — never by adding an s.
If c is a maximum (respectively, minimum) point of f on some set S, then we can say that
the function f attains its maximum (respectively, minimum) for S at c. Note that we
are permitting that the same maximum (respectively, minimum) value could be attained
at more than one point.

An “existence” theorem.
1
Example 5.51 The function f defined by f (x) = has no global minimum (value)
1 + x2
(on its domain, which is R).
Proof: If we make x large enough, we can cause the value of f to be smaller than any
1
positive number we take: we see that lim = 0. But no matter how large we
x→±∞ 1 + x2
take |x|, the function value is always strictly positive, and the function value is never
0. We can’t say that the function has a minimum value of 0, since the function never
attains this value. This function has no minimum!

Fortunately we have105 a theorem that ensures the existence of global extrema under
certain conditions that are often not difficult to satisfy:

Theorem 5.52 (Extreme Value Theorem) [1, p. 272] A function f which is con-
tinuous on a finite, closed interval [a, b] attains both a global maximum and a global
minimum at point(s) of the interval.
103
Here the term preferred by modern mathematicians is local .
104
It doesn’t matter what positive value h has — if there exists an open interval with this property
centred at c.
105
The proof of this theorem is beyond Math 140; it could be studied, for example, in Math 242.
Notes for Lecture Section 002, MATH 140 2008 09 2216

(Remember that continuity on a closed interval [a, b] entails continuity at every point
except a and b, as well as continuity from the right at a, and from the left at b.) This
theorem is “best possible”, in the sense that the conditions of continuity and the closed
interval are needed for it to be true. If the function is permitted to have a discontinuity,
then it is possible to design a function that has no maximum or minimum; similarly, if
the interval is permitted to be open at one or other of its ends, there again we can design
1
a function that lacks extrema. The domain of the function discussed above is
1 + x2
not a finite closed interval, so the theorem does not apply there. Students should try
to construct “counterexamples” to extensions of the theorem, i.e., examples where some
part of the hypotheses is not available, and where the function fails to have extrema.
Note that the Extreme Value Theorem is only an existence theorem — it asserts that
extreme values exist, but gives no information about how to find them.

A “necessary condition” for an extremum. When a function is more than con-


tinuous on an interval, we may have more information available that will enable us to
determine the extrema. For example, we know that differentiability is a stronger property
than continuity. What if a function is known to be differentiable?
106
Theorem 5.53 (Fermat [1, p. 273]) If g is differentiable at c and has a local ex-
tremum there, then g 0 (c) = 0.
This theorem provides a condition that is necessary for the existence of an extremum
at a point where g is differentiable; but it may happen that the condition is satisfied,
and yet g still has no extremum at the point. That is, the condition is necessary for an
extremum, but is not sufficient to ensure that the point is an extremum. As an example,
consider the function g(x) = x3 , and take as the domain either the whole real line R or
any convenient interval around 0. To the right of 0 the function is positive, and to the
left of 0 it is negative. Thus it cannot have an extremum at x = 0, where g(x) = 0;
but the derivative is equal to 0 at x = 0. However, the theorem does provide us a way
of narrowing our search for local extrema: just determine all the points in the domain
where the derivative is 0, or where there is no derivative, and the local extreme points
— if any — will be among them. Remember that a function which is being considered
on a closed interval [a, b] cannot be considered to be differentiable at x = a or x = b,
because that property requires consideration of a limit on both sides of the point, and
the function is not defined on one of those sides; thus the theorem can never exclude the
possibility that the end point of a closed interval is an extremum. For that reason the
method we are developing will require explicit, separate consideration of the end points
of intervals in the domain as candidates for extrema.
106
This result is not always linked to P. Fermat; and there are other well known results that are often
called “Fermat’s Theorem”.
Notes for Lecture Section 002, MATH 140 2008 09 2217

Critical Numbers or Critical Points. For ease in formulating a method for finding
extrema, we define a critical number or critical point of the domain of f to be a number
or “point” c such that either

• f fails to be differentiable at c; or

• f is differentiable at c, and f 0 (c) = 0.

The value of the function at a critical point is a critical value. Then Theorem 5.53 may
be reformulated as follows:

Theorem 5.54 [1, p. 274] The local extrema of f can occur only at critical points of
the domain.

Is a global extremum always a local extremum? If a global extremum of f occurs


at a point a which is in the “interior” of the domain of f , in the sense that a is contained
inside an interval (a−k, a+k) in the domain of f (where k is some positive number), then
this global extremum is also a local extremum. But if, for example, the extremum occurs
at the end point of an interval [a, b] of the domain, where the domain does not contain
the points immediately to the left of a, then a cannot be called a local extremum.107 We
can formalize the situation for a function being considered on one closed interval [a, b]:

Theorem 5.55 (The Closed Interval Method) [1, p. 275] The following procedure
serves to narrow the search for absolute extrema of a continuous function f on a closed
interval [a, b]:

1. Determine all critical points of f in (a, b), and the critical values of f at each of
these critical points.

2. Determine the values of f at the end points, viz., f (a) and f (b).

3. The global maximum of f on [a, b] will be the maximum of the set of values de-
termined in the preceding two steps; analogously, the global minimum will be the
minimum of the set.

It can happen that the preceding method is still not finite, as the set of critical points
could be infinite.
107
There are other ways of defining the use of the term local , and some authors may follow a practice
under which the end points can still be considered local extrema, if they are “one-sided” local extrema.
These variations in definition will not affect you in this course, but you may see slightly different
formulations if you read other textbooks.
Notes for Lecture Section 002, MATH 140 2008 09 2218

Exercise 5.8 The function given by


( 1
x sin when x 6= 0
f (x) = x
0 when x = 0

when considered in the interval −1 ≤ x ≤ 1 has infinitely many critical points. (What
are they?) ¡ ¢
Solution: The derivative of x sin x1 is sin x1 + x cos x1 · − x12 = sin x1 − x1 · cos x1 . This
derivative vanishes when tan x1 = x1 . We can’t solve this equation exactly. The line y = x
crosses the curves y = tan x infinitely often. Each of those crossings as x → ∞ yields a
point closer and closer to 0 where the function f has a horizontal tangent; there are thus
infinitely many critical points in the interval [−1, 1]. In addition there is one point —
just one — where the function is not differentiable. That point is x = 0, since the limit
of the slope of the line segment joining (0, f (0)) to a nearby point on the curve does not
exist, as the slopes oscillate between ±1.

Are end-points of an interval domain critical? If a function f is defined only


on [a, b], it can’t have a derivative at either a or b. Technically, such points satisfy the
textbook’s definition of critical . Some authors call such points critical, and some do
not. You will not be expected to make such a decision; in any case, Part 1 of Theorem
5.55, speaks only of critical points in (a, b); then Part 2 requires you consider the end
points without reference to any question about differentiability. To summarize: it doesn’t
matter whether you wish to consider end-points as critical, you will not need to use that
concept at end-points.
Example 5.56 Find the global extrema of the function |x| on the interval −1 ≤ x ≤ 2.
Solution: This function has no points where the derivative vanishes, but it does have
one point where it is not differentiable; that is x = 0. By definition, this is a critical
point. The global extrema will occur among the points −1, 0, 2: the global minimum is
at x = 0, of value 0; and the global maximum is at x = 2, of value 2.

Example 5.57 ([7, Exercise 4.1.58, p. 287]; cf. [1, Exercise 4.1.68, p. 278]) Find the
absolute (=global) maximum and absolute (=global) minimum values of f (x) = x −
2 cos x on the interval [−π, π].
Solution: This function, being a sum of a polynomial and a cosine, is differentiable ¡ ¢
everywhere. ¡f 0 (x) = ¢1+2 sin x, which vanishes when the sine is − 12 , i.e. at x = 2n − 16 π,
and at x = 2n − 56 π, n being any integer. In the interval given, there are only the
points − π6 ; − 5π
6
.
³ π´ √
π 3
f − = − −2·
6 6 2
Notes for Lecture Section 002, MATH 140 2008 09 2219

µ ¶ √
5π 5π 3
f − = − +2·
6 6 2
We must also check the end-points of the prescribed domain.108
f (−π) = −π + 2
f (π) = π + 2
Of the four values, we find, using a very crude approximation
√ for π, that the largest is
π + 2, attained when x = π, and the minimum is − 6 − 3, attained when x = − π6 .
π

Example 5.58 ([7, Exercise 4.1.68(b), p. 287]) “Use calculus to find the exact maximum
cos x
and minimum values: f (x) = .”
2 + sin x
Solution: (In the (a) part of this problem the textbook asks you to graph the function;
but that graphing is not at all necessary to finding the extrema.)
− sin x · (2 + sin x) − cos x · cos x 2 sin x + 1
f (x) = 2
=−
(2 + sin x) (2 + sin x)2
which vanishes when sin x = − 21 , i.e., when x = − π6 + 2mπ and x = 7π
6
+ 2nπ, where m
and n are any integers. We are confined by the conditions of the problem to the closed
interval 0 ≤ x ≤ 2π, so the critical points are
π 11π
− + 2π =
6 6

and . We must also consider the function at the end-points of the given closed interval.
6
We tabulate the values:
7π 11π
x 0 6 6

end point critical point critical point end point
1
f (x) 2
− √13 √1
3
1
2

Since the function x is an increasing function,
√ √ 1 1
3<4⇒ 3< 4=2⇒ √ >
3 2
so the absolute maximum on the given interval is √13 , and the absolute minimum is − √13 ;
in fact, because the function is periodic with period 2π, these are the absolute extrema
for the natural domain of the function, R.
108
Note that, again, this is not the “natural” domain of the function, but a restricted domain prescribed
by the author of the problem.
Notes for Lecture Section 002, MATH 140 2008 09 2220

4.1 Exercises

[1, Exercise 4.1.50, p. 278] “Find the absolute (=global) maximum and absolute
(=global) minimum values of f on the given interval”: f (x) = x3 − 6x2 + 9x + 2,
on the interval [−1, 4]
Solution: The function is a polynomial, which is differentiable everywhere. Thus
the only critical numbers possessed by f1 are points in the domain where the
derivative vanishes. f (x) = 3x2 − 12x + 9 = 3(x − 3)(x − 1). Thus the critical
points (=critical numbers) are 3 and 1. Following the “Closed Interval Method”,
we have to consider the function’s values at the critical points and at the end points
of the interval; we calculate that

f (3) = 27 − 54 + 27 + 2 = 2
f (1) = 1−6+9+2=6
f (−1) = −1 − 6 − 9 + 2 = −14
f (4) = 64 − 96 + 36 + 2 = 6

We see that the maximum value is 6, and it is attained at 2 maximum points, 1 and
4; the minimum value is −14, and it is attained uniquely at the minimum point
−1. Point 3 is neither a global maximum, nor a global minimum. (Using methods
we have still to develop, we could prove that it is [the only] local minimum.)

[1, Exercise 4.1.54, p. 278] Same instructions as the preceding, with function f (x) =
x2 − 4
and interval [−4, 4].
x2 + 4
Solution: Before applying the Closed Interval Method, I will simplify the function,
so that its behavior will become obvious.
x2 − 4 x2 + 4 − 8 8
f (x) = 2
= 2
=1− 2 .
x +4 x +4 x +4
The subtracted term has numerator and denominator which are both positive.
Since the numerator is constant, the fraction increases as the denominator de-
creased, and the denominator cannot be less than 02 + 4 = 4, a value which is
attained at x = 0. Thus we see — without even appealing to the calculus — that
f attains its minimum when x = 0. But the denominator has no maximum, so the
subtracted term may be made arbitrarily small, This shows that f approaches 1
from below, but never attains this value: f does not have a maximum value. But
wait — we were not asked to consider the function over the maximal domain, only
over the domain −4 ≤ x ≤ 4. The same reasoning we have used shows that the
Notes for Lecture Section 002, MATH 140 2008 09 2221

value of the function increases as we move from 0 to either +4 or −4, where its
8 3
value is, in either case, 1 − = .
20 5
Now let’s use the calculus.
8 16x
f 0 (x) = 0 − 2 · 2x =
(x2 + 4) (x + 4)2
2

for all x in −4 < x < 4; this function is 0 only when x = 0, so x = 0 is a critical


number of f . Because we haven’t been authorized to consider f to the left of −4
or to the right of +4, we really can’t claim to know f 0 (±4), so, technically, ±4
could be considered to be critical numbers of f . Some textbook authors define
their terms slightly differently from yours, and, under their definitions, the end
points ±4 might not be critical points. It doesn’t matter which definitions we use,
however, because we are going to meet the end-points in the next step of the Closed
Interval Method anyhow, as this step requires us to determine the values of the
3
function at the end-points of the interval of definition. As seen earlier, f (±4) = .
5
To complete the application of the Closed Interval Method we consider the values
of the function at the three points — −4, 0, +4:

x −4 0 +4
f (x) 35 −1 35

and conclude that the absolute minimum of −1 on the interval is attained at x = 0,


and the absolute maximum of 35 on the interval is attained at x = ±4.

[1, Exercise 4.1.62, p. 278] Same instructions, f (x) = e−x − e−2x , interval =[0, 1].
Solution: f 0 (x) = −e−x + 2e−2x = e−x (−1 + 2e−x ). In this product, the first
factor, e−x cannot be zero, as no exponential is zero. Hence¡f ¢0 can vanish only
2
when e−x = 21 , i.e. when x = ln 2; we note that f (ln 2) = 21 − 21 = 41 . Checking
the function at the end points, we find that f (0) = 1 − 1 = 0, and f (1) = 1e − e12 .
Comparing the values, again using a very crude approximation for e, we find that
the global maximum is at x = ln 2, with value 41 , and the global minimum is at 0,
with value 0.

[1, Exercise 4.1.74, p. 279] “Show that 5 is a critical number of the function g(x) =
2 + (x − 5)3 , but g does not have a local extreme value at 5.”
Solution: g(x) = 3(x − 5)2 , which vanishes only at x = 5; thus x = 5 is the only
critical point. This function resembles the function x3 considered on page 2216.
If we take x < 5, the function value is less than 2, while, for x > 5, g(x) > 2.
Notes for Lecture Section 002, MATH 140 2008 09 2222

It follows that 5 cannot be either a local maximum or a local minimum, and, a


fortiori it cannot be a global maximum or a global minimum. (The only way in
which 5 could be a local extremum is if we were to consider the function on an
interval ending at 5.)
Notes for Lecture Section 002, MATH 140 2008 09 2223

5.18 Supplementary Notes for the Lecture of November 10th,


2008
Release Date: Monday, November 10th, 2008
subject to correction

5.18.1 §4.2 The Mean Value Theorem


We interrupt our study of extrema to consider two “existence” theorems, whose impor-
tance is often misunderstood, possibly because of a type of problem that often appear
in certain calculus textbooks. The theorems assert the existence of a number with a
particular property, but make no attempt to locate that number; textbook problems
that ask you to find the numbers in question are misleading, since, in applications of
these theorems, we are interested not in the numbers themselves, but only in their exis-
tence, and in crude estimates of the “worst possible” values they may have. Most of the
problems in your textbook do adopt a “correct” stance; but, for example, [1, Exercises
4.2.1-4.2.4, p. 285] and the part of [1, Exercises 4.2.11-4.2.14, p. 285] which states “Then
find all numbers c that satisfy the conclusion of the Mean Value Theorem” are alien to
the spirit of the theorem, unless one interprets them as verifying that there is at least
one point with the desired property.

The Theorems. There are two main theorems in this section, and, while it appears
from the enunciations that one of them is “stronger” than the other, they are equivalent,
in the sense that either of them can be used to prove the other.
109
Theorem 5.59 (Rolle’s Theorem) Let [a, b] be a given closed interval. Let f be
a function such that

1. f is continuous on [a, b];

2. f is differentiable on (a, b); and

3. f (a) = f (b).

Then there exists a number c such that a < c < b and f 0 (c) = 0.

Theorem 5.60 (Mean Value Theorem) Let [a, b] be a given closed interval. Let f
be a function such that

1. f is continuous on [a, b];


109
The name of Michel Rolle (1652 – 1719) is usually pronounced to rhyme with folle — there are two
syllables, Rol-le, but the second syllable is almost silent in normal fast speech.
Notes for Lecture Section 002, MATH 140 2008 09 2224

2. f is differentiable on (a, b).


f (b) − f (a)
Then there exists a number c such that a < c < b and f 0 (c) = .
b−a
It appears that the Mean Value Theorem should be “stronger” than Rolle’s Theorem,
since Rolle’s is the special case of the Mean Value Theorem where f (a) = f (b). But,
in fact, the theorems are equivalent, since the Mean Value Theorem can be proved by
applying Rolle’s Theorem to the appropriate function.

Geometric interpretation of these theorems. The Mean Value Theorem says that,
if we consider the secant to the graph of f obtained by joining the points (b, f (b)), and
(a, f (a)), there will always be at least one point c between a and b where the tangent to
the graph of f is parallel to that secant.

Doesn’t one hypothesis imply another? Since you know that differentiability im-
plies continuity, you may ask why we need both 1. and 2. Indeed, the portion of 1. that
speaks of continuity at the points of (a, b) is totally redundant: continuity at these points
is implied by differentiability. But remember what continuity f on [a, b] means110 :

• continuity of f on the interval (a, b),

• continuity from the right at x = a, and

• continuity from the left at x = b.

Thus we could weaken the first hypothesis of the theorem, to

10 . f is continuous from the right at x = a, and continuous from the left at x = b.

The conclusion of Rolle’s Theorem. If the hypotheses of the theorem are satisfied,
the theorem asserts the existence of a point (c, f (c)) on the graph of f such that a < c < b,
where the tangent is horizontal.

Proof of the Mean Value Theorem from Rolle’s Theorem. Given a function f
which satisfies the conditions of the Mean Value Theorem, we will consider, instead, the
function h defined by

f (b) − f (a)
h(x) = f (x) − f (a) − · (x − a) .
b−a
110
This is a definition, not a theorem.
Notes for Lecture Section 002, MATH 140 2008 09 2225

The continuity and differentiability properties of f imply that h has the same properties,
so h satisfies the first 2 conditions of Rolle’s Theorem. Moreover, we can see that h(a) =
h(b), so the 3rd condition of Rolle’s Theorem is also satisfied. Hence the conclusion of
Rolle’s Theorem holds: there exists a point c such that a < c < b and

f (b) − f (a)
h0 (c) = f 0 (c) − ,
b−a
is zero, and so f 0 (c) has the claimed property.

Corollary 5.61 (to the Mean Value Theorem) Rolle’s Theorem is the special case
where f (a) = f (b).

Corollary 5.62 (to the Mean Value Theorem) If f 0 (x) = 0 for all x in an interval
(a, b), then f is constant on the interval.

(Note that this Corollary, as stated, assumes nothing about f (a) and f (b); so it can’t say
anything about the end points of [a, b].) Proof: If we take any two distinct points x1 , x2
in the interval, we know that between them there is a point c such that f (x2 ) − f (x1 ) =
f 0 (c) · (x2 − x1 ) = 0, since the derivative must be 0 at c. Since f 0 (c) = 0, f (x2 ) = f (x1 ),
proving that, at any point in the interval, the function has the same value as at any
other point; or, equivalently, that the function has the same value at every point.

Corollary 5.63 (to the Mean Value Theorem) If f 0 (x) = g 0 (x) for every point x in
an interval (a, b), then there exists a constant k such that, for all x in the interval (a, b),

g(x) = f (x) + k .

Proof: To the function h defined by h(x) = g(x) − f (x) we apply the corollary immedi-
ately preceding. Since h0 (x) = g 0 (x) − f 0 (x) = 0 for all x, there exists a constant k such
that h( x) = k for all x; equivalently, such that g 0 (x) − f 0 (x) = k for all x.
This is the result I used earlier in these notes, when I solved a differential equation to find
all functions f that had the property that f 00 = (f 0 )2 .

Comparison with Tangential Approximations. In a linear or tangential approx-


imation to a function f near a point a we approximate

f (x) ≈ f (a) + f 0 (a) · (x − a) ,


Notes for Lecture Section 002, MATH 140 2008 09 2226

where the derivative is evaluated at the point a, but where an error is introduced — an
error that we have made no attempt to quantify. If we apply the Mean Value Theorem
on an interval [a, b], then, for x in that interval,

f (x) = f (a) + f 0 (c) · (x − a) ,

where the exact location of c in the interval (a, x) is not known, but where there is no
error in the equation.
Example 5.64 Prove that, for any x in [−1, 1],
π
arcsin x + arccos x =
2
Solution:
Proof using the calculus. Define f (x) = arcsin x + arccos x; then
1 −1
f 0 (x) = √ +√ =0
1 − x2 1 − x2
so, by one of the Corollaries above, f is a constant function over its domain. We
can determine the value of that constant by selecting a convenient value of x in
the domain; we might be tempted to choose x = 1,
π π
arcsin 1 + arccos 1 = +0=
2 2
and to conclude that π2 must be the value of the sum at any point in [−1, 1]. But,
if we were to make this choice, we would be overextending ourselves: our corollary
applies to an open interval, and one of the two functions, arcsin, is not defined in
an open interval containing x = 1. It would be possible to “repair” the corollary
to one that would permit this choice, but the version we have available now does
not permit it: neither of the inverse functions has a derivative at the end-point
of its domain. So, instead, let’s choose a convenient point in the interior of the
domain, where both functions have a derivative. Among the values that should
be convenient to students knowing just a little trigonometry are x = 0, x = ± √12 ,
x = ± 21 , x = ± √13 : for all of these values you should know the arcsine and arccosine.
I shall take x = 0:
π π
arcsin 0 + arccos 0 = 0 + = ,
2 2
which is the same result we obtained earlier by a proof of doubtful validity.
There is one difficulty with this proof: it applies only to the interval −1 < x < +1.
So what about x = −1 and x = +1. We could give an argument based on the
Notes for Lecture Section 002, MATH 140 2008 09 2227

continuity of f from the right at x = −1, and from the left at +1 to argue that
the limit of a constant is that constant, so the function value at the end points is
also π2 . Or, perhaps more easily, we could simply evaluate the function at x = ±1,
since we know the value of the inverse functions at those point.
Trigonometric proof. Define£ θ = arcsin ¤ x, so sin θ = x. Moreover, the image (=range)
of the arcsin function is − π2 , + π2 . Then
³π ´ π π
cos − θ = cos · cos θ + sin · sin θ
2 2 2
= 0 · cos θ + 1 · sin θ
= sin θ = x
Since − π2 ≤ θ ≤ π2 , 0 ≤ π2 − θ ≤ π. By definition of the inverse cosine function,
π
2
− θ = arccos x, so arcsin x + arccos x = π2 .

Warning! When a mathematical theorem requires that a function possess a certain


property for all x in some “universe”, then the failure of that property at even one
point can cause the result of the theorem to fail to apply. So, for example, the function
|x| is differentiable at all but one point. If that point, x = 0, is in the domain we are
considering, then we have no right to claim that the results of the two theorems apply.
For example, on the interval [−1, +1] there is no point where the derivative is equal to
0. In mathematics even one exception can destroy the rule.

A moderately difficult examination problem made much more difficult.


Example 5.65 The following problem appeared on the Final Examination in this course
in December, 2002:
1. [8 MARKS] Determine the derivative of the function
1+x
h(x) = arctan x + arctan 1 − arctan .
1−x
for x 6= 1 . You are expected to simplify your answer.
2. [7 MARKS] Use your solution to (the preceding) question to determine the value
of h(−5) . Only a solution using the previous result will be accepted. Reduce your
answer as much as possible; the examiners are aware that you do not have the use
of a calculator.
3. [THIS IS DIFFICULT, AND WAS NOT PART OF THE EXAMINATION QUES-
TION] Use your solution to (the preceding) question to determine the value of
h(+5) .
Notes for Lecture Section 002, MATH 140 2008 09 2228

Solution:

1. For all points x in the domain of h,


µ ¶
0 d 1+x
h (x) = arctan x + arctan 1 − arctan
dx 1−x
1 1 2
= 2
+0− µ ¶2 ·
1+x 1+x (1 − x)2
1+
1−x
= 0

2. We have shown that h has a derivative, and that the value of that derivative is 0
for all x. By the corollary above, that would appear to imply that the function h
is constant. But we must be careful. The corollary refers to an interval, and we
must also refer to an interval here. We will take the largest possible interval, to
obtain the strongest result.
The domain of the function h is the set of all x for which both arctan x and
1+x
arctan are defined. We know that the arctangent function is defined every-
1−x
1+x
where. But we need to restrict x so that is defined, and this requires the
1−x
exclusion of the point x = 1. Thus the domain of h is a “punctured” real line,
R − {1}. If we wish to apply the corollary, we will need to choose an interval that
avoids the excluded point x = 1. For x < 1 the corollary tells us that the value of
h is constant; in order to determine that constant, we need only evaluate h at a
“convenient” point. For example, take x = 0:

h(x) = h(0)
1+0
= arctan 0 + arctan 1 − arctan
1−0
= arctan 0 + arctan 1 − arctan 1
π π
= 0+ − =0
4 4
for all x < 1. In particular, this tells us that h(−5) = 0.

3. But the preceding computation does not apply for x > 1; all we know is that h is
constant over the interval. It’s not so easy to find a convenient value of x in this
interval, and it will turn out that the constant value of the function for this interval
Notes for Lecture Section 002, MATH 140 2008 09 2229


is not the same as for the interval containing −5.111√Try, for example x = 3; this
is a reasonable choice, since we know that tan π3 = 3. Then we have

h(x) = h( 3)

√ 1+ 3
= arctan 3 + arctan 1 − arctan √
1− 3

π π 1+ 3
= + − arctan √
3 4 1− 3

7π 1+ 3
= − arctan √
12 1− 3
But what is the last term? You may recall an identity found on one of the endpapers
of your textbook:
tan x + tan y
tan(x + y) = .
1 − tan x tan y
Apply this identity with x = π4 and y = π3 , to obtain

7π 1+ 3
tan = √ .
12 1−1· 3
¡ π π¢
As we defined the arctangent function using the ³ π π ´ − 2 , 2 , our inverse tan-
branch
gent function takes its values on the interval − , ; we restate the last result
2 2
in a form that we can use:

−5π 1+ 3
tan = √ ,
12 1−1· 3
and infer that à √ !
1+ 3 5π
arctan √ =− ,
1−1· 3 12
from which it follows that
µ ¶
7π −5π
h(x) = − =π
12 12
for all x > 1.
111
This is not a contradiction to the Corollary to the Mean Value Theorem: the corollary does not
apply to the entire domain of h because there is one point where the domain is broken, and where the
function fails to satisfy the conditions of the theorem; so the most we can do is to apply the corollary
separately on the two halves of the domain; of course, it could have happened that the constant values
for the two halves were the same — but it didn’t happen, and the constants are, indeed, different!
Notes for Lecture Section 002, MATH 140 2008 09 2230

4.2 Exercises
[1, Exercise 4.2.21, p. 286] “Show that a polynomial of degree 3 has at most 3 real
roots.”
Solution: Let the polynomial be named f (x), and suppose that it had 4 or more
distinct roots:
r1 < r2 < r3 < r4 . (191)
Then, by definition of a root, f = 0 at each of these 4 locations; by Rolle’s Theorem
f has a horizontal tangent somewhere between each successive pair of these roots.
That is, there exist real numbers c1 , c2 , c3 such that

r1 < c1 < r2 < c2 < r3 < c3 < r4 ,

f 0 (c1 ) = f 0 (c2 ) = f 0 (c3 ) .


But the derivative of f is a quadratic polynomial, and we are able to calculate its
roots: we know that there cannot exist more than 2 points where f 0 = 0. From
this contradiction we can infer that our assumption that there were 4 distinct roots
was unjustified. By reductio ad absurdum we have shown that the number of roots
cannot exceed 3.
This theorem can also be proved using purely algebraic methods, without any
mention of the calculus.
[1, Exercise 4.2.18, p. 286] “Show that the equation 2x − 1 − sin x = 0 has exactly
one real (solution).” The problem could also have been stated in the following,
equivalent way: “Show that the graphs y = 2x − 1 and y = sin x cross at exactly
one point.”
Solution:
1. We will show that the difference in height between where a line x = u crosses
the graph is equal to zero exactly once. That difference is 2u − 1 − sin u, and
we will denote it by f (u).
2. We use two theorems in this type of proof: the Intermediate Value Theorem
to prove that the function is 0 at least once; and the Mean Value Theorem or
Rolle’s Theorem to show that it is not 0 more than once.
3. Since f (u) is the difference between a polynomial and sin u — two differen-
tiable functions — it is differentiable everywhere;

f 0 (u) = 2 − cos u ≥ 2 − 1 = 1 > 0 .

Since it is differentiable everywhere, it is surely continuous everywhere.


Notes for Lecture Section 002, MATH 140 2008 09 2231

4. When u gets large, 2u also becomes large; but the subtracted sine function
remains between ±1; thus we can certainly find a value of u for which f (u) > 0.
For example, f (π) = 2π − 1 > 0. Similarly, when u becomes large negatively,
¡ ¢
f becomes large negatively; we need only one convenient point: f − π2 =
−π − 1 + 1 = −π < 0.
£ ¤
5. We now have an interval − π2 , π such that f is negative at one end-point,
positive at the other, and continuous throughout. By the Intermediate Value
Theorem, f is zero somewhere in the interval.
6. Suppose that f were equal to 0 twice — not necessarily in the interval, but
twice anywhere on the real line — say at points u = a and u = b. Then
we could apply Rolle’s theorem to conclude that there is a point c such that
a < c < b and f 0 (c) = 0. But we saw earlier that f 0 (u) > 0 for all u.
7. From this contradiction we conclude that it was not correct to assume there
were two points where f = 0; as we have already shown that there is at least
one such point, we can now conclude there is exactly one such point.

[1, Exercise 4.2.23, p. 286] “If f (1) = 10 and f 0 (x) ≥ 2 for 1 ≤ x ≤ 4, how small can
f (4) possibly be?”
Solution: Since f satisfies the hypotheses of the Mean Value Theorem on the given
interval, there exists c such that 1 < c < 4 and

f (4) − f (1)
= f 0 (c)
4−1
⇒ f (4) = f (1) + 3f 0 (c)
= 10 + 3f 0 (c) ≥ 10 + 3 · 2 = 16 .

The preceding is, essentially, the solution in the Student Solutions Manual [3, p.
154], but this solution is not quite complete: the author shows that f (4) ≥ 16, but
does not show that there could exist a function f for which this value of 16 would
be attained. One such function is 2x + 8. This function shows that the inequality
is “best possible”.

[1, Exercise 4.2.29, p. 286] “Use the Mean Value Theorem to prove the inequality

| sin a − sin b| ≤ |a − b|

for all a and b.”


Solution: Here application of the Mean Value Theorem to the function sin x on
the interval [a, b] tells us that there is a number c between a and b for which
Notes for Lecture Section 002, MATH 140 2008 09 2232

sin a − sin b
= cos c; but cosines cannot exceed 1 in absolute value, so
a−b
¯ ¯
¯ sin a − sin b ¯ | sin a − sin b|
¯ ¯
¯ a−b ¯≤1⇔ |a − b|
≤ 1 ⇒ | sin a − sin b| ≤ |a − b| ,

for all a, b.
Notes for Lecture Section 002, MATH 140 2008 09 2233

5.19 Supplementary Notes for the Lecture of November 12th,


2008
Release Date: Wednesday, November 12th, 2008
subject to correction

5.19.1 §4.3 How Derivatives Affect the Shape of a Graph


Review of previous lecture In the preceding lecture we considered a pair of equiv-
alent theorems: Rolle’s Theorem, and The “Mean Value” Theorem. These are existence
theorems, that prove, for a function f which is continuous on a closed interval [a, b] and
differentiable in its interior112 (a, b), of a point c at which f 0 (c) = f (b)−f
b−a
(a)
. I described
three different types of uses of this theorem:

• in the proofs of other theorems, and in several Corollaries, including the following

– a function whose derivative is 0 throughout (a, b) is constant on the interval;


– two functions whose derivatives are equal throughout an interval (a, b) differ
by the same constant throughout the interval

• proofs that a function have a certain precise number of solutions, based on applica-
tions of both the Mean Value Theorem and the Intermediate Value Theorem (where
one part of the argument is a reductio ad absurdum=proof by contradiction);

• proofs of identities by showing that the derivative of some function is zero, and
then evaluating the function at a “convenient” point

In this section we will begin by considering yet another Corollary to the Mean Value
Theorem.

What does f 0 say about f ? The first theorem is another corollary to the Mean Value
Theorem:

Corollary 5.66 (to the Mean Value Theorem:) (“Increasing/Decreasing Test” or


“I/D Test”)

1. If f 0 (x) > 0 at every point of an interval (a, b), then f is increasing on the interval.

2. If f 0 (x) < 0 at every point of an interval (a, b), then f is decreasing on the interval.
112
equivalently, for a function f which is differentiable in an open interval (a, b), continuous from the
right at b, and continuous from the left at a
Notes for Lecture Section 002, MATH 140 2008 09 2234

Remember that the textbook’s definitions of increasing and decreasing [1, p. 20] do not
permit equality; these are concepts that some mathematicians call strictly increasing and
strictly decreasing. There are variations of these results that can be proved to cover cases
when equality may hold. Based on the above results, we can prove the following “Test”.
This “test” describes conclusions that hold when f 0 exists in an interval on both sides of
c — even if it does not exist at c itself. When we speak of the left “side” of c, we mean
an interval of the form (c − ², c), where ² is a positive number — any positive number;
the right “side” is defined analogously. The theorem requires that certain conditions
occur on the “sides”: if they do not hold, then you cannot use this theorem to draw the
conclusions stated.
Theorem 5.67 (First Derivative Test) Suppose that c is a critical number of a con-
tinuous function f .
1. If f 0 changes from positive to negative at c, then f has a local maximum at c.

2. If f 0 changes from negative to positive at c, then f has a local minimum at c.

3. If f 0 is defined on the two sides of c, but does not change sign at c then f has
neither a local maximum nor a local minimum at c.

What does f 00 say about f ?


Definition 5.13 1. If the graph of f lies above all of its tangents on an interval [a, b],
the graph is said to be concave upward on the interval.

2. If the graph of f lies below all of its tangents on an interval [a, b], the graph is said
to be concave downward on the interval.

3. A point P = (c, f (c)) is an inflection point of the graph of the function f if

(a) f is continuous at x = c;
(b) the curve is concave upward on one side of P , and concave downward on the
other side.

Theorem 5.68 (Concavity Test) 1. If f 00 (x) > 0 for all x in I, then the graph of
f is concave upward on I.

2. If f 00 (x) < 0 for all x in I, then the graph of f is concave downward on I.

Theorem 5.69 (Second Derivative Test) Suppose that f 00 is continuous in a neigh-


bourhood of x = c, i.e., in some interval (c − ², c + ²), where ² is some positive real
number.
Notes for Lecture Section 002, MATH 140 2008 09 2235

1. If f 0 = 0 and f 00 (c) > 0, then f has a local minimum at c.

2. If f 0 = 0 and f 00 (c) < 0, then f has a local maximum at c.

3. If f 0 = 0 and f 00 (c) = 0, this theorem gives no information.

Note that the Second Derivative Test is inconclusive when f 00 (c) = 0. Consider the
functions f1 (x) = x3 and f2 (x) = ±x4 at x = 0. All of these functions have a zero first
derivative at x = 0, but the second derivatives also vanish. In the case of f1 there is
no extremum at x = 0; in the case of x4 there is a local minimum; while −x4 has a
local maximum. It is possible to refine the Second Derivative Test by considering higher
derivatives, but that is beyond Math 140.

What should I retain from this section? This section contains definitions and tests
that are required in the analysis of functions: all of these need to be retained and drilled
for examination purposes. However, you will not be expected to be able to reproduce
proofs of these results, although they are all simple consequences of the results in the
preceding two sections of the textbook.

Tabulation of function data. The textbook suggests that routine problems can be
best approached by tabulating the necessary data. If these methods appeal to you, use
them — they tend to keep your information orderly, and to reduce the chance that you
forget something. The rationale for the methods is that we are often interested in the
sign of a derivative or second derivative; since the sign of a product is related to the signs
of its factors, a table can be created where the signs of the factors are tabulated and
the sign of the product deduced. Not all functions can be factored into simple factors,
and for them these tabular methods are of limited value. The best way to prepare for
examination on these topics — after you have thoroughly absorbed the definitions and
theorems is by extensive drill, using the Student Solutions Manual to correct your work
and to guide you as to what constitutes a good solution.

4.3 Exercises As stated earlier, you are advised to work multiple problems and to
verify your solutions in the Student Solutions Manual.

[1, Exercise 4.3.12, p. 295] You are asked to

1. Find the intervals on which f is increasing or decreasing.


2. Find the local maximum and minimum values of f , (if any).
3. Find the intervals of concavity and the inflection points (if any).
Notes for Lecture Section 002, MATH 140 2008 09 2236

x2
Here f (x) = .
x2 + 3
Solution: This is a straightforward example, and the exposition is certainly im-
proved by tabulation of the data. We will be needing the derivatives. The first
derivative may be computed by the Quotient Rule and Chain Rule, but the compu-
tations are even easier if you notice that f (x) = 1 − x23+3 . Thus, by the Reciprocal
Rule and the Chain Rule,
3 6x
f 0 (x) = 0 + · 2x = 2 .
(x2 + 3) 2 (x + 3)2
The denominator is a power of a factor which is always positive; the numerator is
a constant multiple of the polynomial x, which changes sign at x = 0. Extending
the author’s style, I will tabulate, but will also include information about the end
points of the intervals.:

Interval x x2 + 3 f 0 (x) f
x<0 − + − decreasing on (−∞, 0)
x=0 0 + 0 critical point
x>0 + + + increasing on (0, ∞)

Note that f (−x) = f (x): this is an even function, and its graph is symmetric
under reflection in the y-axis. There is only one critical point. As f is differentiable
everywhere, we know that the local extrema will occur only at points where f 0 = 0;
so the only candidate is x = 0. The change of sign of f 0 tabulated above shows, by
the First Derivative Test, that f has a local minimum at x = 0; there are no local
maxima.
The second derivative may be computed by the Quotient Rule:
1(x2 + 3)2 − x · 2(x2 + 3)(2x) 18(1 − x)(1 + x)
f 00 (x) = 6 · 2 4
= .
(x + 3) (x2 + 3)3
To analyze its behaviour, we may again tabulate:

Interval 1 − x 1 + x x2 + 3 f 00 (x) f
x < −1 + − + − concave downward on (−∞, −1)
x = −1 + 0 + 0 inflection point
(because concavity changes)
−1 < x < 1 + + + + concave upward on (−1, −1)
x = +1 0 + + 0 inflection point
(because concavity changes)
x>1 − + + − concave downward on (1, ∞)
Notes for Lecture Section 002, MATH 140 2008 09 2237

0.8

0.6

0.4

0.2

0
-20 -10 0 10 20
x

x2
Figure 14: Graph of the Function and its horizontal asymptote, y = 1
x2 + 3

At the points x = ±1 f 00 is continuous, and changes sign. Consequently both of


these are inflection points of f .
We have determined the nature of the critical point x = 0 by using the First
Derivative Test; we could also have applied the Second Derivative Test there: since
f 00 (0) > 0, the critical point is a local minimum; the minimum value is 0. A sketch
of the graph with its horizontal asymptote is given in Figure 14, page 2237).
[1, Exercise 4.3.14, p. 295] The instructions are the same as for the preceding example,
but the function is
f (x) = cos2 x − 2 sin x ,
no longer a pure polynomial; however, it may be possible to treat it as a polynomial
in a trigonometric function, and to still use the methods used above. This function
is periodic with period 2π, since both the sine and cosine functions have periods
2π; thus it suffices to study its behavior on an interval of length 2π: the author
has selected the interval 0 ≤ x ≤ 2π.
While it has not been requested, we may observe by completion of the square that
f (x) = cos2 x − 2 sin x
= (1 − sin2 x) − 2 sin x = 2 − (sin x + 1)2
√ √
= ( 2 − 1 − sin x)( 2 + 1 + sin x)
This information can also be analyzed in a tabular form, and used to locate the
graph above and below the x-axis:√ the second factor is always positive, but the
first changes sign at x = arcsin( 2 − 1).
f 0 (x) = 2 cos x · (− sin x) − 2 cos x
Notes for Lecture Section 002, MATH 140 2008 09 2238

= −2(cos x)(sin x + 1)
f (x) = −2(cos2 x − sin2 x + cos x − sin x)
00
µ ¶
1
= 4 sin x − (sin x + 1)
2

The sign of the first derivative depends on the sign of the cosine, which is negative
in the 2nd and 3rd quadrants; the sign of the second derivative depends on the
sign of sin x − 12 . We tabulate the signs of the factors and the derivatives:

Interval cos x sin x + 1 f 0 (x) f


0 end point
0 < x < π2 + + − decreasing on (0, π2 )
π
2
0 + 0 critical point
π 3π
2
<x< 2
− + + increasing on ( π2 , 3π
2
)

2
0 0 0 critical point

2
< x < 2π + + − decreasing on ( 3π2
, 2π)
2π end point

We haven’t classified the critical points yet: at the critical point π2 the function
changes from decreasing to increasing, so the point is a local minimum; analogously,

2
is a local maximum. The instructions
¡ ¢ asked for the extremal
¡ ¢ values of the
function, so we need to compute f π2 = 0 − 2 = −1 and f 3π 2
= 0 − 2(−1) = 2.

0
-2 0 2 4 6
-1 x

-2

Figure 15: Graph of the Function cos2 x − 2 sin x


Notes for Lecture Section 002, MATH 140 2008 09 2239

1
Interval sin x − 2
sin x + 1f 00 (x) f
0 < x < π6 − + − concave downwards
π
6
0 + 0 inflection point
π 5π
6
<x< 6 + + + concave upwards

6
0 + 0 inflection point
5π 3π
6
<x< 2 − + − concave downwards

2
− 0 0 NOT inflection point

2
< x < 2π − + − concave downards
¡π 1¢ ¡ ¢
The coordinates of the inflection points are 6 , − 4 and 5π 6
, − 1
4
. There is no
inflection point at x = 3π
2
, even though the second derivative vanishes, because the
concavity does not change at that point. A sketch is given in Figure 15, on page
2238 of these notes.

[7, Exercise 4.3.20, p. 305] You are asked to

1. “Find the intervals on which f is increasing or decreasing.


2. “Find the local maximum and minimum values of f , (if any).
3. “Find the intervals of concavity and the inflection points (if any).”

Here f (x) = x ln x.
Solution: First observe that the domain of f is (0, ∞).
1 1
1. f 0 (x) = 1 · ln x + x · = ln x + 1. f 0 (x) > 0 ⇔ ln x > −1 ⇔ x > . Thus
x ¡1 ¢ e
the function
¡ ¢ is increasing over the interval e
, ∞ , and decreasing over the
interval 0, 1e .
1
2. f 00 (x) = . The second derivative exists at all points of the domain, and is
x
always positive: the graph is concave upwards throughout its domain. There
are no inflection points.

While we have answered the questions that were asked, we still do not have enough
information to graph the function. Investigation of the graph of this function was
continued in [7, Exercise 4.5.43, p. 323] (cf. Figure 16, [9, p. 165].) The graph can
be shown to be tangent to the y-axis at (x, y) = (0, 0).
Notes for Lecture Section 002, MATH 140 2008 09 2240

1.2

0.8

0.4

0
-0.5 0 0.5 1 1.5 2
x
-0.4

Figure 16: Graph of the Function x ln x


Notes for Lecture Section 002, MATH 140 2008 09 2241

5.20 Supplementary Notes for the Lecture of November 17th,


2008
Release Date: Monday, November 17th, 2008
subject to correction

5.20.1 §4.4 Indeterminate Forms and L’Hospital’s Rule


In [1, §2.3, §2.5] we discussed several limit laws that could be used to evaluate a combi-
nation of functions when all of the components had limits, subject to certain restrictions.
The first restriction we met, in [1, §2.3], was in the “Quotient Law”:

f (x) lim f (x)


x→a
lim = if lim g(x) 6= 0 ,
x→a g(x) lim g(x) x→a
x→a

in addition to the requirement that the two limits mentioned exist. Subsequently we
studied the concept of derivative, whose definition involved just such a limit — where
the limit of the denominator (of the ratio giving the slope of the secant to the graph) is
zero. Then, in [1, §2.5], we generalized our definitions to permit limits to have “values”
of ±∞. This raises the question of whether the “limit laws” hold for such limits also.
In [1, Exercise 2.4.44, p. 118] some such laws are proved to hold. The generalizations to
infinite limits can be seen to hold so long as we can avoid certain “forbidden” operations:
0 ∞
like ∞ · 0, ∞ − ∞, , . I have mentioned the possibility of “extending” the real
0 ∞
number system to include two new objects: +∞ and −∞. In practice this is what we
do informally, but it has the downside that we have to avoid certain types of operations
— like those combinations just mentioned — to which we cannot give a meaning. The
issue is not that we don’t know the correct value to attach to these combinations; it is
that we can show that there is no number — either real or ±∞ that can serve as a value,
without creating inconsistencies in the rest of our algebraic system. If we wish to work
with ±∞, then we must avoid these combinations. That means that the limit laws do
not hold for such combinations; but the corresponding limits may still exist, even though
we can’t find them using the limit laws. This is the subject of the present section — to
evaluate the limits of certain combinations of functions where the limit laws fail to hold.
We call these combinations “indeterminate forms”. The first case we consider is a ratio
of two functions whose limits are either both 0 or both one of ∞ or −∞. The name
of the theorem is, in practice, given either the archaic French spelling, L’Hospital’s
Rule, or the more modern spelling, where the silent “s” is replaced by a circumflex on
the preceding vowel: L’Hôpital’s Rule.

Theorem 5.70 (L’Hospital’s Rule) Suppose that


Notes for Lecture Section 002, MATH 140 2008 09 2242

1. f and g are differentiable;

2. g 0 (x) 6= 0 near a, except possibly at a; that one of the following pairs of conditions
holds:

(a) lim f (x) = 0 and lim g(x) = 0


x→a x→a

(b) lim f (x) = ∞ and lim g(x) = ±∞


x→a x→a

(c) lim f (x) = −∞ and lim g(x) = ±∞; and that


x→a x→a

f 0 (x)
3. lim 0 exists or is ∞ or −∞.
x→a g (x)

Then
f (x) f 0 (x)
lim = lim 0 .
x→a g(x) x→a g (x)

An analogous result holds for limits from the right and limits from the left.113

Instructors in Calculus courses often discourage their students from using L’Hospital’s
Rule because it renders the computation of limits so easy: many of the problems that
you will see in this section can be solved without the Rule, and you might wish to see if
you can accomplish that. But a technique that is so powerful is too important to ignore
for macho reasons, so you are expected to become competent in the use of L’Hospital’s
Rule; sometimes it could be the only method you will have to determine a limit; and,
even when other methods are possible, L’Hospital’s Rule will usually be faster. However,
there are skills that you need to acquire to use the Rule effectively, and these will come
from working many examples.
The theorem tells us that the last equation holds only if it known that the second
limit exists. Sometimes we may need to apply the theorem several times in succession.
When we do that, the equations we write are not known to be valid until the limit of
the very last ratio in the sequence has been shown to exist; only then all intervening
equations are justified.
113

Example 5.71 Let’s apply L’Hospital’s Rule first to the case of the derivative of a continuous function
f at x = a. We cannot use the Quotient Law on the ratio f (x)−f x−a
(a)
because both top and bottom have
limit equal to 0. If f is differentiable the conditions of L’Hospital’s Rule are satisfied, and the limit is
0
lim f 1(x) = lim f 0 (x) which will equal f 0 (a), provided we know that f 0 is continuous at x = a. This
x→a x→a
verifies the Rule in a special case, shows that L’Hospital’s Rule is not completely efficient, as it requires
the continuity of f 0 .
Notes for Lecture Section 002, MATH 140 2008 09 2243

Equal signs in applications of l’Hospital’s Rule L’Hospital’s Rule, where applica-


ble, replaces a given limit of certain types of ratios by the limit of another ratio, hopefully
easier to evaluate. If the second limit cannot be evaluated, then the entire application is
worthless. For this reason the authors of the solution manual for your textbook [3] have
H
introduced the novel symbol =. The message conveyed by the H is that the equality
H
will not be justified until the sequence of =’s and =’s ends in the limit of a ratio which
H
can be evaluated. When that last limit has been evaluated, all intervening =’s can be
replaced by =’s.114 This is actually the spirit of some of the other Limit Laws that
we have learned earlier, but we haven’t used a modified equal sign in those cases. For
example, the Product Law replaces the problem of evaluating the limit of a product by
evaluating the limits of its factors; but, if the factors do not all have a limit, or if the
product of the limits is undefined, then the Law fails.
2
ex − 1 − x − x2
Example 5.72 [1, Exercise 4.4.22, p. 304] Evaluate lim .
x
x→0 x3
e −1
Solution: First look at lim , and then at [1, Exercise 4.4.21, p. 304], which is solved
x→0 x
in the Student Solutions Manual. These are a progression of more difficult versions of
the same problem. In the first case we may apply L’Hospital’s Rule since lim (ex − 1) =
x→0
1 − 1 = 0, by the continuity of the exponential function, and the fact that e0 = 1. Then
we have
ex − 1 H ex − 0
lim = lim = e0 = 1 .
x→0 x x→0 1
We could also have proved this without L’Hospital’s Rule, by observing that
¯
ex − 1 ex − e0 d x ¯¯
lim = lim = e = ex |x=0 = 1 .
x→0 x x→0 x − 0 dx ¯x=0

Proceeding to the present example,


x2 2x
ex − 1 − x − 2 H ex − 0 − 1 − 2
lim = lim
x→0 x3 x→0 3x2
H ex − 0 − 22
= lim
x→0 6x
H ex − 0
= lim
x→0 6
114 H
The textbook symbol = is not a “standard” symbol which is understood throughout the mathemat-
ical world, but it is a very useful symbol to help you read the solution manual to your textbook; I will
not always use it in these notes.
Notes for Lecture Section 002, MATH 140 2008 09 2244

whose limit we know by the continuity of ex to be 16 . The intermediate = signs are not
justified until this last step, where we prove that the last limit exists, hence the preceding
one, hence the one before that, etc.
Of course, just because a function whose limit is sought is presented as a quotient
does not guarantee that L’Hospital’s Rule is applicable to the problem. Don’t forget
that the numerator and denominator need to either both have limits equal to 0, or both
have limits of either +∞ or −∞. So, for example, you must not use L’Hospital’s Rule
to solve the following problem:
x + sin x
Example 5.73 [1, Exercise 4.4.31, p. 305] lim =?
x→0 x + cos x
Solution: While the limit of the numerator as x → 0 is, indeed, 0; the limit of the
denominator is 0 + 1 = 1. You may not apply L’Hospital’s Rule, and there is no need
to attempt to do so, since the Quotient Rule applies here, and the limit is the ratio
of the limits, i.e. 01 = 0. (The “answer” that you might have obtained here, using the
inapplicable Rule, is irrelevant, since the operation cannot be justified by the theorem!)

Indeterminate Products L’Hospital’s Rule is available only for quotients of functions


with the properties described earlier: you do not have a counterpart to this theorem for
products. So, if you are presented with a product of functions whose limit is required,
you must either apply one of the other Rules you have available, or attempt to transform
the product into a quotient by moving a factor to its reciprocal in the denominator of
a fraction. This could require some experimentation: if you move the “wrong” factor,
you may transform the function into a ratio where the ratio of the derivatives is “more
complicated” than the original function; unless L’Hospital’s Rule leads to a ratio whose
limit can be found, all steps in the transition to that ratio become invalid.
1
Example 5.74 [1, Example 4.4.52, p. 305] lim (xe x − x) =?
x→∞
1
³ 1 ´
Solution: The function may be factorized as xe x − x = x e x − 1 . Now we can move
one of the factors as a reciprocal into the denominator of a fraction. There are two
choices, either
1
³ 1 ´ 1
ex − 1
xe − x = x e − 1 =
x x
1
x

1
³ 1 ´ or x
xe x − x = x e x − 1 = 1
1
e x −1

Of these the second does not appear to be amenable, and we shall try the first first.115
115
But we could be wrong, and it might be that a very complicated-looking ratio might lend itself to
simplification after the differentiations are carried out.
Notes for Lecture Section 002, MATH 140 2008 09 2245

³ 1 ´
After ascertaining that lim e x − 1 = 1 − 1 = 0, and lim x1 = 0, ensuring that
x→∞ x→∞
L’Hospital’s Rule is applicable, we proceed:
³ 1
´ 1
ex − 1
lim xe − x
x = lim 1
x→∞ x→∞
x
1 ¡ ¢
e x · − x12
= lim
x→∞ − x12
1
= lim e x
x→∞
1
lim
= ex→∞ x (by continuity of ey at y = 0)
= e0 = 1 .

It might have been wiser to change the variable during this process, in order to simplify
the differentiation:
³ 1 ´ 1
ex − 1
lim xe x − x = lim 1
x→∞ x→∞
x
ey − 1
= lim+
y→0 y
(defining y = x1 and letting it → 0+ )
ey
= lim+ = e0 = 1.
y→0 1

And we could have observed that the quotient was precisely the difference quotient which
d y
defines dy e evaluated at y = 0.

Indeterminate Differences The example just worked could also be interpreted as


a difference of two functions, both of which become infinite. In such a situation the
Difference Law is not applicable, but it may still be possible to transform the function
into a quotient to which L’Hospital’s Rule is applicable.
µ ¶
1 1
Example 5.75 (cf. [1, Exercise 4.4.47, p. 305]): lim − =?
x→1 ln x x − 1
Solution: In this case the transformation to a quotient appears to “complicate” the
function, but we are still able to proceed.
µ ¶
1 1 x − 1 − ln x
lim − = lim
x→1 ln x x − 1 x→1 (x − 1) ln x

1 − x1
= lim
x→1 1 ln x − (x − 1) · 1
x
Notes for Lecture Section 002, MATH 140 2008 09 2246

1
x2
= lim
x→1 1
+ x12
x
1
= lim
x→1 x + 1
1
=
2
1
(by continuity of x+1
at x = 1.)
In this last example we might have been tempted to eliminate the negative powers of x;
the saving in effort would be insignificant:
1 − x1 x−1
lim 1 = lim
x→1 1 ln x − (x − 1) · x→1 x ln x + x − 1
x
1
= lim
x→1 1 ln x + x · 1 + 1 − 0
x
1 1 1 1
= lim = = (by continuity of ln x
at x = 1).
x→1 ln x + 2 0+2 2

Indeterminate Powers Since a function of the form f (x)g(x) with f (x) > 0 may be
¡ ¢g(x)
interpreted as eln f (x) = eg(x)·ln f (x) , limits of functions of this type may be found by
finding the limit of the exponent, and then applying the continuity of the exponential
function to infer that the limit of the exponential is equal to the exponential of the
limit. Alternatively, one may use logarithmic differentiation and evaluate the limit of
the logarithm of the given function. But, when the time comes to apply L’Hospital’s
Rule, be sure to check that the Rule is, indeed, applicable.
õ ¶2x+1 !
2x − 3
Example 5.76 [1, Exercise 4.4.64, p. 305] lim =?
x→∞ 2x + 5
Solution: To apply “logarithmic differentiation”, first assign a name to the function;
then take its logarithm and find the limit of the logarithm:
µ ¶2x+1
2x − 3
y =
2x + 5
µ ¶
2x − 3
ln y = (2x + 1) ln
2x + 5
= (2x + 1) (ln(2x − 3) − ln(2x + 5))
ln(2x − 3) − ln(2x + 5)
= 1
2x+1
ln(2x − 3) − ln(2x + 5)
⇒ lim ln y = lim 1
x→∞ x→∞
2x+1
Notes for Lecture Section 002, MATH 140 2008 09 2247

But note that I have made a tactical error here: by expressing the logarithm of the
quotient as the difference of the logarithms, I have created an indeterminate difference,
and am unable to establish its limit, so I can’t show that l’Hospital’s Rule is applicable.
I need to observe that
µ ¶ µ ¶
2x − 3 2x − 3
lim ln = ln lim
x→∞ 2x + 5 x→∞ 2x + 5

(by continuity of ln)


µ ¶
2 − x3
= ln lim
x→∞ 2 + 5
x
= ln 1 = 0 ;
1
and, obviously, lim = 0. Now we can return to the earlier expression and apply
x→∞ 2x + 1
L’Hospital’s Rule:
¡ ¢
ln 2x−3
2x+5
lim ln y = lim 1
x→∞ x→∞
2x+1
ln(2x − 3) − ln(2x + 5)
= lim
x→∞ 1
2x + 1
2 2

= lim 2x − 3 2x + 5
x→∞ 1
− ·2
(2x + 1)2
−8(2x + 1)2
= lim
x→∞ (2x − 3)(2x + 5)
1
−8(2 + )2
= lim x
x→∞ 3 5
(2 − )(2 + )
x x
= −8

Then
õ ¶2x+1 !
2x − 3
lim = lim y
x→∞ 2x + 5 x→∞

= lim eln y
x→∞
lim ln y
= ex→∞
Notes for Lecture Section 002, MATH 140 2008 09 2248

(by continuity of the exponential)


−8
= e

4.4 Exercises

[7, Exercise 4.4.6, p. 313] “Find the limit. Use l’Hospital’s Rule where appropriate.
If there is a more elementary method, consider using it. If l’Hospital’s Rule doesn’t
x+2
apply, explain why. lim 2 .”
x→−2 x + 3x + 2

Solution: This problem does not require l’Hospital’s Rule. If the denominator of
the fraction is factorized, into (x + 1)(x + 2), then the fraction can be seen to equal
1
, whose limit may be found by the Quotient Law; or by using our result that
x+1
rational functions are continuous at points which are not discontinuities, since the
1
only discontinuity of is at x = −1, and we are taking the limit as x → −2.
x+1
1
Thus we see that the limit is −2+1 = −1.
Since both numerator and denominator approach 0 as x → −2, we may use
1 1
l’Hospital’s Rule. The ratio of the derivatives is → = −1, as
2x + 3 −4 + 3
before.
Mathematicians prefer not to use l’Hospital’s Rule where the problem has an easy
solution without it. This may derive from the “macho” element of traditional
mathematics. Sometimes on examinations you may be asked to find a limit and
explicitly instructed not to use l’Hospital’s Rule.
e3t − 1
[1, Exercise 4.4.12, p. 304] “Find the limit. ... lim ”.
t→0 t
Solution: Since both numerator and denominator approach 0, this problem may
be solved by l’Hospital’s Rule. (More precisely, we may attempt to solve it using
l’Hospital’s Rule: the Rule does not always yield a solution — if the ratio of deriva-
tives does not have a limit, or if it is not again amenable to another application
of l’Hospital’s Rule then the Rule fails to provide any information.) The ratio of
3e3t
derivatives is . By the continuity of the exponential, the limit may be obtained
1 3t
by evaluating this last function at t = 0, which is 3e1 , to which the Quotient Law
may be applied, yielding a limit of 3e0 = 3. Of course, this problem could also be
viewed as finding the value at t = 0 of dtd (e3t ), and l’Hospital’s Rule would not be
needed.
ln 2
[1, Exercise 4.4.58, p. 305] “Find the limit. ... lim x 1+ln x ”.
x→∞
Notes for Lecture Section 002, MATH 140 2008 09 2249

Solution:
ln 2 ¡ ¢ ln 2
lim x 1+ln x = lim eln x 1+ln x
x→∞ x→∞

lim e((ln x)· 1+ln x )


ln 2
=
x→∞
µ ¶
ln 2
lim (ln x) ·
= ex→∞ 1 + ln x

We now investigate the limit in the exponent.


µ ¶
ln 2 ln x
lim (ln x) · = (ln 2) · lim
x→∞ 1 + ln x x→∞ 1 + ln x
1
= (ln 2) · lim 1
x→∞
ln x
+1
lim 1
x→∞
= (ln 2) · ¡ ¢
lim ln1x + 1
x→∞
1
= (ln 2) · = ln 2
0+1
which permits us to assert that
ln 2
lim x 1+ln x = eln 2 = 2 .
x→∞

We didn’t have to use l’Hospital’s Rule, but needed the fact that lim ln t = ∞;
t→∞
but we would have needed to use this fact to apply l’Hospital’s Rule anyhow. This
fact follows from the observation that, as t → ∞, t passes through values which
are powers en , and ln (en ) = n → ∞ as n → ∞.

Example 5.77 This lecture would be incomplete without an example where a limit of
an indeterminate fraction does exist, but where l’Hospital’s Rule breaks down and is
x + sin x
unable to reveal that limit. Consider lim . Here lim (x + sin x) = ∞ = lim x ,
x→∞ x x→∞ x→∞
so we cannot use the Quotient Law. But, if we apply L’Hospital’s Rule, we obtain
x + sin x H 1 + cos x
lim = lim .
x→∞ x x→∞ 1
Hitherto, in the examples of l’Hospital’s Rule that we have studied, the limit of the
last ratio could be calculated, and that would validate the differentiation operations,
H
permitting us to replace = by =. But, in this case, the limit of the numerator does not
Notes for Lecture Section 002, MATH 140 2008 09 2250

exist, while the limit of the denominator is 1. So, in this case, the attempted application
of l’Hospital’s Rule must be aborted, with no progress in evaluating the limit of the
original function. That does not imply that the original limit does not exist; in fact
µ ¶
x + sin x sin x
lim = lim 1 +
x→∞ x x→∞ x
sin x
= lim 1 + lim
x→∞ x→∞ x
by the Sum Law
sin x
= 1 + lim
x→∞ x
since the limit of a constant is that constant
= 1 + 0.

(Since
1 sin x 1
−1 ≤ sin x ≤ 1 ⇒ − ≤ ≤ for x > 0 .
x x x
As x → ∞, the extremes of the last pair of inequalities both approach 0, so the “Squeeze”
sin x
Theorem ensures that lim = 0.)
x→∞ x

5.20.2 Sketch of Solutions to Problems on the December, 2003 Final Exa-


mination
We do not have files of solutions to old examinations, as I try to discourage students from
studying from old exams. However, the following solutions were prepared for discussion
in my classes in November, 2004, and I am making them available to you. I plan to
discuss a different examination with this year’s class, but you are invited to ask me or
the TA’s about any of the following examination questions. The sketches I provide in
these notes do not constitute “official” solutions of the examination questions, but are
designed to explain the questions to current student.
Students were advised that there were two kinds of problems on this examination,
each clearly marked as to its type:

• “Some of the questions on this paper require that you SHOW ALL YOUR WORK!
Their solutions are to be written in the space provided on the page where the
question is printed. When that space is exhausted, you may write on the facing
page. Any solution may be continued on the last pages, or the back cover of the
booklet, but you must indicate any continuation clearly on the page where the
question is printed!
Notes for Lecture Section 002, MATH 140 2008 09 2251

• “Some of the questions on this paper require only BRIEF SOLUTIONS ; for these
you are expected to write the correct answer in the box provided; you are not asked
to show your work, and you should not expect partial marks for solutions that are
not completely correct.

“You are expected to simplify your answers wherever possible.”

1. BRIEF SOLUTIONS
[2 MARKS EACH] Give the limit in each of the following cases. If the limit does
not exist, or is +∞ or −∞, write “DOES NOT EXIST”, +∞, or −∞ respectively.
15 − 2x − x2
(a) lim− =?
x→3 x−3
Solution: The intention was that students would observe that the quadratic
numerator factors into −(x−3)(x+5), so that the function is equal to −(x+5)
at all points except x = 3, where it is undefined. For the purpose of finding
the one-sided limit as x → 3− , where the value of the function at x = 3 is
irrelevant, we can use the textbook result that polynomials are continuous
everywhere; hence the limit of −x − 5 is obtained by replacing x + 5 by that
function’s value at x = 3, i.e., by −3 − 5 = −8.
Of course, since students did not have to show their work, they could have
found this limit also by using l’Hospital’s Rule:

15 − 2x − x2 −2 − 2x
lim− = lim− = −2 − 6 = −8
x→3 x−3 x→3 1

7x5 + 2x3 − x2 + 11
(b) lim =?
x→−∞ x5 − 3x4 + 2
Solution: For the limit of a quotient of polynomials as x becomes infinite
positively or negatively, I recommend dividing out the leading power of x:
¡ ¢
7x5 + 2x3 − x2 + 11 x5 7 + x22 − x13 + x115
lim = lim ¡ ¢
x→−∞ x5 − 3x4 + 2 x→−∞ x5 1 − x3 + x25
7 + x22 − x13 + 11
x5
= lim
x→−∞ 1 − x3 + x25
7
=
1
Here also, l’Hospital’s Rule could have been used (5 times).
Notes for Lecture Section 002, MATH 140 2008 09 2252

tan 5x
(c) lim =?
x→0 sin 4x
Solution: We try to transform the function into a product of quotients of the
form sinx x :

tan 5x 5x sin 5x 1 4x
lim = lim · ·
x→0 sin 4x x→0 5x cos 5x 4x sin 4x
sin 5x 1 4x 5
= lim · · ·
x→0 5x cos 5x sin 4x 4
sin 5x 1 4x 5
= lim · lim · lim · lim
x→0 5x x→0 cos 5x x→0 sin 4x x→0 4
1 5 5
= 1· ·1· =
1 4 4
This problem could have been solved using l’Hospital’s Rule.
30 + 6x
(d) lim − =?
x→−5 |5 + x|
Solution: This is a limit as x → 5− ; to the left side of 5, |5 + x| = −(5 + x).
Accordingly
30 + 6x
lim − = lim− −6 = −6 .
x→−5 |5 + x| x→5
√ √
(e) lim ( x2 + x − x2 − 4x) =?
x→∞
Solution: This function is a difference of two functions, both of which become
infinite as x → ∞. We need to transform it into a function which does not
involve an undefined combination of functions which are becoming infinite.
The “standard”√way of attacking
√ this problem is to rationalize the surds, by
multiplying by x2 + x − x2 − 4x and dividing by the same quantity. We
obtain
(x2 + x) − (x2 − 4x) 5x
√ √ =√ √
2 2
x + x + x − 4x x + x + x2 − 4x
2

At this point we could, perhaps, use l’Hospital’s Rule. But a simpler attack
is to divide numerator and denominator by x; since x is positive, this is
equivalent to dividing the surds by x2 under the root signs. We obtain
5x 5
√ √ =q q
x2 + x + x2 − 4x 1+ 1
+ 1− 4
x x

5
which approaches 2
as x → ∞.
Notes for Lecture Section 002, MATH 140 2008 09 2253

(f) lim x2 ex =?
x→−∞
Solution: While some of the preceding problems could be solved by l’Hospital’s
Rule, but also by other — preferred — methods, this problem, for most stu-
dents, would need l’Hospital’s Rule.

x2 2x 2
lim x2 ex = lim = lim = lim −x = 0
x→−∞ x→−∞ e−x x→−∞ −e−x x→−∞ e

2. BRIEF SOLUTIONS
[2 MARKS EACH] Determine each of the following derivatives.
d sin t2
(a) e =?
dt
Solution: This function is the composition of three functions: first t 7→ t2 ; then
u 7→ sin u, finally v 7→ ev . To differentiate we begin with the last function:
the derivative of ev is ev ; here v = sin t2 . We multiply by the derivative with
respect to u of sinu; here u = t2 . Finally we multiply by the derivative of t2 .
The product is ¡ ¢
2
esin t · cos t2 · 2t .
d
(b) ln(e4x ) =?
dx
Solution: We expect students to observe that, as the exponential and loga-
rithm functions are mutual inverses, this function is equal to 4x, so its deriva-
tive is 4. Students could fail to make this observation and still solve the
problem correctly, using the Chain Rule.
d 4
(c) |x | =?
dx
Solution: Since the exponent is even, |x4 | = x4 , so the derivative is simply
4x3 . Students who attempted to solve the problem from first principles would
likely fail to observe that it is differentiable at x = 0, but could still determine
the correct derivative elsewhere.
d √
(d) arcsin x =?
dx
Solution: This is a straight-forward Chain Rule problem:
d √ 1 1 1
arcsin x = p · √ = √
dx 1 − |x| 2 x 2 x − x2

and is defined only for 0 < x < 1.


Notes for Lecture Section 002, MATH 140 2008 09 2254

(e) If y(x) satisfies the equation


5 2 4 0
(y(x)) + x (y(x)) + 9x = 1 , then y (0) =?
Solution: It was intended that this problem be solved by implicit differentia-
tion with respect to x:

5y 4 y 0 + 1y 2 + x · 2y · y 0 + 36x3 = 0

which can be solved for y 0 in terms of x and y.


(f) If g(x) =, g 0 (0) =?
Solution: We might be tempted to solve this problem by “logarithmic differ-
entiation”. In the latter we find the logarithm of g(x):
x x
ln g(x) = · ln(x6 ) = · 6 ln x = 3x ln x
2 2
g 0 (x)
whose derivative, g(x)
is 3(ln x + 1). Hence the derivative of g(x) is

3x3x · (ln x + 1) .

However, this derivation applies only for x > 0, so it is not relevant for the
problem given. We do see, though, that as x → 0+ , this function approaches
−∞, suggesting that there is no derivative anyhow, as the slope is approaching
the vertical.
So how can we attack this problem? We will have to appeal to the definition
of the derivative, i.e.,
x3x − 1
lim .
x→0 x
In order to compute this limit, we will have to determine the two one-sided
limits separately. Since
x3x − 1 e3x ln x − 1
lim = lim+
x→0+ x x→0 x
and we can show by l’Hospital’s Rule that x ln x → 0 as x → 0+ , the nu-
merator approaches 0, as does the denominator, we may apply l’Hospital’s
Rule:
e3x ln x − 1
lim+ = lim+ 3(ln x + 1)x3x .
x→0 x x→0
Thus the earlier formula does apply in the limit, which is found to be −∞.
And, as x → 0− , we have
x3x − 1 −e3x ln(−x) − 1
lim− = lim− .
x→0 x x→0 x
Notes for Lecture Section 002, MATH 140 2008 09 2255

An analysis similar to the case from the right shows that the limit here is +∞.
Our definition of derivative does not permit infinite values: this function is not
differentiable at 0. In fact, the graph has a cusp at the point (x, y) = (0, 1).

3. BRIEF SOLUTIONS
[3 MARKS EACH]

(a) Give equations for all of the vertical asymptotes of the graph of
x
f (x) = .
x2 +x−2
If there is none, write “NONE”.
Solution: The denominator factorizes into (x + 2)(x − 1) which is zero at
x = −2 and x = 1. The numerator remains non-zero at these points, so the
quotient is undefined and approaches ±∞ as x → −2 and x → 1 from either
side. Hence x = −2 and x = 1 are both asymptotes of this graph.
(b) Determine all values of the constant c that will make the function
½
−4x + c when x < 0
f (x) =
(x + c)2 when x ≥ 0

continuous from the right at x = 0. If there is none, write “NONE”.


Solution: Suppose first that the question had read “Determine all values of the
constant c that will make the function continuous at x = 0.” Then we could
proceed as follows. The functions that are being pieced together are both
polynomials, and have one-sided limits at all values. As x → 0− , f (x) →
−4(0) + c = c. As x → 0+ , f (x) → (0 + c)2 = c2 . For continuity these values
must be equal: c = c2 or c(c − 1) = 0, so c = 0 or 1. Both of these values
make the function continuous (not just continuous from one side).
But the problem asked that the function be “continuous from the right at
x = 0”. Since the second line of the definition of the function gives its value
as (x + c)2 whenever x ≥ 0, this function is continuous from the right for all
values of c.
(c) Determine all horizontal asymptotes to the following curve; if there is none,
write “NONE”.
y = arctan(x2 )
Solution: As x → ±∞, x2 → ∞, and its arctangent→ π2 . Thus there is just
one horizontal asymptote, y = π2 .
Notes for Lecture Section 002, MATH 140 2008 09 2256

3
(d) If tanh x = , sinh x =?
5
Solution: This problem can be approached in a variety of ways. If a student
remembered the identity, tanh2 x + sech2 x = 1, he could then argue that
sech2 x = 25
16
. Then, observing that the hyperbolic secant is always positive,
he could conclude the correct value.
But we don’t expect you to memorize this identity. So, instead, solve the
equation
ex − e−x 3
x −x
=
e +e 5
x
to obtain e = ±2, where the + sign must be taken, because real exponentials
are never negative, so
1 1
ex − ex
2− 2 3
sinhx = = = .
2 2 4

4. BRIEF SOLUTIONS
[4 MARKS EACH] The examiners are aware that you do not have a calculator.

(a) Determine all points on the curve

y 3 − x2 = 4

where the tangent is horizontal. If there is none, write “NONE”.


Solution: By implicit differentiation with respect to x we see that 3y 2 ·y 0 −2x =
0. There is no point on the curve where y = 0, as this would entail that x2
be negative. Hence the only way in which y 0√= 0 is that x = 0. The point of
contact of this tangent with the curve is (0, 3 4).
(b) On the interval −2 < x < 0, determine the values of x at which the function
f has a local (=relative) minimum, if it is known that

f 0 (x) = (2x + 1)(x + 1)2 (x + 3) .

If there is no local minimum, write “NONE”.


Solution: The derivative vanishes at x = − 12 , x = −1, and x = −3. Of these
3 points, the last is outside the domain of the function. The other 2 yield
two critical points. To determine which — if either — of them yields a local
minimum, we must use either the First or the Second Derivative Test. Because
Notes for Lecture Section 002, MATH 140 2008 09 2257

of the complication of the function, it is easiest to use the First Derivative


Test:
x 2x + 1 x + 1 (x + 1)2 x + 3 f 0 f
−2 ≤ x < −1 − − + + + decreasing
x = −1 critical point
−1 < x < − 12 − + + + − decreasing
x = − 12 critical point
1
−2 < x ≤ 0 + + + + + increasing
At x = −1 the function does not change from increasing/decreasing to de-
creasing/increasing, so this is NOT a local maximum; at x = − 12 it changes
from decreasing to increasing, so the function has a local minimum.
(c) Find f (2) if it is known that
x2 − x + 1
f 0 (x) = , and
x
f (1) = 5 .

Solution: The general antiderivative of f is


Z µ ¶
1 x2
F (x) = x−1+ dx = − x + ln x + C.
x 2
1 11
Imposing the condition that f (1) = 5 yields 5 = 2
− 1 + C, so C = 2
. It
follows that
22 11 11
f (2) = − 2 + ln 2 + C = 2 − 2 + ln 2 + = ln 2 + .
2 2 2
5. SHOW ALL YOUR WORK!
[3 MARKS EACH]
For each of the following descriptions of a function or functions, either
• give an example of functions with the properties stated, or
• name or state a theorem, law, or rule which can be used to show that no such
function or functions exist.

(a) f (3) = 0, f (1) = −4, f 0 (x) ≤ 1 for all x.


Solution: By the Mean Value Theorem there exists c such that 1 < c < 3 and
f 0 (c) = 0−(−4)
3−1
= 2 > 1, which contradicts the hypothesis that the value of the
derivative can never exceed 1. From this contradiction we conclude that no
such function f can exist.
Notes for Lecture Section 002, MATH 140 2008 09 2258

(b) f is continuous on [−4, 4], f (−3) = −1, f (3) = 2, f (x) 6= 0 for all x.
Solution: A continuous function must, by the Intermediate Value Theorem,
assume all values between any two values it assumes. Thus there must be a
point where f takes on the value 0 that lies between the values of −1 and
2 that is is known to assume. From this contradiction we conclude that no
function f of this type exists.
³ ´
(c) lim f (x) = 3, f lim x = 2 .
x→4 x→4
Solution: If f were continuous at 4, then, since lim x = 4, the two hypotheses
x→4
would be contradictory. We conclude that f cannot be continuous at 4. An
example of a function f of this type is
½
3 when x 6= 4
f (x) =
2 when x = 4

(d) f 0 (1) = 5 and f is not continuous at x = 1 .


Solution: We know that a function which has a derivative at x = 1 must be
continuous at x = 1. Thus the description is self-contradictory, and there
cannot exist any such function f .

6. SHOW ALL YOUR WORK!


[12 MARKS] A balloon leaves the ground 100 metres from an observer, and rises
vertically at the rate of 40 metres per minute. Determine the rate at which the
angle of inclination of the observer’s line of sight (the angle between the line of
sight and the horizontal) is increasing at the instant when the balloon is exactly
100 metres above the ground?
Solution: It appears that the examiners intended that the observer’s eye be at
ground level. Let’s place that eye at the origin, and the balloon on the x-axis
at position (100, 0) at time t = 0. The position of the balloon at time t will
be (100, 40t). The angle subtended at the observer’s eye at time t is, therefore,
2
40t
arctan 100 , whose derivative is 5
2 . The balloon will be at height 100 when
1+( 2t
5 )
t = 25 , at which time the value of the derivative is 15 radian/minute.

7. SHOW ALL YOUR WORK!



Let f (x) = x x + 3 .
(a) [2 MARKS] State the domain of f .

Solution: x + 3 is defined precisely when x ≥ −3; the product with x does
not restrict the domain. Thus the domain of f is x ≥ −3.
Notes for Lecture Section 002, MATH 140 2008 09 2259

(b) [4 MARKS] Find the intervals of increase and the intervals of decrease of f .
(c) [4 MARKS] Find the absolute (global) maximum and minimum values of f
— if any — and the points where they are attained.
Solution: Differentiation by the product rule, followed by algebraic simplifi-
cation, yields
3 x+2
f 0 (x) = · √ .
2 x+3
This function is 0 when x = −2, and is not defined when x = −3. For
−3 < x < −2 f 0 < 0 so f is decreasing; for −2 < x f 0 > 0 so f is increasing.
It follows that x = −2 is a global minimum of f . The global minimum value
of f is, therefore f (−2) = −2.
(d) [3 MARKS] Determine the intervals of concavity upwards and the intervals of
concavity downwards, and the inflection points, if any.
Solution: We find after another differentiation and simplification that
3 x+4
f 00 (x) = · √ .
4 ( x + 3)3

This function is positive throughout the domain of f , so the graph is always


concave upward, and there are not inflection points.
(e) [3 MARKS] Sketch the graph of y = f (x).

8. SHOW ALL YOUR WORK!


Let t represents time measured in seconds, and let C be a constant that is to
be determined. A particle moves so that its position on the x axis at time t is
x(t) = t3 − 12t2 + 36t + C.

(a) [3 MARKS] Determine the acceleration of the particle as a function of time.


Solution:

v(t) = x0 (t) = 3t2 − 24t + 36


a(t) = v 0 (t) = x00 = 6t − 24

(b) [6 MARKS] The speed of the particle is the absolute value of its velocity.
Determine the time intervals when the speed is increasing.
Solution: v(t) = 3(t − 2)(t − 6), while a(t) = 6(t − 4). The various signs will
change only at values t = 2, 4, 6. Let us tabulate the behavior of the velocity
Notes for Lecture Section 002, MATH 140 2008 09 2260

and speed first:


d|v|
t t−2 t−6 v |v| dt
t<2 − − + v 6(t − 4)
2<t<6 − + − −v −6(t − 4)
t>6 + + + v 6(t − 4)

We see that the derivative of the speed will be changing sign at t = 4. We


have
d|v|
t dt
t<2 −
2<t<4 +
4<t<6 −
6<t +
Thus the speed is increasing in the intervals 2 < t < 4 and 6 < t.
(c) [3 MARKS] Determine the value of C if it is known that the particle is at the
origin the whenever the acceleration is 0.
Solution: The acceleration has already been determined to equal 6(t − 4), and
is 0 precisely at time t = 4. We are thus being told that

0 = x(4) = 43 − 12(42 ) + 36(4) + C = 16 + C

so C = −16.
Notes for Lecture Section 002, MATH 140 2008 09 2261

5.21 Supplementary Notes for the Lecture of Wednesday, Novem-


ber 19th, 2008
Release Date: Wednesday, November 19th, 2008
subject to correction

5.21.1 §4.5 Summary of Curve Sketching


Read the textbook! The author’s preamble to the section, [1, pp. 307-308] provides an
excellent introduction to the issues, and to the limitations of both plotting isolated points
on a curve and using calculators and computers to produce graphs.

Limitations of the method of plotting points. If you attempt to graph a curve


only by plotting a number of points — even a large number — you expose yourself to
the risk that either

• your selection of points is too sparse at critical places where the graph changes
rapidly, and so you miss those important features; or

• your selection happens to show undue emphasis on features which are not repre-
sentative of the global behavior of the function.

It is always important to obtain some information about specific points on a curve. The
author warns that such information alone is often misleading. A good policy is that you
should never base any serious conclusions on the behavior of a function at specific points
unless you have some theoretical justification.116

Limitations of graphing calculators and computers. Graphing calculators and


computers are limited by the technical sophistication of the software. In principle the
software should be able to do anything you can do, and more: but, at the moment it
may not be able to do that.
Here again, the issue is that the global behavior of a function may not be best rendered
by selection of specific points in the domain, even if the points are close together.
116
Sometimes we do have such justification! For example, when we tabulate the factors whose product
is f 0 , and then indicate where each of the factors is positive, we are justified in concluding that the sign
of the derivative in an interval can be determined by selecting any point in the interval. Or, when we
use a corollary to the Mean Value Theorem to prove that a function is constant over an interval, we are
justified in determining that constant by choosing any point we like in the interval.
Notes for Lecture Section 002, MATH 140 2008 09 2262

Guidelines for Sketching a Curve The textbook recommends 8 steps to follow when
sketching, and emphasizes that “not every item is relevant to every function”. Try to
use this as a checklist when you solve problems from the chapter.

A. Domain: Remember that sometimes the domain is not the “natural domain” — the
largest set of values where the definition of the function makes sense; sometimes
you are explicitly instructed to consider the function over a restricted subset of
that “natural” domain. But sometimes — particularly in problems stated verbally
— the “instruction” to restrict the function is implicit in other information given
about the problem. Thus it may happen that there are real numbers where the
function is defined, but where the function in the question does not “make sense”.

B. Intercepts: Technically, this term refers to the distance between two designated
points on a line. In this context the line is usually one of the coordinate axes,
and the points in question are the origin and the point where the curve crosses the
axis; we interpret distance as directed distance, so the y-intercept of the graph of
f is usually taken to be the y-coordinate117 of the point where the curve crosses
the y-axis, i.e., the value f (0); and each x-intercept is the x-coordinate118 of a
point where the curve crosses the x-axis, i.e., where f (x) = 0. If you are given an
explicit formula for f , then it is often easy to determine f (0); if the function is
defined implicitly, you may be faced with solving an equation to determine f (0).
I have often used the word “intercept” to describe the corresponding coordinates of
points of intersection of the graph with lines parallel to the coordinate axes. This
kind of information is often helpful, particularly if the graph has symmetry about
lines parallel to but distinct from the coordinate axes.

C. Symmetry: The textbook reminds you of the concepts of evenness and oddness
discussed in [1, §1.1, p. 19], and in these notes. Where a function is known to be
even or odd it suffices to consider its behavior on only half of its domain. The full
graph of an even function can be obtained from the graph for non-negative positive
x by reflecting the graph in the y-axis; the full graph of an odd function can be
obtained from the graph for non-negative x by rotating that portion of the graph
through an angle of π radians around the origin.
The textbook also discusses the property of periodicity under which a function may
echo over its entire domain the behavior over one portion of the domain. If f has
period p, it has the property that

f (x) = f (x + p) for all real numbers x. (192)


117
the ordinate
118
the abscissa
Notes for Lecture Section 002, MATH 140 2008 09 2263

Examples are

• sin and cos, which have period 2π


• tan and cot which have period π
• x − [x] (or x − bxc), which have period 1.

The author defines period to be the shortest distance for repetition to take place.
Sometimes we are more casual with this term, and may use it to mean any distance
for which (192) holds.

D. Asymptotes: The textbook reminds you of the two types of asymptotes we have
studied:

Horizontal Asymptotes: These are lines y = L with the property that either
lim f (x) = L or lim f (x) = L. It can happen that a graph has 2 distinct
x→∞ x→−∞
horizontal asymptotes, just one asymptote to which it is asymptotic both as
x → ∞ and x → −∞, or no horizontal asymptotes at all. You should be able
to give examples of these situations.
Vertical Asymptotes: These are lines y = a with the property that one of the
following 4 situations holds:

lim = +∞ lim = +∞
x→a+ x→a−
lim = −∞ lim = −∞
x→a+ x→a−

Of course, if we know either that lim = +∞ or lim = −∞, we have more


x→a x→a
than enough information to claim that y = a is a vertical asymptote.

The textbook also discusses the possibility of “Slant Asymptotes”. This topic is
omitted from the syllabus of Math 140.

E. Intervals of Increase or Decrease: I have discussed methods for obtaining this


information and presenting it in tabular form in connection with [1, §4.3].

F. Local Maximum and Minimum Values: This topic has also been discussed in
connection with [1, §4.3]; remember that you have 2 tests available for classifying
critical points, and it may happen that one of these is much more useful than the
other in a particular application.

G. Concavity and Points of Inflection: This also has been discussed in connection
with [1, §4.3]. Remember that it is not enough to find points where f 00 (x) = 0.
We have seen instances where this happens, and yet the point is not an inflection
Notes for Lecture Section 002, MATH 140 2008 09 2264

0 y
-6 -4 -2 0 2 4
x

-2

-4

Figure 17: Portion of the graph of y = sin x − tan x, showing its vertical asymptotes

point119 ; and instances where there is an inflection point and yet there is no 2nd
derivative available; for example, the continuous function x · |x| changes concavity
at x = 0, but has no 2nd derivative there; so x = 0 is an inflection point.

H. Sketch the Curve: The textbook offers practical assistance in the use of the pre-
ceding information to plot the graph. The author suggests that, where you have
determined a point (x, f (x)) on the curve, you may sometimes wish to determine
the value of f 0 (x), in order to know the slope at which the curve passes through
the point.

Example 5.78 [7, Exercise 4.5.32, p. 323] To sketch the graph of f (x) = sin x − tan x.
Solution: (cf. Figure 17, page 2264 in these notes)
119
e.g., x4
Notes for Lecture Section 002, MATH 140 2008 09 2265

A. Domain: The sine function is defined for all real numbers; but the tangent function
is not defined at odd multiples of π2 ; hence the domain of f is R − {(2n + 1) π2 |n =
any integer}.

B. Intercepts: The y-intercept is f (0) = 0 − 0 = 0. The x-intercepts will be those x


for which sin x = tan x, equivalently, for which sin x(1 − sec x) = 0, equivalently,
for which either sin x = 0 or cos x = 1. sin x = 0 when x is an integer multiple
of π, while cos x = 1 when x is an integer multiple of 2π: thus the second factor
vanishes only when the first vanishes, and it suffices to look only at the first. To
summarize, the x-intercepts are the integer multiples of π.

C. Symmetry: This function is odd , since f (−x) = sin(−x) − tan(−x) = − sin x +


tan x = −f (x). It is also periodic, since the sine function has period 2π and the
tangent function has period π: f has period 2π. Thus it suffices to study the
function over any interval of length 2π — the local extrema, inflection points, etc,
will be repeated periodically over the whole domain.

D. Asymptotes: We know that

lim +
f (x) = 0 − (−∞) = ∞
x→−( π2 )

lim − f (x) = 0 − (∞) = −∞


x→( π2 )

lim + f (x) = 0 − (−∞) = ∞


x→( π2 )

lim −
f (x) = 0 − (∞) = −∞
x→( 3π
2 )

and the pattern will repeat over the whole domain. There are thus vertical asymp-
totes at all odd multiples of π2 ; in each case the asymptote occurs for two reasons,
because a single infinite one-sided limit is sufficient for a vertical asymptote.
A non-constant function with a positive period cannot have a horizontal asymptote,
since the maximum distance of the graph from any horizontal line will be oscillating
as we pass to ±∞.

E. Intervals of increase or decrease:


cos3 x − 1 (cos x − 1)(cos2 x + cos x + 1)
f 0 (x) = cos x − sec2 x = = .
cos2 x cos2 x
The numerator is a product of two functions. Of these, the second function is a
quadratic function which cannot be zero, since its discriminant is 12 − 4 < 0. Thus
Notes for Lecture Section 002, MATH 140 2008 09 2266

the only way in which f 0 (x) = 0 is when cos x = 1, which, as mentioned earlier,
occurs when x = 2nπ, where n is any integer. These are the only critical numbers,
and they are infinitely many. The odd multiples of π2 are, indeed, points where
the derivative is not defined; but, as they are not in the domain of the function,
they are not critical numbers. More generally, we need to determine where the
derivative is positive and where it is negative. It is never positive, since the factor
cos2 x + cos x + 1 always has the sign of the square term of the quadratic, here +;
and the denominator is a square, so it also is positive; the factor cos x − 1 cannot
be positive, but it can be 0 — when x is ¡ an even
¢ ¡ integer
¢ multiple
¡ ¢of π. Thus the
function is decreasing on the intervals − π2 , 0 , 0, + π2 , π2 , + 3π
2
, and then the
π
pattern repeats itself periodically: two intervals of length 2 , and one interval of
length π.

F. Local Maximum and Minimum Values: We found the critical numbers in the
preceding paragraph. As the function is decreasing on both sides of each of these
points, the function has no local extrema.

G. Concavity and Points of Inflection: Differentiating, we find that


¡ ¢
f 00 (x) = − sin x − 2 sec2 x · tan x = − sin x 1 + 2 sec3 x .

Here the factor 1 + 2 sec3 x cannot be 0, but will alternate between positive and
negative as x passes over the discontinuities at the odd multiples of π2 . The factor
sin x can be 0, and will change sign at the mid point of each of the intervals over
which f is defined. Thus the sign of f 00 will change at each of those midpoints, so
each of those mid points — the integer multiples of π — will be an inflection point
of the graph.

All of the foregoing could be suggesting that the function has branches that all look the
same. But, while the function is periodic with period 2π, there will be 2 branches in
each interval of length 2π, and they will not have exactly the same shape. The difference
is that the curve will be tangent to the x-axis at points 0, ±2π, ±4π, . . ., but not at the
intervening x-asymptotes. The tangency occurs at the points separating the two shorter
intervals of decrease.
sin x
Example 5.79 Consider the graph of the function f (x) = .
x
Notes for Lecture Section 002, MATH 140 2008 09 2267

0.8

0.6

0.4

0.2

0
5 10 15 20 25 30
x
-0.2

sin x
Figure 18: Graph of f (x) = for x > 0
x
Notes for Lecture Section 002, MATH 140 2008 09 2268

A. Domain: This function has as its domain R − {0}, since the fraction is not defined
when the denominator is 0. However, we know that lim x → 0f (x) = 1

B. Intercepts: There is no y-intercept, since 0 is not in the domain. The graph will
cross the x-axis whenever sin x = 0, i.e., precisely at the integer multiples of π —
infinitely often.

C. Symmetry: This function is even:

sin(−x) − sin x sin x


f (−x) = = = = f (x)
−x −x x
for all x. It is not periodic, since the maximum magnitude of f (x) gets smaller as
x grows.

D. Asymptotes: The curve has no vertical asymptotes, since there is no place where
a one-sided limit is infinite: even at x = 0 the limit is finite. As x → ∞,
1 sin x 1
− ≤ ≤
x x x
and the extremes both approach 0; hence, by the Squeeze Theorem, lim = 0, and
x→∞
y = 0 is a horizontal asymptote. The same limit holds as x → −∞, since the
function is even, and the graph to the left of x = 0 is the mirror image of the graph
to the right.

E. Intervals of Increase and Decrease: At this point the author’s proposed rubrics
for investigation become difficult for this function. We find that the derivative is
x cos x − sin x
f 0 (x) = ,
x2
so the tangent is horizontal when tan x = x, i.e., where the curve y = x crosses the
graph of the tangent function. It is beyond this course to be able to extract further
information about precisely where this happens, except to observe that there is
going to be one crossing in every interval of length π. So there are going to be
infinitely many critical points; they are located “near” the odd multiples of π2 .

When x is equal to an odd multiple of π2 , the graph touches one of the curves y = ± x1 ; it
is, in that sense, asymptotic to that pair of curves; more precisely, the distance between
y = f (x) and y = x1 approaches 0 as x → ±∞ and also the distance between the given
curve and y = − x1 . So the curve is oscillating between those two curves, all the while
approaching the y-axis. A sketch is given in Figure 18
Notes for Lecture Section 002, MATH 140 2008 09 2269

Slant Asymptotes Students in MATH 140 are not expected to be familiar with the
concept of slant asymptotes. More generally, we could wish to explore situations where
curves are asymptotic to other kinds of curves than lines; such topics will not be consid-
ered in MATH 140.

4.5 Exercises I will work several problems to indicate the way in which I would ap-
proach problems of this type. But there are so many essentially different kinds of func-
tions, you are advised to work many problems in this section. This is time-consuming,
but essential.

[7, Exercise r
4.5.22, p. 323] Use the guidelines of this section to sketch the graph of
x
f (x) = .
x−5
Solution: (cf. Figure 19, page 2270)
Notes for Lecture Section 002, MATH 140 2008 09 2270

4
y

0
-4 -2 0 2 4 6 8 10
x

r
x
Figure 19: Graph of f (x) = and its horizontal asymptote
x−5
Notes for Lecture Section 002, MATH 140 2008 09 2271

A. Domain: f is the composition of more than one function. We can interpret it


as resulting from the application of the square root function to the function
x
x 7→ . This last function is defined for all x except 5. But the square
x−5
root function is defined when its argument is non-negative; thus we have to
restrict x according to the following sequence of inequalities:
x
≥0
x−5

 x ≥ 0 and x − 5 > 0
⇔ or

x ≤ 0 and x − 5 < 0
⇔x>5 or x ≤ 0.
Thus the domain is made up of two non-intersecting infinite intervals: the
function is not defined for any x in the interval (0, 5].
B. Intercepts: The y-intercept is f (0) = 0. And the only solution to the equation
f (x) = 0 is the point x = 0. Thus both intercepts of f with the coordinate
axes are both 0.
C. Symmetry: We can check that f (−x) 6= f (x) (for any x in the domain), so
the function is not even; it is surely not odd, since an odd function would
have a positive value at some x paired with a negative value at −x, and the
present function is never odd. It is also not periodic, although we have not
discussed in this course the details of proving that.
D. Asymptotes: For valuess of x that are large in absolute value, it is convenient
r
1 5
to view the function as 5 or as 1+ . Either of these functions
1− x x−5

can be seen to approach 1 = 1 as x → ±∞. Thus y = 1 is the only
horizontal asymptote (and the graph is asymptotic to it both as x → ∞
and as x → −∞). Indeed, the function approaches the value 1 from below
as x → −∞, and approaches the value 1 from above as x → +∞. This
observation is made only to help in the sketching: a limit as x → ∞ or as
x → −∞ need not be approached “monotonically”: it is possible for a curve
to cross its horizontal asymptote, although that does not happen here.
p x
The graph is also asymptotic to the vertical line x = 5, since lim+ x−5 =
x→5
+∞.
E. Intervals of Increase and Decrease:
r
0 1 x−5 −5
f (x) = ·
2 x (x − 5)2
Notes for Lecture Section 002, MATH 140 2008 09 2272


 5

 − 1 3 if x>5
= 2x (x − 5) 2
2

 5

 − 1 3 if x<0
2(−x) 2 (5 − x) 2

which is always negative. Thus the function is decreasing over its entire do-
main, except for the end-point x = 0, where f 0 is not defined. Since the
domain consists of two separate disconnected intervals, this property of be-
ing decreasing applies separately to each of the two intervals (−∞, 0) and
(5, +∞).
F. Local Maximum and Minimum Values: There are no critical points; (we
follow the textbook’s terminology, under which an end-point is not a critical
point). By Fermat’s Theorem there is only one point where an extremum
could occur, and that is at x = 0, when f (0) = 0. This is evidently a
local minimum. Indeed, this is the global minimum of the function, since a
square root cannot be less than 0. There is no local maximum, and no global
maximum either.
G. Concavity and Points of Inflection: Since
5 1
f 0 (x) = − · p ,
2 x(x − 5)3
µ ¶
00 5 1 1 · (x − 5)3 + 3x(x − 5)2
f (x) = − · − · p ·,
2 2 ( x(x − 5)3 )3
5 (x − 5)2 (4x − 5)
= · ³p ´3
4
x(x − 5)3
r µ ¶
5 x−5 5 4x − 5 p
= (4x − 5) = x(x − 5) .
4 x3 4 x2

We could set up a chart, analyzing the various factors. We would find that
the second derivative is positive for x > 5 and negative for x < 0: the graph
is concave downward for x < 0, and upward for x > 5. There are no points
where the concavity changes within the domain, so no inflection points.
H. Sketch the Graph: This will be done at the lecture.

[7, Exercise 4.5.50, p. 323] To sketch the curve y = ex − 3e−x − 4x (cf. Figure 20,
page 2273).
Notes for Lecture Section 002, MATH 140 2008 09 2273

y 0
-2 -1 0 1 2 3
x
-2

-4

Figure 20: Graph of y = ex − 3e−x − 4x


Notes for Lecture Section 002, MATH 140 2008 09 2274

Solution:
A. Domain: The exponential function ex is defined for all real x; e−x also has
the whole of R as its domain; and 4x also has domain R: so the domain of
f (x) = ex − 3e−x − 4x is R.
B. Intercepts: The y-intercept is f (0) = 1 − 3 − 0 = −2. We will defer consider-
ation of the x-intercepts until we have more information about f .
C. Symmetry: f (−x) = e−x − 3ex + 4x. The equality f (x) = f (−x) would thus
x −x
be equivalent to the equation e −e
x
= 2, or to sinh x = x. While this equation
may have some solutions — x = 0 is one such value — it is not always true; if
it were true, then the derivatives of the two sides of the equation would also
be equal, and cosh x would be constant, 1, which it certainly is not; thus f
is not an even function. Similarly, it would follow from f (x) = −f (−x), that
ex is negative; from such a contradiction we know that f is not odd either.
After we have investigated the limits as x → ∞ we will be able to conclude
that the function is not periodic either.
D. Asymptotes: The graph has no vertical asymptotes, since, being continuous
everywhere, it has a finite limit at every point in its domain.
µ ¶
x 3 x
lim f (x) = lim e 1 − x − 4 x
x→∞ x→∞ e e
x 1
We can show, by L’Hospital’s Rule, that lim x = lim x = 0; evidently
x→∞ e x→∞ e
lim 3x = 0. Hence, by the Product Law,
x→∞ e
µ ¶
x 3 x
lim f (x) = lim e 1 − x − 4 x
x→∞ x→∞ e e
µ ¶
x 3 x
= lim e · lim 1 − x − 4 x
x→∞ x→∞ e e
µ µ ¶ ³ x ´¶
x 3
= lim e · lim lim 1 − lim − lim 4 x
x→∞ x→∞ x→∞ x→∞ ex x→∞ e
x
= lim e · (1 − 0 − 0)
x→∞
= lim ex · 1 = +∞
x→∞

In a similar way we can show that


³ x ´
lim f (x) = lim e−x e2x − 3 − 4 −x
x→−∞ x→−∞
³ e
x ´
= lim e−x · lim e2x − 3 − 4 −x
x→−∞ x→−∞ e
Notes for Lecture Section 002, MATH 140 2008 09 2275

µ ³ ¶
−x −2x x ´
= lim e · lim e − 3 − lim 4 −x
x→−∞ x→∞ x→−∞ e
−x
= lim e · (0 − 3 − 0) = −∞
x→−∞

Thus there are no horizontal asymptotes to the graph of f .


E. Intervals of increase or decrease:
e2x − 4ex + 3 (ex − 1)(ex − 3)
f (x) = ex + 3e−x − 4 = = .
ex ex
The critical points are x = ln 1 = 0 and x = ln 3. We tabulate the signs of
the derivative:
Interval ex − 1 ex − 3 ex f0 f
x<0 − − + + increasing
x=0 0 −2 1 0 critical point
0 < x < ln 3 + − + − decreasing
x = ln 3 + 0 + 0 critical point
ln 3 < x + + + + increasing

By the First Derivative Test, both critical points are local extrema: x = 0 is
a local maximum, and x = ln 3 is a local minimum.
We can now return to the question of the x-intercepts. There can be no zero
a of f such that a < 0, since then, applying the Mean Value Theorem to
f on the interval a ≤ x ≤ 0, we would find that f (0)−f 0−a
(a)
= f 0 (c) > 0, so
−2−0
0−a
> 0, which would imply that a > 0, contradicting the hypothesis. Since
f (0) = −2, and since f 0 < 0 for 0 < x < ln 3, the function values in this
interval will be negative. f (ln 3) = 2 − 4 ln 3 = 2 − ln 81 < 0. Finally, consider
f on the interval ln 3 < x: here the function is increasing; by a familiar use
of the Mean Value Theorem, the graph cannot cross the x-axis more than
once. Since f (3) = e3 − e33 − 12 > (2.7)3 − (2.7)3 3
3 − 12 > (2.7) − 12 > 0, we

know by the Intermediate Value Theorem that there is a point x between ln 3


and 3 where f (x) = 0 — hence it is a unique point, and this is the second
x-intercept.
G. Concavity and Points

of Inflection: Differentiating, we find that f 00 (x) =

(e + 3)(e − 3)
x x √
ex − 3e−x = . The factor ex
+ 3 is always positive, as is the
e x

factor e . The factor e − 3 is negative for x < ln23 and positive for x > ln23 .
x x

Thus, to the left, the graph is concave downwards; and, to the right of ln23 ,
the graph is concave downwards. Thus x = ln23 is an inflection point.
Notes for Lecture Section 002, MATH 140 2008 09 2276

5.21.2 §4.6 Graphing with Calculus and Calculators


Because we are asking student to avoid the use of calculators in MATH 140/141, this
section is not examination material. You are, however, urged to peruse it, since some
of the skills required in solving the problems is similar to what you will be expected to
know.

4.6 Exercises You might wish to try some of the problems to see how far you can get
without having to resort to the use of a calculator.

[1, Exercise 4.6.26, p. 321] : “Describe how the graph of f varies as (the parameter)
c varies...You should investigate how maximum and minimum points and inflection
points move when c changes. You should also identify any transitional values of c
at which the basic shape of the curve changes. f (x) = x3 + cx.”
Solution:

A. Domain: The domain of f is the entire real line.


B. Intercepts: The y-intercept is f (0) = 0. The x-intercepts are the solutions to
x3 + cx = 0: √
when c > 0 the only x-intercept is 0; when c ≤ 0 the x-intercepts
are 0 and ± −c.
C. Symmetry: f (−x) = (−x)3 + c(−x) = −f (x). Thus these functions are
always odd . They are never even, and never periodic.
D. Asymptotes: As lim = ±∞, there are no horizontal asymptotes. There are
x→±∞
also no vertical asymptotes, as the function is continuous on the whole real
line.
E. Intervals of Increase or Decrease: f 0 (x) = 3x2 + c. When c > 0, f 0 > 0
for all x, and the function is always increasing. When c = 0, the function
is increasing for x > 0, and decreasing
p c for x < 0. And, p when c < p0, the
function is increasing for x < − − 3 , decreasing for − − 3 < x < − 3c ,
c
p p
and increasing again for x > − 3c . The points x = ± − 3c are then critical
points.
F. Local Maximum and Minimum Values: For c > 0 there are no critical
points, and no local extrema. For c = 0 there is one critical point, x = 0;
but this point is not a local extremum since f < 0 to its left and f > 0 to
its right; so here
p also there are no local extrema. But, for c < 0, thep
critical
point x = − − 3 is a local maximum, and the critical point x = − 3c is
c

a local minimum. These properties may be shown by either the First or the
Second Derivative Tests.
Notes for Lecture Section 002, MATH 140 2008 09 2277

G. Concavity and Points of Inflection: f 00 (x) = 6x for all c: The graph is


always concave upward for x > 0, and concave downward for x < 0. Thus
x = 0 is always an inflection point!
H. Sketch the Curve: This may be done at the lecture.

5.21.3 Sketch of Solutions to Problems on one of several versions of the


December, 2004 Final Examination
Students were advised that there were two kinds of problems on this examination, each
clearly marked as to its type:
• “Some of the questions on this paper require that you SHOW ALL YOUR WORK!
Their solutions are to be written in the space provided on the page where the
question is printed. When that space is exhausted, you may write on the facing
page. Any solution may be continued on the last pages, or the back cover of the
booklet, but you must indicate any continuation clearly on the page where the
question is printed!

• “Some of the questions on this paper require only BRIEF SOLUTIONS ; for these
you are expected to write the correct answer in the box provided; you are not asked
to show your work, and you should not expect partial marks for solutions that are
not completely correct.
“You are expected to simplify your answers wherever possible.”
1. BRIEF SOLUTIONS
[3 MARKS EACH] Give the numeric value of each of the following limits if it exists;
if the limit is +∞ or −∞, write +∞ or −∞ respectively. In all other cases write
”NO FINITE OR INFINITE LIMIT”.
|y|
(a) lim =
y→−∞ y
ANSWER ONLY

Solution: For negative y, |y| = −y. It follows that


|y| −y
lim = lim = lim (−1) = −1 ,
y→−∞ y y→−∞ y y→−∞

the limit of a constant function.


Notes for Lecture Section 002, MATH 140 2008 09 2278

sin u
(b) lim =
u→−∞ u

ANSWER ONLY

Solution: Don’t confuse this question with the theorem that lim sinu u = 1. As
u→0
u → ∞ the numerator here oscillates between −1 and +1, but the denom-
inator is approaching −∞, so the quotient approaches 0. Here is a formal
proof:
Beginning with the inequalities

−1 ≤ sin u ≤ 1

we divide all members by u < 0, reversing the inequalities:


1 sin u 1
− ≥ ≥
u u u
and allow u → ∞. The extremes approach 0; hence, by the Squeeze Theorem,
sin u
lim = 0.
u→−∞ u
µ ¶2n
1
(c) lim 1 + =
n→∞ n
ANSWER ONLY

µ ¶n
1
Students should know the result that lim 1 + = e; the present function
n→∞ n
is the square of that function, so the limit is e2 .
µ ¶
1 1
(d) lim − =
x→0 x sin x
ANSWER ONLY

Solution: This was [7, Exercise 45, p. 314]. It was a bad choice for a question
which requires no solutions, since we saw that some students solved it using
Notes for Lecture Section 002, MATH 140 2008 09 2279

fallacious logic, and we still gave them the marks because I did not wish to
penalize them for proofs that were not required, even if they were very, very
wrong.
What was wrong was that some students reasoned that the two subtracted
terms both approached +∞, so the difference was 0. This is nonsense, but it
does happen to yield the correct answer in this case. Here is a correct solution:
µ ¶ µ ¶
1 1 sin x − x
lim − = lim
x→0 x sin x x→0 x sin x

where numerator and denominator both approach 0. We apply l’Hospital’s


Rule:
µ ¶
sin x − x cos x − 1
lim = lim
x→0 x sin x x→0 sin x + x cos x

sin x
= lim
x→0 cos x + cos x − x sin x

after a second application of l’Hospital’s Rule. In this last fraction the de-
nominator approaches 2, but the numerator approaches 0. We may use the
0
Quotient Law, and conclude that the limit of the original function is = 0.
2
¡ 2 2
¢
(e) lim ln(3t ) − ln(t + 7) =
t→∞

ANSWER ONLY

This problem resembles [7, Exercise 63, p. 314].


¡ ¢ 3t2
lim ln(3t2 ) − ln(t2 + 7) = lim ln 2
t→∞ t→∞ t +7
3
= lim ln
t→∞ 1 + t72
µ ¶
3
= ln lim = ln 3 .
t→∞ 1 + 72
t

Note that, contrary to the inference you might make from the location of the
problem in the textbook, you do not need to use l’Hospital’s Rule here.
√ √
x + 8 − 2x
(f) lim =
x→8 x2 − 8x
Notes for Lecture Section 002, MATH 140 2008 09 2280

ANSWER ONLY

Solution:
Ã√ √ ! √ √ √ √
x + 8 − 2x x + 8 − 2x x + 8 + 2x
lim = lim ·√ √
x→8 x2 − 8x x→8 x2 − 8x x + 8 + 2x
µ ¶
(x + 8) − 2x
= lim √ √
x→8 (x2 − 8x)( x + 8 + 2x)
−1
= lim √ √ .
x→8 x( x + 8 + 2x)
The denominator approaches 8 × (4 + 4) = 64, while the numerator → −1, so
1
the function approaches − 64 as x → 8.

2. BRIEF SOLUTIONS
[3 MARKS EACH] Determine each of the following derivatives, and simplify your
answers as much as possible.
µ ¶
d x2 + 3x
(a) =
dx x

ANSWER ONLY

µ ¶
d x2 + 3x d
Solution: = (x + 3) = 1 + 0 = 1.
dx x dx
d ¡ −3t ¢
(b) t =
dt

ANSWER ONLY

Solution:
d ¡ −3t ¢ d ¡ ln t ¢−3t
t = e
dt dt
Notes for Lecture Section 002, MATH 140 2008 09 2281

d −3t ln t
= e
dt
d
= e−3t ln t · (−3t ln t)
dt µ ¶
−3t ln t 1
= e · (−3) · (t) + ln t · 1
t
= e−3t ln t · (−3) · (1 + ln t)
= t−3t · (−3) · (1 + ln t) .

d ¡ ¢
(c) tan(e2s ) − e2 tan s =
ds

ANSWER ONLY

Solution:
d ¡ ¢
tan(e2s ) − e2 tan s = sec2 (e2s ) · e2s · 2 − e2 tan s · 2 sec2 s .
ds
d ¡ ¢
(d) cosh2 (3y) =
dy

ANSWER ONLY

Solution:
d ¡ ¢
cosh2 (3y) = 2 cosh(3y) · sinh(3y) · 3 .
dy
(This answer could be further simplified, as follows, but I didn’t expect stu-
dents to know the identity required for hyperbolic functions:

6 cosh(3y) · sinh(3y) = 3 sinh(6y) .)

d ¡ 2 ¢
(e) cos X − cos(X 2 ) =
dX
Notes for Lecture Section 002, MATH 140 2008 09 2282

ANSWER ONLY

Solution:
d ¡ 2 ¢ ¡ ¢
cos X − cos(X 2 ) = 2 cos X · (− sin X) + sin X 2 · 2X
dX
3. BRIEF SOLUTIONS

(a) [5 MARKS] Find an equation for a line through the point (−2, 0) which is
tangent to the curve y = x2 and is not horizontal.

ANSWER ONLY

Solution: On the curve y = x2 , the slope of the tangent at the point (x0 , x20 )
is 2x0 . An equation for the tangent line is, thus,
y − x20 = 2x0 (x − x0 )
or y = 2x0 x−x20 . For this tangent line to pass through (−2, 0), the coordinates
of that point must satisfy the equation of the tangent, i.e., 0 = 2x0 (−2) − x20 ,
implying that x0 = 0 or x0 = −4. The first of these alternatives gives a
horizontal tangent; hence x0 = −4, and the tangent line sought is
8x + y + 16 = 0 .
(Some students grabbed the only constant they could find in the problem —
−2 — and simply found the equation of the tangent line at the point on the
curve with x = −2. This tangent line does not pass through the point (−2, 0),
and is not the line that the question is looking for.)
(b) [5 MARKS] Determine values of the constants a and b that will make the
following function continuous at x = −6.
 √
 x + 31 if x < −6
f (x) = a+b if x = −6

a(x + 5) if x > −6
Notes for Lecture Section 002, MATH 140 2008 09 2283

ANSWER ONLY

Solution: This function is pieced together from a root function, a constant,


and a linear function. The functions
√ a + b (a constant), and a(x + 5) are
continuous everywhere, and x + 31 is continuous on (−31, ∞); but there
could be discontinuities where the piecing together occurs, i.e., at x = −6.
The function is defined at that point. Thus all we need to check is that the
limit should exist at x = −6, i.e., that the limits from left and right separately
should exist and they should be equal; and that the common value of these
two one-sided limits should be the function value.

The limit from the left will be lim−1 x + 31 = 5; the limit from the right will
x→−6
be lim + a(x + 5) = −a. Thus the equality a = −5 is required for continuity,
x→−6
and will ensure that the 2-sided limit of the function exists at x = −6. We
must now impose the condition that this limit, 5, be equal to the function
value, a + b, i.e., to −5 + b. It follows that b = 10.
(c) [5 MARKS] Determine values of the constants k and ` that will make the
following function differentiable at x = 1.
½
kx2 + ` if x ≤ 1
g(x) =
6x − 4 if x > 1

ANSWER ONLY

Solution: The functions being pieced together here are polynomials, which
we know to be differentiable everywhere. As in the preceding problem, the
only point where there could be a question is where they are pieced together,
at x = 1. At that point we know that differentiability implies continuity.
We will first make the function continuous there. The limit from the left is
lim−1 (kx2 + `) = k + `. The limit from the right is lim+ = 6 − 4 = 2; thus we
x→1 x→1
have
k + ` = 2.
The difference quotient from the right is (6x−4)−(k+`)
x−1
= 6x−6
x−1
= 6 . Thus, if the
function is differentiable at x = 1, the value of the derivative will be 6. We
Notes for Lecture Section 002, MATH 140 2008 09 2284

now look at the difference quotient from the left:


kx2 + ` − (k + `)
= k(x + 1) .
x−1
As x → 1, this quotient approaches 2k, which must be equal to 6; hence k = 3
and ` = −1.

4. SHOW ALL YOUR WORK!

(a) [6 MARKS] Coffee is draining from a conical filter of depth 10 cm and diameter
10 cm (at the top) into a cylindrical coffee pot of diameter 12 cm, at the rate
of 100 cm3 /min. Determine how fast, in cm/min, the level of coffee in the pot
is rising when the coffee in the filter is 3 cm deep?
Solution: This was the easier part of a 2-part problem that had constituted a
written assignment for the students. This part of the problem did not depend
on the shape of the filter. If the coffee is h cm deep in the cylinder, the volume
V is π62 h, so the rate of change of volume with time t is
dV dh
100 = = 36π .
dt dt
100
The rate of increase of volume is, therefore the constant, .
36π
(b) [6 MARKS] You are given that y = y(t) is a function of t satisfying t3 y +ty 3 =
2. Assuming that y(1) = 1, determine the values of y 0 (1) and y 00 (1).
Solution: Differentiating the equation for y implicitly with respect to t yields
dy dy
3t2 y + t3 · + y 3 + t · 3y 2 · =0
dt dt
or
dy
t(t2 + 3y 2 )
+ y(3t2 + y 2 ) = 0 .
dt
Setting t = 1 in the original equation yields y 3 + y − 2 = 0; y 3 + y − 2 factorizes
into (y − 1)(y 2 + 2y + 2) = 0, of which y = 1 is the only solution. Substituting
dy
t = 1, y = 1 in the equation for the derivative yields = −1. Finally we
dt
must differentiate, for example, the equation
y(3t2 + y 2 )
y0 = − .
t(t2 + 3y 2 )
We obtain
00 y 0 (3t2 + y 2 )(6ty + 2y 2 y 0 )(t3 + 3ty 2 ) − y(3t2 + y 2 )(3t2 + 3y 2 + 6tyy 0 )
y =−
t2 (t2 + 3y 2 )2
0
which vanishes when (t, y, y ) = (1, 1, −1).
Notes for Lecture Section 002, MATH 140 2008 09 2285

5. SHOW ALL YOUR WORK!

(a) [4 MARKS] Prove that the function x3 + 9x2 + 33x assumes the value −8 at
least once.
Solution: The function takes on the value 0 > −8 when x = 0. It takes on the
value −1+9−33 = −25 < −8 at x = −1. Since the function is continuous, we
may apply the Intermediate Value Theorem to infer that the function takes
on the value −8 somewhere in the interval −1 < x < 0.
(b) [8 MARKS] Using the Mean Value Theorem or Rolle’s Theorem — no other
methods will be accepted — prove carefully that x3 + 9x2 + 33x takes on the
value −8 at most once.
Solution: Suppose the function took on the value −8 twice. Since the func-
tion is differentiable everywhere, it satisfies the conditions of the Mean Value
Theorem. There would have to be a point x = c in the interval −1 < x < 0
at which f 0 (c) = 0. But this is contradicted by

f 0 (x) = 3(x2 + 6x + 11) = 3(x + 3)2 + 6 ≥ 6 > 0

for all x. We conclude that our assumption of the existence of 2 roots is false
— the function attains the value −8 only once.

6. SHOW ALL YOUR WORK!


[10 MARKS] Using the calculus carefully, determine how to express 8 as the sum
of 2 nonnegative real numbers such that the sum of the square of the first and the
cube of the second is as small as possible.
Solution: Denote the second number by b, so the first will be 8 − b. Then we wish
to minimize the sum f (b) = (8 − b)2 + b3 . We find that

f 0 (b) = 2(8 − b)(−1) + 3b2 = (3b + 8)(b − 2)

so the only critical point in the domain 0 ≤ b ≤ 8 is b = 2, at which f 00 (b) = 6b + 2


evaluates to 14 > 0, so b = 2 is a local minimum, where the function value is
f (b) = 62 + 23 = 44. At the end points of the domain we have f (0) = 0 + 512 > 44
and f (8) = 64 + 0 > 44. Hence the sum is minimized globally when the second
summand is 2, and the first is 8 − 2 = 6.

7. SHOW ALL YOUR WORK!


µ ¶2
x
Let f (x) = .
x+3
Notes for Lecture Section 002, MATH 140 2008 09 2286

(a) [1 MARKS] State the domain of f .


Solution: The fraction is meaningful only where the denominator is non-zero.
Thus the domain of the fraction, and hence of its square f , is R − {−3}.
(b) [4 MARKS] Find the intervals of increase and the intervals of decrease of f .
Solution: µ ¶ µ ¶
0 x d 3 6x
f (x) = · 1− =
x+3 dt x−3 (x + 3)3
which vanishes only at x = 0. Because the degree of the factor x − 0 is 1,
which is odd, the function changes sign at 0. It is decreasing for −3 < x < 0
and increasing for 0 < x. In the interval x < −3, the degree of the negative
factor x + 3 is odd, and the degree of the negative factor x is also odd, so the
derivative is positive for x < −3.
(c) [4 MARKS] Determine the intervals of concavity upwards and the intervals of
concavity downwards, and the inflection points, if any.
Solution: Another differentiation yields

x − 23
f 00 (x) = −12 .
(x + 3)4

The odd factor x − 23 changes sign at x = 23 , so x = 32 is an inflection


¡ point:
¢
the graph is concave upward in both the intervals (−∞, −3) and −3, − 23 .
For x > 32 the graph is concave downwards.
(d) [3 MARKS] Sketch the graph of y = f (x), showing — clearly labelled — all
horizontal and all vertical asymptotes.
Solution: There is a vertical asymptote at x = −3, because the function
approaches ∞ from both sides of x = −3. As x → ±∞, f (x) → 1, so y = 1
is a horizontal asymptote: the graph is above the horizontal asymptote for
x, −3, and below it in the first quadrant. It crosses that asymptote from above
to below at x = − 23 .

8. SHOW ALL YOUR WORK!


[6 MARKS] Consider the function f (x) = 5x + 9 near x = −1. Is it, or is it not
true that f is continuous at x = −1? If the statement is true, prove it carefully,
using the ²-δ definition. If it is false, prove that carefully.
Solution: This topic has been deleted from this year’s syllabus, and I will not
provide a solution here.
Notes for Lecture Section 002, MATH 140 2008 09 2287

5.22 Supplementary Notes for the Lecture of November 24th,


2008
Release Date: Monday, November 24th, 2008
subject to correction

5.22.1 §4.7 Optimization Problems


Here is a variation of the steps the textbook suggests for solving optimization problems.

Spend some time understanding the problem: What quantity is to be optimized? What
is the “independent” variable? In Calculus 1 we can consider only problems with a
single independent variable; if your formulation of the problem appears to involve
more than one variable, there must be a relationship between those variables, and
you will have to determine that before you can complete your solution. (In Calculus
3 you will consider functions of several independent variables.)

The textbook suggests that you sketch the geometry, if the problem has been stated
in geometric language. But remember that your sketch is not a formal part of a
mathematical proof: it will be very helpful to you, but you must not attempt to
base any of your reasoning on the sketch.

Introduce notation for the independent variable(s) and the function (=“dependent vari-
able”), and express the dependencies that hold between them. You may subse-
quently find that your formulation of the problem is not an easy one to solve, and
you may wish to return to this step to express the dependent quantity in a different
way. Don’t be stubborn to continue with a formulation that proves to be difficult
for you.

In your formulation be careful to describe precisely the domain of the function that you
are optimizing. Remember that the domain may not be the entire natural domain
of the function, as there may be constraints in the problem that restrict certain
quantities.

It is usually the global extrema that are of interest in optimization problems. This
means that you will have to consider the end points of closed intervals. You may
also have to consider functions whose domains are not closed intervals.

The First Derivative Test for Global (=Absolute) Extreme Values. In the
discussion on page 2288 of the solution of [1, Exercise 4.7.12, p. 328] I shall draw a
conclusion from the facts that there is just one critical point, and that the derivative is
always of one sign on either side of the point, and that those signs are different. We are
Notes for Lecture Section 002, MATH 140 2008 09 2288

able to infer that the critical point is a global extremum. Hitherto most of the problems
in which we considered global extrema were defined on a finite, closed interval. We had
used this type of reasoning previously in connection with the First Derivative Test, which
is a test to a local extremum. The textbook calls this reasoning the First Derivative Test
for Absolute Extreme Values [1, p. 324].

4.7 Exercises

[1, Exercise 4.7.12, p. 328] “A box with a square base and open top must have a
volume of 32,000 cm3 . Find the dimensions of the box that minimize the amount
of material used.”
Solution: The box has three spatial dimensions, but two are the same. Is this a
problem with 2 independent variables? No. We can call the side of the base x,
and the height h; but these two variables are related, since x2 h = 32000. We can
use this last equation to express either of x and h in terms of the other. Let’s wait
before we decide which of them to take as our independent variable.
The area of the base is x2 , and the area of the sides is 4x · h, so the quantity we
wish to minimize is x2 + 4xh. If we were to take h as the independent variable,
this area expression would involve fractional powers of h, which would be slightly
unpleasant to work with; so I decide to express the area in terms of x, and to
eliminate h, even though the alternative approach could have been used, with a
bit more difficulty. Denote the area by
32000 128000
A(x) = x2 + 4xh = x2 + 4x · = x 2
+ .
x2 x
The domain of the function is 0 < x < ∞. We determine the derivatives of A(x):
128000
A0 (x) = 2x −
x2
256000
A00 (x) = 2 +
x3
The only critical point of A is at x = 40. Since A00 (40) > 0, this is a local
minimum. Alternatively, we could have used the First Derivative Test. Indeed,
the First Derivative Test is better since the fact that A0 is decreasing for x < 40
and increasing for x > 40 shows that this local minimum is, in fact, a global
minimum (cf. [1, First Derivative Test for Global (Absolute) Extreme Values, p.
324]). Finally we have to answer the question as stated: the dimensions of the box
using the smallest amount of material are
Notes for Lecture Section 002, MATH 140 2008 09 2289

side of base = 40 cm.


32000
height = cm. = 20 cm.
1600

(Note that the same method could be used for variants of this problem. For ex-
ample, the box could have a top, or the cost per unit area of the base could be
different from that cost for the sides, as in [1, Exercise 4.7.14, p. 328].)

[1, Exercise 4.7.14, p. 328] A rectangular storage container with an open top is to
have a volume of 10 m3 . The length of its base is twice the width. Material for the
base costs $10 per square meter. Material for the sides costs $6 per square meter.
Find the cost of materials for the cheapest such container.
Solution: We are given the relative lengths of the sides of the base of the box. If
the shorter of these sides has length x, the longer will have length 2x. If the height
of the box is denoted by h, then the constraint on the volume is that x · 2x · h = 10,
5
so the box has height h = 2 . We thus have all dimensions of the box expressed in
x
terms of one unknown: we would be unable to solve the problem with the methods
of MATH 140 if we weren’t able to express it as a single-variable problem. The
dollar cost of the material for the base will be x · (2x)µ· 10¶= 20x2 , while the dollar
5 180
cost of material for the sides will be (x+2x+x+2x)· 2 ·6 = . It is the sum
x x
180
of these costs that we must minimize, a function I will denote by f (x) = 20x2 + ;
x
the domain is x > 0, since we have given no meaning to negative length.
Differentiation yields
¡ ¢
0 180 40 x3 − 92
f = 40x − 2 = .
x x2
To determine the local extrema we will need to factorize the numerator; to apply
the First Derivative Test for Global Extreme Values, we will need to determine
the sign of this numerator. Since the numerator is a difference of cubes, we can
factorize as follows:
³ q ´µ q ³q ´2 ¶
3 9
x− 2 x + x 2 + 3 92
2 3 9

f 0 (x) = .
x2
r
3 9
The first factor changes sign at the critical point x = . The quadratic factor
2
in the numerator does not factorize further, and cannot change sign; it always has
Notes for Lecture Section 002, MATH 140 2008 09 2290

the sign of the leading term, i.e., of x2 , i.e., the sign +. This can also be seen by
completing the square:
r Ãr !2 Ã r !2 Ã r !2
9 9 1 9 3 3 9
x2 + x
3 3 3
+ = x+ + ,
2 2 2 2 4 2
which, being a sum of two squares, cannot be negative; moreover, since the second
of these square is not zero, the sum is strictly positive. As the denominator
r of this
3 9
fraction is a square, the sign of the f 0 (x) is positive when x < , and negative
r r 2
3 9 3 9
when x > : f 0 changes from negative to positive at x = , and, as this is
2 2
the only critical point, it is the global minimum point. The problem asks for the
value of the minimum attained at this point. It is
Ãr ! r r r µ ¶ r
3 9 3 81 3 2 3 2 9 3 2
f = 20 + 180 = 20 · + 180 = 270 .
2 4 9 9 2 9

[1, Exercise 4.7.24, p. 329] “Find the dimensions of the rectangle of largest area that
has its base on the x-axis and its other 2 vertices above the x-axis, and lying on
the parabola y = 8 − x2 .”
Solution: In this problem the author has prescribed a coordinate system, so you
don’t have to decide where to place it. He has not named the points, so you may
wish to do so; while this naming could be shown on your figure, a formal proof
should have the points named in the wording of the proof. For example, I will
assert that the vertices of the base are A(a, 0) and B(b, 0), and, without limiting
the generality, I will decide to let A be the leftmost of the two, so that a < b.
The vertices above them will then be C(a, 8 − a2 ) and D(b, 8 − b2 ). As this is to
be a rectangle, the side CD must be parallel to AB, which is on the x-axis; thus
8 − a2 = 8 − b2 , so b = ±a; since B is distinct from A, I will have to have b = −a.
Since I decided that B should be to the right of A, I will now specify that a < 0:
the vertices of the rectangle are now A(a, 0), B(−a, 0), C(a, 8 − a2 ), D(−a, 8 − a2 );
remember, a < 0. The base of this rectangle has length |2a| and the height is
8 − a2 , so the area — the function we wish to maximize — is
f (a) = (8 − a2 )|2a| .
And what is the domain
√ of f ? We
√ are told that C and D must lie above the x-axis,
2
so 8 − a ≥ 0 and√ − 8 ≤ a ≤ 8. Earlier we restricted a to be non-positive; thus
the domain is − 8 ≤ a ≤ 0. We can simplify the formula we gave for f (a):
f (a) = 2a3 − 16a .
Notes for Lecture Section 002, MATH 140 2008 09 2291


Hence f 0 (a)
q = 6a2
−16, for − 8 < a < 0. The critical points of this function would
8
be a = ± if a could take any value. But our restricting a to be negative leaves
3
q
only one of these points in the domain: a = − 83 . To determine the extrema of
f we evaluate the function at the critical point and at the end point of the closed
q
interval that is the domain, and find that f = 0 at the end points, and f = 643
2
3
at the critical
q point. We conclude that the dimensions of the largest rectangle are:
2 16
width is 4 3
, and height is 3
.
In this solution I did not follow one of the pieces of advice I gave you: I persisted
with an awkward formulation of the problem (with my variable a restricted to be
negative) rather than going back to the drawing board and reformulating it in a
more intuitive way. While I did obtain the answer, it was not the simplest way of
attacking the problem.

[1, Exercise 4.7.66, p. 332] “A steel pipe is being carried down a hallway 9 feet wide.
At the end of the hall there is a right-angled turn into a narrower hallway 6 feet
wide. What is the length of the longest pipe that can be carried horizontally around
the corner?”
Solution: The textbook provides a figure. The figure shows an angle named θ
between the pipe and the outer wall of the 9-foot hallway, where it is touching the
inner corner of the hallway and tight against the two outer walls. In this way the
author has helped you by eliminating the doubts as to which independent variable
to use.
Let’s call the maximum length of the pipe f (θ); the permitted values for θ are
0 < θ < π2 : note that θ cannot equal either 0 or π2 . Call the length of pipe that can
fit tightly at angle θ f (θ); expressing it in terms of the widths of the two hallways
we find it is a sum,
f (θ) = 9 csc θ + 6 sec θ .
Its derivative, after simplification, is
6 sin3 θ − 9 cos3 θ (cos θ) · (6 tan3 θ − 9)
f 0 (θ) = = .
sin2 θ · cos2 θ sin3 θ
We need to determine the global minimum for f , in order to determine the longest
pipe that can turn the corner horizontally.
q The function has just one q critical
point in the interval, when tan θ = 3 96 , corresponding to θ = arctan 3 32 . For
q q q
3 9 3 9
tan θ < 6
0
, f < 0; and, for tan θ > 6
, f > 0: thus θ = arctan 3 32 is a
0

local minimum of f . Since these inequalities hold throughout the domain, we can
Notes for Lecture Section 002, MATH 140 2008 09 2292

conclude that the point is, in fact, a global minimum (cf. “First Derivative
q Test for
3
Global (Absolute) Extreme Values” [1, p. 324]). When tan θ = 3
2
,

f (θ) = 9 csc θ + 6 sec θ


p
= (6 + 9 cot θ) 1 + tan2 θ
à r !s µ ¶ 23
3 2 3
= 6+9 1+
3 2

and this is the longest length of pipe that can turn the corner.

Example 5.80 Consider the following problem from the Review Exercises: [1, Review
Exercise 52 p. 349] “Find the point on the hyperbola xy = 8 that is closest to the point
(3, 0).” µ ¶
8
Solution: As independent variable take the abscissa of a point x, on the hyper-
x
bola; thus the domain will be the positive and negative coordinate axes, with the origin
removed. As a first attack on the problem we could try to minimize the function
s µ ¶2
8
f (x) = (x − 3)2 + −0 ;
x

this would work, but would produce a computationally difficult problem. Instead I will
minimize the square of the distance:
µ ¶2
2 2 8
g(x) = f (x) = (x − 3) + −0 , (193)
x

since the square and square root functions are both increasing functions, so the minimum
distance occurs precisely where the minimum square distance occurs.
We compute the derivatives, and find that

g 0 (x) = 2(x − 3) − 128x−3

g 00 (x) = 2 + 384x−4 > 0


which tells us, by the Second Derivative Test, that any local extremum is a local mini-
mum. Examining the first derivative more carefully, we find that

2(x4 − 3x3 − 64) 2(x − 4)(x3 + x2 + 4x + 16)


g 0 (x) = = . (194)
x3 x3
Notes for Lecture Section 002, MATH 140 2008 09 2293

Let’s examine the domain of g more carefully. The graph has two branches — one in
the first quadrant, and one in the third quadrant. The distance of (3, 0) from the 3rd
quadrant is surely at least 3, while its distance to the branch of the graph in the first
8
quadrant is no more than the vertical distance, . Thus the closest point to the given
3
point will be on the branch in the first quadrant, and we need not even consider the other
branch. This observation has been made be examining the geometry. Another argument
would simply examine the sum in (193), and observe that the second summand is a
square, so it is positive; if x < 0, the first sum cannot be less than 32 . We see from
factorization (194) that one critical number is x = 4. There cannot be any more critical
numbers on the positive axis (0, ∞), since the polynomial factor x3 + x2 + 4x + 16 cannot
be zero for positive x. But there could be critical numbers on the negative coordinate
axis; in fact, there is one at approximately x = −1.344389236. By arguing that the
closest point we seek could not be on the negative branch of the graph, we avoid having
to deal with this second critical point.
As for the critical number x = 4, the derivative is negative to the left of it, and positive
to the right of it, so, by the First Derivative Test, x = 4 is a local minimum. Indeed,
x = 4 is a global minimum [1, First Derivative Test for Global (Absolute) Extreme
Values, p. 324]. The y-coordinate is 84 = 2, so the closest point is (4, 2).
The point that we have ignored on the negative branch of the graph is of no interest
in this problem as stated; but a small change in the problem would make it relevant. We
could simply ask for the closest point on the negative branch of the graph to the point
(3, 0); or, equivalently, for the closest point on the given curve to the point (−3, 0).

5.22.2 §4.8 Newton’s Method – OMIT


This section is not part of the syllabus of MATH 140.

5.22.3 Sketch of Solutions to Problems on One Version of the December,


2005 Final Examination
Students were advised that there were two kinds of problems on this examination, each
clearly marked as to its type:

• “Some of the questions on this paper require that you SHOW ALL YOUR WORK!
Their solutions are to be written in the space provided on the page where the
question is printed. When that space is exhausted, you may write on the facing
page. Any solution may be continued on the last pages, or the back cover of the
booklet, but you must indicate any continuation clearly on the page where the
question is printed!
Notes for Lecture Section 002, MATH 140 2008 09 2294

• “Some of the questions on this paper require only BRIEF SOLUTIONS ; for these
you are expected to write the correct answer in the box provided; you are not asked
to show your work, and you should not expect partial marks for solutions that are
not completely correct.
“You are expected to simplify your answers wherever possible.”
1. BRIEF SOLUTIONS
[2 MARKS EACH] Give the numeric value of each of the following limits if it exists;
if the limit is +∞ or −∞, write +∞ or −∞ respectively. In all other cases write
“NO FINITE OR INFINITE LIMIT”.
³p ´
(a) lim 2
y +y+y =
y→−∞

ANSWER ONLY

Solution:
à !
³p ´ ³p ´ py 2 + y − y
lim y2 + y + y = lim y2 + y + y · p
y→−∞ y→−∞ y2 + y − y
à !
(y 2 + y) − y 2
= lim p
y→−∞ y2 + y − y
 
 (y 2 + y) − y 2 
= lim  r ³ ´

y→−∞  
y 2 1 + y1 − y
 
2 2
(y + y) − y 
= lim  p q
y→−∞
y 2 1 + y1 − y
 
2 2
(y + y) − y 
= lim  q
y→−∞
|y| 1 + y1 − y
(the square root is always non-negative)
 
y
= lim  q 
y→−∞
−y 1 + y1 − y
Notes for Lecture Section 002, MATH 140 2008 09 2295

as y < 0
 
1
= lim  q 
y→−∞ 1
− 1+ y
−1
1 1
= =− .
−1 − 1 2
¡ ¢
(b) lim e−x sinh x =
x→∞

ANSWER ONLY

Solution:
¡ ¢ ex − e−x
lim e−x sinh x = lim e−x ·
x→∞ x→∞ 2
−2x
1−e 1−0 1
= lim = = .
x→∞ 2 2 2
√ √
x + 8 + 2x
(c) lim =
x→8 x2 + 8x

ANSWER ONLY

Solution:
√ √ √ √
x + 8 + 2x 8+8+ 2·8
lim =
x→8 x2 + 8x 82 + 8 · 8
4+4 1
= = .
64 + 64 16
µ ¶
1 1
(d) lim + =
x→1 1 − x ln x

ANSWER ONLY
Notes for Lecture Section 002, MATH 140 2008 09 2296

Solution: We cannot use the Difference Law because neither summand has a
limit as x → ∞. Accordingly we transform the function into one that may be
more amenable; we begin by taking the fractions to a common denominator.
µ ¶
1 1 ln x − (x − 1)
lim + = lim .
x→1 1 − x ln x x→1 (1 − x) ln x
Next we apply l’Hospital’s Rule, since numerator and denominator both ap-
proach 0:
1
ln x − (x − 1) x
−1
lim = lim
x→1 (1 − x) ln x x→1 − ln x + 1−x
x
1−x
= lim
x→1 −x ln x + 1 − x
−1
= lim
x→1 −1 ln x − x − 1
x
by a second application of the 0/0 form of L’Hospital’s Rule .
−1 1
= lim = .
x→1 −1 · 0 − 1 − 1 2
sin ex
(e) lim =
x→∞ cos ex

ANSWER ONLY

Solution: The quotient is equal to tan ex . As x → ∞, ex → ∞. The tangent


has no limit as its argument becomes large, but takes on all real values as ex
passes through values in an interval of length π.
2. BRIEF SOLUTIONS
[2 MARKS EACH] Evaluate each of the following, and simplify your answers as
much as possible.
µ ¶
d x2 + 3x
(a) =
dx x

ANSWER ONLY
Notes for Lecture Section 002, MATH 140 2008 09 2297

Solution:
µ ¶
d x2 + 3x d
= (x + 3) for x 6= 0
dx x dx
= 1 + 0 = 1.

(The function is not defined for x = 0.)


d u
(b) (u ) =
du

ANSWER ONLY

Solution: This problem can be solved in different ways. For example,


d u d ¡¡ ln u ¢u ¢
(u ) = e
du du
since logarithm and exponential are mutual inverses
d ¡ u ln u ¢
= e
du
by the exponent rules
¡ ¢ d
= eu ln u · (u ln u)
du
by the Chain Rule
³ u´
= uu · 1 · ln u +
u
= uu · (ln u + 1) .

Equivalently, one could use the method of “logarithmic differentiation”: De-


fine y = uu , and take logarithms, to obtain ln y = u ln u; then differentiate
both sides of the equation with respect to u (the left side implicitly), and solve
d
the resulting equation to obtain du y in terms of u alone.
(c) An antiderivative F (x) of f (x) = 5x4 + 2x5 such that F (0) = 3 is

ANSWER ONLY
Notes for Lecture Section 002, MATH 140 2008 09 2298

Solution: All antiderivatives of this polynomial have the form F (x) = x5 +


2 6
6
x + C, where C is a constant of integration. Imposing the initial condition
that F (0) = 3 yields 3 = 0 + 0 + C; hence the antiderivative sought is
1
F (x) = x5 + x6 + 3 .
3

(d) If f (x) = x3 + 7, its inverse function f −1 (x) =

ANSWER ONLY

3

Solution: Solving
√ the equation y = x +7 yields a unique solution, x = 3
y − 7.
−1
Thus f (y) = y − 7. But the problem asked that the inverse function be
3

expressed in terms of an independent variable named x, not y; hence



f −1 (x) = 3 x − 7 .

d ¡ 4¢
(e) |x| =
dx

ANSWER ONLY

We cannot differentiate the absolute value function at x = 0. However, |x|2 =


x2 , so
d ¡ 4¢ d ¡ 4¢
|x| = x = 4x3 .
dx dx
3. SHOW ALL YOUR WORK!

(a) [6 MARKS] Showing all your work, determine values of the constants a and
b that will make the following function continuous everywhere.
 1

 (1 + x) x if x > 0

f (x) = a + bx if −1 ≤ x ≤ 0

 sin(x + 1)
 if x < −1
x+1
Notes for Lecture Section 002, MATH 140 2008 09 2299

Solution: The three pieces of the function are, indeed, continuous everywhere;
but we need to ensure continuity where the pieces are combined, i.e., at the
points x = −1 and x = 0. We know that lim − sin(x+1) x+1
= 1, and that
x→−1
lim + (a + bx) = a − b. The two-sided limit will exist if and only if these two
x→−1
one-sided limits are equal, i.e., if and only if

a − b = 1.
1
As x → 0+ , (1 + x) x → e; as x → 0− , a + bx → a. For the limit as x → 0 to
exist we must thus impose the condition that

a = e.

We can now solve the two equations, to conclude that b = e − 1.


(b) [4 MARKS] Determine whether f is differentiable at x = −1. (For the purpose
of this question you may assume that e is approximately 2.72.)
Solution: We must consider the limit as x → −1 of the difference quotient
( (a+bx)−(a−b)
f (x) − f (−1) x+1
= b = e − 1 if 0 > x > −1
= sin(x+1)
−(a−b)
sin(x+1)
−1
x+1 x+1
= x+1
if x < −1
x+1 x+1

We may apply l’Hospital’s Rule to the lower ratio, obtaining


sin(x+1)
−1
x+1 sin(x + 1) − (x + 1)
lim − = lim −
x→−1 x+1 x→−1 (x + 1)2
cos(x + 1) − 1
= lim −
x→−1 2(x + 1)
− sin(x + 1)
= lim − = 0 6= e − 1 .
x→−1 2x
Since the limit from the left is different from the limit from the right, the
2-sided limit does not exist; the derivative is defined to be that 2-sided limit
— consequently the function is not differentiable at x = −1.

4. SHOW ALL YOUR WORK!


[5 MARKS] Let f (x) = x2 ex . Prove carefully by mathematical induction that

dn f ¡ 2 ¢ x
(x) = x + 2nx + (n − 1)n ·e
dxn
Notes for Lecture Section 002, MATH 140 2008 09 2300

for all positive integers n.


Solution: Let S(n) denote the nth case of the preceding alleged equality. S(0)
states that ¡ ¢
f (x) = x2 + 0x + 0) ,
which is known to be true, as it is our definition of f . Suppose that it is known
that
dn f ¡ ¢
n
(x) = x2 + 2nx + (n − 1)n · ex .
dx
Then, differentiating with respect to x, we obtain
µ ¶
d dn f x
¡ 2 ¢ x
(x) = (2x + 2n) · e + x + 2nx + (n − 1)n ·e
dx dxn
¡ ¢
= x2 + 2(n + 1)x + n(n + 1) · ex

which is precisely S(n + 1). We may conclude, by the Principle of Mathematical


Induction, that S(n) is true for all non-negative integers n.
(This topic was not included in the syllabus of MATH 140 2007 09.)

5. SHOW ALL YOUR WORK!


[5 MARKS] Let g(x) = 2x − 3 + cos x. Use Rolle’s Theorem or the Mean Value
Theorem, to prove carefully that there exists exactly one real number x such that
g(x) = 0. (π may be taken to be approximately 3.14.)
Solution:

(a) Since g(0) = 2(0) − 3 + 1 = −2 < 0, and g(π) = 2π − 3 − 1 > 6 − 3 − 1 = 2 > 0,


and since the function g is continuous (being a sum of functions known to be
continuous), we may conclude by the Intermediate Value Theorem that there
is a point x such that 0 < x < π at which g(x) = 0.
(b) Suppose that there were two such zeros for the function g. Then, by Rolle’s
Theorem applied to the interval bounded by those zeros, we could conclude
that g 0 = 0 at some point. But g 0 (x) = 2 − sin x, and this function ranges
in value between 1 and 3 — it cannot be zero! From this contradiction we
infer that the assumed existence of two zeros for the function is false — the
function cannot equal zero more than once.
(c) Thus the number of zeros is not less than 1, and not more than 1: we conclude
that there is a unique zero.
Notes for Lecture Section 002, MATH 140 2008 09 2301

6. SHOW ALL YOUR WORK!


[10 MARKS] A rectangular poster is to be printed on a rectangular board of min-
imum area, leaving margins at the 4 sides. The top and bottom margins are each
10 cm, and the side margins are each 4 cm. If the printed area on the poster is
fixed at 1,000 cm2 , find the best dimensions for the board. Show all of your work,
and justify all of your statements. In your solution, you are expected to carefully
apply either the First or the Second Derivative Test, naming the test as you apply
it.
Solution: Denote the width and height of the board by w, h, measured in centime-
tres; these functions will be assumed to be positive, but we cannot assume any
upper bound for either of them. Then the width and height of the printed area
are, respectively, w − 8 and h − 20, and the constraint on the printed area is that

(w − 8)(h − 20) = 1000 ,


1000
which implies that w = + 8 . The area of the board, which is to be minimized,
h−20
is µ ¶
1000 8(h2 + 105h)
f (h) = wh = +8 ·h= .
h − 20 h − 20
Differentiating f yields, after reduction

8(h − 70)(h + 30)


f 0 (h) = ,
(h − 20)2

hence the only internal extremum is h = 70; (the value h = −30 does not lie in the
domain). For 0 < h < 70 f 0 (h) < 0; and for 70 < h f 0 (h) > 0. Hence h attains its
only local minimum at this point, and h = 70 is the global maximum point. The
minimum use of board will occur uniquely when h = 70 and
1000
w= + 8 = 28 .
70 − 20

7. SHOW ALL YOUR WORK!



Let f (x) = x2 − 1 .

(a) [1 MARK] State the domain of f .


Solution: The function is defined wherever the argument of the square root is
non-negative; i.e., where |x| ≥ 1.
Notes for Lecture Section 002, MATH 140 2008 09 2302

(b) [1 MARK] State precisely where f is differentiable.


Solution: This function is differentiable everywhere except where x = ±1.
While the function is, indeed, defined at those points, it is not differentiable
for two different reasons. First, differentiability requires that the function be
defined on both sides of the point, and f is not defined for x in the interval
−1 < x < 1. Secondly, the one-sided limits of the difference quotient on the
side where the limit is defined is infinite, as the tangents are approaching the
vertical. We do not admit a value of ∞ or −∞ when we speak of a function
as being defined .
(c) [2 MARKS] Define when a line x = a is a vertical asymptote to the graph of
f.
Solution: A line x = a is a vertical asymptote if the either of the one-sided
limits of the function is ±∞.
(d) [2 MARKS] Either
i. Find all vertical asymptotes; or
ii. Explain why the graph has no vertical asymptotes.
Solution: This function does not approach infinity, even though the tangent
lines to the curve are approaching the vertical.
(e) [2 MARKS] Determine the global maximum value of f , or explain why there
is none.
Solution: As x → ∞, and also as x → −∞, f (x) → ∞. Thus the function
has no global maximum.
(f) [2 MARKS] Determine the global minimum value of f , or explain why there
is none.
Solution: This function is a square root, and cannot be negative. Since it is
equal to 0 at two places — x = ±1 — it attains a global minimum there.

8. SHOW ALL YOUR WORK!


A function f (t) satisfies, for all real numbers t, the equation

t3 + f (t)3 + 6t2 · f (t) = 8 .

(a) [5 MARKS] Find an equation for the tangent to the graph y = f (t) at the
point (t, y) = (2, 0).
Solution: Differentiating the given equation implicitly with respect to t yields

3t2 + 3f (t)2 · f 0 (t) + 12t · f (t) + 6t2 · f 0 (t) = 0 ,


Notes for Lecture Section 002, MATH 140 2008 09 2303

which can be solved to yield

t2 + 4tf (t)
f0 = − .
(f (t))2 + 2t2
1
When t = 2 and f (t) = 0, this yields f 0 (2) = − . An equation of the tangent
2
is y − 0 = − 21 (t − 2), or t + 2y = 2.
(b) [5 MARKS] Showing all your work, determine the value of f 00 (2).
Solution: Differentiating the ratio obtained for f 0 (t) yields

(2t + 4f (t) + 4tf 0 (t)) ((f (t))2 + 2t2 ) − t(t + 4f (t))(2f (t)f 0 (t) + 4t)
f 00 = −
((f (t))2 + 2t2 )2

which yields ¡1 ¢
00 (4 − 4) 4
+ 8 − 2(2)(8) 1
f (2) = − = .
(0 + 8)2 2
Notes for Lecture Section 002, MATH 140 2008 09 2304

5.23 Supplementary Notes for the Lecture of November 26st,


2008
Release Date: Wednesday, November 26st, 2008
subject to correction

5.23.1 §4.9 Antiderivatives


Definition 5.14 A function F is an antiderivative of a function f on an interval I if
F 0 = f on that interval.

The term antiderivative is a “calculus book term” rather than a “mathematician’s term”.
Mathematicians are more likely to use the traditional term indefinite integral . Textbooks
sometimes differentiate between the terms antiderivative and indefinite integral , but
these are likely to be rationalizations to justify the newer term. In practice the terms
are interchangeable. The traditional notation for an antiderivative of a function f (x) is
Z
f (x) dx .

We can reformulate Corollary 5.63 to the Mean Value Theorem as


Corollary 5.81 [to the Mean Value Theorem] If F is one antiderivative of f on an
interval I, then the most general antiderivative of f on I is

F (x) + C ,

where C is an arbitrary constant.


Where we wish to denote the family of all antiderivatives of f (x), we usually write
Z
f (x) dx + C ,

where C or some other symbol is understood to represent a constant of integration that


ranges over all real numbers, and will possibly be determined by additional information
that is yet to be supplied about the particular antiderivative that is sought. C is not the
only letter that is used to represent this constant.

Tables of Antiderivatives Much of the information we have been collecting about


derivatives can be recast into information about antiderivatives. For example, if we know
that F and G are antiderivatives of f and g, then we know the antiderivative for f + g
shown in Table 5.81 on page 2304. There are several observations to be made from this
table:
Notes for Lecture Section 002, MATH 140 2008 09 2305

Function One antiderivative


f (x) F (x)
g(x) G(x)
f (x) + g(x) F (x) + G(x)
n xn+1
x (n 6= −1)
n+1
1
ln |x|
x
ex ex
cos x sin x
sin x − cos x
2
sec x tan x
sec x tan x sec x
csc x cot x − csc x
1
arctan x
1 + x2
1
−arccot x
1 + x2
1
√ arcsin x
1 − x2
1
√ − arccos x
1 − x2

Table 8: Some Antiderivatives


Notes for Lecture Section 002, MATH 140 2008 09 2306

1. There are several functions having two or more apparently different antiderivatives.
This is, in one sense, no surprise, as we saw above that a function with at least
one antiderivative has infinitely many. What may be surprising is that we are
seeing functions that we may not have observed earlier to be essentially the same
— differing only by an additive constant.
2. While there is an antiderivative given for the sum of two functions with known
antiderivatives, there is none given for their product. There will be situations
when we will know the antiderivatives of two functions, but we will not have one
available for their product! This is not because an antiderivative “does not exist”,
but because we may not have constructed the elements out of which to describe
an antiderivative. We will return to this topic in MATH 141. One example will be
the functions
2 1
xex and
x
of which we know such antiderivatives as
2
ex
and ln |x|
2
2
respectively, but where we will not have available an antiderivative of ex .
3. A substantial portion of our time in MATH 141 will be spend in investigating pro-
cedures for determining antiderivatives for functions. While the tedious determina-
tion of antiderivatives is more and more being done by computers, the procedures
followed in those searches are very much the business of mathematicians and the
other users of mathematics.

Rectilinear Motion The theory in this section is a restatement of the material in [1,
§3.7, pp. 221-223]. To solve some of the problems you will need to know the acceleration
due to gravity, usually taken to be approximately 32 feet/sec−2 . I will illustrate with
some problems.
Example 5.82 [7, Exercise 14, p. 358] Find the most general antiderivative of the func-
tion
sin θ
h(θ) =
cos2 θ
and check your answer by differentiation.
Solution: In MATH 141 we will see a general way of attacking problems of this type.
For now the solution depends upon observing, for example, that
d
sin θ = − cos θ .

Notes for Lecture Section 002, MATH 140 2008 09 2307

Thus
d
sin θ cos θ d 1
= − dθ = = sec θ
cos2 θ cos2 θ dθ cos θ
so one antiderivative is sec θ. This could also have been noticed if it had been observed
that
sin θ sin θ
h(θ) = = = tan θ · sec θ ,
cos2 θ 1
cos θ ·
sec θ
which, it may be recalled, is the derivative of sec θ. The most general antiderivative is
sec θ + C.
As in the previous problem solved, the functions are undefined at certain real num-
bers: the constants of integration could be different on the different intervals where the
functions are defined. In this case the singularities occur at the odd integer multiples of
π
2
, so there are infinitely many intervals of length π for which a constant of integration
needs to be specified, and these could be combined arbitrarily, depending on the other
information known about the function.

Example 5.83 [7, Exercise 4.10.42, p. 359] “Find f if it is known that f 000 (x) = sin x,
f (0) = f 0 (0) = f 00 (0) = 1.”
Solution: What we will be finding is an antiderivative of an antiderivative of an an-
tiderivative of f , and we will impose the preceding 3 “initial” values on the function to
determine the values of the “constants of integration”. We begin by interpreting the
equation f 000 (x) = sin x as
0
(f 00 ) (x) = sin x .
One antiderivative of sin x is − cos x, so the most general antiderivative is − cos x + C,
where C is a constant to be determined. Thus we know that

f 00 (x) = − cos x + C

and hence
1 = f 00 (0) = − cos 0 + C = −1 + C ,
from which we conclude that C = 2, so

f 00 (x) = − cos x + 2 .

We interpret this last equation as


0
(f 0 ) (x) = − cos x + 2 .
Notes for Lecture Section 002, MATH 140 2008 09 2308

An antiderivative of − cos x is − sin x, and an antiderivative of 2 is 2x, so we infer that

f 0 (x) = − sin x + 2x + K ,

where K is another constant of integration; use another letter to name that constant so
that it is not confused with the earlier constant. Imposing another of the initial value
conditions, we have
1 = f 0 (0) = − sin 0 + 2 · 0 + K = K ,
which determines this constant of integration:

f 0 (x) = − sin x + 2x + 1 .

Finally, an antiderivative of − sin x + 2x + 1 is cos x + x2 + x, so

f (x) = cos x + x2 + x + L ,

where L is a third constant of integration. The third initial condition gives

1 = f (0) = cos 0 + 02 + 0 + L

so L = 0, and f (x) = cos x + x2 + x.


(In some problems the information given will be such that the evaluation of the
constants may have to wait until the general antiderivative has been determined, and
then a set of equations will be determined, to be solved for the constants.)

4.10 Exercises

[1, Exercise 12, p. 345] Find the most general antiderivative of the function

5 − 4x3 + 2x6
g(x) =
x6
and check your answer by differentiation.
Solution: The “trick” here is not to attempt to work with the given ratio, but
to transform the function into a sum: you know the antiderivatives of a sum of
functions with known antiderivatives, but you know little about the antiderivatives
of a quotient. Since

5 − 4x3 + 2x6 5 4 2
6
= 6− 3+
x x x 1
= 5x − 4x + 2x0 ,
−6 −3
Notes for Lecture Section 002, MATH 140 2008 09 2309

one antiderivative is −x−5 + 2x−2 + 2x1 , and the most general antiderivative is

−x−5 + 2x−2 + 2x1 + C

where C is any real constant. We check by differentiation:


d ¡ −5 ¢
−x + 2x−2 + 2x1 + C = −(−5)x−5−1 + 2(−2)x−2−1 + 2(1)x1−1 + 0
dx
which can be seen to equal the given function after the summands are taken to a
common denominator.
Note, however, that the given functions are not defined at x = 0, and can surely
not be continuous there. Thus there could be different constants of integration that
apply for the two intervals, (0, ∞) and (−∞, 0) where the functions are defined.

[1, Exercise 30, p. 345] Find f , if it is known that f 0 (x) = 8x3 +12x+3, and f (1) = 6.
Solution: We have seen that we can find the antiderivative of a sum of powers by
d 4
taking the appropriate constant multiples of powers of x. Since x = 4x3 , an
dx
antiderivative of 8x3 is 2x4 ; analogously we find antiderivatives of the other two
summands; hence the most general antiderivative of f 0 (x) is

f (x) = 2x4 + 6x2 + 3x + C ,

were C is a constant to be determined. Imposing the condition that f (1) = 6 yields

6 = f (1) = 2(1)4 + 6(1)2 + 3(1) + C = 11 + C

from which we conclude that C = −5, and that

f (x) = 2x4 + 6x2 + 3x − 5 .

3
[1, Exercise 40, p. 345] Find f , if it is known that f 00 (t) = √ , f (4) = 20, f 0 (4) = 7.
t
00
Solution: The most general antiderivative of f is of the form
1
f 0 (t) = 3 (k) t− 2 +1 + C

where C is a constant of integration, and k is a constant that needs to be determined


by differentiation. Doing that, we find that
µ ¶
00 1 −1
f (t) = 3(k) t 2 +0
2
Notes for Lecture Section 002, MATH 140 2008 09 2310

µ ¶
1
so 3 = 3(k) , k = 2 and
2
1
f 0 (t) = 6t 2 + C ; (195)
a second such operation gives
µ ¶
2 1 +1
f (t) = 6 t 2 + Ct + E (196)
3

where E is a second constant of integration. The constant C may be determined


in equation (195) by setting t = 4 there:
³ 1´
7 = f 0 (4) = 6 4 2 + C = 12 + C ,

implying that C = 7 − 12 = −5. Substituting x = 4 in equation (196) yields


³ 3´
20 = f (4) = 4 4 2 + (−5)(4) + E ,

implying that E = 20 − 4(8) − (−20) = 8, so


3
f (t) = 4t 2 − 5t + 8 .

[1, Exercise 4.9.48, p. 345] “Find a function f such that f 0 (x) = x3 and the line
x + y = 0 is tangent to the graph of f .”
x4
Solution: The candidates are all functions f (x) = +C, where C is some constant,
4µ ¶
a4
to be determined. A general point on this curve is a, + C , at which the slope
4
of the tangent is f 0 (a) = a3 . (I am using the letter a to denote the abscissa of the
general point in order that I can comfortably talk about the equation of the tangent,
and have the letter x available for its traditional use.) An equation for the tangent
at this general point is
µ 4 ¶
a
y− + C = a3 (x − a) .
4

For this to be the same line as x + y = 0, they must have the same slope, so

−1 = a3 ,
Notes for Lecture Section 002, MATH 140 2008 09 2311

µ ¶
1
implying that a = −1, i.e., that the point of tangency is −1, + C . Thus the
4
tangent line is
µ ¶
1
y− +C = −(x + 1)
4
3
or x+y = C − .
4
We have imposed the condition that the tangent line have the desired slope, but the
given line passes through the origin; imposing that condition (that (x, y) = (0, 0)
satisfy the equation) here gives
3
0+0=C − ,
4
from which we conclude that C = 34 , so the desired function is

x4 + 3
f (x) = .
4
(This problem could be solved more elegantly by a more algebraic treatment; I have
avoided that route since not all students in the course are taking or have taken a
course in linear algebra.)

[7, Exercise 64, p. 360] A particle is moving with the following data; find the position
of the particle:

s00 (t) = 10 + 3t − 3t2


s(0) = 0
s(2) = 10

Solution: We know that position at time t is given by the displacement function


s(t); that velocity is v(t) = s0 (t), and that acceleration is a(t) = v 0 (t) = s00 (t).
Note that this is the first problem I have considered where the data are not the
initial values of the function and its derivatives, but are “boundary” values of the
function at different times.
The most general antiderivative of a(t) = s00 (t) is
3
v(t) = s0 (t) = 10t + t2 − t3 + C1 ,
2
Notes for Lecture Section 002, MATH 140 2008 09 2312

where C1 is a constant of integration. A second operation of antidifferentiation


gives
3 1 1
s(t) = 5t2 + · t3 − t4 + C1 t + C2 , (197)
2 3 4
where C2 is a second constant of integration. Imposing the boundary data gives
1 1
0 = s(0) = 5 · 0 + ·0− · 0 + C1 · 0 + C2
2 4
1 1
10 = s(2) = 5 · 4 + · 8 − · 16 + C1 · 2 + C2
2 4
implying that C1 = −5 and C2 = 0, so
1 1
s(t) = 5t2 + t3 − t4 − 5t + 0 ,
2 4

[1, Exercise 4.9.66, p. 346] “Two balls are thrown upward from the edge of a cliff
(432 feet above the ground). The first is thrown with a speed of 48 feet/s, and the
other is thrown a second later with a speed of 24 feet/s. Do the balls ever pass
dv
each other?” Assume that the acceleration is a(t) = = −32 feet/s2 .
dt
Solution: This problem is based on [1, Example 7, pp. 344].
Let’s set up a coordinate system for the moving balls. They will move along the
y-axis, and I will take the origin to be at the bottom of the cliff, with the positive
direction upward, and distance on the axis measured in feet. I will measure time
from the release of the first ball, and will denote the positions of the first and
second balls at time t respectively by y1 (t) and y2 (t).
The motion of the first ball is governed by the equation y100 (x) = −32. The most
general antiderivative of −32 is −32t + C1 — this is the velocity of the first ball.
If we impose the initial velocity condition for that ball, we obtain

48 = y10 (0) = −32(0) + C1 ,

from which we conclude that C1 = 48, and that the velocity of the first ball is given
by
y10 (t) = −32t + 48 .
From this we conclude that the position of the first ball is given by the equation

y1 (t) = −16t2 + 48t + K1 ,


Notes for Lecture Section 002, MATH 140 2008 09 2313

where K1 is a constant of integration that must be determined. Imposing the


information we have about the initial position of the ball, we obtain

432 = y1 (0) = −16(02 ) + 48(0) + K1 ,

so K1 = 432, and the position of the first ball is given by

y1 (t) = −16t2 + 48t + 432 .

Proceeding to the second ball, its acceleration satisfies the equation y200 (x) = −32,
so its velocity will be
y20 (t) = −32t + C2 .
When we impose the initial velocity condition for the second ball — but be careful,
the release time is t = 1 here — we obtain

24 = y20 (1) = −32(1) + C2 ,

so C2 = 56 and
y20 (t) = −32t + 56
for t ≥ 1; the equation is not valid for t < 1. A second iteration of antidifferentia-
tion yields
y2 (t) = −16t2 + 56t + K2
and we impose the condition that the ball be at position y2 = 432 when t = 1:

432 = −16(12 ) + 56(1) + K2

so K2 = 392, and
½ ¾
−16t2 + 56t + 392 when t>1
y2 (t) = .
432 when 0 ≤ t ≤ 1

Finally we are ready to answer the question. The excess of height of the first ball
over the second at time t is
½ ¾
−8t + 40 if t>1
y1 (t) − y2 (t) = .
−16t2 + 48t if 0 ≤ t ≤ 1

Indeed, this difference is 0 twice: when t = 0 and when t = 5. The solution t = 0


is just before the release of the first ball, and is certainly not the intention of the
formulator of the problem. The balls then pass each other 4 seconds after release
of the second ball.
Notes for Lecture Section 002, MATH 140 2008 09 2314

(If we had naively considered only the first line of this piecewise-defined function, it could
have happened, for different initial data, that we might have obtained a value of t in the
interval 0 < t < 1 as solution; that would be using a piece of the parabolic arc of the 2nd
ball which would not be applicable, since the 2nd ball is stationary before t = 1.)
The wording is somewhat ambiguous, as it appears to permit one ball to pass the other
when the other is stationery, which was probably not the intention. A more careful
solution might proceed as follows: First determine the position of the first ball:


 432 when t < 0 √
y1 (x) = −16t + 48t + 432 when 0 ≤ t ≤√3+32 13
2

 0 when t > 3+32 13

by solving to determine when the ball hits the ground. Then do the same calculations
for the 2nd ball:

 432 when t < 1
y2 (x) = −16t 2 + 56t + 392 = −8(2t − 13)(t + 3) when 1 ≤ t ≤ 13
2

0 when t > 13
2

When we work with y1 − y2 we need to restrict t to the interval from

t=1

to √
t = the minimum of 32 (1 + 13) and 13
2 ,

i.e. 1 ≤ t ≤ 13
2 . The value of t = 5 which we found is in that interval, so it is the point
we seek; had it been outside of the interval it would have been irrelevant, and the balls
would not pass in midair.)

Example 5.84 A Problem on the First Derivative Test for Global Extrema [1,
Review Exercise 50, p. 349]. “Find two positive integers such that the sum of the
first number and four times the second number is 1000, and the product of the numbers
is as large as possible.”
Solution: While the problem is phrased in terms of two numbers, we have to express it
as a single variable problem, since that is the only calculus machinery we have available
before MATH 222. Call the second number u, so the first number will be 1000 − 4u.
(Note that this definition is “cleaner” than one that would involve assigning a name to
the first number; but both approaches would lead to the same final answer.)
The function we wish to maximize is (1000 − 4u)u, and I propose to call it P (u).
The domain of P for the purpose of the problem is 0 ≤ u ≤ 250, since it would not be
Notes for Lecture Section 002, MATH 140 2008 09 2315

meaningful to permit the function P (u) to be negative. Thus the domain is a closed
interval. We have

P (u) = (1000 − 4u)u = 4(250u − u2 ) ,


P 0 (u) = 4(250 − 2u) , and
P 00 (u) = −8u .

The only critical number is u = 250


2
= 125. We apply the Closed Interval Method, and
tabulate the function values at the critical point and the end points of the domain:

u 125 0 250
P(u) 62,500 0 0

The global maximum occurs at u = 125, so the numbers must be 125 and 1, 000−4(125) =
500.

5.23.2 Draft Solutions to the December, 2006, Final Examination in MATH


140 2006 09 (one version)
Release Date: Monday, 26 November, 2007
(draft, subject to correction)

Students were advised that there were two kinds of problems on this examination,

• Some of the questions on this paper require that you SHOW ALL YOUR WORK!
Their solutions are to be written in the space provided on the page where the
question is printed. When that space is exhausted, you may write on the facing
page. Any solution may be continued on the last pages, or the back cover of the
booklet, but you must indicate any continuation clearly on the page where the
question is printed!

• Some of the questions on this paper require only BRIEF SOLUTIONS ; for these
you are expected to write the correct answer in the box provided; you are not asked
to show your work, and you should not expect partial marks for solutions that are
not completely correct.

1. BRIEF SOLUTIONS
[2 MARKS EACH] Give the numeric value of each of the following limits if it exists;
if the limit is +∞ or −∞, write +∞ or −∞ respectively. In all other cases write
“NO FINITE OR INFINITE LIMIT”.
Notes for Lecture Section 002, MATH 140 2008 09 2316

sin 2x
(a) lim =
x→∞ x

ANSWER ONLY

Solution: We know that, for all x,


−1 ≤ sin 2x ≤ +1 .
1
For positive x the inequalities are preserved when multiplied by :
x
1 sin 2x 1
− ≤ ≤+ .
x x x
1 1
As x → ∞, both − and approach 0; hence, by the Squeeze Theorem, the
x x
sin 2x
middle member of the inequalities must also approach 0, i.e., lim = 0.
x→∞ x
sin 2x
(b) lim =
x→0 x

ANSWER ONLY

Solution:
sin 2x sin 2x
lim = lim 2
x→0 x x→0 2x
sin 2x
= 2 lim
x→0 2x
sin y
= 2 lim defining y = 2x
y→0 y
= 2 × 1 = 2.
µ ¶x
2 7
(c) lim 1− + 2 =
x→∞ x x

ANSWER ONLY
Notes for Lecture Section 002, MATH 140 2008 09 2317

Solution:
 µ ¶ x
µ ¶x 2 7
ln 1 − + 2 
2 7 x x
lim 1− + 2 = lim e
x→∞ x x x→∞
µ µ ¶ ¶
2 7
ln 1 − + 2 · x
= lim e x x by the exponent rules
x→∞
µ µ ¶ ¶
2 7
lim ln 1 − + 2 · x
= ex→∞ x x by continuity of exponential
ln (1 − 2y + 7y 2 )
lim+
= ey→0 y defining y = 1
x
where l’Hospital’s Rule may be applied
 
−2 + 14y
 
 2

 lim
1 − 2y + 7y 

 
y→0+

1 

= e
−2 + 14y
lim+
= ey→0 1 − 2y + 7y 2
−2
= e1
= e−2

|x|
(d) lim =
x→0 x

ANSWER ONLY

Solution: The limit from the right is +1, while the limit from the left is −1.
Hence there is NO FINITE OR INFINITE LIMIT.

(e) lim ( x2 + 3x + x) =
x→−∞

ANSWER ONLY
Notes for Lecture Section 002, MATH 140 2008 09 2318

Solution:
à √ !
√ √ x 2 + 3x − x
lim ( x2 + 3x + x) = lim ( x2 + 3x + x) · √
x→−∞ x→−∞ x2 + 3x − x
√ √
( x2 + 3x + x)( x2 + 3x − x)
= lim √
x→−∞ x2 + 3x − x

( x2 + 3x)2 − x2
= lim √
x→−∞ x2 + 3x − x
(x2 + 3x) − x2
= lim √
x→−∞ x2 + 3x − x
3x
= lim √
x→−∞ 2
x + 3x − x
3x
= lim q ¡ ¢
x→−∞
x2 1 + x3 − x
3x
= lim √ q
x→−∞ 3
x2 · 1 + x
−x
3x
= lim q
x→−∞ 3
|x| 1+ x
−x
3x
= lim q since x < 0
x→−∞ 3
−x 1+ x
−x
3
= lim q
x→−∞ 3
− 1+ x
−1
lim 3
x→−∞
= Ã r !
3
lim − 1+ −1
x→−∞ x
lim 3
x→−∞
= Ã r !
3
lim − 1 + − lim 1
x→−∞ x x→−∞

3 3
= =− .
−1 − 1 2

2. BRIEF SOLUTIONS
Notes for Lecture Section 002, MATH 140 2008 09 2319

[3 MARKS EACH] For each of the following functions answer the question; if the
object requested does not exist, write “NONE”.

(a) The horizontal asymptotes to the graph of g(x) = 2 tanh x + 1 are

ANSWER ONLY

Solution:
µ ¶
ex − e−x
lim (2 tanh x + 1) = lim 2 x +1
x→∞ x→∞ e + e−x
µ x ¶
e − e−x
= lim 2 x + lim 1
x→∞ e + e−x x→∞
µ ¶
1 − e−2x
= 2 lim + lim 1
x→∞ 1 + e−2x x→∞
µ ¶
1−0
= 2 +1=3
1+0
µ x ¶
e − e−x
lim (2 tanh x + 1) = lim 2 x +1
x→−∞ x→∞ e + e−x
µ x ¶
e − e−x
= lim 2 x + lim 1
x→−∞ e + e−x x→−∞
µ 2x ¶
e −1
= 2 lim + lim 1
x→−∞ e2x + 1 x→−∞
µ ¶
0−1
= 2 + 1 = −2 + 1 = −1
0+1

Hence the horizontal asymptotes are y = 3 and y = −1.


p
(b) The vertical asymptotes to the graph of h(x) = |x2 − 1| are

ANSWER ONLY

The function h is continuous for all real x, and has no points where a one-sided
limit is infinite ⇒ there are NO vertical asymptotes.
Notes for Lecture Section 002, MATH 140 2008 09 2320

(c) A ladder 4 metres long rests against a vertical wall. If the bottom of the
ladder slides away from the wall at a speed of 1 metre per second, how fast is
the angle between the top of the ladder and the wall changing when the angle
is π6 radians?

ANSWER ONLY

Solution: If we denote by x the distance from the foot of the wall (at the
origin) to the foot of the ladder (on the positive x-axis), and by θ the angle
between the ladder and the wall, then sin θ = x4 . Differentiation with respect
dθ 1 dx 1
to time, t, yields cos θ = = metres/second. When θ = π6 radians,
dt 4 dt 4

3 dθ 1 2 1
cos θ = 2 , so = ·√ = √ .
dt 4 3 2 3
 2

 x if x < −1

0 if −1 ≤ x ≤ 0
(d) The removable discontinuities of m(x) = are at the

 2x if 0 < x < 3

2x if 3<x
following points:

ANSWER ONLY

Solution: This function is made by piecing together polynomials. Since poly-


nomials are continuous everywhere, the only points where m could be dis-
continuous would be where two polynomials are brought together; thus the
candidates are x = −1, 0, 3. We calculate the limits from left and right at
each of these three points:

lim m(x) = lim x2 = 1


x→−1− x→−1−
lim m(x) = lim 0 = 0 6= 1
x→−1+ x→−1+
lim m(x) = lim 0 = 0
x→0− x→0−
lim m(x) = lim 2x = 0 = lim− m(x)
x→0+ x→0+ x→0
lim m(x) = lim 2x = 6
x→3− x→3−
Notes for Lecture Section 002, MATH 140 2008 09 2321

lim m(x) = lim 2x = 6 = lim− m(x)


x→3+ x→3+ x→3

At x = −1 the limits from the two sides exist, but they are different. There
is no way to remove this discontinuity by redefining the function at x = −1.
At x = 0 the limits from the two sides exist, are equal, and are equal to the
function value. Thus x = 0 is not a discontinuity.
At x = 3 the limits from the two sides exist and are equal, but the function
is not defined there. Thus x = 3 is a discontinuity; but it is removable — we
have only to define the function there so that its defined value is equal to the
value of the 2-sided limit. We extend the definition of m by defining m(3) = 6,
and that fills the hole in the graph, and renders the function continuous —
so x = 3 is a removable discontinuity, the only such discontinuity.
3. BRIEF SOLUTIONS
[2 MARKS EACH] Evaluate each of the following, and always simplify your answers
as much as possible.
à 1 2
!
d x 2 + 3x 3
(a) =
dx x

ANSWER ONLY

Solution:
à !
d ³ −1 ´
1 2
d x 2 + 3x 3 − 13
= x + 3x
2
dx x dx
µ ¶
1 −3 1 4
= − x +3 −
2 x− 3
2 3
1 3 4
= − x− 2 − x− 3
2
d ln(ln u)
(b) e =
du

ANSWER ONLY
Notes for Lecture Section 002, MATH 140 2008 09 2322

Solution:
d ln(ln u) d
e = ln u
du du
1
=
u
1
(c) An antiderivative F (x) of f (x) = such that F (0) = 3 is
1 + x2

ANSWER ONLY

The most general antiderivative is F (x) = arctan x + C, where C is a constant


of integration. If we impose the given “initial” condition, we obtain 3 =
F (0) = arctan 0 + C, which implies that C = 3, so the antiderivative sought
is F (x) = arctan x + 3.
d ³ √
2
´
(d) t arcsin(t) + 1 − t =
dt

ANSWER ONLY

Solution:
d ³ √ ´ d d√
t arcsin(t) + 1 − t2 = (t arcsin(t)) + 1 − t2
dt dt dt
1 1 1
= 1 arcsin(t) + t · √ + ·√ · (−2t)
1−t 2 2 1 − t2
= arcsin t

4. SHOW ALL YOUR WORK!

(a) [1 MARKS] For a real number a in the domain of a function f , define precisely
what is meant by f 0 (a).
f (x) − f (a)
Solution: f 0 (a) = lim .
x→a x−a
Notes for Lecture Section 002, MATH 140 2008 09 2323

(b) [4 MARKS] Let f (x) = |1 − x|. Carefully, using the preceding definition,
either determine f 0 (1), or prove that f is not differentiable at x = 1. (No
marks will be given for a solution that uses the Differentiation Rules.)
Solution:

f (x) − f (1) −(1 − x) − 0


lim+ = lim+
x→1 x−1 x→1 x−1
= lim+ 1 = 1
x→1
f (x) − f (1) (1 − x) − 0
lim− = lim−
x→1 x−1 x→1 x−1
= lim− (−1) = −1 .
x→1

Since the one-sided limits are different, the two-sided limit — which is the
derivative — does not exist at x = 1.

5. SHOW ALL YOUR WORK!

(a) [1 MARKS] For a real number a in the domain of a function f , define precisely
what is meant by f 0 (a).
f (x) − f (a)
Solution: f 0 (a) = lim .
x→a x−a
(b) [4 MARKS] Let f (x) = |1 − x|. Carefully, using the preceding definition,
either determine f 0 (1), or prove that f is not differentiable at x = 1. (No
marks will be given for a solution that uses the Differentiation Rules.)
Solution:

f (x) − f (1) −(1 − x) − 0


lim+ = lim+
x→1 x−1 x→1 x−1
= lim+ 1 = 1
x→1
f (x) − f (1) (1 − x) − 0
lim− = lim−
x→1 x−1 x→1 x−1
= lim− (−1) = −1 .
x→1

Since the one-sided limits are different, the two-sided limit — which is the
derivative — does not exist at x = 1.
(c) [4 MARKS] It is known that x and y are related by the equation

x + y = 1 + x2 y 2 . (198)
Notes for Lecture Section 002, MATH 140 2008 09 2324

dy
Using any valid method, determine when x = 0.
dx
Solution: If we differentiate the given equation implicitly with respect to x,
we obtain
1 1
·√ · (1 + y 0 ) = 0 + 2xy 2 + x2 · 2y · y 0 .
2 x+y

When x = 0, y = 1 + 0, so y = 1. Substitution in the equation we obtained
yields 12 · 1 · (1 + y 0 ) = 0 + 0 + 0, implying that y 0 = −1.

6. SHOW ALL YOUR WORK!


[8 MARKS] Showing all your work, determine constants a, b, c so that the tangent
lines at x = −1 and x = 5 to the graph of

y = f (x) = ax2 + bx + c (199)

have slopes 6 and −2 respectively, and that the graph pass through the point (1, 4).
Solution: Differentiation
½ of (199) yields y¾0 = f 0 (x) = 2ax + b. Since f 0 (−1) = 6
6 = 2a(−1) + b
and f 0 (5) = −2, ⇒ 12a = −8 ⇒ a = − 32 ⇒ b = 14 .
−2 = 2a(5) + b 3

Hence f (x) = 13 (−2x2 + 14x + 2c). Imposing the condition that 4 = f (1) yields
4 = 31 (−2 + 14 +
¡ 2c) ⇒ c¢= 0, f (x) = 23 (−x2 + 7x). To summarize, we have shown
2 14
that (a, b, c) = − 3 , 3 , 0 .

7. SHOW ALL YOUR WORK!


A particle is moving in the plane on the curve C whose equation is

x = 2y − 4 + ey .

(a) [4 MARKS] Use the Mean Value Theorem to prove that C crosses the y-axis
exactly once. (You may assume it is known that 2 < e < 3.)
Solution: Since f (y) = 2y − 4 + ey is a sum of continuous functions, it is
continuous, and we may apply the Intermediate Value Theorem. Since f (2) =
4 − 4 + e2 > 0, and f (0) = 0 − 4 + 1 = −3 < 0, there must exist a point (0, y)
on C where 0 < y < 2; the information that 2 < e < 3 is not needed. Suppose
that there were 2 points, say (0, y1 ) and (0, y2 ), such that y1 < y2 and

0 = 2y1 − 4 + ey1 = 2y2 − 4 + ey2 .

Since f (y) = 2y − 4 + ey is differentiable everywhere, Rolle’s Theorem implies


the existence of c such that y1 < c < y2 and f 0 (c) = 0. But f 0 (y) = 2 + ey > 0
Notes for Lecture Section 002, MATH 140 2008 09 2325

for all y. From this contradiction we infer that there exists at most one point
(0, a) such that 0 = 2a − 4 + ea ; since we have already proved that there exists
at least one such point, we may conclude that C crosses the y-axis in exactly
one point (0, a).
(b) [3 MARKS] If P (0, a) is the point where C crosses the y-axis, give a linear
approximation in terms of a for the y-coordinate of the point on C where the
x-coordinate is 12 .
dx
Solution: We know that = 2 + ey = 2 + (x − 2y + 4) = x − 2y + 6. We are
dy¡ ¢
asked to approximate f −1 21 linearly in terms of f −1 (0) = a.
µ ¶ ¯
−1 1 −1 dy ¯¯
f ≈ f (0) + · ∆x
2 dx ¯x=0
1 1
= a+ a
·
2+e 2
1 1 −4a2 + 12a + 1
= a+ · = .
0 − 2a + 6 2 (3 − a)4

8. SHOW ALL YOUR WORK!


[8 MARKS] Showing all your work, use the calculus to find two positive real num-
bers such that the sum of the first and the square of the second is 300, and the
product of the two numbers is as large as possible.
Solution: Call the positive numbers being sought a, b, a being the first. The
constraint is a + b2 = 300. We wish to maximize ab = (300 − b√ 2
)b, which√we
2
call g(b), on the domain where b > 0 and b = 300 − a < 300, so b < 300 = 10 3;
thus the domain is an open interval.
g 0 (g) = 300 − 3b2 = 3(10 − b)(10 + b) has zeroes at b = ±10, but only b = 10 is in
the domain of g. For b < 10, g 0 (b) > 0; while, for b > 10, g 0 (b) < 0. By the First
Derivative Test for Global Maxima, the global maximum of g is√at b = 10. Hence
the first number sought is 300 − 102 = 200, while the second is 300 − 200 = 10.

9. SHOW ALL YOUR WORK!


Let f (x) = x − ln(1 + x2 ) .

(a) [2 MARKS] Showing all your work, determine the intervals where f is increas-
ing, and the intervals where it is decreasing.
1 (1 − x)2
Solution: f 0 (x) = 1 − · 2x = . The numerator is always non-
1 + x2 1 + x2
negative, while the denominator is positive. Thus f is increasing on each of
Notes for Lecture Section 002, MATH 140 2008 09 2326

the intervals where it is defined, i.e., on (−∞, 1) and on (1, +∞). It is never
decreasing.
(b) [2 MARKS] Showing all your work, determine whether f has local extrema,
and classify them, if any, as maxima or minima.
Solution: The only critical point is x = 1. This is the only candidate for a local
extremum. But the 1st derivative test shows that this is not an extremum,
as the function is less to the left of x = 1 and greater to the right. It follows
that there are no local extrema; a fortiori there are no global extrema (since
the domain of f is all of R).
(c) [3 MARKS] Showing all your work, determine all inflection points for f .
00 −2(1 − x)(1 + x2 ) − (1 − x)2 · 2x −2(1 − x)(1 + x)
Solution: f (x) = 2 2
= .
(1 + x ) (1 + x2 )2
Thus f 00 changes sign at both x = ±1. As the function is continuous at
both of these points, they are both inflection points; there are no others.
(d) [1 MARK] Sketch the graph of f .
Solution: see Figure 21 on Page 2327 of these notes.
Notes for Lecture Section 002, MATH 140 2008 09 2327

x
0
0

-2

Figure 21: Graph of the Function x − ln(1 + x2 )


Notes for Lecture Section 002, MATH 140 2008 09 2328

5.24 Supplementary Notes for the Lecture of December 01st,


2008
Release Date: Monday, December 01st, 2008
subject to correction

5.24.1 Solutions to December, 2007, Final Examination in MATH 140 2007


09 (One of several versions)
Instructions
1. Fill in the above clearly.

2. DO NOT TEAR PAGES FROM THIS BOOK! All your writing — even rough work — must
be handed in. You may do rough work anywhere in the booklet.

3. This is a CLOSED BOOK examination. CALCULATORS ARE NOT PERMITTED. Transla-


tion dictionaries are permitted; no other dictionaries are permitted.

4. OTHER CALCULUS EXAMINATIONS ARE BEING WRITTEN AT THIS TIME. THIS IS


THE EXAMINATION IN MATH 140 ONLY!

5. The examination booklet consists of this cover, Pages 1 through 7 containing questions; and
Pages 8, 9, and 10, which are blank. Your neighbour’s version may not be the same as yours.

6. There are two kinds of problems on this examination, each clearly marked as to its type.

• Some of the questions on this paper require that you SHOW ALL YOUR WORK!
Their solutions are to be written in the space provided on the page where the question
is printed. When that space is exhausted, you may write on the facing page. Any
solution may be continued on the last pages, or the back cover of the booklet, but you
must indicate any continuation clearly on the page where the question is printed!
• Some of the questions on this paper require only BRIEF SOLUTIONS ; for these you
are expected to write the correct answer in the box provided; you are not asked to show
your work, and you should not expect partial marks for solutions that are not completely
correct.

You are expected to simplify your answers wherever possible.


You are advised to spend the first few minutes scanning the problems. (Please inform the
invigilator if you find that your booklet is defective.)

7. A TOTAL OF 70 MARKS ARE AVAILABLE ON THIS EXAMINATION.


Notes for Lecture Section 002, MATH 140 2008 09 2329

1. BRIEF SOLUTIONS
[2 MARKS EACH] Give the numeric value of each of the following limits if it exists;
if the limit is +∞ or −∞, write +∞ or −∞ respectively. In all other cases write
“NO FINITE OR INFINITE LIMIT”.
x + x2
(a) lim =
x→∞ 1 − 2x2

ANSWER ONLY

Solution: (cf. [1, Exercise 4.4.16, p. 304])


1
x + x2 x
+1
lim = lim1
x→∞ 1 − 2x2 x→∞ 2 − 2
x
dividing numerator and denominator by the leading power, x2
1
lim + lim 1
= x→∞ x x→∞
1
lim 2 − lim 2
x→∞ x x→∞
0+1 1
= =− .
0−2 2

sin (3x2 )
(b) lim =
x→0 (sin 3x)2

ANSWER ONLY

Solution:
à µ ¶2 !
sin (3x2 ) sin 3x2 3x 3x2
lim = lim · ·
x→0 (sin 3x)2 x→0 3x2 sin 3x (3x)2
µ ¶2
sin 3x2 3x 3x2
= lim · lim · lim
x→0 3x2 x→0 sin 3x x→0 (3x)2

by the Product Law for Limits


1 1
= 1 · 12 · = .
3 3
Notes for Lecture Section 002, MATH 140 2008 09 2330

µ ¶
1
(c) lim+ arctan − =
x→0 x

ANSWER ONLY

1
As x → 0+ . − → −∞. By the construction of the inverse tangent function,
x
π
since limπ + tan x = −∞, the desired limit will be − .
x→− 2 2
u
ln
(d) lim 3 =
u→3 u − 3

ANSWER ONLY

Solution: Note that this problem should have been formulated for the right
limit as u → 3+ , since the function is not defined when u < 3. We can apply
l’Hospital’s Rule, since the limits of numerator and denominator are both 0.
u u 1
ln ·
lim 3 = lim+ 3 3
u→3+ u − 3 u→3 1
1
= lim+
u→3 u

1
if the latter limit exists. But this limit surely exists, and is equal to . By
3
1
L’Hospital’s Rule, the original limit will then also equal .
³√ ´ 3

(e) lim u2 + 2u + 4 − u2 − 3u + 1 =
u→−∞

ANSWER ONLY

Solution:
³√ √ ´
lim u2 + 2u + 4 − u2 − 3u + 1
u→−∞
Notes for Lecture Section 002, MATH 140 2008 09 2331

à !
³√ √ ´ √u2 + 2u + 4 + √u2 − 3u + 1
= lim u2 + 2u + 4 − u2 − 3u + 1 · √ √
u→−∞ u2 + 2u + 4 + u2 − 3u + 1
5u + 3
= lim √ √
u→−∞ u2
+ 2u + 4 + u2 − 3u + 1
¡ ¢
u 5 + u3
= lim q ¡ ¢ q ¡ ¢
u→−∞
u 1 + u + u2 + u2 1 − u3 +
2 2 4 1
u2
¡ ¢
u 5 + u3
= lim q q
u→−∞
|u| 1 + u + u2 + |u| 1 − u3 + u12
2 4

¡ ¢
5 + u3
= lim q q
u→−∞
− 1 + u + u2 − 1 − u3 + u12
2 4

since u < 0
5 5
= =− .
−1 − 1 2

2. BRIEF SOLUTIONS
[3 MARKS EACH] For each of the following functions answer the question; if the
object(s) requested does/do not exist, write “NONE”.

(a) The horizontal asymptotes to the graph of g(x) = 2 arctan x − 1 are

ANSWER ONLY

π
Solution: Since lim (2 arctan x−1) = 2· −1 = π−1, y = π−1 is a horizontal
x→∞ 2
−π
asymptote to the graph of g. Since lim (2 arctan x−1) = 2· −1 = −π−1.
x→−∞ 2
y = −π − 1 is also a horizontal asymptote to the graph of g.
 1 

 x2 −4
if x 6= −2, 0, 2 

 
6 if x = 2
(b) If f is defined by f (x) = , the vertical asymptotes

 5 if x = 0 

 
−4 if x = −2
to the graph of f are
Notes for Lecture Section 002, MATH 140 2008 09 2332

ANSWER ONLY

Solution: The vertical asymptotes are located where the limit of the function
from either left or right is ±∞. The only component of this function which
1 1
ever has an infinite limit is the ratio 2 = , which has an
x −4 (x + 2)(x − 2)
infinite limit only at x = ±2. Both the points x = ±2 have neighbourhoods
which are contained entirely in the domain of f — it’s not relevant that the
function has unusual definitions at the points. In either case the limit of the
function is infinite on one — in fact on both — sides of the point. It happens
that the function changes sign at the points, and that the one-sided limits are
opposite signs of infinity, but that also is irrelevant to the question of whether
the vertical lines at these points are vertical asymptotes. But there is no
vertical asymptote at x = 0: even though the function has a discontinuity at
this point, it is a jump discontinuity, not an infinite discontinuity.
(c) Air is being pumped into a spherical balloon so that its volume increases at
a rate of 10 cm3 /s. How fast is the radius of the balloon increasing when the
radius is 12 cm?

ANSWER ONLY

Solution: Except for changed data, this problem appears in the textbook as
[1, Example 3.9.1, p. 241]. Students were expected to know the formula for the
4
volume V of a sphere as a function of its radius r. That formula is V = πr3 .
3
dV dr
Differentiation implicitly with respect to time yields = 4πr2 · . We
dt dt
dV
can substitute = 10, r = 12, and solve to conclude that, for these data,
dt
dr 5
= .
dt 288π
3. BRIEF SOLUTIONS
[3 MARKS EACH] Evaluate each of the following, and always simplify your answers
as much as possible.
d¡ x¢
(a) x ln x =
dx
Notes for Lecture Section 002, MATH 140 2008 09 2333

ANSWER ONLY

Solution: This problem can be solved by “logarithmic” differentiation. Equiv-


alently one can proceed as follows:
d ¡ x ¢ d ³¡ ln x ¢ lnxx ´
x ln x = e
dx dx
d ¡ ln x· x ¢
= e ln x
dx
d x
= (e )
dx
= ex .
d
(b) cos(arcsin u) =
du

ANSWER ONLY

Solution:
Simplifying later: By the Chain Rule,
d 1 1 u
cos(arcsin u) = − sin(arcsin u)· √ = −u· √ = −√ .
du 1−u 2 1−u 2 1 − u2
Beginning with simplification: arcsin u lies between − π2 and π2 ; sin(arcsin u) =

u; hence cos(arcsin u) = ± 1 − u2 , where we must take the + sign, be-
cause the cosine is positive in quadrants I, IV. Hence
d d√ 1 1 u
cos(arcsin u) = 1 − u2 = · √ · (−2u) = − √ .
du du 2 1 − u2 1 − u2
(c) An antiderivative F (x) of f (x) = sinh x such that F (0) = −1 is

ANSWER ONLY
Notes for Lecture Section 002, MATH 140 2008 09 2334

Solution: The most general antiderivative of sinh x is F (x) = cosh x + C.


where C is a constant of integration. Imposing the given initial condition
yields −1 = F (0) = cosh 0 + C = 1 + C, implying that C = −2, so the
antiderivative desired is cosh x − 2.
s
t2 3−t
(d) Where f (t) = , f 0 (2) =
1 − t (3 + t)2

ANSWER ONLY

Solution: This function can be differentiated using the Product, Quotient,


and Chain Rules. Alternatively, using “logarithmic” differentiation, we have
¯ 2 ¯ ¯ ¯
¯ t ¯ 1 ¯ 3−t ¯
ln |f (t)| = ln ¯¯ ¯ + ln ¯ ¯
1 − t ¯ 2 ¯ (3 + t)2 ¯
1
= 2 ln |t| − ln |1 − t| + ln |3 − t| − ln |3 + t|
2
1 0 2 1 1 1 1
⇒ · f (t) = − · (−1) + · (−1) −
f (t) t 1−t 23−t 3+t
1 2 1 1 1 1
⇒ · f 0 (2) = − · (−1) + · (−1) −
f (2) 2 −1 23−2 3+2
1 1 7
= 1−1− − =−
2 5 10
(The use of absolute signs may put some students off. We have to deal with
the problem that f (t) is negative near t = 2, and that the factor 1 − t is also
negative near that point: this means that we can’t talk about the logarithm
of f (t), nor about the logarithm of 1 − t there. Another fix would be to write
s
t2 3−t
−f (t) =
t − 1 (3 + t)2

and now to take the logarithm of both sides in the usual way:
1
ln(−f (t)) = 2 ln t − ln(t − 1) + (ln(3 − t) − 2 ln(3 + t))
2
and then differentiate both sides in the usual way:
µ ¶
f 0 (t) 2 1 1 −1 1
− = + + −2
f (t) t t−1 2 3−t 3+t
Notes for Lecture Section 002, MATH 140 2008 09 2335

etc.)
r µ ¶
4 1 4 0 4 7 14
But f (2) = = − . Hence f (2) = − · − = .
−1 25 5 5 10 25

4. SHOW ALL YOUR WORK!

(a) [6 MARKS] Use Rolle’s Theorem and the Intermediate Value Theorem to
show that the curve y = 1 + 2x + x3 + 4x5 crosses the x-axis exactly once.
Solution:
i. This is [1, Exercise 17 p. 286]. The given function is a polynomial —
it is differentiable everywhere on R. Since differentiability is a stronger
property than continuity, this function satisfies the conditions of the In-
termediate Value Theorem on any finite closed interval in R.
ii. There are points where the function is negative, and other points where
it is positive. TO find the former we can observe that the dominant
term in this polynomial is 4x5 . If we choose x large enough negatively
we can arrange for this term to be negative, and for the value of y also
to be negative. For example, when x = −10, y = 1 − 20 − 1000 −
40, 000 = −401, 19 < 0. We don’t need to take such a large number –
we could experiment with smaller negative numbers, e.g., when x = −1,
y = 1 − 2 − 1 − 4 = −6 < 0. When x = 0 y = 1 > 0. Hence, by the
Intermediate Value Theorem, there is at least one value of x between −1
and 0 where y = 0. But there could be more than one.
iii. Suppose there were 2 values of x somewhere in R at both of which y = 0
— i.e., that the curve crossed the x-axis at least twice. Call the two
points a and b, and assume, for convenience, that a < b. The function is
differentiable on the interval (a, b), and it is continuous on [a, b], since in
fact it is differentiable everywhere on the line. By Rolle’s Theorem there
will exist c such that a < c < b and y 0 = 0 at x = c. But y 0 (x) = 2 +
3x2 + 20x4 ≥ 2 since squares can’t be less than 0. From this contradiction
we infer that there did not exist points a, b where the curve crossed the
x-axis — that there is, at most one such point.
iv. As we proved above that there is at least one such point, we can now
conclude that there is exactl one point where the curve crosses the x-axis.
(b) [4 MARKS] Showing all your work, determine the value of the constant K
Notes for Lecture Section 002, MATH 140 2008 09 2336

that will make the following function continuous at x = 0:



2
 Kx

if x > 0
f (x) = 1 − cos x .

 8 if x ≤ 0

Solution: The function is defined at and to the left and right of x = 0. More
precisely, it is defined for all x ≤ 0, and for 0 < x < π2 . Calculating the
one-sided limits of f at x = 0, we find that lim− f (x) = lim− 8 = 8. To
x→0 x→0
Kx2
evaluate lim+ f (x) = lim− we observe that limits of numerator and
x→0 x→0 1 − cos x
denominator are both 0, so we can’t use the Quotient Law for Limits. But we
can apply L’Hospital’s Rule:
d
Kx2 dx
(Kx2 ) 2Kx
lim− = lim d
= lim− = 2K.
x→0 1 − cos x x→0− dx
(1 − cos x) x→0 sin x

For the function to have a limit at x = 0 we must have equality between the
two one-sided limits, i.e., we must have 2K = 8, so K = 4. For continuity
we need to show that, with this value for K, the limit at x = 0 must equal
f (0). But the limit has already been seen to be 8, and the second line of
the definition states that this is the function value. Hence the function is
continuous if and only if K = 4.
Notes for Lecture Section 002, MATH 140 2008 09 2337

5.25 Supplementary Notes for the Lecture of December 02nd,


2008
Release Date: Tuesday, December 02nd, 2008
subject to correction

5.25.1 Solutions (continued) to December, 2007, Final Examination in MATH


140 2007 09 (One of several versions)
5. SHOW ALL YOUR WORK!
The equation x5 + x2 y + y 3 = 4y + 3 defines y implicitly as a function of
x near the point (x, y) = (1, 2). Showing all your work
(a) [3 MARKS] determine the value of y 0 at (x, y) = (1, 2);
Solution: By “implicit” differentiation with respect to x we obtain
5x4 + 2x · y 0 + x2 · y 0 + 3y 2 · y 0 = 4y 0 , (200)
implying that
5x4 + 2xy
y 0 (x) = .
4 − x2 − 3y 2
5 + 2(2)
When (x, y) = (1, 2), this implies that y 0 = = −1.
4 − 1 − 12
(b) [3 MARKS] determine the value of y 00 at (x, y) = (1, 2); and
Solution: Differentiate implicitly in equation (200) with respect to x:
2
20x3 + 2 (y + xy 0 ) + 2x · y 0 + x2 · y 00 + 6y (y 0 ) + 3y 2 y 00 = 4y 00 .
32
When (x, y, y 0 ) = (1, 2, −1), this yields y 00 = −.
9
(c) [3 MARKS] estimate y when x = 0.97 by using the tangent line to the curve
at the point (x, y) = (1, 2).
Solution: We evaluate the function at a = 1, taking ∆x = 0.97 − 1 = −0.03.
Since, when x = 1, y 0 = −1, the linearization of the function yields
y 0 (1 + (−0.03)) ≈ y(1) + (−0.03) · y 0 (1) = 2 − (0.03)(−1) = 2.03 .

6. SHOW ALL YOUR WORK!



 60
for 0 ≤ x ≤ 2
[10 MARKS] The function f is defined by f (x) = 1 + x2 . A

20 − 4x for 2 < x ≤ 5
rectangle with sides parallel to the coordinate axes has one vertex at the origin,
Notes for Lecture Section 002, MATH 140 2008 09 2338

one on the positive x-axis, one on the positive y-axis; and the fourth on the graph
of f . Showing all your work, use the calculus — no other method will be accepted
— to determine the maximum area of such a rectangle.
Solution: The objective function is the area of a rectangle with two of its sides
along the coordinate axes and its upper right-hand vertex on the curve. If we
denote the abscissa (x-coordinate) of that vertex by x, then the area, which I shall
denote by A(x), is given by
 60x
 if 0 ≤ x ≤ 2
A(x) = x · f (x) = 1 + x2

x(20 − 4x) if 2 < x ≤ 5

We calculate the derivative to be



 60(1 + x)(1 − x) if 0 < x < 2

0
A (x) = (1 + x2 )2

 20 − 8x if 2 < x < 5

I have excluded the end points of the interval since they must be tested separately
— and, anyhow, we don’t know the function on both sides of these points, so we
can’t find its derivative. As for the point x = 2, where the two parts of the curve
come together, we aren’t sure that the function has a derivative there: we would
have to compare limits from left and right of the difference quotients, which would
have to be calculated using the two different parts of the definition of the function.
While I haven’t actually proven that the function is not differentiable at x = 2,
there is no harm in including that point in the list of points where the value of
the function must be computed.120 In addition to the points x = 0, 2, 5 mentioned
above, we must also test the function at the critical points where the derivative
vanishes, i.e., at x =-1and + 1 and x = 52 ; but −1 is not in the interval of interest,
so only +1 need be considered. Thus the maximum must be attained at one or
more points in the list x = 0, 1, 2, 52 , 5. We calculate the following values:

x A(x)
0 0
1 30
2 24
5
2
25
5 0
120
The slope of the left part of the curve at x = 2 is − 180
25 = −7.2, while the slope of the right branch
of the curve is −4.
Notes for Lecture Section 002, MATH 140 2008 09 2339

from which we see that the global maximum occurs at x = 1, and the value of that
maximum — which is what is sought in the problem — is 30.

7. SHOW ALL YOUR WORK!


2
For x ≥ 0, define f (x) = xe−2x .

(a) [3 MARKS] Showing all your work, determine the intervals of its domain
where f is increasing, and the intervals where it is decreasing.
(b) [3 MARKS] Showing all your work, determine whether f has local extrema,
and classify them, if any, as maxima or minima. You are expected to base
your classification on tests studied in this course.
(c) [3 MARKS] Showing all your work, determine all inflection points for f .
(d) [1 MARK] Sketch the graph of f .

Solution:

(a) Differentiation yields


2 2 2 2
f 0 (x) = 1e−2x + x · e−2x · (−4x) = (1 − 4x2 )e−2x = (1 + 2x)(1 − 2x)e−2x .

In the product shown the first factor is positive for x > − 12 , and negative for
x < − 12 , the second factor is positive for x < 12 , and negative for x > 21 , and
the exponential factor is always positive. If we subdivide R at the points − 12
and + 12 , we see from the sign of the derivative that the function is increasing
between − 12 and + 21 , and decreasing inside the two residual rays x < − 12 and
x > 12 .
(b) The function is differentiable on the whole of R, and has 2 critical points.
The only candidates for local extrema are at the two critical points — the
1
points where f 0 = 0, i.e., at x = ± . Since the first derivative is equal to
2
the product of 1 linear factor associated with each of these critical points, the
first derivative changes sign at each of these points. Hence both of the critical
1
points are local extrema. The point x = is a local maximum because f 0
2
1
changes sign from positive to negative there. The point x = − is not a local
2
minimum: while f 0 changes sign from negative to positive at that point, the
domain prescribed for the function was x ≥ 0, so this point is outside of the
interval of interest.
Notes for Lecture Section 002, MATH 140 2008 09 2340

(c) Differentiation of f 0 yields


¡ ¡ ¢ ¢ 2
f 00 (x) = −8x + 1 − 4x2 (−4x) e−2x
à √ !à √ !
3 3 2
= 16x x − x+ e−2x .
2 2

3
We see that f 00 = 0 at x = 0, ± . Each of these points corresponds to
00
2
a linear factor of f of degree √ 1, so the concavity changes at each of them.
3
However, the point x = − is outside of the prescribed domain of the
2
function, so it is of no interest. The point x = 0 is an endpoint of the domain,
and we are not permitted to consider the behavior of the function to its left;
consequently x = 0 is also not √ a possible inflection point. Thus, for the
3
prescribed domain, only x = is an inflection point.
2
(d) A sketch of graph appears in Figure 22 on page 2341 of these notes.

5.25.2 Some additional solved problems in my notes


[1, Review Exercise 54, p. 349] “Find the volume of the largest (right) circular cone
that can be inscribed in a sphere of radius r.”
Solution: A right circular cone is a cone constructed from a disk by erecting a line
perpendicular to the plane of the disk, at the centre thereof, and joining a point on
that line — the apex of the cone to all points in the boundary of the disk. Suppose
that the sphere is formed from the circle x2 + y 2 = r2 by revolving the circle about
the y-axis. Think of the base of the cone as cutting the xy-plane in a line parallel
to the x-axis — say the line y = a, where −r√≤ a ≤ +a; denote the √ points of
intersection of this line with the circle by A(− r2 − a2 , a) and B(− r2 − a2 , a);
the apex of the cone will be the point (0, r) in the xy-plane. The mid-point of AB
is the point (0, a), which is the centre of the circular base of the cone. We are now
ready to describe the function to be optimized.
The volume of the cone is
1
× (area of base) × (height) .
3

The base is a disk of radius r2 − a2 , so its area is π(r2 − a2 ); the height of the
cone is the distance from the apex C(0, r) to the centre (0, a) of the base, i.e., r − a.
Notes for Lecture Section 002, MATH 140 2008 09 2341

0.3

0.25

0.2

0.15

0.1

0.05

0
0 0.5 1 1.5 2 2.5 3
x

2
Figure 22: Graph of xe−2x

Thus we wish to maximize the function


1 π
f (a) = · π(r2 − a2 )(r − a) = (r − a)2 (r + a)
3 3
over the domain − r ≤ a ≤ +r . Differentiation yields
0 π¡ ¢
f (a) = 2(r − a)(−1)(r + a) + (r − a)2
3
π π¡ ¢
= − (r − a)(r + 3a) = − r2 + 2ar − 3a2
3 3

f 00 (a) = − (r + a) .
3
The only points of the domain that could be critical points of the function are
Notes for Lecture Section 002, MATH 140 2008 09 2342

r
a = r and a = − , and the value a = r is excluded because it is an end-point of
3
the domain. By the Second Derivative Test, the fact that
³ r´ 4πr
f 00 − =− <0
3 9
tells us that the point is a local maximum. At the end points of the domain f = 0,
r
so the global maximum is at the unique critical point, a = − . The problem asks
3
for the maximum value of the volume; it is
³ r ´ 32
f − = πr3 .
3 81
[1, Review Exercise 58, p. 349] “A metal storage tank with volume V is to be con-
structed in the shape of a right circular cylinder surmounted by a hemisphere.
What dimensions will require the least amount of metal.”
Solution: Let r be the radius of the hemisphere and the base of the cylinder, and
let h be the height of the cylinder. Then the volume is
1 4π 3
V = · r + πr2 h
2 3
and so we may solve to obtain h in terms of r:
V 2r
h= 2
− .
πr 3
The area of sheet metal used (including the base) is
1 5π 2 2V
· 4πr2 + 2πr · h + πr2 = ·r +
2 3 r
which we shall denote by A(r). The domain of function A is 0 < r < ∞ — not a
closed interval!
The derivatives are
10π 2V 2(5πr3 − 3V )
A0 (v) = ·r− 2 = ,
3 r 3r2
10π 4V
A00 (v) = + 3 > 0.
3 r
µ ¶ 13
3V
The first derivative vanishes only at ; from the second derivative we see

that this is a local minimum point of A. Moreover, if we factorize the formula for
A0 as à µ ¶ 31 ! à µ ¶ 31 µ ¶ 32 !
0 10π 3V 2 3V 3V
A (r) = 2 r − r + r+
3r 5π 5π 5π
Notes for Lecture Section 002, MATH 140 2008 09 2343

we see that the last factor is always positive (because it is a quadratic factor without
real factors, so its sign is always that of the square term); consequently the deriva-
tive changes sign from negative to positive as r passes from left to right through
¡ ¢ 13
the value r = 3V 5π
. It follows that the function attains its global minimum at
¡ 3V ¢ 13
r = 5π .
Information for Students in MATH 140 2008 09 3001

6 Assignments from Previous Years


6.1 Fall 1998 Problem Assignments
(For most of these problems the answer was in the back of the (then) text-book [31][33],
but the full solution was not in the Student Solution Manual [32][34].)

Assignment Due Exercise Numbers


Number Date
1 25 Sept./98 Chapter 2 Miscellaneous Problems: 2,
5, 9, 12, 17, 21, 26, 28, 32, 35, 38, 40,
50, 56, 57, 59, 61
2 9 Oct./98 §3.1: 11, 17, 20, 36, 41
§3.2: 2, 5, 9, 15, 21, 33, 45, 48, 53
§3.3: 14, 15, 17, 44, 50, 53
3 23 Oct./98 §3.4: 45, 50
§3.5: 2, 6, 9, 21, 27, 45
§3.6: 21, 26, 32
§3.7: 62, 68, 74
4 6 Nov./98 §3.8: 17, 21, 26, 29, 35, 47
(deferred to §3.9: 3, 8, 11, 23, 27, 50, 51, 59
9 Nov.) Chapter 7 Miscellaneous Problems: 3,
11, 17, 21
Chapter 8 Miscellaneous Problems: 5, 9
5 23 Nov./98 §4.2: 3, 8, 13, 20, 26, 40, 41, 42
§4.3: 15, 23, 29, 33
§4.4: 5, 11, 15, 30
§4.5: 21, 32
§4.6: 31, 49
§4.7: 2, 6, 9, 15, 29, 47

Table 9: 1998 Problem Assignments

6.2 Fall 1999 Problem Assignments


6.2.1 First Fall 1999 Problem Assignment, with Solutions
Students were advised as follows in the problem set and its solutions:
Information for Students in MATH 140 2008 09 3002

• This assignment is intended to help you determine gaps in your background. It


will not be graded, but the solutions will be made available on the Web. If you
have difficulty with the problems or the solutions which will be posted, you should
discuss these with one of the tutors.

• Do not use a calculator when solving these problems, even for simple arithmetic.
(You may, however, wish to use a calculator afterwards to verify whether your
answers are ‘reasonable’.)

• Do not attempt to approximate square roots, or π. Where possible, however,


formulæ should always be simplified .

• Unless specifically stated in degrees, all angles should be assumed to be expressed


in radians. (Remember that the straight angle, 180◦ is equal to π radians, so an
π πd
angle of 1◦ is equal to 180 radians, and d◦ is equal to 180 radians.)

• Caveat lector! As in any duplicated materials, there could be misprints or errors.

1. (a) Give a right-angled triangle in which one angle is α = π3 , and use the triangle
to determine the values of sin α, cos α, tan α.
(b) Give a right-angled triangle in which one angle is β = π4 , and use the triangle
to determine the values of sin β, cos β, cot β, sec β.
It is not sufficient to state the values: you should explain how you determine them
in terms of the lengths of the sides of your triangle.
Solution:

(a) We take the right half of an equilateral triangle, each of whose sides has length
2. In ∆DBC ∠C = π3 = α. DB bisects ∠ADC and meets AC in its midpoint,
B. Then

|BD| 3
sin α = =
|CD| 2
|BC| 1
cos α = =
|CD| 2

|BD| 3 √
tan α = = = 3
|BC| 1

(b) Here we take an isosceles ∆EF G, in which EF = EG = 1, and ∠F = 45◦ = β.

|EG| 1
sin β = =√
|F G| 2
Information for Students in MATH 140 2008 09 3003

D
·T
· T
· T
· T
· T
· T
· √ T
F @
2 · 3 T 2
· T @
· T @
· T @ √
· T 1 @
2
· T @
· T @
· T @
· 1 90◦ 1 60◦T 90◦ 45@

· T 1 @
A B C E G

|EF | 1
cos β = =√
|F G| 2
|EF |
cot β = =1
|EG|

|F G| 2 √
sec β = = = 2
|EF | 1

(Note: Some students may have been taught never to leave a surd in the
denominator, as in the fraction √12 above. This convention derives from the

difficulties, in the days before calculators, of working with numbers like 2.
While it still is useful, for hand calculations, to confine surds to the numerator,
you may work with fractions like √12 if you are happy with them.)

2. State, without proof,

(a) a formula which expresses sin(x + y) in terms of the sines and cosines of x
and y.
(b) a relationship between sin x and sin(−x), and another between cos x and
cos(−x).
(c) By applying the results of parts 2b, 2a above, determine a formula that ex-
presses sin(x − y) in terms of the sines and cosines of x and y.
Information for Students in MATH 140 2008 09 3004

The formulæ you state should be valid for all values of x and y.
Solution:

(a)
sin(x + y) = sin x cos y + cos x sin y (201)
[31, A-15 (6)]
(b) sin(−x) = − sin x; cos(−x) = − cos x [31, A-15 (4)]
(c) Replacing y by −y throughout (201), one obtains

sin(x − y) = sin x cos(−y) + cos x sin(−y)


= sin x cos y + cos x(− sin y)
= sin x cos y − cos x sin y (202)

π π
3. (a) By substituting x = α = 3
and y = β = 4
in the formula in 2c, determine
the value of sin 15◦ .
π
(b) Using the result of 3a, determine the value of cos 12 .
Solution:

(a)
µ ¶
π π
sin = sin − i4
12 3 p
π π π π
= sin cos − cos sin
à √3 ! µ 4 3
¶ µ ¶ µ
4

3 1 1 1
= · √ − · √
2 2 2 2
√ √ √
3−1 6− 2
= √ = .
2 2 4
√ √ √
( 3−1)2
(b) π
cos2 12=1− 8
= 2+4 3 ; hence cos 12
π
is one of the square roots of 2+4 3 .
Since theq
angle is in the first quadrant, the sign is positive. It follows that

π 2+ 3
cos 12 = 4
. This can be seen — you were not expected to see this —

3+1
that the square root is √
2 2
.

4. State, without proof, formulæ which express cos(x + y) and cos(x − y) in terms of
the sines and cosines of x and y.
Information for Students in MATH 140 2008 09 3005

Solution:

cos(x + y) = cos x cos y − sin x sin y (203)


cos(x − y) = cos x cos y + sin x sin y (204)

[31, A-15 (7)]

5. (a) Specialize the formula in Problem 4 (by making a suitable choice for y in
terms of x) to express cos 2x in terms of sin x and cos x.
(b) Use the identity sin2 x + cos2 x = 1 to express cos 2x in terms of cos x alone;
and in terms of sin x alone.
(c) Apply your formulæ in 5b to determine the values of cos π6 and cos π2 from the
π
sines of 12 and π4 .
Solution:

(a) Taking y := x in (203) yields

cos 2x = cos2 x − sin2 x (205)

(b)

cos 2x = cos2 x − (1 − cos2 x) = 2 cos2 x − 1 (206)


= (1 − sin2 x) − sin2 x = 1 − 2 sin2 x (207)

(c)
π ³π´ ³π´
2
cos = cos 2 = 1 − 2 sin
6 12√ 12 √ √
( 3 − 1)2 1+3−2 3 3
= 1−2· =1−2· =
8 8 2
µ ¶2
π π 1
cos = 1 − 2 sin2 = 1 − 2 √ =0
2 4 2

6. Find an equation for each of the following lines in the plane:

(a) The line through the points (−1, 4) and (−2, 1).
(b) The line through the point (4, −1) which is perpendicular to the line

2x + y = −7 . (208)

Solution: (This problem is from the 1998 Final examination in 189-112A.)


Information for Students in MATH 140 2008 09 3006

1−4
(a) The slope of the line will be (−2)−(−1)
= 3, so an equation is y−4 = 3(x−(−1))
or y = 3x + 7.
(b) The slope of the line we seek will be the negative reciprocal of the line y =
−2x − 7, which has slope −2; thus the line will have slope 21 . One equation
will be y − (−1) = 21 (x − 4), or x − 2y = 6.
7. (a) It is claimed that the equation (2x + y + 7)2 = 0 represents the same points
as the equation 2x + y + 7 = 0. Determine whether the claim is correct.
(b) Determine what is represented by the equation (2x + y)2 = (−7)2 , i.e. by the
equation obtained by squaring both sides of equation (208).
Solution:
(a) The equation of a line is a constraint satisfied by the coordinates of its points,
and satisfied by the coordinates of no other points. As (2x + y + 7)2 may
be zero if and only if 2x + y + 7 = 0, this is another equation for the same
line. (Some authors would, however, say that this is “the equation of two
coincident lines”.)
(b) (The symbol ⇔ means that the statements which it connects are logically
equivalent.)
(2x + y)2 = (−7)2 ⇔ (2x + y)2 − (−7)2 = 0
⇔ (2x + y − 7)(2x − y + 7) = 0
⇔ 2x + y − 7 = 0 or 2x + y + 7 = 0
Thus the given equation is satisfied by the coordinates of the points on either
of the parallel lines 2x + y − 7 = 0 and 2x + y + 7 = 0; the equation represents
the union of the sets of points on the two lines.
8. Determine the centre and radius of the circle x2 + y 2 + 4x − 6y + 3 = 0.
Solution: (This problem is from the 1998 Final examination in 189-112A.) We
complete the squares separately.
x2 + y 2 + 4x − 6y + 3 = 0
⇔ (x2 + 4x) + (y 2 − 6y) + 3 = 0
à µ ¶2 ! µ ¶2 à µ ¶2 ! µ ¶2
4 4 −6 −6
⇔ x2 + 4x + − + y 2 − 6y + − +3=0
2 2 2 2
⇔ (x + 2)2 + (y − 3)2 = 4 + 9 − 3 = 10

⇔ (x − (−2))2 + (y − 3)2 = 4 + 9 − 3 = ( 10)2

Hence the centre of the circle is (−2, 3), and its radius is 10.
Information for Students in MATH 140 2008 09 3007

9. Simplify completely the following


à √ formula,
!−1 leaving your answer free of negative
√6
3
a3 b2
exponents or radicals: b· √ .
a6 b3
Solution: (This problem is from the 1998 Final examination in 189-112A.)
Ã√ !−1 √

6
3
a3 b2 √
6 a6 b3
b· √ = b· √
3
a6 b3 a3 b2
1 3 1 3 5
b 6 a3 b 2 a3 b 6 + 2 a3 b 3
= 2 = 2 = 2
a1 b 3 ab 3 ab 3
5 2
= a3−1 b 3 − 3 = a2 b

10. Solve the equation


1 1 4
+ = . (209)
x x−2 3
Solution: (This problem is from the 1998 Final examination in 189-112A.) If we
multiply both sides of the equation121 by 3x(x − 2), we obtain the polynomial
equation 3(x − 2) + 3x = 4x(x − 2), which reduces to 4x2 − 14x + 6 = 0, then to
2x2 − 7x + 3 = 0, which is equivalent to (2x − 1)(x − 3) = 0. The product on the
left can be zero only with at least one of the factors is zero, i.e. when either x = 21
or x = 3. These are the only solutions to (209).
11. Determine the “natural” (i.e. largest possible) domain of definition of each of the
following functions. Explain your results.

x
(a) f (x) = .
2 − sin x
1
(b) .
log3 (x + 1)
Solution: (This problem is from the 1998 Final examination in 189-112A.)
121
We should verify, when we have found the alleged solutions to the equation, that this operation of
multiplying both sides by the same quantity did not entail the introduction of any values which were
not solutions. For example, if we had multiplied by 3x2 (x − 2), the subsequent computations would have
produced a 3rd degree equation whose solutions would be 0, 12 and 3 — but x = 0 is evidently not a
solution, since the left side is not even defined there. Similarly, if we had multiplied by 3x(x − 2)(x + 11),
we would have obtained x = 11 as one of the solutions of the polynomial equation, but it is certainly
not a solution of the original equation. While the procedure could be made more rigorous, the safest
policy is to verify that all claimed solutions are indeed solutions of the original equation — particularly
when, in the course of your solution, you have multiplied both sides of an equation by an expression
that could be 0.
Information for Students in MATH 140 2008 09 3008

(a) The denominator is defined for all x. The numerator is defined for all non-
negative x. The ratio function is defined only for non-zero denominators,
so we must ensure that 2 − sin x is not zero; but this can never happen, as
| sin x| ≤ 1. Thus the largest possible domain for this function is the set of all
non-negative real numbers.
(b) Logarithms are defined only for positive numbers, so we must require that x +
1 > 0, i.e. that x > −1. But the logarithm here appears in the denominator,
which can assume any value except 0. log3 (x+1) = 0 precisely when x+1 = 1,
i.e. x = 0. Thus the largest possible domain for this function is the union of
the set −1 < x < 0 with the set x > 0 of positive real numbers.

12. Factorize the following polynomial completely:

(x2 + 4xy + 4y 2 ) − (3x + 6y)

Solution: (This problem is from the 1998 Final examination in 189-112A.) Students
were expected to observe that the first summand is a perfect square.

(x2 + 4xy + 4y 2 ) − (3x + 6y) = (x + 2y)2 − 3(x + 2y)


= (x + 2y)(x + 2y − 3)

is the desired factorization.

13. Solve the system of equations:

3x − 7y = 1
4x + 3y = 5

Solution: (This problem is from the 1998 Final examination in 189-112A.) Students
will study the systematic solution of systems of linear equations in courses on
linear algebra and matrices. This problem is simply to detect whether you are
able to solve a routine small system, even if your methods are not systematic.
Subtracting 3 times the second equation from 4 times the first equation yields
11
−37y = −11, implying that, if there is a solution, y = 37 ; this, substituted into
the first equation, yields x = 38 37
. (This solution can be verified by substituting
in the second equation. While that is hardly necessary in the present problem,
substitution should be undertaken in large linear systems, unless care has been
taken to avoid the possibility that some constraint has been lost. This is beyond
the present course, but will be studied in courses in linear algebra. What is at
issue is the possibility that a system of equations may have no solutions at all, or
may have infinitely many solutions.)
Information for Students in MATH 140 2008 09 3009

14. Find all solutions to the equation

log2 (3x + 2) + log2 (x + 1) = 2 (210)

Solution: (This problem is from the 1998 Final examination in 189-112A.)

log2 (3x + 2) + log2 (x + 1) = 2


⇒ log2 ((3x + 2)(x + 1)) = 2 = log2 22
⇔ 3x2 + 5x + 2 = 4
⇔ 3x2 + 5x − 2 = 0
⇔ (3x − 1)(x + 2) = 0
1
⇔ x= or x = −2
3
But the value x = −2 is extraneous, as the logarithm is not defined at −2 + 1 =
−1 < 0, or at 3 · (−2) + 2 = −4 < 0. We conclude that the only possible
solution to (210) is x = 13 , and verify that it does indeed satisfy the equation, as
log2 3 + log2 34 = log2 4 = log2 22 = 2.

15. Determine all real numbers x such that

(a) 5 tan2 x − sec2 x = 11


5
(b) cos x + sec x = 2

Solution:

(a)

5 tan2 x − sec2 x = 11 ⇔ 5 tan2 x − (tan2 x + 1) = 11


⇔ 4 tan2 x = 12

⇔ tan x = ± 3

The smallest solutions in absolute value are x = ± π3 . From the period-


icity
© of the tangent
ª function we conclude that the set of all solutions is
1
(n ± 3 )π : n ∈ Z where Z is the set of all integers.
(b)
5 1 5
cos x + sec x = ⇔ cos x + =
2 cos x 2
2
⇔ 2 cos x − 5 cos x + 2 = 0
⇔ (2 cos x − 1)(cos x − 2) = 0
Information for Students in MATH 140 2008 09 3010

1
⇔ cos x = or cos x = 2
2
1
⇒ cos x = as | cos x| ≤ 1
µ 2 ¶
1
⇔ x = 2n ± π
3

16. Show that


1 x2 − x + 1
≤ 2 ≤3 (211)
3 x +x+1
for any real number x.
Solution: Let us first prove the inequalities
x2 + x + 1
≤ x2 − x + 1 ≤ 3(x2 + x + 1)
3
i.e. equivalently, the two inequalities

x2 + x + 1 ≤ 3(x2 − x + 1)
x2 − x + 1 ≤ 3(x2 + x + 1)

The first of these is equivalent to 2(x − 1)2 ≥ 0, which is true because the left side
is a square, hence non-negative; the second is equivalent to 2(x + 1)2 ≥ 0, which is
non-negative for the same reason. In order to pass from these two inequalities to
(211) we need to divide by x2 + x + 1. This is a quadratic polynomial, having no
real roots; its sign is that of the leading coefficient x2 , i.e. it is positive; this can
be shown by observing that x2 + x + 1 = (x + 12 )2 + 34 ≥ 0 + 34 > 0. So dividing
the members of an inequality by this positive quantity preserves the inequalities,
thereby yielding the desired pair of inequalities.
2
A more elegant solution can be found if we define z = xx2 +x+1
−x+1
, and transform this
equation to yield a quadratic equation for x in terms of z:

x2 − x + 1 − z(x2 + x + 1) = 0
⇔ (1 − z)x2 − (1 + z)x + (1 − z) = 0

For this equation to admit a solution for every x, the discriminant must be non-
negative, i.e.
(1 + z)2 − 4(1 − z)2 ≥ 0
This is equivalent to

[(1 + z) − 2(1 − z)][(1 + z) + 2(1 − z)] ≥ 0


Information for Students in MATH 140 2008 09 3011

¡ ¢ ¡ ¢
which is equivalent to − z − 13 (z − 3) ≥ 0. For the product z − 31 (z − 3) to
be non-positive, z must lie between the roots, i.e. z must be such as to make one
of the factors negative and the other positive. This can be achieved only with
1
3
≤ z ≤ 3, as required.

6.2.2 Second Fall 1999 Problem Assignment


1. Determine if lim f (x), lim f (x), and/or lim f (x) exist for each of the following
x→1+ x→1− x→1
functions, and evaluate those that do exist.

(a) f (x) = x2 + x − 2
½ 2
x − 3 for x ≤ 1
(b) f (x) =
2 − x for x > 1
½ √
2 − x for x < −3
(c) f (x) = 3
x − x for 2 > x ≥ −3
2. In each of the following cases, evaluate the limit, or show that it does not exist.
µ ¶
1 1
(a) lim −
x→3 x2 − x − 6 x − 3
µ ¶
5 1
(b) lim −
x→3 x2 − x − 6 x − 3
122
3. In each of the following cases, evaluate the limit, or show that it does not exist.

3x + 2 x
(a) lim
x→∞ 1−x
|2x − 3|
(b) lim
x→−∞ x + 2

1
x+
4. Evaluate lim x2
x→0 2
− 3x2
x2
123
5. Find a value for a so that
µ ¶
x2 + 1
lim 3x + 1 − a
x→∞ x+1
exists as a finite limit, and evaluate that limit.
122
Added 21.09.99: This problem should be omitted. It is based upon [31, §§4.5,4.7], which material
may not have been discussed in the lectures before the due date of the assignment.
123
See footnote 122.
Information for Students in MATH 140 2008 09 3012


 2 when x ≤ −2
6. Sketch the graph of f (x) = x + 4 when −2 < x < 4 .

20 − x2 when 4≤x
Find the values of x for which f (x) is not continuous.
7. Find values of a and b which will make the following function continuous:
 2
 x + 2a when 0 < x < 1

f (x) = bx + a when 1 ≤ x < 2 .
 2b − 3 when

2≤x
x
x2 − 2x − 3
8. Let f (x) = . Determine where f (x) is not continuous, and where
4 − x2
x2 − 2x − 3
f (x) = 0. Then solve the inequality ≥ 0. [Hint: One method to solve
4 − x2
this problem uses the Intermediate Value Theorem.]
3 1 − 4x
9. Solve the inequality ≤ . [Hint: Bring everything to one side of the
x+1 x−1
equation and simplify first.]

6.2.3 Third Fall 1999 Problem Assignment, with Solutions


Caveat lector! There could be misprints and other errors in these draft solutions.
1. (a) Apply the
r definition of the derivative as a limit to determine the derivative of
7x − 1
f (x) = for any x in the domain.
3
Solution:
q q
7(x+h)−1 7x−1
3
− 3
0
f (x) = lim
h→0
q h q
q q
7(x+h)−1 7x−1 7(x+h)−1 7x−1
3
− 3 3
+ 3
= lim ·q q
h→0 h 7(x+h)−1
3
+ 7x−13
7(x+h)−1 7x−1
3
− 3 1
= lim ·q q
h→0 h 7(x+h)−1 7x−1
3
+ 3
7h
3 1
= lim ·q q
h→0 h 7(x+h)−1 7x−1
3
+ 3
Information for Students in MATH 140 2008 09 3013

7 1
= lim ·q q
h→0 3 7(x+h)−1 7x−1
3
+ 3
7 1
= lim q q
3 h→0 7(x+h)−1 7x−1
3
+ 3
7 1
= q
6 7x−1
3

(b) Determine the value of f 0 (7) from the result of 1a.


Solution: √749−1 = 7
24
6 3

2. You will find below the definitions for a number of functions. If the domain of
definition is not stated, then you are to assume the domain to be as large as
possible.

• Determine the derivative of each function wherever it is defined in the domain.


• List any points in the domain where the function does not possess a derivative,
and explain precisely why there fails to be a derivative.
• If possible, determine, for those functions where requested, the equation of
the tangent or the normal to the graph of the function at the given point.
x
(a) f1 (x) = ; tangent at (1, f1 (1)).
5 − 8x
Solution: The domain of f1 is R − { 85 }, the set of all non-zero real numbers x
for which the denominator is non-zero; the function is continuous and differ-
entiable at every point of the domain. f10 (x) = 1(5−8x)−x(−8)
(5−8x)2
5
= (5−8x) 2 , at every
5
point x in the domain, i.e. at all real numbers except x = 8 . As f10 (1) = 59 ,
the equation of the tangent at (1, − 13 ) is y − (− 13 ) = 59 (x − 1), i.e. 5x − 9y = 8.
(b) f2 (x) = (5 + 6x2 + x4 ) − (1 + 2x + x2 )2 + 4x(1 + x); normal at (5, f2 (5)).
Solution: As f2 is a polynomial, its domain is the entire real line; it is dif-
ferentiable (hence continuous) at all points of R also. f20 (x) = (12x + 4x3 ) −
2(1 + 2x + x2 )(2 + 2x) + (4 + 8x) = −12x2 + 8x; hence f20 (5) = −260, and the
1
slope of the normal at the point (x, y) = (5, f2 (5)) is 260 . The equation of the
1
normal is y − (−396) = 260 (x − 5).
(c) f3 (x) = (1 − x)2 .
Solution: f3 is a polynomial, and is both defined and differentiable at all real
numbers. f30 (x) = −2(1 − x).
Information for Students in MATH 140 2008 09 3014


(d) f4 (x) = − 10x − x2 ; tangent at (1, −3).
Solution: The domain of f4 consists of all real numbers x such that 10x−x2 ≥
0, i.e. where the product x(x−10) is non-positive: that is, all x in the interval
0 ≤ x ≤ 10. The derivative will be defined at every point in the domain except
at the end-points of the interval, at one of which the tangent approaches the
1
vertical; f40 (x) = − 12 (10x − x2 )− 2 (10 − 2x) = √10x−x
x−5
2 . At the point x = 1,

f 0 = − 12 · 13 ·8 = − 34 . The equation of the tangent there is y−(−3) = − 43 (x−1),


i.e. 4x + 3y + 5 = 0.
3
(e) f5 (x) = |3 − 2x|; tangent at x = .
2
Solution: The function is defined for all real x. As
½
3 − 2x 3 − 2x ≥ 0
f5 (x) = ,
2x − 3 3 − 2x < 0
½
−2 x < 32
f5 (x) = .
2 x > 32
The derivative does not exist at the point x = 23 , since the difference quotient
approaches +2 on one side of this point, and −2 on the other, so there is no
2-sided limit to the difference quotient.
 3 
 x 
when x 6= 0
(f) f6 (x) = |x| ; tangent and normal at (0, 0).
 
0 when x = 0
Solution: The function has all real numbers in its domain; the only point that
might have caused difficulty would have been 0, as the given quotient is not
defined there; however, the value at x = 0 is given separately. Differentiating
f within each of the defining half-lines gives
½
0 2x when x > 0
f6 (x) =
−2x when x < 0
To determine the value of the derivative at x = 0 we have to appeal to first
h3 h3
−0 −0 h2 −0
principles. f60 (0) = limh→0 |h|
h
. Since lim+ |h|
h
= lim+ h
= lim+ h = 0,
h→0 h→0 h→0
and the limit from the left is also equal to 0, we can assert that the 2-sided
limit exists, so f60 (0) = 0. The line through the origin with slope 0 is the
x-axis, which is tangent to the curve at that point.
½ 2
x − 6x when x < 1
(g) f7 (x) = ; tangent at (1, −5).
−1 − 4x when x ≥ 1
Solution: f7 is defined for all x. The function was created by piecing together
a branch of a parabola for x < 1, and a ray of a straight line for x ≥ 1.
Information for Students in MATH 140 2008 09 3015

Strictly within each of the constituent half-lines the function is obviously


differentiable: for x < 1, f70 (x) = 2x − 6; while, for x > 1, f70 (x) = −4; note
that we were unable to make any assertion about x = 1. Indeed, at x = 1
the limit from the right of the difference quotient is again seen to be −4. The
limit from the left is the value 2(1) − 6 = −4, which can be seen from first
principles. As here again the limits from left and right are equal, the limit
exists, and the function is differentiable at x = 1. The line through (1, −5)
with slope 4 has equation y + 5 = 4(x − 1), or 4x − y = 9.
 2
 x − 6x when x < 1
(h) f8 (x) = 0 when x = 1 ; normal at (1, −5).

−1 − 4x when x > 1
Solution: f8 is similar to f7 , except for the definition of the function at x = 1.
This time, when we attempt to determine the derivative at x = 1 by first
principles, we find that the quotient f (1+h)−f
h
(1)
= f (1+h)
h
has infinite limits —
i.e. does not have a (real) limit from the two sides. As the limit does not
exist, the function is not differentiable at x = 1, and there can be no normal.

3. (The functions referred to below were defined in Problem 2 above.)


(a) Determine the value of the function f7 ◦ f7 at any point x in its domain. [This
problem is technically difficult. Try to understand what makes the problem
difficult. Your solution to this problem will not be graded.]
Solution:


 (x2 − 6x)2 − (x2 − 6x) if x2 − 6x < 1 and x<1

(−1 − 4x)2 − (−1 − 4x) if x≥1 and −1 − 4x < 1
f7 ◦f7 (x) =

 −1 − 4(x2 − 6x) if x<1 and x2 − 6x ≥ 1

−1 − 4(−1 − 4x) if x≥1 and −1 − 4x ≥ 1
To study the four combinations of inequalities we observe that x2 − 6x =
(x − 3)2 − 9. The inequalities simplify to
 2 2 2


 (x − 6x) − (x − 6x) if 3 − 10 < x < 1

(−1 − 4x)2 − (−1 − 4x) if x ≥ 1√
f7 ◦ f7 (x) =

 −1 − 4(x2 − 6x) if x ≤ 3 − 10

−1 − 4(−1 − 4x) never.
i.e.
 2 √
 (x − 6x)(x2 − 6x − 1) if 3− 10 < x < 1
f7 ◦ f7 (x) = 2(8x2 + 6x + 1) if x ≥ 1√

−4x2 + 24x − 1 if x ≤ 3 − 10
Information for Students in MATH 140 2008 09 3016

(b) Determine the value of the functions f1 ◦ f4 and f4 ◦ f1 at all points in their
domains. Find the derivatives of these functions in two ways:
i. by differentiation of the formulæ you have determined
ii. by applying the chain rule
and show that the results are the same.
Solution:

− 10x − x2
(f1 ◦ f4 )(x) = √
5 − 8(− 10x − x2 )
1
= − 1
8 + 5(10x − x2 )− 2
s µ ¶ µ ¶2
x x
(f4 ◦ f1 )(x) = − 10 −
5 − 8x 5 − 8x
s
x(50 − 81x)
= −
(5 − 8x)2
Both methods of differentiation should lead to the same results:
5(x − 5)
(f1 ◦ f4 )0 (x) = √
5 + 8 10x − x2
205x − 125
(f4 ◦ f1 )0 (x) = p
(5 − 8x)2 x(50 − 81x)
Don’t panic! These computations are much to difficult for an examination!
The purpose was to test your perseverence, under conditions where two meth-
ods had to lead to the same answer, so you could verify your work.
1+s 1 √ dy
(c) Suppose that y = , s = t − , t = x. Determine the value of at
1−s t dx
x = 2.
Solution:
dy dy ds dt
= · ·
dx ds dt dx µ ¶
2 1 1
= 2
· 1− 2 · √
(1 − s) t 2 x

When x = 2, t = 2, s = √1 .
Hence
2
µ ¶ √
dy 2 1 1 3 2+4
=³ ´2 · 1 − · √ = .
dx 1 2 2 2 2
1 − √2
Information for Students in MATH 140 2008 09 3017

4. For each of the following functions, and for the given closed interval,
• find all critical points;
• find all local maxima and local minima;124
• find all global maxima and global minima, or explain why none of either exists.
All claims should be supported by careful reasoning; show all your work.
(a) g1 (x) = (x − 3)5 ; interval [2, 4]
Solution: g10 (x) = 5(x − 3)4 for 2 < x < 4. The derivative is thus defined
at all points in the interior of the interval; we cannot speak of a derivative
at the end points, since the behavior of the function must be known on both
sides of a point where a derivative is to be determined; (we could speak of
1-sided derivatives, but have not done so in this course). The critical points
will be those points where the derivative is zero, i.e. only the point x = 3. To
determine local and global extrema we must consider the value of the function
at the critical point and also at the end-points of the interval. Since g1 (3) = 0,
g1 (2) = −1, and g1 (4) = 1, we see that the global maximum, of value 1, occurs
at x = 4, and that the global minimum, of value −1, occurs at x = 2. The
point x = 3 is neither a local minimum nor a local maximum: for x < 3 the
function value is less than g1 (3), while, for x > 3, the function value is greater
than g1 (3).
· ¸
2 3 5
(b) g2 (x) = 6 − 36x + 15x − 2x ; interval ,5
2
Solution: The function g2 , being a polynomial,
£5 ¤ is defined at all points. As the
definition of g2 has been specified
¡ ¢ as 2 , 5 , the function will have a derivative
at all points in the interval 25 , 5 ; we have no information about g2 for x < 52 ,
nor for x > 5, so we do not have a derivative at either of those end-points.
Where it is defined, g20 (x) = −36 + 30x − 6x2 = −6(x − 2)(x − 3), which
vanishes at x = 2 and x = 3. But x = 2 is not in the domain of definition
of the function, so this point is irrelevant. We thus have just one critical
point, x = 3; there g2 (3) = 6 − 108 + 135 − 54 = −21. At the end-points,
g2 ( 52 ) = − 33
2
, g2 (5) = −49. As g2 ( 52 ) > g2 (3) > g2 (5), the global maximum is
5
at 2 , and the global minimum at 5.

 −1

when −1 ≤ x < 0
(c) Solution: g30 (x) = x2 . For h 6= 0, the value of
 1
 − when 0 < x ≤ 1
x2
124
THIS PROBLEM WAS NOT TO BE GRADED: It was announced in the lectures that this topic
would not be discussed until [31, Chapter 4]; accordingly this part of the question was to be omitted.
Information for Students in MATH 140 2008 09 3018

the difference quotient g3 (h)−0


h
is h12 , which has no finite limit as h → 0; hence
the function is not differentiable at x = 0. The derivative is not zero anywhere
in the domain; thus the only critical point is x = 0. Evaluating the function
at this critical point, and at the two end-points, we obtain g3 (−1) = −1,
g3 (0) = 0, g(1) = 1. We may not apply [33, Theorem 3, p. 144] here, because
this function is not continuous throughout the domain of definition. In fact,
it is discontinuous at x = 0. As x → 0− , g3 (x) → −∞; and, as x → 0+ ,
g3 (x) → ∞; these one-sided limits are not equal real numbers, so no limit
exists; for continuity they must be equal, and must equal the function value,
here 0. Because the function becomes positively and negatively infinite inside
the domain of definition, it can have neither a global maximum nor a global
minimum. There is no need to evaluate it at the critical point, nor at the
end-points of the interval of definition.
(d) Solution: This problem differs from the preceding problem in that the limit
as x → 0, while still not existing as a real number, is ∞. In this case there
can be no global maximum. The cited theorem still does not apply, as there is
still a discontinuity at x = 0. The function values at x = ±1 are both 1, and,
from examination of the behavior of the function near these points, might be
suspected of being global minima. However, the value at x = 0 is lower. The
global minimum is thus 0, assumed only at the point x = 0; everywhere else
the function value is strictly larger.
(e) Solution:
√ The domain of definition of g5 is the whole√real line. The derivative
of x for x > 0 is 2√1 x ; similarly, the derivative of −x for x < 0 is − 2√1−x .
Investigation of the difference quotient g5 (h)−0
h
as h → 0, shows that the limit
does not exist as a finite number; in fact is it ∞, and the function may be
said to have a vertical tangent. Thus x = 0 is a critical point, because the
function lacks a derivative there. The function is, however, continuous, so we
may determine the global extrema by studying its behavior at the end-points
and the critical point: g5 (−4) = 2, g5 (0) = 0, g5 (9) = 3. The global maximum
is thus at x = 9, and the global minimum is at x = 0.

5. [35, Examples XLVI.17] (Cambridge Math. Tripos 1930) The graph of the function
ax + b
h(x) = has (2, −1) as a critical point. Determine a and b, and show
(x − 1)(x − 4)
that the critical point is a local maximum. (Note: It is intended that this problem
be solved without using concepts from Chapter 4 of your textbook (involving higher
derivatives.))
2
Solution: For x different from 1 and 4, h0 (x) = a(x −5x+4)−(ax+b)(2x−5)
(x−1)2 (x−4)2
. As the
derivative exists at x = 2, it can be a critical point only because the derivative is
Information for Students in MATH 140 2008 09 3019

zero; this implies that a(22 −5·2+4)−(a·2+b)(2·2−5) = 0, i.e. b = 0. We are also


2a+0 x
told that h(2) = −1, hence (2−1)(2−4) = −1, so a = 1. Thus h(x) = (x−1)(x−4) . As
(x−2) 2
h(2) − h(x) = − (x−1)(x−4) , this difference is non-negative in an interval surrounding
x = 2 (since the numerator is non-negative, and the denominator is negative). Thus
h(x) ≤ h(2), so 2 is a local maximum.

6. Determine values of the constants a, b, c which will cause the curve y = ax3 + bx2 +
cx + d to pass through the points (2, 6) and (−1, 6) and to be tangent to the line
y = 3x + 1 at the point (1, 4).
Solution: We are given four kinds of information: three points through which the
curve passes, and the equation of the tangent at one of these points. Imposing the
condition that the curve pass through the points (2, 6), (−1, 6), and (1, 4) yields
three linear equations:

8a + 4b + 2c + d = 6
−a + b − c + d = 6
a+b+c+d = 4

Finally, as the tangent at x = 1 has slope 3, we know that 3ax2 + 2bx + c|x=1 = 3,
i.e. 3a+2b+c = 3. Solving the 4 linear equations simultaneously yields (a, b, c, d) =
(−1, 3, 0, 2).

7. Find all lines with slope −3 which are normal to the curve 64y = x3 .
1 3 3 2
Solution: At the point (x, 64 x ) on the curve the slope of the tangent is 64 x , so
64
the slope of the normal is − 3x2 . Imposing ¡ the condition
¢ that this equal −3, we
obtain that x = ± 83 . Through
¡ ¢ the point ± 8
3
, ± 8
27
the equation of the line with
8 8 8·28
slope 3 is y ∓ 27 = −3 x ∓ 3 , i.e. 3x + y = ± 27 .

8. Find the volume of the uncovered box of greatest volume that can be made by
cutting equal squares out of the corners of a piece of sheet metal which is 21 cm.
× 5 cm., and turning up the sides.
Solution: If the side of the square cut from each corner is of lengh x, where 0 ≤ x ≤
5
2
, then the volume obtained after the sides are folded up is x · (21 − 2x)(5 − 2x) =
4x3 − 52x2 + 105x. Setting the derivative, 12x2 − 104x + 105 (i.e. (2x − 15)(6x − 7))
equal to zero, we find that x = 15 2
or x = 67 . The second of these is the only critical
point; the first is not in the interval of definition of the function. To determine the
maximum we compare the value of the volume at x = 76 with the volume at the
end-points, both of which give volume 0. At x = 67 the volume is 76 · 56 3
· 83 = 1568
27
3
cm. , which exceeds the value of 0 at the end-points. This is the largest volume.
Information for Students in MATH 140 2008 09 3020

9. Show that, among all right-angled triangles whose hypotenuse is 10 units long, the
triangle whose area is maximum is isosceles.
Solution: Denote the lengths of the non-hypotenuse sides by a and b. As √ these are
2 2 2 2
constrained by the equation
√ x + b = 10 = 100, we know that b = 100 − a .
1
The area is therefore 2 a 100 − a2 . We may take the domain to be 0 ≤ a ≤ 10.
2
The derivative of the area function is, after reduction, √50−a 2 , which is zero when
√ 100−a
a = ±5 2. Of these two values, only the positive one is in the domain of definition
of the function. At the end-points of the domain the function is zero; while, at
the critical point we have found, the function value is positive; this, then, is the
maximum point. For this value of a, b = a, which was to be proved.

6.2.4 Fourth Fall 1999 Problem Assignment


dy
1. Find if
dx
¡ ¢
(a) y = ln 1 + x2
2
(b) y = e−x

x
(c) y = x

2. Find an equation for the tangent line at x=0 to the graph of y = ex − e−x .
1
3. Find the greatest value of f (x) = ln x .
x2
4. If f (x) = e−x sin x , find the values of x where

(a) f 0 (x) = 0 ;
(b) f 00 (x) = 0 .
dy sin−1 2x
5. (a) Find if y= .
dx sin−1 x
µ ¶
−1 x+1 dz
(b) Show that if z = tan + tan−1 x , then =0.
x−1 dx
6. A picture a metres high is placed on a wall with its base b metres above
an observer’s eye. If the observer stands x metres away from the wall, find

(a) the angle α subtended by the picture at the observer’s eye; and
(b) the distance x which will give the maximum value for α.
Information for Students in MATH 140 2008 09 3021

7. Use the tangent line approximation to find an approximate value for

(a) ln(0.94) ;
(b) cos−1 (0.47) .

8. Find an equation for the line tangent to the curve x4 + x2 y 2 + y 4 = 21 at the


point (1, 2) .
d2 y
9. If x3 + y 3 + 6xy = −5 , find at the point (−1, 2) .
dx2
ln x
10. Evaluate lim . [Hint: Write down the limit definition of f 0 (1) , where
x→1 x − 1
f (x) = ln x .]

6.2.5 Fifth Fall 1999 Problem Assignment


1. For each of the functions,

(x2 − 1)2 1 − sin x


f (x) = g(x) =
4x2 √ cos x
−x2
h(x) = xe k(x) = ln x

determine all of the information requested below, and sketch a graph of the func-
tion. Show all your work.

(a) For any discontinuities, determine whether they are removable.


(b) Where is the function increasing? Where is it decreasing?
(c) Where is the graph concave upwards? Where is it concave downwards?
(d) Where, if any, are the local extrema? In each case you should indicate whether
the extremum is a local maximum or a local minimum, and provide justifica-
tion for your choice.
(e) Where, if any, are the global extrema? Again, justification is expected.
(f) Where, if any, are the intercepts (with the coordinate axes)?
(g) Where, if any, are the inflection points?
(h) Is the graph symmetric about any vertical line x = a? (The graph of F is
symmetric about x = a if the equation remains unchanged under the trans-
formation x − a → a − x; i.e. if F (x) = F (2a − x). In particular, the graph
is symmetric about the y-axis if F (x) = F (−x). Such a function is said to be
even.)
Information for Students in MATH 140 2008 09 3022

(i) Is the graph symmetric under rotation about the origin? (The graph of F is
symmetric under rotation about the origin if the equation remains unchanged
under the transformation (x, y) → (−x, −y); i.e. if F (−x) = −F (x). Such a
function is said to be odd .)
(j) Determine all horizontal and all vertical asymptotes to the graph.
2. (a) Show that at least one of the hypotheses of the Mean Value Theorem fails to
hold for the following functions; show also that the conclusion of the theorem
fails to hold for each function, where the interval is [a, b] = [−1, 1].
½ 1
when x 6= 0
i. f1 (x) = x .
0 when x = 0
2
ii. f2 (x) = x 3 .
(b) [31, Problem 4.3.48] [33, Problem 4.3.48] Show that the function f2 (x) de-
fined in Problem 2(a)ii above does satisfy the conclusion of the Mean Value
Theorem on the interval [−1, 27].
(c) Using the Mean Value Theorem, show carefully that the equation x7 + x − 12
has exactly one real solution. Then show — without using a calculator —
that this solution lies between x = 1 and x = 2.
3. Consider the function f3 (x) = x cos x.
(a) Determine the intercepts of the graph of f3 on the coordinate axes.
(b) Show that the critical points of f3 occur at points of intersection of y = tan x
1
with the curve y = .
x
1
(c) Show that, wherever it is defined, the function tan x − is increasing. Use
x
this
µ fact to show that there
¶ is exactly one critical point of f3 in each interval
(2n − 1)π (2n + 1)π
, , where n is any integer. [Hint: Use a Corollary to
2 2
the Mean Value Theorem.]
(d) Sketch the graph of f3 .
1
4. Use the Second Derivative Test in finding all points on the curve y = 2 which
2x
are closest to the origin.
125
5. In each of the following cases, evaluate the limit, or show that it does not exist.
125
This problem and the next were originally included in Assignment 2, but were subsequently omitted,
because they are based upon [31, §§4.5,4.7], which material had not been discussed in the lectures before
the due date of Assignment 2.
Information for Students in MATH 140 2008 09 3023


3x + 2 x
(a) lim
x→∞ 1−x
|2x − 3|
(b) lim
x→−∞ x + 2

6. Find a value for a so that


µ ¶
x2 + 1
lim 3x + 1 − a
x→∞ x+1

exists as a finite limit, and evaluate that limit.

6.3 2000/2001 Problem Assignments, with Solutions


6.3.1 First 2000/2001 Problem Assignment, with Solutions
Instructions

• The basic principle you should follow in any mathematics course is that every statement
should be justified. While it may be that in some cases you are not able to provide a
flawless logical argument, that should always be your goal. Getting the right answer is
always desirable, but will usually not be enough.

• Notwithstanding the preceding comment, you should not spend disproportionately long
amounts of time on any one question. Solutions will be posted on the Web.

• While the textbook in [17, 1.1] describes four different ways in which to represent a func-
tion, we usually regard the representation by an explicit formula as the most desirable.
In particular, graphical representation should be used only to assist you in visualizing a
solution, not as the final solution; it will normally not be acceptable to explain a step in
a proof by reference to a graph — but it is a good policy to make a quick sketch of the
graph of any function you have to work with.

• You should always attempt, in an algebraic representation, to make your functions as


“simple” as possible; there may be different “simple” ways of representing the function.
Use your good judgment: the simplification is intended to help you!

• Do not approximate numbers unless you are asked to do so. Thus, π should not be
replaced by 3.1415926...; of course, you should know the values of the standard trigono-
metric functions at familiar multiples and submultiples of π, and use this information√to
simplify your answers, where applicable. For example, you should replace sin π3 by 23 ,
but not by 0.8660....
It is usually preferable to leave fractions in the form m
n , rather than rewriting as decimal
fractions. This preference applies even when the decimal expansion is finite, as in, for
Information for Students in MATH 140 2008 09 3024

example, 34 = 0.75. One reason for this preference is that decimal fractions are often
interpreted as approximations, rather than as the exact value; so by writing 0.75 instead
of 34 you could be obscuring the fact that your datum is exact.

• You may assume that, over an interval [c, d], the maximum and minimum values of a
linear function — that is a function of the form f (x) = ax+b, where a and b are constants
— are attained at the end points of the interval.

1. Let a function f be defined by f (x) = |2x| + |2x + 3|.

(a) Find algebraic formulæ which give the value of f (x) without using the absolute
value function. For this purpose it will be necessary to break the domain up
into several parts, as the formula will be different in different subintervals.
(b) Showing all your work, determine the domain and the range126 of f .
(c) Determine whether or not f has an inverse function. If it does, determine a
formula for f −1 (x).

Solution:

(a)
½
−2x when x<0
Since |2x| =
2x when x≥0
½
−2x − 3 when 2x + 3 < 0
and |2x + 3| =
2x + 3 when 2x + 3 ≥ 0
(
−2x − 3 when x < − 23
or, equivalently |2x + 3| = ,
2x + 3 when x ≥ − 23

we may add the inequalities to obtain



 −2x − 2x − 3 when x < − 32
|2x| + |2x + 3| = −2x + 2x + 3 when − 32 ≤ x < 0

2x + 2x + 3 when x ≥ 0

 −4x − 3 when x < − 23
= 3 when − 32 ≤ x < 0

4x + 3 when x ≥ 0
126
Many mathematicians prefer to use the term image instead of range.
Information for Students in MATH 140 2008 09 3025

(b) While the description of the function changes as one moves from one interval
to the next, the function is defined for all x — i.e. the domain is the whole
real line.
For the interval −∞ < x ≤ − 32 , the function value ranges between +∞ and
3; it remains at 3 through the middle interval; and, for − 32 < x < ∞, again
takes on all values which are ≥ 3. Thus the range is [3, ∞).
[How could one prove, rigorously, that the stated intervals do indeed constitute the
range of the function? For example, when x < − 23 , we reverse the inequality when
we multiply by −4, to obtain −4x > 6; adding −3 to both sides of the inequality
yields −4x − 3 > 3, showing that the portion of the range of the function for this
part of the domain is contained in the interval (3, ∞). And, if y is any real number
such that 3 < y, then 6 < y + 3; so, multiplying both sides of the inequality by − 14 ,
y+3 3
and
³ thereby
´ reversing
³ ´the inequality, we obtain − 4 < − 2 . But this tells us that
f − y+34 = −4 y+3 4 − 3 = y. We have thus shown that every number y in the
interval (3, ∞) is in the range: thus the range of the function corresponding to this
portion of the domain is precisely the interval (3, ∞).]
(c) This function is not invertible, since there are values which are attained at
more than one point in the domain. For example, the function takes on the
same value for all x such that − 23 ≤ x ≤ 0.
2. Let a function g be defined by

g(x) = 2x − 5 . (212)
Showing your work, answer each of the following questions for g.
(a) Determine the domain.
(b) Determine the range.
(c) Determine whether the function has an inverse function. If it does, find a
formula for g −1 (x), and determine the domain and range.
Solution:
(a) g is the composition of two functions. In the first phase x is mapped on
to 2x − 5. The second function applied is the square root. The mapping
x 7→ 2x − 5 is defined for all x; that is, its domain is the whole real line.
However, the mapping to the square root is not defined for negative numbers.
Thus we cannot proceed unless 2x − 5 ≥ 0, i.e. unless
5
x≥ . (213)
2
Inequality (213) gives the domain of g.
Information for Students in MATH 140 2008 09 3026

(b) Any non-negativeµ 2 real¶ number r is attained as a value of the function g;


r +5
specifically, g = r. Thus the range of g is the interval [0, ∞).
2
(c) We may square both sides127 of (212) to obtain (g(x))2 = 2x − 5, so x =
(g(x))2 + 5
, showing that x is determined as soon as g(x) is known. Thus g
2
x2 + 5 128
is invertible, and its inverse is given by x 7→ .
2
3. A curve is symmetric about the y-axis if the presence of any point (x0 , y0 ) on the
curve implies the presence of (−x0 , y0 ); a curve is symmetric about the x-axis if
the presence of any point (x0 , y0 ) on the curve implies the presence of (x0 , −y0 );
a curve is symmetric about the origin if the presence of any point (x0 , y0 ) on the
curve implies the presence of (−x0 , −y0 ). A function F is odd if its graph is
symmetric about the origin, i.e. for any x in the domain, F (−x) = −F (x) (which
is equivalent to saying that if (x, F (x)) is on the graph, then so is (−x, −F (x))).
F is even if its graph is symmetric about the y-axis; i.e., for any x in the domain,
F (−x) = F (x) (which is equivalent to saying that if (x, F (x)) is on the graph, then
so is (−x, F (x))).
For each of the following functions F , determine whether

• F is odd.
• F is even.
• the graph of F is symmetric about the x-axis.
• the graph of F is symmetric about the y-axis.

(a) F (x) = | sin x|


(b) F (x) = cos x
³ π´
(c) F (x) = cos x −
2
(d) F (x) = 0
1
(e) F (x) = x3 +
x
127
√ √
More precisely, we can multiply both sides of the equation g(x) − 2x − 5 = 0 by g(x) + 2x − 5.
128
Note that, while this formula is meaningful even when x < 0, we are specifically restricting the
domain of this inverse function to be the range of the function g. With that proviso we find that the
pair of functions g and g −1 have the property that the domain ¡ of each
¢ is the range of the other, and
vice-versa. Thus the domain of g −1 is [0, ∞), and the range is 52 , ∞
Information for Students in MATH 140 2008 09 3027

1
(f) F (x) = x3 −
x
2
(g) F (x) = G(x ), where G is any function.

Solution:
Before beginning our solution, we make the following observations:

• The graph of F is symmetric about the y-axis ⇔ F is even.


• If the graph of F is symmetric about the x-axis, then, for any x0 in the
domain, both points (x0 , F (x0 )) and (x0 , −F (x0 )) will lie on the graph. But
the graph of a function can never meet two distinct points on the same vertical
line x = x0 . It follows that these points cannot be distinct, i.e. that F (x0 ) =
−F (x0 ), i.e. that F (x0 ) = 0 for all points x0 in the domain. Thus only the
0 function, or restrictions of this function to subsets of the real line can have
this symmetry property.
• The only function which is both even and odd is the 0 function (or restrictions
of this function to subsets of the real line). For both conditions F (−x) = F (x)
and F (−x) = −F (x) must hold for all x, implying that F (x) = 0 for all x.

(a) | sin(−x)| = |−sin x| = | sin x|, so this function is even; (hence, it not being the
0 function, it is not odd). By the preceding comments, its graph is symmetric
about the y-axis, but not about the x-axis.
(b) cos(−x)
¡ π ¢= cos x for all x, so this function is even. It is not odd; for example,
cos − 4 = 2 6= − √12 = − cos π4 .
√1

¡ ¢
(c) By a well-known property of the cosine function, cos x − π2 = cos ¡ x cos ¢π2 +
sin x sin π2 = (cos x) · 0 + (sin
¡ x) π· ¢1 = sin x for all x. Hence cos −x − 2 =
π

sin(−x) = − sin x = − cos x − 2 , so this function is odd. As the function


is not identically 0, it is not even; its graph is not symmetric about either
coordinate axis.
(d) The 0 function is both even and odd, and its graph is symmetric about both
coordinate axes.
µ ¶
3 1 3 1
(e) (−x) + =− x + , so this function is odd. It is not even, and its
−x x
graph is symmetric about neither coordinate axis.
1
(f) The function x3 − has the same symmetry properties as the preceding
x
function.
Information for Students in MATH 140 2008 09 3028

(g) G((−x)2 ) = G(x2 ), so this function is even, and its graph is symmetric about
the y-axis. The function is not odd, and the graph is not symmetric about
the x-axis.

4. Let
x2 − 4
f (x) =
x−1
1
g(x) =
f (x − 1)
³ π´
h(x) = sin x −
4
1
k(x) = ³ π´
h x−
4
(a) Determine an explicit formula for (f ◦ g)(x).
(b) Describe the domains of the functions f , g, f ◦ g, h and k.
(c) [BONUS QUESTION] Describe the domain of g ◦ f .

Wherever possible in this problem, algebraic expressions should be simplified.


Solution:

(b) f . Since f is defined as a ratio of polynomials, its domain is the set if all real
numbers where the denominator is not zero; as the denominator is x − 1,
the domain of f is all real numbers except 1.
g.
1 1 1 x−2
g(x) = = 2 = 2 = 2
f (x − 1) (x − 1) − 4 x − 2x − 3 x − 2x − 3
(x − 1) − 1 x−2
The preceding statement is valid wherever g is defined. That will be the
intersection of the set of x such that f (x − 1) is defined, and the set
where the fraction 1f (x − 1) is defined; i.e., it will be the set of x where
f (x − 1) is defined, from which we must remove all points x such that
f (x − 1) = 0. Since f (x) is defined for x 6= 1, f (x − 1) is defined for all
x such that x − 1 6= 1, i.e. such that x 6= 2. And f (x − 1) = 0 precisely
when (x − 1)2 − 4 = 0, equivalently when x − 1 = ±2, i.e. when x = −1
or x = 3. Therefore the domain of g is the set R − {−1, 2, 3}.
f ◦ g. The first function to be applied is g. Its domain consists of R −
{−1, 2, 3}. But the application of f requires, further, that we exclude
Information for Students in MATH 140 2008 09 3029

points x where g(x) = 1, since 1 is not in the domain√ of f . Thus we


3 ± 13
exclude points x such that x2 − 3x − 1 = 0, i.e. . The domain is,
2
therefore, ( √ √ )
3 + 13 3 − 13
R − −1, 2, 3, ,
2 2
(We could also have determined this by excluding from R the roots of the
denominator of the ratio of polynomials that we have determined above
for f ◦ g.)
h. h(x) is defined for all real numbers x.
¡ ¢
g. k(x) is not defined at points where sin x − π2 is zero, i.e. where x − π2 is
an integer multiple of π. Thus the points that have to be excluded from
2n + 1
R are all numbers of the form π, where n is any integer.
2
(a)
µ ¶2
x−2
−4
x2 − 2x − 3
(f ◦ g)(x) =
x−2
2
−1
x − 2x − 3
(x − 2)2 − 4(x2 − 2x − 3) 1
= 2 2
· 2
(x − 2) − (x − 2x − 3) x − 2x − 3
4 3 2
4x − 16x − 9x + 52x + 32
=
(x2 − 3x − 1)(x2 − 2x − 3)

(c) We can also express (g ◦ f )(x) as a ratio of polynomials in x:

f (x) − 2
(g ◦ f )(x) =
f (x)2
− 2f (x) − 3
x2 − 4
−2
= µ 2 x − 1
¶2
x −4 x2 − 4
−2 −3
x−1 x−1
((x2 − 4) − 2(x − 1)) (x − 1)
=
(x2 − 4)2 − 2(x2 − 4)(x − 1) − 3(x − 1)2
(x2 − 2x − 2)(x − 1)
= 4
x − 2x3 − 9x2 + 14x + 5
Information for Students in MATH 140 2008 09 3030

This ratio is not defined at points where the denominator is zero. However,
that denominator is a quartic function of x, and its roots are not obvious.129
Instead, let us attack this problem the same way we dealt with f ◦ g above.
First we have to exclude the points where f is not defined, i.e., the point x = 1.
Next we have to exclude points x such that f (x) is not in the domain of g:
x2 − 4 x2 − 4
these are the solutions to each of the equations = 3, = 2, and
x−1 x−1
x2 − 4
= −1, i.e., the roots of the three quadratic polynomials, x2 − 3x − 1,
x−1
x2 − 2x − 2, and x2 + x − 5. It follows that the domain of g ◦ f is
( √ √ √ √ )
3 + 13 3 − 13 −1 + 21 −1 − 21 √ √
R − 1, , , , , 1 + 3, 1 − 3
2 2 2 2

5. (a) Showing all your work, and without using tables, computers, slide rules, or a
calculator , determine the value of log16 2.
(b) Showing all your work, determine all values of x for which
µ ¶x
x 1
2 = . (214)
2

(c) Showing all your work, determine all values of x for which log2 (ln x) = 1.
(d) Showing all your work, simplify cos(x + y) cos(x − y) − cos2 x − cos2 y, where
x and y are any real numbers.
Solution:
(a) By [17, p. 70]
ln 2 1 1 1 1
log16 2 = = = = =
ln 16 ln 16 log2 16 log2 24 4
ln 2
(b) By the Laws of Exponents [17, p. 58]
µ ¶x µ ¶x
x 1 x 1
1=1 = 2· =2 · ,
2 2
hence µ ¶x
1 1 20
= x = x = 2−x .
2 2 2
129
Although, in fact, this quartic polynomial does factorize into the product of two quadratic polyno-
mials, x2 − 3x − 1 and x2 + x − 5, whose roots can be found in the usual way.
Information for Students in MATH 140 2008 09 3031

Thus (214)⇔ 2x = 2−x . Applying the inverse function log2 to both sides
of this equation yields the equivalent statement x = −x, which is, in turn,
equivalent to x = 0.
(c) Applying the exponential function (to base 2) to both sides of the hypothesis
log2 (ln x) = 1 yields ln x = 21 = 2. Now apply the exponential function, this
time to base e, to obtain the equivalent statement x = e2 .
6. (a) [17, Exercise 14, p. A34] A circle has radius 10 cm. Showing all your work,
determine the length of the arc subtended by a central angle of 72◦ .
(b) [17, Exercise 34, p. A35] Showing all your work, find the remaining 5 trigono-
4 3π
metric ratios for the angle θ, if it is known that csc θ = − , and < θ < 2π.
3 2
(c) (Adapted from [17, Exercise 70, p. A35]) Find all values of x that satisfy the
equation cos x + sin 2x = 0.
(d) Determine the value of cos(x + y) cos(x − y) − cos2 x − cos2 y, as x and y range
over the real numbers.
Solution:
72
(a) The entire circle has circumference 2π · 10 cm. The angle of 72◦ is 360 = 15
of the central angle subtending the entire circle. Hence the arc has length
1
5
× 20π = 4π cm.
1 3
(b) First observe that sin θ = =− .
csc θ 4
Now, since θ is in the 4th quadrant, the cosine is positive. This resolves the
2 2
sign choice when
r we solve for
√ cos θ in the equation sin θ + cos θ = 1, so we
9 7
have cos θ = + 1 − = . The remaining ratios can now be computed
16 4
without ambiguity:
−3
sin θ 4 3
tan θ = = √ = −√
cos θ 7 7
4
√ √
7 −4 7
cot θ = cos θ · csc θ = · =−
4 3 3
1 4
sec θ = =√
cos θ 7
(c) (There are many ways of approaching problems of this type, so the following
is only one possibility. Of course, all methods lead to the same answers.)
cos x + sin 2x = 0 ⇔ cos x + 2 sin x cos x = 0 (double-angle formula)
Information for Students in MATH 140 2008 09 3032

⇔ cos x(1 + 2 sin x) = 0


1
⇔ cos x = 0 or sin x = −
2
The solution set will, therefore, be the union of the solution sets for the two
equations shown.
π
cos x = 0. The solutions to this equation are all x of the form x = + nπ,
2
where n is any integer.
sin x = − 21 . One solution is x = − π6 ; another solution is x = − 5π 6
; remember
that the graph of the sine function is symmetric about the line x = 3π 2
(as it is about all odd integer multiples of π2 ): any value which is realized
between 3π 2
and 2π will also be attained at a mirror-image point between

π and 2 . These two solutions each give rise to an infinite set of solutions,
2π units¡ apart. The
¢ full
¡ 5set of solutions
¢ is all real numbers of either of the
1
forms
¡ 11 − + 2m
¢6 ¡ 7 π, −
¢ 6 + 2m π or, equivalently of either of the forms
6
+ 2m π, 6 + 2m π where m is any integer.
The set of solutions to the original equation is, therefore, the union of the two
lists above:
½µ ¶ µ ¶ µ ¶ ¯ ¾
1 7 11 ¯
+ m π, + 2m π, ¯
+ 2m π ¯ m any integer
2 6 6

(d) (There are several possible approaches to this problem. The following may
not be the simplest.)
cos(x + y) cos(x − y) − cos2 x − cos2 y
= (cos x cos y − sin x sin y)(cos x cos y + sin x sin y) − cos2 x − cos2 y
= cos2 x cos2 y − sin2 x sin2 y − cos2 x − cos2 y
= cos2 x cos2 y − (1 − cos2 x)(1 − cos2 y) − cos2 x − cos2 y
= cos2 x cos2 y − (1 − cos2 x − cos2 y + cos2 x cos2 y) − cos2 x − cos2 y = −1
for all x and for all y.
7. (a) [17, Exercise 8, p. A-23] Show that the following equation represents a circle,
and determine its centre and radius:
16x2 + 16y 2 + 8x + 32y + 1 = 0

(b) [17,
¡ 1 Exercise
¢ 36, p. A-15] Find an equation of the line through the point
2
2
, − 3
which is perpendicular to the line 4x − 8y = 1.
Solution:
Information for Students in MATH 140 2008 09 3033

(a) We first group the terms in x and y separately, as quadratic functions. Then
we scale to arrange for the coefficients of x2 and of y 2 to be 1, and complete
the square in each of these:

16x2 + 16y 2 + 8x + 32y + 1 = 0


⇔ (16x2 + 8x) + (16y 2 + 32y) = −1
µ ¶
1 1
⇔ x + x + (y 2 + 2y) = −
2
2 16
à µ ¶2 ! à µ ¶2 !
1 1 1 1
⇔ x2 + x + · + y 2 + 2y + ·2
2 2 2 2
µ ¶2 µ ¶2
1 1 1 1
=− + · + ·2
16 2 2 2
µ ¶2
1 1 1
⇔ x+ + (y + 1)2 = − + +1=1
4 16 16
¡ ¢
which is the equation of the circle with centre − 41 , −1 and radius 1.
(b) The slope of the line 4x − 8y = 1 is 12 ; so the line we seek, being perpendicular
to that line, must have slope − 11 = −2. The line through the given point,
2
with slope −2, has equation
µ ¶ µ ¶
2 1
y− − = −2 x −
3 2

which simplifies to 2x + y = 31 .

8. (a) [17, Exercise 84(a), p. A36] Showing all your work, determine the exact value
of arctan(−1).
(b) [17, Exercise 86(b), p. A36] Showing all your work, determine the exact value
of arcsin 1.
(c) [17,¡Exercise 92, p. A36]
¢ Showing all your work, determine the exact value of
sin arcsin 13 + sin−1 32 .
(d) (Adapted from [17, Exercise 96, p. A36]) Simplify sin (−2 cos−1 x).
Solution:

(a) The inverse tangent


¡ π πfunction
¢ inverts the restriction of the tangent function to
the interval − 2 , + 2 . In that interval the only x such that tan x = −1 is
− π4 . Hence arctan −1 = − π4 .
Information for Students in MATH 140 2008 09 3034

(b) The inverse


£ π sineπ ¤function inverts the restriction of the sine function to the
π
interval − 2 , + 2 . In that interval the only x such that sin x = 1 is 2 . Hence
arcsin 1 = π2 .
(c) Let’s apply the formula for the sine of a sum:
µ ¶
1 −1 2
sin arcsin + sin
3 3
µ ¶ µ ¶ µ ¶ µ ¶
1 −1 2 1 −1 2
= sin arcsin · cos sin + cos arcsin · sin sin
3 3 3 3
µ ¶ µ ¶
1 2 1 2
= · cos sin−1 + cos arcsin ·
3 3 3 3

Now sin−1 23 is in the first quadrant, so its cosine is positive, and equal to
q √
+ 1 − 49 = 35 ; arcsin 13 is also in the first quadrant, so its cosine is also

2 2
positive, and is equal to 3
. Substituting these values gives
µ ¶ µ ¶ √ √
1 −1 2 1 2 5+4 2
· cos sin + cos arcsin · = .
3 3 3 3 9

(d)
¡ ¢ ¡ ¢
sin −2 cos−1 x = − sin 2 cos−1 x
¡ ¢ ¡ ¢
= −2 sin cos−1 x · cos cos−1 x

We know that, for all x, cos cos−1£x =¤x. Also, since the inverse cosine function
takes its values in the interval 0, π2 , and the sine function is non-negative

throughout that interval, we also know that sin cos−1 x = + 1 − x2 . It follows
that ¡ ¢ √
sin −2 cos−1 x = −2x 1 − x2 .

6.3.2 Second 2000/2001 Problem Assignment, with Solutions


In all of these problems you were expected to justify every statement you made, and to
show all your work.

1. For the curve y = 2x3 , find the slope MP Q of the secant line through the points
P = (1, 2) and Q = (2, 16), i.e. the points with x = 1 and x = 2.
2·23 −2·13
Solution: The slope of the line joining P and Q is 2−1
= 14.
Information for Students in MATH 140 2008 09 3035

2. Given 
 x3 + 2 if x ≤ −1
2
f (x) = x + x + 1 if −1 < x < 1

x4 + 2 if x≥1
find the following limits, if they exist; or explain why the limit does not exist.
Justify your answers.

(a) lim f (x)


x→−1+

(b) lim f (x)


x→−1−

(c) lim f (x)


x→−1

(d) lim+ f (x)


x→1

(e) lim− f (x)


x→1

(f) lim f (x)


x→1

Solution:

(a) For x to the right of −1 (but to the left of +1) f (x) = x2 + x + 1; hence
lim + f (x) = (−1)2 + (−1) + 1 = 1.
x→−1

(b) For x to the left of −1, f (x) = x3 + 2; hence lim − f (x) = −13 + 2 = 1
x→−1

(c) Since the one-sided limits from both left and right exist at −1 and have the
same value, lim f (x) exists, and its value is the common value of the one-
x→−1
sided limits, i.e. 1.
(d) For x to the right of 1, f (x) = x4 + 2; hence lim+ f (x) = 14 + 2 = 3.
x→1

(e) For x to the left of 1 (but to the right of −1) f (x) = x2 + x + 1; hence
lim− f (x) = 12 + 1 + 1 = 3.
x→1

(f) Since the one-sided limits from both left and right exist at 1 and have the
same value, lim f (x) exists, and its value is the common value of the one-
x→1
sided limits, i.e. 3.

3. Given that lim f (x) = 5, lim g(x) = 0, and lim h(x) = −8, find the following
x→3 x→3 x→3
limits, if they exist. If a limit does not exist, explain why.

(a) lim (f (x) + h(x))


x→3
Information for Students in MATH 140 2008 09 3036

(b) lim (x2 f (x))


x→3

(c) lim (f (x))2


x→3
f (x)
(d) lim
x→3 2h(x)
g(x)
(e) lim
x→3 (x)
f

f (x)
(f) lim
x→3 g(x)
2h(x)
(g) lim f (x)−h(x)
x→3
p
(h) lim 3
h(x)
x→3

Solution:

(a) lim (f (x) + h(x)) = lim f (x) + lim h(x) = 5 + (−8) = −3. (We have used the
x→3 x→3 x→3
Sum Law.)
(b) Since we know that the limit of x as x → 3 is 3, we can use the Product Law:
³ ´2
2
lim (x f (x)) = lim x · lim f (x) = 32 · 5 = 45.
x→3 x→3 x→3

(c) Again by the Product Law, the limit of a square is the square of the limit:
³ ´2
lim (f (x)) = lim f (x) = 52 = 25.
2
x→3 x→3

(d) First we use the Constant Multiple Law to determine the limit of the de-
nominator. Then we use the Quotient Law, since the limit of the denom-
inator exists, and is not zero, and the limit of the numerator also exists:
f (x) 5 5
lim = =− .
x→3 2h(x) 2(−8) 16
g(x) 0
(e) Again by the Quotient Law, lim = = 0.
x→3 f (x) 5

(f) But, in this case, the limit of the denominator is 0. Since the limit of the
numerator exists and is non-zero, the limit of the quotient does not exist.
(g) The Difference Law is used for the denominator; then the Constant Multiple
2h(x) 2(−8) 16
Law and the Quotient Law: lim f (x)−h(x) = 5−(−8) = − 13 .
x→3
p √
(h) By the Root Law, lim 3 h(x) = 3 −8 = −2.
x→3
Information for Students in MATH 140 2008 09 3037

4. Use the Intermediate Value Theorem to show that there is a solution to the equation
x3 + 2x2 − 42 = 0 in the interval (0, 3).
Solution: Let f (x) = x3 + 2x2 − 42. This function is continuous everywhere on
R. Since f (0) = −42, and f (3) = 33 + 2(9) − 42 = 3, and since −42 < 0 < +3,
the function must assume the intermediate value 0 at a point in the open interval
0 < x < 3.

5. Given 
 2x3 + 16 if x ≤ −2
2
f (x) = x + bx + c if −2 < x < 2

3x4 − 48 if x≥2
determine values for b and c so that f is continuous everywhere. Justify your
answer.
Solution: In each of the intervals (−∞, −2], (−2, 2), [2, ∞) the function is a poly-
nomial, and is therefore continuous at all points. The only possibly problematic
points are x = −2 and x = +2. We will determine limits from the left and right at
each of these two points; then we equate the two one-sided values, and obtain con-
ditions on b and c which we attempt to satisfy. lim − 2x3 + 16 = 2(−2)3 + 16 = 0,
x→−2
lim + x2 + bx + c = (−2)2 + b(−2) + c = 4 − 2b + c; lim− x2 + bx + c = 22 + b(2) + c =
x→−2 x→2
4 4
4 + 2b + c, lim+ 3x − 48 = 3(2) − 48 = 0. We solve the equations 0 = 4 − 2b + c
x→2
and 4 + 2b + c = 0, to obtain b = 0, c = −4. With this pair of values — and only
these values — f is continuous everywhere.

6. The displacement in meters of a particle moving in a straight line is given by


s = t3 + sin tπ, where t is measured in seconds. Find the average velocity in meters
per second over the time period [1, 5].
53 + 0 − 13 − 0
Solution: Average velocity = = 31 meters per second.
5−1
x−5
7. Find the value of lim+ .
x→5 |x − 5|
x−5
Solution: For all x > 5, |x−5|
= 1. Hence the limit, as x approaches 5 from the
right is 1.
3x
8. Find the value of x at which the curve y = x+7
has a vertical asymptote.
Solution: The domain of the function is R − {−7}: at all points except for −7
the function is a ratio of polynomials, each having a limit, where the limit of the
denominator is non-zero. Hence, by the Quotient Law, the limit of the function
Information for Students in MATH 140 2008 09 3038

exists (as a real number). However, as x → −7− , the denominator approaches 0


from the left, while the numerator approaches −21; so the ratio becomes infinitely
3x
large positively; this is what we mean when we write lim − x+7 = +∞. Similarly,
x→−7
as x → −7+ , the denominator approaches 0 from the right, while the numerator
approaches −21; so the ratio becomes infinitely large negatively; this is what we
3x
mean when we write lim + x+7 = −∞. It is in cases like these, where the one-sided
x→−7
limits are infinite positively or negatively, that we say that the curve has a vertical
asymptote.
x3 +8
9. Find the value of the limit lim .
x→2 x+2

x3 + 8 lim (x3 + 8) 23 + 8
x→2
Solution: lim = = = 16. (Problem: Find the limit
x→2 x + 2 lim (x + 2) 2+2
x→2
3
of the given quotient as x → −2. Solution: lim x +8 = lim x2 − 2x + 4 =
x→−2 x+2 x→−2
22 + 2 · 2 + 4 = 12.)
¡ 1 ¢
10. Find the value of lim x+2 + x24−4 .
x→−2
¡ 1 ¢
Solution: lim x+2 + x24−4 = lim (x+2)(x−2)
x+2
= lim 1
= 1
= − 14 .
x→−2 x→−2 x→−2 x−2 −2−2

11. Find the value of lim √x−4 .


x→4 x−2
√ √
( x−2)( x+2) √
Solution: For positive x, different from 4, √x−4 = √ = x + 2. Hence
√ x−2 x−2
the limit, as x → 4 is 4 + 2 = 4.
1
12. How close to 4 do we have to take x so that 4x + 10 is within of 26?
100
Solution:
1
|4x + 10 − 26| < 0.01 ⇔ |4(x − 4)| <
100
1
⇔ |x − 4| <
400

(x+1)2
13. At what value of x does the function x2 −1
have a removable discontinuity?
(x+1)2
Solution: x2 −1 = x+1
x−1
for all x except x = −1, where the ratio is not defined
because both numerator and denominator of the fraction become 0 there. Every-
where except at x = 1 this is a ratio of continuous functions, and the denominator
Information for Students in MATH 140 2008 09 3039

is non-zero. Only at x = 1 or at x = −1 can this function have any type of


discontinuity.
As x → 1− , the function approaches −∞; and, as x → 1+ , the function approaches
+∞. Thus x = 1 is an infinite discontinuity: it cannot be “removed” by the device
of defining the function at the point, since no matter what value we would choose
to give the function there, it could still not be continuous.
But a different situation holds at x = −1. Since the function is equal to x+1 x−1
near (but not at) x = −1, its one-sided limit from either side of −1 is 0, so the
limit exists at the point. However, the function is not defined at the point, since
our definition was expressed as a ratio that becomes meaningless there. We can
“remove” the discontinuity by extending the function to the domain R − {1} —
we simply define the value of the newly extended function to be 0 there. Now the
new function is continuous at every point except −1.
(x+1)2
14. At what value of x does the function x2 −1
have an infinite discontinuity?
Solution: See the solution to Problem 13.
|x−2|
15. At what value of x does the function x−2
have a jump discontinuity?
Solution: This function is equal to +1 when x > 2, and to −1 when x < 2; it is
not defined at x = 2. The limits from the left and right are different at x = 2,
so the function has no limit there, and is said to be discontinuous. But, as both
one-sided limits exist, the discontinuity is a jump discontinuity.
1
16. Find the distance between the two values of x at which the function x2 −3x+2
is
discontinuous.
1 1
Solution: x2 −3x+2 = (x−1)(x−2) . Expressed as a ratio of two polynomials — which
are continuous everywhere — this function is also continuous everywhere, except
possibly at points where the denominator is zero. That happens precisely at x = 1
and x = 2. At either of these points the function is not even defined — hence it is
discontinuous at both of these points. The distance between the two discontinuities
is, therefore, |2 − 1| = 1.
(x2 −1)(x2 +3x+2)
17. Find the values of x where the function f (x) = (x2 −1)2 (x+2)(x−3)
has a removable
discontinuity.
Solution: This function is a ratio of polynomials, and polynomials are continuous
everywhere. This function will also be continuous everywhere, except where it is not
defined. That can happen only when the denominator is zero. So the discontinuities
of the function are at x = −1, 1, −2, 3. We arrived at this information without even
considering the factorization of the numerator.
Information for Students in MATH 140 2008 09 3040

Let us now factorize both numerator and denominator:


(x2 − 1)(x2 + 3x + 2) (x − 1)(x + 1)2
= .
(x2 − 1)2 (x + 2)(x − 3) (x − 1)2 (x + 1)2 (x − 3)

We can cancel equal non-zero factors in numerator and denominator. Thus


1
f (x) =
(x − 1)(x + 3)

provided x is distinct from 1, −1, and −2. As x → 1, and also as x → −3, this
ratio becomes infinite; so the limits as we approach these two points are infinite;
in fact the function approaches +∞ from one side of each of these points, and −∞
from the other side. So 1 and −3 are infinite discontinuities. But, as x → −2, the
ratio does not become infinite: the limit is 13 from either side. Nor, as x → −1,
does the ratio become infinite: the limit is 14 from either side; but the function is
not defined at either x = 2 or x = −1. We can “remove” these discontinuities by
defining a new function which takes on the values 13 at x = −2, 14 at x = −1, and,
elsewhere, behaves like f (x).
½
−3x if x ≤ 1
18. Find the real number(s) c for which the function f (x) =
(x − c)(x + c) if x > 1
is continuous on (−∞, +∞).
Solution: For x < 1 the function is defined by the upper line of the array; as
x → 1− , f (x) approaches the value (−3) · 1 = −3. For x > 1 it is the second
line of the array which defines the function. Here, as x → 1+ , (x − c)(x + c) →
(1 − c)(1 + c). To make the function continuous at x = 1 it is necessary and
sufficient that −3 = (1 − c)(1 + c), i.e. that c = 2 or c = −2.
r
8x + 3x2
19. Evaluate lim .
x→∞ 13x2 − 9
Solution:
r s
2 1
8x + 3x x2 8x + 3x2
lim = lim 1 ·
x→∞ 13x2 − 9 x→∞
x2
13x2 − 9
s
1
x2
(8x + 3x2 )
= lim 1
x→∞
x2
(13x2 − 9)
s
8
x
+3
= lim
x→∞ 13 − x92
Information for Students in MATH 140 2008 09 3041

q
8
x
+3
= lim q
x→∞
13 − x92
q
lim x8 + 3
x→∞
= q (Quotient Law)
lim 13 − x92
x→∞
q ¡ ¢
lim x8 + 3
x→∞
= q ¡ ¢ (Root Law)
lim 13 − x92
x→∞

0+3
= √ (Sum Law)
13 − 0
√ r
3 3
= √ =
13 13

20. Determine whether there exists a real number x which is exactly 10 more than its
5th power.
Solution: (cf. [19, Solution to Exercise 2.6.59]). The problem may be paraphrased
as asking whether there exists a solution x to the equation x5 + 10 = x; or, equiv-
alently, whether the function f (x) = x5 − x + 10 is ever zero. Now f is continuous
everywhere; f (0) = 10 > 0, and f (−2) = −32 + 2 + 10 < 0. By the Intermediate
Value Theorem there will exist a point in the interval (−2, 0) at which f (x) = 0.

6.3.3 Third 2000/2001 Problem Assignment, with Solutions


1. Let f and g be functions whose domain is R, and which possess derivatives at every
point in R. Suppose also that the following data are given about f , g and their
derivatives.
x f (x) g(x) f 0 (x) g 0 (x)
1 1 −1 4 −2
2 −2 3 0 2
3 5 0 5 6
Showing all your work, determine each of the following, or explain why either
• it does not exist; or
• you do not have enough information to find the value.
µ ¶¯
d f (x) ¯¯
(a)
dx g(x) ¯x=1
Information for Students in MATH 140 2008 09 3042

¯
d ¯
(b) (f (t) + g(t))¯¯
dt t=2
µ ¶¯
d 1 ¯
(c) ¯
dx g(x) ¯x=3
µ ¶
d 0
(d) (f (x) + g (x)) (2)
dx
µ x ¶¯
d e ¯
(e) ¯ (Do not attempt to approximate e.)
dx f (x) ¯x=1
µ ¶¯
d f (x) ¯¯
(f)
dx f (x) ¯ x=2

Solution:

(a) By the Quotient Rule,


µ ¶¯
d f (x) ¯¯ f 0 (1) · g(1) − f (1) · g 0 (1) 4 · (−1) − 1 · (−2)
¯ = 2
= = −2 .
dx g(x) x=1 g(1) (−1)2

(b) By the Sum Rule,

d
(f (x) + g(x)) (2) = f 0 (2) + g 0 (2) = 0 + 2 = 2 .
dx
1
(c) Since g(3) = 0, the function is not defined at x = 3. The derivative of
g(x)
g at x = 3 is defined in terms of g(3); so, if g is not defined there, it cannot
have a derivative there either.
(d) We do not know whether the function g 0 has a derivative at x = 2 — equiv-
alently, whether the function g has a second derivative at x = 2. Without
this information, and the actual value of that derivative if it exists, we cannot
evaluate the derivative of the sum.
(e) By the Quotient Rule,
µ ¶¯ d x¯
¯
d ex ¯¯ dx
e x=1· f (1) − e1 · f 0 (1)
=
dx f (x) ¯x=1 f (1)2
e1 (f (1) − f 0 (1))
= = e(1 − 4) = −3e
f (1)2
Information for Students in MATH 140 2008 09 3043

(f) This case can also be attacked using the Quotient Rule, and we find that
µ ¶
d f (x) f 0 (2) · f (2) − f (2) · f 0 (2) 4·1−1·4
(2) = 2
= = 0.
dx f (x) f (2) 12

f (x)
Alternatively, one can observe that the function is defined wherever
f (x)
f (x) 6= 0; in particular, it is defined at x = 2; wherever it is defined, its value
is 1. Thus this is a constant function! The derivative of any constant function
is zero.

2. Suppose that a particle moves in a straight line with its position at time t given
by the formula f (t) = et (sin t + cos t).

(a) Find the velocity, the speed, and the acceleration of the particle at time t.
(b) Determine the average velocity during the time interval from t = 0 to t = 2π.
(c) Determine the smallest positive value of t — if any — when the particle
returns to the origin.
(d) Determine the smallest positive value of t — if any — when the particle is
stationary — i.e. the velocity is 0.
(e) Determine whether there is a maximum distance that the particle attains
away from the origin. If there is a maximum distance, determine what it is.

[The velocity is given by the derivative of f (t) (which is called the displacement;
the speed is the magnitude of the velocity. The acceleration is the derivative of the
velocity with respect to time.]
Hints: You may wish to use the identities:
³ π´ 1
sin x + = √ (sin x + cos x)
4 2
³ π´ 1
cos x + = √ (− sin x + cos x)
4 2
which are consequences of the addition formulæ and the known values for the sine
and cosine of π4 . Do not attempt to simplify complicated formulæ involving π.
Solution:

(a) The velocity at time t is f 0 (t) = et (cos t − sin t) + et (sin t + cos t) = 2et cos t.
The speed is the magnitude of the velocity, i.e., 2et | cos t|. (We may remove
the exponential factor outside the absolute signs since exponentials are always
Information for Students in MATH 140 2008 09 3044

d 0
positive.) The acceleration is the derivative of the velocity, i.e. f (t) =
¡ t ¢ dt
2 e (− sin t) + et cos t = 2et (− sin t + cos t).
√ ¡ ¢
(b) Using one of the given identities, we see that f (t) = 2 · et · sin x + π4 . The
average velocity from t = 0 to t = 2π is

f (2π) − f (0)
2π − 0
1 ³√ 2π ³ π´ √ ³ π ´´
= 2e · sin 2π + − 2 · e0 · sin
2π 4 4
1 √ ¡ 2π ¢ π e2π − 1
= 2 e − 1 sin =
2π 4 2π
√ ¡ ¢
(c) We seek the smallest positive value t when f (t) = 0, i.e. when
¡ et 2¢sin t + π4 =
0. Since et is never zero, this equation is equivalent to sin t + π4 = 0, whose
solutions are t + π4 = nπ, where n is any integer. The smallest positive value
will then be 34 π.
(d) 2et cos t will be zero precisely when the cosine is zero, i.e. when t is an odd
integer multiple of π2 .
t
(e) We have
√ seen above that f (t) ¡ is πa¢product of e (which is never 0), the con-
stant 2, and the factor sin t + 4 , which oscillates in value between +1 and
−1. Because of the exponential factor et the amplitude of the oscillations is
increasing as t becomes large. The limit lim f (t) does not exist. We can see
t→∞
this by exhibiting values of t where the function is arbitrarily large positively,
and others where it is arbitrarily large negatively. For example, if we take
π
t = 2nπ + , we find the values oscillating between et and −et , and we know
4
that the exponential function approaches infinity.

3. Determine the derivative of each of the following functions. You may not use the
Chain Rule, but you may use any of the Rules and Theorems in [17, §§3.1–3.4],
including the General Power Rule.
µ ¶
1 1
(a) 2 ex + x
e
1
(b) 2
(ex − e−x )
x4 + 2x2 − x − 5
(c) √
x
(d) xe − ex
Information for Students in MATH 140 2008 09 3045

Solution:
(a)
µ µ ¶¶ µµ ¶¶
d 1 x 1 1 d x 1
e + x = e + x Constant Multiple Rule
dx 2 e 2 dx e
µ ¶
1 dex 0 · ex − 1 · ex
= + Quotient Rule
2 dx e2x
ex − e−x
=
2
ex + e−x
(b) Analogously to the preceding, .
2
(c)
x4 + 2x2 − x − 5 7 3 1 1
√ = x 2 + 2x 2 − x 2 − 5x− 2
x
µ 4 ¶
2
d x + 2x − x − 5 d ³ 7 3 1 1
´
⇒ √ = x 2 + 2x 2 − x 2 − 5x− 2
dx x dx
µ ¶
7 5 3 1 1 −1 1 3
= x 2 + · 2x 2 − x 2 − − 5x− 2
2 2 2 2
7 5 1 1 1 5 3
= x 2 + 3x 2 − x− 2 + x− 2
2 2 2
d e
(d) (x − ex ) = e · xe−1 − ex , by the Generalized Power Rule and properties of
dx
the exponential.
4. Showing all your work, determine the derivative of each of the following functions
from first principles: that is, you are to evaluate a limit in each case, and are not
to use any of the “Rules” for evaluating derivatives.
(a) f (x) = 4x − 6

(b) g(x) = 2x + 3
1
(c) h(x) = 3 . [Hint: Remember the factorization a3 − b3 = (a − b)(a2 + ab + b2 ).]
x
Solution:
(a) For all x,
(4(x + h) − 6) − (4x − 6) 4h
f 0 (x) = lim = lim =4
h→0 h h→0 h
Information for Students in MATH 140 2008 09 3046

(b) For all x,


p √
2(x + h) + 3 − 2x + 3
g 0 (x) = lim
h→0
p h p
√ √
2(x + h) + 3 − 2x + 3 2(x + h) + 3 + 2x + 3
= lim ·p √
h→0 h 2(x + h) + 3 − 2x + 3
(2(x + h) + 3) − (2x + 3)
= lim p √
h→0 h( 2(x + h) + 3 + 2x + 3)
2
= lim p √
h→0 2(x + h) + 3 + 2x + 3
1
= √
2x + 3

(c) For all x,


1 1
3
− 3
(x + h) x
h0 (x) = lim
h→0 h
x3 − (x + h)3 −3x2 h − 3xh2 − h3
= lim = lim
h→0 hx3 (x + h)3 h→0 hx3 (x + h)3
−3x2 − 3xh − h2
= lim
h→0 x3 (x + h)3
lim (−3x2 − 3xh − h2 ) −3x2
h→0
= = = −3x−4
lim (x3 (x + h)3 ) x6
h→0

5. Determine precisely where the function f , defined below, is differentiable.




 −x − 2 if x ≤ −1


 −x2 if −1 < x < 0
f (x) = 1 if x=0



 −x2 if 0 < x ≤ +2

4 − 4x if x > +2

Solution: The function is a polynomial in each of the intervals (−∞, −1], (−1, 0),
(0, 2), and (2, ∞). As polynomials are differentiable [17, §3.1], the only places
where differentiability can fail is at the points x = −1, 0, 2. We check each of them
separately.
Information for Students in MATH 140 2008 09 3047

f (x) − f (−1)
x = −1: We have to determine whether the limit lim exists. By
x→−1 x − (−1)
[17, Theorem 2.3.1, p. 107], the two one-sided limits must exist and be equal.
But
f (x) − f (−1) (−x − 2) − (1 − 2)
lim − = lim −
x→−1 x − (−1) x→−1 x − (−1)
−x − 1
= lim − = −1 while
x→−1 x+1
f (x) − f (−1) −x2 − (1 − 2)
lim + = lim +
x→−1 x − (−1) x→−1 x − (−1)
−x2 + 1
= lim + = lim + (−x + 1) = 2 6= −1
x→−1 x+1 x→−1

As the one-sided limits are not equal, the function is not differentiable at this
point.
2
x = 0: Here we have to consider the existence of lim f (x)−fx
(0)
, that is lim −xx−1 .
x→0 x→0
But this limit cannot exist, since the ratio becomes infinitely large close to
0, as the numerator approaches −1, but is divided by an arbitrarily small
denominator. Thus the limit does not exist, and the function fails to be
differentiable at x = 0.
We could have argued this alternatively by appealing to [17, Theorem 2.9.4,
p. 169], since the function is discontinuous at x = 0, so it cannot possibly be
differentiable there.
f (x)−f (2)
x = 2: This time we have to consider the existence of lim x−2
. We observe
x→2
2
from the definition that f (2) is defined to be −2 = −4. Then

f (x) − f (2) −x2 − (−4)


lim− = lim−
x→2 x−2 x→2 x−2
= lim− (−x − 2) = −4 ; and
x→2
f (x) − f (2) (4 − 4x) − (−4)
lim+ = lim+
x→2 x−2 x→2 x−2
= lim+ −4 = −4
x→2

As the limits from the left and right are equal, the function is, indeed, differ-
entiable at the point x = 2.

6. The curve y = 3x5 − 20x3 − 675x + 12 has some points with horizontal tangents.
Showing all your work, find all such points.
Information for Students in MATH 140 2008 09 3048

d ¡ 5 ¢
Solution: Setting 3x − 20x3 − 675x + 12 equal to zero, we find that 15x4 −
dx
60x2 − 675 = 0, or, equivalently, (x2 )2 − 4x2 − 45 = 0. This is a quadratic equation
(in x2 ), and its solutions are x2 = −5 and x2 = 9. There are no real solutions to
x2 = −5, so we may confine ourselves to the second equation, whose solutions are
x = ±3. Thus there are exactly 2 points with horizontal tangents: (3, −1824) and
(−3, 1848).
7. Determine the value of each of the following limits, or explain why they do not
exist. Do not use l’Hôpital’s Rule.
tan3 πx
(a) lim
x→0 x3
(b) lim(cot 5θ)2 · (sin 3θ) · tan(−2θ)
θ→0

Solution:
(a)
õ ¶3 ³ !
tan3 πx sin πx π ´3
lim = lim ·
x→0 x3 x→0 πx cos πx
µ ¶3 ³
sin πx π ´3
= lim lim (Product Rule)
x→0 πx x→0 cos πx
µ ¶3 µ ¶3
sin πx limx→0 π
= lim (Quotient Rule)
x→0 πx limx→0 cos πx
µ ¶3 µ ¶3
sin y limx→0 π
= lim (where y = πx)
y→0 y limx→0 cos πx
³ π ´3
3
= 1 · = π3
1
(b)
¡ ¢
lim (cot 5θ)2 · (sin 3θ) · tan(−2θ)
θ→0
à µ ¶2 µ ¶ µ ¶ !
1 5θ sin 3θ tan(−2θ)
= lim cos2 5θ · · · · 3θ · · 2θ
θ→0 25θ2 sin 5θ 3θ −2θ
õ ¶2 µ ¶ µ ¶!
6 5θ sin 3θ sin(−2θ) 1
= − lim cos2 5θ · lim · · · lim
25 θ→0 θ→0 sin 5θ 3θ −2θ θ→0 cos(−2θ)

6 6
= − · 1 · 12 · 1 · 1 · 1 = −
25 25
Information for Students in MATH 140 2008 09 3049

8. Determine all points (x, cos2 x) on the graph of the function cos2 x (i.e. cos x · cos x)
at which
(a) the tangent to the curve is parallel to the line y = x + 1.
(b) the tangent to the curve is horizontal
(c) the tangent to the curve is vertical
(d) the tangent to the curve is parallel to the line y = 2x − 3
(e) the normal to the curve passes through the origin, and |x| > 1.
[Hints: The first and last parts of the problem are more difficult than the others.
Remember the identities involving sin 2x and cos 2x. The normal to the curve at
a point is the line through the point which is perpendicular to the tangent.]
Solution:
(a) The derivative of (cos x)2 is cos x·(− sin x)+(− sin x)·cos x = −2 sin x·cos x.130
We impose the condition that this be equal to the slope of the line y = x + 1,
i.e., that
−2 sin x · cos x = 1 (215)
The points we seek will be the solutions to equation (215). A simple attack
is to observe that this equation is equivalent to
sin 2x = −1 . (216)
¡3 ¢
The general solution¡ to the
¢ last equation is 2x = 2 + 2n π, where n is any
integer; hence x = 43 + n π, where n is any integer. Thus there are infinitely
many points where the tangent¡ has¢ slope11: the x-coordinates are as indicated;
the y-coordinates are cos 4 + n π = 2 . But note that the line y = 21 cuts
2 3

the curve also in infinitely many points where the tangent has slope −1.
(b) We have to solve the equation −2 sin x·cos x = 0, or, equivalently, −2 sin 2x =
0. The solutions to this equation are 2x = nπ, i.e. x = nπ 2
, where n is any
integer.
(c) The function is differentiable everywhere; that means that the derivative has
a (finite) value at every point, and so the tangent cannot be vertical. There
are no such points on the curve.
(d) The value of the derivative is −2 sin x · cos x, or − sin 2x. As a sine cannot
have magnitude exceeding 1, the slopes of tangents to this curve can never
equal 2. There are no points of this type!
130
We are avoiding using the Chain Rule, as the assignment was due before the Rule had been fully
discussed in the lectures.
Information for Students in MATH 140 2008 09 3050

1
(e) The slope of the normal at the point (t, cos2 t) is − . The equation of
− sin 2t
the normal is
1
y − cos2 t = (x − t)
sin 2t
The normal will pass through the origin if the coordinates of the origin satisfy
the equation, that is if t = cos2 t sin 2t. But this implies that t is the product
of two factors, neither of which can exceed 1 in magnitude, so there are no
points with this property outside the interval −1 ≤ t ≤ 1.

6.3.4 Fourth 2000/2001 Problem Assignment, with Solutions


Solutions to most of the problems can be found in the Student Solutions Manual [19].
1. (a) [17, Exercise 3.5.3, p. 221] Write the composite function cos(tan x) in the form
f (g(x)). [Identify the “inner” function u = g(x), and the “outer” function
dy
y = f (u).] Then find the derivative .
dx
(b) Find an equation for the tangent to the curve y = cos(tan x) at the point
π
x= .
4
(c) Find an equation for the normal to the curve y = cos(tan x) at the point

x= .
4
Solution:

(a) In [19, Exercise 3.5.3, p. 82] the derivative is shown to have value − sin tan x ·
sec2 x. ³π ´ √ 2
(b) The slope of the tangent at , cos 1 is − sin 1 · 2 . Hence an equation of
4
the tangent is ³ π´
y − cos 1 = −2 sin 1 · x − .
4
µ ¶
5π √
(c) The slope of the tangent at , cos(1) is − sin(1) · (− 2)2 = −2 sin 1;
4
1
hence the slope of the normal is . An equation of the tangent is
2 sin 1
µ ¶
1 5π
y − cos 1 = · x− .
2 sin 1 4

or x − 2 sin 1 · y = − sin 2 + .
4
Information for Students in MATH 140 2008 09 3051

2. Find the derivative of each of the following functions:

(a) [17, Exercise 3.5.19, p. 222] (2x − 5)4 (8x2 − 5)−3


2
(b) [17, Exercise 3.5.21, p. 222] xe−x
1
(c) [17, Exercise 3.5.29, p. 222] 5− x

Solution:

(a) [19, Exercise 3.5.19, p. 82]


(b) [19, Exercise 3.5.21, p. 82] Note that the original function is odd ; and the
derivative is even. This is an instance of [17, Exercise 3.5.73, p. 224].
(c) [19, Exercise 3.5.29, p. 82]. (This problem could be solved from first princi-
ples. Simply observe that 5 = eln 5 , apply the usual rules for exponents, and
differentiate an exponential [now to base e] using the Chain Rule.)

3. [17, Exercise 3.5.56, p. 222] A table of values for f , g, f 0 , g 0 is given:

x f (x) g(x) f 0 (x) g 0 (x)


1 3 2 4 6
2 1 8 5 7
3 7 2 7 9

(a) If F (x) = (f ◦ f )(x), determine F 0 (2).


(b) If G(x) = (g ◦ g)(x), determine G0 (3).

Solution:

(a) F 0 (2) = f 0 (f (2)) · f 0 (2) = f 0 (1) · f 0 (2) = 4 · 5 = 20.


(b) G0 (3) = g 0 (g(3)) · g 0 (3) = g 0 (2) · g 0 (3) = 7 · 9 = 63.
dy
4. In each of the following cases, find by implicit differentiation.
dx
y
(a) [17, Exercise 3.6.11, p. 230] = x2 + 1
x−y

(b) [17, Exercise 3.6.13, p. 230] xy = 1 + x2 y
(c) [17, Exercise 3.6.17, p. 230] cos x − y = xex

Solution:

(a) Two different approaches are given in [19, Exercise 3.6.11, p. 86].
Information for Students in MATH 140 2008 09 3052

(b) [19, Exercise 3.6.13, p. 86]


(c) [19, Exercise 3.6.18, p. 86]
5. (a) [17, Exercise 3.6.23, p. 230] Regarding y as the “independent” variable, and x
as the “dependent” variable (i.e. regarding x as a function of y) apply implicit
differentiation to the equation
y 4 + x2 y 2 + yx4 = y + 1
dx
to determine .
dy
dy dy dx
(b) Use the same equation to determine , and verify that · = 1 (a fact
dx dx dy
which follows from the Chain Rule by interpreting y as a function of x which,
in turn, is a function of y, i.e. taking the point of view y = y(x(y))).
Solution:
(a) [19, Exercise 3.6.23, p. 86]
(b) Differentiating both sides of the given equation with respect to x yields
¡ ¢ ¡ ¢
4y 3 · y 0 + 2xy 2 + x2 · 2yy 0 + y 0 x4 + y · 4x3 = y 0 + 0
¡ ¢
⇒ 4y 3 + 2x2 y + x4 − 1 y 0 = −2xy 2 − 4yx3
2xy 2 + 4yx3
⇒ y0 =
1 − 4y 3 − 2x2 y − x4
³ √ ´
6. (a) Determine the derivative of cot−1 x − 1 + x2
(b) [17, Exercise 3.6.41, p. 231] Determine the derivative of arcsin(x2 ).
Solution:
(a)
d ³ −1 ³ √ ´´
cot x − 1 + x2
dx
1 d ³ √
2
´
= − ¡ √ ¢2 · x − 1 + x
1 + x − 1 + x2 dx
µ ¶
1 x
= − ¡ √ ¢ · 1− √
1 2 + 2x2 − 2x 1 + x2 1 + x2

1 1 + x2 − x 1
= − √ ¡√ ¢· √ =−
2 1+x 2 2
1+x −x 1+x 2 2(1 + x2 )
Information for Students in MATH 140 2008 09 3053

(b) [19, Exercise 3.6.41, p. 88]


7. (a) [17, Exercise 3.6.35] Find all points on the curve 2(x2 + y 2 )2 = 25(x2 − y 2 )
where the tangent is horizontal. (You may assume that the origin is not one
of these points.)
dx
(b) For the same curve, determine all points where = 0, i.e. where the tangent
dy
is vertical. (You may assume that the origin is not one of these points.)
Solution:
(a) [19, Exercise 3.6.35, p. 88] (It is shown that the four points with horizontal
tangents lie on a certain circle centred at the origin. To determine the points
one must solve the equation of this circle with the equation of the given curve.)
(b) One finds by implicit differentiation that
dx 25y + 4y(x2 + y 2 )
=
dy 25x − 4x(x2 + y 2 )
This derivative vanishes when y = 0 or 4(x2 + y 2 ) = −25. The latter situation
cannot occur, as a sum of squares cannot be negative; hence y = 0. That
is, the points with vertical tangents are at points where the curve meets the
5
x-axis: these are the points where x = 0 or x = ± √ . But being on the x-axis
2
is a necessary condition for having a vertical tangent, and is not sufficient. At
the origin there are two tangents,³ and neither
´ of them is vertical. The only
5
points with vertical tangents are ± √2 , 0 .

8. (cf. [17, Exercise 3.7.53, p. 239]) Find constants ci (i = 0, 1, 2) such that the
function f ( t) = c2 t2 + c1 t1 + c0 has the properties that f (−1) = 0, f 0 (−1) = 7,
f 00 (−1) = 10.
Solution: Imposing the given conditions yields the three equations
c2 − c1 + c0 = 0
−2c2 + c1 = 7
2c2 = 10
from which we conclude that c0 = 22, c1 = 27, c2 = 5.
9. [17, Exercise 3.8.27, p. 246] Find the domain of the function f , and determine its
derivative, where f (x) = x2 ln(1 − x2 ).
Solution: [19, Exercise 3.8.27, p. 94]
Information for Students in MATH 140 2008 09 3054

10. Determine the first and second derivatives of the function f (x) = | ln(sec x+tan x)|.
Solution:
d 1 d
ln(sec x + tan x) = · (sec x + tan x)
dx sec x + tan x dx
1
= · (sec x · tan x + sec2 x) = sec x
sec x + tan x
d2 d
2
ln(sec x + tan x) = sec x = sec x · tan x
dx dx
The problem referred to the derivatives of f (x) = | ln(sec x + tan x)|; the logarithm
will be negative precisely when sec x+tan x < 1; this last inequality may be shown,
using trigonometric identities, to be equivalent to
³x π ´ ³x´
cos + · sin < 0.
2 4 2
For the range 0 < x < 4π, we find, examining the signs of the two factors as
0 < x2 < π4 , ... 7π
4
< x2 < 2π, that the product is negative except when ¡0 < x2 < π
¢ 4
x 5π 1
or π < 2 < 4 . Thus, if we define s(x) to be +1 when 2nπ < x < 2n + 2 π,
where n is any integer, and −1 everywhere else, f 0 (x) = s(x) · sec x, and f 00 (x) =
s(x) · sec x tan x; neither function is defined at odd integer multiples of π2 .
11. Using “logarithmic differentiation”, or otherwise, determine the derivative of
sin2 x · tan4 x
(a) [17, Exercise 3.8.37, p. 246] y = .
(x2 + 1)2
(b) y = xln x
Solution:
(a) [19, Exercise 3.8.37, p. 95]
1 1
(b) Since ln y = (ln x)2 , · y 0 = 2 ln x · . Hence y 0 = 2x(ln x)−1 ln x.
y x
12. (a) [17, Exercise 3.7.19, p. 238] Find the first, second, and third derivatives of
g(t) = t3 e5t .
(b) [17, Exercise 3.7.13, p. 238] Find the second, third, and fourth derivatives of
x
.
1−x
(c) [17, Exercise 3.7.29, p. 238] Find y 00 where y is defined implicitly by the
equation x3 + y 3 = 1. Simplify your answer as much as possible.
Solution:
Information for Students in MATH 140 2008 09 3055

(a) [19, Exercise 3.7.19, p. 90]


d ¡ ¢
g 000 (t) = (25t3 + 30t2 + 6t)e5t
¡dt 2 ¢ ¡ ¢
= 75t + 60t1 + 6t0 e5t + 5 25t3 + 30t2 + 6t e5t
¡ ¢
= 125t3 + 225t2 + 90t1 + 6t0 e5t

(b) [19, Exercise 3.7.13, p. 90] This problem can be attacked naı̈vely, with re-
peated applications of the Quotient Rule. Another approach is to observe
x 1
that = −1 + . Hence
1−x 1−x
d x d ¡ ¢
= 0+ (1 − x)−1
dx 1 − x dx
d
= (−1) · (1 − x)−2 · (1 − x) = (1 − x)−2
dx
d2
x d ¡ ¢
2
= (1 − x)−2 = (−2) · (1 − x)−3 · (−1) = 2 · (1 − x)−3
dx 1 − x dx
d3 x d ¡ −3
¢
= 2 (1 − x) = 2(−3) · (1 − x)−4 · (−1) = 3 · 2 · (1 − x)−4
dx3 1 − x dx
d4 x d ¡ ¢
4
= 3·2 (1 − x)−4 = 3 · 2 · (−4) · (1 − x)−5 · (−1)
dx 1 − x dx
= 4 · 3 · 2 · (1 − x)−5

(c) [19, Exercise 3.7.29, p. 91]


³ x ´n
13. Evaluate the following limits, if possible; you may assume that lim 1 + = ex .
n→∞ n
In each case it is suggested that you make a substitution of the form m = kn, where
k is a constant that you choose, and then replace lim by lim when k > 0, or by
n→∞ m→∞
lim when k < 0.
m→−∞
µ ¶n
2
(a) lim 1 + .
n→∞ n
³ x ´n
(b) lim 1 − .
n→∞ 3n
Solution:
n
(a) Define m = , so n = 2m. Then
2
µ ¶n µ ¶2m
2 1
lim 1 + = lim 1 +
n→∞ n m→∞ m
Information for Students in MATH 140 2008 09 3056

õµ ¶m ¶2 !
1
= lim 1+
m→∞ m
µ µ ¶m ¶2
1
= lim 1 + Product Law
m→∞ m
= e2
m
(b) Define m = −3n, so n = − . Then
3
µ ¶n r³
1 x ´m
lim 1− = lim 3 1 −
n→∞ 3n m→−∞ m
r ³ x ´m
= 3
lim 1 − Root Law
m→−∞ m
r ³
3 x ´−t
= lim 1 + (t = −m)
t→∞ t
s
1
= 3 lim
¡ ¢
t→∞ 1 + x t
t
1 1 x
= r ³ ´ = √3 x
= e− 3
3 x t e
lim 1 +
t→∞ t

6.3.5 Fifth 2000/2001 Problem Assignment, with Solutions


1. [17, Exercise 3.9.12, p. 251] Prove the following identity about hyperbolic functions,
using only the definitions of these functions in terms of exponentials:

cosh(x + y) = cosh x · cosh y + sinh x · sinh y

Solution:

cosh x · cosh y + sinh x · sinh y


ex + e−x ey + e−y ex − e−x ey − e−y
= · + ·
2 2 2 2
(ex+y + ex−y + e−x+y + e−x−y ) + (ex+y − ex−y − e−x+y + e−x−y )
=
4
2ex+y + 2e−x−y
= = cosh(x + y)
4
Information for Students in MATH 140 2008 09 3057

4
2. [17, Exercise 3.9.21, p. 251] If tanh x = , find the values of the other 5 hyperbolic
5
functions at x.
Solution: [19, Exercise 3.9.21, p. 96]. (The problem could also be solved by using
the given information to show that e2x = 9, hence ex = +3, and thence determining
the values of the other functions.)
3. Use the definitions of the hyperbolic functions to find the following limits:
(a) [17, Exercise 3.9.23(b), p. 252] lim tanh x
x→−∞

(b) [17, Exercise 3.9.23(c), p. 252] lim sinh x


x→∞
(c) [17, Exercise 3.9.23(e), p. 252] lim sechx
x→∞
sinh x
(d) [17, Exercise 3.9.52, p. 252] lim
x→∞ ex
Solution:
(a) [19, Exercise 3.9.23(b), p. 96]
(b) [19, Exercise 3.9.23(c), p. 96]
(c) [19, Exercise 3.9.23(e), p. 96]
sinh x 1¡ −2x
¢ 1
(d) Since x
= 1 − e , and lim e−x = 0, the limit as x → ∞ is .
e 2 x→∞ 2
4. Determine the derivative of each of the following functions:
(a) [17, Exercise 3.9.32, p. 252] g(x) = sinh2 x
(b) [17, Exercise 3.9.38, p. 252] f (t) = ln(sinh t)
x √
(c) [17, Exercise 3.9.45, p. 252] x sinh−1 − 9 + x2
3
Solution:
(a) g 0 (x) = 2 sinh x·cosh x, which could be expressed more compactly as sinh(2x).
d 1
(b) f (t) = · cosh t = coth t
dt sinh t
(c) [19, Exercise 3.9.45, p. 98]
5. (a) [17, Exercise 3.10.7, p. 257] A street light is mounted at the top of a 15-
foot-tall pole. A man 6 feet tall walks away from the pole with a speed of 5
feet/second along a straight path. How fast is the tip of his shadow moving
when he is 40 feet from the base of the pole?
Information for Students in MATH 140 2008 09 3058

(b) The man extends his arms horizontally so that the distance between his fin-
gertips is 6 feet. What is the rate of increase of the shadow of his extended
arms on the ground when his feet are 40 feet from the base of the pole? You
may assume that his shoulders are 5 feet above the ground.
Solution:
(a) [19, Exercise 3.10.7, p. 99]
(b) We can consider two sets of similar triangles similar to the triangles considered
in part (a). Let A denote the top of the lamppost, and B its foot; let C be
a point on the man, at shoulder-height, and let D be a point on the man’s
feet; let G be the point where the line through A and C meets the ground.
15 |AB| |AG| |BG|
Then triangles ABG and CDG are similar, so = = = ,
5 |CD| |CG| |DG|
so |BG| = 3|DG|, |AG| = 3|CG|, and |BD| = 2|DG|.
Now let E be the end of the man’s horizontally-extended left arm, and let F
be the end of the shadow cast by that arm on the ground. Here triangles AGF
|AG| |GF | |GF | 3
and ACE are similar, so = , so, since |AG| = 3|CG|, = ;
|AC| |CE| |CE| 2
9
since |CE| = 3, |GF | = . The shadow of the extended arms is 2 × 29 = 9
2
feet long. As this shadow is of constant length, its rate of change is 0.
6. [17, Exercise 3.10.25, p. 258] Boyle’s Law states that, when a sample of gas is
compressed at a constant temperature, the pressure P , and volume V satisfy the
equation P V = C, where C is a constant depending on the sample. Suppose that,
at a certain instant, the volume is 600 cm3 , the pressure is 150kPa, and the pressure
is increasing at a rate of 20 kPa/minute. At what rate is the volume decreasing at
this instant?
Solution: [19, Exercise 3.10.25, p. 100]
7. Use differentials (or, equivalently, a linear approximation) to estimate the following
numbers:
(a) [17, Exercise 3.11.34, p. 265]
(b) [17, Exercise 3.11.35, p. 265]
(c) [17, Exericse 3.11.36, p. 265]
Solution:
(a) Let y = f (x) = x6 , so dy = 6x5 dx. (1.97)6 = (2 − 0.03)6 = f (2 − 0.03) ≈
f (2) + f 0 (2) · (−0.03) = 26 + 6 · 25 · (−0.03) = 58.24.
Information for Students in MATH 140 2008 09 3059

(b) [19, Exercise 3.11.35, p. 104]


dx
(c) Let y = g(x) = ln x. Then dy = g 0 (x) dx = . Then ln(1.07) = g(1+0.07) ≈
x
0.07
g(1) + g 0 (1) · 0.07 = ln 1 + = 0.07.
1
8. [17, Exercise 4.2.32, p. 294] Prove the identity

2 arcsin x = arccos(1 − 2x2 ) (x ≥ 0)

by using the method of [17, Example 6, p. 2902].


1
Solution: Define f (x) = 2 arcsin x − arccos(1 − 2x2 ). Then f 0 (x) = √ −
1 − x2
−1 1 4x 1 4x
p ·(0−4x) = 2 √ −p = 2√ − √ =
1 − (1 − 2x2 )2 1−x 2 22 |x|2 (1 − x2 ) 1 − x 2x 1 − x2
2

0; note that we have used the hypothesis that x > 0. Hence, by [17, Theorem 4.2.5,
p. 291], f (x) =constant. We can determine the value of the constant by select-
π
ing a “convenient” value; for example, take x = 1: then, since arcsin 1 = , and
2
arccos(−1) = π, we find that the constant is equal to 0.
Note that this argument does not hold for x = 0. There, however, we can simply
observe that f (0) = 2 arcsin 0 − arccos 1 = 2 · 0 − 0 = 0.

9. [17, Exercise 4.2.6] Let f (x) = (x − 1)−2 . Show that f (0) = f (2), but there is no
number c such that 0 < c < 2 and f 0 (c) = 0. Why does this not contradict Rolle’s
Theorem?
Solution: f (0) = (−1)−2 = 1; f (2) = 1−2 = 1. Since f 0 (x) = −2(x − 1)−3 ,
wherever it is defined, the value of the derivative ranges, as 0 ≤ x < 1, from 2 to
∞; similarly, as 1 < x ≤ 2, the derivative ranges between −∞ and −2. Thus the
derivative never takes values between −2 and 2.
This is not a counterexample to Rolle’s Theorem, since that theorem requires
that the function be differentiable throughout the open interval, and the present
function fails to be differentiable at one point in the interval — namely the point
x = 1. That failure to be differentiable, even though it is only at a single point,
renders Rolle’s Theorem inapplicable.

10. [17, Exercise 4.2.17, p. 293] Show that the polynomial x5 + 10x + 3 has exactly one
real root.
Solution: f (0) = 3 > 0, but f (−1) = −12 < 0; by the Intermediate Value Theorem
this function — which, being a polynomial, is continuous everywhere, in particular
Information for Students in MATH 140 2008 09 3060

in the interval −1 < x < 0 — must take on the value 0 at least once between −1
and 0. However, the Intermediate Value Theorem does not exclude the possibility
that there is more than one place where f = 0. If, however, f were to vanish at
distinct points x1 and x2 , then, by Rolle’s theorem, f 0 would be zero somewhere
between x1 and x2 . But f 0 (x) = 5x4 + 10; being the sum of a square and a positive
number, this cannot be zero.

11. For each of the following functions

• find the “critical numbers”;


• find the vertical and horizontal asymptotes;
• find the intervals of increase or decrease;
• find the local maximum and minimum values;
• find the intervals of concavity and the inflection points;
• use the information you have determined above to sketh the graph.

(A “critical number” of a function is a point in the domain of the function where


either the derivative is not defined, or the derivative is defined and equal to zero.
This term is sometimes used (by other authors) with a slightly different definition.)

(a) (cf. [17, Exercise 4.3.39, p. 304]) f1 (x) = x2 + 1 − x
1 + x2
(b) [17, Exercise 4.3.43, p. 304] f2 (x) =
1 − x2

(c) [17, Exercise 4.3.45, p. 304] f3 (x) = x2 + 1 − x
(d) [17, Exercise 4.3.47, p. 304] f4 (x) = ln(1 − ln x)
1
(e) [17, Exercise 4.3.49, p. 304] f5 (x) = e− x+1

Solution:
µ ¶ µ ¶
3 3
(a) (cf. [19, Exercise 4.3.39, p. 132]) Since f1 (x) = x 1 + , and since 1 + →
x x
1 as either x → ∞ or x → −∞, lim f1 (x) = ±∞, so there are no horizontal
x→±∞
asymptotes. All limits of f1 (x) as x approaches any (finite) point are finite,
so there are no vertical asymptotes either. (However, the graph of f1 does
have a vertical tangent at x = −3.)
(b) [19, Exercise 4.3.43, p. 133]. The derivative is defined everywhere in the
domain of the function, and vanishes when x = 0, so x = 0 is the unique
“critical number”.
Information for Students in MATH 140 2008 09 3061

(c) [19, Exercise 4.3.45, p. 134]. The derivative is defined everywhere, and never
vanishes, so there are no “critical numbers”.
(d) [19, Exercise 4.3.47, p. 134]. The derivative is defined everywhere in the
domain of the function, and never vanishes, so there are no “critical numbers”.
(e) [19, Exercise 4.3.49, p. 134]. The derivative is defined everywhere in the
domain of the function, and never vanishes, so there are no “critical numbers”.

6.3.6 Sixth 2000/2001 Problem Assignment, with Solutions


1. In each of the following cases you are to find the limit, if it exists. Since the subject
of [17, §4.4] is l’Hôpital’s Rule, you should try to use that rule, if it is possible.
However, if l’Hôpital’s Rule cannot be applied, explain why. Mathematicians usu-
ally regard the avoidance of l’Hôpital’s Rule as a challenge; try to find another way
to evaluate those limits where you use the Rule.
ex − 1
(a) [17, Exercise 4.4.9, p. 312] lim
x→0 sin x
ln x
(b) [17, Exercise 4.4.15, p. 312] lim
x→∞ x
ln x
(c) [17, Exercise 4.4.17, p. 312] lim+
x→0 x
1 − cos x
(d) [17, Exercise 4.4.27, p. 312] lim
x→0 x2
2x − arcsin x
(e) [17, Exercise 4.4.37, p. 312] lim
x→0 2x + arccos x
2x − arcsin x
(f) [17, Exercise 4.4.38, p. 312] lim
x→0 2x + arctan x
³ √ ´
(g) [17, Exercise 4.4.51, p. 312] lim x − x2 − 1
x→∞
³ 1
´
(h) [17, Exercise 4.4.57, p. 312] lim (1 − 2x) x
x→0

Solution: We shall follow the notation of the Student Solution Manual [19], and
H
write = when we are applying l’Hôpital’s Rule.

(a) [19, Exercise 4.4.9, p. 138]


ex − 1 H ex
lim = lim
x→0 sin x x→0 cos x
Information for Students in MATH 140 2008 09 3062

lim ex e0
x→0
= = =1
lim cos x cos 0
x→0
ex − 1 ex − 1 x
lim = lim ·
x→0 sin x x→0 x sin x
ex − e0 x
= lim · lim (Product Law)
x→0 x x→0 sin x
¯
d x ¯¯
= e ·1
dx ¯x=0

= e0 = 1

(b) [19, Exercise 4.4.15, p. 138].


(c) L’Hôpital’s rule cannot be used, since the limit of the numerator is (negatively)
infinite, while the limit of the denominator is 0. As x → 0+ , ln x → −∞,
1 1
while → +∞, so the product ln x · approaches −∞.
x x
(d) [19, Exercise 4.4.27, p. 138]. Here is one way to avoid using l’Hôpital’s Rule:
µ ¶
1 − cos x 1 − cos x 1 + cos x
lim = lim ·
x→0 x2 x→0 x2 1 + cos x
µ ¶ õ ¶2 !
1 − cos2 x 1 sin x 1
= lim · = lim ·
x→0 x2 1 + cos x x→0 x 1 + cos x
µ µ ¶¶2
sin x 1
= lim ·
x→0 x lim (1 + cos x)
x→0
1 1
= 12 · =
1+1 2
(e) [19, Exercise 4.4.37, p. 138].
(f)
1
2− √
2x − arcsin x H 1 − x2
lim = lim
x→0 2x + arctan x x→0 1
2+
1 + x2

2 − lim 1 1 − x2
x→0
=
1
2 + lim
x→0 1 + x2
Information for Students in MATH 140 2008 09 3063

2−1 1
= =
2+1 3
It is possible to avoid the Rule, but the result looks very much like what we
have proved above:
arcsin x − arcsin 0
2x − arcsin x 2−
lim = lim x−0
x→0 2x + arctan x x→0 arctan x − arctan 0
2+
x−0
arcsin x − arcsin 0
2 − lim
=
x→0 x−0
arctan x − arctan 0
2 + lim
x→0 x−0
¯
d ¯
2− arcsin x¯¯
dx
= ¯ x=0
d ¯
2+ arctan¯¯
dx x=0

1
2− √
1 − 02 1
= =
1 3
2+
1 + 12

(g) [19, Exercise 4.4.51, p. 139].


(h) [19, Exercise 4.4.57, p. 139].

2. Discuss each of these functions under the following headings, using the guidelines
of the same names in [17, pp. 315–316]

A. Domain
B. Intercepts
C. Symmetry
D. Horizontal or Vertical Asymptotes. (Do not attempt to investigate slant
asymptotes.)
E. Intervals of Increase or Decrease
F. Local Maximum and Minimum Values
G. Concavity and Points of Inflection
Information for Students in MATH 140 2008 09 3064

Then sketch the graph of the function.

(a) [17, Exercise 4.5.5, p. 321] f1 (x) = x4 + 4x3


1
(b) [17, Exercise 4.5.17, p. 321] f2 (x) = 3
x −x
p
(c) [17, Exercise 4.5.29, p. 321] f3 (x) = x + |x|
(d) [17, Exercise 4.5.47, p. 322] f4 (x) = ln(x2 − x)
(e) [17, Exercise 4.5.39, p. 322] f5 (x) = sin(2x) − 2 sin x

Solution:

(a) [19, Exercise 4.5.5, p. 142]


(b) [19, Exercise 4.5.17, p. 145]
(c) [19, Exercise 4.5.29, p. 147]
(d) [19, Exercise 4.5.47, p. 150]
(e) [19, Exercise 4.5.39, p. 148]

3. Before attempting these problems, try [17, Exercise 4.7.7, p. 335]; then compare
your solution with that in [19, Exercise 4.7.7, p. 162].

(a) [17, Exercise 4.7.11, p. 335] If 1200 cm2 of material is available to make a box
with a square base and an open top, find the largest possible volume of the
box.
(b) [17, Exercise 4.7.15, p. 335] Showing all your work, use the calculus to find
the point on the line y = 4x + 7 that is closest to the origin. (Use the calculus,
even though you may know methods for solving this problem that require no
calculus.)
(c) [19, Exercises 4.7.25 and 4.7.27, p. 162] A right circular cylinder is inscribed in
a sphere of radius r. Showing all your work, find the largest possible volume
and the largest possible surface area of such a cylinder. (You may assume
that the cylinder has been inscribed so that its axis passes through the centre
of the sphere.)

Solution:

(a) [19, Exercise 4.7.11, p. 162]


(b) [19, Exercise 4.7.15, p. 163]
Information for Students in MATH 140 2008 09 3065

(c) The solutions in [19, Exercises 4.7.25 and 4.7.27, p. 164] use, as the variable,
either the half-height or the radius of the base of the inscribed cylinder. You
might wish to try to solve the problem another way, using, as your variable,
the angle subtended at the centre of the sphere by the radius of the base.

4. Showing all your work, find the most general antiderivative of the following func-
tions. Check your answer by differentiation.

(a) [17, Exercise 4.10.3, p. 356] g1 (x) = 1 − x3 + 5x5 − 3x7


√ √
(b) [17, Exercise 4.10.7, p. 356] g2 (x) = x + 3 x
− 12
(c) [17, Exercise 4.10.15, p. 356] g3 (x) = 2x + 5 (1 − x2 )

Solution:

(a) [19, Exercise 4.10.3, p. 175]


(b) [19, Exercise 4.10.7, p. 175]
(c) [19, Exercise 4.10.15, p. 175]

5. Showing all your work, find the functions that have the listed properties. Check
your answers by differentiation and substitution in the differential equation.

(a) [17, Exercise 4.10.19, p. 356] h001 (x) = 6x + 12x2


√ 1
(b) [17, Exercise 4.10.27, p. 356] h02 (x) = 3 x − √ , h2 (1) = 2.
x
1
(c) [17, Exercise 4.10.39, p. 357] h003 (x) = 3 , x > 0, h3 (1) = 0 = h3 (2).
x
Solution:

(a) [19, Exercise 4.10.19, p. 175]


(b) [19, Exercise 4.10.27, p. 176]
(c) [19, Exercise 4.10.39, p. 176]

6.4 Fall 2006 Written Assignments, with Draft Solutions


6.4.1 Draft Solutions to W1 , First Written Assignment, Fall, 2006
Distribution Date: Released to students on October 21st, 2006
Draft version, subject to correction
Your completed solution to this assignment was to be submitted, with a copy of this
question sheet at your Tutorial, during the week September 26-30, 2006. You were
Information for Students in MATH 140 2008 09 3066

asked to “Please slip the assignment into your folded quiz answer paper to Quiz Q1 .
All materials must bear your name and/or student number. No other method of
submission is acceptable.”

The assignment question You are given formulæ for functions f and g.

1. Determine a formula for each of the functions f ◦ f , f ◦ g, g ◦ f , g ◦ g, simplified as


much as you can.

2. Determine for each of the functions f , g, f ◦ f , f ◦ g, g ◦ f , g ◦ g, its domain. Where


the domain is not all of R, you must explain your work — it’s not enough just to
write down the answers.

TUTORIAL DAY f (x) g(x)



MONDAY, 25 Sept., 2006 x2 − 4 x−1
1 1
WEDNESDAY, 27 Sept., 2006 x− 4x x
−x 1
THURSDAY, 28 Sept., 2006 e ln x
FRIDAY, 29 Sept., 2006 sin x1 arcsin x
2

While you are expected to submit solutions for only the problem assigned to your tutorial
day, you should also try the other 3 (but not hand in solutions).

Solutions

Marking Scheme: TOTAL = 20 MARKS

MONDAY: 1. Since f is a polynomial, there are no real numbers where it is unde-


fined. Domain(f ) = R

2. Since g(x) = x − 1, it is defined only where the argument of the square root
is non-negative, i.e., where x ≥ 1: this is the Domain(g).
3. f has been shown to be defined for all x. We compose it with itself, finding
that f (f (x)) = (x2 − 4)2 − 4 = x4 − 8x2 + 12. This is also a polynomial, which
is defined for all x, so Domain(f ) = R.
4. We have already shown that the domain of g is the interval [1, ∞). In that
interval
f (g(x)) = |x − 1| − 4 = x − 5 .
This last is the formula for f ◦g, but it is valid only where x ≥ 1. Even though
the function x − 5 has all of R as its domain, Domain(f ◦ g) = [1, ∞).
Information for Students in MATH 140 2008 09 3067

5. The domain of f is unrestricted. Hence f (x) − 1√is defined for all x, and is
always equal to x√2 − 5. But we cannot evaluate x2 − 5 unless x2 − 5 ≥ 0,
i.e., unless |x| ≥ 5.
√ √
Domain(g ◦ f ) = (−∞, − 5] ∪ [ 5, ∞)

6. When we compose g with itself, the first application of g will be defined only
on
p√the domain of g, i.e., for x ≥ 1. But the second application of g will give
x − 1 − 1, and will not be defined unless the argument
√ of the last square
root applied
¡√ is
¢2non-negative. Thus we require that x − 1 ≥ 1, which implies
2
that x − 1 ≥ 1 , i.e., that |x − 1| ≥ 1, which is equivalent to x ≥ 2 or
x ≤ 0. We have already required that x ≥ 1. This condition is inconsistent
with x ≤ 0, and is implied by x ≥ 2; hence
Domain(g ◦ g) = {x|x ≥ 2} = [2, ∞) .

function formula domain


2
f √x −4 R
g x−1 [1, ∞)
4 2
f ◦f x − 8x + 12 R
f ◦g √x − 5 [1,
√∞) √
g◦f 2
p√ x − 5 R − (− 5, + 5)
g◦g x−1−1 [2, ∞)

Table 10: Summary of solutions to Fall, 2006 written assignment W1 , MONDAY version

WEDNESDAY: 1. In the development of the real number system we do not assign


a meaning to 10 ; we can’t! No matter what real number r we would want to
choose as the value of 01 , we find that one of our other basic properties of
the real number system would be violated if we used r as the value of this
quotient. We resolve this issue by restricting fractions ab to cases where b 6= 0.
Thus Domain(g) = R − {0}.
1
2. The same reasoning tells us that 4x is defined only for x 6= 0; when we
subtract this function from the function x, which is defined everywhere, we
obtain another function defined away from 0; thus Domain(f ) = R − {0}.
3. We wish to compose f with itself. For the first application we require that
x 6= 0. The value of that first application, i.e.,
¡ ¢¡ ¢
1 4x2 − 1 (2x − 1)(2x + 1) 1 x − 12 x + 12
x− = = = ·
4x x x 4 x
Information for Students in MATH 140 2008 09 3068

must not be equal to 0 if it is to be the point where we apply the function f


again. Thus we have to insist that x be different from each of ± 12 , and also
different from 0:
½ ¾
1 1
Domain(f ◦ f ) = R − − , 0, + .
2 2
In that domain the function has the following formula:
µ ¶
1 1 16x4 − 12x2 + 1
(f ◦ f )(x) = x − − ¡ 1
¢ = .
4x 4 x − 4x 4x(4x2 − 1)

4. To follow g by f we first require that x 6= 0, in order to apply g. The value


of g(x) is never 0, so there is no restriction for the application of f . Thus the
domain of f ◦ g is R − {0}, and the formula is
1 1 4 − x2
(f ◦ g)(x) = − = .
x 4 · x1 4x

5. To follow f by g, we need to arrange first that f be defined: that needs x 6= 0.


Then we need to arrange that the value of f be non-zero, in order that it lie
2
in the domain of g. The value of f (x) in general is 4x x−1 = (2x−1)(2x+1)
x
, so it
1
will be non-zero if and only if x is different from ± 2 . Thus
½ ¾
1 1
Domain(g ◦ f ) = R − − , 0, + .
2 2
A simplified formula for the function is given by
1 4x
(g ◦ f )(x) = 1 = 2
.
x − 4x 4x − 1
Note that, even though this rational function is defined when x = 0, the
1
formula does not apply to g ◦ f at that point, since 4x is not defined there.
6. To apply g twice in succession we obtain
1
(g ◦ g)(x) = g(g(x)) = 1 = x.
x

1
However, the equation does not apply when x = 0, since x
is not defined
there. The domain of the composition g ◦ g is R − {0}.
THURSDAY: 1. The exponential is defined for all exponents. Whatever value we
choose for x, −x is in the domain of the exponential, so the domain of f is R.
Information for Students in MATH 140 2008 09 3069

function formula domain


1
f x − 4x R − {0}
1
g R − {0}
x
16x4 −12x2 +1
© ª
f ◦f 4x(4x2 −1)
R − − 21 , 0, + 21
4−x2
f ◦g 4x
R
© −1{0} 1 ª
4x
g◦f 4x2 −1
R − − 2 , 0, + 2
g◦g x R − {0}

Table 11: Summary of solutions to Fall, 2006 written assignment W1 , WEDNESDAY


version

2. The logarithm is defined only for positive numbers. So we cannot venture


outside of (0, ∞). For x in the interval (0, 1) the logarithm is negative; at
x = 1 it is equal to 0; and for x > 1 it is positive. Since we will be taking the
reciprocal, ln1x , we have to exclude the point 1. Thus

Domain(f ) = (0, 1) ∪ (1, ∞) .

3. Since the domain of f is the whole line, we can apply the function a second
time indiscriminately, and we have

Domain(f ◦ f ) = R .

4. To follow g by f we first must arrange that g be applicable. That requires


that x be in the union (0, 1) ∪ (1, ∞). It matters not what the value of g is
at x, since f is defined everywhere. Hence

Domain(f ◦ f ) = (0, 1) ∪ (1, ∞) .

The formula does not admit simplification,


1
(f ◦ g)(x) = e− ln x .

5. f and g are mutual inverses: If we follow one by the other we obtain the
identity function, i.e., x. The logarithm of f (x) is, therefore −x. Thus far
there is no restriction on x. However, in calculating g, we wish to take a
reciprocal: for this we require that the logarithm be different from 0: −x 6= 0
if and only if x 6= 0. Thus

Domain(g ◦ f ) = R − {0} .
1
The formula for the composition function is simply −x
or − x1 .
Information for Students in MATH 140 2008 09 3070

6. We wish to compose g with itself. In the first application we have to require


that x be contained in the union (0, 1) ∪ (1, ∞). The value of this function
must be restricted: it must be in the domain of the natural logarithm function,
and it must be such that the natural logarithm there is non-zero. To be in the
domain of the function ln we require that ln1x be positive, equivalently that
ln x be positive, equivalently that x > 1. And, we have to exclude that point
x = e, since g(e) = 1, and a second application of g would not be possible,
since ln 1 = 0 and 01 is undefined. Thus

Domain(g ◦ g) = (1, e) ∪ (e, ∞) .

The formula for the function does not admit much simplification:
1 1
(g ◦ g)(x) = ¡ 1 ¢ =− .
ln ln x
ln ln x

function formula domain


−x
f e R
1
g ln x
(0, 1) ∪ (1, ∞)
−e−x
f ◦f e R
− ln1x
f ◦g e (0, 1) ∪ (1, ∞)
g◦f − x1 R − {0}
1
g◦g − ln ln x (1, e) ∪ (e, ∞)

Table 12: Summary of solutions to Fall, 2006 written assignment W1 , THURSDAY


version

FRIDAY: 1. x1 is defined for all x 6= 0. Then the sine is defined everywhere, so the
domain of f is R − {0}.
2. The arcsine function is defined only on the interval [−1, 1]. That is the domain
of g.
3. To compose f with itself the first application requires only that x 6= 0. For
a second application we require only that sin x1 6= 0, which is equivalent to
1
x
6= nπ where n is any integer. Hence we have
½ ¾
1 1 1
Domain(f ◦ f ) = R − 0, ± , ± , ± , . . . .
π 2π 3π
Information for Students in MATH 140 2008 09 3071

4. To follow g by f we first must restrict x to be in the domain of g, i.e.,


[−1 ≤ x ≤ 1]. Then we must also restrict x so that the value g(x) 6= 0, i.e.,
that arcsin x 6= 2(0), which is equivalent to excluding the point x = 0. Hence

Domain(f ◦ g) = [−1, 0) ∪ (0, 1] .

The formula is simply


µ ¶
2
(f ◦ g)(x) = sin .
arcsin x

and does not admit significant simplifications.


5. The values of the sine function are always in the interval [−1, 1], and the
arcsine is defined at all such points. To follow f by g we must, however, first
restrict x to be non-zero, in order that f be applicable. Hence

Domain(g ◦ f ) = R − {0} .
arcsin(sin 1
)
The formula (g ◦ f )(x) = 2
x
does admit simplifications. For example,
when |x| > π2 , (g ◦ f )(x) = 1
2x
.
£ π π¤
6. The arcsine function takes£ its values
¤ in the interval − 2 , 2 ; hence g takes
its values in the interval − π4 , π4 , which is entirely contained in the interval
[−1, 1]. This means that a second application of the arcsine function is always
possible. Hence the domain of g ◦ g is still [−1, 1].

function formula domain


1
f sin x R − {0}
arcsin x
g ¡2 ¢ © 1 ª
f ◦f sin¡ sin x1 ¢ R − {0} − ± nπ |n = 1, 2, 3, 4, . . .
2
f ◦g sin arcsin x
[−1, 0) ∪ (0, 1]
arcsin(sin x1 )
g◦f 2
R − {0}
arcsin( arcsin x
)
g◦g 2
2
[−1, 1]

Table 13: Summary of solutions to Fall, 2006 written assignment W1 , FRIDAY version

6.4.2 Draft Solutions to W2 , Second Written Assignment, Fall, 2006


Release Date: Published on WebCT on November 11th, 2006.
Information for Students in MATH 140 2008 09 3072

Students’ completed solutions to this assignment were to be submitted, with a copy of


this question sheet at Tutorials during the week October 16-20, 2006. Students were
asked to “Please slip the assignment into your folded quiz answer paper to Quiz Q2 .
All materials must bear your name and/or student number. No other method of
submission is acceptable.”

The assignment questions

1. This first problem concerns a pair of functions that may not be discussed at the lec-
tures, except incidentally. The purpose of the problem is to allow you to investigate
the functions yourself. You are not asked for rigorous proofs in this problem.
This problem involves, in part, sketching some graphs. The functions in question
have discontinuities. Please use the following convention. If f has a discontinuity
at x = a, please enlarge the point (a, f (a)); this convention is particularly useful
where a function is, for example, continuous from the right but not from the left —
in that way the reader can see immediately what type of discontinuity is present.
Another way of showing such discontinuities is through judicious use of square and
round brackets. For example, the graph of
½
−1 if −4 ≤ x < 0
f (x) =
1 if 0 ≤ x ≤ 4

could be sketched as in Figure 23 on page 3072. You are expected to solve this

6 6
s [

- -

)
or

Figure 23: Showing a discontinuity in a graph

problem by using information from your graphs — you are not expected to provide
rigorous justifications for all statements. You do not need to use graph paper —
just try to have your graphs approximately to scale, so that the properties of the
functions can be clearly visible.
Information for Students in MATH 140 2008 09 3073

Definition 6.1 The floor function bxc is defined as follows: bxc is the largest
integer n such that n ≤ x. (Your textbook [7, Example 10, p. 110] calls this
function the Greatest Integer Function, and denotes it by [[x]].)
The ceiling function dxe is defined to be the smallest integer n such that n ≥ x.
(This function used to be called the Least Integer Function.)

(a) Sketch a graph of each of the functions bxc, dxe for −2 < x < 2. (You don’t
need to use graph paper for these problems, but you may if you wish.)
(b) Sketch graphs of the functions f (x) = b2xc + d2xe and g(x) = b2xc − d2xe
for −1 ≤ x ≤ 1.
(c) Describe the following sets:
½ ¯ ¾
¯
i. a ¯¯ lim− bxc does not exist =
x→a
½ ¯ ¾
¯
ii. a ¯¯ lim− dxe does not exist =
x→a
½ ¯ ¾
¯
iii. a ¯¯ lim+ bxc does not exist =
x→a
½ ¯ ¾
¯
iv. a ¯¯ lim+ dxe does not exist =
x→a
½ ¯ ¾
¯
v. a ¯¯ lim− b2xc = 2a =
x→a
½ ¯ ¾
¯
vi. a ¯¯ lim− bxc = lim− dxe =
x→a x→a
½ ¯ ¾
¯
vii. a ¯¯ lim− bxc = lim+ bxc =
x→a x→a

(The “set-builder notation” used here is described in [7, Appendix A, p. A3].)

2. Prove the statement assigned for your tutorial day by using the Principle of Math-
ematical Induction. For this problem you are expected to provide a very careful
proof. (One version of this problem was on the final examination in MATH 140
2005 09.)

Students whose tutorial is on Monday: Let f (x) = x2 ex . Prove carefully by


mathematical induction that
dn f ¡ ¢
n
(x) = x2 + 2nx + (n − 1)n · ex
dx
for all positive integers n.
Information for Students in MATH 140 2008 09 3074

Students whose tutorial is on Wednesday: Let f (x) = x2 e−x . Prove care-


fully by mathematical induction that
dn f n
¡ 2 ¢ −x
(x) = (−1) x − 2nx + (n − 1)n ·e
dxn
for all positive integers n.
Students whose tutorial is on Thursday: Let f (x) = 2xe2x . Prove carefully
by mathematical induction that
dn f
(x) = 2n (2x + n) · e2x
dxn
for all positive integers n.
Students whose tutorial is on Friday: Let f (x) = 2xe−2x . Prove carefully by
mathematical induction that
dn f
(x) = (−2)n (2x − n) · e−2x
dxn
for all positive integers n.

Solutions

1. (a) The graph of bxc consists of horizontal line segments of length 1, each includ-
ing its left end-point but not its right; on the x-axis the graph includes the
interval [0, 1), and the next portion of the graph to the right is 1 unit higher,
etc.
The graph of dxe also consists of horizontal line segments of length 1, this
time each including its right end-point but not its right; on the x-axis the
graph includes the interval (−1, 0], and the next portion of the graph to the
right is 1 unit higher, etc.
(b) The graph of bxc + dxe consists of open line segments of length 1 lacking both
end points, together with points at even heights — the value of the function
at an integer n is 2n; the pattern is centred around the origin, which is a point
of the graph. The given function was, however b2xc + d2xe: the effect is to
compress the previous graph horizontally — the line segments have length 12 .
The function b2xc − d2xe is mostly constant: its value is −1 at every point
which is not half an integer; at the half-integers the function value is 0.
Information for Students in MATH 140 2008 09 3075

(c) To solve this problem, observe from the graphs or otherwise that

lim bxc = bac (217)


x→a+
½
bac if a is not an integer
lim− bxc = (218)
x→a bac − 1 if a is an integer
lim dxe = dae (219)
x→a−
½
dae if a is not an integer
lim+ dxe = (220)
x→a dae + 1 if a is an integer
½ ¯ ¾
¯
i. a ¯¯ lim− bxc does not exist is empty. The floor function is constant
x→a
“near” any point x; for points which are not integers, the constant value
is the same on both sides, and the limit of the function is the function
value. At integers the constant value on the left side is 1 unit less than
the constant value on the right. This means that the limit from either
side exists, since, in a one sided “neighbourhood” of the point on one side
the function behaves “locally” like a constant. Thus the answers to this
question and, by analogous reasoning, the next 3 are all the same: the
set ¯of points where the one-sided limits do not exist is empty!
½ ¾
¯
ii. ¯
a ¯ lim− dxe does not exist is empty.
x→a
½ ¯ ¾
¯
iii. ¯
a ¯ lim+ bxc does not exist is empty.
x→a
½ ¯ ¾
¯
iv. ¯
a ¯ lim+ dxe does not exist is empty.
x→a
½ ¯ ¾
¯
v. ¯
a ¯ lim− b2xc = 2a = By (218) the defining condition for this set is
x→a

b2ac = 2a when 2a is not an integer; and


b2ac − 1 = 2a when 2a is an integer.
When 2a is not an integer, b2ac < 2a; when 2a is an integer, b2ac = 2a.
Thus equality cannot hold at any time!
½ ¯ ¾
¯
¯
vi. The defining condition for a ¯ lim− bxc = lim− dxe is
x→a x→a

bac = dae if a is not an integer; and


bac − 1 = dae if a is an integer,
conditions that can never hold, since, in the first case

bac < a < dae


Information for Students in MATH 140 2008 09 3076

and, in the second case,


bac − 1 = a − 1 < a = dae .
vii. The condition lim− bxc = lim+ bxc is satisfied precisely when a is not an
x→a x→a
integer.
2. The second problem involved the Principle of Mathematical Induction. While this
topic was discussed in detail in MATH 140 2005 09, we did not have time for
a thorough treatment in the lectures this year. Nevertheless, we believe it is an
appropriate topic for a written assignment: the students had to read up about
the principle from their textbook, look up worked examples, and then answer the
question as posted. Because the topic has not seen significant discussion in the
lectures, I would like the grading to be generous; students have been assured that
this topic will not be on the final examination this year.
Students whose tutorial is on Monday: f (x) = x2 ex . Let Sn denote the state-
ment
dn f ¡ 2 ¢ x
(x) = x + 2nx + (n − 1)n ·e , (221)
dxn
for positive integers n.
“Anchor” of Induction, or “Base Step”: Applying the Product Rule of
Differentiation, we obtain
df
(x) = (2x)ex + (x2 )ex
dx ¡ ¢
= x2 + 2(1)x + (0)(1) ex
which proves S1 .
Induction Hypothesis: Assume that Sn is true.
Induction Step: Then
µ ¶
dn+1 d dn f
f (x) = (x)
dxn+1 dx dxn
d ©¡ 2 ¢ ª
= x + 2nx + (n − 1)n ex
dx
by the Induction Hypothesis
¡ ¢
= (2x + 2n) ex + x2 + 2nx + (n − 1)n ex
by the Product Rule
¡ 2 ¢
= x + (2n + 2)x + ((n − 1)n + 2n) ex
¡ ¢
= x2 + 2(n + 1)x + n(n + 1) ex
proving Sn+1 .
Information for Students in MATH 140 2008 09 3077

Conclusion: Hence, by Mathematical Induction, Sn is true for all positive


integers n.
Students whose tutorial is on Wednesday: f (x) = x2 e−x . Let Sn denote the
statement
dn f ¡ 2 ¢ −x
(x) = x − 2nx + (n − 1)n ·e , (222)
dxn
for positive integers n.
“Anchor” of Induction, or “Base Step”: Applying the Product Rule of
Differentiation, we obtain
df
(x) = (2x)e−x − (x2 )e−x
dx ¡ ¢
= −x2 + 2(1)x + (0)(1) e−x
¡ ¢
= (−1)1 x2 − 2(1)x + (0)(1) e−x

which proves S1 .
Induction Hypothesis: Assume that Sn is true.
Induction Step: Then
µ ¶
dn+1 d dn f
f (x) = (x)
dxn+1 dx dxn
d © ¡ ¢ ª
= (−1)n x2 − 2nx + (n − 1)n e−x
dx
by the Induction Hypothesis
¡ ¢
= (−1)n (2x − 2n) e−x − (−1)n x2 − 2nx + (n − 1)n e−x
by the Product Rule
¡ ¢
= (−1)n+1 x2 − (2n + 2)x + ((n − 1)n + 2n) e−x
¡ ¢
= (−1)n+1 x2 − 2(n + 1)x + n(n + 1) e−x

proving Sn+1 .
Conclusion: Hence, by Mathematical Induction, Sn is true for all positive
integers n.
Students whose tutorial is on Thursday: f (x) = 2xe2x . Let Sn denote the
statement
dn f
n
(x) = 2n (2x + n) · e2x , (223)
dx
for positive integers n.
Information for Students in MATH 140 2008 09 3078

“Anchor” of Induction, or “Base Step”: Applying the Product Rule of


Differentiation, we obtain
df
(x) = 2e2x + 2(2x)e2x
dx
= 21 (2x + 1)e2x

which proves S1 .
Induction Hypothesis: Assume that Sn is true.
Induction Step: Then
µ ¶
dn+1 d dn f
f (x) = (x)
dxn+1 dx dxn
d © n ª
= 2 (2x + n)e2x
dx
by the Induction Hypothesis
= 2 (2) e2x + 2n (2x + n) · 2 · e2x
n

by the Product Rule


= 2 (4x + (2n + 2)) e2x
n

= 2n+1 (2x + (n + 1)) e2x

proving Sn+1 .
Conclusion: Hence, by Mathematical Induction, Sn is true for all positive
integers n.
Students whose tutorial is on Friday: f (x) = 2xe−2x . Let Sn denote the
statement
dn f
(x) = 2n (2x − n) · e−2x , (224)
dxn
for positive integers n.
“Anchor” of Induction, or “Base Step”: Applying the Product Rule of
Differentiation, we obtain
df
(x) = 2e−2x − 2(2x)e−2x
dx
= 21 (−2x + 1)e−2x = (−2)1 (2x − 1)e−2x

which proves S1 .
Induction Hypothesis: Assume that Sn is true.
Information for Students in MATH 140 2008 09 3079

Induction Step: Then


µ ¶
dn+1 d dn f
f (x) = (x)
dxn+1 dx dxn
d © ª
= (−2)n (2x − n)e−2x
dx
by the Induction Hypothesis
= (−2)n (2) e−2x + (−2)n (2x − n) · (−2) · e−2x
by the Product Rule
= (−2)n (−4x + (2n + 2)) e−2x
= (−2)n+1 (2x − (n + 1)) e−2x

proving Sn+1 .
Conclusion: Hence, by Mathematical Induction, Sn is true for all positive
integers n.

6.4.3 Draft Solutions to W3 , Third Written Assignment, Fall, 2006


Release Date: November 11th, 2006; subject to correction.
Your completed solution to this assignment was to be submitted, with a copy of this
question sheet at your Tutorial, during the week October 30 - November 03, 2006. You
were asked to “Please slip the assignment into your folded quiz answer paper to Quiz
Q3 . All materials must bear your name and/or student number. No other method of
submission is acceptable.”

The assignment questions

1. A function y = y(x) is defined implicitly by the following equation. You are,


showing all your work, to determine the values of y 0 (x) and y 00 (x) at the given
point.
Tutorial Day Defining Equation Evaluate at the Point
Monday x3 + y 4 = 2xy (x, y) = (1, 1)
Wednesday x4 + y 3 = 2xy (x, y) = (1, 1)
Thursday x3 − y 4 = 2xy (x, y) = (−1, 1)
4 3
Friday −x + y = 2xy (x, y) = (1, −1)

2. The six hyperbolic functions are defined in terms of sinh and cosh, which, in turn,
are defined in terms of exponentials. These six functions satisfy identities that
Information for Students in MATH 140 2008 09 3080

resemble properties of the trigonometric functions. Beginning with each function


expressed in terms of exponentials, and showing all your work, prove the identity
assigned below to your Tutorial day:

Tutorial Day Identity to be Proved


Monday cosh(x + y) = cosh x · cosh y + sinh x · sinh y
tanh x − tanh y
Wednesday tanh(x − y) =
1 − tanh x · tanh y
Thursday d
dx
tanh x = sech2 x
d
Friday dx
csch x = −csch x · coth x

Solutions

1. Students whose tutorial is on Monday: Differentiating the defining equation


“implicitly” with respect to x, we obtain

3x2 + 4y 3 · y 0 = 2(y + x · y 0 ) , (225)

which we evaluate at (x, y) = (1, 1), to obtain y 0 (1) = − 21 . We differentiate


equation (225) implicitly with respect to x to obtain

6x + 12y 2 · (y 0 )2 + 4y 3 · y 00 = 2(y 0 + y 0 + xy 00 ) (226)


¡ ¢
and substitute the values (x, y, y 0 ) = 1, 1, − 21 , obtaining that y 00 (1) = −4.
(We could have solved equation (225) for

3x2 − 2y
y0 = ,
2x − 4y 3
and then differentiated with respect to x to obtain

(6x2 − 24xy 3 + 4y) + (−16y 3 + 36x2 y 2 − 4x)y 0


y 00 (x) = .
(2x − 4y 3 )2
¡ ¢
Then we could again have substituted (x, y, y 0 ) = 1, 1, − 21 . That, however,
would not have made the calculations any easier.)
Students whose tutorial is on Wednesday: Differentiating the defining equa-
tion “implicitly” with respect to x, we obtain

4x3 + 3y 2 · y 0 = 2(y + x · y 0 ) , (227)


Information for Students in MATH 140 2008 09 3081

which we evaluate at (x, y) = (1, 1), to obtain y 0 (1) = −2. We differentiate


equation (227) implicitly with respect to x to obtain
12x2 + 6y 2 · (y 0 )2 + 3y 2 · y 00 = 2(y 0 + y 0 + xy 00 ) (228)
0 00
and substitute the values (x, y, y ) = (1, 1, −2), obtaining that y (1) = −44.
(We could have solved equation (227) for y 0 and then differentiated with re-
spect to x; that, however, would not have made the calculations any easier.)
Students whose tutorial is on Thursday: Differentiating the defining equation
“implicitly” with respect to x, we obtain
3x2 − 4y 3 · y 0 = 2(y + x · y 0 ) , (229)
0 1
which we evaluate at (x, y) = (−1, 1), to obtain y (1) = 2
. We differentiate
equation (229) implicitly with respect to x to obtain
6x − 12y 2 · (y 0 )2 − 4y 3 · y 00 = 2(y 0 + y 0 + xy 00 ) (230)
¡ ¢
and substitute the values (x, y, y 0 ) = 1, 1, 21 , obtaining that y 00 (1) = − 11
2
.
0
(We could have solved equation (229) for y and then differentiated with re-
spect to x; that, however, would not have made the calculations any easier.
Students whose tutorial is on Friday: Differentiating the defining equation “im-
plicitly” with respect to x, we obtain
−4x3 + 3y 2 · y 0 = 2(y + x · y 0 ) , (231)
which we evaluate at (x, y) = (1, −1), to obtain y 0 (1) = 2. We differentiate
equation (231) implicitly with respect to x to obtain
−12x2 + 6y · (y 0 )2 + 3y 2 · y 00 = 2(y 0 + y 0 + xy 00 ) (232)
and substitute the values (x, y, y 0 ) = (1, 1, 2), obtaining that y 00 (1) = 44. (We
could have solved equation (231) for y 0 and then differentiated with respect
to x; that, however, would not have made the calculations any easier.
2. Students whose tutorial is on Monday: I believe it will be easier to start this
proof on the right side of the identity, which is the more complicated.
cosh x · cosh y + sinh x · sinh y
ex + e−x ey + e−y ex − e−x ey − e−y
= · + ·
2 2 2 2
(ex + e−x ) (ey + e−y ) + (ex − e−x ) (ey − e−y )
=
4
x+y x−y −x+y
(e +e +e + e−x−y ) + (ex+y − ex−y − e−x+y + e−x−y )
=
4
2ex+y + 2e−(x+y)
= = cosh(x + y) .
4
Information for Students in MATH 140 2008 09 3082

¤
Students whose tutorial is on Wednesday: In this example as well, I believe
it will be easier to start this proof on the right side of the identity, which is
the more complicated.
tanh x − tanh y
1 − tanh x · tanh y
sinh x sinh y

cosh x cosh y
=
sinh x sinh y
1− ·
cosh x cosh y
sinh x · cosh y − cosh x · sinh y
=
cosh x · cosh y − sinh x · sinh y
ex − e−x ey + e−y ex + e−x ey − e−y
· − ·
= x 2 −x y 2 −y 2 2
e +e e +e ex − e−x ey − e−y
· − ·
2 2 2 2
x+y x−y −x+y −x−y x+y
(e +e −e −e ) − (e − ex−y + e−x+y − e−x−y )
=
(ex+y + ex−y + e−x+y + e−x−y ) − (ex+y − ex−y − e−x+y + e−x−y )
2ex−y − 2e−x+y
=
2ex−y + 2e−x+y
ex−y − e−(x−y)
= x−y
e + e−(x−y)
x−y
e − e−(x−y)
sinh(x − y)
= x−y 2 −(x−y) = = tanh(x − y)
e +e cosh(x − y)
2
Students whose tutorial is on Thursday:
 x 
µ ¶ e − e−x µ ¶
d d sinh x d  2  d ex − e−x
tanh x = =  =
dx dx cosh x dx ex + e−x dx ex + e−x
2
(e + e )(e + e ) − (e − e−x )(ex − e−x )
x −x x −x x
=
(ex + e−x )2
(e2x + 2 + e−2x ) − (e2x − 2 + e−2x ) 4
= x −x 2
= x
(e + e ) (e + e−x )2
µ ¶2 µ ¶2
2 1
= x −x
= = (sech x)2 = sech2 x
e +e cosh x
Information for Students in MATH 140 2008 09 3083

Students whose tutorial is on Friday:


µ ¶
d d 1
csch x =
dx dx sinh x
µ ¶
d 2
=
dx ex − e−x
ex + e−x
= −2 x
(e − e−x )2
ex + e−x
= − µ x 2 −x ¶2
e −e
2
µ ¶ µ ¶
1 cosh x
= − · = −csch x · coth x
sinh x sinh x
¤

6.4.4 Draft Solutions to W4 , Fourth Written Assignment, Fall, 2006


Distribution Date: Monday, November 27th, 2006
Completed solutions to this assignment were due to be submitted, with a copy of this
question sheet at Tutorials during the week November 20-24, 2006. Students were
instructed to “Please slip the assignment into your folded quiz answer paper to Quiz
Q4 . All materials must bear your name and/or student number. No other method of
submission is acceptable.”

(x − a)2 − b2
The assignment question Consider the function f (x) = on the interval
(x − a)2 + b2
a − 2b ≤ x ≤ a + b, where a and b are constants determined from your student number,
as follows:
Starting from the left, list the non-zero digits in your student num-
ber. Ignore the first of these. The second is a; the third is b.
(It could happen that a = b.) Showing all your work, modelled on the solution in the
textbook of Example 8, page 284,
1. Find the global (absolute) maximum and minimum values of the function
(x − a)2 − b2
f (x) =
(x − a)2 + b2
using the “Closed Interval Method” on the interval a − 2b ≤ x ≤ a + b.
Information for Students in MATH 140 2008 09 3084

2. Apply the First Derivative Test to any critical points in the interval.
3. Apply the Second Derivative Test to any critical points in the interval.
4. Explain how you know that f does not have a global maximum on the interval
−∞ < x < +∞.
5. Sketch the graph of f , showing the extrema you have found.
Solution:
1.
(x − a)2 − b2 2b2
f (x) = = 1 −
(x − a)2 + b2 (x − a)2 + b2
4(x − a)b2
⇒ f 0 (x) =
((x − a)2 + b2 )2
2
¡ √ ¢¡ √ ¢
4b b − 3(x − a) b + 3(x − a)
⇒ f 00 (x) =
((x − a)2 + b2 )3
Since f 0 is defined for all x, we can set f 0 (x) equal to 0 to find all critical points:
the only solution is x = a; f (a) = −1.
The values of f at the end-points of the given interval are 53 and 0. The global
maximum is attained at x = a − 2b, with the value 35 ; the global minimum is at
x = a, with the value −1.
2. For x < a, f 0 (x) < 0, while, for x > a, f 0 (x) > 0. (Usually we should only
be looking “within a small neighbourhood” of x = a, but, for this function, the
derivative has the same sign throughout the ray to the left of x = a, and the same
sign throughout the ray to the right of x = a.) Thus f 0 changes from negative to
positive at the critical point, and the point must be a local (relative) minimum.
−4b2 (−b)(b) 4
3. f 00 (a) = = > 0, implying that the critical point is a local (relative)
(b2 )3 b2
minimum.
2b 2
4. As x → ±∞, f (x) → 1. But, for all x, 1 − f (x) = (x−a) 2 +b2 > 0. What could the

global maximum be? If you claimed the maximum occurred at some point x = x0 ;
then, by taking x > x0 , I would have a point where f (x) > f (x0 ), and x0 couldn’t
be a maximum point. And you can’t claim the maximum value is 1, since the value
1 is never attained. There is no maximum!
5. see Figure 24 on page 3085
Information for Students in MATH 140 2008 09 3085

0.5

0
-20 -10 0 10 20
x

-0.5

-1

(x − a)2 − b2
Figure 24: Graph of the Function and its horizontal asymptote, y = 1,
(x − a)2 + b2
when a = 2, b = 5
Information for Students in MATH 140 2008 09 3086

7 Some Tests and Quizzes from Previous Years


7.1 Fall 1998 Class Quiz, with Solutions
(This quiz was given to both sections of 189-140A in the middle of October, 1998. No
grades were recorded. The solutions were posted on the Web.)
1. Determine the global maxima and global minima of the function
f (x) = 5x2/3 − x5/3 on the interval −2 ≤ x ≤ 4. (You may take 1.6 as an ap-
1
proximation to 4 3 . You may find it helpful to make a rough sketch, but your
solutions must not depend on that sketch.)
Solution:
2 −1 5 2
f 0 (x) = 5 · · x 3 − · x3
3 3
5(2 − x)
= 1 ,
3x 3
provided x 6= 0. When x = 0, the function lacks a derivative. The derivative
vanishes (i.e. is equal to zero) if and only if x = 2. Thus the list of critical points131
for this function, whose domain is the closed interval [−2, 4], is
Points where there is no derivative:
(0, f (0)) = (0, 0)
Points where the derivative vanishes:
2
(2, f (2)) = (2, 3 · 2 3 )
End-points of the domain of definition:
1
(−2, f (−2)) = (−2, 7 · 4 3 )
2
(4, f (4)) = (4, 1 · 4 3 )
Both the global maximum and the global minimum must be attained at points
among these 4. Comparing the function values — if necessary students could have
1
used the approximation of 1.6 for 4 3 — shows that the maximum occurs at the
1
point (−2, 7 · 4 3 ), and the minimum at the point (0, 0).
(A graph of this function may be found in [31, Example 6 of §3.5].)
131
The definition [31, p. 144] of critical point in the text-book is unclear as to whether or not the
end-points of the interval of definition are to be considered critical . If you choose to consider end-points
as not being included in this definition, then you must include them with the (other) critical points in
your list of candidates for extrema.
Information for Students in MATH 140 2008 09 3087

2. A rectangle, with its base on the x-axis, is constructed so that its upper two vertices
are on the semi-circle x2 + y 2 = 4, y ≥ 0. Determine the maximum possible area
for such a rectangle, and determine where the maximum value is attained.
Solution: There are several ways of approaching this problem; we present two.
(a) A rectangle inscribed in the semicircle must have its upper side parallel to the
x-axis, so the coordinates of the upper vertices
√ may be taken to be (±x, y);
2 2
as x + y = 4, and y √ is non-negative, y = 4 − x2 . Thus the vertices of the
rectangle will be (±x, 4 − x2 ) and (±x, 0). The area, which we may denote
by A(x) is then given by

A(x) = |(2x) · 4 − x2 | .
(The absolute signs are needed since the intention of the problem is that
the area should be non-negative.) We can now approach the problem in two
equivalent ways.
i. We may√ take the domain of A(x) to be −2 ≤ x ≤ 2, since the circle has
radius 4 = 2. Differentiating yields
√ 1 1
A0 (x) = 2 4 − x2 + 2x · √ · (−2x)
2 4 − x2
4 − 2x2
= √
4 − x2
However, this is valid only for x > 0, because of the absolute signs; for
x<0
2x2 − 4
A0 (x) = √ .
4 − x2
Note that there is no derivative at x = 0.
Thus the critical points are
(0, A(0)) = (0, 0)
√ √ √
(− 2, A(− 2)) = (− 2, 4)
√ √ √
( 2, A( 2)) = ( 2, 4)
and the end points,
(−2, f (−2)) = (−2, 0)
(2, f (2)) = (2, 0)
Comparing the values, we√find the maximum of A = 2 to be attained at
both of the points x = ± 2. The minimum value of 0 is attained at 3
points: x = ±2 and x = 0.
Information for Students in MATH 140 2008 09 3088

ii. The preceding approach can be modified, in that we can restrict the
domain to be 0 ≤ x ≤ 2, by the symmetry of the function. In this
approach the only critical points are
√ √ √
( 2, A( 2)) = ( 2, 4)
and the end points,
(0, A(0)) = (0, 0)
(2, A(2)) = (2, 0)
and we obtain the same extreme values.
(b) A more elegant approach would observe that all points on the semi-circle have
coordinates of the form (2 cos t, 2 sin t), where 0 ≤ t ≤ π. Here again we could
treat two cases, according as we permit the domain to be the full interval
stated, or only 0 ≤ t ≤ π2 . In either case the area is α(t) = |4 cos t · 2 sin t| =
|4 sin 2t|. For convenience we will take the domain to be 0 ≤ t ≤ π2 . Then,
observing that α(t) = 4 sin 2t, we have α0 (t) = 4(cos 2t) · 2, which vanishes
only where 2t = π2 , so t = π4 . Other than the end-points of the interval, this
is the only critical point, and the area there is α( π4 ) = 4 sin π2 = 4. At the
end-points we have α(0) = sin 0 = 0, and α( π2 ) = 0. Thus the maximum value
of 4 occurs only at t = π4 , and the minimum value of 0 occurs at t = 0 and
t = π2 .

7.2 Last Three Tutorial Quizzes in 2000/2001 (many versions)


7.2.1 Fourth 2000/2001 Tutorial Quizzes
T01
Distribution Date: Monday, October 23rd, 2000 — 13:30 to 14:30 h.
1. Evaluate the following limit, but do not use L’Hôpital’s rule:
sin(x2 − 1)
lim .
x→1 (x − 1)

dy 2
2. Find if y = 3x .
dx
1
3. 132
Find an equation for the tangent line to the graph of exy = x − +e at the
¡ 1¢ y
point e, e .
132
corrected, 7 November 2000
Information for Students in MATH 140 2008 09 3089

T02, T03, T05


Distribution Date: Monday, October 23rd, 2000 — 14:30 to 15:30 h.

1. Evaluate the following limit, but do not use L’Hôpital’s rule:


x + tan x
lim .
x→0 sin x
dy
2. Find if y = tan(3x ).
dx
3. Find an ³equation ´ for the tangent line to the graph of x2 (x2 + y 2 ) = y 2 at the
1 1
point √ , √
2 2
.

T04, T06
Distribution Date: Monday, October 23rd, 2000 — 15:30 to 16:30 h.

1. Use the limit definition of the derivative to find f 0 (0) for


½ 2 ¡ ¢
x sin x1 for x 6= 0
f (x) = .
0 for x = 0

dy 2
2. Find if y = sec(5x ) .
dx
3. If the line y = 3x − 1 is tangent to the graph of y = f (x) at the point
with x = 1 , find the equation of the tangent line to the graph of y = [f (x)]2
at the point with x = 1 .

T07
Distribution Date: Wednesday, October 25th, 2000 — 13:30 to 14:30 h.

1. Use the limit definition of the derivative to find f 0 (0) for


( 1 − cos x
when x = 6 0
f (x) = x .
0 when x = 0

dy 1
2. Find if y = 2 2x .
dx
3. If g(x) = 2x · f (x2 − 2x + 2) and f (1) = 3 , find g 0 (1) .
Information for Students in MATH 140 2008 09 3090

T08, T09, T11


Distribution Date: Wednesday, October 25th, 2000 — 14:30 to 15:30 h.
1. Use the limit definition of the derivative to find f 0 (8) for f (x) = x1/3 .
µ ¶
dy 1
2. Find if y = x tan .
dx 3x

3. If f (x) = ex·g(x) and g(0) = 5 , find f 0 (0) .

T10, T12, T15


Distribution Date: Wednesday, October 25th, 2000 — 15:30 to 16:30 h.
1. Use the limit definition of the derivative to find f 0 (1) for f (x) = x−1/3 .
dy ¡1¢
2. Find if y = x sec−1 2x
.
dx
3. Find the equation(s) of the line(s) through the origin, tangent to the graph of
y = ex .

T16
Distribution Date: Wednesday, October 25th, 2000 — 16:30 to 17:30 h.
1. Use the limit definition of the derivative to find f 0 (0) for f (x) = sec x .
µ ¶
dy −1 2x
2. Find if y = tan .
dx 1 − x2
3. Find equations(s) of the line(s) through the origin, tangent to the graph of
y = x2 + 9 .

T13
Distribution Date: Thursday, October 26th, 2000 — 16:00 to 17:00 h.
1. Use the limit definition of the derivative to find f 0 (0) for f (x) = ex sin x .
µ x ¶
dy −1 e − e−x
2. Find if y = tan .x
dx 2
3. Find the equations(s) of the line(s) through the origin, tangent to the graph of
2
y = ex .
Information for Students in MATH 140 2008 09 3091

T14

Distribution Date: Thursday, October 26th, 2000 — 17:00 to 18:00 h.

1. Use the limit definition of the derivative to find f 0 (0) for

f (x) = e3x (1 − cos x) .


µ ¶
dy −1 x
2. Find if y = sin √ .
dx 1 + x2
3. Find equations(s) of the line(s) through the origin, tangent to the graph of

y = e2x .

7.2.2 Fifth 2000/2001 Tutorial Quizzes


T01

Distribution Date: Monday, November 13th, 2000 — 13:30 to 14:30 h.

dy x
1. Find dx if y = x x−1 .
2. Water is pouring into a leaky tank at the rate of 10m3 /h. The tank
is a cone with the vertex down, 9m deep and 6m in diameter at the
top. The surface of the water is rising at a rate of 0.2m/h when the
depth is 6m. How fast is the water leaking out at that time?

T02, T03, T05

Distribution Date: Monday, November 13th, 2000 — 14:30 to 15:30 h.



x
dy
1. Find dx if y = x x−1 .
2. How fast must you let out the line if the kite you are flying is 30m
high, 40m horizontally away from you, and moving horizontally away
from you at 10m/min?
Information for Students in MATH 140 2008 09 3092

T04, T06
Distribution Date: Monday, November 13th, 2000 — 15:30 to 16:30 h.
dy √
1. Find dx if y = ( x + 1)x .
2. A ferris wheel you are riding has diameter 20m and is rotating at 1
revolution per minute. How fast are you rising or falling when you
are 6m horizontally away from the vertical line through the centre of
the wheel?

T07
Distribution Date: Wednesday, November 13th, 2000 — 13:30 to 14:30 h.
dy
1. Find dx if y = (x2 + 1)1/x .
2. A policeman stationed at a fixed distance from a highway aims a
radar gun at a car. When the gun is pointing at an angle of 45◦ to
the highway, the radar gun records the rate at which the distance of
the car from the gun is increasing at 100km/h. How fast is the car
travelling?

T08, T09, T11


Distribution Date: Wednesday, November 13th, 2000 — 14:30 to 15:30 h.
dy
1. Find dx if y = (xx + 1)1/2 .
2. [CORRECTED] A balloon is released and rises vertically from point
A and is tracked from point B, 100m horizontally away. If the balloon
is rising at the rate of k m/sec., how fast is the angle of elevation of
the balloon, as observed at point B, increasing when it is 200m above
A?

T10, T12, T15


Distribution Date: Wednesday, November 13th, 2000 — 15:30 to 16:30 h.
dy
1. Find dx if y = xln x .
2. A man 6ft tall walks towards a building at the rate of 5ft/sec. If
there is a light on the ground 50ft away from the building, how fast
is the man’s shadow shrinking when he is 30ft from the building?
Information for Students in MATH 140 2008 09 3093

T16

Distribution Date: Wednesday, November 13th, 2000 — 16:30 to 17:30 h.

1
dy
1. Find dx if y = x 2x .
2. A spherical balloon is being inflated so that its volume is increasing
at the rate of 5m3 /min. At what rate is the surface area increasing
when the radius is 6m?
(Volume = 43 πr3 , Area = 4πr2 , where r is the radius.)

T13

Distribution Date: Thursday, November 14th, 2000 — 16:00 to 17:00 h.

dy
1. Find dx if y = (x2x + 1)1/3 .
2. A 10m long ladder has one end on the ground, and is supported part
way along its length by a fence 3m high, so that part of the ladder
projects past the fence. If the end on the ground is 4m from the base
of the fence and is being dragged away at 0.2m/sec, how fast is the
vertical height of the other end of the ladder changing?

T14

Distribution Date: Thursday, November 14th, 2000 — 17:00 to 18:00 h.

dy
¡ 1 ¢x
1. Find dx if y = 2x .
2. A light shines from a pole 50ft high. A ball is dropped from the same
height from a point 30ft away from the light. In t secs the ball falls
16t2 ft. How fast is the shadow of the ball moving along the ground
after 0.5 secs?
Information for Students in MATH 140 2008 09 3094

7.2.3 Sixth 2000/2001 Tutorial Quizzes


T01

Distribution Date: Monday, November 27th, 2000 — 13:30 to 14:30 h.

Let f (x) = (2x2 + x + 1)ex .

1. [5 MARKS] Determine the domain of f .

2. [5 MARKS] Determine all horizontal asymptotes and all vertical asymptotes of the
graph of f .

3. [5 MARKS] Determine the intervals where f increases, and the intervals where it
decreases.

4. [5 MARKS] Determine all local maxima and all local minima of f .

5. [5 MARKS] Determine the intervals where the graph is concave upwards, and the
intervals where it is concave downwards.

6. [5 MARKS] Determine all inflection points of the graph.

7. [5 MARKS] Sketch the graph.


xa
(You may use, without proof, the fact that lim = 0 for any real number a.)
x→∞ ex

T02, T03, T05

Distribution Date: Monday, November 27th, 2000 — 14:30 to 15:30 h.

Let f (x) = (2x2 − 3x + 2)ex .

1. [5 MARKS] Determine the domain of f .

2. [5 MARKS] Determine all horizontal asymptotes and all vertical asymptotes of the
graph of f .

3. [5 MARKS] Determine the intervals where f increases, and the intervals where it
decreases.

4. [5 MARKS] Determine all local maxima and all local minima of f .

5. [5 MARKS] Determine the intervals where the graph is concave upwards, and the
intervals where it is concave downwards.
Information for Students in MATH 140 2008 09 3095

6. [5 MARKS] Determine all inflection points of the graph.

7. [5 MARKS] Sketch the graph.


xa
(You may use, without proof, the fact that lim = 0 for any real number a.)
x→∞ ex

T04, T06

Distribution Date: Monday, November 27th, 2000 — 15:30 to 16:30 h.

Let f (x) = (6x2 + x + 5)ex .

1. [5 MARKS] Determine the domain of f .

2. [5 MARKS] Determine all horizontal asymptotes and all vertical asymptotes of the
graph of f .

3. [5 MARKS] Determine the intervals where f increases, and the intervals where it
decreases.

4. [5 MARKS] Determine all local maxima and all local minima of f .

5. [5 MARKS] Determine the intervals where the graph is concave upwards, and the
intervals where it is concave downwards.

6. [5 MARKS] Determine all inflection points of the graph.

7. [5 MARKS] Sketch the graph.


xa
(You may use, without proof, the fact that lim = 0 for any real number a.)
x→∞ ex

T07

Distribution Date: Wednesday, November 29th, 2000 — 13:30 to 14:30 h.

Let f (x) = (2x2 − 5x + 4)ex .

1. [5 MARKS] Determine the domain of f .

2. [5 MARKS] Determine all horizontal asymptotes and all vertical asymptotes of the
graph of f .

3. [5 MARKS] Determine the intervals where f increases, and the intervals where it
decreases.
Information for Students in MATH 140 2008 09 3096

4. [5 MARKS] Determine all local maxima and all local minima of f .

5. [5 MARKS] Determine the intervals where the graph is concave upwards, and the
intervals where it is concave downwards.

6. [5 MARKS] Determine all inflection points of the graph.

7. [5 MARKS] Sketch the graph.


xa
(You may use, without proof, the fact that lim = 0 for any real number a.)
x→∞ ex

T08, T09, T11


Distribution Date: Wednesday, November 29th, 2000 — 14:30 to 15:30 h.
1 64
Let f (x) = − .
x−1 x+1
1. [5 MARKS] Determine the domain of f .

2. [5 MARKS] Determine all horizontal asymptotes and all vertical asymptotes of the
graph of f .

3. [5 MARKS] Determine the intervals where f increases, and the intervals where it
decreases.

4. [5 MARKS] Determine all local maxima and all local minima of f .

5. [5 MARKS] Determine the intervals where the graph is concave upwards, and the
intervals where it is concave downwards.

6. [5 MARKS] Determine all inflection points of the graph.

7. [5 MARKS] Sketch the graph.

T10, T12, T15


Distribution Date: Wednesday, November 29th, 2000 — 15:30 to 16:30 h.
64 1
Let f (x) = − .
x−1 x+1
1. [5 MARKS] Determine the domain of f .

2. [5 MARKS] Determine all horizontal asymptotes and all vertical asymptotes of the
graph of f .
Information for Students in MATH 140 2008 09 3097

3. [5 MARKS] Determine the intervals where f increases, and the intervals where it
decreases.

4. [5 MARKS] Determine all local maxima and all local minima of f .

5. [5 MARKS] Determine the intervals where the graph is concave upwards, and the
intervals where it is concave downwards.

6. [5 MARKS] Determine all inflection points of the graph.

7. [5 MARKS] Sketch the graph.

T16

Distribution Date: Wednesday, November 29th, 2000 — 16:30 to 17:30 h.


1 64
Let f (x) = − .
x−1 x+8
1. [5 MARKS] Determine the domain of f .

2. [5 MARKS] Determine all horizontal asymptotes and all vertical asymptotes of the
graph of f .

3. [5 MARKS] Determine the intervals where f increases, and the intervals where it
decreases.

4. [5 MARKS] Determine all local maxima and all local minima of f .

5. [5 MARKS] Determine the intervals where the graph is concave upwards, and the
intervals where it is concave downwards.

6. [5 MARKS] Determine all inflection points of the graph.

7. [5 MARKS] Sketch the graph.

T13

Distribution Date: Thursday, November 30th, 2000 — 16:00 to 17:00 h.


1 16
Let f (x) = − .
4x + 1 x + 2
1. [5 MARKS] Determine the domain of f .
Information for Students in MATH 140 2008 09 3098

2. [5 MARKS] Determine all horizontal asymptotes and all vertical asymptotes of the
graph of f .

3. [5 MARKS] Determine the intervals where f increases, and the intervals where it
decreases.

4. [5 MARKS] Determine all local maxima and all local minima of f .

5. [5 MARKS] Determine the intervals where the graph is concave upwards, and the
intervals where it is concave downwards.

6. [5 MARKS] Determine all inflection points of the graph.

7. [5 MARKS] Sketch the graph.

T14

Distribution Date: Thursday, November 30th, 2000 — 17:00 to 18:00 h.

Let f (x) = ln |x + 2| − 4 ln |x − 1| .

1. [5 MARKS] Determine the domain of f .

2. [5 MARKS] Determine all horizontal asymptotes and all vertical asymptotes of the
graph of f .

3. [5 MARKS] Determine the intervals where f increases, and the intervals where it
decreases.

4. [5 MARKS] Determine all local maxima and all local minima of f .

5. [5 MARKS] Determine the intervals where the graph is concave upwards, and the
intervals where it is concave downwards.

6. [5 MARKS] Determine all inflection points of the graph.

7. [5 MARKS] Sketch the graph.


Information for Students in MATH 140 2008 09 3099

7.3 2005/2006 Written Assignments


7.3.1 First 2005/2006 Written Assignment W1 , with Sketch of Solutions
The assignment question You are given formulæ for functions f and g.

1. Determine a formula for each of the functions f ◦ f , f ◦ g, g ◦ f , g ◦ g, simplified as


much as you can.

2. Determine for each of the functions f , g, f ◦ f , f ◦ g, g ◦ f , g ◦ g, its domain. Where


the domain is not all of R, you must explain your work — it’s not enough just to
write down the answers.
TUTORIAL DAY f (x) g(x)
2 1
MONDAY, 26 Sept., 2005 √1 − x x
WEDNESDAY, 28 Sept., 2005 2x − 1 x2 + 1
THURSDAY, 29 Sept., 2005 1
x−1
sin2 x
FRIDAY, 30 Sept., 2005 cos x 1 + x1

Solutions:

Marking Scheme: 1. 2 MARKS for each of the 4 formulæ for the compositions
2. 2 MARKS for each of the 6 domains. But, where the domain is not all of R,
the student must provide an acceptable explanation in order to receive full
marks.
3. THE CERTIFICATE OF ORIGINALITY must be signed. If the student has
not signed it, insist on seeing it signed before returning the graded assignment.
TOTAL = 20 MARKS

MONDAY: 1. Since f is a polynomial, there are no real numbers where it is unde-


fined. Domain(f ) = R
2. Since g(x) = x1 , it is defined only where the fraction is defined. In the devel-
opment of the real number system we do not assign a meaning to 10 ; we can’t!
No matter what real number r we would want to choose as the value of 10 , we
find that one of our other basic properties of the real number system would
be violated if we used r as the value of this quotient. We resolve this issue by
restricting fractions ab to cases where b 6= 0. Thus Domain(g) = R − {0}.
3. f (f (x)) = 1 − (1 − x2 )2 = 2x2 − x4 . Since f is defined for all x, we may apply
the function again to f (x). It follows that Domain(f ◦ f ) = R.
Information for Students in MATH 140 2008 09 3100

¡ ¢2
4. f (g(x)) = 1 − x1 = 1 − x12 . Since g(x) is not defined for x = 0, 0 will
be excluded from the domain of the composition. As f is defined for all x,
the composition is also defined for all x except this one excluded value. Thus
Domain(f ◦ g) = R − {0}.
1
5. g(f (x)) = 1−x 2 . There is no restriction when the first function, here f , is

applied. But we must exclude the possibility that its value be 0, since g will
not be defined there. Thus we exclude all x such that 1 − x2 = 0, equivalently,
that x = ±1. It follows that Domain(g ◦ f ) = R − {−1, 1}.
6. g(g(x)) = 11 = x. One might be tempted to say that the domain of the
x
composition is R. But we would be forgetting that the first application of g
was restricted to x 6= 0. So the composition g ◦ g differs from the function x
only in the fact that its domain excludes the point 0: Domain(g ◦g) = R−{0}.

WEDNESDAY: 1. The square root 2x − 1 is defined only where the argument is
1
£ 1 negative, i.e., where 2x − 1 ≥ 0, equivalently, where x ≥ 2 . Domain(f ) =
not
2
, ∞).
2. Since g is a polynomial, it is defined for all real numbers x. Thus Domain(g) =
R.
p √
3. f (f (x)) = 2 2x − 1 − 1. The first application of f requires that x ≥ 12 .
The result of this application is a non-negative number, which, when√ doubled
and added to −1, must be non-negative also.£ We thus require 2 2x − 1 ≥ 1,
which implies that x ≥ 85 . Domain(f ◦ f ) = 85 , ∞).
p √
4. f (g(x)) = 2(x2 + 1) − 1 = 2x2 + 1. The first function yields a number
no less than 1, which, when doubled and the result decreased by 1, yields
a positive number, which certainly has a non-negative square root. Hence
Domain(f ◦ g) = R.
¡√ ¢2
5. g(f (x)) = 2x − 1 + 1 = |2x − 1| + 1. The application of f requires that
x ≥ 12 . Thus 2x − 1 ≥ 0, and we may remove the absolute sign and further
simplify to g(f (x)) = 2x. But, unlike the polynomial 2x, this function has
a£ more restricted domain, because of the restriction on f : Domain(g ◦ f ) =
1
2
, ∞).
2
6. g(g(x)) = (x2 + 1) . This is no restriction on the applicability of g, and so
the 2nd application is also unrestricted. Domain(g ◦ g) = R.
1
THURSDAY: 1. The reciprocal x−1 is defined only where x − 1 6= 0, i.e., where
x 6= 1. Domain(f ) = R − {1}. (See solution MONDAY.2.)
2. Since g is a power of the sine function, which is defined for all x, Domain(g) =
R.
Information for Students in MATH 140 2008 09 3101

1 x−1
3. f (f (x)) = 1
−1
= 2−x
. This rational expression conceals one restriction in
x−1
the domain, so let’s examine the steps in computing this composition. In
the first application of f we must avoid the value x = 1. No matter what
happens in the second application, the value x = 1 has now been irrevocably
removed from the domain of the composition. In the second application we
1
have to ensure that x−1 − 1 6= 0. But this is equivalent to x 6= 1, 2. Thus
Domain(f ◦ f ) = R − {1, 2}.
4. f (g(x)) = sin21x−1 = − sec2 x. When we first apply g there is no restriction, as
the sine function and its square are applicable everywhere. However, we have
to ensure that the result of¡the ¢ application of g is different from 1, i.e., that
sin x 6= ±1, i.e., that x 6= n π2 , where n is any odd integer. Thus
n πo
Domain(f ◦ g) = R − all odd integer multiples of .
2
¡ 1 ¢
5. g(f (x)) = sin2 x−1 . The first step in calculating f is to take the reciprocal
of a non-zero real number; we need to restrict x to be different from 1, and
no other restriction is needed. The subsequent application of g imposes no
further restriction, as the sine function is defined for all real numbers. Hence
Domain(g ◦ f ) = R − {1}.
¡ ¢
6. g(g(x)) = sin2 sin2 x . There are no restrictions on x: Domain(g ◦ g) = R.

FRIDAY: 1. The cosine function is defined for all x. Domain(f ) = R.


2. The only restriction in evaluating g is that x1 must be well defined, so x 6=
0. This restriction is not affected by the subsequent addition of 1 to the
reciprocal. Domain(g) = R − {0}.
3. f (f (x)) = cos(cos x). The cosine function is defined everywhere, so one ap-
plication can follow another without restriction. Domain(f ◦ f ) = R.
¡ ¢
4. f (g(x)) = cos 1 + x1 . In applying g first we need to ensure that x 6= 0. The
subsequent application of f is unrestricted, as its domain is all of R. Hence
Domain(f ◦ g) = R − {0} .
5. g(f (x)) = 1 + cos1 x = 1 + sec x. The first step in calculating f is to evaluate a
cosine. That is possible for all x. However, we cannot proceed to the second
step of the calculation if that cosine is equal to 0, and that is the only obstacle
to completing the calculation. It follows that
n πo
Domain(g ◦ f ) = R − all odd integer multiples of .
2
Information for Students in MATH 140 2008 09 3102

6. g(g(x)) = 1+ 1+1 1 = 2x+1


x+1
. In the first application of g we must avoid the value
x
x = 0 in order to take the reciprocal of x. But, for a second application to be
possible, we need to ensure that g(x) 6= −1, i.e., that x1 6= −1, equivalently
that x 6= −1. Domain(g ◦ g) = R − {−1, 0}.

7.3.2 Second 2005/2006 Written Assignment W2 , with Sketch of Solutions


Your completed solution to this assignment should be submitted, with a copy of this
question sheet at your Tutorial, during the week October 17-21, 2005. Please slip the
assignment into your folded quiz answer paper. All materials must bear your name
and/or student number. No other method of submission is acceptable.

The assignment questions with solutions: Prove the following statement by using
the Principle of Mathematical Induction.

Students whose tutorial is on Monday:

n(n + 1)(2n + 1)
12 + 22 + 32 + . . . + n2 =
6
for all positive integers n.
Solution: Define S(n) to be the preceding equation.

“Base” or “Anchor” Case: Since


1(1 + 1)(2 + 1) 6
= = 1 = 12 ,
6 6
S(1) is true.
Induction Step: Suppose that it is known that S(k) is true, where k is any
positive integer. Then

12 + 22 + 32 + . . . + k 2 + (k + 1)2
¡ 2 ¢
= 1 + 22 + 32 + . . . + k 2 + (k + 1)2
k(k + 1)(2k + 1)
= + (k + 1)2
6
by the Induction Hypothesis
k+1 k+1
= · (k(2k + 1) + 6(k + 1)) = · (2k 2 + 7k + 6)
6 6
k+1
= · (2k + 3)(k + 2)
6
Information for Students in MATH 140 2008 09 3103

(k + 1)(k + 2)(2k + 3)
= ,
6
(k + 1)((k + 1) + 1)(2(k + 1) + 1)
=
6
which is S(k + 1).
Conclusion: We may conclude that S(n) is true for all positive integers n.
Students whose tutorial is on Wednesday:

If n is a positive integer, then 5n + 3 is divisible by 4.

Solution: Let S(n) denote the preceding statement.


“Base” or “Anchor” Case: When n = 1, 51 + 3 = 8, which is equal to 2 × 4.
Thus 51 + 3 is divisible by 4, and S(1) is true.
Induction Step: Suppose it is known that S(k) is true, i.e., that 5k + 3 is a
multiple of 4, i.e., that there exists an integer b such that 5k + 3 = b · 4. Then

5k+1 + 3 = 5 · 5k + 3
by the exponent rules
¡ ¢ ¡ ¢
= 4 5k + 5k + 3 since 5 = 4 + 1
¡ k¢
= 4 5 + b · 4 by the Induction Hypothesis
¡ ¢
= 5k + b · 4 ,

which is a multiple of 4. We have thus proved that S(k + 1) is true.


Conclusion: We may conclude that S(n) is true for all positive integers n.
Students whose tutorial is on Thursday: A sequence of real numbers a1 , a2 , . . . , an ,
. . . is defined133 as follows:
1
a1 = ,
2
1
For n = 1, 2, 3, . . . an+1 = an + .
(n + 1)(n + 2)
Prove that
1 1 1 1
2
+ 2 + 2 + ... + < an for all n ≥ 1.
2 3 4 (n + 1)2
Solution: Let the preceding inequality be designated as S(n).
133
We call a definition of this type a recursive definition.
Information for Students in MATH 140 2008 09 3104

1
“Base” or “Anchor” Case: The case n = 1 reads < a1 . Since
22
1 1 1
a1 = > = 2,
2 4 2
statement S(1) is true.
Induction Step: Suppose it is known that S(k) is true, i.e., that
1 1 1 1
+ + + . . . + < ak .
22 32 42 (k + 1)2
Then
1 1 1 1
2
+ 2 + 2 + ... +
2 3 4 ((k + 1) + 1)2
µ ¶
1 1 1 1 1
= 2
+ 2 + 2 + ... + 2
+
2 3 4 (k + 1) (k + 2)2
1
< ak +
(k + 2)2
by the Induction Hypothesis
1
< ak +
(k + 1)(k + 2)
since the denominator is reduced, so the fraction is increased
= ak+1 by the definition of ak+1
proving Sk+1 .
Conclusion: We may conclude that S(n) is true for all positive integers n.
Students whose tutorial is on Friday: A sequence of functions f0 , f1 , . . . , fn , . . . is
defined by
f0 (x) = 2x + 1 ,
For n = 0, 1, 2, . . . fn+1 = f0 ◦ fn .
Prove that
fn (x) = 2n+1 x + 2n+1 − 1 .
Solution: Let S(n) denote the preceding sentence.
“Base” or “Anchor” Step: When n = 0,
2n+1 x + 2n+1 − 1 = 2x + 2 − 1 = 2x + 1 = f0 (x)
so S(0) is true.
Information for Students in MATH 140 2008 09 3105

Induction Step: Suppose it is known that S(k) is true. Then


fk+1 (x) = (f0 ◦ fk ) (x) by definition of fk+1
= f0 (fk (x)) by definition of ◦
¡ ¢
= f0 2k+1 x + 2k+1 − 1
by the Induction Hypothesis
¡ ¢
= 2 2k+1 x + 2k+1 − 1 + 1 by definition of f0
= 2(k+1)+1 x + 2(k+1)+1 − 1
proving Sk+1 .
Conclusion: Hence, by the Principle of Induction, S(n) is true for all n ≥ 0.

Solution to the “Fun Problem” (Student were not asked to submit a solu-
tion.): What is wrong with the following “proof” by induction?
Claim: All birds are of the same colour.
Fallacious Proof: Let Sn be the statement
“any n birds are of the same colour”.
The statement is clearly true for n = 1, because all members of a set of one bird have
the same colour. Assume that it is always true that any k birds are of the same colour.
(We call this the Induction Hypothesis.) Now take k + 1 birds. Put one aside. There
are k birds left; by the Induction Hypothesis, the k birds are of the same colour. For
simplicity, say they are all black. Replace the bird previously taken aside into the group,
and take out a different bird. Again, there are k birds remaining, which by assumption
must have the same colour. In particular, the bird that was taken out in the first place
must now have the same colour as all the other birds, namely black. The bird taken out
was also black. Therefore, the k + 1 birds must all be black. But that means that we
have proved the statement Sn for all natural numbers by induction. Yet, the statement
is obviously false. Where is the fallacy in this proof?
The Fallacy: We have given what appears to be a general proof that Sk implies Sk+1 .
That general proof is almost correct. The problem is that it is not valid in the case
S1 ⇒ S2 . In that one induction step, the assumption that the kth case implies the next
is defective. Consider a set containing exactly 2 birds. If you remove 1 bird, then the
resulting set of 1 bird does, indeed, consist of birds of the same colour. But the removal
of another element of the set of 2 — i.e., of the only possible other element — produces
a set of 1 bird which is totally disjoint from the one we discussed earlier. These 2 sets
of size 2 − 1 do not overlap; so, while they each consist of elements of the same colour,
there is no justification in concluding that those colours are the same for both sets of
size 2 − 1.
Information for Students in MATH 140 2008 09 3106

7.3.3 Third 2005/2006 Written Assignment W3


Your completed solution to this assignment should be submitted, with a copy of this
question sheet at your Tutorial, during the week October 31 – November 4, 2005. Please
slip the assignment into your folded quiz answer paper. All materials must bear your
name and/or student number. No other method of submission is acceptable.

Certificate: Your assignment will not be graded unless you attach or include the fol-
lowing completed certificate of originality, signed in ink:
I have read the information on the web page

http://www.mcgill.ca/integrity/studentguide/,

and assert that my work submitted for W3 , R7 , and R8 does not violate
McGill’s regulations concerning plagiarism.

Signature(required) Date(required)

Instructions:
• This assignment consists of problems taken from YOUR VERSIONS of WeBWorK
assignments R6 and R7 .
• There is no place in this assignment where the use of a calculator is required
or permitted.
• The tutor may grade only part of your solutions, but may check whether you have
submitted solutions to all of the problems.
• You are expected to write FULL SOLUTIONS to the following problems; it is not suf-
ficient to get the correct answer — you should show a full explanation, comparable to
solutions in the Student Solutions Manual to the textbook.
• You are expected either to attach a printout of the questions from your
assignments, or to copy the questions verbatim on to your answer sheet before
beginning the solution.

The assignment questions:


1. Problem 1 on R6: DO NOT USE A CALCULATOR; YOU MAY EXPRESS
PORTIONS OF YOUR SOLUTION USING INVERSE TRIGONOMETRIC FUNC-
TIONS.
Information for Students in MATH 140 2008 09 3107

2. Problem 4 on R6

3. Problem 14 on R6: Do not use l’Hospital’s Rule anywhere in your solutions.

4. Problem 16 on R6

5. Problem 9 on R7

7.3.4 Fourth 2005/2006 Written Assignment W4


Your completed solution to this assignment should be submitted, with a copy of this
question sheet at your Tutorial, during the week November 14 – November 18, 2005.
Please slip the assignment into your folded quiz answer paper. All materials must bear
your name and/or student number. No other method of submission is acceptable.

Certificate: Your assignment will not be graded unless you attach or include the fol-
lowing completed certificate of originality, signed in ink:

I have read the information on the web page

http://www.mcgill.ca/integrity/studentguide/,

and assert that my work submitted for W4 , R9 , and R10 does not violate
McGill’s regulations concerning plagiarism.

Signature(required) Date(required)

Instructions:
• This assignment consists of 2 problems taken from YOUR version of WeBWorK as-
signment R9 . One of the problems is extended beyond the version on your WeBWorK
assignment.

• There is no place in this assignment where the use of a calculator is required


or permitted.

• The tutor may grade only part of your solutions, but may check whether you have
submitted solutions to all of the problems.

• You are expected to write FULL SOLUTIONS to the following problems; it is not suf-
ficient to get the correct answer — you should show a full explanation, comparable to
solutions in the Student Solutions Manual to the textbook.
Information for Students in MATH 140 2008 09 3108

• You are expected either to attach a printout of the questions from your
assignments, or to copy the questions verbatim on to your answer sheet before
beginning the solution.

The assignment questions:

1. Problem 6 on R9 :

(a) Solve the problem as stated.


(b) Find the normal line to the curve at the point of contact of the tangent.

2. Problem 19 on R9 . (This problem resembles Exercise 23, p. 261 in your text-


book.)

7.3.5 Fifth 2005/2006 Written Assignment W5


Your completed solution to this assignment should be submitted, with a copy of this
question sheet at your Tutorial, during the week November 28 – December 2, 2005.
Please slip the assignment into your folded quiz answer paper. All materials must bear
your name and/or student number. No other method of submission is acceptable.

Certificate: Your assignment will not be graded unless you attach or include the fol-
lowing completed certificate of originality, signed in ink:

I have read the information on the web page

http://www.mcgill.ca/integrity/studentguide/,

and assert that my work submitted for W5 , R11 , and R12 does not
violate McGill’s regulations concerning plagiarism.

Signature(required) Date(required)

Instructions:
• This assignment consists of an extension of one problem taken from YOUR version of
WeBWorK assignment R11 . Note that the problem given below asks more than the
WeBWorK problem.
Solution of the problem may require the use of L’Hospital’s Rule, which you may not
have seen yet in your lectures. Part of your assignment is to read ahead in your textbook,
Information for Students in MATH 140 2008 09 3109

and learn enough about that rule that you may use it if you need to. Example 2 on page
309 of the textbook, may prove helpful.

• There is no place in this assignment where the use of a calculator is required


or permitted.

• The tutor may grade only some parts of your solutions, but may check whether you have
submitted solutions to all of the parts.

• You are expected to write FULL SOLUTIONS; it is not sufficient to report the correct
answer — you should show a full explanation, comparable to solutions in the Student
Solutions Manual to the textbook.

The assignment question: Consider the function f (x) = x2 eax with the value of a
in Problem 13 of your WeBWorK assignment R11 . Show all your work! It is not
sufficient to make unsubstantiated statements.

1. Find an equation for each horizontal asymptote to the graph of f .

2. Find an equation for every vertical asymptote, or show that there is none.

3. Determine all critical numbers.

4. Determine the global maximum of the function, if there is one, or show that there
is none.

5. Determine the global minimum of the function, if there is one, or show that there
is none.

6. Determine the locations of all inflection points of this graph. You are expected
to refer to the textbook definition of inflection point and to demonstrate that the
points you have given are, indeed, inflection points.

7. Give the intervals where the graph is concave downward, and the intervals where
the graph is concave upward.

8. Give a rough sketch of your graph, showing the approximate locations of the in-
flection points and the intervals of upward and downward concavity. (The sketch
may be freehand, and need not be to scale.)
Information for Students in MATH 140 2008 09 3110

7.4 Quizzes from MATH 140 2006 09, with Draft Solutions
7.4.1 Draft Solutions to Quiz Q1
Distribution Date: Released to students 21 October, 2006
Draft, subject to correction.

The following sketches of solutions will be given for one specific set of data for each of
the versions of the quiz.

Instructions to students
1. Show all your work. Marks are not given for answers alone.

2. You must enclose this question sheet in your folded answer sheet.
3. Time = 20 minutes

4. No calculators are permitted.

Monday version
1. By using appropriate algebraic operations, compute the following limit or show
that it does not exist:
x−4
lim √
x→4 5 − x + 21

Don’t use L’Hospital’s rule or epsilon-delta methods!


Solution:

(a) As x√ → 4 the numerator has limit 0, and the limit of the denominator,
5 − x + 21, is also 0; so the Quotient Law may not be used.
(b) Rationalize the denominator and simplify the ratio:

x−4 (x − 4)(5 + x + 21)
√ = √ √
5 − x + 21 (5 − x + 21)(5 + x + 21)

(x − 4)(5 + x + 21)
=
25 − (x + 21)

(x − 4)(5 + x + 21)
=
√4 − x
= −(5 + x + 21)
Information for Students in MATH 140 2008 09 3111

(c) Now the Limit Laws may be used:


√ q
lim (−5 − x + 21) = lim (−5) − lim (x + 21)
x→4 x→4 x→4

= −5 − 25 = −10

If a student used L’Hospital’s Rule, she should have received a grade of 0, as it was
explicitly forbidden to use that method, and we often make a similar restriction on
examination questions.

2. Find a formula for the inverse of the function

1 + 6ex
f (x) =
2 + 7ex
What is the range (image) of the inverse function?
Solution:

(a) Name the “dependent” variable y:


1 + 6ex
y=
2 + 7ex

(b) Solve for ex in terms of y:


1 − 2y
ex =
7y − 6
(c) Using the logarithm, solve further for x in terms of y:
1 − 2y
x = ln
7y − 6

(d) Describe the inverse function:


1 − 2y
f −1 (y) = ln
7y − 6
The usual convention would be to rename the independent variable of the
inverse function as x, but this step is not needed if the preceding step was
carried out using another name (y) for the independent variable:
1 − 2x
f −1 (x) = ln
7x − 6
Information for Students in MATH 140 2008 09 3112

(e) The student is to give the image of the inverse function. The instruction (1)
is clear: Marks are not given for answers alone.
This problem could be solved directly or indirectly. To solve it indirectly,
recall that the image of f −1 is the domain of f . The value of f (x) is defined
as a fraction, defined everywhere except where the denominator is 0, i.e.,
everywhere except where 2 + 7ex = 0; equivalently, except where ex = − 72 .
But an exponential cannot be negative, so the denominator is non-zero for all
values of x, and the domain of f is R.
To find the image directly is harder; I would not expect students in MATH
140 to be able to articulate a convincing argument of this type. The ratio
1−2x
7x−6
is undefined when x = 67 ; it is positive only where 21 < x < 67 ; as we
wish to take
¡ 1 ¢+a logarithm, the function is defined only for x in−1this interval.
As x → 2 , the fraction approaches 0 from the right, and f (x) → −∞.
¡ ¢−
As x → 76 , the fraction approaches ∞ and f −1 (x) → +∞. The function is
continuous, and thus (by the Intermediate Value Theorem) takes on all real
values; that is, the image of f −1 is R.

Wednesday version

1. By using appropriate algebraic operations, compute the following limit or show


that it does not exist:
µ ¶
1 1
lim − 2
t→0 5t 7t + 5t

Don’t use L’Hospital’s rule or epsilon-delta methods!


Solution:

(a) Algebraic manipulations before evaluating the limit:


1 1 1 1
− 2 = −
5t 7t + 5t 5t (7t + 5)t
µ ¶
1 1 1
= − ·
5 7t + 5 t
7t 1
= ·
5(7t + 5) t
7t
=
5(7t + 5)t
Information for Students in MATH 140 2008 09 3113

(b) Divide numerator and denominator by factor t, which is not zero, as we are
computing the limit as t → 0:
7t 7
=
5(7t + 5)t 5(7t + 5)

for t near, but not equal to 0.


(c) Apply the limit laws — I don’t expect students to give their reasons as pre-
cisely as the following:
µ ¶ lim 7
7 t→0
lim =
t→0 5(7t + 5) lim 5(7t + 5)
t→0
7
=
25
TA’s were instructed not to give any marks if either

• the student uses L’Hospital’s Rule; or


• the student computes a difference like ∞ − ∞.

2. Find a formula for the inverse of the function


7 − 2 ln x
f (x) = .
6 + 1 ln x
What is the range (image) of the inverse function?
Solution:

(a) Name the dependent variable y:

7 − 2 ln x
y=
6 + 1 ln x

(b) Solve for ln x in terms of y:


7 − 6y
ln x = .
y+2

(c) Using exponential, solve for x in terms of y:


7 − 6y
x=e y+2 .
Information for Students in MATH 140 2008 09 3114

(d) Describe the inverse function precisely:


7 − 6y
f −1 (y) = e y + 2 .

The usual convention would be to rename the independent variable of the


inverse function as x, but this step is not needed if the preceding step was
carried out using another name (y) for the independent variable:
7 − 6x
f −1
(x) = e x + 2 .

(e) The student is to give the image of the inverse function. The instruction (1)
is clear: Marks are not given for answers alone!
This problem could be solved directly or indirectly. To solve it indirectly, recall
that the image of f −1 is the domain of f . The value of f (x) is a fraction,
defined everywhere except where the denominator is 0, i.e., everywhere except
where 6 + 1 ln x = 0; equivalently, where ln x = − 16 , i.e., where x = e−6 . The
point e−6 must be deleted from the maximum domain of the function ln, i.e.,
from the interval (0, +∞). Thus the domain of f is
¡ −6 ¢ ¡ −6 ¢
0, e ∪ e , +∞ ,

and this must be the image, or range of the inverse function.


To find the image directly is harder. If a student attempts this approach, use
good judgement in assigning marks. One approach would observe that
7−6x
lim − e x+2 = 0
x→−2
7−6x
lim e x+2 = e−6
x→−∞
7−6x
lim + e x+2 = +∞
x→−2
7−6x
lim e x+2 = e−6 .
x→+∞

The Intermediate Value Theorem, applied to the interval (−∞, −2) yields
function values in the interval (−∞, e−6 ); applied to the interval (−2, +∞)
it yields function values in the interval (e−6 , +∞). It’s unrealistic to expect
Calculus 1 students to argue in this way.
Information for Students in MATH 140 2008 09 3115

Thursday version
1. By using appropriate algebraic operations, compute the following limit or show
that it does not exist:
1
−1
lim 2 x 4
x→4 x + 3x − 28

Don’t use L’Hospital’s rule or epsilon-delta methods!


Solution:
(a) Algebraic manipulations before evaluating the limit:
1
x
− 14 4−x
4x
=
x2 + 3x − 28 x2 + 3x − 28
4−x
x
=
4(x + 7)(x − 4)
(4 − x)
=
4x(x + 7)(x − 4)

(b) Divide numerator and denominator by the factor x − 4, which cannot be 0 as


we are computing the limit x → 4:
(4 − x) −1
=
4x(x + 7)(x − 4) 4x(x + 7)
for x near, but not equal to 4.
(c) Now apply the Limit Laws:
1 µ ¶
− 14
x −1
lim 2 = lim
x→4 x + 3x − 28 x→4 4x(x + 7)
lim (−1)
x→4
=
lim (4x(x + 7))
x→4
−1 1
= =−
4 · 4(4 + 7) 176

2. Find a formula for the inverse of the function

f (x) = x2 + 8x + 25

defined for all x ≥ 0. What is the domain of the inverse function?


Solution:
Information for Students in MATH 140 2008 09 3116

(a) Name the dependent variable:

y = x2 + 8x + 25 .

(b) Solve for x in terms of y, keeping in mind that x has been restricted to lie in
the interval x ≥ 0. Students may use the “quadratic formula”, although I
prefer to “complete the square”:

y = x2 + 8x + 25 = (x + 4)2 + 9
p
⇔ x+4=± y−9
p
⇔ x = −4 ± y − 9.

But the lower choice


√ of sign would make x < 0, which has been excluded.
Thus x = −4 + y − 9.
(c) Even with the resolution of the double sign there is still√restriction on y that
is required to ensure that x ≥ 0. While the expression y − 9 is meaningful
for y ≥ 9, we need to ensure that it is no less than 4; this requires that
y ≥ 25. (This observation could have been made earlier, by examining the
sum x2 + 8x + 25 when x ≥ 0.)
(d) Give a clear formula for the inverse function:
p
f −1 (y) = −4 + y − 9 (y ≥ 25)

or √
f −1 (x) = −4 + x−9 (x ≥ 25) .
(e) The domain of √ the inverse function is not the interval [9, +∞), even though
that is where x − 9 is meaningful! The function is defined only on the image
of f , which excluded the points in [9, 25). Thus the domain of f −1 is [25, ∞).

Friday version

1. By using appropriate algebraic operations, compute the following limit or show


that it does not exist:

x2 − 36
lim √ √
x→6 6− x

Don’t use L’Hospital’s rule or epsilon-delta methods!


Solution:
Information for Students in MATH 140 2008 09 3117

(a) Algebraic manipulations before evaluating the limit:


x2 − 36 (x − 6)(x + 6)
√ √ = √ √
6− x 6− x
√ √ √ √
( x + 6)( x − 6)(x + 6)
= √ √
6− x
We could also √have proceeded
√ by factorizing the numerator as a difference of
4th powers of x and 6.
√ √
(b) Division of numerator and denominator by the factor x − 6, which is non-
zero, since we are evaluating the limit as x → 6, but need not permit x = 6:
√ √ √ √ √ √
( x + 6)( x − 6)(x + 6) ( x + 6)(x + 6) √ √
√ √ = = −( x + 6)(x + 6) ,
6− x −1
for x near but not equal to 6.
(c) Now apply the Limit Laws:
³ √ √ ´ √ √
lim −( x + 6)(x + 6) = − lim ( x + 6) · lim (x + 6)
x→6 x→6 x→6
√ √
= −2 6 · (6 + 6) = −24 6 .

2. Find a formula for the inverse of the function


f (x) = x2 − 10x + 21
defined for all x ≤ 0. What is the domain of the inverse function?
Solution:
(a) Name the dependent variable:
y = x2 − 10x + 21 .

(b) Solve for x in terms of y, keeping in mind that x has been restricted to lie in
the interval x ≤ 0. Students may use the “quadratic formula”, although I
prefer to “complete the square”:
y = x2 − 10x + 21 = (x − 5)2 − 4
p
⇔ x−5=± y+4
p
⇔ x = 5 ± y + 4.
But the upper√choice of sign would make x > 0, which has been excluded.
Thus x = 5 − y + 4.
Information for Students in MATH 140 2008 09 3118

(c) Even with the resolution of the double sign, there is still further restriction

on y that is required to ensure that x ≤ 0. While the expression y + 4 is
meaningful for y ≥ −4, we need to ensure that it is no less than 5; this requires
that y ≥ 21. (This observation could have been made earlier, by examining
the sum x2 − 10x + 21 when x ≤ 0.)
(d) Give a clear formula for the inverse function:
p
f −1 (y) = 5 − y + 4 (y ≥ 21)
or √
f −1 (x) = 5 − x+4 (x ≥ 21) .
(e) The domain of√the inverse function is not the interval [−4, +∞), even though
that is where x + 4 is meaningful! The function is defined only on the image
of f , which excluded the points in [−4, 21). Thus the domain of f −1 is [21, ∞).

7.4.2 Draft Solutions to Quiz Q2


Distribution Date: November 11th, 2006
(subject to correction)

The following draft sketches of solutions will be given for one specific set of data for each
of the versions of the quiz.

Instructions to students
1. Show all your work. Marks are not given for answers alone.
2. You must enclose this question sheet in your folded answer sheet.
3. Time = 20 minutes
4. No calculators are permitted.

Monday version
1. By using appropriate algebraic manipulations, compute the following limit or show
that it does not exist:
³√ √ ´
lim x2 + 9x − x2 + 4x
x→−∞

Don’t use L’Hospital’s rule or epsilon-delta methods!


Solution:
Information for Students in MATH 140 2008 09 3119

(a) If a student attempts to evaluate the limit of the difference by using the
Difference Law, you should give ZERO for the entire problem. The difference
must be transformed BEFORE applying any of the Limit Laws!
(b) Multiply and divide by a function which will rationalize the given function
(at the expense of introducing a complicated denominator):
³√ √ ´ ¡√x2 + 9x + √x2 + 4x¢
x2 + 9x − x2 + 4x · ¡√ √ ¢
x2 + 9x + x2 + 4x

(c) Simplify the numerator:


³√ √ ´ ³√ √ ´
x + 9x − x + 4x · x + 9x + x + 4x = (x2 +9x)−(x2 +4x) = 5x
2 2 2 2

(d) The function has now been transformed from a difference of the form ∞ − ∞,

which we can’t evaluate, to a ratio of the form ∞ , which we also can’t evaluate.
But we can divide numerator and denominator by a factor which will then
permit us to use the Quotient Law. To do this we need a factor like x in the
denominator. This is obtained using the property of exponents that
√ √ √
AB = A · B

where A and B are non-negative. In this case we have


s µ ¶
√ 9
x2 + 9x = x2 1 +
x
s µ ¶
√ 4
2
x + 4x = 2
x 1+
x

(e) Using the fact that x2 = |x| we have
³√ √ ´ x 5
2 2
x + 9x − x + 4x = ·q q
|x| 1+ 9
+ 1+ 4
x x

(f) Now, as x → −∞, the first factor → −1, and the second → 25 ; so the limit is
− 52 .
You should not give full marks if the student’s solution doesn’t make it clear how
she is using the fact that the limit is taken as x → −∞; that is, if the argument
written down could apply equally well to the limit at +∞, then something is
missing in the solution, and full marks have definitely not been earned!
Information for Students in MATH 140 2008 09 3120

2. Use the limit definition of the derivative to compute the derivative of


1
f (x) =
(x + 8)2

at x = −7. Don’t use any of the differentiation rules of Chapter 3 of Stewart!


Solution:

(a) Give the definition of the derivative as a limit (several alternatives are possi-
ble):
f (−7 + h) − f (−7)
f 0 (−7) = lim
h→0 h
(b) Express that general limit in the particular case of the given function
1 1
0 (−7+h)2
− (−7)2
f (−7) = lim
h→0 h

(c) Simplify the fraction algebraically:


1 1
(−7+h)2
− (−7)2 (−7)2 − (−7 + h)2
lim = lim
h→0 h h→0 h(−7)2 (−7 + h)2

(d) Simplify the numerator by expanding the squares:

(−7)2 − (−7 + h)2 14h − h2


lim = lim
h→0 h(−7)2 (−7 + h)2 h→0 h(−7)2 (−7 + h)2

(e) Divide numerator and denominator by h:

14 − h
lim
h→0 (−7)2 (−7 + h)2

(f) Now the Quotient Law may be used:

14 − h 14 − 0 14 2
lim = = = .
h→0 (−7)2 (−7 + h)2 (−7)2 (−7 + 0)2 492 343

Don’t take off marks if students can’t compute the numerical powers without
a calculator.
Information for Students in MATH 140 2008 09 3121

Wednesday version
1. Show that the equation
3x = 2x + 3
has at least one solution between x = −2 and x = 0. Justify all your steps!
Solution:
(a) The first step in a problem like this is to identify a function to which to apply
the Intermediate Value Theorem. One such function — not the only one —
is f (x) = 3x − (2x − 3).
(b) A next step is to identify the closed interval on which to apply the theorem.
Here the data have been given, the student must simply show that she has
extracted this information from the problem: the interval is [−2, 0].
(c) The student must at least OBSERVE that the function is continuous on the
given closed interval. That is, a word like continuous or continuity should
appear explicitly in the solution.
(d) The function must be evaluated at the end-points of the given interval, and
it must be shown that the two values are on opposite sides of 0:
1 10
f (−2) = − (−4 + 3) = >0
9 9
1
f (0) = − (0 + 3) = −2 < 0
1
(e) Conclude by applying the Intermediate Value Theorem that the function as-
sumes the value 0 somewhere in the interval.
2. Let
2x2 + 2x + 1
f (x) = .
4ex
Compute f 0 (x) and find the equation of the tangent line to the graph of f (x) at
x = 0. You may use all the differentiation rules you know, but you are expected
to show all steps of your solution.
Solution:
(a) There could be different ways in which the differentiation rules can be imple-
mented; only one is given here. Steps must be shown in the solution — do
not give full marks if only the answer is given!
(4x + 2) · (4ex ) − (2x2 + 2x + 1) · (4ex )
f 0 (x) =
(4ex )2
−2x2 + 2x + 1
= .
4ex
Information for Students in MATH 140 2008 09 3122

(b) Hence f 0 (0) = 14 .


(c) When x = 0, f (x) = 41 .
¡ ¢
(d) An equation for the tangent line at the point (x, y) = 0, 41 on the graph of
f is
1 1
y − = (x − 0)
4 4
or x − 4y = −1.

Thursday version

1. Let 
 9

 if x < −3

 x
f (x) = ab if x = −3 .





−6 + b + ln(x + 4) if x > −3
Showing all your work, determine what values must be chosen for a and b in order
to make this function continuous at x = −3?
Solution:

(a) Find the limit from the left at x = −3. This entails identifying which line of
9
the definition is to be used. Here lim − f (x) = −3 = −3.
x→−3

(b) Find the limit from the right at x = −3. This entails identifying which line
of the definition is to be used. Here lim + f (x) = −6 + b + ln(−3 + 4) =
x→−3
−6 + b + 0 = b − 6.
(c) Continuity requires that the limit exists; that is equivalent to the existence of
the 2 one-sided limits, and the equality of these 2 one-sided limits:

−3 = b − 6

so b = 3.
(d) Continuity requires that the limit be equal to the function value. Use the
preceding facts to observe the value of the 2-sided limit to be −3.
(e) Finally, the limit must be equal to the function value:
1
−3 = ab ⇒ −3 = 6a ⇒ a = − .
2
Information for Students in MATH 140 2008 09 3123

2. Let
−4 + 9ex
f (x) = .
−3 + 7ex
Compute f 0 (x) and determine the equation of the tangent line to the graph of f
at x = 0.
Solution:

(a) Applying the Differentiation Rules, we obtain

(9ex ) · (−3 + 7ex ) − (−4 + 9ex ) (7ex )


f 0 (x) =
(−3 + 7ex )2
ex
= .
(−3 + 7ex )2
1
(b) When x = 0, f 0 (0) = 42
.
−4 + 9 5
(c) When x = 0, f (x) = = .
−3 + 7 2
(d) Hence an equation for the line through the point x = 0 and having the correct
slope is
5 1
y − = (x − 0) .
2 16
Or, equivalently, x − 16y = −40.

Friday version
1. Use the limit definition of the derivative to compute the derivative of

f (x) = 3x + 4 .

Don’t use any of the differentiation rules of Chapter 3 of Stewart!


Solution:

(a) Give the definition of the derivative as a limit (several alternatives are possi-
ble):
f (x + h) − f (x)
f 0 (x) = lim
h→0 h
(b) Express that general limit in the particular case of the given function
p √
3(x + h) + 4 − 3x + 4
f 0 (x) = lim
h→0 h
Information for Students in MATH 140 2008 09 3124

(c) Simplify the fraction algebraically:


p √ p √
3(x + h) + 4 − 3x + 4 3x + (3h + 4) − 3x + 4
lim = lim
h→0 h h→0 h

(d) Rationalize the numerator by multiplying the fraction by


p √
3x + (3h + 4) + 3x + 4
p √
3x + (3h + 4) + 3x + 4

(e) Simplify the numerator


Ãp √ ! Ãp √ !
3x + (3h + 4) − 3x + 4 3x + (3h + 4) + 3x + 4
lim p √
h→0 h 3x + (3h + 4) + 3x + 4
(3x + 3h + 4) − (3x + 4)
= lim p √
h→0 h( 3x + (3h + 4) + 3x + 4)
3h
= lim p √
h→0 h( 3x + (3h + 4) + 3x + 4)

(f) Divide numerator and denominator by h:


3
lim p √
h→0 3x + (3h + 4) + 3x + 4

(g) Now the Quotient Law may be used (or continuity may be invoked):
3 3 3
lim p √ =p √ = √
h→0 3x + (3h + 4) + 3x + 4 3x + 4) + 3x + 4 2 3x + 4

2. Let f be a differentiable function defined for all real numbers. Assume it is known
that f (0) = 9. Compute the derivative of the function

xf (x) − 8
g(x) =
ex
at x = 0.
Solution:
Information for Students in MATH 140 2008 09 3125

(a)

(5xf (x) − 8)0 · ex − (5xf (x) − 8) · (ex )


g 0 (x) =
(ex )2
by the Quotient Rule
(5xf (x))0 · ex − (5xf (x) − 8) · (ex )
=
(ex )2
by the Sum Rule
(5f (x) + 5xf 0 (x)) · ex − (5xf (x) − 8) · (ex )
=
(ex )2
by the Product Rule
5(f (x) + xf 0 (x) − xf (x)) + 8
=
ex
(b)
5f (0) + 8
g 0 (0) = = 5 · 9 + 8 = 53.
1

7.4.3 Draft Solutions to Quiz Q3


Distribution Date: November 11th, 2006; (subject to correction)

The following draft sketches of solutions will be given for one specific set of data for
each of the versions of the quiz; in some problems the variations of the versions involve
different functions, so please be careful.

Instructions to students
1. Show all your work. Marks are not given for answers alone.

2. You must enclose this question sheet in your folded answer sheet.

3. Time = 20 minutes

4. No calculators are permitted.

Monday version
1. In answering each of the following problems, show all your work, and simplify your
answer as much as possible.

(a) Find f 0 (x), when f (x) = arcsin(4x − 1).


Information for Students in MATH 140 2008 09 3126

sin 5x
(b) Evaluate lim without using L’Hospital’s Rule.
x→0 sin 2x

Solution:
(a) i.
1 d
f 0 (x) = p · (4x − 1)
1 − (4x − 1)2 dx
ii. Evaluating the 2nd factor:
1
f 0 (x) = p ·4
1 − (4x − 1)2
iii. Simplifying the first factor:
1 2
f 0 (x) = √ ·4= .
8x − 16x2 2x − 4x2

(b) Remember, if the student’s proof is essentially using L’Hospital’s Rule, then
she should not receive any marks.
i. Transform so that numerator and denominator involve limits of sin x/x:
sin 5x
sin 5x 5x
= 5x ·
sin 2x sin 2x 2x
2x
ii.
 
sin 5x
sin 5x  5x 
lim = lim  5x · 
x→0 sin 2x x→0 sin 2x 2x
2x
sin 5x
lim 5x
= x→0 5x · lim
sin 2x x→0 2x
lim
x→0 2x
sin 5x
lim 5
= 5x→0 5x · lim
sin 2x x→0 2
lim
2x→0 2x

iii.
sin 5x 1 5 5
lim = · =
x→0 sin 2x 1 2 2
Information for Students in MATH 140 2008 09 3127

2. Suppose that
4x2 + 9y 2 = 36 . (233)

(a) Find y 0 only by implicit differentiation. Your solution will involve both x and
y.
(b) Assuming y > 0, solve the given equation explicitly for y, and differentiate
the solution with respect to x to obtain an explicit formula for y 0 .
(c) Substitute the solution you found for y in part (b) into your derivative y 0
found in part (a), and show that the result is the same as the value you found
for y 0 in (b).

Solution:

(a) Differentiate equation (233) implicitly with respect to x:

dx dy
4 · 2x · + 9 · 2y =0
dx dx
which can be solved to yield
dy 4x
=− .
dx 9y

(b) If y > 0,
2√
y= 9 − x2 . (234)
3
Differentiating the explicit formula for f in (234) yields

dy 2 1 1 2x 1
= · ·√ · (−2x) = − · √ .
dx 3 2 9 − x2 3 9 − x2

(c)
4x 4x
− = − ¡2√ ¢
9y 9 3 9 − x2
2 1
= − ·√
3 9 − x2
which is the value that we determined in part (b) by explicit differentiation.
Information for Students in MATH 140 2008 09 3128

Wednesday version
1. In answering each of the following problems, show all your work, and simplify your
answer as much as possible.
¡√ ¢
(a) Find f 0 (x), when f (x) = arctan x2 + 1 − x .
sin 7x2
(b) Evaluate lim without using L’Hospital’s Rule.
x→0 sin 3x2

Solution:
(a) i.
0 1 d ³√ 2 ´
f (x) = ¡√ ¢2 · x +1−x
1+ x2 + 1 − x dx
ii. µ ¶
0 1 1 2x
f (x) = ¡√ ¢2 · ·√ −1
1+ x2 + 1 − x 2 x2 + 1
iii. Final simplification:
1 1
f 0 (x) = − · 2
2 x +1
(b) Remember, if the student’s proof is essentially using L’Hospital’s Rule, then
she should not receive any marks.
i. Transform so that numerator and denominator involve limits of functions
of the form sin x/x:
sin 7x2
sin 7x2 7x2 · 7x
2
=
sin 3x2 sin 3x2 3x2
3x2
ii.
 
sin 7x2
sin 7x2  7x2 7x2 
lim = lim  · 
x→0 sin 3x2 x→0 sin 3x2 3x2
3x2
sin 7x2
lim 2 7x2
= x→0 7x 2 · lim 2
sin 3x x→0 3x
lim
x→0 3x2
sin 7x2
lim 2 7
= 7x →0 7x 2 · lim
2

sin 3x x→0 3
lim
3x2 →0 3x2
Information for Students in MATH 140 2008 09 3129

iii.
sin 7x2 1 7 7
lim = · =
x→0 sin 3x2 1 3 3
2. Suppose that ¡ ¢2
x2 + 64y 2 = x2 + 2y 2 − 24x .
Determine an equation — simplified as much as possible — for the tangent to this
curve at the point with coordinates (0, 4).
Solution:
(a) Differentiate the given equation implicitly with respect to x:
¡ ¢
2x + 64 · 2y · y 0 = 2 x2 + 2y 2 − 24x · (2x + 4yy 0 − 24)

(b) Substitute (x, y) = (0, 4) in the equation for y 0 to determine the value of that
derivative when x = 4:

0 + 512y 0 = 2(32)(16y 0 − 24) ⇒ y 0 = 3 .

Alternatively, one could determine a general formula for y 0 for any point on
the curve, and then make the substitution.
(c) Give the equation with of the line through (4, 3) with the given slope:

y − 4 = 3(x − 0) or y = 3x + 4 .

Thursday version
1. In answering each of the following problems, show all your work, and simplify your
answer as much as possible.

(a) Find f 0 (x), when f (x) = x arcsin x + −x2 + 1.
Solution:
i.
1 d√ 2
f 0 (x) = 1 · arcsin x + x · √ + −x + 1
−x2 + 1 dx
ii.
1 1 1
f 0 (x) = 1 · arcsin x + x · √ + ·√ · (−2x)
−x2 + 1 2 −x2 + 1
iii. Final simplification:
1 1
f 0 (x) = 1 · arcsin x + x · √ −x· √ = arcsin x .
−x2 + 1 −x2 + 1
Information for Students in MATH 140 2008 09 3130

sin(cos 2x)
(b) Evaluate lim without using L’Hospital’s Rule.
x→0 sec 2x
Solution: For this problem l’Hospital’s Rule could not apply, since the limit of
the denominator exists, and is not zero. You certainly should not give more
than 0 if a student appeals to l’Hospital’s Rule here!
i. The limit of the denominator is sec 0 = 1, by the continuity of the secant
function.
ii. The limit of the numerator is

lim sin(cos 2x) = sin(lim cos 2x)


x→0 x→0
since the sine function is continuous at lim cos 2x
x→0
= sin 1.

sin(cos 2x) lim sin(cos 2x) sin 1


x→0
iii. By the Quotient Law, lim = = = sin 1.
x→0 sec 2x lim sec 2x 1
x→0

2. Suppose that ¡ ¢
3(x2 + y 2 )2 = 25 x2 − y 2 .
Determine an equation — simplified as much as possible — for the tangent to this
curve at the point with coordinates (2, 1).
Solution:
(a) Differentiate the given equation implicitly with respect to x:

3 · 2 · (x2 + y 2 ) · (2x + 2yy 0 ) = 25 (2x − 2yy 0 )

(b) Solve for


x 25 − 6x(x2 + y 2 )
y0 = · .
y 6(x2 + y 2 ) + 25
and evaluate at (x, y) = (2, 1):
2
y0 = − .
11
Alternatively, substitute (x, y) = (2, 1) before solving for the value of the
derivative at the point.
(c) Give the equation with of the line through (2, 1) with the given slope:
µ ¶
2
y−1= − (x − 2) or 2x + 11y = 15 .
11
Information for Students in MATH 140 2008 09 3131

Friday version
1. In each of the following problems, show all your work, and simplify your answer as
much as possible.
¡√ ¢
(a) Find f 0 (x), when f (x) = −x2 + 1 arccos x.
t3
(b) Evaluate lim without using L’Hospital’s Rule.
t→0 tan3 (8t)

Solution:

(a) i.
1 1 √ d
f 0 (x) = √ · (−2x) · arccos x + −x2 + 1 · arccos x
2 −x + 1
2 dx
ii.
1 1 √ −1
f 0 (x) = √ · (−2x) · arccos x + −x2 + 1 · √
2 −x2 + 1 1 − x2
iii. Simplification:
1 1 √ −1
f 0 (x) = √ · (−2x)) · arccos x + −x2 + 1 · √
2 −x2 + 1 1 − x2
x arccos x
= −√ −1
1 − x2

(b) Remember, if the student’s proof is essentially using L’Hospital’s Rule, then
she should not receive any marks.
i. Transform so that numerator and denominator involve limits of functions
of the form sint t or tant t :

t3 1 1 cos3 8t 1
3
= µ ¶ 3 · 3
= µ ¶3 · 3
tan 8t tan 8t 8 sin 8t 8
8t 8t
ii. Now prepare to use the Quotient Law for Limits:
t3 cos3 8t 1
lim 3
= lim µ ¶3 · 3
t→0 tan 8t t→0 sin 8t 8
8t
lim cos3 8t 1
t→0
= µ ¶3 · 3
sin 8t 8
lim
t→0 8t
Information for Students in MATH 140 2008 09 3132

lim cos3 8t 1
= µ t→0 ¶3 · 3
sin 8t 8
lim
t→0 8t
³ ´3
lim cos 8t 1
= µ 8t→0 ¶3 · 3
sin 8t 8
lim
8t→0 8t

iii. Complete the computations and simplify:

13 1 1
= 3
· 3 =
1 8 512
2. Suppose that
y 2 (y 2 − 16) = x2 (x2 − 5) .
Determine an equation — simplified as much as possible — for the tangent to this
curve at the point with coordinates (0, 4).
Solution:

(a) Prior to implicit differentiation, prepare by replacing the products of polyno-


mials by sums of constant multiples of powers:

y 4 − 16y 2 = x4 − 5x2 .

(b) Differentiate implicitly with respect to x:

4y 3 y 0 − 16yy 0 = 4x3 − 10x .

(c) Determine a general formula for the derivative:

x 2x2 − 5
y0 = ·
y 2(y 2 − 2)

(d) Evaluate y 0 at the given point:

y 0 (0) = 0 .

(e) Give the equation of the line with that slope through the given point: y = 4.
Information for Students in MATH 140 2008 09 3133

7.4.4 Draft Solutions to Quiz Q4


Distribution Date: Mounted on WebCT on December 01, 2006
Draft, subject to correction.
The following sketches of solutions will be given for one specific set of data for each of
the versions of the quiz.

Instructions to students
1. Show all your work. Marks are not given for answers alone.
2. You must enclose this question sheet in your folded answer sheet.
3. Time = 45 minutes.
4. No calculators are permitted.

Monday version
1. [10 MARKS] Find a function
f (x) = ax2 + bx + c (235)
where a, b and c are constants to be determined, if f has all of the following
properties:
(a) f (1) = 0
(b) f 0 (3) = −13
(c) f 00 (−7) = −4
You must show a FULL solution—a formula for the function alone could be worth
zero marks.
Solution:
(a) Imposition of the first boundary condition yields
0 = f (1) = a + b + c . (236)

(b) Differentiation of equation (235) yields


f 0 (x) = 2ax + b . (237)
Imposition of the second boundary condition yields
−13 = f 0 (3) = 2a(3) + b . (238)
Information for Students in MATH 140 2008 09 3134

(c) Differentiation of equation (237) yields

f 00 (x) = 2a . (239)

Imposition of the third boundary condition yields

−4 = f 00 (−7) = 2a , (240)

implying that a = −2.


(d) Back substitution of this value in equation (238) yields

−13 = 2(−2)(3) + b ,

implying that b = −1. Back substitution of the values of a and b in equation


(236) yields
0 = (−2) + (−1) + c ,
which we may solve to obtain c = 3. The function we seek is now completely
determined:
f (x) = −2x2 − 1x + 3 .

2. [10 MARKS] Use logarithmic differentiation to find the derivative of the function
y = x4x .
Solution:

(a) First apply logarithms to both sides of the given defining equation:

ln y = 4x ln x .

(b) Next differentiate implicitly with respect to x:

y0 1
= 4 ln x + 4(x) = 4 ln x + 4 ,
y x
implying that
y 0 = 4y · (ln x + 1) .
(c) Now restore the known explicit formula for y(x):

y 0 = 4x4x · (ln x + 1) .

3. [10 MARKS] If sinh x = −1, find the value of ex and then the value of cosh x.
Solution:
Information for Students in MATH 140 2008 09 3135

x −x
(a) In terms of exponentials, the given equation yields e −e
2
= −1 which implies
2x x
that e − 1 = −2e , which may be viewed as a quadratic equation,

(ex )2 + 2ex − 1 = 0 ,

implying that ex = −1 ± 2. But, of the two values given, one must be
discarded, as an exponential cannot be negative. Thus we have shown that

ex = 2 − 1 .

(b) We may now determine the value of the hyperbolic cosine:


1
ex +
cosh x = ex
√ 2
2−1+ √1
2−1
=
2
√ √
2−1+ √1 · √2+1
2−1 2+1
=
2√
√ 2+1
2−1+ 1
=
√ 2
= 2.

(The instructions clearly stated that it was necessary to find ex first. Otherwise one
could have applied the identity cosh2 x−sinh2 x = 1 to determine that cosh x = ±2;
then one could have observed that the hyperbolic cosine, being a positive multiple
of the sum of exponentials, must be positive, etc.)

4. [10 MARKS] Find the global (absolute) extrema of the function

ln(4x)
y=
x

on the closed interval [1, 8]. You must show all of your work—correct numerical
answers alone may not earn any marks.
Solution:

(a) To find the critical points, differentiate


1 − ln(4x)
y 0 (x) = .
x2
Information for Students in MATH 140 2008 09 3136

(b) Observe that there are no points in the given interval where the function fails
to be differentiable.
(c) Determine the critical points by setting the derivative equal to 0: ln 4x = 1 ⇒
x = 4e < 1, which is not in the interval. Thus there are NO critical points!
(d) Compare the values of the function at the end points:
y(1) = ln 4 = 2 ln 2
ln 32 5
y(8) = = ln 2
8 8
so the global maximum is at 1, and the maximum value is ln 4; the global
minimum is at 8, and the minimum value is 85 ln 2.

Wednesday version
1. [10 MARKS] Find a function
f (x) = a cos x + b sin x + c (241)
where a, b and c are constants to be determined, if f has all of the following
properties:
(a) f (0) = −1

(b) f 0 (− π4 ) = −2 2
(c) f 00 (π) = 1
You must show a FULL solution—a formula for the function alone could be worth
zero marks.
Solution:
(a) Imposition of the first boundary condition yields
−1 = f (0) = a + b(0) + c = a + c . (242)

(b) Differentiation of equation (241) yields


f 0 (x) = −a sin x + b cos x . (243)
Imposition of the second boundary condition yields
√ ³ π´ −1 1
0
−2 2 = f − = −a · √ + b · √ , (244)
4 2 2
implying that
a + b = −4 . (245)
Information for Students in MATH 140 2008 09 3137

(c) Differentiation of equation (243) yields

f 00 (x) = −a cos x − b sin x . (246)

Imposition of the third boundary condition yields

1 = f 00 (π) = −a(−1) + b(0) , (247)

implying that a = 1.
(d) Back substitution of this value in equation (245) yields

1 + b = −4 ,

implying that b = −5. Back substitution of the values of a and b in equation


(242) yields
−1 = 1 + c ,
which we may solve to obtain c = 4. The function we seek is now completely
determined:
f (x) = 1 cos x − 5 sin x + 4 .

2. [10 MARKS] Use logarithmic differentiation to find the derivative of the function
y = x2/x .
Solution:

(a) First apply logarithms to both sides of the given defining equation:
2
ln y = · ln x .
x

(b) Next differentiate implicitly with respect to x:


1
y0 x
· x − (ln x) · 1
=2· ,
y x2
implying that
1 − ln x
y 0 = 2y · .
x2
(c) Now restore the known explicit formula for y(x):

2 1 − ln x
y 0 = 2x x · .
x2
Information for Students in MATH 140 2008 09 3138

3. [10 MARKS] If tanh x = 35 , find the value of ex and then the value of cosh x.
Solution:
(a) Write the given equation in terms of exponentials:
ex − e−x 3
x −x
= .
e +e 5
(b) Solve for ex : 2e2x = 8 ⇒ ex = ±2 ⇒ ex = +2 since exponentials cannot be
negative.
(c) Now express cosh x in terms of exponentials, and, through that, determine
the value of cosh x.
1
ex + e−x 2+ 2 5
cosh x = = = .
2 2 4

4. [10 MARKS] Use a linear approximation to estimate the number 99.9. You
must show all your work, giving the precise values assigned to any variables or
increments.
Solution:

(a) Select function, e.g., f (x) = x. Also select the reference point, and, thereby
the increment: a = 100, ∆x = −0.1. Thus f (a) = 10.
1
(b) Determine the derivative of f , and evaluate at a: f 0 (x) = √
2 x
. At a = 100,
f 0 (a) = 2√1100 = 20
1
.
(c) Complete the approximation:
f (a + ∆x) ≈ f (x) + f 0 (a) · ∆x
1
= 10 + · (−0.1)
20
= 10 − 0.025 = 9.975 .

Thursday version
1. [10 MARKS] The volume of a right circular cone is V = 13 πr2 h, where r is the
radius of the base and h is the height. Assume that the height of such a cone
remains constant at 6 centimeters, while its volume is expanding at a constant
rate of 4 cubic centimeters per second. Find the rate of change of the radius when
the radius is 2 centimeters. Simplify your answer as much as is possible without a
calculator.
dV
Solution: V = π3 hr2 ⇒ dV
dt
= π3 h · 2r · dr
dt
⇒ dr
dt
= 2
dt
πrh
= 2
4
π(2)(6)
= 1

.
3 3
Information for Students in MATH 140 2008 09 3139

2. [10 MARKS] Use logarithmic differentiation to find the derivative of the function
y = x9 cos x .
Solution:

(a) First apply logarithms to both sides of the given defining equation:

ln y = 9 cos x · ln x .

(b) Next differentiate implicitly with respect to x:

y0
= 9(− sin x) · ln x + 9 cos x · 1x ,
y
implying that
y 0 = 9y · ((− sin x) · ln x + 9 cos x · 1x) .
(c) Now restore the known explicit formula for y(x):

y 0 = 9x9 cos x · ((− sin x) · ln x + 9 cos xx) .

3. [10 MARKS] Find the global (absolute) extrema of the function



y = x 2 − x2

on the closed interval [−1, 1]. You must show all of your work—correct numerical
answers alone may not earn any marks.
Solution:

(a) Find the derivative of the function:


√ 1 1 2(1 − x)(1 + x)
y0 = 2 − x2 + x · √ = √
2 2−x 2 2 − x2

(b) Determine the critical points of the function on the given interval: f 0 (x) =
0 ⇒ x = ±1; there are no critical points of the type where the derivative does
not exist. And the only critical points are at the end-points of the interval.
Our definition of critical point does not include end-points; but it doesn’t
matter, because end-points must be checked separately in any case.
(c) Under the Closed Interval Method we need check only the end-points. Since
y(−1) = −1 and y(+1) = +1, the global maximum — of value +1 — occurs
at x = +1; while the global minimum — of value −1 — occurs at x = −1.
Information for Students in MATH 140 2008 09 3140

4. [10 MARKS] Use a linear approximation to estimate the number ln(1.05). You
must show all your work, giving the precise values assigned to any variables or
increments.
Solution:
(a) Select function, e.g., f (x) = ln x. Also select the reference point, and, thereby
the increment: a = 1, ∆x = +0.05. Thus f (a) = 0.
1
(b) Determine the derivative of f , and evaluate at a: f 0 (x) = x
. At a = 1,
f 0 (a) = 11 = 1.
(c) Complete the approximation:
f (a + ∆x) ≈ f (x) + f 0 (a) · ∆x
1
= 0 + · (0.05)
1
= 0.05 .

Friday version
1. [10 MARKS] The volume of a right circular cone is V = 13 πr2 h, where r is the
radius of the base and h is the height. Assume that the volume of such a cone
remains constant at 4 cubic centimeters, while its radius is increasing at a constant
rate of 2 centimeters per second. Find the rate of change of the height when the
radius is 3 centimeters. Simplify your answer as much as is possible without a
calculator.
π 3V 12 1 dh
Solution: V = 3
· r2 h ⇒ h = π
· 1r2 = π
· r2
. Hence dt
= − 24
π
· 1r3 · dr
dt
8
= − 9π .
2. [10 MARKS] Use logarithmic differentiation to find the derivative of the function
y = (ln x)6/x .
Solution:
(a) First apply logarithms to both sides of the given defining equation:
ln ln x
ln y = 6 .
x
(b) Next differentiate implicitly with respect to x:
µ 1 ¶
y0 x ln x
· x − ln ln x · 1
=6 ,
y x2
implying that µ ¶
1
0 ln x
− ln ln x
y = 6y .
x2
Information for Students in MATH 140 2008 09 3141

(c) Now restore the known explicit formula for y(x):


µ 1 ¶
0 6
ln x
− ln ln x
y = 6(ln x) x .
x2

3. [10 MARKS] Use a linear approximation to estimate the number (0.99)6 . You
must show all your work, giving the precise values assigned to any variables or
increments.
Solution:

(a) Select function, e.g., f (x) = x6 . Also select the reference point, and, thereby
the increment: a = 1, ∆x = −0.01. Thus f (a) = 1.
(b) Determine the derivative of f , and evaluate at a: f 0 (x) = 6x5 . At a = 1,
f 0 (a) = 11 = 1.
(c) Complete the approximation:

f (a + ∆x) ≈ f (x) + f 0 (a) · ∆x


1
= 0 + · (0.05)
1
= 0.05 .

4. [10 MARKS] Show that the polynomial 5x4 − 9x − 4 has exactly 2 real roots.
Carefully justify all your steps and name the theorems you’re using!
Solution:

7.5 Quizzes from MATH 140 2007 09, with Draft Solutions
7.5.1 Draft Solutions to Quiz Q1
Release Date: 11 October, 2007
These are sketches of possible solutions to types of problems on the quiz. They haven’t
been fully checked, and there could be some errors that have to be corrected.

There were many versions of these quizzes. The solutions below are to typical examples,
with specific choices of data. In all cases it was intended that students provide supporting
explanations for their answers.
¡√ ¢
1. [3 MARKS] Find the domain of the function h(x) = (x + 8)2 4 5x − 25 .
Solution: This function is the product of two functions. Thepfirst, the polynomial
2
(x + 8) , is defined for all real numbers x. But the second, 5(x − 5), is defined
4
Information for Students in MATH 140 2008 09 3142

only if the function under the root sign is not negative. Thus this latter factor
requires that x ≥ 5, and that restriction is sufficient for h to be well defined: the
domain is (5, +∞).

x+1
2. [3 MARKS] Find the domain of the function f (x) = .
x−2
Solution: This function is the quotient of two functions. The numerator is defined
when the argument of the square root is not negative, i.e., for x ≥ −1. The
denominator is defined for all x; but, it being a denominator, we require that it be
non-zero, requiring the exclusion of the value x = 2 from the domain. Thus the
domain of f is obtained by puncturing the interval [−1, +∞) at x = 2:

[−1, 2) ∪ (2, +∞) .

3. [3 MARKS] If f (x) = 2x − 6 and g(x) = 4 − x2 , evaluate the expressions f (g(5)),


g(f (5)), f (f (5), showing all your work.
Solution:
¡ ¢
f (g(x)) = f 4 − x2
¡ ¢
= 2 4 − x2 − 6 = −2x2 + 2
⇒ f (g(5)) = −2(25) + 2 = −48
g(f (x)) = g(2x − 6)
= 4 − (2x − 6)2 = −4x2 + 24x − 32
⇒ g(f (5)) = −4(25) + 24(5) − 32 = −12
f (f (x)) = f (2x − 6)
= 2(2x − 6) − 6 = 4x − 18
⇒ f (f (5)) = 4(5) − 18 = 2 .

8 x − 11
4. [3 MARKS] Find the domain of (f ◦g)(x) if f (x) = , g(x) = . Express
x − 11 x+9
as an interval or union of intervals.
µ ¶
x − 11 8 4 x+9
Solution: (f ◦ g)(x) = f (g(x)) = f = x−11 =− · , at all
x+9 x+9
− 11 5 x + 11
points in the domain of g which are also in the domain of f . We must be cautious
not to base our reasoning on the reduced form of the function given finally above:
all steps in the computation of the composite function must be meaningful at any
point in its domain. In this final form the factor x + 9 does not appear in the
denominator, and we might be tempted to conclude that −9 is in the domain
Information for Students in MATH 140 2008 09 3143

of the composite function; but that is not the case, because the first step in the
calculation requires that g be defined, and −9 is not in the domain of g.
The first function to be applied is g, whose domain is R − {−9}, since we must
delete from the domain of the numerator function x − 11 the one point where the
denominator would be equal to 0. Then we have to apply the function f , whose
domain is — by similar reasoning — R − {11}; but it is not x = 11 that must be
deleted from the domain R − {−9} — we must delete the value x, if any, where
g assumes the value 11: that is, we must delete from the domain any point(s) x
x − 11
such that 11 = . The only point x with this property is x = −11. Thus the
x+9
domain of f ◦ g is R − {−9, −11}.

5. [3 MARKS] Showing all your work, find the inverse function f −1 (x) of f (x) =
7 − 5x
.
6 − 2x
7 − 5x
(a) Set y = f (x) = . Note that the point x = 3, which causes the denomi-
6 − 2x
nator of the fraction to be 0, is not in the domain of f .
7 − 6y
(b) Solve for x in terms of y: x = ; excluded is the value y = 52 .
5 − 2y
(c) Rewrite, naming the independent variable x: the inverse function is given by
7 − 6x
f −1 (x) = . Its domain is R − { 52 }.
5 − 2x
[3 MARKS] Showing all your work, find the inverse function f −1 (x) of f (x) =
6. √
10x + 7.
√ 7
(a) Set y = f (x) = 10x + 7. Note that the domain of f is x ≥ − .
10
y2 − 7
(b) Solve for x in terms of y: squaring and reduction yield x = .
10
(c) Rewrite, naming the independent variable x: the inverse function is given by
x2 − 7
f −1 (x) = . Its domain is [0, +∞] (corresponding to the fact that the
10
image of f is [0, +∞]. (For the purposes of this course it wouldn’t be wrong to
think of the domain of f −1 as being R; but the algebraic relationship between
f and its inverse is more attractive if we take the domain of each to be the
image of the other.)

7. [3 MARKS] Showing all your work, find the inverse function f −1 (x) of f (x) =
¡ ¢1
6 − x3 5 .
Information for Students in MATH 140 2008 09 3144

¡ ¢1
(a) Set y = f (x) = 6 − x3 5 . Note that the domain of f is R.
(b) p
Solve for x in terms of y: taking the 5th power and reducing yield x =
3
6 − y5.
(c) Rewrite, naming
√ the independent variable x: the inverse function is given by
−1 3
f (x) = x = 6 − x5 . Its domain is R.

8. [3 MARKS]
√ Showing all your work, find the inverse function f −1 (x) of f (x) =
9 + x + 2.

(a) Set y = f (x) = 9 + x + 2. Note that the domain of f is x ≥ −2, the set of
all points where the square root is defined.
(b) Solve for x in terms of y: subtracting 9 from both sides and squaring yield
x = (y − 9)2 − 2.
(c) Rewrite, naming the independent variable x: the inverse function is given by
f −1 (x) = (x − 9)2 − 2 = x2 − 18x + 79. Its domain is [9, +∞) (corresponding
to the fact that the image of f is [9, +∞]. For the purposes of this course it
wouldn’t be wrong to think of the domain of f −1 as being R; but the algebraic
relationship between f and its inverse is more attractive if we take the domain
of each to be the image of the other.)

[3 MARKS] Showing all your work, find the inverse function f −1 (x) of f (x) =
9. √
81 − x2 , (0 ≤ x ≤ 9).

(a) Set y = f (x) = 81 − x2 . If we had not additional information, we could
observe that the domain of f is −9 ≤ x ≤ 9, the set of all points where
the square root is defined. However, we have been told in advance that the
domain is only the interval 0 ≤ x ≤ 9.
p
(b) Solve for x in terms of y: squaring both sides and reducing yield x = ± 81 − y 2 .
If we had no additional information, we would have to stop here, and assert
that f does not have an inverse. However, we are told that the domain of
f is 0√≤ x ≤ 9; this permits us to drop the minus sign, and to assert that
y = 81 − x2 . We √ can also observe that the values of this function range
between 0 and + 81 = 9.
(c) Rewrite, naming
√ the independent variable x: the inverse function is given by
−1
f (x) = 81 − x2 . Its domain is [0, 9] (corresponding to the fact that the
image of f is [0, 9]). Thus it happens that f is its own inverse!

10. [3 MARKS] Showing all your work, use the Laws of Logarithms to rewrite the
following expression in a form with no logarithm of a product, quotient, root, or
Information for Students in MATH 140 2008 09 3145

power: s
x4 + 4
ln .
(x2 + 1) (x3 − 8)3

Solution:
s µ ¶ 12
x4 + 4 x4 + 4
ln = ln
(x2 + 1) (x3 − 8)3 (x2 + 1) (x3 − 8)3
1 x4 + 4
= ln
2 (x2 + 1) (x3 − 8)3
1³ ¡ 4 ¢ ³¡
2
¢¡ 3 ¢3 ´´
= ln x + 4 − ln x + 1 x − 8
2
1¡ ¡ 4 ¢ ¡ ¡ ¢ ¡ ¢¢¢
= ln x + 4 − ln x2 + 1 + 3 ln x3 − 8
2
1¡ ¡ 4 ¢ ¡ ¢ ¡ ¢¢
= ln x + 4 − ln x2 + 1 − 3 ln x3 − 8
2

11. [3 MARKS] Showing all your work, use the Laws of Logarithms to rewrite the
following expression in a form with no logarithm of a product, quotient, root, or
power: r q
3 √
ln x y z .

Solution:
r q µ q ¶1
3 √ 3 √ 2
ln x y z = ln x y z
µ q ¶
1 3 √
= ln x y z
2
µ µq ¶¶
1 3 √
= ln x + ln y z
2
1³ ³¡ √ ¢ 1 ´´
= ln x + ln y z 3
2µ ¶
1 1 ¡ √ ¢
= ln x + ln y z
2 3
µ ¶
1 1 1 √
= ln x + ln y + ln z
2 3 3
µ ¶
1 1 1 1
= ln x + ln y + · ln z
2 3 3 2
Information for Students in MATH 140 2008 09 3146

1 1 1
= ln x + ln y + ln z
2 6 12

12. [3 MARKS] Showing all your work, use the Laws of Logarithms to rewrite the
following expression in a form with no logarithm of a product, quotient, root, or
power: Ã !
4

z x
ln p .
6 2
y + 4y + 16

Solution:
à √ !
z4 x ¡ √ ¢ ³p ´
ln p = ln z 4 x − ln 6 y 2 + 4y + 16
6
y 2 + 4y + 16
³ 1
´ ³¡ ¢1 ´
= ln z 4 x 2 − ln y 2 + 4y + 16 6
¡ 4¢ ³ 1´ ³¡ ¢ 16 ´
2
= ln z + ln x − ln y + 4y + 16
2

1 1 ¡ ¢
= 4 ln z + ln x − ln y 2 + 4y + 16
2 6

13. [3 MARKS] Showing all your work, solve the equation x6 2x − 2x = 0.


Solution:
¡ ¢
x6 2x − 2x = 0 ⇔ x6 − 1 2x = 0
⇔ x6 − 1 = 0
since 2x is non-zero for all values of x ,

of which both +1 and −1 are (the only real) solutions.

14. [3 MARKS] Showing all your work, solve the equation e2x − 3ex + 2 = 0.
Solution: The given equation is a quadratic equation in ex . It factorizes to yield

(ex − 1) (ex − 2) = 0 .

This equation is satisfied when either of the two factors is 0, i.e., when either Case
1: ex = 1 or Case2: ex = 2.

Case 1: ex = 1. To solve this equation take (natural) logarithms of both sides, to


find that x = 0 uniquely.
Case 2: ex = 2. Here again, if we take logarithms of both sides, we obtain x =
ln 2.
Information for Students in MATH 140 2008 09 3147

Thus the given equation is satisfied only for x = 0, ln 2.

15. [3 MARKS] Showing all your work, solve the following equation for x: log2 5 +
log2 x = log2 7 + log2 (x − 2).
Solution:

log2 5 + log2 x = log2 7 + log2 (x − 2) ⇔ log2 (5x) − log2 (7(x − 2)) = 0


µ ¶
5x
⇔ log2 =0
7(x − 2)
5x
⇔ =1
7(x − 2)
taking exponentials to base 2
⇔ x=7

16. [3 MARKS] Showing all your work, solve the following equation for x: log5 (x +
2) − log5 (x − 2) = 2.
Solution:

log5 (x + 2) − log5 (x − 2) = 2 ⇔ log5 ((x − 2)(x + 2)) = 2


⇔ 5(log5 ((x−2)(x+2))) = 52
⇔ (x − 2)(x + 2) = 25
⇔ x2 = 29

⇔ x = ± 29

17. [3 MARKS] Showing all your work, determine for which value(s) of x the following
equation is true: ln(x + 6) = ln x + ln 6.
Solution:

ln(x + 6) = ln x + ln 6 ⇔ ln(x + 6) − ln(6x) = 0


µ ¶
x+6
⇔ ln =0
6x
x+6
⇔ =1
6x
taking logarithms to base e on both sides of the equation
6
⇔ x + 6 = 6x ⇔ x = .
5
Information for Students in MATH 140 2008 09 3148

18. [3 MARKS] Showing all your work, solve for x: log2 (log3 x) = 4.

log2 (log3 x) = 4 ⇔ 2log2 (log3 x) = 24


⇔ log3 x = 24 = 16
⇔ 3log3 x = 316
⇔ x = 316 .

19. [3 MARKS] Solve the inequality x2 ex − 6ex < 0.


Solution:
¡ ¢
x2 ex − 6ex < 0 ⇔ x2 − 6 ex < 0
⇔ x2 − 6 < 0
since ex is positive for all x
³ √ ´³ √ ´
⇔ x+ 6 x− 6 <0
√ √
⇔ − 6<x< 6

since the product can be negative only if precisely one of the two factors is negative,
and that can be arranged only by placing x between the roots of the quadratic
function x2 − 6.

20. [3 MARKS] Showing all your work, find in the interval [0, 2π) all solutions to the
equation sin x cos x − 4 sin x = 0.
Solution:
sin x cos x − 4 sin x = 0 ⇔ (sin x) · (cos x − 4) = 0.
But the second factor can never be zero, since the cosine cannot exceed 1 in magni-
tude. Thus we can divide by this second factor, leaving sin x = 0, which is satisfied
in the given interval for x = 0, π.

21. [3 MARKS] Showing all your work, find, in the interval [0, 2π), all solutions of the
equation sin2 x = 4 sin x + 5.
Solution: The given equation is quadratic in sin x:

sin2 x = 4 sin x + 5 ⇔ (sin x + 1)(sin x − 5) = 0 .

Of the two factors on the left side of this equation, the second can never be 0,
since the magnitude of the sine cannot exceed 1. Thus the equation is equivalent
to sin x + 1 = 0, which is equivalent to sin x = −1, which is satisfied in the given
interval only for x = 3π
2
.
Information for Students in MATH 140 2008 09 3149

22. [3 MARKS] Showing all your work, find, in the interval [0, 2π), all solutions of the
equation sin2 x − cos2 x = 0. (Hint: You may wish to use a trigonometric identity.)
Solution:
sin2 x − cos2 x = 0 ⇔ − cos 2x = 0 .
The cosine is equal to 0 only at odd integer multiples of π2 . For x ∈ [0, 2π),
2x ∈ [0, 4π); in this interval there are 4 such multiples: 2x = π2 , 3π
2
, 5π
2
, 7π
2
; hence
π 3π 5π 7π
x = 4 , 4 , 4 , 4 . (A more naive way to solve the equation would be to factorize
the difference of squares into (sin x − cos x)(sin x + cos x). The product can vanish
only with either of the factors is zero. None of the factors is 0 when the cosine is
0; (why?). Hence the equation is equivalent to, after dividing by cos2 x, (tan x −
1)(tan x+1) = 0. These values for the tangent function are attained at the bisectors
of the quadrants, etc.

23. [3 MARKS] Showing all your work, find, in the interval [0, 2π], all solutions of the
equation 2 sin x cot x + cot x = −1 − 2 sin x.
Solution:

2 sin x cot x + cot x = −1 − 2 sin x ⇔ (2 sin x + 1) (cot x + 1) = 0


⇔ 2 sin x + 1 = 0 or cot x + 1 = 0
or both.

Case 1: 2 sin x + 1 = 0 ⇔ sin x = − 12 ⇒ x = 7π 11π


6
, 6 in the given interval [0, 2π).
3π 7π
Case 2: cot x + 1 → x = 4
, 4 in the given interval [0, 2π).

Thus, in the interval 0 ≤ x < 2π, there are precisely the 4 solutions given above.

24. [3 MARKS] Find all solutions of the following equation in the interval [0, 2π):
3 cot x − tan x = 0.
Solution: First observe that the tangent cannot be 0 at any solution to this equa-
tion, since the cotangent
√ must be defined.
√ Multiplying the equation by tan x yields
2
3 − tan x = 0 ⇔ ( 3 − tan x)( 3 + tan x) = √0. The equation is thus satis-
fied precisely for points where the tangent is ± 3, i.e., at the multiples of π3 :
x = π3 , 2π
3
, 4π
3
, 5π
3
.

25. [3 MARKS] Use an addition or subtraction formula to simplify the following equa-
tion. Then find all solutions in the interval [0, 2π): cos x cos 2x + sin x sin 2x = 0.
Solution: cos x cos 2x + sin x sin 2x = 0 ⇔ cos(2x − x) = 0 ⇔ cos x = 0 which is
satisfied in the given interval only for x = π2 and x = 3π
2
.
Information for Students in MATH 140 2008 09 3150

26. [3 MARKS] Use an addition or subtraction formula to simplify the following equa-
tion. Then find all solutions in the interval [0, 2π): sin 7x cos 6x − cos 7x sin 6x = 0.
Solution: sin 7x cos 6x − cos 7x sin 6x = 0 ⇔ sin(7x − 6x) = 0 ⇔ sin x = 0, which
is satisfied in the given interval only for x = 0, π.

27. [3 MARKS] Use a double- or half-angle formula to solve the following equation in
the interval [0, 2π): sin 2x + cos x = 0.
Solution: sin 2x + cos x = 0 ⇔ 2 sin x · cos x + cos x = 0 ⇔ (2 sin x + 1)(cos x) = 0.
The equation is satisfied when at least one of the 2 factors is zero. The first factor
is satisfied when sin x = − 12 , hence at x = 7π6
and x = 11π6
in the given interval.
The second equation is satisfied for x = 2 and x = 3π
π
2
in the given interval. These
are the 4 solutions to the given equation in the given interval.

7.5.2 Draft Solutions to Quiz Q2


Release Date: Monday, October 29th, 2007
draft, subject to correction

Monday versions There were many versions of these quizzes. The solutions below
are to typical examples, with specific choices of data. In all cases it was intended that
students provide supporting explanations for their answers.
x2 + 9x + 8
1. [5 MARKS] Showing all your work, evaluate lim . (It is not acceptable
x→−8 x2 + 7x − 8
to simply state the value!)
Solution: The limit cannot be found immediately using the Quotient Law, since
the limits of numerator and denominator are both 0. We can, however, transform
the fraction into one to which the Quotient Law can be applied. This will require
factorizing both of the quadratic polynomials, either by observation, by using the
quadratic formula, or by completion of the square.

x2 + 9x + 8 (x + 8)(x + 1)
lim = lim
x→−8 x2 + 7x − 8 x→−8 (x + 8)(x − 1)
x+1
= lim
x→−8 x − 1
lim (x + 1)
x→−8
=
lim (x − 1)
x→−8
−8 + 1 7
= = ,
−8 − 1 9
Information for Students in MATH 140 2008 09 3151

since limits of polynomials may be found by substitution, as they are continuous


functions.
½
4 − x2 if x ≤ 2
2. [5 MARKS] Let f (x) = .
x − 1 if x > 2
(a) Justifying every step in your solution, find lim− f (x) and lim+ f (x) .
x→2 x→2
(b) Explain carefully whether lim f (x) exists; if it does exist, state its value.
x→2
(c) Indicate which of the following is the graph of f . (Here 5 sketches were offered,
and the student was to select one of them.)

Solution:

(a) For the limit from the left the first line of the piecewise definition applies.

lim f (x) = lim− (4 − x2 ) = 4 − 22


x→2− x→2
by the continuity of the polynomial 4 − x2
= 0.

For the limit from the right the second line of the piecewise definition applies.

lim f (x) = lim+ (x − 1) = 2 − 1


x→2+ x→2
by the continuity of the polynomial x − 1
= 1.

(b) Since the two one-sided limits are different, the two-sided lim f (x) does not
x→2
exist.
(c) Two of the graphs presented look almost the same; they differ in the value
shown for the point x = 2, which should be attached to the parabola, rather
than to the ray.
f (x) − f (6)
3. [5 MARKS] Showing all your work, evaluate the limit of the ratio as
x−6
x → 6, where f (x) = (x + 3)(x − 1).
Solution:
f (x) − f (6) (x + 3)(x − 1) − (6 + 3)(6 − 1)
lim = lim
x→6 x−6 x→6 x−6
x2 + 2x − 48
= lim
x→6 x−6
Information for Students in MATH 140 2008 09 3152

(x + 8)(x − 6)
= lim
x→6 x−6
factorizing by observation,
the quadratic formula, or completion of the square
= lim (x + 8)
x→6
dividing by the factor x − 6, which is non-zero
= 6+8
polynomials are continuous, find limit by substitution
= 14.

4. [5 MARKS] Showing all your¡ work, determine


¢ the following limit, or explain why
−1 2 4
it does not exist: lim tan x −x .
x→∞

Solution: We begin by evaluating the limit as x → ∞ of x2 −x4 . We cannot use the


Addition (Subtraction) Law here, since that leads to a difference of the form ∞−∞,
which we cannot evaluate. However, we can observe that x2 − x4 = x2 (1 − x2 ). As
x → ∞, factor x2 → +∞, and factor 1 − x2 → −∞, so the product approaches
−∞. Thus the problem reduces to determining the value of lim tan−1 y. This
y→−∞
limit is − π2 , by the way in which we defined the inverse tangent function: the
graph
¡ π πwas¢ obtained by inverting the graph of the tangent function on the interval
−2, 2 .

Wednesday versions There were many versions of these quizzes. The solutions below
are to typical examples, with specific choices of data. In all cases it was intended that
students provide supporting explanations for their answers.
x2 − 5x
1. [5 MARKS] Showing all your work, evaluate lim .
x→5 x2 − 4x − 5

Solution: We cannot apply the Quotient Law to this rational function, since both
numerator and denominator approach 0 as x → 5. Instead we must transform the
function before seeking the limit. We do this by dividing numerator and denomi-
nator by the factor x − 5, which is not zero (since we avoid the point x = 5 which
is being approached).

x2 − 5x x(x − 5)
lim = lim
x→5 x2 − 4x − 5 x→5 (x + 1)(x − 5)
x
= lim
x→5 x + 1
dividing numerator and denominator by x − 5
Information for Students in MATH 140 2008 09 3153

lim x
x→5
= by the Quotient Law
lim (x + 1)
x→5
5
=
5+1
since polynomials are continuous functions
5
= .
6

x2 − 64
2. [5 MARKS] Let F (x) = .
|x − 8|
(a) Justifying every step in your solution, determine the values of lim+ F (x) and
x→8
lim− F (x), or argue that they do not exist.
x→8

(b) Explain carefully whether lim F (x) exists; if it does exist, state its value.
x→8

(c) Indicate which of the following is the graph of F .(Here 5 sketches were offered,
and the student was to select one of them.)

Solution:
x2 − 64 (x − 8)(x + 8)
(a) When x > 8, F (x) = = = x + 8, so lim+ F (x) =
|x − 8| x−8 x→8
lim+ (x + 8) = 8 + 8 = 16, since the one-sided limit of a polynomial may be
x→8
evaluated by substitution, by virtue of the fact that polynomials are continu-
x2 − 64 (x − 8)(x + 8)
ous everywhere. When x < 8, F (x) = = = −(x + 8),
|x − 8| −(x − 8)
so lim− F (x) = − lim− (x + 8) = −(8 + 8) = −16.
x→8 x→8

(b) Since the two one-sided limits at x = 8 are different, there is no two-sided
limit at x = 8: the limit does not exist.
(c) We have seen that, for x > 8, F (x) = x+8, while, for x < 8, F (x) = −(x+8).
You know how to graph these two lines, so it’s just a matter of doing so, and
checking the given sketches; alternatively, some of the cases could have been
eliminated using partial information. For example, since the function is known
to be discontinuous, two sketches that had an unbroken curve could not be
correct; etc.
f (x) − f (3)
3. [5 MARKS] Showing all your work, evaluate the limit of the ratio as
x−3
x → 3, where f (x) = (x + 4)3 .
Information for Students in MATH 140 2008 09 3154

Solution: We cannot apply the Quotient Law to this rational function, since both
numerator and denominator approach 0 as x → 3. Instead we must transform the
function before seeking the limit. We do this by dividing numerator and denomi-
nator by the factor x − 3, which is not zero (since we avoid the point x = 3 which
is being approached).

(x + 4)3 − (3 + 4)3 ((x + 4) − 7)((x + 4)2 + (x + 4)7 + 72 )


lim = lim
x→3 x−3 x→3 x−3
factoring a − b = (a − b)(a2 + ab + b2 )
3 3
¡ ¢
= lim x2 + 15x + 93
x→3
2
= 3 + 15(3) + 93 = 147
since polynomials are continuous functions
and limits can be evaluated by substitution

4x2 − 8x + 6
4. [5 MARKS] Showing all your work, evaluate lim . (It is not accept-
x→∞ 3x2 + 6x − 6
able to simply state the value.)
Solution: We can’t use the generalized Quotient Law here, since numerator and

denominator both before infinite, and we can’t evaluate a quotient of the type .

So we we need to transform the function prior to attempting to determine its limit.
In the case of limits at a real point we found it convenient to factorize polynomials;
here it is convenient to factor out the “leading” power of x.

Thursday versions There were many versions of these quizzes. The solutions below
are to typical examples, with specific choices of data. In all cases it was intended that
students provide supporting explanations for their answers.
x3 − 27
1. [5 MARKS] Showing all your work, evaluate lim .
x→3 x2 − 9

Solution: The Quotient Law may not be used immediately here, since numerator
and denominator both approach 0. We can, however, factorize both of these poly-
nomials and cancel the common factor, x − 3, which will not be zero, since, as we
approach the value x = 3, we are not concerned with the value of the function at
x = 3. Students needed to remember the factorization of a difference of cubes:
a3 − b3 = (a − b) (a2 + ab + b2 ).

x3 − 27 (x − 3) (x2 + 3x + 9)
lim = lim
x→3 x2 − 9 x→3 (x − 3)(x + 3)
Information for Students in MATH 140 2008 09 3155

x2 + 3x + 9
= lim
¡ 2x + 3
x→3
¢
lim x + 3x + 9
x→3
=
lim (x + 3)
x→3
32 + 3(3) + 9
=
3+3
by continuity of the polynomials
27 9
= =
6 2

f (x) − f (3)
2. [5 MARKS] Showing all your work, evaluate the limit of the ratio as
x−3
1
x → 3, where f (x) = . (Do not use any differentiation formulas.)
x+4
Solution:
1 1
f (x) − f (3) −
lim = lim x + 4 3 + 4
x→3 x−3 x→3 x−3
7 − (x + 4)
(x + 4)7
= lim
x→3 x−3
3−x
= lim
x→3 (x + 4)7(x − 3)
−1
= lim
x→3 (x + 4)7
−1 1
= =−
7(7) 49
since the limit of a polynomial may be determined by substitution, as polynomials
are continuous everywhere.
3. [5 MARKS] Showing all your work, determine all discontinuities of the function
6
f (x) = . (Some variants of this problem had a sine function in the
1 + cos 2x
denominator.)
Solution: This is a rational function — a ratio of polynomials. Polynomials are
defined for all x. Ratios thereof are defined wherever the denominator is non-
zero. Here the numerator polynomial is a non-zero constant: the discontinuities
are precisely the “zeroes” of the denominator. 1 + cos 2x = 0 precisely where
Information for Students in MATH 140 2008 09 3156

cos 2x = −1. The general solution is, therefore all points x such that 2x is an odd,
integer multiple of π radians; hence the set of discontinuities is
½ ¯ ¾
¯ (2n + 1)π
x ¯¯x = ,n ∈ Z .
2

4. [5 MARKS] Showing all your work, determine the limit as x → ±∞ of


√ √
x2 + 3x ∓ x2 + 7x .

Solution: For convenience I will work just the case x → +∞. We can’t solve this
problem by using the Addition/Subtraction Law immediately, since that leads to
a difference ∞ − ∞ which we can’t evaluate. So we have to resort to a “standard
trick” to transform to a different calculation.
√ √ ³√ √ ´ √x2 + 3x + √x2 + 7x
2 2
x + 3x − x + 7x = x + 3x − x + 7x · √
2 2 √
x2 + 3x + x2 + 7x
(x2 + 3x) − (x2 + 7x)
= √ √
x2 + 3x + x2 + 7x
−4x
= √ √ .
x + 3x + x2 + 7x
2

This transformation will, however, not be sufficient, since, as x → ±∞, numerator


and denominator both approach ∞, and we again have an incommensurable ratio,

this time of the form . So here we have to undertake another transformation, to

render finite the parts of the fraction. We do this by dividing out a factor x.
−4x −4x
√ √ = q ¡ ¢ q ¡ ¢
x2 + 3x + x2 + 7x x 1 + x + x2 1 + x7
2 3

−4x
= √ q √ q
x2 · 1 + x3 + x2 · 1 + 7
x
−4x
= q q
|x| 1 + x3 + |x| 1+ 7
x

 √ 3−4√ 7 if x > 0
1+ x + 1+ x
= √ −4 √
 if x < 0
− 1+ x3 − 1+ x7

In the version I am considering,


√ √ −4 −4
lim x2 + 3x ∓ x2 + 7x = lim q q = = −2 .
x→∞ x→∞ 3
1+ x + 1+ 7 2
x
Information for Students in MATH 140 2008 09 3157

µ ¶
2 8 6
x 4− + 2
4x2 − 8x + 6 x x
lim = lim µ ¶
x→∞ 3x2 + 6x − 6 x→∞ 2 6 6
x 3+ − 2
x x
 
8 6
4− + 2
 x2 x x 
= lim  2 ·
x→∞ x 6 6 
3+ − 2
x x
8 6
4− + 2
= lim x x
x→∞ 6 6
3+ − 2
µ x x ¶
8 6
lim 4 − + 2
x→∞ x x
= µ ¶
6 6
lim 3 + − 2
x→∞ x x
lim (4 − 0 + 0)
x→∞
=
lim (3 + 0 − 0)
x→∞
4
=
3

Friday versions There were many versions of these quizzes. The solutions below are
to typical examples, with specific choices of data. In all cases it was intended that
students provide supporting explanations for their answers.
1. [5 MARKS] Showing all your work, use the fact that

2x ≤ f (x) ≤ x3 + 1 for 0 ≤ x ≤ 3

to evaluate lim f (x).


x→1
Solution: The function f is not described explicitly — we know only that its graph
lies between the line y = 2x and the curve y = x3 + 1. That cubic curve cuts the
line y = 2x in the point (x, y) = (1, 2), at precisely the limiting value of x that is
being approached. Since the graph of f lies on each line x=constant between the
line and the curve, it is trapped at the point (1, 2) when x = 1.
The only result presented in this course in the form of a pair of inequalities of this
type is the “Squeeze” Theorem; so this is what students should have been thinking
about. The limit of the function at the left is equal to 2(1)=2; and the limit of the
polynomial on the right is (1)3 + 1 = 2 also, since limits of polynomials — they
Information for Students in MATH 140 2008 09 3158

being continuous functions — can be evaluated by substitution. The conclusion,


by the Squeeze Theorem, is that lim f (x) = 2.
x→1

f (x) − f (3)
2. [5 MARKS] Showing all your work, evaluate the limit of the ratio as
√ x−3
x → 3, where x + 3.
Solution: We cannot use the Quotient Law immediately, since numerator and de-
nominator both approach 0. So we transform the rational function in the usual way,
by “rationalization” (multiplying numerator and denominator by the “conjugate”
of, in this case, the numerator):
√ √
f (x) − f (3) x+3− 3+3
lim = lim
x→3 x−3 x→3 x−3√
µ√ √ √ ¶
x+3− 3+3 x+3+ 3+3
= lim ·√ √
x→3 x−3 x+3+ 3+3
à !
(x + 3) − (3 + 3)
= lim ¡√ √ ¢
x→3 (x − 3) x + 3 + 3 + 3
à !
x−3
= lim ¡√ √ ¢
x→3 (x − 3) x + 3 + 3 + 3
1
= lim √ √
x→3 x+3+ 3+3
r
1 2 2
= √ √ =√ = .
6+ 6 6 3
½
(x − 8)3 if x < 0
3. [5 MARKS] For the function f (x) = , determine all points
(x + 8)3 if x ≥ 0
x at which f is discontinuous. Show all y our work, and determine — if they exist
— limits from the left and from the right at the points of discontinuity.
Solution: This function is defined to be a polynomial in a domain consisting of R
decomposed into two disjoint half-lines (rays) — one of them including the point at
which the line is broken. The definition of continuity at a point x = a is concerned
with limits from the left and right as x approaches a. Where the function is the
same polynomial on both sides of a, we know from properties of polynomials, that
it is continuous at x = a. The one point remaining to study is the point where
the two polynomials are pasted together, here the point x = 0. The limit from the
right at x = 0 is lim (x + 8)3 = (0 + 8)3 = 512; the limit from the left is, by the
x→0+
same reasoning, lim (x − 8)3 = (0 − 8)3 = −512. Since the two one-sided limits
x→0−
Information for Students in MATH 140 2008 09 3159

are different, the function is not continuous at x = 0, but is continuous at all other
points on R.
4. [5 MARKS] Showing all your work, determine all values
½ of the constant c for which
cx + 55 , x ≤ 4
the function f is continuous on (−∞, +∞): f (x) = .
cx2 − 5 , x > 4
Solution: Except possibly at the point x = 4 the function f is a polynomial, and
we know that polynomials are continuous wherever they are defined; but this result
is meaningful only on open intervals, since the definition of continuity requires our
being able to investigate the function on both sides of the point in question. We are
left with only the point x = 4. As x → 4− , the one-sided limit may be evaluated by
substituting in the appropriate polynomial, i.e., in cx+55: lim− (cx+55) = c(4)+55.
x→4
As x → 4+ , the one-sided limit may be evaluated by substituting in the appropriate
polynomial, i.e., cx2 − 5: lim− (cx2 − 5) = c(16) − 5. f will be continuous at x = 4
x→4
if and only if the two one-sided limits are equal. That gives rise to the equation
4c + 55 = 16c − 5 ⇔ 12c = 60 ⇔ c = 5 .
Thus there is just one value for c that will make this function continuous on R, the
value c = 5.

Extra Problem
1. [5 MARKS] Showing all your work, use the fact that
1 ≤ f (x) ≤ x2 + 8x + 17 for all x
to evaluate lim f (x).
x→−4
Solution: The function f is not described explicitly — we know only that its graph
lies between the line y = 1 and the parabola y = x2 + 8x + 17. That parabola
has the property that x2 + 8x + 17 = (x + 4)2 + 1; thus the curve is never below
the line y = 1, and touches that line when x = −4, which is precisely the limiting
value that is being approached. Since the graph of f lies between the line and the
curve, it is trapped at the point (−4, 1) when x = −4.
The only result presented in this course in the form of a pair of inequalities of
this type is the “Squeeze” Theorem; so this is what students should have been
thinking about. The limit of the constant at the left is equal to 1; and the limit of
the polynomial on the right is (−4)2 + 8(−4) + 17 = 16 − 32 + 17 = 1 also, since
limits of polynomials — they being continuous functions — can be evaluated by
substitution. The conclusion, by the Squeeze Theorem, is that lim f (x) = 1.
x→−4
Information for Students in MATH 140 2008 09 3160

7.5.3 Draft Solutions to Quiz Q3


Release Date: Tuesday, November 20th, 2007
draft, subject to correction

Monday versions There were many versions of these quizzes. The solutions below
are to typical examples, with specific choices of data. In all cases it was intended that
students provide supporting explanations for their answers.
1. Two sides of a triangle are 7 m and 4 m in length and the angle between them is
increasing at a rate of 0.09 rad/s. Showing the details of your work, find the rate
(in m2 /sec) at which the area of the triangle is increasing when the angle between
the sides of fixed length is π3 . (Recall that, by the Law of Sines, the area of the
triangle is one-half the product of the lengths of 2 sides and the sine of the angle
between them.)
Solution: Denote the size of the angle between the sides of fixed lengths 7 and 4 m
by θ, the area of the triangle by A(θ), and time in seconds by t. Then we know that
7·4 dA dθ π
A(θ) = · sin θ = 14 sin θ. It follows that = 14 cos θ · = 14 cos · 0.09 =
2 dt dt 3
1
14 · · 0.09. The area is increasing at a rate of 0.63 square metres per second.
2
2. Showing the details of your work,¡ find an ¢ equation of the tangent line to the curve
y = sec x − 8 cos x at the point π3 , −2 . (The instructors are aware that you do
not have the use of a calculator.)
dy d
Solution: The slope of the tangent is = (sec x − 8 cos x) = (sec x)(tan x) −
dx √dx √
8(− sin x), whose value at x = π3 is 2 · √13 + 8 23 = 143 3 . An equation of the tangent
line is √
14 3 ³ π´ √ √
y = −2 + x− or 14x − 3 3y = 14π − 6 3.
3 3
¡ √ ¢
3. Showing all your work, differentiate the function h(x) = ln x + x2 − 6 . You are
expected to simplify your answer as much as possible.
Solution:
³ ´ µ ¶
d √ 1 1 2x
ln x + x2 − 6 = √ · 1+ · √
dx x + x2 − 6 2 x2 − 6

1 x2 − 6 + x
= √ · √
x + x2 − 6 x2 − 6
1
= √
x2 − 6
Information for Students in MATH 140 2008 09 3161

4. In the equation y 4 + x2 y 2 + yx4 = y + 3 regard y as the independent variable, and


x as the dependent variable; showing all your work, use implicit differentiation to
dx
find a formula for .
dy
Solution: Differentiating implicitly with respect to y yields
dx 2 dx
4y 3 + 2x · · y + x2 · 2y + 1 · x4 + y · 4x3 · =1+0
dy dy
Collecting terms yields
¡ ¢ dx
2xy 2 + 4x3 y = 1 − 4y 3 − 2x2 y − x4 ,
dy
implying that
dx 1 − 4y 3 − 2x2 y − x4
= .
dy 2xy 2 + 4x3 y
If you are uncomfortable using y as the independent variable, you could first de-
dy
termine , and then take the reciprocal, since these derivatives behave as though
dx
they were fractions.

Wednesday versions There were many versions of these quizzes. The solutions below
are to typical examples, with specific choices of data. In all cases it was intended that
students provide supporting explanations for their answers.

1. If a snowball melts so that its surface area decreases at a rate of 1 cm2 /min, find
the rate (in cm/min) at which the diameter decreases when the diameter is 17 cm.
(Recall that the area of a sphere is 4 times the area of the disk which the sphere
cuts from a plane through its centre.)
Solution: Let’s denote the area of the snowball by A(r), where r is the radius.
Then, as per the hint given, A = 4πr2 . Since the problem is stated in terms of the
diameter — call it D — we have A(D) = πD2 . Taking time derivatives we have
d dD dA
A(D(t)) = 2πD · . We are told that = −1 (since the hypothesis is in
dt dt dt
terms of decrease of area, and our convention is that the derivative is the rate of
dD 1 1
increase. When D = 17, the equation yields =− =− cm/min. This
dt 2πD 34π
is the rate of increase of diameter. Hence the rate of decrease of diameter will be
1
+ .
34π
Information for Students in MATH 140 2008 09 3162

2. Showing all your work, find the derivative of f (x) = x5x .


Solution: This problem can be solved with or without using logarithmic differenti-
ation.
without using logarithmic differentiation:
d d ¡ 5x ¢
f (x) = x
dx dx
d ³¡ ln x ¢5x ´
= e
dx
d ¡ 5x ln x ¢
= e
dx
¡ ¢ d
= e5x ln x · (5x ln x)
dx
¡ 5x ln x ¢ ³ x´
= e · 5 1 ln x +
x
= x5x · 5 (ln x + 1)

using logarithmic differentiation:


f (x) = x5x ⇒ ln f (x) = 5x ln x
d d
⇒ (ln f (x)) = (5x ln x)
dx µ dx ¶
f 0 (x) 1
⇒ = 5 1 ln x + x
f (x) x
0
f (x)
⇒ = 5 (ln x + 1)
f (x)
¡ ¢
⇒ f 0 (x) = 5 (f (x)) (ln x + 1) = 5 x5x (ln x + 1) as before.

3. It is known that x and y are related by the equation xy = 10 + x6 y. Showing all
dy
your work, use implicit differentiation to find a formula for .
dx
Solution: Differentiating the equation implicitly with respect to x yields
y + xy 0
√ = 0 + 6x5 y + x6 y 0 ,
xy
so
y + xy 0 = (10 + x6 y)(6x5 y + x6 y 0 ) .
Solving this equation for y0 yields
y (6x5 (10 + x6 y) − 1)
y0 = .
x (1 − x5 (10 + x6 y))
Information for Students in MATH 140 2008 09 3163


Note that I have simplified the presentation slightly by replacing xy by the ex-
pression given by the original equation.
3 cos(cos θ)
4. Showing all details of your work, find the limit lim . (The instructors
θ→0 sec θ
are aware that you do not have use of a calculator.)
Solution:

3 cos(cos θ) 3 cos(lim cos θ)


θ→0
lim =
θ→0 sec θ lim sec θ
θ→0
3 cos(1)
=
sec 1
= 3 cos2 1.

Without assistance you aren’t able to approximate cos 1, although you could ob-
serve that the answer could also be expressed as 32 (cos 2 + 1).
This problem appeared in several different versions: the trigonometric functions in
the numerator were either sines or cosines, in any combination.

Thursday versions There were many versions of these quizzes. The solutions below
are to typical examples, with specific choices of data. In all cases it was intended that
students provide supporting explanations for their answers.
sin5 θ
1. Showing all details of your work, find the limit lim .
θ→0 (2θ)3

Solution:
õ ¶5 !
sin5 θ sin θ
lim = lim 8θ2 ·
θ→0 (2θ)3 θ→0 θ
µ ¶5 ³ ´
sin θ 2
= lim · 8 lim θ ·
θ→0 θ θ→0
5
= 1 · 0 = 0.

2. Showing all your work, find the derivative of the function f (x) = xtan x .
Solution: This problem can be solved with or without using logarithmic differenti-
ation.
Information for Students in MATH 140 2008 09 3164

without using logarithmic differentiation:


d d ¡ tan x ¢
f (x) = x
dx dx
d ³¡ ln x ¢tan x ´
= e
dx
d ¡ (ln x)(tan x) ¢
= e
dx
¡ ¢ d
= e(ln x)(tan x) · ((ln x)(tan x))
dx
µ ¶
¡ (ln x)(tan x) ¢ 1 2
= e · · tan x + (ln x) · sec x
x
µ ¶
tan x tan x 2
= x · + (ln x)(sec x)
x

using logarithmic differentiation:

f (x) = xtan x ⇒ ln f (x) = (tan x)(ln x)


d d
⇒ (ln f (x)) = ((tan x)(ln x))
dx dx
f 0 (x) tan x
⇒ = (sec2 )(ln x) +
f (x) x
0
µ ¶
f (x) tan x 2 tan x
⇒ =x (sec )(ln x) + as before.
f (x) x

3. The altitude of a triangle is increasing at a rate of 4 cm/min while the area of


the triangle is increasing at a rate of 1 cm2 /min. At what rate is the base of the
triangle changing when the altitude is 5 cm and the area is 66 cm2 . (Recall that
the area of a triangle is one-half the product of its base and its altitude.)
Solution: Let’s denote the altitude, the length of the base, and the area of the
bh 2A
triangle respectively by h, b, A. The hint reminds you that A = . Hence b = .
2 h
Differentiating this equation with respect to time — let’s denote it by t — gives
à !
dA dh
db · h − A ·
= 2 dt dt
.
dt h2

Applying the given data yields


µ ¶
db 1 · 5 − 66 · 4
=2 .
dt 52
Information for Students in MATH 140 2008 09 3165

4. Showing all your work, use implicit differentiation to find an equation of the tangent
line to the curve y 2 = x3 (26 − x) at the point (1, 5).
Solution: (cf. Figure 25, on page 3165)

Figure 25: Problem 4 on Thursday Quiz Q3

Solution: Differentiating the equation of the curve implicitly with respect to x


yields 2y · y 0 = 78x2 − 4x3 . At the point (x, y) = (1, 5) the tangent will have slope
422 37
78x2 − 4x3 2y = = . The equation of the tangent at (1, 5) is y−5 = 37 5
(x−1),
10 5
equivalently 37x − 5y = 12.
Information for Students in MATH 140 2008 09 3166

Friday versions There were many versions of these quizzes. The solutions below are
to typical examples, with specific choices of data. In all cases it was intended that
students provide supporting explanations for their answers.
1. Showing all your work, use implicit differentiation to find an equation of the tangent
line to the curve 3(x2 + y 2 )2 = 25(x2 − y 2 ) at the point (2, 1). (cf. Figure 26, on
page 3166)

Figure 26: Problem 1 on Friday Quiz Q3

Solution: Differentiating the equation of the curve implicitly with respect to x


yields 2 · 3(x2 + y 2 ) · (2x + 2yy 0 ) = 25 (2x − 2yy 0 ), implying that
x(50 − 12(x2 + y 2 ))
y0 = . (248)
y(12(x2 + y 2 ) + 50)
Information for Students in MATH 140 2008 09 3167

−20 2
At the point (x, y) = (2, 1) the tangent has slope = − , and its equation is
110 11
2
y =1− 11
(x − 2), or 2x + 11y = 15.

2. Showing all your work, find y 0 if x7y = y 9x .


Solution: I am going to differentiate implicitly with respect to x: it will be simpler
if I first take logarithms of both sides of the equation.

x7y = y 9x ⇒ 7y ln x = 9x ln y
³ µ ¶
0 y´ y0
⇒ 7 y ln x + = 9 1 ln y + x ·
x y
y(9x ln y − 7y)
⇒ y0 =
x(7y ln x − 9x)

3. Gravel is being dumped from a conveyor belt at a rate of 20 ft3 /min, and its
coarseness is such that it forms a pile in the shape of a cone whose base diameter
and height are always equal. How fast (in ft/min) is the height of the pile increasing
when the pile is 8 ft high? (In case you have forgotten, the volume of the cone is
one-third of the product of the area of its base and its height.)
Solution: If we denote the diameter of the base of the cone by D, and its volume
1 πD2 D2 h
and height respectively by V and h, then V = · ·h= . In the present
3 4 12
application we are told that D = H, so the relationship between volume and height
h3
is given by V = . Differentiating with respect to time — which I will denote
12
dV
dV h2 dh dh 4
by t — yields = · , implying that = dt . Inserting the given data
dt 4 dt dt h2
yields
dh 4 · 20 5
= 2
or
dt 8 4
ft/min.

4. Differentiate the function y = ln |10 − x − 4x2 |.


Solution: While it is possible to obtain the correct answer by ignoring the absolute
signs, that was not an acceptable step in a solution, since the argument of the
absolute value function is sometimes negative. You needed either to recall that
d 1
|x| = [1, Equation 4, p, 217], or to treat the two cases separately, where the
dx x
argument is positive and where it is negative. While this may seem to be a minor
Information for Students in MATH 140 2008 09 3168

detail, it was the essence of the problem, which appears in the textbook as [1,
Exercise 3.6.17, p. 220].

d ¯ ¯ 1 d ¡ ¢
ln ¯10 − x − 4x2 ¯ = · 10 − x − 4x 2
dx 10 − x − 4x2 dx
1 8x + 1
= 2
· (−1 − 8x) = 2 .
10 − x − 4x 4x + x − 10

Unused problems The problems below were considered for use in some version of the
quizzes, but were eventually not used on any version. For some of these no solution has
been prepared.

1. A plane flying with a constant speed of 360 km/h passes over a ground radar station
at an altitude of 1 km, climbing at an inclination of π3 radians above the horizontal.
Showing the details of your work, determine the rate at which the distance from
the plane to the radar station is increasing a minute later?[Hint: Recall the Law
of Cosines: the length of a side of a triangle is equal to the square root of the sum
of the squares of the other two sides, diminished by the product of the lengths of
those other two sides and the cosine of the angle between them.] (Your instructors
are aware that you do not have the use of a calculator.)
Solution: Denote the distance in km of the plane from the ground radar station by
D, and time in minutes by t. The angle opposite the line joining the plane to the
π π
station measures + radians. After t minutes have elapsed, the plane will have
2 3
t
travelled · 360 = 6t km; the distance of the plane from the station is
60
r
5π √
D = 12 + (6t)2 − 2 · 1 · (6t) cos = 1 + 36t2 + 6t .
6
The rate of change of this distance will be
dD 1 1
= ·√ · (72t + 6) .
dt 2 1 + 36t2 + 6t
dD 1 1 39 2340
When t = 1, = ·√ · (72 + 6) = √ km/min, or √ km/h.
dt 2 1 + 36 + 6 43 43
sin x − cos x
2. Showing the details of your work, evaluate limπ (Do not use L’Hospital’s
x→ 4 cos 2x
Rule, even if you know it.) [Hint: Use a trigonometric identity, for example, a dou-
ble angle formula.]
Information for Students in MATH 140 2008 09 3169

Solution: As x → π4 , both numerator and denominator approach 0. Thus we can’t


use the Quotient Rule. There are a variety of trigonometric identities that could
be used to simplify this problem. For example, we could replace the denominator
by cos2 x − sin2 x:
sin x − cos x sin x − cos x
limπ = limπ 2
x→ 4 cos 2x x→ 4 cos2 x − sin x

sin x − cos x
= limπ
x→ 4 (cos x − sin x)(cos x + sin x)

−1
= limπ
x→ 4 cos x + sin x

The numerator and denominator of this last ratio do not approach 0, so the Quo-
−1 1
tient Rule may be used, giving the value of the limit as = −√ .
1 1 2
√ +√
2 2
3. Showing the details of your
³ π work,
´ find an equation of the normal line to the curve
y = 8 tan x at the point ,8 .
4
d
Solution: The slope of the tangent at the point (x, y) on the curve is (8 tan x) =
dx
8 sec x; hence the slope of the normal line through that point is −18 sec2 x =
2

cos2 x ³π ´ −1 1
− . At the point , 8 , the normal has slope 2 = − . The equation of
8 4 8 16
the normal line is, therefore,
1 ³ π´
y =8− x− .
16 4
4. Suppose that F (x) = f (g(x)), and g(2) = 4, g 0 (2) = 7, f 0 (2) = 1, and f 0 (4) = 9.
Express F 0 (2) in terms of the derivatives of the given functions, and then determine
its value at the given point.
Solution: F 0 (x) = f 0 (g(x)) · g 0 (x). When x = 2, this yields
F 0 (2) = f 0 (g(2)) · g 0 (2)
= f 0 (4) · g 0 (2)
= 9 × 7 = 63.
5. A table of values for f , g, f 0 , g 0 is given:
x f g f 0 g0
1 3 2 1 3
2 2 1 7 5
3 1 3 3 9
Information for Students in MATH 140 2008 09 3170

If h(x) = f (g(x)), and H(x) = g(f (x), determine the values of h0 (1) and H 0 (1).
Solution:

h0 (1) = f 0 (g(1)) · g 0 (1)


= f 0 (2) · g 0 (1) = 7 × 3 = 21
0
H (1) = g 0 (f (1)) · f 0 (1)
= g 0 (3) · f 0 (1) = 9 × 1 = 9 .

6. A table of values for f, g, f 0 , and g 0 is given. If H(x) = f (g(x)), and H(x) =


g(f (x)), find h0 (1) and H 0 (1), showing clearly how you are making use of the data
in the table.
x f (x) g(x) f 0 (x) g 0 (x)
1 3 2 7 5
2 1 6 5 5
3 1 5 5 7

Solution:

h0 (1) = f 0 (g(1)) · g 0 (1)


= f 0 (2) · g 0 (1) = 5 × 5 = 25
0
H (1) = g 0 (f (1)) · f 0 (1)
= g 0 (3) · f 0 (1) = 7 × 7 = 49 .

7. A ladder 4 feet long rests against a vertical wall. Let θ by the angle between the
top of the ladder and the wall, and let x be the distance from the bottom of the
ladder to the wall. if the bottom if the ladder slides away from the wall, how fast
does x change with respect to θ when theta = π3 , You are expected to show all the
details of your work.
Solution: Denote the point of contact of the ladder with the walls by A(0, y), the
location of the foot of the ladder by P (x, 0); the foot of the ladder is at the origin
O(0, 0). The distance of the foot of the ladder from the origin is |OP | = x, and the
dx
velocity of the moving foot is, therefore, . Since x = |OP | = |AP | sin θ = 4 sin θ,
dt
we have
dx d
= (4 sin θ) = 4 cos θ .
dθ dθ
π ¡1¢
When θ = , dx = 4 cos π
= 4 = 2.
3 dθ 3 2
Information for Students in MATH 140 2008 09 3171

8. Showing all your work, use implicit differentiation to find all points on the curve
3(x2 + y 2 )2 = 25(x2 − y 2 ) where the tangent is horizontal.
Solution: (cf. Figure 26, on page 3166) Setting equal to zero the general formula
for the slope, we obtain x(50 − 12(x2 + y 2 )) = 0. This implies that the points
25
with horizontal tangents lie either on the y − axis or on the circle x2 + y 2 = .
6
The y-intercept of the curve is the origin, where the tangents apparently are not
horizontal; the method of implicit differentiation is not adequate to resolve this
question. To find the points of intersection
µ of the circle¶and
µ the given curve,
¶ we
5 5 5 5
solve their equations, and obtain (x, y) = ± √ , √ , ± √ , − √ .
2 2 2 6 2 2 2 6
9. A plane flying horizontally at an altitude of 2 miles, and a speed of 485 miles per
hour passes directly over a radar station. Showing the details of your work, find
the rate at which the distance from the plane to the station is increasing when it
is 10 miles away from the station.
Solution: Zero the clock when the plane passes over
p the station. After time t the
distance from the plane to the station is D(t) = 22 + (485t)2 , and the rate of
change of distance is
dD 1 d 4852 · 2t
= p · (22 + (485t)2 ) = p .
dt 2 22 + (485t)2 dt 22 + (485t)2
p
When the plane is 10 miles from the station, D(t) = 22 + (485t)2 = 10, and the
rate of change of distance (in miles per hour) is
4852 · 2t √ √
= 97 96 = 482 6 .
10
10. Find an equation of the tangent line to the curve y = 2 sin(sin x), at the point
(π, 0).
11. Differentiate the function y = ln (2x−x + xe−x ).
12. Find an equation of the tangent line to the curve y = ln (x2 − 35) at the point
(6, 0).
13. Showing all your work, find y 0 if y = ln (4x2 + 9y 2 ).

7.5.4 Draft Solutions to Quiz Q4


Release Date: Friday, November 30th, 2007
draft, subject to correction
Information for Students in MATH 140 2008 09 3172

Monday versions There were many versions of these quizzes. The solutions below
are to typical examples, with specific choices of data. In all cases it was intended that
students provide supporting explanations for their answers.
1. [3 MARKS] Showing all your work, find the derivative of y = esinh rx (where r is a
prescribed constant).
Solution: We apply the Chain Rule:
dy d
= esinh rx · sinh rx
dx dx
d
= esinh rx · cosh rx · rx
dx
= esinh rx · cosh rx · r

2. [6 MARKS] Let f (x) = x − 4 sin x, (0 < x < 5π). Showing all your work
(a) [2 MARKS] find the open interval(s) on which the graph of f is concave
upward;
(b) [2 MARKS] find the open interval(s) on which the graph of f is concave
downward;
(c) [2 MARKS] find all inflection points; or show that there is none.
Solution:
(a)
f 0 = 1 − 4 cos x
⇒ f 00 = 4 sin x
The curve is concave upwards where sin x > 0, i.e., on the intervals 0 < x < π,
2π < x < 3π, 4π < x < 5π in the given domain.
(b) Since f 00 = 4 sin x, the curve is concave downwards on the intervals 1π < x <
2π, 3π < x < 4π in the given domain.
(c) At a point (x, y) where the graph y = f (x) of a function f has an inflection
point [1, p. 291] the function must be continuous, and the concavity must
change from upwards to downwards or vice versa. Since f is a sum of a
polynomial and a trigonometric function, it is continuous everywhere. In the
prescribed domain it changes concavity at the points
(π, π − 4 sin π) = (π, π)
(2π, 2π − 4 sin 2π) = (2π, 2π)
(3π, 3π − 4 sin 3π) = (3π, 3π)
(4π, 4π − 4 sin 4π) = (4π, 4π) .
Information for Students in MATH 140 2008 09 3173

These 4 points are the inflection points of the graph in the given domain.
3. [6 MARKS] Showing all your work, find the linearization L(x) of the function
1
f (x) = √ at a = 0, and use it to approximate the value of f (5). You do not
7+x
need to simplify your numerical answer, as the instructors are aware that you do
not have the use of a calculator.
Solution:
1 1 3
f (x) = √ ⇒ f 0 (x) = − · (7 + x)− 2
7+x 2
1 3
⇒ f 0 (a) = − · 7− 2
2
⇒ L(x) = f (a) + f 0 (a) · (x − a)
1 1 3 1 ³ x´
⇒ L(x) = 7 2 − · 7− 2 · (x − 0) = √ 1 −
2 7 14
1 1 3 9
⇒ L(5) = 7 2 − · 7− 2 · (5) = 3 ≈ 0.242977 . . . .
2 2 · 72
The correct value is approximately 0.288675. . . .
4. [6 MARKS] Let g(x) = 100 + 8x3 + x4 . Showing all your work
(a) find the interval(s) of increase, or prove there is none;
(b) find the interval(s) of decrease, or prove there is none;
(c) find all local maximum value(s), if any;
(d) find all local minimum value(s), if any;
(e) find all inflection point(s).
Solution:
(a)
g 0 = 24x2 + 4x3 = 4x2 (x + 6)
The first derivative is the product of an even number of factors x, and an odd
number of factors x + 6. The factor x2 is always non-negative; but the factor
x + 6 changes sign at x = −6. The first derivative changes sign at x = −6; at
x = 0 it is momentarily equal to 0.
Interval x2 x + 6 g 0 g
−∞ < x < −6 > 0 < 0 < 0 decreasing
−6 < x < 0 > 0 > 0 > 0 increasing
0 < x < +∞ > 0 > 0 > 0 increasing
Information for Students in MATH 140 2008 09 3174

The intervals of increase are (−6, 0) and (0, +∞). NOTE TO GRADERS:
Please give the students full marks even if they have merged these two intervals
into one interval, (−6, +∞).
(b) The interval of decrease is (−∞, −6).
(c)

g 00 = 24x + 12x2 = 12x(2 + x)


g 0 (x) = 0 ⇒ x = 0 or x = −6
g 00 (0) = 0
g 00 (−6) = 12(−6)(−4) > 0

The critical points are x = 0, −6; as the function is differentiable everywhere,


these are the only candidates for local extrema. The 2nd derivative test fails
at x = 0, since g 00 = 0 there. But we can use the first derivative test. In the
interval (−6, 0) g is increasing; and, in the interval (0, +∞) the function is
also increasing. Thus x = 0 is not an extremum; it is, in fact, an inflection
point. There is no local maximum for this function.
(d) Since g 00 (−6) > 0, x = −6 is a local minimum point, by the 2nd derivative
test; the minimum value is

g(−6) = 100 − 8 · 216 + 1296 = −332 .

5. We can apply the Quotient Rule. An easier method is to proceed as follows:


µ ¶ µ ¶
0 d 7 + cosh x d 14 − 7 + cosh x
G (x) = =
dx 7 − cosh x dx 7 − cosh x
µ ¶
d 14
= −1
dx 7 − cosh x
d
(− cosh x)
= −14 dx −0
(7 − cosh x)2
− sinh x 14 sinh x
= −14 2
=
(7 − cosh x) (7 − cosh x)2

6. Showing all your work, determine the global (absolute) maximum and global (ab-
x
solute) minimum values of 2 on the interval [−1, 16].
x + 64
Information for Students in MATH 140 2008 09 3175

x
Solution: Let f (x) = .
x2 + 64
1(x2 + 64) − x(2x)
f 0 (x) =
(x2 + 64)2
(8 − x)(8 + x)
=
(x2 + 64)2
The critical points are x = ±8, and the end-points of the domain — where the 2-
sided derivative is not defined — are −1 and 16. These 4 points are the candidates
for absolute extrema. There is no need to use the first or 2nd derivative tests,
since, in any case, we must test 3 of these points. The point x = −8 is outside of
the domain of the function, and can be ignored.
1
f (−1) = −
65
8 1
f (8) = =
128 16
16 1 1
f (16) = = < = f (8).
320 20 16
The global maximum occurs at x = 8, where the global maximum value is seen to
1
be . The global minimum occurs at x = −1, where the global minimum value
16
1
is seen to be − .
65

Wednesday versions There were many versions of these quizzes. The solutions below
are to typical examples, with specific choices of data. In all cases it was intended that
students provide supporting explanations for their answers.
1. [6 MARKS] Showing all your work, determine the global (absolute) maximum and
x
the global (absolute) minimum values of 2 on the interval [−1, 8].
x + 16
Solution: This rational function is continuous wherever it is defined; its domain is
the whole of R. Since the given interval is closed, the function will attain a global
x
maximum and a global minimum at points in the interval. Define f (x) = 2 ;
x + 16
(I am giving the function a name!) Differentiation yields
1(x2 + 16) − x(2x) (4 − x)(4 + x)
f 0 (x) = 2 = .
2
(x + 16) (x2 + 16)2
The critical points of the function are x = ±4; but only x = 4 is in the domain of
the function. Including the end-points of the domain, we thus have 3 candidates
Information for Students in MATH 140 2008 09 3176

for extrema: x = −1, x = 4, x = 8. We need only evaluate the function at these


3 points and compare the values; there is no advantage to be gained in using the
First or Second Derivative Tests:
x f (x)
1
−1 − 17
4 4
= 1 .
32 8
8 1
8 80
= 10

The global maximum point is at x = 4, and the global maximum value is 81 . The
1
global minimum point is at x = −1, and the global minimum value is − 17 . The
point x = 18 is neither a maximum point nor a minimum point. Note that the
question asked for the values, not the locations where those extremal values are
attained.
2. [6 MARKS] Showing all your work, find all inflection points of the function f (x) =
3x + 3 cos x, whose x-coordinates lie in the open interval (0, 10).
Solution:

f 0 (x) = 3 − 3 sin x
f 00 (x) = −3 cos x.

The textbook definition of an inflection point [1, p. 219] is a point where the
function is continuous, and the curve changes concavity. This function, being a
sum of a polynomial and a cosine, is continuous for all x. The concavity changes
where the second derivative changes sign, which is at odd integer multiples of π2 .
In the given interval those points ¡are at¢ x¡ = π2 , ¢x ¡= 3π
2
, x¢ = 5π
2
. The inflection
π 3π 3π 9π 5π 15π
points of the curve are, therefore, 2 , 2 , 2 , 2 , 2 , 2 .
3. [6 MARKS] Showing all your work, determine the values of a and b for which the
function f (x) = x3 + ax2 + bx + 4 has a local maximum when x = −9, and a local
minimum when x = −5.
Solution:

f (x) = x3 + ax2 + bx + 4
f 0 (x) = 3x2 + 2ax + b
f 00 (x) = 6x + 2a

The domain of definition of this function, not otherwise specified, is the whole of
R. It has 2 local extrema: these must be the two roots of the polynomial f 0 (x),
Information for Students in MATH 140 2008 09 3177

i.e., of 3x2 + 2ax + b. From the information given we know that this polynomial has
the same roots as (x − (−9)) · (x − (−5)), i.e., as x2 + 14x + 45. This can happen
only if
3 2a b
= = ,
1 14 45
hence a = 21 and b = 135. We find that f 00 (−9) = 6(−9) + 2(21) = −12 < 0: by
the Second Derivative Test there is a local maximum at x = −9, as required; also
we find that f 00 (−5) = 6(−5)+2(21) = 12 > 0: by the Second Derivative Test there
is a local minimum at x = −5, as required. The First Derivative Test could also
have been used, but would have been more complicated. IT WAS NECESSARY
TO INCLUDE THIS STEP; IT COULD HAVE HAPPENED THAT THE TYPES
OF EXTREMA WERE NOT WHAT THE QUESTION REQUIRED, IN WHICH
CASE THE ANSWER WOULD HAVE BEEN THAT “THERE ARE NO VAL-
UES OF a, b WHICH WILL MAKE THIS FUNCTION HAVE THE DESIRED
PROPERTIES”.

4. [6 MARKS] Some of the following formulas may be correct; others may contain
errors in the calculations. You are to check each statement. if the calculations are
correct, write the word CORRECT beside it. If the calculations contain errors,
find the place where the first error occurs, write a big X there, and write down
what would have been the correct value at that point in the calculations.
1£ x ¤ 1 ¡ −x ¢ 1¡ ¢
(a) sinh(−x) = e − e−x = −e − ex = − ex − e−x = − sinh x
2 2 2
(b) (cosh x + sinh x)n = cosh xn + sinh xn = enx = cosh nx + sinh nx
1 + tanh x cosh x − sinh x 2ex
(c) = = −x = e2x
1 − tanh x cosh x + sinh x 2e
1 £ −x ¤ 1 ¡ −x ¢ 1¡ x ¢
(d) cosh(−x) = e − e−(−x) = −e + ex = e + e−x = cosh x
2 2 2
1 £ −x ¤ 1 ¡ −x ¢ 1¡ ¢
(e) sinh(−x) = e − e−(−x) = e − ex = − ex − e−x = − sinh x
2 2 2
Solution:

(a) The given statement is incorrect at the very first equal sign. The correct
1 £ −x ¤
statement would be sinh(−x) = e − e−(−x) . While there may be other
2
errors in the string of equations, the question asked only that you mark the
FIRST error.
x −x x −x
(b) Since cosh x = e +e2
, and sinh x = e −e2
, cosh x + sinh x = ex . Hence
x n nx
the nth power is, indeed, equal to (e ) = e by the exponent rules. That
Information for Students in MATH 140 2008 09 3178

exponential is, by the same reasoning, equal to cosh(nx) + sinh(nx). Finally,


cosh xn +sinh xn = ex , and this is not the same as (ex )n . Thus, to summarize,
n

the very first equal sign is again incorrect!


sinh x
(c) If we replace tanh x by the ratio , we find that
cosh x
sinh x
1 + tanh x 1+ cosh x cosh x + sinh x
= sinh x
=
1 − tanh x 1− cosh x
cosh x − sinh x
so the first error is in the very first equal sign: the second fraction has numer-
ator and denominator reversed.
(d) The first equal sign is correct. However, the second equal sign is followed
immediately by a minus sign which is unjustified.
(e) This statement is completely correct.
5. [6 MARKS] Showing all your work, find the linearization L(x) of the function
f (x) = ln 9x at a = 19 , and use it to approximate the value of f (9). You should
simplify your answer as much as possible, but the instructors are aware that you
do not have the use of a calculator.
Solution:
f (x) = ln 9x
1
⇒ f 0 (x) =
µ ¶ x
1
⇒ f0 = 9
9
µ ¶ µ ¶ µ ¶
1 0 1 1
⇒ L(x) = f +f · x−
9 9 9
µ ¶ µ ¶
1 1
= ln 9 · +9· x−
9 9
= 9x − 1 .
Hence L(9) = 9(9)−1 = 80. This “approximate” value is a very bad approximation;
the correct value of the function is approximately 4.39445. Why is this so bad an
approximation? For one thing, the point where we wish to approximate the value
is “far” from the point of tangency. If we had used the approximation to find,
say, f (0.1), we would have obtained the approximation −0.1, which is not so bad
an approximation of ln(0.9), whose correct value is −0.1054. Here the percentage
error is approximately 5%, whereas the error in approximating f (9) was about
75.6
4.4
= 1700%.
Information for Students in MATH 140 2008 09 3179

Thursday versions There were many versions of these quizzes. The solutions below
are to typical examples, with specific choices of data. In all cases it was intended that
students provide supporting explanations for their answers.

1. [6 MARKS] Showing all your work, determine the global (absolute) √ maximum,
and the global (absolute) minimum values of the function f (t) = t 36 − t2 on the
interval [−4, 6].
Solution: This function is a product of a polynomial and a root function, and is
continuous wherever it is defined; its domain is the portion of R where the argument
of the square root function is non-negative, i.e., [−6, 6]. Since the given interval is
closed, the function will attain a global maximum and a global minimum at points
in the interval. Differentiation yields
√ 1 1
f 0 (t) = 1 36 − t2 + t · · √ · (−2t)
2 36 − t2
2(18 − t2 )
= √
36 − t2
√ √
2(3 2 − t)(3 2 + t)
= √ .
36 − t2

The critical points of the function are t = ±3 2, and possibly 6, the right end-
points of the domain. There is some ambiguity in the definition of critical point;
but it doesn’t matter whether we include the end-points as critical points, since we
have to test them in any case as part of the “Closed Interval Method”. Including
the end-points
√ of the domain, we thus have 3 candidates for extrema: t = −4,
t = 3 2, x = 6. We need only evaluate the function at these 3 points and compare
the values; there is no advantage to be gained in using the First or Second Derivative
Tests:
t √ f (t) √
−4
√ −4 20 = −8 5 < 0 .
3 2 18 > 0
6 0

The global maximum point is at t = 3 2, and the global maximum value is √18.
The global minimum point is at t = −4, and the global minimum value is −8 5.
The point t = 6 is neither a maximum point nor a minimum point. Note that the
question asked for the values, not the locations where those extremal values are
attained.

2. [9 MARKS] Let f (t) = t + cos t (−π ≤ t ≤ π). Showing all your work
Information for Students in MATH 140 2008 09 3180

(a) find all interval(s) of increase, if any;


(b) find all interval(s) of decrease, if any;
(c) find all local maximum value(s), if any;
(d) find all local minimum value(s), if any;
(e) determine the interval(s) where the graph of f is concave upward;
(f) determine the interval(s) where the graph of f is concave downward;
(g) determine all inflection point(s).

Solution:
¡ ¢
(a) f 0 (t) = 1−sin t, which is never negative, but is equal to 0 when t = 2n + 12 π.
The domain of¡the function
¢ ¡was given
¢ as −π ≤ t ≤ π. The intervals of increase
π π
are, therefore −π, 2 and 2 , π . (NOTE TO GRADERS: Please accept the
union of the closures of these intervals — [−π, π] — or (−π, π) as valid answers
also.)
(b) Since the first derivative is never negative, this function is never decreasing!
(c) The first derivative vanishes only at π2 , thus t = π
2
is a critical point. Since

f 00 (t) = − cos t ,

the second derivative vanishes at the point; thus we cannot apply the Second
Derivative Test to determine whether this critical point is a local extremum.
But we know that the function is increasing on both sides of t = π2 , so the
First Derivative Test tells us that the point is not a local extremum!
(d) There are no other critical points, so there are no local minima.
(e) In the given domain of definition the second derivative is positive for −π <
t < − π2 and for π2 < t < π; these are the intervals where the graph of the
function is concave upward.
(f) In the given domain of definition the second derivative is negative for − π2 <
t < π2 ; this is the interval where the graph of the function is concave downward.
(g) The textbook definition of an inflection point [1, p. 219] is a point where the
function is continuous, and the curve changes concavity. This function, being
a sum of a polynomial and a cosine, is continuous for all t. The concavity
changes where the second derivative changes sign, which is at odd integer
multiples of π2 . In the given interval those points are at t = − π2 , x = + π2 .
These are the only inflection points.
Information for Students in MATH 140 2008 09 3181

3. [5 MARKS] Some of the following formulas may be correct; others may contain
errors in the calculations. You are to check each statement. If the calculations are
correct, write the word CORRECT beside it. If the calculations contain errors,
find the place there the first error occurs, write a big X there, and write down what
would have been the correct value at that point in the calculations. (There were
many possible identities in this problem. Two that appeared on some versions are
the following.)
sinh x 1¡ x ¢ 1¡ x ¢
1+ e + e−x + e − e−x
(a) 1 + tanh x = cosh x = cosh x + sinh x = 2
1¡ x ¢
2
1¡ x ¢=
1 − tanh x sinh x cosh x − sinh x
1− e − e−x + e + e−x
cosh x 2 2
ex + e−x + ex − e−x = 2ex = e2x
ex − e−x − ex + e−x 2e−x
2 2
(b) cosh x + sinh x = 21 (ex + e−x ) + 12 (ex − e−x ) = 12 (2ex ) = ex
Solution:
(a) This identity is correct.
(b) The first error appears at the first equal sign. The member to the right of
the equal sign should have fractions of 41 , not 12 . (The next equal sign is
also incorrect.) It is true that cosh x + sinh x = ex , but that is irrelevant, as
students were asked to mark the FIRST equal sign that was not correct.
4. [6 MARKS] Showing all your work, determine all the critical numbers of the func-
tion f (θ) = 2 cos θ − sin2 θ.
Solution: The critical points are the points where either f 0 = 0 or where f is
not differentiable. This function is a sum of multiples of sines and cosines, and is
differentiable everywhere; f p rime(θ) = −2 sin θ − 2 sin θ cos θ = −2 sin θ(1 + cos θ).
The derivative vanishes where either sin θ = 0 or where 1 + cos θ = 0, or both. The
sine vanishes when its argument is an integer multiple of π; the cosine is equal to
−1 when its argument is an odd multiple of π. Thus the critical numbers are all
the integer multiples of π, since the first factor vanishes at all of these points; the
vanishing of the second factor occurs only at some of these points, so it adds no
points to the list of critical points. The set of critical points is infinite!
This function 2
¡ √ is
¢¡
periodic with
√ ¢period equal to 2π; f (θ) = 2 cos θ − (1 − cos θ) =
cos θ + 1 + 2 cos θ + 1 − 2 . When θ is an even integer multiple of π the func-
tion has its local maxima, of value 2; when θ is an odd integer multiple of π the function
has its local minima of −2. It is an even function. f 00 (θ) = 2(2 cos θ + 1)(cos θ − 1). The
second factor is either negative or 0; the first factor changes sign around the points where
¡ ¢
cos θ = − 12 ; thus the inflection points are at θ = 2n ± 23 π, where n is any integer.
Information for Students in MATH 140 2008 09 3182

5. (a) [4 MARKS] √ Showing all your work, find the linear approximation of the func-
tion g(x) = 3 1 + x at x = 0.

(b) [1 MARK] Use the linear approximation from part (a) to approximate 3 0.93.
Your answer should be simplified as much as possible, but you are not expected
to perform calculations for which you would normally require a calculator.

(c) [1 MARK] Use the linear approximation from part (a) to approximate 3 1.08.
Your answer should be simplified as much as possible, but you are not expected
to perform calculations for which you would normally require a calculator.
Solution:
√ 1 2 1
(a) Since g(x) = 3
(1 + x)− 3 . Hence g(0) = 1, g 0 (0) = . The
1 + x, g 0 (x) =
3 3
linear approximation about x = 0 is L(x) = 1 + 31 (x − 0).
√ √
(b) To approximate 3 0.93 = 3 1 − 0.07 we take L(−0.07) = 1 − 0.07 3
= 293
300

0.97667; the correct value is 0.9761....
√ p
(c) To approximate 3 1.08 = 3 (1 + 0.08) we take L(0.08) = 1 + 0.08 3
= 308
300

1.02667; the correct value is 1.0260....

Friday versions There were many versions of these quizzes. The solutions below are
to typical examples, with specific choices of data. In all cases it was intended that
students provide supporting explanations for their answers.

1. [6 MARKS] State Rolle’s Theorem, giving the precise conditions that a function
must satisfy on a given interval for the conclusion to hold. Then, showing all your
work, investigate whether the following function satisfies those conditions on the
given interval: √
f (x) = x x + 15, [−15, 0].
If the conditions of the theorem are satisfied, explain what result the theorem
provides; if the conditions of the theorem are not satisfied, determine whether or
not the conclusion of the theorem is still true for this function and interval.
Solution: Rolle’s Theorem [1, p. 280] states that, if a function f satisfies the
following 3 hypotheses:

(a) f is continuous on the closed interval [a, b];


(b) f is differentiable on the open interval (a, b); and
(c) f (a) = f (b)

then there exists a number c such that a < c < b and f 0 (c) = 0.
Information for Students in MATH 140 2008 09 3183

(a) The given function the product of a polynomial and a root function, both
known to be continuous; so it is also continuous wherever it is defined . Its
domain of definition is the interval where the square root function is defined,
i.e., x ≥ −15. We are told in the question to confine our attention to the
interval −15 ≤ x ≤ 0, so this replaces the maximal domain with which I
began this discussion. The known continuity of the function applies to all
the points in (−15, 0), where the continuity is 2-sided. It also applies to the
points x = −15 and x = 0, where the continuity need only be one-sided — to
the right at x = −15, and to the left at x = 0. This is what the conditions of
the theorem mean for a domain which is a closed interval.

(b) Since the functions x, x, and x + 15 are differentiable on the interiors of
intervals where they are defined, so are the
√ compositions and products that
can be formed from them. The function x x + 15 is differentiable everywhere
in (−15, 0).
(c) Finally, f (0) = 0 = f (−15).

Thus the conditions of Rolle’s Theorem are satisfied. The conclusion of Rolle’s
theorem is that there exists a point c such that −15 < c < 0 where f 0 (c) =
f (0) − f (−15) 0−0
. Here that ratio is equal to = 0; thus the theorem simply
0 − (−15) 15
asserts that there is a point c in (−15, 0) where the tangent is horizontal. (Students
were not asked to find a point c with the desired property. A simple computation
shows that there is only one such point: the point is x = − 15 2
.)

2. [6 MARKS] Showing all your work, find the global (absolute)


√ maximum and the
global (absolute) minimum values of the function f (t) = 3 t·(12−t) on the interval
[0, 12].
Solution:

(a) The given function is a product of a “root” function and a polynomial, and is
continuous wherever the factors are defined. Since it is continuous on a closed
interval, it will attain global extrema there.
(b) We have, by the Product and Quotient Rules of Differentiation,

3
f (t) = t · (12 − t)
1 1− 1 1
f 0 (t) = · t 3 · (12 − t) + t 3 · (−1)
2
12 − t − 3t 4 3−t
= 2 = · 2 .
3t 3 3 t3
Information for Students in MATH 140 2008 09 3184

(The function could also be treated as a sum of multiples of powers of t, and


the differentiation would be simpler!)We see that this function has a critical
point at t = 3. Such a point could be a local extremum, or might be only an
inflection point. We don’t need to follow up on this part of the investigation
here, since we are interested only in global extrema.)
(c) To locate the global extrema, we need to test the function at every (internal)
critical point, and also at the end-points of the interval, i.e., at 0 and 12. I
tabulate the results:
x f (x)
0 √ 0
3 3 3√· 9 > 0
3
12 120
We see that the global minimum value is 0, and that it is attained at two
points
√ in this closed interval — at the end-points. The global maximum value
3
is t · (12 − t), a value which is attained at only one point in the interval.
The problem, as stated, asked only for the extremal values.

3. Showing all your work, find the linear approximation of the function g(x) = 9 1 + x
at a = 0, and use it to approximate g(81).
Solution:
√ 1 8
g(x) = 9
1 + x ⇒ g 0 (x) = · (1 + x)− 9
9
1
⇒ g 0 (a) =
9
⇒ L(x) = g(a) + g 0 (a) · (x − a)
1 1
⇒ L(x) = 1 + (x − 0) = (x + 9)
9 9
90
⇒ L(81) = = 10.
9
The correct value is approximately 1.6295. The approximation is so poor because
the point of contact of the tangent line is so far from the point where the approxi-
mation is needed. Usually, when approximations of this type are taken, one chooses
a to be a convenient point where the function’s value is easy to compute.

4. [12 MARKS] Use the definitions of the hyperbolic functions to find the limits:

(a) lim tanh x


x→∞

(b) lim tanh x


x→−∞
Information for Students in MATH 140 2008 09 3185

(c) lim sinh x


x→0
(d) lim sinh x
x→−∞

(e) lim sech x


x→∞
(f) lim coth x
x→∞
(g) lim+ coth x
x→0
(h) lim coth x
x→0− ∞
(i) lim csch x
x→∞

Solution:
(a)
ex − e−x
lim tanh x = lim
x→∞ x→∞ ex + e−x
ex (1 − e−2x )
= lim x
x→∞ e (1 + e−2x )

1 − e−2x
= lim
x→∞ (1 + e−2x )
1−0
= lim =1
x→∞ 1 + 0

(b)
ex − e−x
lim tanh x = lim
x→−∞ x→−∞ ex + e−x
e−x (e2x − 1)
= lim −x 2x
x→−∞ e (e + 1)
2x
e −1
= lim 2x
x→−∞ e +1
0−1
= lim = −1
x→−∞ 0 + 1

(c)
ex − e−x
lim sinh x = lim
x→0 x→0 2
1−1
= =0
2
Information for Students in MATH 140 2008 09 3186

(d)

ex − e−x
lim sinh x = lim
x→−∞ x→−∞ 2
x
e e−x
= lim − lim
x→−∞ 2 x→−∞ 2
e−x
= 0 − lim
x→−∞ 2
= −∞

(e)
2
lim sech x = lim
x→∞ x→∞ ex + e−x
Here the denominator is the sum of two terms; the first approaches +∞, while
the second approaches 0 as x → ∞. Thus the denominator → ∞, and the
fraction → 0.
(f)
1
lim coth x = lim
x→∞ x→∞ tanh x

By Part (a) above the denominator approaches 1; as the numerator is equal


to 1, the hyperbolic cotangent also approaches 1 as x → ∞.
(g)

ex + e−x
lim coth x = lim+ x
x→0+ x→0 e − e−x
ex (1 + e−2x )
= lim+ x
x→0 e (1 − e−2x )

1 + e−2x
= lim+
x→0 1 − e−2x

When x > 0 ex > 1, so e−x < 1, and 1 − e−2x > 0. The denominator is
approaching 0 as x → 0+ ; the numerator approaches 2. Thus the faction
remains positive, but approaches +∞.
(h) This problem is similar to the preceding, except that the denominator is ap-
proaching 0 on the negative side; the one-sided limit is −∞.
Information for Students in MATH 140 2008 09 3187

(i)
2
lim csch x = lim
x→∞ x→∞ ex − e−x
The term ex approaches +∞, while the term e−x approaches 0; the denomina-
tor approaches +∞, but the numerator is a positive constant. The hyperbolic
cosecant approaches 0.

8 Examinations from Previous Years

1. These examinations have been taken from the original versions in computer files.
Occasionally an examination has been changed or corrected in the examination
room: such changes may not have been incorporated into these versions.

2. Listed below are only the texts of the problems on most of the examinations from
1996-May 2005; the actual formats of most of the examinations can be seen at the
following URL:

http://www.math.mcgill.ca/brown/math140a.html

3. It is possible that the form and format of the examination in MATH 140 will
undergo further changes in December, 2006. Further information may be given at
the lectures.

4. The syllabus on which the following examinations was based has undergone only
minor changes over the years. Some changes derive from the use of different text-
books. The examinations can serve as a circumstantial evidence of what a student
in Math 140 was expected to know by examination time. Notwithstanding the
foregoing, you are advised not to base your preparation for your examination on a
study of these examinations. The best way to prepare for your examination is to
work through your textbook, using the Student Solutions Manual and consultations
with the tutors to help you evaluate the quality of your solutions to problems.

5. Solutions to these examinations are not available: in some cases you may be able
to find similar, solved problems in the Student Solutions Manual.
Information for Students in MATH 140 2008 09 3188

8.1 December 1996 Final Examination in 189-122A


1
− 12
x
1. [4 MARKS] Evaluate lim .
x→2 x − 2

x−9
2. [4 MARKS] Evaluate lim √ .
x→9 x−3
3. [4 MARKS] Evaluate lim x cot 3x .
x→0

4. [5 MARKS] Find an equation for the straight line which is normal to the graph of
f (x) = x2 at x = −3 .

5. [8 MARKS] The surface area of a sphere is increasing at a rate of 10 square metres


per hour. At what rate is the volume increasing, when the radius is 2 metres?
(Note: The volume of a sphere of radius r is 34 πr3 ; the surface area is 4πr2 .)

6. [8 MARKS] Show that the equation 6x4 − 7x + 1 = 0 does not have more than
two distinct real roots.

7. [10 MARKS] A right circular cylinder is inscribed in a right circular cone of height
H and radius R. Determine the dimensions of the cylinder with the largest possible
volume. What is that largest volume?
1
8. [8 MARKS] Show that the function f (x) = 2 + (1 − x3 ) 5 has an inverse. Deter-
mine f −1 (x) .

9. The function f (x) = x − cos x (−π ≤ x ≤ π) is differentiable.

(a) [2 MARKS] Show that this function has an inverse.


(b) [4 MARKS] Calculate the derivative of the inverse function.
(c) [4 MARKS] Evaluate the derivative of f −1 (x) at all points x such that f (x) =
−1 .

10. [8 MARKS] Use the mean value theorem to show that, when
x>1,
x−1
< ln x < x − 1 .
x
µ ¶
x4
11. Let f (x) = ln .
x−1
(a) [3 MARKS] Specify the domain of f .
Information for Students in MATH 140 2008 09 3189

(b) [3 MARKS] Determine the interval(s) where f increases, and the interval(s)
where f decreases.
(c) [3 MARKS] Determine the concavity of the graph of f , and find the points
of inflection.
(d) [3 MARKS] Sketch the graph, using the information determined above.

12. Let f (x) = xe−x .

(a) [3 MARKS] Specify the domain of f .


(b) [3 MARKS] Determine the interval(s) where f increases, and the interval(s)
where it decreases.
(c) [3 MARKS] Determine the concavity of the graph of f , and find the points
of inflection.
(d) [3 MARKS] Sketch the graph, using the information determined above.

13. [7 MARKS] Given the curve x3 + y 3 = 1 + 3xy 2 , verify that the point (x, y)
dy d2 y
= (0, 1) is on the curve. Find and 2 at (x, y) = (0, 1) .
dx dx

8.2 December 1997 Final Examination in 189-140A


x2 − 4
1. [8 MARKS] For the function f (x) =
|x − 2|
(a) Find the left-hand and right-hand limits at x = 2 .
(b) Determine whether the two-sided limit exists at x = 2 .
(c) Sketch the graph of y = f (x) .

2. [2 MARKS] Multiple Choice: Circle the correct answer (A, B, C, D, or E.)


√ √
9 + x − 9 − 3x
lim
x→0 x

A B C D E
2
=0 = =1 does not exist (or = is a real number r dif-
3 2
∞, or = −∞). ferent from 0, , 1.
3
Information for Students in MATH 140 2008 09 3190

3. [8 MARKS] Apply the intermediate value property of continuous functions to prove


that the equation x3 − 4x2 + 1 = 0 has at least three different solutions. (Hint:
Denoting f (x) = x3 − 4x2 + 1 , we have f (+1) = −2.)
60 − t
4. [8 MARKS] At time t, the radius of a leaking spherical balloon is r =
12
centimetres. Determine the rate (in cm.3 /second) at which the volume is decreasing
when t = 30 . (Hint: You may assume that the volume of a sphere of radius r is
4
3
πr3 .)

5. [8 MARKS] Determine an equation for the straight line that passes through the
point (1, 5) and is tangent to the curve y = x3 .

6. [2 MARKS] Multiple Choice: Circle the correct answer (A, B, C, D, or E.)

x − 4 tan x
lim
x→0 sin x

A B C D E
2
=0 = =1 does not exist (or = is a real number r dif-
3 2
∞, or = −∞). ferent from 0, , 1.
3

7. [8 MARKS] Showing all your work, determine the maximum area of a rectangle
with a base that lies on the x-axis, and with two upper vertices that lie on the
graph of the equation y = 4 − x2 .

8. [8 MARKS] A tank is in the shape of an inverted right circular cone of height 800
cm., whose top is a disk of radius 160 cm. Water is running out of a small hole
at the vertex (apex) of the cone, which is at the bottom. Showing all your work,
determine the rate of change of volume V with respect to height h, at a time when
the height is 600 cm. (Hint: You may assume that the volume of a right circular
cone of height h, whose base has radius r, is 31 πr2 h.)

9. [2 MARKS] Multiple Choice: Circle the correct answer (A, B, C, D, or E.)


dy π
If y = sin 2x cos 3x , the value of when x = is
dx 2

A B C D E
2 2
0 1 −1 different from 0, , 1, −1.
3 3
Information for Students in MATH 140 2008 09 3191

10. [8 MARKS] A covered rectangular box is to be constructed with volume 576 cubic
centimetres, with its bottom twice as long as it is wide. Determine the dimensions
of the box that will minimize its total surface area (including the cover).

11. [5 MARKS] Showing all your work, determine the maximum and minimum values
of f (x) = 3 − |x − 2| on the interval [1, 4] .

12. [5 MARKS] Multiple Choice: Circle the correct answer (A, B, C, D, or E.)
¡ ¢
The function f (x) = 2x2 − 3x e−x has a global maximum on the half-line x ≥ 0 .
The maximum value is

A B C D E

√ 1 9 9 −3
− e −√ e 4 none of the preceding
e e2 8
4 values.

13. [8 MARKS] Showing all your work, sketch the graph of the function

e2x
f (x) = ,
e2x + 3
identifying asymptotes, critical points, and inflection points. Show clearly where
the graph is concave upward and where it is concave downward.

14. [8 MARKS]

(a) State the Mean Value Theorem.


(b) Given that f (x) = |x| for −1 ≤ x ≤ 1 , does it follow from the Mean Value
Theorem that there exists a point c such that −1 ≤ c ≤ 1 and f 0 (c) = 0 ?
Explain your answer.

15. [2 MARKS] Multiple Choice: Circle the correct answer (A, B, C, D, or E.)
dy
If x5 − y 5 = 2x2 y 2 , the value of when (x, y) = (1, −1) is
dx

A B C D E
5 5 5 5
−1 1 − none of −1, 1, − , .
3 3 3 3
Information for Students in MATH 140 2008 09 3192

16. [10 MARKS] Showing all your work, sketch the graph of the function

x2 − x − 2
f (x) = − 1,
(1 − x)2
identifying asymptotes, critical points, and inflection points. Show clearly where
the graph is concave upward and where it is concave downward.

8.3 December 1998 Final Examination in 189-140A


x3 − 27
1. [4 MARKS] Showing your work, find lim .
x→3 x − 3

2. [5 MARKS] Given that a function f has the property that

|f (x) − 2| ≤ (x − 1)2 ,

determine lim f (x).


x→1

3. (a) [4 MARKS] Define v(x) = x3 − 4x2 + x + 3 . Use the Intermediate Value


Theorem to show that the equation v(x) = 0 has a solution between x = 1
and x = 2.
(b) [4 MARKS] By examining the behavior of v(x) as x → ∞ and as x → −∞,
or otherwise, discuss the existence of other solutions to v(x) = 0.

4. [8 MARKS]
√ Find equations for all straight lines which are both normal to the curve
y = x − 3 and parallel to the straight line y = −2x + 11.

5. For the function g(x) = |1 − x2 |,

(a) [3 MARKS] Sketch the graph of g.


(b) [3 MARKS] Show that g is continuous at x = 1.
(c) [2 MARKS] Determine whether g is differentiable at x = 1.
sin x
6. (a) [3 MARKS] Determine h0 (x) , if h(x) = .
1 − 2 cos x
(b) [3 MARKS] If u(x) = ln ((2x − 1)3 ), determine u0 (x).

7. [12 MARKS] Determine the slope of the curve

(x + y)2 − (x − y)2 = x4 + y 4

at all point(s), other than the origin, where the curve meets the line x = y.
Information for Students in MATH 140 2008 09 3193

8. [6 MARKS] A child is building a snowman by rolling a snowball on the ground; its


volume is increasing at the rate of 8 cubic centimetres per minute. Find the rate
at which the radius is increasing when the snowball is 75 centimetres in diameter.
(The volume of a sphere of radius r is 34 πr3 .)
£ ¤
9. [8 MARKS] The domain of the function F is to be taken to be the interval −2, 12 .
On that interval,
F (x) = x3 + x2 − x + 1 .
Determine the global maximum ( = absolute maximum), global minimum ( = abso-
lute minimum), local maxima ( = relative maxima), and local minima ( = relative
minima).
10. [10 MARKS] A closed box with a square base is to have a volume of 2, 000 cubic
centimetres. The material for the top and bottom of the box costs $3 per square
centimeter, while the material for the sides costs $1.50 per square centimetre.
Determine the dimensions of the least expensive box.

11. For the function f (x) = x2 ,


x2 − 4
(a) [2 MARKS] Determine the (largest possible) domain.
(b) [4 MARKS] Determine all local extrema, and all points of inflection.
(c) [2 MARKS] Determine precisely where the function is increasing, decreasing,
concave upward, concave downward.
(d) [2 MARKS] Determine all horizontal and all vertical asymptotes, if any.
(e) [1 MARK] Sketch the graph.
12. It is given that the function f , defined by f (x) = x + x3 has an inverse, denoted
by f −1 .
(a) [3 MARKS] Determine the value of f −1 (2).
³ ´
(b) [3 MARKS] Determine the value of d (f −1 ) (2).
dx
13. [4 MARKS] Determine the derivative of the function

m(x) = xx .

14. [4 MARKS] Determine the value of the following limit, if it exists:


3
ex − 1
lim
x→0 x − sin x
Information for Students in MATH 140 2008 09 3194

8.4 May 1999 Supplemental Examination in 189-140A


x2 − 4
1. (a) Evaluate lim .
x→2 x − 2

(b) Let f be the function with domain ∞ < x < ∞ given by


½ 2
x − 2x if x ≤ 1
f (x) =
x−2 if x > 1

i. Is f continuous at x = 1? Explain (justify) your answer.


ii. Is f differentiable at x = 1? Explain (justify) your answer.

2. Find the derivative of each of the following functions. (You need not simplify your
answers.)

sin(2x2 − 1)
(a) F (x) =
(x2 + 1)3
µ ¶
−1 1
(b) G(x) = tan √
x3 + 1
2 −2x
(c) H(x) = x e ln x

[a, b], and theorem. g 0 (x) > 0

3. It is given that the function f , defined by f (x) = x + x3 has an inverse, denoted


by f −1 .

(a) Determine the value of f −1 (2).


³ ´
(b) Determine the value of d (f −1 ) (2).
dx
4. (a) Find an equation for the line tangent at the point (1, 1) to the curve y 2 =
x3 (2 − x).
(b) A woman who is 1.75 metres tall walks at a rate of 1.5 metres per second
away from a lamp that is at the top of a 4-metre high lamp post. At what
rate is her shadow lengthening when she is 30 metres from the lamp post.

5. Find the area of the largest rectangle which can be inscribed in a semicircle of
radius 1, where one side of the rectangle lies on the diameter of the semicircle, and
the other two vertices lie on the semicircle.

6. Consider the function h(x) = 4x3 − 15x2 + 12x + 7, with domain −∞ < x < ∞.
Information for Students in MATH 140 2008 09 3195

(a) Find all points at which h has a local maximum, a local minimum, or a point
of inflection. Justify all of your answers.
(b) Find the global (absolute) maximum and the global (absolute) minimum of h
on the interval [0, 3].
2x2
7. Sketch the curve y = , indicating any horizontal or vertical asymptotes.
x2 − 1
8. Let the function u be defined by u(x) = ln(2x + 1).

(a) What is the (maximum possible) domain of u. (Explain.)


(b) Sketch the graph of u(x) = ln(2x + 1).

8.5 December 1999 Final Examination in 189-140A




 x + sin 3x
 x>0
tan 4x
1. Let f (x) = , where a is a constant, to be determined.

 (x − a)2
 x≤0
4
(a) [6 MARKS] Determine each of lim− f (x), lim+ f (x) or explain why either or
x→0 x→0
both do not exist.
(b) [3 MARKS] Use the information of part (a) to determine all values, if any, for
the constant a, which will make f continuous at x = 0.
2 −3
2. [8 MARKS] Let m(x) = xx − (x2 ) . Determine the value of m0 (1).

3. [8 MARKS] If y is defined implicitly as a function of x by 2x2 − 3xy + 5y 2 = 10,


d2 y
determine the value of 2 when (x, y) = (1, −1).
dx
2+x
4. For the function h(x) = arctan ,
1 − 2x
(a) [1 MARK] State the (largest possible) domain.
dh
(b) [5 MARKS] Determine for all points x where the derivative exists.
dx
(c) [2 MARKS] Give an example of a function different from h which has exactly
the same domain and exactly the same derivative as h.
x
5. The function u is defined by u(x) = , for −∞ < x < ∞.
1 + x2
Information for Students in MATH 140 2008 09 3196

(a) [3 MARKS] Determine the intervals where the function u is increasing, and
those where it is decreasing.
(b) [3 MARKS] Find all critical points. In each case determine whether the point
is a maximum or minimum point, or neither.
(c) [3 MARKS] Determine the intervals where the graph of u is concave upwards,
and those where it is concave downwards.
(d) [3 MARKS] Determine all inflection points of the graph.
(e) [2 MARKS] Determine all horizontal or vertical asymptotes of the graph.
(f) [4 MARKS] Sketch the graph.
(x − 1)(x + 1) 2x(x2 − 3)
You may assume that u0 (x) = − , and that u 00
(x) = .
(x2 + 1)2 (x2 + 1)3
(For each of parts (a) through (e) you are expected to show all your work and your
results, clearly marked by the question number, e.g., 5(c); it is not sufficient to
provide information only on your graph.)

6. [8 MARKS] Use the Intermediate Value Theorem and/or the Mean Value Theorem
1
and/or properties of G0 (x) to show that the function G(x) = x2 − e 1+x assumes
the value 0 for exactly one real number x such that 0 < x < 2. Show all your work.
1
[Hint: You may assume that e 3 < 2.]

7. [8 MARKS] Triangle OBC, in the first quadrant, has vertex O at the origin, vertex
B on the x-axis, and vertex C on the y-axis. If the vertices are constrained so that
the line joining B and C passes through the point (2, 3), determine the minimum
area for triangle OBC. Show all your work.

8. Showing all your work, evaluate the following limits, if they exist:
√ √
(a) [8 MARKS] lim ( x2 + x − x2 − x).
x→∞
tan x − sin x
(b) [8 MARKS] lim .
x→0 x3
9. [8 MARKS] Showing all your work, determine all lines with slope 3 which are
normal to the curve 64y + x3 = 0, (i.e. which are perpendicular to the tangent at
each point where they meet the curve).

10. [9 MARKS] Showing all your work, determine the (global) maxima and minima
of the function R(x) = 3x4 + 4x3 − 6x2 − 12x on the closed interval −2 ≤ x ≤ 2.
[Hint: x3 + x2 − x − 1 = (x2 − 1)(x + 1).]
Information for Students in MATH 140 2008 09 3197

8.6 December 1999 Special Final Examination in 189-140A


1 − x + ln x
1. [4 MARKS] Evaluate lim− .
x→1 1 + cos πx
2
2. [6 MARKS] When f (x) = x−2x , evaluate f 0 (1).
2x
3. [5 MARKS] Evaluate lim .
x→0 tan 3x
4. [5 MARKS] Find an equation for the straight line which is normal to the graph of
f (x) = x3 at x = −2 .

5. [10 MARKS] The volume of a sphere is increasing at a rate of 10 cubic metres per
hour. At what rate is the surface area increasing, when the radius is 2 metres?
(Note: The volume of a sphere of radius r is 34 πr3 ; the surface area is 4πr2 .)

6. [10 MARKS] Show that the equation 3x4 − 28x + 8 = 0 does not have more than
two distinct real roots.
dy d2 y
7. [10 MARKS] Determine the value of and 2 at the point (x, y) = (1, −1) when
dx dx
x and y are related by the equation x3 + xy + y 4 = 1.

8. [10 MARKS] A box with a square base and an open top is to have volume 62.5
cubic centimetres. Neglect the thickness of the material used to make the box, and
find the dimensions that will minimize the amount of material used.
µ ¶
x3
9. Let f (x) = ln .
x2 − 1
(a) [4 MARKS] Specify the domain of f , and any points of discontinuity.
(b) [5 MARKS] Determine all horizontal asymptotes and all vertical asymptotes.
(c) [5 MARKS] Determine the interval(s) where f increases, the interval(s) where
f decreases, and all local extrema.
(d) [3 MARKS] Determine the concavity of the graph of f , and find all points of
inflection.
(e) [3 MARKS] Sketch the graph, using the information determined above.
µ ¶
−1 8π
10. [5 MARKS] Showing all your work, determine sin sin .
3
Information for Students in MATH 140 2008 09 3198

8.7 December 2000 Final examination in 189-140A


1. (a) [5 MARKS] Find the derivative of ln cosh(x2 ) .
µ ¶
2t
(b) [5 MARKS] Find the derivative of arctan . You are expected to
1 + t2
simplify your answer.

2. [10 MARKS] Determine the equation of the tangent to the curve

x2 + y sin x + tan2 y = 1

at the point (1, 0) .

3. [10 MARKS] Use a linear approximation to estimate the value of


√3

5
0.97 + 0.97 .

4. [10 MARKS] Showing all your work, determine the domain of the following func-
tion, and all vertical or horizontal asymptotes to its graph.
2x + 5
f (x) = √ .
x2 − 36

5. [10 MARKS] Let f (x) = |2x + 5|. Show that there is no value of c such that

f (−4) = f (0) + f 0 (c) · (−4 − 0) .

Explain why this does not contradict the Mean Value Theorem.

6. [10 MARKS] A farmer wants to fence an area of 20,000 square metres in a rectan-
gular field, and then divide it into three parts by fences parallel to one of the sides
of the rectangle. What shape of field will minimize the cost of the fencing? Show
all your work.

7. For the function h(x) = x2 e2x , showing all your work

(a) [5 MARKS] Determine the intervals where h is increasing, and where it is


decreasing.
(b) [5 MARKS] Determine all local maxima, and all local minima.
(c) [5 MARKS] Determine intervals of concavity, and all inflection points.
(d) [5 MARKS] Sketch a graph of the function.
Information for Students in MATH 140 2008 09 3199

8. (a) [5 MARKS] Find the function f that satisfies all of the following conditions:

f 00 (x) = 3ex + 5 sin x , f (0) = 1 , f 0 (0) = 2 .

(b) [5 MARKS] Determine all values of the constant c that will make the
following function continuous everywhere:
½
cx + 1 if x < 5
f (x) =
cx2 − 1 if x ≥ 5

8.8 May 2001 Supplemental/Deferred Examination in 189-140A


√ √
1. (a) [5 MARKS] Showing all your work, evaluate lim ( x + 2 − x) .
x→∞

sin(x2 − 1)
(b) [5 MARKS] Showing all your work, evaluate lim .
x→−1 x+1
x2 + 1
2. (a) [5 MARKS] Find the derivative of .
x2 − 2
µ ¶
1
(b) [5 MARKS] Find the derivative of cos √ .
x+1
(c) [5 MARKS] Find the derivative of x sin−1 x .

3. [10 MARKS] Showing all your work, determine the greatest and least values at-
sin x
tained by the function f (x) = .
(7 − 2 cos x)2

3 3 dy d2 y
4. [10 MARKS] If x − xy + y = 1 , determine and when x = 1
dx dx2
and y = 0.

5. [10 MARKS] The illumination I of an object by a light source is given by the


S
formula I = 2 units, where S is the strength of the light source, and d is the
d
distance of the object from the light source. If two light sources, one 8 times as
strong as the other, are placed 3 units apart, where should an object be placed on
the line between them so as to receive the least illumination?

6. [10 MARKS] Use a “linear” or “tangent-line” approximation at x = 4 to compute


√ 1
an approximate value for f (x) = x + √ at x = 3.97.
x
7. For the function f (x) = x5 + x6 , showing all your work
Information for Students in MATH 140 2008 09 3200

(a) [5 MARKS] Determine the intervals where f is increasing, and those where it
is decreasing.
(b) [5 MARKS] Determine all local maxima, and all local minima.
(c) [5 MARKS] Determine intervals of concavity, and all inflection points.
(d) [5 MARKS] Sketch a graph of the function.
1
8. (a) [5 MARKS] If f 0 (x) = x − 1 and f (1) = , find f (x).
5
1
(b) [5 MARKS] If f 0 (x) = and f (0) = 0, use the Mean Value Theorem
1 + x3
to show that
2
f (2) − f (0) =
1 + c3
2
for some c such that 0 < c < 2, and deduce that < f (2) < 2. (Use the
9
Mean Value Theorem — do not determine an exact formula for f (x).)

8.9 December 2001 Final Examination in 189-140A


A TOTAL OF 137 MARKS ARE AVAILABLE ON THIS EXAMINATION.

1. In each of the following cases evaluate the limit, or explain why the limit does not
exist. Show all your work. Do not use l’Hôpital’s Rule.
x−2
(a) [5 MARKS] lim−
x→2 |x − 2|
sin x − sin(3x)
(b) [5 MARKS] lim
x→0 x2 + 6x
x+4
(c) [5 MARKS] lim
x→−4 x2 + 5x + 4

2. In each of the following cases evaluate the limit, or explain why the limit does not
exist. Show all your work. If you wish, you may use l’Hôpital’s Rule.
2
ex − 1
(a) [10 MARKS] lim .
x→0 sec x − 1

sinh(x + 1)
(b) [5 MARKS] lim . (Hint: Express the functions in terms of
x→∞ cosh(x)
exponentials.)
Information for Students in MATH 140 2008 09 3201

3. Showing your work, find the derivative of each of the following functions at the
points indicated, or explain why the function fails to be differentiable there. In
all cases you are expected to simplify your answers as much as possible, but the
examiners are aware that you do not have a calculator.
π
(a) [5 MARKS] a(x) = ex sin x at x = − .
2

x+5
(b) [5 MARKS] b(x) = √ at x = 4 .
x−5
eln(2x)
(c) [5 MARKS] c(x) = at x = 4.
ln(e3x )
4. A function K(x) is defined as follows, where α and β are constants to be evaluated:
½
α + x − x2 if x < 2
K(x) =
x2 − β(x − 2) − 4 if x ≥ 2

(a) [8 MARKS] Showing all your work, determine all values of α and β — if any
— that will make K continuous at x = 2.
(b) [7 MARKS] Showing all your work, determine all values of α and β — if any
— that will make K differentiable at x = 2.

5. Suppose that y = 1 − xy 4 .
dy
(a) [10 MARKS] Showing all your work, determine the values of and
dx
d2 y
when x = 0 .
dx2
(b) [5 MARKS] Find an equation for the tangent to the curve y = 1 − xy 4
at the point where x = 0 .
1 1
6. Let f (x) = − − 1.
x x−1
(a) [3 MARKS] Determine the domain of f .
(b) [5 MARKS] Determine the intervals where f is increasing, and the intervals
where it is decreasing.
(c) [5 MARKS] Determine the intervals where the graph of f is concave upwards,
and the intervals where it is concave downwards.
(d) [5 MARKS] Determine the local extrema of f , or prove that there are no
local extrema.
Information for Students in MATH 140 2008 09 3202

(e) [5 MARKS] Determine the inflection points of the graph of f , or prove that
there are no inflection points.
(f) [4 MARKS] Determine the horizontal asymptotes (if any) and the vertical
asymptotes (if any) to the graph of f .
(g) [3 MARKS] Sketch the graph of f .

7. A line ` with positive slope m is drawn through the point (−4, 9) in the plane.

(a) [1 MARK] Find an equation for `.


(b) [2 MARKS] ` intersects the x-axis in a point A and the y-axis in a point B.
Find the coordinates of A and B.
(c) [12 MARKS] Showing all your work, determine which values of m minimize
the area of the triangle AOB (where O denotes the origin).

4
8. (a) [6 MARKS] Showing all your work, find a linear approximation to 10020 .
(b) [2 MARKS] Carefully state the Mean Value Theorem.
(c) [7 MARKS] Showing all your work, apply the Mean Value Theorem to show
that the function arctan x − x is equal to zero only at x = 0 .
(d) [2 MARKS] Prove that the curves y = x and y = arctan x intersect only
at the origin.

8.10 May 2002 Supplemental/Deferred Examination in 189-


140A
1. Compute the derivatives of the functions

(a) [4 MARKS] f (x) = 2x x3



(b) [4 MARKS] g(x) = sinh( x4 + 1)
sec x
(c) [4 MARKS] h(x) =
arctan x
2. Suppose that the function f satisfies
f (x) − 3
lim =4
x→1 x2 − 1

(a) [6 MARKS] Find lim f (x).


x→1

(b) [6 MARKS] Given that f is continuous at x = 1, use the definition of the


derivative to show that f is differentiable at x = 1 and find f 0 (1).
Information for Students in MATH 140 2008 09 3203

3. The equation
y 5 + xy 2 + x3 = 4x + 3
defines y implicitly as a function of x near the point (2, 1).

(a) [6 MARKS] Determine the values of y 0 and y 00 at this point.


(b) [4 MARKS] Use the tangent line approximation to estimate y when x = 1.98.
x3
4. Let f (x) = .
x2 − 4
(a) [3 MARKS] Compute and simplify f 0 (x) and f 00 (x).
(b) [3 MARKS] Determine, if any, all vertical and horizontal
asymptotes of f .
(c) [3 MARKS] Determine the intervals where f is increasing, and the intervals
where f is decreasing.
(d) [3 MARKS] Determine all local maxima and all local minima.
(e) [3 MARKS] Determine all intervals where (the graph of) f is concave upward
and all intervals where (the graph of) f is concave downward.
(f) [3 MARKS] Determine all inflection points.
(g) [3 MARKS] Sketch the graph of f .

5. Evaluate the following limits, using l’Hôpital’s rule or otherwise:


tan(3x)
(a) [5 MARKS] lim
sin(5x)
x→0
¡ 1/x ¢
(b) [6 MARKS] lim xe − x
x→∞

6. A rectangle is inscribed with its base on the x-axis, its upper left vertex on the
y-axis and its upper right vertex on the graph of the function y = e−2x .

(a) [6 MARKS] Find the dimensions of the rectangle that maximize its area. Fully
justify your answer!
(b) [6 MARKS] Find the dimensions of the rectangle that minimize its circum-
ference. Fully justify your answer!

7. [10 MARKS] Use the mean value theorem, or properties


√ of the derivative, to show
that the graphs of y = arcsin x and y = 2x 2 intersect exactly once for x

2
between 2 and 1.
Information for Students in MATH 140 2008 09 3204

8. In each of the following problems, find the function that satisfies all the stated
conditions:

(a) [6 MARKS] f 0 (x) = e2x − sin x, f (0) = 0.


1
(b) [6 MARKS] g 00 (x) = 2 , g 0 (1) = 0, g(1) = 0.
x

8.11 December 2002 Final Examination in MATH 140 2002 09


A TOTAL OF 140 MARKS ARE AVAILABLE ON THIS EXAMINATION.

1. In each of the following cases evaluate the limit, or explain why the limit does not
exist. Show all your work. Do not use l’Hôpital’s Rule.
5+u
(a) [5 MARKS] lim .
u→−5 u2 − 25

x2
(b) [5 MARKS] lim
x→0 sin2 3x

(c) [10 MARKS] lim x(cosh ln x − sinh ln x).


x→∞

2. In each of the following cases evaluate the limit, or explain why the limit does not
exist. Show all your work. If you wish, you may use l’Hôpital’s Rule.
2 − x2 − 2 cos x
(a) [7 MARKS] lim .
x→0 x4
(b) [8 MARKS] lim x (2 arctan x − π) .
x→∞

3. In each of the following problems you are expected to show your work, and to
simplify your answer as much as possible. The examiners are aware that you do
not have the use of a calculator.

(a) [5 MARKS] Find the value of the derivative of a(x) = tan(1 + x2 ) at x = 0 .


(b) [8 MARKS] Find the value of the derivative of f (x) = 2 · xln x at x = e3 .
(c) [7 MARKS] Find a function f (x) such that f (−1) = 7 and, for x < 0 ,
2
f 0 (x) = .
x
4. Let A be the point (6, −3) on the curve C with equation x2 = y 2 (y + 7) .

(a) [12 MARKS] Showing all your work, determine an equation for the tangent
to the curve at A .
Information for Students in MATH 140 2008 09 3205

(b) [8 MARKS] A point P is moving along the curve C . Let t represent time.
dy
Determine the value of when P is at position A on the curve, if it is
dt
known that, at that moment,
dx
= 4 m/s.
dt
1
5. Let f (x) = x(4 − x2 ) 2 .
(a) [3 MARKS] Determine the (largest possible) domain of f .
(b) [5 MARKS] Determine the intervals where f is increasing, and the intervals
where it is decreasing.
(c) [5 MARKS] Determine the local extrema of f , or prove that there are no
local extrema.
(d) [5 MARKS] Prove that f 00 (x) = 2x(x2 − 6)(4 − x2 )−3/2 . Then determine the
intervals where the graph of f is concave upwards, and the intervals where
it is concave downwards.
(e) [5 MARKS] Determine the inflection points of the graph of f , or prove that
there are no inflection points.
(f) [4 MARKS] Determine every horizontal asymptote to the graph of f , or prove
that there is none; determine every vertical asymptote to the graph of f , or
prove that there is none.
(g) [3 MARKS] Sketch the graph of f .
6. (a) [8 MARKS] Determine the derivative of the function
1+x
h(x) = arctan x + arctan 1 − arctan .
1−x
for x 6= 1 . You are expected to simplify your answer.
(b) [7 MARKS] Use your solution to question 6.(a) to determine the value of
h(−5) . Only a solution using the previous result will be accepted. Reduce
your answer as much as possible; the examiners are aware that you do not
have the use of a calculator.

7. Let f (x) = sin3 x + 3 · cos3 x .
(a) [10 MARKS] Showing all your work, determine all local maxima (= relative
maxima) and all local minima (= relative minima) x of f such that
π π
− <x< .
2 2
Information for Students in MATH 140 2008 09 3206

(b) [5 MARKS] Determine the global maximum (= absolute maximum) and


global minimum (= absolute minimum) value of f on the interval
π π
− ≤x≤ .
2 2
(c) [5 MARKS] Showing all your work, use a linear approximation to estimate
the value of √
sin3 62◦ + 3 · cos3 62◦ .

◦ 1 ◦ 3
You may assume that the sine of 30 is , and that the sine of 60 is .
2 2

8.12 May 2003 Supplemental/Deferred Examination in MATH


140 2002 09
1. In each of the following cases, evaluate the limit or explain why the limit does not
exist. Do not use L’Hôpital’s Rule!
2x3 − 3x2 + 7
(a) [3 MARKS] lim
x→∞ −x3 + 5x − 1

x+4−3
(b) [3 MARKS] lim
x→5 x−5
(tan 2x) · (cos 2x)
(c) [3 MARKS] lim
x→0 x
½
2x − 4a2 if x ≤ 2
2. Let a be a real number, and f (x) = 2 .
a(x − 6x + 8) if x > 2

(a) [5 MARKS] Determine the value(s) of a — if any — for which f is continuous.


(b) [5 MARKS] Explain why f is not differentiable at x = 2 if a = −2.

3. Compute the derivatives of the following functions. Simplify your answers!


sin x
(a) [4 MARKS] f (x) = .
ex
µ ¶
1
(b) [4 MARKS] g(x) = arctan .
x
³ √ ´
(c) [4 MARKS] h(x) = ln x + 1 + x2 .

4. (a) [5 MARKS] Determine the function f if it is known that f 00 (x) = 6x − 6,


f (0) = −2 and f (1) = 3.
Information for Students in MATH 140 2008 09 3207

3
(b) [5 MARKS] Let g(x) = √ . Find an antiderivative G of g such that
Ã√ ! 1 − x2
3
G = 0.
2

5. Consider the curve C with equation 2x3 + 2y 3 = 9xy .

(a) [2 MARKS] Verify that the point P = (1, 2) lies on C.


(b) [4 MARKS] Determine the equation of the tangent line to C at P .
d2 y
(c) [6 MARKS] Determine when x = 1. Is C concave upward or concave
dx2
downward at P ?

6. [10 MARKS] Use the Mean Value Theorem to prove that x = 0 is the only solution
of the equation sinh x = x. Carefully explain your reasoning!

7. [10 MARKS] A particle is moving along the parabola 4y = (x + 1)2 in such a


way that its x-coordinate is increasing at the constant rate of 5 units per second.
Determine how fast the distance from the particle to the origin is changing at the
instant the particle is at the point (3, 4).

8. [10 MARKS] A rectangle with sides parallel to the coordinate axes has one vertex
at the origin, one on the positive x-axis, one on the positive y-axis, and its fourth
1
vertex in the first quadrant on the curve y = . What is the maximum
1 + x2
possible area of such a rectangle? Fully justify your answer!

9. Let f (x) = (x2 + 4x + 4)e−x .

(a) [2 MARKS] Determine all x — if any — for which f (x) = 0 .


(b) [4 MARKS] Determine all horizontal asymptote(s) — if any — to the graph
of f .
(c) [4 MARKS] Find all local extrema — if any — of f ; determine which are
local maxima and which are local minima. Determine the intervals where f
is increasing, and the intervals where f is decreasing.
(d) [4 MARKS] Find all inflection points — if any — of f . Determine the intervals
where the graph of f is concave upward, and the intervals where the graph is
concave downward.
(e) [3 MARKS] Sketch the graph of f .
Information for Students in MATH 140 2008 09 3208

8.13 December 2003 Final Examination in MATH 140 2003 09


Students were advised that there were two kinds of problems on this examination, each
clearly marked as to its type:
• “Some of the questions on this paper require that you SHOW ALL YOUR WORK!
Their solutions are to be written in the space provided on the page where the
question is printed. When that space is exhausted, you may write on the facing
page. Any solution may be continued on the last pages, or the back cover of the
booklet, but you must indicate any continuation clearly on the page where the
question is printed!

• “Some of the questions on this paper require only BRIEF SOLUTIONS ; for these
you are expected to write the correct answer in the box provided; you are not asked
to show your work, and you should not expect partial marks for solutions that are
not completely correct.
“You are expected to simplify your answers wherever possible.”
1. BRIEF SOLUTIONS
[2 MARKS EACH] Give the limit in each of the following cases. If the limit does
not exist, or is +∞ or −∞, write “DOES NOT EXIST”, +∞, or −∞ respectively.
15 − 2x − x2
(a) lim− =
x→3 x−3
ANSWER ONLY

7x5 + 2x3 − x2 + 11
(b) lim =
x→−∞ x5 − 3x4 + 2
ANSWER ONLY

tan 5x
(c) lim =
x→0 sin 4x
ANSWER ONLY
Information for Students in MATH 140 2008 09 3209

30 + 6x
(d) lim − =
x→−5 |5 + x|
ANSWER ONLY

√ √
(e) lim ( x2 + x − x2 − 4x) =
x→∞

ANSWER ONLY

(f) lim x2 ex =
x→−∞

ANSWER ONLY

2. BRIEF SOLUTIONS
[2 MARKS EACH] Determine each of the following derivatives.
d sin t2
(a) e =
dt
ANSWER ONLY

d
(b) ln(e4x ) =
dx
ANSWER ONLY

d 4
(c) |x | =
dx
ANSWER ONLY
Information for Students in MATH 140 2008 09 3210

d √
(d) arcsin x =
dx
ANSWER ONLY

(e) If y(x) satisfies the equation


(y(x))5 + x (y(x))2 + 9x4 = 1 , then y 0 (0) =
ANSWER ONLY

¡ ¢x
(f) If g(x) = x6 2 , g 0 (0) =

ANSWER ONLY

3. BRIEF SOLUTIONS
[3 MARKS EACH]

(a) Give equations for all of the vertical asymptotes of the graph of
x
f (x) = .
x2 +x−2
If there is none, write “NONE”.
ANSWER ONLY

(b) Determine all values of the constant c that will make the function
½
−4x + c when x < 0
f (x) =
(x + c)2 when x ≥ 0

continuous from the right at x = 0. If there is none, write “NONE”.


Information for Students in MATH 140 2008 09 3211

ANSWER ONLY

(c) Determine all horizontal asymptotes to the following curve; if there is none,
write “NONE”.
y = arctan(x2 )
ANSWER ONLY

3
(d) If tanh x = , sinh x =
5
ANSWER ONLY

4. BRIEF SOLUTIONS
[4 MARKS EACH]
(a) Determine all points on the curve
y 3 − x2 = 4
where the tangent is horizontal. If there is none, write “NONE”.
ANSWER ONLY

(b) On the interval −2 < x < 0, determine the values of x at which the function
f has a local (=relative) minimum, if it is known that
f 0 (x) = (2x + 1)(x + 1)2 (x + 3) .
If there is no local minimum, write “NONE”.
Information for Students in MATH 140 2008 09 3212

ANSWER ONLY

(c) Find f (2) if it is known that

x2 − x + 1
f 0 (x) = , and
x
f (1) = 5 .

ANSWER ONLY

The examiners are aware that you do not have a calculator.

5. SHOW ALL YOUR WORK!


[3 MARKS EACH]
For each of the following descriptions of a function or functions, either

• give an example of functions with the properties stated, or


• name or state a theorem, law, or rule which can be used to show that no such
function or functions exist.

(a) f (3) = 0, f (1) = −4, f 0 (x) ≤ 1 for all x.


(b) f is continuous on [−4, 4], f (−3) = −1, f (3) = 2, f (x) 6= 0 for all x.
³ ´
(c) lim f (x) = 3, f lim x = 2 .
x→4 x→4
0
(d) f (1) = 5 and f is not continuous at x = 1 .

6. SHOW ALL YOUR WORK!


[12 MARKS] A balloon leaves the ground 100 metres from an observer, and rises
vertically at the rate of 40 metres per minute. Determine the rate at which the
angle of inclination of the observer’s line of sight (the angle between the line of
sight and the horizontal) is increasing at the instant when the balloon is exactly
100 metres above the ground?
Information for Students in MATH 140 2008 09 3213

7. SHOW ALL YOUR WORK!



Let f (x) = x x + 3 .
(a) [2 MARKS] State the domain of f .
(b) [4 MARKS] Find the intervals of increase and the intervals of decrease of f .
(c) [4 MARKS] Find the absolute (global) maximum and minimum values of f
— if any — and the points where they are attained.
(d) [3 MARKS] Determine the intervals of concavity upwards and the intervals of
concavity downwards, and the inflection points, if any.
(e) [3 MARKS] Sketch the graph of y = f (x).

8. SHOW ALL YOUR WORK!


Let t represents time measured in seconds, and let C be a constant that is to
be determined. A particle moves so that its position on the x axis at time t is
x(t) = t3 − 12t2 + 36t + C.
(a) [3 MARKS] Determine the acceleration of the particle as a function of time.
(b) [6 MARKS] The speed of the particle is the absolute value of its velocity.
Determine the time intervals when the speed is increasing.
(c) [3 MARKS] Determine the value of C if it is known that the particle is at the
origin the whenever the acceleration is 0.

8.14 May 2004 Supplemental/Deferred Examination in MATH


140 2003 09
Students were advised that there were two kinds of problems on this examination, each
clearly marked as to its type:
• “Some of the questions on this paper require that you SHOW ALL YOUR WORK!
Their solutions are to be written in the space provided on the page where the
question is printed. When that space is exhausted, you may write on the facing
page. Any solution may be continued on the last pages, or the back cover of the
booklet, but you must indicate any continuation clearly on the page where the
question is printed!
• “Some of the questions on this paper require only BRIEF SOLUTIONS ; for these
you are expected to write the correct answer in the box provided; you are not asked
to show your work, and you should not expect partial marks for solutions that are
not completely correct.
Information for Students in MATH 140 2008 09 3214

“You are expected to simplify your answers wherever possible.”

1. BRIEF SOLUTIONS
[2 MARKS EACH] Find each of the following. If the item requested does not exist,
or is +∞ or −∞, write “DOES NOT EXIST”, +∞, or −∞ respectively.

(a) If g(x) = 2 + x + ex , find g −1 (3).


ANSWER ONLY

(b) Find a formula for the inverse of the function

r(x) = ln(x + 2) .

ANSWER ONLY


t2 + 4 − 2
(c) Find lim .
t→0 t2
ANSWER ONLY

³ √ ´
(d) lim x+ x2 + 3x =
x→−∞

ANSWER ONLY

(You may do rough work in this space, or on the back of the preceding page.)

2. BRIEF SOLUTIONS
[2 MARKS EACH] Determine each of the following derivatives.
d ³ 3t ´
(a) 2 =
dt
Information for Students in MATH 140 2008 09 3215

ANSWER ONLY

µ ¶
d ex
(b) =
dx x2
ANSWER ONLY

d ¡ 2 ¢
(c) (x + 1)50 (x4 + 1)3 =
dx
ANSWER ONLY

d
(d) sin(sin 2x) =
dx
ANSWER ONLY

3. SHOW ALL YOUR WORK!


Full explanations are required for all of your statements.

(a) [4 MARKS] Determine the domain of the function


√ √
r(x) = x − −x .

(b) [4 MARKS] Determine whether the function


½
x − 2 for x < 3
h(x) =
5 − x for x ≥ 3

is continuous at x = 3.
(c) [4 MARKS] Determine whether the function f (x) = |x − 4|5 is differentiable
at x = 4.
Information for Students in MATH 140 2008 09 3216

4. SHOW ALL YOUR WORK!


[6 MARKS] A plane flying horizontally at an altitude of 2 km and a speed of 500
km/h passes directly over a certain radar station. Determine the rate at which the
(shortest) distance from the plane directly to the station is increasing when the
plane is 3 km away from the station.

5. [8 MARKS] SHOW ALL YOUR WORK!


Showing all your work, and naming any theorems that you use, prove that the
equation x5 − 7x + 5 = 0 has exactly one root in the closed interval [−1, +1].

6. SHOW ALL YOUR WORK!

(a) [6 MARKS] Find an equation for the line tangent to the curve y 2 = x3 (2 − x)
at the point (1, 1).
(b) [8 MARKS] A particle is moving on the x axis so that its position at time t
is given by
x(t) = 2t3 − 7t2 + 4t + 1 .
Find
i. The times when the velocity is 0.
ii. The acceleration at each time when the velocity is 0.

7. SHOW ALL YOUR WORK!

(a) [2 MARKS] State the Mean Value Theorem.


(b) [8 MARKS] For the function
½
2|x + 1| when x ≤ −1
f (x) =
x(1 − |x|)(1 + |x|) when x > −1

decide whether f satisfies the hypotheses of the Mean Value Theorem on the
interval [−2, 3]. Then answer the appropriate one of the following questions:
I. If f satisfies the conditions of the Mean Value Theorem on the interval
[−2, 3], explain precisely what the conclusion of the theorem is for this
function.
II. If f does not satisfy the conditions of the Mean Value Theorem, explain
precisely where it fails to satisfy those conditions.
Information for Students in MATH 140 2008 09 3217

8. SHOW ALL YOUR WORK!


Let x and y = f (x) be related by the equation

ey + x · y = e .

Showing all your work,

(a) [2 MARKS] determine f (0);


(b) [3 MARKS] determine f 0 (0);
(c) [5 MARKS] determine f 00 (0).

9. SHOW ALL YOUR WORK!



Let f (x) = x 4 − x2 .

(a) [2 MARKS] State the domain of f .


(b) [4 MARKS] Find the intervals of increase and the intervals of decrease of f .
(c) [4 MARKS] Find the absolute (global) maximum and minimum values of f
— if any — and the points where they are attained.
(d) [3 MARKS] Determine the intervals of concavity upwards and the intervals of
concavity downwards, and the inflection points, if any.
(e) [2 MARKS] Sketch the graph of y = f (x).

10. [9 MARKS] SHOW ALL YOUR WORK!


A metal box with a square base and an open top must have a volume of 500 m3 .
Find the dimensions of the box that minimize the total amount of metal used.

8.15 December 2004 Final Examination in MATH 140 2004 09


(One of several versions)
Students were advised that there were two kinds of problems on this examination, each
clearly marked as to its type:

• “Some of the questions on this paper require that you SHOW ALL YOUR WORK!
Their solutions are to be written in the space provided on the page where the
question is printed. When that space is exhausted, you may write on the facing
page. Any solution may be continued on the last pages, or the back cover of the
booklet, but you must indicate any continuation clearly on the page where the
question is printed!
Information for Students in MATH 140 2008 09 3218

• “Some of the questions on this paper require only BRIEF SOLUTIONS ; for these
you are expected to write the correct answer in the box provided; you are not asked
to show your work, and you should not expect partial marks for solutions that are
not completely correct.

“You are expected to simplify your answers wherever possible.”

1. BRIEF SOLUTIONS
[3 MARKS EACH] Give the numeric value of each of the following limits if it exists;
if the limit is +∞ or −∞, write +∞ or −∞ respectively. In all other cases write
”NO FINITE OR INFINITE LIMIT”.
|y|
(a) lim =
y→−∞ y

ANSWER ONLY

sin u
(b) lim =
u→−∞ u

ANSWER ONLY

µ ¶2n
1
(c) lim 1+ =
n→∞ n
ANSWER ONLY

µ ¶
1 1
(d) lim − =
x→0 x sin x
ANSWER ONLY

¡ ¢
(e) lim ln(3t2 ) − ln(t2 + 7) =
t→∞
Information for Students in MATH 140 2008 09 3219

ANSWER ONLY

√ √
x + 8 − 2x
(f) lim =
x→8 x2 − 8x
ANSWER ONLY

2. BRIEF SOLUTIONS
[3 MARKS EACH] Determine each of the following derivatives, and simplify your
answers as much as possible.
µ ¶
d x2 + 3x
(a) =
dx x

ANSWER ONLY

d ¡ −3t ¢
(b) t =
dt

ANSWER ONLY

d ¡ ¢
(c) tan(e2s ) − e2 tan s =
ds

ANSWER ONLY

d ¡ ¢
(d) cosh2 (3y) =
dy
Information for Students in MATH 140 2008 09 3220

ANSWER ONLY

d ¡ 2 ¢
(e) cos X − cos(X 2 ) =
dX

ANSWER ONLY

3. BRIEF SOLUTIONS

(a) [5 MARKS] Find an equation for a line through the point (−2, 0) which is
tangent to the curve y = x2 and is not horizontal.

ANSWER ONLY

(b) [5 MARKS] Determine values of the constants a and b that will make the
following function continuous at x = −6.
 √
 x + 31 if x < −6
f (x) = a+b if x = −6

a(x + 5) if x > −6

ANSWER ONLY

(c) [5 MARKS] Determine values of the constants k and ` that will make the
following function differentiable at x = 1.
½
kx2 + ` if x ≤ 1
g(x) =
6x − 4 if x > 1
Information for Students in MATH 140 2008 09 3221

ANSWER ONLY

4. SHOW ALL YOUR WORK!

(a) [6 MARKS] Coffee is draining from a conical filter of depth 10 cm and diameter
10 cm (at the top) into a cylindrical coffee pot of diameter 12 cm, at the rate
of 100 cm3 /min. Determine how fast, in cm/min, the level of coffee in the pot
is rising when the coffee in the filter is 3 cm deep?
(b) [6 MARKS] You are given that y = y(t) is a function of t satisfying t3 y +ty 3 =
2. Assuming that y(1) = 1, determine the values of y 0 (1) and y 00 (1).

5. SHOW ALL YOUR WORK!

(a) [4 MARKS] Prove that the function x3 + 9x2 + 33x assumes the value −8 at
least once.
(b) [8 MARKS] Using the Mean Value Theorem or Rolle’s Theorem — no other
methods will be accepted — prove carefully that x3 + 9x2 + 33x takes on the
value −8 at most once.

6. SHOW ALL YOUR WORK!


[10 MARKS] Using the calculus carefully, determine how to express 8 as the sum
of 2 nonnegative real numbers such that the sum of the square of the first and the
cube of the second is as small as possible.

7. SHOW ALL YOUR WORK!


µ ¶2
x
Let f (x) = .
x+3
(a) [1 MARKS] State the domain of f .
(b) [4 MARKS] Find the intervals of increase and the intervals of decrease of f .
(c) [4 MARKS] Determine the intervals of concavity upwards and the intervals of
concavity downwards, and the inflection points, if any.
(d) [3 MARKS] Sketch the graph of y = f (x), showing — clearly labelled — all
horizontal and all vertical asymptotes.
Information for Students in MATH 140 2008 09 3222

8. SHOW ALL YOUR WORK!


[6 MARKS] Consider the function f (x) = 5x + 9 near x = −1. Is it, or is it not
true that f is continuous at x = −1? If the statement is true, prove it carefully,
using the ²-δ definition. If it is false, prove that carefully.

8.16 May 2005 Supplemental/Deferred Examination in MATH


140 2004 09
Students were advised that there were two kinds of problems on this examination, each
clearly marked as to its type:
• “Some of the questions on this paper require that you SHOW ALL YOUR WORK!
Their solutions are to be written in the space provided on the page where the
question is printed. When that space is exhausted, you may write on the facing
page. Any solution may be continued on the last pages, or the back cover of the
booklet, but you must indicate any continuation clearly on the page where the
question is printed!
• “Some of the questions on this paper require only BRIEF SOLUTIONS ; for these
you are expected to write the correct answer in the box provided; you are not asked
to show your work, and you should not expect partial marks for solutions that are
not completely correct.
“You are expected to simplify your answers wherever possible.”
1. BRIEF SOLUTIONS
[3 MARKS EACH] Give the numeric value of each of the following limits if it exists;
if the limit is +∞ or −∞, write +∞ or −∞ respectively. In all other cases write
“NO FINITE OR INFINITE LIMIT”.
µ ¶
2x 8
(a) lim − =
x→4 x−4 x−4
ANSWER ONLY

1 − cos u
(b) lim =
u→∞ u2
ANSWER ONLY
Information for Students in MATH 140 2008 09 3223

µ ¶−n
1
(c) lim 1− =
n→∞ n
ANSWER ONLY

µ ¶
1 1
(d) limπ − =
x→ 2 x sin x

ANSWER ONLY

x+2
(e) lim √ √ =
x→−2 −x − 2
ANSWER ONLY

2. BRIEF SOLUTIONS
[3 MARKS EACH] Determine each of the following derivatives or antiderivatives,
and simplify your answers as much as possible. If the derivative or antiderivative
does not exist, write “DOES NOT EXIST”.
µ ¶
d x2 + 3x
(a) =
dx x

ANSWER ONLY

d ¡ −3t ¢
(b) t =
dt

ANSWER ONLY
Information for Students in MATH 140 2008 09 3224

d
(c) (sinh(ln s) − ln(sinh s)) =
ds

ANSWER ONLY

d ¡ ¢
(d) At the point y = −1, 2y · | − y| + 3 · |y|4 =
dy

ANSWER ONLY

(e) A function f such that f 00 (θ) = cos θ − sin θ, f (0) = 4, f 0 (π) = −3.

ANSWER ONLY

3. BRIEF SOLUTIONS

(a) [5 MARKS] Find all points — if any — on the graph of the function
f (x) = 2 sin x + sin2 x (−π ≤ x ≤ π)
at which the tangent line is horizontal.

ANSWER ONLY

(b) [5 MARKS] Determine values of the constants a and b that will make the
following function continuous at x = 24.
 √
 49 − x if x < 24
f (x) = a−b if x = 24

b(x − 19) if x > 24
Information for Students in MATH 140 2008 09 3225

ANSWER ONLY

(c) [5 MARKS] Determine the domain of the function

sin(arcsin x) − arcsin(sin x) .

ANSWER ONLY

4. SHOW ALL YOUR WORK!

(a) [6 MARKS] You are given that y = y(x) is a function of x satisfying

2x3 − 3xy 4 + 5xy − 10 = 0 .

Express the value of y 0 (x) in terms of x and y.


(b) [9 MARKS] Suppose the lengths of the sides of a triangle are a, b and c. Define
s = 12 (a + b + c). Heron’s formula for the area T of the triangle is
p
T = s(s − a)(s − b)(s − c) .

If the lengths of the sides are all increasing at the rate of 1 cm/min, determine
— using logarithmic differentiation or otherwise — the rate of change of the
ds
area, T , when a=3 cm, b=4 cm, c= 5 cm. (Hint: First find .)
dt

5. SHOW ALL YOUR WORK!

(a) [4 MARKS] Prove that the function x3 + 9x2 + 33x assumes the value −8 at
least once.
(b) [8 MARKS] Using the Mean Value Theorem or Rolle’s Theorem — no other
methods will be accepted — prove carefully that x3 + 9x2 + 33x takes on the
value −8 at most once.
Information for Students in MATH 140 2008 09 3226

6. SHOW ALL YOUR WORK!


[10 MARKS] Determine all points on the curve y = 1 − 40x3 + 3x5 at which the
slope of the tangent line has a local maximum, and all points where the slope of
the tangent line has a local minimum. Then discuss whether the slope has global
maxima and global minima.

7. SHOW ALL YOUR WORK!


1
Let f (x) = e− x .

(a) [1 MARKS] State the domain of f .


(b) [4 MARKS] Find the intervals of increase and the intervals of decrease of f .
(c) [4 MARKS] Determine the intervals of concavity upwards and the intervals of
concavity downwards, and the inflection points, if any.
(d) [3 MARKS] Sketch the graph of y = f (x), showing — clearly labelled — all
horizontal and all vertical asymptotes.

8. SHOW ALL YOUR WORK!


[6 MARKS] Let f (x) = 5x − 8. Given a real number ² > 0, find a real number δ
— depending on ² — such that |f (x) − 7| < ² whenever 0 < |x − 3| < δ.

8.17 December, 2005, Final Examination in MATH 140 2005


09 (One of several versions)
Students were advised that there were two kinds of problems on this examination, each
clearly marked as to its type:

• “Some of the questions on this paper require that you SHOW ALL YOUR WORK!
Their solutions are to be written in the space provided on the page where the
question is printed. When that space is exhausted, you may write on the facing
page. Any solution may be continued on the last pages, or the back cover of the
booklet, but you must indicate any continuation clearly on the page where the
question is printed!

• “Some of the questions on this paper require only BRIEF SOLUTIONS ; for these
you are expected to write the correct answer in the box provided; you are not asked
to show your work, and you should not expect partial marks for solutions that are
not completely correct.

“You are expected to simplify your answers wherever possible.”


Information for Students in MATH 140 2008 09 3227

1. BRIEF SOLUTIONS
[2 MARKS EACH] Give the numeric value of each of the following limits if it exists;
if the limit is +∞ or −∞, write +∞ or −∞ respectively. In all other cases write
“NO FINITE OR INFINITE LIMIT”.
³p ´
(a) lim y2 + y + y =
y→−∞

ANSWER ONLY

¡ ¢
(b) lim e−x sinh x =
x→∞

ANSWER ONLY

√ √
x + 8 + 2x
(c) lim =
x→8 x2 + 8x

ANSWER ONLY

µ ¶
1 1
(d) lim + =
x→1 1 − x ln x

ANSWER ONLY

sin ex
(e) lim =
x→∞ cos ex

ANSWER ONLY
Information for Students in MATH 140 2008 09 3228

2. BRIEF SOLUTIONS
[2 MARKS EACH] Evaluate each of the following, and simplify your answers as
much as possible.
µ ¶
d x2 + 3x
(a) =
dx x

ANSWER ONLY

d u
(b) (u ) =
du

ANSWER ONLY

(c) An antiderivative F (x) of f (x) = 5x4 + 2x5 such that F (0) = 3 is

ANSWER ONLY

(d) If f (x) = x3 + 7, its inverse function f −1 (x) =

ANSWER ONLY

d ¡ 4¢
(e) |x| =
dx

ANSWER ONLY
Information for Students in MATH 140 2008 09 3229

3. SHOW ALL YOUR WORK!

(a) [6 MARKS] Showing all your work, determine values of the constants a and
b that will make the following function continuous everywhere.
 1

 (1 + x) x if x > 0

f (x) = a + bx if −1 ≤ x ≤ 0

 sin(x + 1)
 if x < −1
x+1

(b) [4 MARKS] Determine whether f is differentiable at x = −1. (For the purpose


of this question you may assume that e is approximately 2.72.)

4. SHOW ALL YOUR WORK!


[5 MARKS] Let f (x) = x2 ex . Prove carefully by mathematical induction that

dn f ¡ 2 ¢ x
(x) = x + 2nx + (n − 1)n ·e
dxn
for all positive integers n.

5. SHOW ALL YOUR WORK!


[5 MARKS] Let g(x) = 2x − 3 + cos x. Use Rolle’s Theorem or the Mean Value
Theorem, to prove carefully that there exists exactly one real number x such that
g(x) = 0. (π may be taken to be approximately 3.14.)

6. SHOW ALL YOUR WORK!


[10 MARKS] A rectangular poster is to be printed on a rectangular board of min-
imum area, leaving margins at the 4 sides. The top and bottom margins are each
10 cm, and the side margins are each 4 cm. If the printed area on the poster is
fixed at 1,000 cm2 , find the best dimensions for the board. Show all of your work,
and justify all of your statements. In your solution, you are expected to carefully
apply either the First or the Second Derivative Test, naming the test as you apply
it.

7. SHOW ALL YOUR WORK!



Let f (x) = x2 − 1 .

(a) [1 MARK] State the domain of f .


(b) [1 MARK] State precisely where f is differentiable.
Information for Students in MATH 140 2008 09 3230

(c) [2 MARKS] Define when a line x = a is a vertical asymptote to the graph of


f.
(d) [2 MARKS] Either
i. Find all vertical asymptotes; or
ii. Explain why the graph has no vertical asymptotes.
(e) [2 MARKS] Determine the global maximum value of f , or explain why there
is none.
(f) [2 MARKS] Determine the global minimum value of f , or explain why there
is none.

8. SHOW ALL YOUR WORK!


A function f (t) satisfies, for all real numbers t, the equation

t3 + f (t)3 + 6t2 · f (t) = 8 .

(a) [5 MARKS] Find an equation for the tangent to the graph y = f (t) at the
point (t, y) = (2, 0).
(b) [5 MARKS] Showing all your work, determine the value of f 00 (2).

8.18 May, 2006, Supplemental/Deferred Examination in MATH


140 2005 09
Students were advised that there were two kinds of problems on this examination, each
clearly marked as to its type:

• “Some of the questions on this paper require that you SHOW ALL YOUR WORK!
Their solutions are to be written in the space provided on the page where the
question is printed. When that space is exhausted, you may write on the facing
page. Any solution may be continued on the last pages, or the back cover of the
booklet, but you must indicate any continuation clearly on the page where the
question is printed!

• “Some of the questions on this paper require only BRIEF SOLUTIONS ; for these
you are expected to write the correct answer in the box provided; you are not asked
to show your work, and you should not expect partial marks for solutions that are
not completely correct.

“You are expected to simplify your answers wherever possible.”


Information for Students in MATH 140 2008 09 3231

1. BRIEF SOLUTIONS
[2 MARKS EACH] Give the value of each of the following limits if it exists; if the
limit is +∞ or −∞, write +∞ or −∞ respectively. In all other cases write “NO
FINITE OR INFINITE LIMIT”.
sin e−x
(a) lim =
x→∞ cos e−x

ANSWER ONLY

³p p ´
(b) If a and b are real numbers, lim y 2 + ay − y 2 − by =
y→∞

ANSWER ONLY

¡ ¢
(c) lim e−x cosh x =
x→−∞

ANSWER ONLY

sin x + sin π6
(d) limπ =
x→ 6 x + π6

ANSWER ONLY

x3
(e) lim+ =
x→0 x − sin x

ANSWER ONLY
Information for Students in MATH 140 2008 09 3232

2. BRIEF SOLUTIONS
[2 MARKS EACH] Evaluate each of the following, and simplify your answers as
much as possible.
(a) An antiderivative G(x) of g(x) = ex + 4x − 1 such that G(0) = 5 is

ANSWER ONLY

µ ¶
d y(cos 2y)
(b) =
dy cos2 y − sin2 y

ANSWER ONLY

d ¡ −x ¢
(c) x =
dx

ANSWER ONLY

d
(d) (x arctan(4x)) =
dx

ANSWER ONLY

(e) If A(t) = ln (t3 + 7), its inverse function A−1 (t) =

ANSWER ONLY
Information for Students in MATH 140 2008 09 3233

3. SHOW ALL YOUR WORK!

(a) [5 MARKS] Showing all your work, determine all values of the constants a
and b that will make the following function continuous everywhere.
½
x(2a − x) if x ≤ 0
f (x) =
x(x2 − 6x + 12) + b if x > 0

(b) [5 MARKS] By examining one-sided limits, determine all values of the con-
stants a and b that will make f differentiable at x = 0.

4. SHOW ALL YOUR WORK!


[5 MARKS] Two sides of a triangle are 3 m. and 4 m. in length, and the angle
between them is increasing at the rate of 1 degree/second. Find the rate at which
the area of the triangle is increasing when the angle between the sides of fixed
π
length is . [Remember that the area of the triangle is one-half the product of the
3
lengths of the base and the altitude (or height).]

5. SHOW ALL YOUR WORK!


[5 MARKS] Use Rolle’s Theorem or the Mean Value Theorem, to prove carefully
that there exists no function f such that f (1) = 5, f (4) = 14, and f 0 (x) ≤ 2 for
−3 ≤ x ≤ 8.

6. SHOW ALL YOUR WORK!


[10 MARKS] Showing all your work, determine the points on the curve y = −3x5 +
40x3 − 1 where the tangent line has the largest positive slope. In your solution,
you are expected to carefully apply either the First or the Second Derivative Test,
naming the test as you apply it.

7. SHOW ALL YOUR WORK!



Let f (x) = x x2 − 1 .

(a) [1 MARK] State the domain of f .


(b) [1 MARK] State precisely where f is differentiable.
(c) [4 MARKS] Investigate whether the graph of f has horizontal and/or vertical
asymptotes, and determine them all, if there are any.
(d) [4 MARKS] Investigate whether the graph of f has global extremum points,
and determine them all, if there are any.
Information for Students in MATH 140 2008 09 3234

8. SHOW ALL YOUR WORK!


A function y = f (x) satisfies, for all real numbers x, the equation

ey + xy = e .

(a) [5 MARKS] Find an equation for the tangent to the graph y = f (x) at the
point (x, y) = (0, 1).
(b) [5 MARKS] Showing all your work, determine the value of f 00 (0).

8.19 December, 2006, Final Examination in MATH 140 2006


09 (One of several versions)
Students were advised that there were two kinds of problems on this examination,

• Some of the questions on this paper require that you SHOW ALL YOUR WORK!
Their solutions are to be written in the space provided on the page where the
question is printed. When that space is exhausted, you may write on the facing
page. Any solution may be continued on the last pages, or the back cover of the
booklet, but you must indicate any continuation clearly on the page where the
question is printed!

• Some of the questions on this paper require only BRIEF SOLUTIONS ; for these
you are expected to write the correct answer in the box provided; you are not asked
to show your work, and you should not expect partial marks for solutions that are
not completely correct.

1. BRIEF SOLUTIONS
[2 MARKS EACH] Give the numeric value of each of the following limits if it exists;
if the limit is +∞ or −∞, write +∞ or −∞ respectively. In all other cases write
“NO FINITE OR INFINITE LIMIT”.
sin 2x
(a) lim =
x→∞ x

ANSWER ONLY

sin 2x
(b) lim =
x→0 x
Information for Students in MATH 140 2008 09 3235

ANSWER ONLY

µ ¶x
2 7
(c) lim 1− + 2 =
x→∞ x x

ANSWER ONLY

|x|
(d) lim =
x→0 x

ANSWER ONLY


(e) lim ( x2 + 3x + x) =
x→−∞

ANSWER ONLY

2. BRIEF SOLUTIONS
[3 MARKS EACH] For each of the following functions answer the question; if the
object requested does not exist, write “NONE”.
(a) The horizontal asymptotes to the graph of g(x) = 2 tanh x + 1 are

ANSWER ONLY

p
(b) The vertical asymptotes to the graph of h(x) = |x2 − 1| are

ANSWER ONLY
Information for Students in MATH 140 2008 09 3236

(c) A ladder 4 metres long rests against a vertical wall. If the bottom of the
ladder slides away from the wall at a speed of 1 metre per second, how fast is
the angle between the top of the ladder and the wall changing when the angle
is π6 radians?

ANSWER ONLY

 2

 x if x < −1

0 if −1 ≤ x ≤ 0
(d) The removable discontinuities of m(x) = are at the

 2x if 0 < x < 3

2x if 3<x
following points:

ANSWER ONLY

3. BRIEF SOLUTIONS
[2 MARKS EACH] Evaluate each of the following, and always simplify your answers
as much as possible.
à 1 2
!
d x 2 + 3x 3
(a) =
dx x

ANSWER ONLY

d ln(ln u)
(b) e =
du

ANSWER ONLY
Information for Students in MATH 140 2008 09 3237

1
(c) An antiderivative F (x) of f (x) = such that F (0) = 3 is
1 + x2

ANSWER ONLY

d ³ √ ´
(d) t arcsin(t) + 1 − t2 =
dt

ANSWER ONLY

4. SHOW ALL YOUR WORK!

(a) [1 MARKS] For a real number a in the domain of a function f , define precisely
what is meant by f 0 (a).
(b) [4 MARKS] Let f (x) = |1 − x|. Carefully, using the preceding definition,
either determine f 0 (1), or prove that f is not differentiable at x = 1. (No
marks will be given for a solution that uses the Differentiation Rules.)

(c) [4 MARKS] It is known that x and y are related by the equation x + y =
dy
1 + x2 y 2 . Using any valid method, determine when x = 0.
dx

5. SHOW ALL YOUR WORK!


[8 MARKS] Showing all your work, determine constants a, b, c so that the tangent
lines at x = −1 and x = 5 to the graph of

y = f (x) = ax2 + bx + c

have slopes 6 and −2 respectively, and that the graph passes through the point
(1, 4).

6. SHOW ALL YOUR WORK!


A particle is moving in the plane on the curve C whose equation is

x = 2y − 4 + ey .
Information for Students in MATH 140 2008 09 3238

(a) [4 MARKS] Use the Mean Value Theorem to prove that C crosses the y-axis
exactly once. (You may assume it is known that 2 < e < 3.)
(b) [3 MARKS] If P (0, a) is the point where C crosses the y-axis, give a linear
approximation in terms of a for the y-coordinate of the point on C where the
x-coordinate is 12 .

7. SHOW ALL YOUR WORK!


[8 MARKS] Showing all your work, use the calculus to find two positive real num-
bers such that the sum of the first and the square of the second is 300, and the
product of the two numbers is as large as possible.

8. SHOW ALL YOUR WORK!


Let f (x) = x − ln(1 + x2 ) .

(a) [2 MARKS] Showing all your work, determine the intervals where f is increas-
ing, and the intervals where it is decreasing.
(b) [2 MARKS] Showing all your work, determine whether f has local extrema,
and classify them, if any, as maxima or minima.
(c) [3 MARKS] Showing all your work, determine all inflection points for f .
(d) [1 MARK] Sketch the graph of f .

8.20 May, 2007, Supplemental/Deferred Examination in MATH


140 2006 09
Students were advised that there were two kinds of problems on this examination,

• Some of the questions on this paper require that you SHOW ALL YOUR WORK!
Their solutions are to be written in the space provided on the page where the
question is printed. When that space is exhausted, you may write on the facing
page. Any solution may be continued on the last pages, or the back cover of the
booklet, but you must indicate any continuation clearly on the page where the
question is printed!

• Some of the questions on this paper require only BRIEF SOLUTIONS ; for these
you are expected to write the correct answer in the box provided; you are not asked
to show your work, and you should not expect partial marks for solutions that are
not completely correct.
Information for Students in MATH 140 2008 09 3239

1. BRIEF SOLUTIONS
[2 MARKS EACH] Give the numeric value of each of the following limits if it exists;
if the limit is +∞ or −∞, write +∞ or −∞ respectively. In all other cases write
“NO FINITE OR INFINITE LIMIT”.

(a) lim cos 2x =


x→∞

ANSWER ONLY

tan x
(b) lim =
x→−π sin 2x

ANSWER ONLY

µ ¶
|x − 2|
(c) lim− =
x→2 2−x

ANSWER ONLY


x2 + 1
(d) lim+ =
x→1 x−1

ANSWER ONLY


(e) lim (2x + 4x2 + 2x) =
x→−∞

ANSWER ONLY
Information for Students in MATH 140 2008 09 3240

2. BRIEF SOLUTIONS
[3 MARKS EACH] For each of the following functions answer the question; if the
object requested does not exist, write “NONE”.

(a) If g(x) = −2 + x + e2x , then g −1 (−1) =

ANSWER ONLY


(b) The horizontal asymptotes to the graph of x2 + 1
g(x) = 2x are
−1

ANSWER ONLY

(c) A snowball melts so that its surface area decreases at a rate of 1 cm2 /minute.
Find, when the radius is 3 cm., the rate at which the radius is decreasing.

ANSWER ONLY

(d) All values


½ of the constant c that will make the function
−2x + c if x < 0
f (x) = continuous everywhere are
(x + c)3 if x ≥ 0

ANSWER ONLY

3. BRIEF SOLUTIONS
[2 MARKS EACH] Evaluate each of the following, and always simplify your answers
as much as possible.
d ¡ ln x ¢
(a) x =
dx
Information for Students in MATH 140 2008 09 3241

ANSWER ONLY

µ ¶
d ln(e2u )
(b) =
du u

ANSWER ONLY

(c) The antiderivative F (x) of f (x) = ex + 2(1 + x2 )−1 such that F (0) = −1 is

ANSWER ONLY

µ ¶
d (x + 1) (x2 + 1) (3x3 + x)
(d) =
dx x−2

ANSWER ONLY

4. SHOW ALL YOUR WORK!


[8 MARKS] Use the Mean Value Theorem, and possibly other theorems, to carefully
show that the equation x5 + 10x + 3 = 0 has exactly one real solution.

5. SHOW ALL YOUR WORK!

(a) [4 MARKS] Find an equation of the tangent at (0, ln 2) to the curve y =


ln (ex + e2x )
½
sinh x if x > 0
(b) [4 MARKS] Suppose f (x) = . Showing all your work,
tan x if x ≤ 0
determine whether f is differentiable at x = 0.
Information for Students in MATH 140 2008 09 3242

6. SHOW ALL YOUR WORK!


[8 MARKS] Suppose that x and y are related by the equation

x3 − 2xy + y 3 = 5 .

d2 y
Find 2 , evaluated when (x, y) = (1, 2).
dx
7. SHOW ALL YOUR WORK!
[8 MARKS] Showing all your work, use the calculus to find the rectangle of largest
area that can be inscribed in an equilateral triangle, if one side of the rectangle
must lie on the base of the triangle.

8. SHOW ALL YOUR WORK!


2
Let f (x) = x 3 (x − 5) .

(a) [2 MARKS] Showing all your work, determine the intervals where f is increas-
ing, and the intervals where it is decreasing.
(b) [2 MARKS] Showing all your work, determine whether f has local extrema,
and classify them, if any, as maxima or minima.
(c) [3 MARKS] Showing all your work, determine all inflection points for f .
(d) [1 MARK] Sketch the graph of f .

8.21 December, 2007, Final Examination in MATH 140 2007


09 (One of several versions)
Instructions
1. Fill in the above clearly.

2. DO NOT TEAR PAGES FROM THIS BOOK! All your writing — even rough work — must
be handed in. You may do rough work anywhere in the booklet.

3. This is a CLOSED BOOK examination. CALCULATORS ARE NOT PERMITTED. Transla-


tion dictionaries are permitted; no other dictionaries are permitted.

4. OTHER CALCULUS EXAMINATIONS ARE BEING WRITTEN AT THIS TIME. THIS IS


THE EXAMINATION IN MATH 140 ONLY!

5. The examination booklet consists of this cover, Pages 1 through 7 containing questions; and
Pages 8, 9, and 10, which are blank. Your neighbour’s version may not be the same as yours.
Information for Students in MATH 140 2008 09 3243

6. There are two kinds of problems on this examination, each clearly marked as to its type.
• Some of the questions on this paper require that you SHOW ALL YOUR WORK!
Their solutions are to be written in the space provided on the page where the question
is printed. When that space is exhausted, you may write on the facing page. Any
solution may be continued on the last pages, or the back cover of the booklet, but you
must indicate any continuation clearly on the page where the question is printed!
• Some of the questions on this paper require only BRIEF SOLUTIONS ; for these you
are expected to write the correct answer in the box provided; you are not asked to show
your work, and you should not expect partial marks for solutions that are not completely
correct.
You are expected to simplify your answers wherever possible.
You are advised to spend the first few minutes scanning the problems. (Please inform the
invigilator if you find that your booklet is defective.)
7. A TOTAL OF 70 MARKS ARE AVAILABLE ON THIS EXAMINATION.
1. BRIEF SOLUTIONS
[2 MARKS EACH] Give the numeric value of each of the following limits if it exists;
if the limit is +∞ or −∞, write +∞ or −∞ respectively. In all other cases write
“NO FINITE OR INFINITE LIMIT”.
x + x2
(a) lim =
x→∞ 1 − 2x2

ANSWER ONLY

sin (3x2 )
(b) lim =
x→0 (sin 3x)2

ANSWER ONLY

µ ¶
1
(c) lim+ arctan − =
x→0 x

ANSWER ONLY
Information for Students in MATH 140 2008 09 3244

u
ln
(d) lim 3 =
u→3 u − 3

ANSWER ONLY

³√ √ ´
(e) lim u2 + 2u + 4 − u2 − 3u + 1 =
u→−∞

ANSWER ONLY

2. BRIEF SOLUTIONS
[3 MARKS EACH] For each of the following functions answer the question; if the
object(s) requested does/do not exist, write “NONE”.

(a) The horizontal asymptotes to the graph of g(x) = 2 arctan x − 1 are

ANSWER ONLY

 1


 x2 −4
if x 6= −2, 0, 2 

 
6 if x=2
(b) If f is defined by f (x) = , the vertical asymptotes

 5 if x=0 

 
−4 if x = −2
to the graph of f are

ANSWER ONLY

(c) Air is being pumped into a spherical balloon so that its volume increases at
a rate of 10 cm3 /s. How fast is the radius of the balloon increasing when the
radius is 12 cm?
Information for Students in MATH 140 2008 09 3245

ANSWER ONLY

3. BRIEF SOLUTIONS
[3 MARKS EACH] Evaluate each of the following, and always simplify your answers
as much as possible.
d¡ x¢
(a) x ln x =
dx

ANSWER ONLY

d
(b) cos(arcsin u) =
du

ANSWER ONLY

(c) An antiderivative F (x) of f (x) = sinh x such that F (0) = −1 is

ANSWER ONLY

s
t2 3−t
(d) Where f (t) = , f 0 (2) =
1−t (3 + t)2

ANSWER ONLY
Information for Students in MATH 140 2008 09 3246

4. SHOW ALL YOUR WORK!

(a) [6 MARKS] Use Rolle’s Theorem and the Intermediate Value Theorem to
show that the curve y = 1 + 2x + x3 + 4x5 crosses the x-axis exactly once.
(b) [4 MARKS] Showing all your work, determine the value of the constant K
that will make the following function continuous at x = 0:

2
 Kx

if x > 0
f (x) = 1 − cos x .

 8 if x ≤ 0

5. SHOW ALL YOUR WORK!


The equation x5 + x2 y + y 3 = 4y + 3 defines y implicitly as a function of
x near the point (x, y) = (1, 2). Showing all your work
(a) [3 MARKS] determine the value of y 0 at (x, y) = (1, 2);
(b) [3 MARKS] determine the value of y 00 at (x, y) = (1, 2); and
(c) [3 MARKS] estimate y when x = 0.97 by using the tangent line to the curve
at the point (x, y) = (1, 2).

6. SHOW ALL YOUR WORK!



 60
for 0 ≤ x ≤ 2
[10 MARKS] The function f is defined by f (x) = 1 + x2 . A

20 − 4x for 2 < x ≤ 5
rectangle with sides parallel to the coordinate axes has one vertex at the origin,
one on the positive x-axis, one on the positive y-axis; and the fourth on the graph
of f . Showing all your work, use the calculus — no other method will be accepted
— to determine the maximum area of such a rectangle.
7. SHOW ALL YOUR WORK!
2
For x ≥ 0, define f (x) = xe−2x .
(a) [3 MARKS] Showing all your work, determine the intervals of its domain
where f is increasing, and the intervals where it is decreasing.
(b) [3 MARKS] Showing all your work, determine whether f has local extrema,
and classify them, if any, as maxima or minima. You are expected to base
your classification on tests studied in this course.
(c) [3 MARKS] Showing all your work, determine all inflection points for f .
(d) [1 MARK] Sketch the graph of f .
Information for Students in MATH 140 2008 09 3247

8.22 May, 2008, Supplemental/Deferred Examination in MATH


140 2007 09
Instructions
1. Fill in the above clearly.

2. DO NOT TEAR PAGES FROM THIS BOOK! All your writing — even rough work — must
be handed in. You may do rough work anywhere in the booklet.

3. This is a CLOSED BOOK examination. CALCULATORS ARE NOT PERMITTED. Transla-


tion dictionaries are permitted; no other dictionaries are permitted.

4. OTHER CALCULUS EXAMINATIONS ARE BEING WRITTEN AT THIS TIME. THIS IS


THE EXAMINATION IN MATH 140 ONLY!

5. The examination booklet consists of this cover, Pages 1 through 7 containing questions; and
Pages 8, 9, and 10, which are blank. Your neighbour’s version may not be the same as yours.

6. There are two kinds of problems on this examination, each clearly marked as to its type.

• Some of the questions on this paper require that you SHOW ALL YOUR WORK!
Their solutions are to be written in the space provided on the page where the question
is printed. When that space is exhausted, you may write on the facing page. Any
solution may be continued on the last pages, or the back cover of the booklet, but you
must indicate any continuation clearly on the page where the question is printed!
• Some of the questions on this paper require only BRIEF SOLUTIONS ; for these you
are expected to write the correct answer in the box provided; you are not asked to show
your work, and you should not expect partial marks for solutions that are not completely
correct.

You are expected to simplify your answers wherever possible.


You are advised to spend the first few minutes scanning the problems. (Please inform the
invigilator if you find that your booklet is defective.)

7. A TOTAL OF 70 MARKS ARE AVAILABLE ON THIS EXAMINATION.

1. BRIEF SOLUTIONS
[2 MARKS EACH] Give the value of each of the following limits if it exists; if the
limit is +∞ or −∞, write +∞ or −∞ respectively. In all other cases write “NO
FINITE OR INFINITE LIMIT”.
x + x3
(a) lim =
x→−∞ 1 − 5x3
Information for Students in MATH 140 2008 09 3248

ANSWER ONLY

sin(sec x)
(b) lim =
x→0 sec(sin x)

ANSWER ONLY

tanh 2x
(c) lim+ =
x→0 arctan 2x

ANSWER ONLY

ln (eu )
(d) lim =
u→3 eln u

ANSWER ONLY

³√ ´
(e) lim u2 + 4u + 1 + u + 3 =
u→−∞

ANSWER ONLY

2. BRIEF SOLUTIONS
[3 MARKS EACH] In each of the following problems fill in the blank with all
the information requested; if the object/objects requested does/do not exist, write
“NONE”.
³ ´
(a) The horizontal asymptotes to the graph of g(x) = (ex − 3)2 e−x + 4 are
2
Information for Students in MATH 140 2008 09 3249

ANSWER ONLY


(b) A particle is moving along the curve y = x. How fast is the distance from
the particle to the origin changing when the particle passes through the point
(x, y) = (9, 3), if, at that time, y is increasing at the rate of 1 cm/sec?

ANSWER ONLY

(c) List all of the inflection points — if any — of the graph of the function
f (x) = 3 (x5 − 1) − 10x3 (x − 1):

ANSWER ONLY

3. BRIEF SOLUTIONS
[3 MARKS EACH] Evaluate each of the following, and always simplify your answers
as much as possible.
½
x2 − 2x + 2 if x > 4
(a) Find the values of the constants: f (x) = is
−x2 + Ax + B if x ≤ 4
differentiable at x = 4 when (A, B) =

ANSWER ONLY

d
(b) ln | csc u + cot u| =
du

ANSWER ONLY
Information for Students in MATH 140 2008 09 3250

(c) If, for x > 0, f 00 (x) = x−2 , and it is known that f (1) = 0, and f 0 (2) = 0, then
f (x) =

ANSWER ONLY

µ ¶
2
(d) The tangent line to the curve y = ln −xex at the point (−1, 1) has equa-
tion

ANSWER ONLY

4. SHOW ALL YOUR WORK!

(a) [2 MARKS] Showing all yourµ work, ¶determine the domain of the function
√ x−1
f (x) = 2 arctan( x) − arcsin .
x+1
(b) [4 MARKS] Determine the derivative of f .
(c) [4 MARKS] Using the result of question (4b), evaluate f (9); your answer must
be simplified as much as possible; the examiners are aware that you do not
have a calculator.

5. SHOW ALL YOUR WORK!


The equation
x3 + y 3 + 3(x2 + y 2 ) + xy = 5
defines y implicitly as a function of x near the point (x, y) = (−1, −1). Showing
all your work

(a) [3 MARKS] determine the value of y 0 at (x, y) = (−1, −1);


(b) [3 MARKS] determine the value of y 00 at (x, y) = (−1, −1); and
(c) [3 MARKS] estimate y when x = −0.99 by using the tangent line to the curve
at the point (x, y) = (−1, −1).
Information for Students in MATH 140 2008 09 3251

6. SHOW ALL YOUR WORK!


[10 MARKS] A rectangular storage container with an open top is to have a volume
of 1000 cm3 . The length of its base is 3 times its width. Material for the base costs
$4 per square meter, while material for the sides costs $9 per square meter. Find
the costs of materials for the cheapest such container.

7. SHOW ALL YOUR WORK!



 ¡ x−4 ¢ if x < 0
Define f (x) = 4 sin¡2 x −¢ cos x if 0 ≤ x ≤ 4π
3
.
 15

x if x > 4π
3

(a) [4 MARKS] Showing all your work, determine the intervals of its domain
where f is increasing, and the intervals where it is decreasing.
(b) [4 MARKS] Showing all your work, determine whether f has local extrema,
and classify them, if any, as maxima or minima. You are expected to base
your classification on tests studied in this course, not on observations from
your graph.
(c) [2 MARKS] Sketch the graph of f .
Information for Students in MATH 140 2008 09 4001

9 WeBWorK
9.1 Frequently Asked Questions (FAQ)
9.1.1 Where is WeBWorK?
WeBWorK is located on Web servers of the Department of Mathematics and Statistics,
and is accessible at the following URL’s:

http://msr04.math.mcgill.ca/webwork/m140f08
or
http://msr05.math.mcgill.ca/webwork/m140f08

If your student number ends with an odd digit — 1, 3, 5, 7, or 9 — you should access
the URL

http://msr05.math.mcgill.ca/webwork/m140f08;

if your student number ends with an even digit — 0, 2, 4, 6, or 8 — you should access

http://msr04.math.mcgill.ca/webwork/m140f08.

If you access WeBWorK through WebCT, the link on your page will have been pro-
grammed to take you to the correct WeBWorK server automatically.

9.1.2 Do I need a password to use WeBWorK?


You will need a user code and a password.

Your user code. Your user code will be your 9-digit student number.

Your password. The WeBWorK system is administered by the Mathematics and


Statistics Department, and is not accessible through the myMcGill Portal; your initial
password will be different from your MINERVA password, but you could change it to
that if you wish. Your initial password will be your 9-digit student ID number. You will
be able to change this password after you sign on to WeBWorK.134
134
If you forget your password you will have to send a message to Professor Brown so that the system
administrator may be instructed to reset the password at its initial value.

UPDATED TO December 2, 2008


Information for Students in MATH 140 2008 09 4002

Your e-mail address. The WeBWorK system requires each user to have an e-mail
address. After signing on to WeBWorK, you should verify that the e-mail address
shown is the one that you prefer. You should endeavour to keep your e-mail address up
to date, since the instructors may send messages to the entire class through this route.
We suggest that you use either your UEA135 or your po-box address. You may be able
to forward your mail from these addresses to another convenient address, (cf. §1.8.1.)

9.1.3 Do I have to pay an additional fee to use WeBWorK?


WeBWorK is available to all students registered in the course at no additional charge.

9.1.4 When will assignments be available on WeBWorK?


Each assignment will have a begin date and a due date. The assignment is available to
you after the begin date; answers (not full solutions) will be made available soon after
the due date.

9.1.5 Do WeBWorK assignments cover the full range of problems that I


should be able to solve in this course?
The questions on the WeBWorK assignments (A1 through A7 ) are a sampling of some
types of problem you should be able to solve after successfully completing this course.
Some types of calculus problems do not lend themselves to this kind of treatment, and
may not appear on the WeBWorK assignments. Use of WeBWorK does not re-
place studying the textbook — including the worked examples, attending
lectures and tutorials, and working exercises from the textbook — using the
Student Solutions Manual [3] to check your work. Students are cautioned not
to draw conclusions from the presence, absence, or relative frequencies of problems of
particular types, or from particular sections of the textbook. Certain sections of the
textbook remain examination material even though no problems are included in the
WeBWorK assignments.

9.1.6 May I assume that the distribution of topics on quizzes and final
examinations will parallel the distribution of topics in the WeBWorK
assignments?
No! While the order of topics on WeBWorK assignments should conform to the order
of the lectures, there are some topics on the syllabus that will not appear in WeBWorK
135
Uniform E-mail Address
Information for Students in MATH 140 2008 09 4003

questions. Use WeBWorK for the areas it covers, and supplement it by working prob-
lems from your textbook. Also, remember that WeBWorK — which checks answer
only — cannot ascertain whether you are using a correct method for solving problems.
But, if you write out a solution to an odd-numbered textbook problem, you can compare
it with the solution in the Solutions Manual; and, if in doubt, you can show your work
to a Teaching Assistant at one of the many office hours that they hold through the week.

9.1.7 WeBWorK provides for different kinds of “Display Mode”. Which


should I use?
“Display Mode” is the mode that you enter when you first view a problem; and, later,
when you submit your answer. You may wish to experiment with the different formats.
The default is jsMath mode, which should provide reasonable looking mathematical for-
mulæ; jsMath requires the presence of certain fonts on your computer. If you experience
any difficulty in viewing in jsMath mode, just change the mode to images mode, which
should also look similar to the version that you print out (cf. next question); but viewing
in images mode may be slower. The lowest quality is plain Text mode, which is essentially
the way the author of the problem entered his data into the system; this mode is related
to the TEX and LATEX systems that mathematicians use in typesetting their documents;
the notes that you are reading here were prepared using LATEX; it contains formatting
instructions in a “markup” language, and is difficult for inexperienced readers.

9.1.8 WeBWorK provides for printing assignments in “Portable Document


Format” (.pdf ), “PostScript” (.ps) forms. Which should I use?
Most newer home computers have already been loaded with the Acrobat Reader for .pdf
files; if the Reader has not been installed on your computer136 , you will find instructions
for downloading this (free) software in §1.5.5 of these notes. If you are not happy with
.pdf files, and wish to print and view PostScript files, you may require such (free) software
as Ghostscript and Ghostview, available at
http://pages.cs.wisc.edu/ ghost/
Most computers available to you on campus should be capable of printing in either
of .pdf and PostScript formats.

9.1.9 What is the relation between WeBWorK and WebCT (my Courses)?
There is none. WebCT is the proprietary system of Web Course Tools that has been
implemented by McGill University as Courses. You may access the web page for this
136
At the time these notes were written, the latest version of the Reader for my operating system was
8.1.0 but recent, earlier versions should also work properly.
Information for Students in MATH 140 2008 09 4004

course, and WeBWorK through your myCourses account137 , and WebCT will link you
to the appropriate server for WeBWorK. If you follow this route to WeBWorK, you
will still have to log in when you reach the WeBWorK site. At the present time we will
be using WebCT primarily for the posting of grades, and as a convenient repository for
links to notes and announcements in the course.

9.1.10 Which browser should I use for WeBWorK?


We recommend that you use Internet Explorer, Netscape, or Mozilla. While other
browsers may give satisfactory results, your instructors and tutors do not have time
to correct errors in your WeBWorK records that could be attributed to idiosyncracies
in another browser. Information about browsers supported by WebCT may be obtained
at
http://www.mcgill.ca/myCourses/

9.1.11 What do I have to do on WeBWorK?


After you sign on to WeBWorK, and click on “Begin Problem Sets”, you will see a
list of Assignments, each with a due date. Since there is no limit to the number of
attempts at problems on P0 or the other “Practice” assignments, you may play with
these assignments to learn how to use the WeBWorK software.
You may print out a copy of your assignment by clicking on “Get hard copy”. This is
your version of the assignment, and it will differ from the assignments of other students
in the course. You should spend some time working on the assignment away from the
computer. When you are ready to submit your solutions, sign on again, and select the
same assignment. This time click on “Do problem set”. You can expect to become more
comfortable with the system as you attempt several problems; but, in the beginning,
there are likely to be situations where you cannot understand what the system finds
wrong with some of your answers. It is useful to click on the Preview Answers button to
see how the system interprets an answer that you have typed in. As the problems may
become more difficult, you may have to refer to the “Help” page, and also to the “List
of functions” which appears on the page listing the problems. Don’t submit an answer
until you are happy with the interpretation that the Preview Answers button shows that
the system will be taking of your answer.

9.1.12 How can I learn how to use WeBWorK?


As soon as your instructor announces that the WeBWorK accounts are ready, sign on
and try assignment P0 , which does not count. The system is self-instructive, so we will
137
http://www.mcgill.ca/mycourses
Information for Students in MATH 140 2008 09 4005

not burden you with a long list of instructions.

You will need to learn how to enter algebraic expressions into WeBWorK as it is coded
to read what you type in a way that may different from what you expect. For example,
the symbol ^ is used for writing exponents (powers). If you type 2^3, WeBWorK will
interpret this as 23 = 8. However, if you type 2^3+x, WeBWorKwill interpret it as
23 + x, i.e. as 8 + x; if you wish to write 23+x , you have to type 2^(3+x). You may obtain
more information from the List of Available Functions, available online, or at
http://webwork.math.rochester.edu/webwork_system_html
/docs/docs/pglanguage/availablefunctions.html

9.1.13 Where should I go if I have difficulties with WeBWorK?


If you have difficulties signing on to WeBWorK, or with the viewing or printing func-
tions on WeBWorK, or with the specific problems on your version of an assignment,
you may send an e-mail distress message directly from WeBWorK by clicking on the
Feedback button. You may also report the problem to your instructor and/or your
tutor, but the fastest way of resolving your difficulty is usually the Feedback . Please
give as much information as you can. (All of the instructors and tutors are able to view
from within WeBWorK the answers that you have submitted to questions.)
If your problem is mathematical, and you need help in solving a problem, you should
consult one of the tutors at their office hours; you may go to any tutor’s office hours, not
only to the hours of the tutor of the section in which you are registered.

9.1.14 Can the WeBWorK system ever break down or degrade?


Like all computer systems, WeBWorK can experience technical problems. The systems
manager is continually monitoring its performance. If you experience a difficulty when
online, please click on the Feedback button and report it. If that option is not available
to you, please communicate with either instructor by e-mail.
If you leave your WeBWorK assignment until the hours close to the due time on
the due date, you should not be surprised if the system is slow to respond. This is
not a malfunction, but is simply a reflection of the fact that other students have also
been procrastinating! To benefit from the speed that the system can deliver under normal
conditions, do not delay your WeBWorK until the last possible day! If a systems failure
interferes with the due date of an assignment, arrangements could be made to change that
date, and an e-mail message could be broadcast to all users (to the e-mail addresses on
record), or a message could be posted on WeBCT or the WeBWorK sign-on screen.138
138
But slowness of the system just before the due time will not normally be considered a systems
failure.
Information for Students in MATH 140 2008 09 4006

9.1.15 How many attempts may I make to solve a particular problem on


WeBWorK?
Practice Assignments P1 — P7 are intended to prepare you for Assignments A1 — A7 ,
and permit unlimited numbers of attempts; your grades on these “Practice” do not
count in your term mark. For the problems on assignments A1 — A7 you will normally
be permitted about 5 tries: read the instructions at the head of the assignment. You
should not need to use more than one or two of these tries, as you should have learned
how to solve the problems on the Practice assignments.

9.1.16 Will all WeBWorK assignments have the same length? the same
value?
The numbers of problems on the various assignments may not be the same, and the
individual problems may vary in difficulty. Assignments A1 — A7 will count equally in
the computation of your grade.

9.1.17 Is WeBWorK a good indicator of examination performance?


A low grade on WeBWorK has often been followed by a low grade on the examination.
A high grade on WeBWorK does not necessarily indicate a likely high grade on the
examination.
To summarize: WeBWorK alone is not enough to prepare this course; but students
who don’t do WeBWorK appear to have a poor likelihood of success in MATH 140:
that is one reason why we have made the WeBWorK assignments compulsory.
Information for Students in MATH 140 2008 09 5001

10 Contents of the DVD disks for


Larson/Hostetler/Edwards
These excellent disks were produced to accompany the textbook, Calculus of a Single
Variable: Early Transcendental Functions, 3rd Edition[28] (called LHE in the charts be-
low). The correspondence shown to sections of [7] are only approximate. (NOTE THAT
THIS BOOK DOES NOT FOLLOW STEWART’S CONVENTIONS FOR INVERSE
SECANT/COSECANT!)
[All references in this table are to the 5th edition of Stewart, [7].]

DVD LHE Stewart


# Section Subject Minutes Section
1 P Chapter P: Preparation for Calculus
1 P.1 Graphs and Models 45
1 P.2 Linear Models and Rates of Change 27 A10
1 P.3 Functions and Their Graphs 48 1.1
1 P.4 Fitting Models to Data 21 1.2
1 P.5 Inverse Functions 48 1.6
1 P.6 Exponential and Logarithmic Functions 30 1.5
DVD LHE Stewart
# Section Subject Minutes Section
1 1 Chapter 1: Limits and Their Properties
1 1.1 A Preview of Calculus 11 2.1
1 1.2 Finding Limits Graphically and Numeri- 25 2.2, 2.4
cally
1 1.3 Evaluating Limits Analytically 28 2.3
1 1.4 Continuity and One-Sided Limits 22 2.5
1 1.5 Infinite Limits 18 2.6
DVD LHE Stewart
# Section Subject Minutes Section
1 2 Chapter 2: Differentiation
1 2.1 The Derivative and the Tangent Line Prob- 68 2.1
lem
1 2.2 Basic Differentiation Rules and Rates of 34 2.3
Change
1 2.3 The Product and Quotient Rules and 25 3.2, 3.7
Higher Order Derivatives
Information for Students in MATH 140 2008 09 5002

DVD LHE Stewart


# Section Subject Minutes Section
2 2 Chapter 2 (continued): Differentiation
2 2.4 The Chain Rule 44 3.5
2 2.5 Implicit Differentiation 50 3.6
2 2.6 Derivatives of Inverse Functions 17 3.5, 3.8, 3.9
2 2.7 Related Rates 34 3.10
2 2.8 Newton’s Method 26 4.9
DVD LHE Stewart
# Section Subject Minutes Section
2 3 Chapter 3: Applications of Differentiation
2 3.1 Extrema on an Interval 41 4.1
2 3.2 Rolle’s Theorem and the Mean Value The- 15 4.2
orem
2 3.3 Increasing and Decreasing Functions and 19 4.3
the First Derivative Test
2 3.4 Concavity and the Second Derivative Test 24 4.3
2 3.5 Limits at Infinity 23 2.6
2 3.6 A Summary of Curve Sketching 43 4.5
2 3.7 Optimization Problems 37 4.7
2 3.8 Differentials 51 3.11
DVD LHE Stewart
# Section Subject Minutes Section
3 4 Chapter 4: Integration
3 4.1 Antiderivatives and Indefinite Integration 40 4.10
DVD LHE Stewart
# Section Subject Minutes Section
4 7 Chapter 7: Integration by Parts Trigono-
metric Substitution Partial Fractions
L’Hôpital’s Rule
4 7.7 Indeterminate Forms and L’Hôpital’s Rule 22 4.4

(The coverage extends to part of the material for Math 141 as well.)
Information for Students in MATH 140 2008 09 6001

11 References

11.1 Stewart Calculus Series


[1] J. Stewart, Single Variable Calculus (Early Transcendentals), Sixth Edition. Thom-
son * Brooks/Cole (2008). ISBN 0-495-01169-X.

[2] J. Stewart, Calculus (Early Transcendentals), Sixth Edition. Thomson *


Brooks/Cole (2008). ISBN 0-495-01166-5.

[3] D. Anderson, J. A. Cole, D. Drucker, Student Solutions Manual for Stewart’s Single
Variable Calculus (Early Transcendentals), Sixth Edition. Thomson * Brooks/Cole
(2008). ISBN 0-495-01240-8.

[4] J. Stewart, Single Variable Calculus (Early Transcendentals), Sixth Edition. Thom-
son * Brooks/Cole (2008); bundled with Student Solutions Manual for Stew-
art’s Single Variable Calculus (Early Transcendentals), Sixth Edition. Thomson
* Brooks/Cole (2008). ISBN 0-495-42966-X.

[5] R. St. Andre, Study Guide for Stewart’s Single Variable Calculus (Early Transcen-
dentals), Sixth Edition. Thomson * Brooks/Cole (2008). ISBN 0-495-01239-4.

[6] J. Stewart, Multivariable Calculus (Early Transcendentals), Sixth Edition. Thom-


son * Brooks/Cole (2008). ISBN 0-495-?????-?.

[7] J. Stewart, Single Variable Calculus (Early Transcendentals), Fifth Edition. Thom-
son * Brooks/Cole (2003). ISBN 0-534-39330-6.

[8] J. Stewart, Calculus (Early Transcendentals), Fifth Edition. Thomson *


Brooks/Cole (2003). ISBN 0-534-39321-7.

[9] D. Anderson, J. A. Cole, D. Drucker, Student Solutions Manual for Stewart’s Single
Variable Calculus (Early Transcendentals), Fifth Edition. Thomson * Brooks/Cole
(2003). ISBN 0-534-39333-0.

[10] J. Stewart, Single Variable Calculus (Early Transcendentals), Fifth Edition. Thom-
son * Brooks/Cole (2003); bundled with Student Solutions Manual for Stew-
art’s Single Variable Calculus (Early Transcendentals), Fifth Edition. Thomson
* Brooks/Cole (2003). ISBN 0-17-6425411.

[11] J. Stewart, Single Variable Essential Calculus (Early Transcendentals). Thomson


* Brooks/Cole (2006). Thomson * Brooks/Cole (2003). ISBN 0-495-10957-6.
Information for Students in MATH 140 2008 09 6002

[12] J. Stewart, Calculus (Early Transcendentals), Fifth Edition. Thomson *


Brooks/Cole (2003); bundled with Student Solutions Manual for Stewart’s Single
Variable Calculus (Early Transcendentals), Fifth Edition. Thomson * Brooks/Cole
(2003). ISBN 0-534-10307-3.

[13] R. St. Andre, Study Guide for Stewart’s Single Variable Calculus (Early Transcen-
dentals), Fifth Edition. Thomson * Brooks/Cole (2003). ISBN 0-534-39331-4.

[14] Text-specific DVDs for Stewarts Calculus, early transcendentals, 6th edition [vide-
orecording]. Thomson * Brooks/Cole (c2008). ISBN 0-495-01243-2. 4 DVD disks.

[15] Interactive Video Skillbuilder CD for Stewart’s Calculus: Early Transcendentals,


5th Edition. Thomson * Brooks/Cole (2003). ISBN 0-534-39326-8.

[16] H. Keynes, J. Stewart, D. Clegg, Tools for Enriching Calculus, CD to accompany


[7] and [8]. Thomson * Brooks/Cole (2003). ISBN 0-534-39731-X.

[17] J. Stewart, Single Variable Calculus (Early Transcendentals), Fourth Edition.


Brooks/Cole (1999). ISBN 0-534-35563-3.

[18] J. Stewart, Calculus (Early Transcendentals), Fourth Edition. Brooks/Cole (1999).


ISBN 0-534-36298-2.

[19] D. Anderson, J. A. Cole, D. Drucker, Student Solutions Manual for Stewart’s


Single Variable Calculus (Early Transcendentals), Fourth Edition. Brooks/Cole
(1999). ISBN 0-534-36301-6.

[20] J. Stewart, L. Redlin, S. Watson, Precalculus: Mathematics for Calculus, Enhanced


Review Edition. Thomson * Brooks/Cole. (2006). ISBN: 0-495-39276-6.

[21] J. Stewart, Trigonometry for Calculus. Thomson * Brooks/Cole. ISBN: 0-17-


641227-1.

11.2 Other Calculus Textbooks


11.2.1 R. A. Adams

[22] R. A. Adams, Calculus, Single Variable, Fifth Edition. Addison, Wesley, Longman,
Toronto (2003). ISBN 0-201-79805-0.

[23] R. A. Adams, Calculus of Several Variables, Fifth Edition. Addison, Wesley, Long-
man, Toronto (2003). ISBN 0-201-79802-6.
Information for Students in MATH 140 2008 09 6003

[24] R. A. Adams, Calculus: A Complete Course, Fifth Edition. Addison, Wesley, Long-
man, Toronto (2003). ISBN 0-201-79131-5.
[25] R. A. Adams, Student Solution Manual for Adams’, Calculus: A Complete Course,
Fifth Edition. Addison, Wesley, Longman, Toronto (2003). ISBN 0-201-79803-4.
[26] R. A. Adams, Calculus: A Complete Course, with Solution Manual, Fifth Edition.
Addison, Wesley, Longman, Toronto (2003). ISBN 0-131-30565-4.
[27] R. A. Adams, Calculus: A Complete Course Manual, Sixth Edition. Addison,
Wesley, Longman, Toronto (2006). ISBN 0-321-27000-2.

11.2.2 Larson, Hostetler, et al.

[28] Calculus Instructional DVD Program, for use with (inter alia) Lar-
son/Hostetler/Edwards, Calculus of a Single Variable: Early Transcendental Func-
tions, Third Edition [29]. Houghton Mifflin (2003). ISBN 0-618-25097-2.
[29] R. Larson, R. P. Hostetler, B. H. Edwards, D. E. Heyd, Calculus, Early Transcen-
dental Functions, Third Edition. Houghton Mifflin Company, Boston (2003). ISBN
0-618-22307-X.

11.2.3 Edwards and Penney

[30] C. H. Edwards, Jr., and D. E. Penney, Single Variable Calculus, Early Transcen-
dentals, Sixth Edition. Prentice Hall, Englewood Cliffs, NJ (2002). ISBN 0-13-
041407-7.
[31] C. H. Edwards, Jr., and D. E. Penney, Calculus with Analytic Geometry, Early
Transcendentals Version, Fifth Edition. Prentice Hall, Englewood Cliffs, NJ (1997).
ISBN 0-13-793076-3.
[32] C. H. Edwards, Jr., and D. E. Penney, Student Solutions Manual for Calculus with
Analytic Geometry, Early Transcendentals Version, Fifth Edition. Prentice Hall,
Englewood Cliffs, NJ (1997). ISBN 0-13-079875-4.
[33] C. H. Edwards, Jr., and D. E. Penney, Single Variable Calculus with Analytic
Geometry, Early Transcendentals Version, Fifth Edition. Prentice Hall, Englewood
Cliffs, NJ (1997). ISBN 0-13-793092-5.
[34] C. H. Edwards, Jr., and D. E. Penney, Student Solutions Manual for Single Variable
Calculus with Analytic Geometry, Early Transcendentals Version, Fifth Edition.
Prentice Hall, Englewood Cliffs, NJ (1997). ISBN 0-13-095247-1.
Information for Students in MATH 140 2008 09 6004

11.2.4 Others, not “Early Transcendentals”

[35] G. H. Hardy, A Course of Pure Mathematics, 10th edition. Cambridge University


Press (1967).

[36] H. S. Hall, S. R. Knight, Elementary Trigonometry, Fourth Edition. Macmillan


and Company, London (1905).

[37] S. L. Salas, E. Hille, G. J. Etgen, Calculus, One and Several Variables, 10th Edition.
John Wiley & Sons, Inc. (2007). ISBN 0471-69804-0.

11.3 Other References


[38] D. Ebersole, D. Schattschneider, A. Sevilla, K. Somers, A Companion to Calculus.
Brooks/Cole (1995). ISBN 0-534-26592-8.

[39] McGill Undergraduate Programs Calendar 2007/2008. Also accessible at


http://coursecalendar.mcgill.ca/ugcal200708/wwhelp/wwhimpl/js/html/wwhelp.htm

You might also like